Arihant NTA CUET UG Exam Guide For Section 3 General Test With Practice Sets and Solved Paper For 2024 Exams 9358896086, 9789358896084


105 10 9MB

English Pages 330 [303]

Report DMCA / Copyright

DOWNLOAD PDF FILE

Recommend Papers

Arihant NTA CUET UG Exam Guide For Section 3 General Test With Practice Sets and Solved Paper For 2024 Exams
 9358896086, 9789358896084

  • 0 0 0
  • Like this paper and download? You can publish your own PDF file online for free in a few minutes! Sign Up
File loading please wait...
Citation preview

A-PDF Watermark DEMO: Purchase from www.A-PDF.com to remove the watermark

https://sscstudy.com/

https://sscstudy.com/

https://sscstudy.com/

Contents Oneliner Current Affairs

1-32

General Knowledge

1-50

General Mental Ability

1-84

1. Analogy 2. Classification

3-8 9-12

9. Logical Sequence of Words

47-49

10. Inserting the Missing Characters

50-53

3. Series

13-18

11. Ranking Test

54-57

4. Coding and Decoding

19-24

12. Sitting Arrangement

58-61

5. Alphabet Test and Jumbling

25-30

13. Puzzle Test

62-65

6. Mathematical Operations

31-34

14. Venn Diagram

66-71

7. Direction Sense Test

35-41

15. Cube and Dice

72-75

8. Blood Relations

42-46

16. Non-Verbal Reasoning

76-84

Numerical Ability 1. Number System

1-102 3-9

10. Simple and Compound Interest

49-53

2. HCF and LCM

10-13

11. Mixture and Alligation

54-57

3. Square Root and Cube Root

14-20

12. Time and Work

58-62

4. Indices and Surds

21-24

13. Time, Speed and Distance

63-69

5. Simplification

25-28

14. Area and Perimeter

70-75

6. Average

29-32

15. Volume and Surface Area

76-80

7. Ratio and Proportion

33-37

16. Algebra

81-87

8. Percentage

38-41

17. Geometry

88-95

9. Profit, Loss and Discount

42-48

18. Data Interpretation

96-102

Quantitative Reasoning

3-10

Logical and Analytical Reasoning

3-14

Practice Sets (1-5)

3-22

https://sscstudy.com/

https://sscstudy.com/

CUET (UG) All About

Exam Common University Entrance Test (CUET (UG) – 2022) will be conducted in 13 mediums across India for admission into the Undergraduate Programmes in Computer Based Test (CBT) mode for all the Central Universities (CUs) like DU, JNU, JMI, BHU, AMU, etc. in India for the academic year 2022-23.

EVENTS

DETAILS

Online submission of Application Form

06 April to 06 May, 2022 (up to 5:00 pm)

Last date of successful transaction of Examination Fee

06 May, 2022 (up to 11:50 pm)

Correction in the particulars of Application Form on Website only

To be announced later on the website

Downloading of Admit Card from NTA Website

To be announced later on the website

Date(s) of Examination

First and second week of July, 2022

Duration of Examination

Slot 1: 195 minutes (3:15 hours) Slot 2: 225 minutes (3:45 hours) Slot 1: 09.00 AM to 12.15 PM (IST)

Timing of Examination

Slot 2: 03.00 PM to 06.45 PM (IST)

TESTS DESIGN

There are three Sections in the design of the test:

Section IA

13 Languages (As a medium and “Language”)

Section IB

20 Languages

Section II

27 Domain Speci c Subjects

Section III

General Test

MERIT LIST Merit list will be prepared by participating Universities/ organisations. Universities may conduct their individual counselling on the basis of the score card of CUET (UG) 2022 provided by NTA.

https://sscstudy.com/

https://sscstudy.com/

Broad features of CUET (UG) - 2022 are as follows: Section

Subjects/Tests

Section IALanguages

There are 13* different languages. Any of these languages may be chosen.

Section IBLanguages

There are 19** languages. Any other language apart from those offered in Section IA may be chosen.

Section IIDomain

Section IIIGeneral Test

Questions to be Attempted

Question Type

Duration

40 questions to be attempted out of 50 in each language

Language to be tested through Reading Comprehension (based on different types of passages – Factual, Literary and Narrative, [Literary Aptitude and Vocabulary]

45 minutes for each language

There are 20*** Domains specific subjects being offered under this Section. A candidate may choose a maximum of Six (06) Domains as desired by the applicable University/Universities.

40 questions to be attempted out of 50

Ÿ Input text can be used for MCQ

45 minutes for each Domain Specific Subjects

For any such undergraduate programme/programmes being offered by Universities where a General Test is being used for admission.

60 questions to be attempted out of 75

based questions

Ÿ MCQs based on NCERT Class XII

syllabus only

Ÿ Input text can be used for MCQ

60 minutes

based questions

Ÿ General Knowledge, Current

Affairs, General Mental Ability, Numerical Ability, Quantitative Reasoning (Simple application of basic mathematical concepts arithmetic/algebra geometry/ mensuration/stat taught till Grade 8) Logical and Analytical Reasoning

* Languages (13): Tamil, Telugu, Kannada, Malayalam, Marathi, Gujarati, Odiya, Bengali, Assamese, Punjabi, English, Hindi and Urdu ** Languages (20): French, Spanish, German, Nepali, Persian, Italian, Arabic, Sanskrit, Sindhi, Kashmiri, Konkani, Bodo, Dogri, Maithili, Manipuri, Santhali, Tibetan, Japanese, Russian, Chinese. *** Domain Specific Subjects (27): 1. Accountancy/ Book Keeping, 2. Biology/ Biological Studies/Biotechnology /Biochemistry, 3. Business Studies, 4. Chemistry 5. Computer Science/ Informatics Practices 6. Economics/ Business Economics 7. Engineering Graphics 8. Entrepreneurship 9. Geography/Geology 10. History 11. Home Science 12. Knowledge Tradition and Practices of India 13. Legal Studies 14. Environmental Science 15. Mathematics 16. Physical Education/ NCC /Yoga 17. Physics 18. Political Science 19. Psychology 20. Sociology 21. Teaching Aptitude 22. Agriculture 23. Mass Media/ Mass Communication 24. Anthropology 25. Fine Arts/Visual Arts (Sculpture/ Painting)/Commercial Arts, 26. Performing Arts – (i) Dance (Kathak/ Bharatnatyam/Oddisi/ Kathakali/Kuchipudi/ Manipuri (ii) Drama- Theatre (iii) Music General (Hindustani/ Carnatic/ Rabindra Sangeet/ Percussion/ Non-Percussion). 27. Sanskrit/For all Shastri (Shastri 3 years/ 4 years Honours) Equivalent to B.A./B.A. Honours courses i.e. Shastri in Veda, Paurohitva (Karmakand), Dharamshastra, Prachin Vyakarana, Navya Vyakarana, Phalit Jyotish, Siddhant Jyotish, Vastushastra, Sahitya,Puranetihas, Prakrit Bhasha,Prachin Nyaya Vaisheshik, Sankhya Yoga, Jain Darshan,Mimansa,Advaita Vedanta, Vishihstadvaita Vedanta, Sarva Darshan, a candidate may choose Sanskrit as the Domain.] Ÿ A candidate can choose a maximum of any 3 languages from Section IA and Section IB taken together. However, the (one of the langueges chosen needs to be in lieu of 6th domain specific Subjects). Ÿ Section II offers 27 Subjects, out of which a candidate may choose a maximum of 6 Subjects. Ÿ Section III comprises General Test. Ÿ For choosing Languages (upto 3) from Section IA and IB and a maximum of 6 Subjects from Section II and General Test under Section III, the candidate must refer to the requirements of his/her intended University.

https://sscstudy.com/

https://sscstudy.com/

Broad features of CUET (UG) - 2022 are as follows: Mode of the Test

Computer Based Test-CBT

Test Pattern

Objective type with Multiple Choice Questions

Medium

13 languages ( Tamil, Telugu, Kannada, Malayalam, Marathi, Gujarati, Odiya, Bengali, Assamese, Punjabi, English, Hindi and Urdu) Section IA & IB: Language to be tested through Reading Comprehension (based on different types of passages– Factual, Literary and Narrative [Literary Aptitude & Vocabulary]

Syllabus

Section II : As per NCERT model syllabus as applicable to Class XII only Section III : General Knowledge, Current Affairs, General Mental Ability, Numerical Ability, Quantitative Reasoning (Simple application of basic mathematical concepts arithmetic/ algebra geometry/mensuration/stat taught till Grade 8), Logical and Analytical Reasoning

Registration

Registration will be online at https://cuet.samarth.ac.in/. (available from 06.04.2022)

Level of questions for CUET (UG) -2022: All questions in various testing areas will be benchmarked at the level of Class XII only. Students having studied Class XII Board syllabus would be able to do well in CUET (UG) – 2022.

Number of attempts : If any University permits students of previous years of class XII to take admission in the current year also, such students would also be eligible to appear in CUET (UG) – 2022. Choice of Languages and Subjects : Generally the languages/subjects chosen should be the ones that a student has opted in his latest Class XII Board examination. However, if any University permits any flexibility in this regards, the same can be exercised under CUET (UG) -2022 also. Candidates must carefully refer to the eligibility requirements of various Central Universities in this regard. Moreover, if the subject to be studied in the Undergraduate course is not available in the list of 27 Domain Specific Subject being offered, the candidate may choose the Subject closest to his/her choice for e.g. For Biochemistry the candidate may choose Biology.

https://sscstudy.com/

https://sscstudy.com/

ONELINER CURRENT AFFAIRS

1

ONELINER

CURRENT AFFAIRS National Affairs ■



PM Narendra Modi on March 29, 2022 participated in ‘Grih Pravesham’ of more than 5 lakh beneficiaries of Pradhan Mantri Awaas Yojana-Gramin Madhya Pradesh in India has launched an evacuation mission named Operation Ganga on February 27, 2022 to evacuate Indian nationals from Ukraine due to the







Russia-Ukraine tension ■



Union Cabinet has approved the pan-India implementation of Ayushman Bharat Digital Mission (ABDM) with a 5 years budget of ` 1600 crore for Union Minister of Culture, G Kishan Reddy, inaugurated Devayatanamon February 25, 2022, a one-of-a-kind odyssey (conference) of temple architecture of India, at



Prime Minister Shri NarendraModi inaugurated a 550-tonne capacity ‘Gobar-Dhan (Bio-CNG) Plant’ through video conferencingon February 19, Indore (Madhya Pradesh) 2022 in The Union Minister of Ports, Shipping and Waterways SarbanandaSonowal flagged off India’s first water taxi service on February 17, 2022 in



Union Home Minister Amit Shah launched ‘Smart Card Arms License’ and ‘Shastra App’on February 16, 2022 Delhi Police of the Ministry of Education has approved a new scheme “New India Literacy Programme on February 16, 2022 for the period of 2022-2027 to cover all the Adult Education aspects of

Union Territory ■

The Ministry of Culture has nominated the ‘Sacred Ensembles of Hoysala’ located in Karnataka on January 31, UNESCO’s World Heritage 2022 for



Home Delivery of medicines under …….. for all entitled armed forces personnel and their families started on January 31, 2022.

List for 2022-23

Jammu & Kashmir has become the first Union Territory on February 8, 2022 to be integrated with the

Services e-Health Assistance and Teleconsultation (SeHAT)



‘Pradhan Mantri Kisan Sampada Yojana (PMKSY)’ has been extended till ……….. with an allocation of ` 4600 crore on February 8, 2022. March 2026



Health Minister Mansukh Mandaviya launched revamped website and Mobile App …….. of Central Government Health Schemeon Jan 24, 2022. MyCGHS



……….. government has started an open-air classroom programme, called Paray Shikshalaya (Neighborhood School), for students studying in primary and pre-primary classes on West Bengal February 6, 2022.



The government of India has launched a web portal ………. to share Key Performance Indicators (KPI) related to the Coal Sector on January 21, 2022.





India’s First “District Good Governance Index” for 20 districts of Jammu and Kashmir released on January 21, 2022 and………..district has topped the Jammu Index.



Who was inaugurated ‘AzadiKe Amrit Mahotsav se Swarnim Bharat Ki Ore’programme on January 20, 2022 via video conferencing? PM Narendra Modi



India’s first geological park is being set up on February 1, 2022 at Lamheta village of Jabalpur district in Madhya Pradesh





The Aravalli Biodiversity Park in Gurugram, Haryana, has been declared on February 2,2022 as India’s first Other Effective area-based Conservation Measures (OECM) site



Koyla Darpan

Khadi and Village Industries Commission has canceled the “Khadi Certification” on February 4, 2022 of its oldest Khadi Institution named Mumbai Khadi and Village Industries Association

Maharashtra ■

Union Minister NitinGadkari and Bihar Chief Minister Nitish Kumar jointly inaugurated the ……… ‘Rail-cumRoad-Bridge’ in Bihar on February 11, 14.5 KM long 2022.

and 31, 2022, to celebrate the culture and traditional heritage of the

National Single Window System (NSWS)

Hampi (Karnataka) ■

Ministry of Tribal Affairs has sanctioned ` 2.26 Crores for secondlargest fair in India, MedaramJatara 2022 Festival held from February 16 Telangana to 19, 2022 in

The annual SpitukGustor Festival was observed in Ladakh on January 30

https://sscstudy.com/

The government of India has merged the flame of the Amar Jawan Jyoti on January 21, 2022 at Delhi’s India Gate with the flame at the adjoining National War Memorial



Prime Minister Narendra Modi has declared on January 16, 2022 to mark National Start-up Day January 16 as

https://sscstudy.com/

2 ■

CUET (UG) Section III : General Test

The state governments of Haryana and Himachal Pradesh have signed a MoU on January 21, 2022to revive the Saraswati river through construction of AdiBadri dam







PM Narendra Modi inaugurated 11 new government medical colleges and a new campus of the Central Institute of Classical Tamil (CICT) on January 12, Tamil Nadu 2022 in





Ministry of Railways has approved the renaming of the Kevadiya railway station in Narmada district of Gujarat on Jan 5, 2022, as Ekta Nagar railway station Prime Minister Narendra Modi has declared on January 5, 2022 that starting from the year 2022, December 26 will be observed as ‘Veer Baal Diwas’ Telangana has bagged the top position among all other States of the country in having the highest number of ODF plus villages under the PM Narendra Modi inaugurated 13 Projects and laid the foundation stone of 9 projects worth on January 4, 2022, during an event held at Hapta Kangjeibung in Imphal, Manipur









PM NarendraModi inaugurated and laid foundation stones of 23 projects worth over ` 17500 croreon December 30, 2021 Haldwani (Uttarakhand) in Uttar Pradesh Government has renamed the Jhansi Railway Station in Jhansi as ‘VeeranganaLaxmibai Railway Station’ on December 29, 2021, Queen of Jhansi as a tribute to PM Narendra Modi has inaugurated and laid the foundation stone of several development projects worth over 650 Goa crores rupees on Dec 23, 2021 in PM Narendra Modi has laid the foundation stone of Ganga Expressway on Dec 22, 2021 in Shahjahanpur, Uttar Pradesh



Union Minister Dr. Jitendra Singh has dedicated to the nation “Sardar Patel Leadership Centre” on October 31, 2022 at LalBahadurShastri National Academy of Administration (LBSNAA) Mussoorie in



The Union Minister of Power & New and Renewable Energy, R K Singh has launched a new market segment on October 28, 2022

Pandharpur (Maharashtra)

PM Narendra Modi has inaugurated the first phase of the ` 339 crore KashiVishwanath Corridor project that connects the Kashi Vishwanath temple PM Narendra Modi has inaugurated and laid the foundation stone on December 7, 2021of multiple projects worth ` 18000 crore at

Green Day Ahead Market (GDAM) ■

Dehradun, Uttarakhand ■

NITI Aayog launched the ‘e-Sawaari India Electric Bus Coalition’ in partnership with Convergence Energy Service Ltd (CESL) and World Resources





PM NarendraModi has laid the Foundation Stone of Noida International Airport on November 25, 2022 at Jewar (Uttar Pradesh) President of India, Ram NathKovind visited Sui village in the………….., to inaugurate various public facilities in the village on November 18, 2022.



Prime Minister NarendraModi has inaugurated the Kushinagar International Airport on October 20, Uttar Pradesh 2022 in



Prime Minister NarendraModi dedicated ……. new Defence PSUs, carved out of OFBs, to the nation on October 16, 2022.



Prime Minister NarendraModi inaugurated ……… National Master Plan, from Pragati Maidan in New Delhi on October 14, 2022. PM Gati Shakti

PM Narendra Modi has announced on November 19, 2022 that his government will repeal the three agricultural laws contentious



Centre government has launched …….. in Kolkata for digital monitoring of port operation on Oct 11, 2022.

Union Minister, Piyush Goyal virtually launched India’s first Digital Food Museum on November 18, 2022 in



‘MyPortApp’



PM Narendra Modi has inaugurated the ……….. long Purvanchal Express in Sultanpur district, Uttar Pradesh on 341 km November 16, 2022.



The central government of India promulgated two ordinances on November 15, 2022 for extending the tenure of Directors of the ED and CBI 5 years for up to Union Cabinet chaired by Prime Minister Narendra Modi on November 11, 2022 has approved the declaration of November 15 as Janjatiya Gaurav Divas

https://sscstudy.com/

Who was launched the Indian Space Association (ISpA) via videoconferencing on October 11, 2022. Prime Minister Narendra Modi

Thanjavur, Tamil Nadu



7



Bhiwani district (Haryana) ■

The Atal Innovation Mission (AIM) of NITI Aayog has launched a Digi-book named ………. on October 26, 2022. Innovations for You

Institute, India (WRI India)

Swachh Bharat Mission (Grameen) ■

Union Minister Social Justice and Empowerment, Dr Virendra Kumar has launched National Helpline on December 14, 202 1against atrocities on

and Ganga ghats ■

Every Year ■

PM Narendra Modi laid the foundation stone and dedicated various National Highway and Road projects on November 9, 2022 to the nation in

SCs, STs ■

Chandigarh University ■



Balrampur, Uttar Pradesh ■

PM NarendraModi inaugurated the 25th National Youth Festival on Jan 12, 2022 via video conferencingin Puducherry Defence Minister Rajnath Singh inaugurated Kalpana Chawla Centre on January 10, 2022 for Research in Space Science & Technology at

Prime Minister NarendraModi has inaugurated the SaryuNahar National Project on December 15, 2021 in



Union Minister of Law and Justice, KirenRijiju has inaugurated the first Sports Arbitration Centre of India on Oct 4, 2022 at Ahmedabad in Gujarat



India on October 14, 2021 re-elected for which council?



Who has topped the IIFL Wealth Hurun India Rich List 2021?

United Nations Human Rights Council

Mukesh Ambani ■

The world’s largest Khadi national flag has recently been unveiled in Ladakh

https://sscstudy.com/

ONELINER CURRENT AFFAIRS



The Greenfield Sindhudurg Airport has been inaugurated in which state under the UDAN Scheme? Maharashtra



Which cartoon character has been roped in as the official mascot for the Namami Gange Programme?



Which UT has topped among union territories in the FSSAI’s 3rd State Food Safety Index 2021?





The VAYO NAMAN Programme was organised by Ministry of Social Justice



and Empowerment  ■

The Alibaug white onion of which state has recently received the GI tag? The 2021 State of the Education Report (SOER) for India: “No Teacher, No Class”has been launched by which UNESCO India organisation?



Which state has launched Vatan Prem Gujarat Yojana?



Which state government has launched ‘Mera Kaam Mera Maan’ Scheme to Help Unemployed Youth? Punjab



Which state will host the Defence Expo Gujarat 2022?



Who has been elected as the new Chief Minister of Gujarat? Bhupendra Patel



In which state, Nirmala Sitharaman has inaugurated ‘My Pad, My Right’ project? Tripura





Who has been featured in Time Magazine’s list of ‘100 Most Influential PM Narendra Modi, People of 2021’? Who has been appointed as Punjab’s new Chief Minister?







Which state’s Hathei chilli and Tamenglong mandarin orange has been given the GI tag? Manipur



The BIMSTEC Summit 2022 was held on March 30, 2022 in Virtual mode. It was hosted by Sri Lanka



The UN Sustainable Development Solutions Network (SDSN) published the World Happiness Report 2022 on March 18, 2022. In this, India is ranked at 136th Place



India has been ranked at ………. place in the International IP Index 2022 released by US Chamber of Commerce on February 25, 2022 and US has topped. 43rd



India and France have inked a roadmap on February 22, 2022 to enhance their bilateral exchanges on the blue Ocean Governance economy and



Prime Minister NarendraModi delivered the inaugural address at World Sustainable Development …….. via video conferencing on February 16, Summit 2022.



Who has become the first country in the world to allow unmanned aircraft vehicles (UAVs/Drones) to operate in Israel civilian airspace?



Military officer Paul-Henri Sandaogo Damiba appointed as interim President on February 12, 2022 of Burkina Faso



Norway has topped and India is placed at ….. position, among 167 nations, in the EIU’s 2021 Democracy Index. 46th



………. tops in terms of digital skills readiness as per Salesforce Global India Index on February 7, 2022.



North Korea successfully tested its Hwasong-12 intermediate-range ballistic missile on January 30, 2022 from the Jagang Province area

Which state has government has launched ‘Mission Vatsalya’ for women who lost their husbands to COVID-19?

International Affairs

Maharashtra

PM Narendra Modi launched……, an e-voucher- based digital payment e-RUPI solution, on August 2, 2021.



Which state has celebrated Wanchuwa Festival 2021? Assam



The Parliament on July 27, 2021 passed which bill, aims to replace over 90-year-old Lighthouse Act, 1927? Marine Aids to Navigation Bill, 2021



The Central government on July 19, 2021 decided to set-up which institute in Noida (Gautam Buddha Nagar)? Indian Institute of Heritage









Which state has become first state to recognise Community Forest Resource Rights in Urban Region? Chhattisgarh Which state government has renamed Kakori Train Conspiracy to Kakori Train Uttar Pradesh Action?

Education Minister Ramesh Pokhriyal ‘Nishank’ launched on April 12, 2021 the ‘Implementation Plan for School SARTHAQ Education’, called



Charanjit Singh Channi ■



Which Airport has been renamed as Maharaja Agrasen International Hisar Airport Airport?

Mamata Banerjee, Adar Poonawalla ■

Which year has been set as the target for becoming 'energy independent' by PM Modi? 2047

NITI Aayog on April 13, 2021 launched a national digital repository on health and nutrition, called Poshan Gyan



Orang National Park

India’s 1st Emergency Landing Facility on National Highway has been inaugurated in which state? Rajasthan



In which city, Narendra Singh Tomar has inaugurated world’s second-largest refurbished gene bank? New Delhi

The name of Rajiv Gandhi national park in Assam has been changed to



India’s highest altitude herbal park situated at a height of 11,000 feet was inaugurated in Uttarakhand

The Union Minister for Commerce and Industry Piyush Goyal inaugurated …………… on April 13, 2021 to provide a platform for connecting aqua farmers and buyers. e-SANTA



Maharashtra ■



Jammu & Kashmir

Chacha Chaudhary

The Ministry of Ayush launched which mobile app in the event for the 7th International Day of Yoga on June 11, Namaste Yoga 2021, virtually. Which State Government of has officially notified the Dihing Patkai Wildlife Sanctuary as a National Park on June 9, 2021? Assam Ladakh LG RK Mathur launched which scheme to Distributes 12300 Tablets Among Students of Government School on June 6, 2021? YounTab Raimona Reserve Forest in Assam has been upgraded as the National Park. It will now be the Sixth National Park in Assam



3

Union Minister Piyush Goyal launched which scheme on April 19, 2021 to provide financial assistance to startups? Startup India Seed Fund

https://sscstudy.com/

https://sscstudy.com/

4 ■





CUET (UG) Section III : General Test

PM of Portugal ……… , has been re-elected after his centre-left Socialist Party secured landslide victory in the 2022 Portuguese legislative electionon January 30, 2022. Antonio Costa The world’s largest canal lock has been inaugurated on January 28, 2022 at Ijmuiden, a small port city, in the Port of Amsterdam, The Netherlands PM of India NarendraModi hosted the first ………. on 27 January 2022, through virtual platform.

to a four-and-a-half day starting January 1,2022. The UAE ■











The Russian, Chinese and Iranian navies undertook naval exercises ………… in the Gulf of Oman from January 18 to 22, 2022. CHIRU-2Q22 ……….. delivered all the contracted 70,000 AK-203 Kalashnikov assault rifles to the Indian Armed Forces on Russia January 25, 2022. The Indonesian parliament has approved a bill on January 21, 2022to change the capital of Indonesia from Nusantara Jakarta to India Extended $ 190-million line of credit to ………. for the Metro Express Project on January 20, 2021.









Mauritius ■

The Oxford University Press (OUP) has picked up ……… as the Children’s Word of the Year 2021, based on their recent research on January 20, 2021.



Anxiety ■

PM Narendra Modi addressed the ……….. Davos Agenda Summit 2022, through video-conferencing on January 17, 2022. World Economic Forum’s (WEF’s)



The iconic ‘Infinity Bridge’ in ………. was formally opened to traffic for the first time on January 16, 2022.





Antigua and ……… has officially joined the International Solar Alliance (ISA) as a member country on Barbuda January 5, 2022. ………. has assumed the rotating presidency of the Council of the European Union with effect from France January 1, 2022. ……….. becomes first country to change its existing five-day workweek

Russian Navy successfully test-fired the ………… Hypersonic Cruise Missile from Frigate – Admiral Gorshkov warship in the Russian Arctic waters on November 22, 2021. Zircon



The WePOWER India Partnership Forum was held on November 17, 2021, to boost the South Asia

Omicron India

35-year-old ……… , has won Chile presidential elections, to become the youngest-ever President-elect of Chile on December 22, 2021. Gabriel Boric ………. was sworn in as new Austria’s chancellor by Alexander Van der Bellen, President of Austria in Hofburg Palace, Vienna, Austria on December 20, 2021. Karl Nehammer Dubai became the world’s first government to turn 100% paperless on December 15, 2021, the announcement was made by UAE Crown Prince Indian-American political advisor GautamRaghavanelected as the head of the …………. of Presidential Personnel White House Office on Dec 13, 2021. United Nations General Assembly (UNGA) has granted Observer Status to ………. by adopting resolution 76/123 on December 10, 2021. German lawmakers ……….. selectedas the new chancellor on December 9, 2021, putting an end to 16 years of conservative rule under Angela Merkel.

Women in Power Sector Professional Network (WePOWER) in India ■

……….. successfully launched a new satellite named Shijian-21 on China October 28, 2021.



India, Israel, United Arab Emirates and ………. have decided to launch a new quadrilateral economic forum on October 21, 2021. The USA



……….., the Leader of the Labour Party in Norway, has assumed the charge of the Prime Minister of Norway with effect from October 14, 2021. Jonas Gahr Store



German rail operator, Deutsche Bahn and industrial group, Siemens launched the first automated & driverless ……… of the world October 12, 2021. train



Colonel Mamady Doumbouya has been sworn-in as the Interim President of Guinea which country?



What is the rank of India in the latest Henley Passport Index for 2021? 90th



The Multidimensional Poverty Index (MPI) Report is released by UNDP



Which University/Institute has topped the ‘THE World University Ranking 2022’? University of Oxford



Which country will host the G-20 leaders’ summit in 2023? India



With which country, India has launched the revamped Strategic Clean Energy Partnership (SCEP)? USA



Aziz Akhannouch has been appointed as the new Prime Minister of Morocco



‘AUKUS’ is a new security partnership for Indo-Pacific between



Who has been chosen as the new Prime Minister of Japan? Fumio Kishida

Olaf Scholz ■

Dubai (United Arab Emirates) ■



…….. has launched Inmarsat-6 F1, a communication satellite, through a Mitsubishi Heavy Industries (MHI) H-IIA204 rocket on Dec 28, 2021. Japan

International Solar Alliance (ISA) ■

World Health Organisation (WHO) has classified the new COVID-19 variant B.1.1.529 on November 29, 202 1as

Egypt has been inducted on Dec 29, 2021as the fourth new member of the BRICS New Development Bank (NDB)

India-Central Asia Summit ■

Japan has introduced a minibus lookalike world’s first dual-mode vehicle (DMV)on December 29, 2021 in Kaiyo its town of







The President of Gambia, ………….., won the second term as President during Gambia’s presidential election Adama Barrow on December 7, 2021. Who has become the World’s newest republic, around 400 years after it became a British colony on Barbados December 1, 2021? PetrFiala has been sworn in as the new Prime Minister of the Czech Republic on Nov 29, 2021by President MilosZeman

https://sscstudy.com/

USA, UK and Australia

https://sscstudy.com/

ONELINER CURRENT AFFAIRS



Which country has become the 9th member of the Shanghai Cooperation Organisation (SCO)? Iran





What was the rank of India in Global Innovation Index 2021? 46th





Narendra Modi has become the first Indian PM to chair ……… debate on August 9, 2021.



The ……….. was India’s top trading partner in the calendar year 2021 with a trade of $ 112.3 billion. US

Which country has launched the first Arctic-monitoring Satellite ‘Arktika-M’? Russia



…….. has launched a new mobile app named Saa?thi on investor education on January 19, 2022. SEBI



Which nation’s President has signed a law enabling him to run for President Russia again?





Who is the new President of Vietnam elected on April 5? Nguyen Xuan Phuc

The RBI has included ……….. Payment Bank in the ‘Second Schedule to the Reserve Bank of India Act, 1934’ on Airtel January 4, 2022.



The global cryptocurrency app Crypto Wire has launched India’s first index of cryptocurrencies named ……… on IC15 January 3, 2022.



Who has issued the guidelines for facilitating small value digital transactions in offline mode on January 1, 2022? Reserve Bank of India



……… and Payment Services Ltd. has collaborated with Paytm for card tokenisation to protect data of the cardholders on Jan. 1, 2022. SBI Cards



The RBI has extended the card-on-file (CoF) tokenization deadline on December 27, 2021 by 6 months i.e., to

Switzerland

United Nations Security Council (UNSC) ■

India has launched a tech platform named ‘UNITE Aware’, in collaboration with United Nations



Nikol Pashinyan has been re-appointed as the PM of Armenia



Which country has become the 5th country to sign International Solar Alliance (ISA) Framework Agreement? Germany









Which European country ratified the Framework Agreement of the International Solar Alliance (ISA) on July 22, 2021? Sweden





Which nation has registered the world's first COVID-19 vaccine for animals?





India has jointly set up a friendship park with which nation? Republic of Korea Arouca Bridge 516, the world’s longest pedestrian suspension bridge and was officially inaugurated on May 2, 2021 Portugal in northern



Which country is building the world’s first platypus sanctuary? Australia



PM Narendra Modi participated in the ‘Leaders’ Summit on Climate’ on April 23, 2021, hosted by President of Italy has launched pilot project named “The Mega Food Park” virtually on April 17, 2021 at Fanidhar Mega Food Park, in Gujarat

June 30, 2022 ■

New Development Bank (NDB) will become the first multilateral agency to open an office in the Gujarat



JP Morgan (largest bank in the USA) announced that it has become the …………. to enter in the metaverse on First Bank February 18, 2022.



RBI reopens Voluntary Retention Route (VRR) on February 10, 2022. With investment limit of ` 250000 crore



The NPCI and UPI ecosystem observed February 1 to 7,2022 as ‘UPI Safety and Awareness Week’ and the whole of February as





As per the first revised estimates released by NSO, the GDP contracted by ……….. in FY21 (2020-21). 6.6% Reserve Bank of India has canceled the license of Independence Co-operative Bank Ltd., ………….. w.e.f. February 3, 2022. Nashik, Maharashtra

https://sscstudy.com/

Bank of Baroda has announced on December 23, 2021, that it has won the first position in overall digital transactions amongst large banks for FY 2020-21



Paytm Payments Bank has been given the scheduled bank status on Dec 10, 2021 by the Reserve Bank of India



The Labour ministry has released the new series of wage rate index (WRI) with the base year being ……….. on 2016 November 26, 2022.



The RBI has issued a revised ……………. Framework on November 5, 2022 for banks to enable supervisory intervention at “appropriate time”.

UPI Safety and Awareness Month ■

The RBI has announced on December 23, 2021 the empanelment of CSB Bank, a private sector lender as an Agency Bank

International Finance Tech (GIFT) City

USA Joe Biden ■

The Reserve Bank of India (RBI) on March 11, 2022 barred which Payments Bank from opening new accounts with immediate effect? Paytm Payments Bank

Russia ■

RBI Governor Shaktikanta Das inaugurated the ......... in Bengaluru on March 24, 2022. Reserve Bank Innovation Hub (RBIH)



Stockholm International Peace Research Institute (SIPRI) released the SIPRI Yearbook 2021 on June 14, 2021

The ICICI Bank Ltd. and HDFC Bank on March 28, 2022 signed an agreement for a 15% state in India Debt Resolution Company Limited

According to SIPRI Yearbook 2021 on June 14, 2021, India has ……. highest nuclear weapons stockpile while United States tops. sixth



Russia has become the first country in the world to register the coronavirus vaccine for animals on April 5, 2021, named Carnivac-Cov

Economy & Banking ■

Who has been appointed as the new Prime Minister of Lebanon, in place of Saad Hariri, on July 26, 2021? Najib Mikati

Which country has banned the Full-Face Coverings in Public Places?

5

Prompt Corrective Action (PCA) ■

Which Indian Conglomerate has acquired the  financial services Piramal Enterprises company, DHFL?



What is the interest rate fixed on the Post Office Savings Account in the third quarter (Oct-Dec) of 2021-22? 4%

https://sscstudy.com/

6

CUET (UG) Section III : General Test



What is the interest rate fixed on Sukanya Samriddhi Accounts for Quarter-3 (Oct-Dec) of 2021-22? 7.6%



Which company has recently acquired 100% stake in digital lending startup Credit Mate? Paytm



Which bank has launched NAV-eCash card for Indian Navy on INS Vikramaditya? SBI



Which company has recently launched the ‘postpe’ solution based on ‘Buy Now Pay Later’ platform? BharatPe













The RBI joined the Central Banks and Supervisors ………. as a Member on April 26, 2021. Network for Greening the Financial System



Which bank has been authorized by the Reserve Bank of India (RBI) to act as an ‘Agency Bank’?

The Reserve Bank of India (RBI) has extended the deadline for processing auto-debit payments till Sept. 30, 2021



Who topped the Forbes billionaires list 2021 with a net worth of $177 billion?















Which digital payments platform has received direct broking license from IRDAI? PhonePe The Central government on July 22, 2021 approved the PLI scheme for which product? Specialty Steel



What is the projected GDP of India for FY22 according to the Economic Survey 2021? 11%

The World Bank on March 24, 2022 created the world’s first kind of ........., also known as the ‘Rhino Bond’ in supoort of South Africa’s efforts to protect the endangered species? Wildlife Conservation Bond (WCB)





IIT-Hyderabad has launched an AI-based job portal named ……….. that helps people with disabilities acquire relevant skills and find jobs. Swarajability



Indian Institute of Science (IISc) commissioned on February 2, 2022, one of India’s most powerful supercomputers Param Pravega



The Global Center of Excellence in Affordable and Clean Energy (GCoE-ACE) has been inaugurated on January 28, 2022 at IIT, Dharwad in Karnataka





Delhi-Jaipur National Highway

The Indian armed forces have signed a contract with the Swedish defence company ………. for the supply of single-shot anti-armour weapon AT4 on January 24, 2022. Saab



ISRO Successfully Tests Vikas engine that would power India’s first humancarry rocket on January 22, 2022 in

The government of India has organised a week-long science exhibition titled ………. from February 22 to 28, 2022. Three more Rafale fighter aircraft landed in India from France on Feb 22, 2022, with

Mahendragiri, Tamil Nadu ■

The Israeli Defence Ministry conducted successful test flight of the ……..… anti-ballistic missile system and its interceptors on Jan 18, 2022. Arrow-3



Indian Navy and Russian Navy undertook ………… on January 14, 2022 at the port of Cochin, in the PASSEX Arabian Sea.



DRDO successfully test-fired an extended range sea-to-sea variant of the BrahMos supersonic cruise missile on January 11, 2022 for the

Indian specific enhancements ■





The Indian Navy received the 12th anti-submarine warfare aircraft P-8I from the US-based aerospace company Boeing ………. on February 23, 2022. ……….. notified Green Hydrogen and Green Ammonia Policy on February 20, 2022. Ministry of Power India’s first commercial-scale biomassbased hydrogen plant will come up at Khandwa (Madhya Pradesh)

https://sscstudy.com/

India’s largest electric vehicle (EV) charging station has been inaugurated in Sector 52 of Gurugramon January 28, 2022 at



Vigyan Sarvatra Pujyate

The LIC on July 20, 2021 launched a new health insurance scheme by which name? Aarogya Rakshak El Salvador has become the first country in the world on June 7, 2021 to bitcoin grant legal tender status to

India has become the first country in the world on February 7, 2022to administer a DNA vaccine named ZyCoV-D (ZydusCadila, Ahmedabad) against COVID-19

Science & Technology

……… has allowed payments banks to act as investment bankers. SEBI Which organization has introduced ‘Regulatory General Anti-Avoidance Rule (GAAR)’ for round tripping? RBI

Pacific Ocean

Which bank has launched ‘UNI– CARBON CARD’ with HPCL? Union Bank of India

NASA planned on February 7, 2022 to retire the International Space Station by 2031 by crashing it into the



Jeff Bezos

Induslnd Bank ■

According to the budget 2021-22, Pensioners (senior citizens) over ............. of age are exempted from filing 75 years returns.



S&P Global Ratings has projected India’s GDP for FY22 at 9.5%

Reliance Industries ■

Who has launched its first Overseas Fund? SBI Mutual Funds

Which bank has topped the MeitY Digital Payment Scorecard for 2020-21?

Which is the highest-ranked Indian company on the Fortune Global 500 list for 2021 in terms of revenue?



Shivalik Small Finance Bank Ltd.

What is the current policy repo rate? 4%

Which Bank has launched ‘Dukandar HDFC Bank Overdraft Scheme’?

The UP-based ……….. commenced operations as a SFB with effect from April 26, 2021.



………… buys 3.9% stake in Bank of India via open market acquisition. LIC

Bank of Baroda ■



Indian Navy ■

Covid vaccine of children between the age group of ………. started from January 3, 2022. 15 to 18 years

https://sscstudy.com/

ONELINER CURRENT AFFAIRS



INS Khukri, the first indigenously built Missile Corvette, was decommissioned on December 29, 2021 after 32 years of service at



The 8th edition of the Exercise Mitra Shakti-21 was held between the Indian Army and Sri Lanka



Name the web portal launched by the government to find solutions to India’s waste problems, through community participation? Waste to wealth

Visakhapatnam, Andhra Pradesh ■

Indian Army has launched on December 27, 2021a contemporary messaging application named



ASIGMA (Army Secure IndiGeneous Messaging Application) ■





The nuclear-capable ballistic missile ‘Agni Prime’ was successfully tested off the coast of Odisha on December 20, 2021 by DRDO India’s first commercial-scale biomass-based hydrogen plant will come up on December 13, 2021 at the Khandwa district of Madhya Pradesh The 3rd edition of the Trilateral Maritime Exercise named SITMEX–21 held from 15 to 16 Nov. 21 of India, Singapore and Thailand in









The seventh Indian Navy frigate of the P1135.6 class was launched at the Yantar Shipyard on November 13, 2021 at Kaliningrad in Russia A new Indian Coast Guard Ship (ICGS) ………… has been dedicated to the nation on October 28, 2021. Sarthak DRDO has carried successful test-firing of surface-to-surface ballistic missile Agni-5 on October 27, 2021from







Armed Forces of India and the United Kingdom (UK) launched Tri-Service exercise ‘Konkan Shakti 2021’ from October 24 to 27, 2021 off the Konkan Arabian Sea coast in the The fifth edition of India-Japan Maritime Bilateral Exercise ………. was held in the Arabian Sea from October 6 JIMEX to 8 ,2021. The 8th edition of the India – Sri Lanka bilateral joint Exercise “Mitra Shakti-21” conductedfrom October 4 to 15, 2021at Combat Training School, Ampara in Sri Lanka The second phase of multilatral maritime exercise ‘Malabar 2021’ concluded on October 15, 2021 in the Bay of Bengal

IIT, Ropar has developed a device dubbed as ……… which can be used as a substitute of Continuous Positive Airway Pressure machine. Jivan Vayu



JIMEX 2021 is the 5th edition of the annual bilateral maritime exercise of Japan India with which country?

Indian Navy’s oldest hydrographic survey ship INS Sandhayak was decommissioned on June 7, 2021 at Naval Dockyard in Visakhapatnam



The first ever automated, driverless train in the world has been unveiled in which country? Germany

IIT Ropar develop ‘FakeBuster’ tool to detect imposters in video conferences & social media manipulated Faces on



India is developing a mobile tech platform called ‘UNITE AWARE’, to make use of technology in the safety UN peacekeepers and security of



Indian Navy’s first destroyer ……… decommissioned after 41 years of service on May 21, 2021 INS Rajput



The 19th edition of the Indian and French Navy bilateral exercise ……….. was conducted in the Arabian Sea from April 25-27 2021. VARUNA-2021

Indian Army contingent participated in Russia Exercise ZAPAD 2021 with



Name the typhoon that has recently hit Taiwan. Chanthu



Which Indian frigate has conducted maritime partnership exercise with Sudanese Navy in Red Sea? INS Tabar



SpaceX has launched the first all-tourist crew into orbit onboard Crew Dragon capsule. Name the three days Inspiration4 mission.

Sports ■



Name the Cyclonic Storm which has recently affected the state of Odisha and Andhra Pradesh. Gulab



36th edition of CORPAT held between Bangladesh India and



Which company has launched ‘AI For All’ initiative in collaboration with CBSE, Ministry of Education? Intel



Which armed force has recently inducted its first two women officers (Prakriti and Diksha) in combat for the first time? Indo Tibetan Border Police

APJ Abdul Kalam Island in Odisha ■





Andaman Sea



What name has been given to India’s Evacuation Mission from Afghanistan?



Which new generation missile was successfully flight-tested by the DRDO Akash-NG on July 23, 2021?



Which Indigenously developed missile was successfully test-fired by the DRDO on July 21, 2021? Man Portable-Anti Tank Guided Missile

https://sscstudy.com/

Indian badminton sensation PV Sindhu on March 27, 2022 won women’s singles title at which tournament Swiss Open 2022



India concluded its campaign at the Singapore Weightlifting International 2022 with ……… medals, including six golds on February 27, 2022. eight



Haryana defeated the Indian Railway, to win Men’s title in ……….. on February 13, 2022.

Which organisation on July 23, 2021 announced to set-up 35 new centres in the country by December 2021? National Centre for Seismology (NCS)

Which team won the prestigious Obaidulla Khan Heritage Hockey Cup Tournament 2022, that was held in Bhopal (Madhya Pradesh) from March 21-27, 2022? Railway Sports Promotion Board (RSPB)



Operation Devi Shakti ■

7

Senior National Volleyball Championship 2021-22 ■

In women’s category of Senior National Volleyball Championship, ……….. defeated the Indian Railway, to lift the trophy on February 12, 2022. Kerala



English club Chelsea defeated ……….. Palmeiras, 2-1, to win the FIFA Club World Cup final 2021, held on February 12, 2022. Brazilian club

https://sscstudy.com/

8 ■





CUET (UG) Section III : General Test

The Rajasthan CM Ashok Gehlot and BCCI President SouravGanguly laid the foundation stone of world’s thirdlargest cricket stadium on February 5, 2022 in Jaipur

Indian teenager Unnati Hooda beat compatriot Smit Toshniwal on January 30, 2022, to claim the women’s singles title at

Argentina defeated Chile on January 28, 2022 ,to win its 6th women’s field hockey championship title at









Japan defeated South Korea in the final on January 28, 2022 to win their 3rd title at Women’s Hockey Asia Cup tournament 2022 and India defeated bronze medal China to win The women’s team from Ladakh has lifted the 9th National Women’s Ice Hockey Championship on January 21, Himachal Pradesh 2022 in In badminton, ace Indian shuttler PV Sindhu bagged the women’s singles title at the Syed Modi International Tournament on January 23, 2022, in Bharath Subramaniyam from Chennai has become on January 9, 2022 the











Indian badminton player LakshyaSen defeated World Champion Loh Kean Yew of Singapore on Jan 17, 2022, in the men’s singles final of India Open 2022 ICSD has given its approval to the All India Sports Council of the Deaf (AISCD) on January 16, 2022 to organise the World Deaf World number six Rafael Nadal won the men’s singles tennis title on January 9, 2022 at 2022 Melbourne Summer Set 1

India won a total of six medals, including two gold and 4 silver medals in the Asian Rowing Championship on December 22, 2021 in Thailand India claimed 41 medals (12 Gold, 15 Silver, 14 Bronze) at the 4th Asian Youth Para Games (AYPG) on December 11, 2021, Asia’s biggest event Riffa city (Bahrain) held at In cricket, Tamil Nadu has lifted the Syed Mushtaq Ali Trophy defeating Karnataka on November 24, 2021 after chasing 152 in India ends with 7 medals at 2021 Asian Archery Championships Karnataka on Nov. 22 ,2021 in Dhaka (Bangladesh)



Australia won their maiden T20 World Cup title as they defeated New Zealand in the final by 8 wickets on Dubai November 14, 2021 in







T20 Cricket Championship in 2023 ■

Shuttler Kidambi Srikanth became the first Indian man to win a silver medal on December 22,2021 at

Arun Jaitley Stadium (Delhi)

India’s first para-badminton academy has been set up in Lucknow, Uttar Pradesh

Senior Indian off-spinner, Harbhajan Singh has announced retirement on December 27, 2021 from all forms of



73rd Grandmaster of India ■

India has lifted the under-19 Asia cricket Cup by defeating Sri Lanka by nine wickets on January 3, 2022 in a rain-interrupted One-Day International final in Dubai

BWF World Championships

Lucknow ■



Which team has won the Durand Cup 2021? FC Goa



Name the winner of the inaugural Meltwater Champions Chess Tour Magnus Carlsen (MCCT).



Name the hockey players who have won the FIH Women’s Player of the Year and  FIH Men’s Player of the Year 2020-21? GurjitKaur and Harmanpreet Singh



cricket ■

Women’s Pan American Cup 2022 ■

Rohan Bopanna and Ram kumar Ramanathan (India) won on January 8, 2022 2022 Adelaide International 1 Tennis Tournament

Indian men’s hockey player ……….. has won the World Games Athlete of the Year award for 2021on January 28, 2022. PR Sreejesh

Odisha Open badminton tournament 2022 ■



China beat Japan 3-1 in a thrilling final in Aarhus, Denmark, on October 20, 2021 to reclaim the Uber Cup Chennai Super Kings (CSK) has defeated Kolkata Knight Riders (KKR) on October 17,2021 in the finals to win 2021 IPL Title the Which franchise won the title of IPL 2021 after beating KKR in the final on October 15, 2021?

IBHA ■

Which country will host the FIFA U-17 Women’s World Cup in 2022? India



Praveen Kumar has clinched silver in Paralympics 2020. He is related to which sport? High Jump



Who has won bronze medal in archery in men's individual recurve open for India at the Tokyo Paralympics 2020? Harvinder Singh



Who has won the formula one Dutch Grand Prix 2021? Max Verstappen



Who has been named the ICC Men’s player of the month for August? Joe Root



Who has won the women’s singles title of US Open 2021? Emma Raducanu



Which country has been suspended from 2022 Beijing Winter Olympics by the IOC? North Korea



Who has won the Formula One Italian Grand Prix 2021? Daniel Ricciardo



Who has won Norway Chess Open 2021 Masters section? D. Gukesh



Name the Formula 1 race driver who has won the Hungarian GP 2021. Esteban Ocon



Who has become India’s 70th Chess Grandmaster? Raja Rithvik



Which country has won CONCACAF Gold Cup 2021 in Football? USA



India men’s hockey team defeated …… to won the bronze medal at the Tokyo Germany Olympics on 5th August.



………… has become the first Indian to win Olympic Gold medal in athletics.

Chennai Super Kings (CSK) ■

Indian player Rupinder Pal Singh has recently announced retirement from which sports? Hockey

https://sscstudy.com/

What is the name of the official mascot for U-17 Women’s World Cup 2022?

Neeraj Chopra

https://sscstudy.com/

ONELINER CURRENT AFFAIRS



33rd Summer Olympics Games in 2024 will be hosted in which city?



Paris, France ■



Which country has won the maximum number of medals in Tokyo Olympics 2020? US Who has been named the ICC Men’s player of the month for July?





Who has become first Indian woman to win two Olympic medals in badminton PV Sindhu sports? PV Sindhu defeated ‘He Bingjiao’ from ……….. by 21-13, 21-15 in women’s singles bronze medal match at Tokyo Olympics 2020. China Which cricket team won the inaugural ICC World Test Championship 2019-21 after beating India on June 23, 2021? Who has won the 2021 French Grand Prix, held on June 20, 2021?

Who has been won the 2021 Azerbaijan Grand Prix, held on June 6, 2021

FIFA has suspended which nation’s football federation? Pakistan



Which football team has won the ISL League Football Winners Shield?







Tamil Nadu teenager Arjun Kalyan on April 20, 2021 has become the



Casper Ruud of Norway beat 15thranked Denis Shapovalov of Canada on May 25, 2021 to win Geneva Open 2021 Max Verstappen (Red Bull-Netherlands) has won the……, held on May 23, 2021.





American professional golfer………., created history by winning the 2021 PGA Championship, at the age of 50 on Phil Mickelson May 23, 2021. English professional player ………. has become the World Snooker Champion 2021 for the fourth time on May 3, 2021. Mark Selby

Which State/UT has topped the medals tally at the 2nd Khelo India Winter National Games in Gulmarg? Who has won the BBC’s Indian sportswoman of the year award?



Which Indian Olympic medallist weightlifter won the ‘BBC Indian Sports Woman of the Year 2021’ award on March 28 2022?



American mathematician Dennis Sullivan has been awarded with which prize for his contributions to topology and dynamic systems, on March 25, 2022? Abel Prize 2022



Physicist Professor, ……….. has become the first Indian to be awarded the Boltzmann Medal on March 2, 2022. Deepak Dhar



The ICC has selected New Zealand batter Daryl Mitchell on February 2,2022as the winner of the



The Forensic Science Laboratory (FSL) won ………… in the Silver category for its work towards combating Crime and Violence against Children on SKOCH Award January 10, 2022.



The Power of the Dog and the West Side Story, won 3 each awards in 79th edition of ………… ceremony held on January 9, 2022. Golden Globe Awards 2022



Zishaan A Latif won …….. in the Photo Journalism category on January 5, Ramnath Goenka Award 2022.



People for the Ethical Treatment of Animals (PETA) India has named Bollywood star ………… its 2021 Person of the Year on December 29, 2021. Alia Bhatt



PM NarendraModi unveiled hologram statue of NetajiSubhas Chandra Bose at India Gate on December 26, 2021 on

ICC Spirit of Cricket Award 2021 ■



The government of Assam has conferred its highest civilian award ….…… on Industrialist Ratan Tata on Assam Baibhav January 27, 2022. The Padma Award 2022 has been conferred upon 128 winners on January 26, 2022, which include 4 Padma Vibhushan, 17 Padma Bhushan 107 Padma Shri and

https://sscstudy.com/

The 2015 film BajrangiBhaijaan fame actress, Harshaali Malhotra has been awarded on January 11, 2022 the 12th Bharat Ratna Dr. Ambedkar Award 2022

Mirabai Chanu ■

37 Years old Shaylyn Ford was crowned on January 15, 2022 as the winner of Mrs World 2022

Awards & Honours ■

Former Japan Prime Minister Shinzo Abe was conferred with the Netaji Award 2022 on January 23,2022 by Netaji Research Bureau



Who is the 1st Indian woman cricketer to score 10000 international runs? Mithali Raj

2021 Monaco Grand Prix ■

20th Dhaka International Film Festival

Koneru Humpy

68th Chess Grandmaster of India ■

PS Vinothraj directed film Koozhangal from India won the best film award in the Asian Film Competition section on January 24, 2022 at the

Jammu & Kashmir ■

The IBF, the apex body of broadcasters in India, is being renamed on May 29, IBDF 2021 as Name a Hong Kong-based mountaineer, has created the record of world’s fastest ascent of Everest by a woman, within the time of just under Tsang Yin-hung 26 hours.



Mumbai City FC

Sergio Perez (Mexico- Red Bull) ■

Who has been selected for the Chhattisgarh Veerni Award?



Max Verstappen (Netherlands-Red Bull) ■

President Ram Nath Kovind approved conferment of the Jeevan Raksha Padak Series of Awards 2021on January 25, 2022, a civilian lifesaving award, to 51 brave citizens of the country

Dutee Chand

New Zealand ■



Stefanos Tsitsipas (Greece) ■

Shakib Al Hasan ■

Who won Monte Carlo Masters 2021 in Tennis and captured maiden Masters 1000 title on April 20, 2021.

9

Parakram Divas ■

DivyaHegde, an Indian Climate Action Entrepreneur from Udupi, Karnataka, has won UN Women’s Award for Leadership Commitment



Seven-time Formula One champion received, …….., a knighthood at Windsor Castle, London on December 17, 2021. Lewis Hamilton

https://sscstudy.com/

10 ■

CUET (UG) Section III : General Test

Actor-model ………… made history as she was crowned Miss Universe 2021, 21 years after India last brought home the title on December 12, 2021. Harnaaz Sandhu



Assamese poet Nilmani Phookan Jr. won the 56th Jnanpith Award and Konkani novelist ………. won the 57th Jnanpith Award on December 8, 2021.



Nizamuddin Revival Project of NizamuddinBasti community, in New Delhi has won the UNESCO Asia-Pacific Awards on December 6, 2021 for Cultural Heritage Conservation 2021



Who has been conferred with the Tata Literature Live! Lifetime Achievement Award for 2021 on November 23, 2021?



South African playwright and novelist, Damon Galgut has won the 2021 Booker Prize on November 8, 2021 for





Who was honoured with the prestigious 51st DadasahebPhalke Award at the 67th National Film Awards ceremony on October 25, 2021?



………… won 22nd Lal Bahadur Shastri National Awardon October 12, 2021? Dr. Randeep Guleria









Name the Ashok Chakra awardee of 2021, who has been given the award Babu Lal posthumously.



Who has won Prof. CR Rao Centenary Gold Medal award?









Who has been awarded the National Youth Award in 2021? Mohammad Azam

National Logistics Excellence Awards ■

International Federation of Film Archives (FIAF) will confer the prestigious 2021 FIAF Award to whom? Amitabh Bachchan



Who has been honoured with International Woman of Courage Award 2021? Gowsalya Shankar



Which singer’s album ‘Folklore’ has won the ‘Album of the year’ award at the 63rd Grammy Awards 2021?



Name the Hindi Writer who has been selected for Vyas Samman 2020.

Taylor Swift

Sharad Pagare ■

Which actor has won the ‘Best Actor (Male)’ award at Dadasaheb Phalke International Film Festival Awards Akshay Kumar 2021?



Who has been chosen to receive Saraswati Samman 2020?



Noted Zimbabwean novelist …………. has been declared as the winner of 2021 Pen Pinter Prize on June 9, 2021. Tsitsi Dangarembga



Dr. Shakuntala Haraksingh Thilsted, an Indian-born global nutrition expert, won the prestigious 2021 World Food Award Who has been honoured with the 51st Dadasaheb Phalke Award? Rajnikanth

Who has won the Bird Photographer of the Year 2021 award? Alejandro Prieto



Philippines Journalist ………honored with 2021 UNESCO/ Guillermo Cano World Press Freedom Prize on May 3, Maria Ressa 2021.

Who has won the Women’s Prize for Fiction 2021 for her novel ‘Piranesi’? ■

Who has been honoured with the ryabhata Award for 2021 by Astronautical Society of India (ASI)?

Sharankumar Limbale ■

Who has received Kirti Chakra award in 2021? Altaf Hussain Bhat



Who is the winner of the fourth Satyajit Ray Award in 2021? BejawadaGopal



The Central government on July 19, 2021 launche which new award?

David Julius and Ardem Patapoutian

Dr. G. Satheesh Reddy

Who has been selected for the prestigious Maharashtra Bhushan Asha Bhosle Award?

Jagdish Bhagwati and C. Rangarajan

Name the winner of the 2021  Nobel Prize for Medicine?

Susanna Clarke



Who has won the Goalkeeper ‘Global Goals Award 2021’? Who has been named the winner of the prestigious National Lokmanya Tilak Cyrus Poonawala Award 2021?

Telugu filmmaker B. Gopal, alias Bejawada Gopal, has been chosen on October 11, 2021for the fourth Satyajit Ray Award

Who has been honoured with the National Florence Nightingale Award 2020? SV Sarasvati

Ghumzile Mlambo-Ngcuka

Rajinikanth ■

Who has been conferred with the Swami Brahmanand Award 2021?



The Promise ■

Who has won the AM Turing Award 2020? Alfred Aho

Anand Kumar

Anita Desai ■



and Environment (LIFE), Delhi

Damodar Mauzo ■

Which Indian organisation has been honored with the 2021 Right Livelihood Award? Legal Initiative for Forest

The Marathi film “Puglya” has won the Best Foreign Language Feature award on April 17, 2021 at

Kangana Ranaut ■

Who has won the Wildlife Photographer of the Year People’s Robert Irwin Choice Award 2021?



Who has been crowned VLCC Femina Miss India World 2020? Manasa Varanasi



Which city’s Police has received the Best Marching Contingent Trophy 2021? Delhi



Who has won the Sanctuary Lifetime Service Award 2020? Theodore Baskaran



Which State’s Chief Minister has got the SKOCH Chief Minister of the Year Award? Andhra Pradesh



Who has won the Nelson Mandela World Humanitarian Award?

Moscow International Film Festival, 2021 ■

Who has been honored with the 51st Dadasaheb Phalke Awards, the highest film honour in India on April 3, 2021? legendary actor Rajinikanth

https://sscstudy.com/

Who has won the Best Actress award at the 67th National Awards 2021?

Ravi Gaikwad ■

Who has won the Best Actor (Male) award at 66th Filmfare Awards 2021? Irrfan Khan

https://sscstudy.com/

11

ONELINER CURRENT AFFAIRS



Which has won the prestigious International King Bhumibol World Soil Day Award 2020?



Indian Council of Agricultural Research ■

Which State’s tableau has won the 1st prize on Republic Day 2021 parade?







Name the former Chief Minister of Sikkim, who passed away on March 28, 2022 at the age of 92 years. Bhim Bahadur Gurung





Who has been appointed as the New MD and CEO of Maruti Suzuki India on March 24, 2022? Hisashi Tekuchi







India’s ................ has been elected the new Chairman of the International Rubber Study Group (IRSG) for a period of two years on February 23, 2022. KN Raghavan







Who has been elected as the President of The Institute of Chartered Accountants of India (ICAI) on February 13, 2022 for the year 2022-23?







Actor and athlete ........ (Bheem in the TV series “Mahabharat”) passes away at the age of 74 years on February 10, 2022. Praveen Kumar Sobti Who has been appointed as the Brand Ambassador of Indian state of Uttarakhand on February 8, 2022?



Who has been appointed as the new director of Vikram Sarabhai Space Centre (VSSC) on February 7, 2022?



Who has been appointed as the official brand ambassador of MediBuddy on February 7, 2022. Amitabh Bachchan

Who has been appointed as the new Advisor to Prime Minister Narendra Modi? Amit Khare



Who has been appointed as the Chairman of Central Board of Direct Taxes (CBDT)? JB Mohapatra

Gopinath appointed as first deputy managing director of ......... from IMF January 21 2022.



Who has become the first woman CMD of Engineers India Ltd.?

Andhra Pradesh girl .......... becomes first Indian to complete NASA’s IASP programme on January 20, 2022.

Vartika Shukla ■

Professional mountaineer, Padma Shri and a retired army officer Major ........….... has passed away on January 19, 2022. Padma Vibhushan awardee,Legendary Kathak maestro, composer and singer, ............ has passed away, due to age-related ailments on January 16, Pandit Birju Maharaj 2022. ………… elected Kuwait’s Haitham Al-Ghais as new secretary general on January 6, 2022. OPEC Alka Mittal becomes first women head of ............... on January 1, 2022. Virender Singh (VS) Pathania took charge on Dec. 31, 2021 as the DirectorGeneral of the Indian Coast Guard (ICG) Somalian Prime Minister ……… has been suspended on December 28, 2021. Senior bureaucrat, Vivek Johri has been appointed on November 30, 2022 as the new Chairman of the Central Board of Who takes charge as CMD of National Research Development Corporation (NRDC) on October 18, 2021?



Who has been appointed as the president of the International Road Federation (IRF) India? Satish Parekh



Name the Uttarakhand Governor who has resigned. Baby Rani Maurya



Name the newly appointed MD & CEO of National Securities Depository (NSDL). Padmaja Chunduru

https://sscstudy.com/

Who has been appointed as joint secretary in Ministry of Cooperation? Abhay Kumar Singh



Oscar Fernandes passed away recently. Politician He was a/an



Who has been appointed as the new CEO of Yahoo? Jim Lanzone



Who has been appointed as the Governor of Manipur? La Ganesan



Who has been appointed as 25th CGA? Deepak Das



Who among the following has become world’s youngest mountaineer to scale Shehroze Kashif K2?



Which BJP leader took oath as the 23rd Chief Minister of Karnataka on July 28, 2021? Basavaraj S. Bommai



Chinmoy Chatterjee passed away recently. He was a ……… . Footballer



Which eminent multilingual Kannada film actress passed away in Bengaluru at the age of 76 years on July 26, 2021? Jayanthi

Amit Rastogi ■

Who has been appointed as the secretary-general of Rajya Sabha? PPK Ramacharyulu

Indirect Taxes and Customs (CBIC). ■

Dr. S. Unnikrishnan Nair ■



Mohamed Hussein Roble

Bollywood actor Akshay Kumar ■

Lieutenant General GAV Reddy has been appointed on February 2, 2022 as the Director-General of the

Oil and Natural Gas Corporation (ONGC) ■

Debashis Mitra ■

Who has been appointed as the first Chairman of the newly set up  Indian Space Association (ISpA)? Jayant Patil

Hari Pal Singh Ahluwalia ■

Chetan Ghate ■

Vivek Ram Chaudhari

Jahnavi Dangeti

Bollywood actor Shah Rukh Khan has been appointed on February 23, 2022 as the brand ambassador for Thums Up Who has been appointed as the new Director of The Institute of Economic Growth on February 20, 2022?

Veteran Indian and Marathi film and television actor, ........ , has passed away on February 5, 2022. Ramesh Deo

Who has been appointed as the new Chief of Air Staff (CAS) of the IAF?



Defence Intelligence Agency

RBI has approved the re-appointment of Rakesh Sharma as Managing Director and Chief Executive Officer of IDBI Bank



Lata Mangeshkar

Uttar Pradesh

In the News ...

Nightingale of India and Queen of Melody .......... has died in Mumbai’s Breach Candy Hospital due to multiple organ failures on February 5, 2022.



Which Indian businessman was declared ‘bankrupt’ by a UK High Court on July 26, 2021? Vijay Mallya

https://sscstudy.com/

12 ■

CUET (UG) Section III : General Test

Which Union Minister was appointed as the Deputy Leader of the House in the Rajya Sabha on July 19, 2021? Mukhtar Abbas Naqvi





Dr Chintan Vaishnav ■

Who has won the 2021 Iranian presidential election on June 18, 2021? Ebrahim Raisi











Name of former Defence Minister, took oath as the Prime Minister of the Israel on June 13, 2021. Naftali Bennett Asian Games gold medal-winning former boxing star …………has passed away, battling liver cancer on June 7, Dingko Singh 2021. Name of Syrian President has been re-elected for the fourth straight 7-year term on May 28,2021. Bashar al-Assad The social media giant Facebook has named ………. as its Grievance Officer for India on its website. Spoorthi Priya Who took the oath as the Chief Minister of Kerala for the second consecutive time on May 20, 2021? Pinarayi Vijayan







AINRC founder leader ……….. has been sworn in as the New CM of Puducherry, for a record fourth time, on May 7, 2021. N. Rangasamy Padma Bhushan awardees Former Attorney General of India, ……….has passed away, battling Covid-19 on Soli Sorabjee April 30, 2021.



Name the Election Commissioner assumed charge with effect from April 13, 2021 as Chief Election Sushil Chandra Commissioner.



Name the US educated female law professor has been elected as the new President of Kosovo on April 7, 2021.



Name the Deputy Governor of RBI who has retired from his service. BP Kanungo

National Assembly of Vietnam has voted ………. as the new Prime Minister of the country on April 5, Pham Minh Chinh 2021. Niger President Mohamed Bazoum has appointed ……….as the new PMof the country effective from April 3, 2021.





Tarun Bajaj has been appointed as the Revenue Secretary







Who has been recommended as the next Chief Justice of India (CJI) w.e.f. April 4, 2021? NV Ramana ■

A short biography of Indian athlete ………… titled ‘Golden Boy Neeraj Chopra’ authored by Navdeep Singh Gill has been released on February 3, Neeraj Chopra 2022.



The Book titled ‘Fearless Governance’ authored by ………. has been released on January 31, 2022. Dr. Kiran Bedi



India’s first innovation centre for graphene is set to come up on January 28, 2022 in Thrissur, Kerala



A new book titled …………., based on West Bengal CM and Trinamool Congress supremo Mamata Banerjee released on January 24, 2022.

Saurabh Garg ■

In which city, International Yoga Festival has been organised? Rishikesh



Name the Indian-American who has been appointed Acting Chief of Staff of NASA. Bhavya Lal



Who has been appointed as the First Chief Compliance Officer of Facebook? Henry Moniz



Who has been appointed acting chief of the Central Bureau of Investigation (CBI)? Praveen Sinha



Who has been appointed as the CEO of Amazon, after Jeff Bezos step down?

Mamata Beyond 2021 ■

India’s “Supermom” tigress, popularly known as ……….. , has passed away at Pench Tiger Reserve (PTR) in Madhya Pradesh, due to old-ageon January 16, Collarwali 2022.



The 18th edition of the Kachai Lemon Festival was held in Manipur on Jan. 13 and 14, 2022 at the Local Ground of Kachai Village in Ukhrul district



India’s first heli-hub with all aviation facilities is proposed on Jan. 12, 2022 to Haryana be built at Gurugram in

Andy Jassy ■



Who has become India’s Youngest Female Pilot at 25? Ayesha Aziz In which state Mandu Festival is celebrated? Odisha

https://sscstudy.com/

The book titled “A Nation To Protect” authored by Priyam Gandhi Mody was Union Health launched by Minister Mansukh Mandaviya

Who has been re-appointed as the Chairman of the Central Board of Direct Taxes? Who has been appointed as the new CEO of Unique Identification Authority of India (UIDAI)?

Award-winning US author Jimmy Soni released on February 22, 2022 his new The Founders : book titled The Story of PayPal and the Entrepreneurs Who Shaped Silicon Valley

Pramod Chandra Mody ■

Indian author Anirudh Suri has come out on February 25, 2022 with his new book titled The Great Tech Game : Shaping Geopolitics and the Destinies of Nations.



Name the first women cricket commentator who passed away on April 4. Chandra Nayudu

Union Minister of Agriculture and Farmers Welfare Narendra Singh Tomar on March 28, 2022 released a book titled Indian Agricultrue Towards 2030 : Pathways for Enhancing Farmers Income, Nutritional Security and Sustainable Food and Farm Systems

Who has been appointed as the new Chairman and Managing Director of SIDBI? S. Ramann



Vjosa Osmani ■

Miscellaneous ■

Ouhoumoudou Mahamadou

BJP leader Himanta Biswa Sarma Replaced ……… as Assam’s New Chief Minister on May 9, 2021. Sarbananda Sonowal





Name the Justice was sworn in as the 48th Chief Justice of India on April 24, 2021. Nuthalapati Venkata Ramana

Who has been appointed as the new Mission Director for AIM?

https://sscstudy.com/

13

ONELINER CURRENT AFFAIRS



Arundhati Bhattacharya, the former chair of the SBI, has come out on Jan. 11, 2022 with her new book titled Indomitable : A Working Woman’s Notes on Work, Life and Leadership.







A new book titled Gandhi’s Assassin: The Making of NathuramGodse and His Idea of India, authored by journalist ………, has been released on January 4, 2022. Dhirendra K. Jha Union Minister Jitendra Singh inaugurated 24th Conference on e-Governance 2020-21 on January 7, 2022 in Hyderabad,Telangana

Ministry of Power is celebrating the Energy Conservation Week from December 8 to 14, 2021 under Who is the author of the book named ‘Chronicles from the Land of the Happiest People on Earth’? Wole Soyinka



What is the theme of the International Day of Older Persons 2021?

Who is the author of the book ‘Human Rights and Terrorism in India’? Subramanian Swamy



What was the theme for World Ozone Day 2021? Montreal Protocol Keeping us, our food and vaccines cool

Digital Equity for All Ages

International Day of Democracy is celebrated on ……… every year.



Which Central Armed Police Force celebrated its 83rd Raising Day on July 27, 2021? CRPF





Who is the author of the book  ‘The Custodian of Trust – A Banker’s Memoir’? Rajnish Kumar 



Who is the writer of the book, titled ‘Know Your Rights and Claim Them : A Guide for Youth’?



Who is the author of the new book titled ‘Balakot Air Strike: How India Avenged Pulwama’? Manan Bhatt



What was the theme of the 75th Independence Day of India?



Angelina Jolie &Geraldine Van Bueren ■





September 15

What was the theme for International Literacy Day 2021?

Which city has become the first Indian city to vaccinate 100% against COVID-19? Bhubaneswar

https://sscstudy.com/

In India, “Muslim Women Rights Day” is observed across the country on August 1

Who is the author of the book titled Ramesh Babu ‘My Own Mazagon’?

Literacy for a human-centered recovery: Narrowing the digital divide

Azadi ka Amrit Mahotsav ■

India’s first Sports Arbitration Centre has been unveiled in which city? Ahmedabad



……….. has retained the tops Atal Rankings (ARIIA) 2021 for third straight year on January 1, 2022. IIT Madras





Nation First, Always First ■

Cricketer Suresh Raina has launched his autobiography on June 14, 2021 titled ‘Believe-What Life taught me’



Harry Potter author JK Rowling is all set to release on Oct. 12, 2021 a new children’s book titled The Christmas Pig

https://sscstudy.com/

14

CUET (UG) Section III : General Test

Practice Questions National Affairs

9. The government of India is committed

17. Which UT has become the first Union

1. The first-ever road project, made out

to become a net zero carbon emitter by which year?

of waste steel has come up in which city of India?

Territory in the country to have ‘District Good Governance Index’?

(a) 2040 (c) 2050

(a) Surat (c) Hyderabad

10. Name the scheme launched by the

(a) Chandigarh (b) Ladakh (c) Jammu and Kashmir (d) Puducherry

(b) Gurugram (d) Jamshedpur

(b) 2030 (d) 2070

Aayog’s Export Preparedness Index 2021?

Ministry of Social Justice and Empowerment for the welfare of the Transgender community and the beggars?

(a) Tamil Nadu (c) Gujarat

(a) SMILE (c) PROUD

2. Which state has topped the NITI (b) Maharashtra (d) Uttar Pradesh

(b) WINNER (d) VALUE

3. Which state’s handmade

11. Which Indian city has been declared

Narasingapettai Nagaswaram got geographical identification tag under the category of musical instruments of class 15?

as the most congested cities in the country as per the TomTom Traffic Index Ranking 2021?

(a) Kerala (c) Odisha

(b) Maharashtra (d) Tamil Nadu

4. Which Indian city has been ranked second-most noise polluted city globally, as per the UNEP ‘Annual Frontier Report, 2022’? (a) Mumbai (c) Kanpur

(b) Moradabad (d) Ghaziabad

5. Which state’s ‘Jingkieng Jri or Living Root Bridge’ has been included in UNESCO’s tentative list of World Heritage Sites? (a) Meghalaya (c) Tripura

(b) Mizoram (c) Assam

6. India has launched which operation to bring back its nationals from Ukraine? (a) Operation Yamuna (b) Operation Saraswati (c) Operation Ganga (d) Operation Bharat

7. Which of the follow ing organisations has launched the ‘Waste to Wealth Creation’ programme for women in the Sundarbans in West Bengal? (a) National Development Council (b) NITI Aayog (c) Small Industries Development Bank of India (SIDBI) (d) Central Vigilance Commission

8. Which state’s Governor has flagged off

(a) New Delhi (c) Mumbai

(b) Bengaluru (d) Chennai

12. Which of the follow ing has launched ‘Operation AAHT’ to curb human trafficking? (a) National Security Guard (b) Central Reserve Police Force (c) Railway Protection Force (d) Border Security Force

13. Which state government launched a

1. According to the Ministry of Defence, for the first time, an Indian Air Force (IAF) shown grand flypast by 75 aircraft or helicopters. 2. 1000 indigenously developed drones used for the ‘Beating the Retreat’ ceremony, along with projection mapping shown for the first time. 3. Beating Retreat ceremony held at Attari-Wagah border on 73rd Republic Day Which of the above is/are correct? (a) Only 1 (c) 2 and 3

(b) 1 and 2 (d) All of these

19. Kazhuveli Wetland has been declared as the 16th Bird Sanctuary of which of the follow ing states?

4-year mentorship programme ‘Project Arohan’?

(a) Tamil Nadu (c) Andhra Pradesh

(a) Tamil Nadu (c) Assam

20. Which state’s Chief Minister has

(b) Uttar Pradesh (d) Jharkhand

14. Consider the follow ing statements : 1. Asia’s biggest Bio-CNG GOBAR- Dhan plant has been inaugurated in Indore (Madhya Pradesh). 2. The plant is based on zero-landfill models. Of the above correct statement(s) is/are (a) Both 1 and 2 (c) Only 2

(b) Only 1 (d) None of these

15. Which State/UT has topped the India Press Freedom Report 2021 released by the Rights and Risks Analysis Group (RRAG)? (a) Uttar Pradesh (b) Tripura (c) Madhya Pradesh (d) Jammu & Kashmir

(b) Karnataka (d) Kerala

launched the ‘Milk Price Incentive Scheme’? (a) Nagaland (c) Uttarakhand

(b) Sikkim (d) West Bengal

21. Which State government has set up General Category Commission (Samanya Varg Aayog) for upper castes? (a) Andhra Pradesh (b) Uttar Pradesh (c) Arunachal Pradesh (d) Himachal Pradesh

22. First Drone Mela in the country has been organised at (a) Jabalpur (c) Gwalior

(b) Orchha (d) Bhopal

23. Which State’s Askot Wildlife

historic first-ever all-women officers offshore sailing expedition of Indian Army?

16. India’s first Graphene Innovation

(a) Karnataka (c) Odisha

(a) Karnataka (c) Tamil Nadu

(b) Telangana (d) Tamil Nadu

18. Consider the follow ing statements :

Centre w ill be established in which of the follow ing state? (b) Telangana (d) Kerala

https://sscstudy.com/

Sanctuary has been declared as the Askot Wildlife Sanctuary Eco Sensitive Zone (ESZ)? (a) Nagaland (b) Sikkim (c) Uttarakhand (d) West Bengal

https://sscstudy.com/

CURRENT AFFAIRS ONELINER

15

24. Which State government has

34. Consider the follow ing statements :

42. Rajiv Gandhi Kisan Nyay Yojana

launched ‘Khel Nursery Scheme 2022-23’ to promote sports?

1. India’s first ever emergency landing strip at National Highway for the Indian Air force Planes has been launched in Rajasthan. 2. With this inauguration, the NH-925 has become India’s first national highway to be used for emergency landing of IAF aircraft. Of the above correct statement(s) is/are

(RGKNY) is the initiative of which Indian State/UT?

(a) Haryana (c) Rajasthan

(b) Bihar (d) Assam

25. Which state has topped the IPF Smart Policing Index 2021 released by the Indian Police Foundation (IPF)? (a) Andhra Pradesh (b) Gujarat (c) Uttar Pradesh (d) Rajasthan

(a) Both 1 and 2 (c) Only 2

(b) Only 1 (d) None of these

26. Which state in India has approved

35. India’s largest open-air fernery has

the proposal to launch the first Cyber Tehsils?

been inaugurated at

(a) Gujarat (c) Madhya Pradesh

(b) Uttarakhand (d) Rajasthan

27. The Indian Railways is setting up the

(a) Ranikhet (c) Dehradun

(b) Darjeeling (d) Rishikesh

36. Match the follow ing.

tallest pier railway bridge of the world in the state of

a VAYO NAMAN Programme

1. Ministry of Commerce and Industry

(a) Uttarakhand (c) Gujarat

b Clean India Programme

2. Ministry of Science & Technology

c

3. Ministry of Youth Affairs and Sports

(b) Tamil Nadu (d) Manipur

28. Which Indian state/UT recently observed the first Aharbal festival to promote tourism?

peve CARE

d National Single Window System (NSWS)

4. Ministry of Social Justice and Empowerment

(a) Jammu & Kashmir (b) Assam (c) Kerala (d) Rajasthan

(a) a-3, b-4, c-2, d-1 (c) a-4, b-3, c-2, d-1

29. Which city’s Patalpani Railway

37. Which state has won the FSSAI’s

Station has been renamed after tribal Icon Tantya Bhil?

State Food Safety Index (SFSI) 2020-21 in large state category?

(a) Patna (c) Indore

(a) Tamil Nadu (c) Gujarat

(b) Jaipur (d) Delhi

(b) a-4, b-2, c-3, d-1 (d) a-4, b-3, c-1, d-2

(b) Madhya Pradesh (d) Maharashtra

30. Which city has been designated as

38. Which cartoon character has been

a part of UNESCO Creative Cities Network (UCCN) in November 2021 by UNESCO?

roped in as the official mascot for the Namami Gange Programme?

(a) Shimla (c) Srinagar

(b) Kargil (d) Prayagraj

31. India has been chosen as the Host Nation for the G20 Summit in which of the upcoming years? (a) 2023 (c) 2024

(b) 2022 (d) 2025

32. India’s tallest air purification tower

(a) Shakitmaan (b) Chacha Chaudhary (c) Chhota Bheem (d) Motu Patlu

39. What is the theme of the 7th edition of the Swachh Survekshan (SS), launched by the government for the year 2022? (a) People First (b) Citizen Participation (c) Waste to Wealth (d) Sustainable Living

40. Which state has received the GI tag for Judima, a home-made rice w ine?

has been installed in which State/UT of the country?

(a) Kerala (c) Nagaland

(a) Delhi (c) Assam

41. The Ministry of Information and

(b) Tamil Nadu (d) Chandigarh

(b) Assam (d) Odisha

overall category ranking of the NIRF India Rankings 2021?

Broadcasting has constituted a Committee to review the existing guidelines of Journalist Welfare Scheme. Who is the head of this committee?

(a) IIT Madras (b) IISc Bengaluru (c) AIIMS Delhi (d) IIT Delhi

(a) Sachidanand Murthy (b) Ravinder Kumar (c) Pankaj Salodia (d) Ashok Kumar Tandon

33. Which institution has topped the

https://sscstudy.com/

(a) Punjab (c) Chhattisgarh

(b) Kerala (d) Andhra Pradesh

43. The Mera Kaam Mera Maan (MKMM) scheme has been launched by which state to help in enhancing skills of unemployed youths? (a) Maharashtra (c) Punjab

(b) Madhya Pradesh (d) Rajasthan

44. PM Modi dedicated to the nation renovated complex of Jallianwala Bagh Smarak. The memorial is based in which city? (a) New Delhi (c) Kolkata

(b) Amritsar (d) Srinagar

45. The Rural Enterprises Acceleration Programme ‘Saath’ has been launched by which State/UT for Self Help Group (SHG) women? (a) Chandigarh (b) Rajasthan (c) Jammu & Kashmir (d) Uttar Pradesh

46. The government of India an July 29, 2021 decided to give 27% reservation to OBCs and 10% to EWSs for undergraduate and postgraduate programmes from the academic year 2021-22 in which field? (a) Defene sector (c) Economic sector

(b) Medial education (d) None of these

47. In which state did the Union Home Minister Amit Shah launh the ‘Harit Sohra Afforestation Campaign’ on July 25, 2021? (a) Nagaland (c) Tripura

(b) Mizoram (d) Meghalaya

48. The Ministry of Defence has unveiled a new system for automated sanction and disbursement of defence pension. What is the name given to that system? (a) MITRA (c) SPARSH

(b) ROSHINI (d) PRAYAS

49. Name the new Union Cabinet Minister of Health and Family Welfare to replace Dr. Harsh Vardhan in cabinet reshuffle. (a) Mahendra Nath Pandey (b) Mansukh Mandaviya (c) G. Kishan Reddy (d) Narayan Tatu Rane

50. Select the incorrect match w ith respect to the newly appointed Governor of the States (a) Himachal Pradesh – Rajendra Vishwanath Arlekar (b) Madhya Pradesh – Mangubhai Chhaganbhai Patel

https://sscstudy.com/

16

CUET (UG) Section III : General Test

(c) Mizoram – Hari Babu Kambhampati (d) Maharashtra – Thaawarchand Gehlot

51. Who has been named as the brand ambassador of Khadi Prakritk Paint, India’s first and only paint made from cow dung? (a) Nitin Gadkari (c) Smriti Irani

(b) Narendra Modi (d) Amit Shah

(a) 12 (c) 7

1. Prime Minister Narendra Modi has unveiled the mYoga mobile app. 2. mYoga app is available in English, Hindi and French. 3. mYoga app is preloaded with many yoga training videos and audio practice sessions. Code

edition of the Dandi March celebrations, which was led by Mahatma Gandhi? (a) 75

(a) Om Birla (c) Ravi Capoor

(a) 5 (c) 3

Chilahati-Haldibari rail link, which w ill connect India w ith which country? (a) Thailand (c) Nepal

an MoU on June 22, 2021 w ith the Government of Fiji for cooperation in the field of

(b) Madhya Pradesh (d) Kerala

(a) Defense sector (b) Agriculture and allied sectors (c) Sports sector (d) IT Sector

(a) The Shower (b) An Engineered Dream (c) Elephants Do Remember (d) Khisa

introduced ministry in the Union Cabinet of Government of India?

68. Name the Yoga dedicated mobile app launched by the Government of India in collaboration w ith WHO on the occasion of International Yoga Day on June 21, 2021

62. The world’s first Double Stack Long

(a) Ministry of Relations (b) Ministry of Services (c) Ministry of Synergy (d) Ministry of Cooperation

(a) s-Yoga (c) m-Yoga

Haul Container Train was flagged off by PM Modi recently. The train w ill run between which states?

Human Freedom Index 2020? (b) 107 (d) 111

(b) p-Yoga (d) i-Yoga

69. Which city has topped the Ease of Living Index 2020, for being the most liveable city of India?

(a) Mahatshtra and Bihar (b) Haryana & Rajathan (c) Jammu & Kashmir & Delhi (d) Tamil Nadu and Odisha

56. What is the rank of India in the

(b) Myanmar (d) Bangladesh

67. The Government of India has signed

non-feature film award at the 67th National Film Awards?

55. Which among these is the newly

(b) M. Venkaiah Naidu (d) Naresh Jain

66. PM Modi has inaugurated the

(b) 7 (d) 10

61. Which film has won the best

(b) Dehradun (d) Nagpur

(d) 100

launched by government to offer LED bulbs at ` 10 in rural areas. The scheme w ill cover how many states in the first phase?

(a) Haryana (c) Uttarakhand

Natural Gas (LNG) facility plant has been inaugurated in which city?

(c) 82

Television channels have been merged to form a single entity called Sansad Television. Who has been appointed as the CEO of this Sansad Television?

‘Launch Pad Scheme’ for above 18 years youths, coming out of child care institutions?

54. India’s first ever private Liquefied

(b) 91

65. The Rajya Sabha and Lok Sabha

(b) 9 (d) 5

60. Which state has launched the

(b) 1 and 3 (d) None of these

(b) Bangladesh (d) Sri Lanka

64. In 2021, India is observing which

59. The Gram Ujala Scheme has been

53. Choose the correct statements

(a) 124 (c) 92

(a) Nepal (c) Maldives

(b) SARTHAQ (d) UDDESHYA

between India and Bangladesh during the visit of PM Modi to the foreign nation to attend golden jubilee celebrations of Bangladesh’s independence in 1971?

(b) Cochin (d) Surat

(a) Pune (c) Ahmedabad

has recently inaugurated ‘Maitri Setu’ bridge, to strengthen connectivity w ith which country?

58. How many MoUs have been signed

India’s first movable freshwater tunnel aquarium ‘Aquatic Kingdom’ in which city?

(a) 1 and 2 (c) 1, 2 and 3

63. Prime Minister Shri Narendra Modi

released ‘Implementation Plan for School Education’ under the National Education Policy (NEP) 2020. What is the name given to this plan? (a) SMARAN (c) ARTH

52. The Indian Railways has set up

(a) Shimla (c) Bengaluru

57. Education Minister Ramesh Pokhriyal

(a) Hyderabad (c) New Delhi

(b) Bengaluru (d) Mumbai

ANSWERS 1. 11. 21. 31. 41. 51. 61.

(a) (c) (d) (a) (d) (a) (b)

2. 12. 22. 32. 42. 52. 62.

(c) (c) (c) (d) (c) (c) (b)

3. 13. 23. 33. 43. 53. 63.

(d) (c) (c) (a) (c) (c) (b)

4. 14. 24. 34. 44. 54. 64.

(b) (a) (a) (a) (b) (d) (b)

5. 15. 25. 35. 45. 55. 65.

(a) (d) (a) (a) (c) (d) (c)

6. 16. 26. 36. 46. 56. 66.

(c) (d) (c) (c) (b) (d) (d)

https://sscstudy.com/

7. 17. 27. 37. 47. 57. 67.

(c) (c) (d) (c) (d) (b) (b)

8. 18. 28. 38. 48. 58. 68.

(b) (d) (a) (b) (c) (d) (c)

9. 19. 29. 39. 49. 59. 69.

(d) (a) (c) (a) (b) (a) (b)

10. 20. 30. 40. 50. 60.

(a) (c) (c) (b) (d) (b)

https://sscstudy.com/

CURRENT AFFAIRS ONELINER

17

International Affairs 1. Which country hosted BIMSTEC

13. PM Modi hosted the first

22. Aziz Akhannouch has been appointed

Summit 2022?

India-Central Asia Summit virtually. How many countries are officially recognised as Central Asian region countries?

as the new Prime Minister of which country?

(a) Sri Lanka (c) Bangladesh

(b) India (d) Thailand

2. Robert Abela has been re-elected as the Prime Minister of which country? (a) Portugal (c) Cyprus

(b) Maldives (d) Malta

3. Which city has been adjudged as the world’s most polluted capital city in IQAir’s 2021 World Air Quality Report? (a) Dushanbe (c) Dhaka

(b) Muscat (d) New Delhi

4. India, along w ith which country,

(a) 5 (c) 16

(b) 9 (d) 24

14. Three countries have jointly topped the Corruption Perceptions Index (CPI) 2021 released by the Transparency International. Which of these country among them? (a) New Zealand, Finland and Denmark (b) Norway, New Zealand and Finland (c) France , Finland and Japan (d) Denmark , Israel and Norway

agreed on ‘Roadmap on Blue Economy and Ocean Governance’?

15. Which country will build world’s first

(a) The USA (c) France

(a) India (c) Singapore

(b) The UAE (d) Japan

5. France government has decided to w ithdraw its military from Mali after nine years, and shift its military base to which country? (a) Chad (c) Niger

(b) Senegal (d) Mauritania

6. Paul-Henri Sandaogo Damiba has been declared as President of (a) Chad (c) Mauritania

(b) Ecuador (d) Burkina Faso

7. Which country has topped the EIU’s 2021 Democracy Index? (a) Singapore (c) New Zealand

(b) Australia (d) Norway

floating city by 2025? (b) China (d) South Korea

16. Which country became the first country to transition to 4.5 day work week? (a) Qatar (b) Oman (c) Saudi Arabia (d) United Arab Emirates

17. Which country has joined the ISA as its 101st member? (a) Singapore (c) France

(b) Germany (d) The USA

18. Which country has planned to establish world’s first Bitcoin City?

(b) Algeria (d) Israel

23. Choose the correct statement regarding Fumio Kishid. 1. He has been appointed as the 100th Prime Minister of China. 2. Hereplaced outgoing party leader Prime Minister Yoshihide Suga. Codes (a) Both 1 and 2 (c) Only 2

(b) Only 1 (d) None of these

24. The World Expo 2020 has been organised in Dubai. What is the theme of the expo? (a) Culture and Education (b) Connecting Minds, Creating the Future (c) Live Green, Live Better (d) Building The World of Tomorrow

25. What was the theme of the 76th session of the United Nations General Assembly (UNGA) held on September 25, 2021? (a) The Future We Want, the UN We Need (b) Effective responses to global crises (c) Making the United Nations relevant to all people (d) Building resilience through hope

26. The first-ever, bronze statue of

8. Xiomara Castro sworn in as the first

(a) United States (c) Singapore

woman President of which of the follow ing country?

19. Which country has set the

Bitcoin’s anonymous creator Satoshi Nakamoto has been unveiled in which city?

Guinness Record for the world’s largest orchestra?

(a) San Salvador (c) Budapest

9. Which country has topped in the

(a) Venezuela (c) Chile

27. PM Modi virtually addressed the 6th

Corruption Perception Index (CPI) 2021 released by Transparency International?

20. Which country has topped the global

(a) Nicaragua (c) Mexico

(a) New Zealand (c) Finland

(b) Guatemala (d) Honduras

(b) Denmark (d) All of these

10. Nusantara has been selected as the new capital of which of the follow ing country? (a) South Korea (c) Vietnam

(b) Japan (d) Indonesia

11. What is the rank of India in the latest Henley Passport Index 2022? (a) 73

(b) 83

(c) 94

(d) 104

12. Which word has been chosen as the Oxford University Press Children’s Word of the Year 2021? (a) Anxiety (c) Sanitizer

(b) Coronavirus (d) Isolation

(b) Switzerland (d) El Salvador

(a) Turkey (c) Morocco

(b) Argentina (d) Ecuador

list of TRACE Bribery Risk Rankings 2021? (a) Iceland (b) Singapore (c) Finland (d) Denmark

21. What was the theme of the BRICS Summit 2021? (a) BRICS@15: Stronger Partnership for a Brighter Future (b) BRICS@15: Intra-BRICS Cooperation for Continuity, Consolidation and Consensus (c) BRICS@15: Collaboration for inclusive growth and shared prosperity in the 4th Industrial Revolution (d) BRICS@15: Economic Growth for an Innovative Future

https://sscstudy.com/

(b) Dubai (d) Madrid

Eastern Economic Forum (EEF) recently. Which country was the host of the EEF Summit 2021? (a) France (c) Germany

(b) Russia (d) Australia

28. The Children’s Climate Risk Index’ introduced by UNICEF on August, 2021. Accordingly, which of the follow ing statement is correct? 1. It ranks countries based on children’s exposure to climate and environmental shocks 2. Climate crisis puts Indian kids at ‘extremely high risk’ 3. The 33 extremely high risk countries emit only 9.38 % of global greenhouse gas emissions

https://sscstudy.com/

18

CUET (UG) Section III : General Test

Codes (a) Only 1 (c) Only 3

(a) Black list (c) Red list

(b) Only 2 (d) All of these

29. Name the President of Afghanistan who has stepped down after the control of the country was taken by the Taliban forces? (a) Ashraf Ghani (c) Rashid Khan

(b) Grey list (d) None of these

(a) China (b) US

Council Secretaries of Shanghai Cooperation Organisation (SCO) held?

47. The first BRICS Employment

38. Which country signed the framework

Global Manufacturing Risk Index?

agreement of the International Solar Alliance (ISA) and Instrument of Ratification w ith India in June, 2021?

(b) Australia (d) China

(a) Portugal (c) Italy

31. Which of the follow ing option is not true regarding UN resolution on vision?

Working Group (EWG) Meeting of 2021 was held recently. Which country is the chair of BRICS in 2021?

(a) It encourages countries to institute a whole of government approach to eye care. (b) It calls on international financial institutions and donors to provide targeted financing. (c) According to the resolution, at least 2 billion people are living with vision impairment. (d) None of the above

39. A 1098-carat diamond has been

32. Abiy Ahmed has won the second

(a) Ukraine (b) Qatar (c) Syria

Press Freedom Index 2021? (a) Norway (c) Switzerland

the latest and 23rd member of the Indian Ocean Rim Association (IORA)? (a) France (c) Australia

Romainville Island Solar Power Plant to which country? (a) Sri Lanka (c) Mauritius

presidential election 2021 of which country ?

an MoU on June 22,2021 w ith the Government of Fiji for cooperation in the field of

Act’ has been passed by which country for the welfare of Pakistani-diaspora female?

took place in a Hybrid format at Cornwall,

(a) Germany (c) United States

(a) France (c) United Kingdom

52. Which country has topped the World

(b) USA (d) Germany

(a) Germany (c) Finland

the new President of which country? (a) Israel (c) Turkey

(b) India (d) France

Happiness Report 2021?

42. Isaac Herzog, has been elected as

(a) Defense sector (b) Agriculture and allied sectors (c) Sports sector (d) IT Sector

(b) Indonesia (d) Seychelles

51. The ‘Malala Yousafzai Scholarship

(d) Iran

41. The 47th G7 Leaders’ Summit 2021

33. The government of India has signed

(b) Afghanistan (d) Malaysia

50. India has handed over 1 MW

(b) Zimbabwe (d) South Africa

40. Ebrahim Raisi has won the Iranian

(b) Ethiopia (d) Egypt

(b) Finland (d) Sweden

49. Which country has been inducted as

discovered by researchers, which is deemed to be the world’s third-largest diamond ever found. The diamond has been recovered in which country?

five-year term as the Prime Minister of which of the follow ing countries?

(b) Russia (d) Brazil

48. Which country has topped the World

(b) Sweden (d) Denmark

(a) Mauritius (c) Botswana

(c) Japan (d) India

(a) South Africa (c) India

30. Which country has topped the 2021

(a) Chile (c) Nigeria

Power Project with which nation?

37. Where was 16th meeting of Security (a) Dushanbe (Tajikistan) (b) Baku (Azerbaijan) (c) Ashgabat (Turmenistan) (d) Tashkant (Uzbekistan)

(b) Mohammed Omar (d) Abdullah Abdullah

(a) France (c) United States

46. Russia has started its biggest Nuclear

(b) Oman (d) Lebanon

(b) Singapore (d) Norway

53. Which country was the largest arms exporter in the world during 2016-2020 as per latest SIPRI report?

43. Which country has become the first in the world to grant legal tender status to bitcoin?

(a) United States (c) Russia

(a) Russia (b) Denmark (c) Italy (d) France

(a) Costa Rica (c) Ecuador

54. As per the 2020 IQAir World Air

35. What is the rank of India in terms of

44. Bashar al-Assad is the President of

recipient of FDI inflows in the year 2020, as per the World Investment Report 2021 by the UNCTAD?

which country, who has been re-appointed for fourth straight term recently?

34. The tallest sandcastle of the world has been inaugurated in which country?

(a) 3rd

(b) 5th

(c) 7th

(b) El Salvador (d) Colombia

(a) Syria (c) The Bahamas

(d) 6th

Quality Report, which city has been adjudged as the most polluted capital city in the world? (a) Beijing (c) Dhaka

(b) Mali (d) Oman

(b) Kathmandu (d) New Delhi

55. Which country was the host of the

36. Financial Action Task Force (FATF)

45. Isaac Herzog, has been elected as

released its updated list of countries in June, 2021, where Pakistan remained in the

the new President of which country? (a) Israel (c) Turkey

(b) China (d) Germany

‘One Planet Summit’ 2021, held in collaboration w ith UN and World Bank? (a) Australia (c) India

(b) Oman (d) Lebanon

(b) France (d) Germany

ANSWERS 1. 11. 21. 31. 41. 51.

(a) (b) (b) (d) (c) (c)

2. 12. 22. 32. 42. 52.

(d) (a) (c) (b) (a) (c)

3. 13. 23. 33. 43. 53.

(d) (a) (c) (b) (b) (a)

4. 14. 24. 34. 44. 54

(c) (a) (b) (b) (a) (d)

5. 15. 25. 35. 45. 55.

(c) (d) (d) (b) (a) (b)

6. 16. 26. 36. 46.

(d) (d) (c) (b) (a)

https://sscstudy.com/

7. 17. 27. 37. 47.

(d) (d) (b) (a) (c)

8. 18. 28. 38. 48.

(d) (d) (d) (d) (a)

9. 19. 29. 39. 49.

(d) (a) (a) (c) (a)

10. 20. 30. 40. 50.

(d) (d) (d) (d) (d)

https://sscstudy.com/

CURRENT AFFAIRS ONELINER

19

Economy & Banking 1. The Dearness Allowance for Central

11. Choose the correct statements

19. The RBI has granted small finance

government employees has been hiked by how much percent?

regarding Economic Survey 2021-22 1. The central theme of Economic Survey 2021-22 is the Agile Approach. 2. The Economic Survey has predicted 8 to 8.5% GDP growth in Financial Year 2022-23. 3. India is the 3rd largest start-up ecosystem in the world after US and China. Codes

bank licence to Unity Small Finance Bank. It is a consortium of which entities?

(a) 1% (c) 3%

(b) 2% (d) 4%

2. Reserve Bank Innovation Hub (RBIH) has been inaugurated in which city and what is the initial capital contribution for RBIH? (a) Mumbai; ` 200 crore (b) Bengaluru; ` 100 crore (c) New Delhi; ` 200 crore (d) Chennai; ` 100 crore

(a) 1 and 2 (c) 1, 2 and 3

3. The ‘AutoFirst’ application has been launched by which bank to offer fully automated auto loans? (a) HDFC Bank (b) ICICI Bank (c) Kotak Mahindra Bank (d) IndusInd Bank

4. Which country will become the first country to adopt India’s UPI platform? (a) Nepal (c) Bangladesh

(b) Myanmar (d) Sri Lanka

5. Which bank has become the first to enter in the Metaverse? (a) Barclays (c) Citibank

(b) JPMorgan (d) UBS

6. Which city in India has highest number of dollar-millionaire households according to the Hurun India Wealth Report 2021? (a) Bengaluru (c) Delhi

(b) Kolkata (d) Mumbai

7. In accordance w ith the Bloomberg

(b) 2 and 3 (d) None of thsee

(b) March 2024 (d) September 2025

9. How many digital banking systems w ill be set-up in 75 districts by Scheduled Commercial Banks? (a) 25

(b) 50

(c) 75

(d) 100

10. Nirmala Sitharaman announced a new tax rule for taxpayers where a taxpayer can file an updated return on payment of taxes w ithin? (a) 1 year (b) 2 years (c) 3 years (d) 4 years

(b) BharatPe (d) PhonePe

21. Bank of Baroda has launched its

(a) bob Ahead (c) bob Unite

(b) Italy (d) Egypt

(b) bob Now (d) bob World

13. Which bank has launched

22. Match the follow ing.

feature-rich savings bank product for women named “Mahila Mitra Plus”?

a. Public Provident Fund

1. 6.9%

(a) RBL Bank (b) Karur Vysya Bank (c) Federal Bank (d) DCB Bank

b. National Savings Certificate

2. 7.6%

c. Sukanya Samriddhi Yojana

3. 6.8%

14. Which payments bank has recently

d. Kisan Vikas Patra

4. 7.1%

gat the RBI’s approval to function as a scheduled payments bank?

(a) a-3, b-4, c-2, d-1 (c) a-4, b-3, c-2, d-1

(a) Paytm Payments Bank (b) Airtel Payments Bank (c) NSDL Payments Bank (d) Jio Payments Bank

15. Which company has partnered w ith Mastercard on Tokenisation for Card-Based Payments?

16. Which organisation has recently

(a) March 2025 (c) December 2024

(a) Paytm (c) Whatsapp

(a) Nepal (c) Iran

(a) Aziz Premji (c) Ratan Tata

Non-Banking Financial Companies (NBFCs) to mandatorily implement ‘Core Financial Services Solution (CFSS)’ by

‘postpe’ solution based on ‘Buy Now Pay Later (BNPL)’ platform?

digital banking platform under which name to bring all banking services under one roof?

(a) PhonePe (b) Paytm (c) Google Pay (d) Amazon Pay

8. The RBI has directed certain

20. Which company has launched the

added as a new member of the BRICS New Development Bank in December 2021?

12. Which of these countries has been

Billionaires Index, who among the follow ing has become Asia’s richest person as on February 8, 2022? (b) Gautam Adani (d) Mukesh Ambani

(a) Centrum and BharatPe (b) PFC Ltd. and Aditya Birla (c) Tata Capital and Bandhan (d) LIC and L&T Finance

launched the HARBINGER 2021 – Innovation for Transformation hackathon? (a) SEBI (c) SBI

(b) SIDBI (d) RBI

17. Under RBI’s new directives, what is the minimum initial capital requirement to set-up a new universal bank? (a) ` 1000 crore (c) ` 700 crore

(b) ` 800 crore (d) ` 500 crore

18. CBI has recently registered the biggest ever bank fraud case by which company? (a) Berger Paints (b) ABG Shipyard (c) Mercator Limited (d) Shalimar Works

https://sscstudy.com/

(b) a-4, b-2, c-3, d-1 (d) a-4, b-3, c-1, d-2

23. Who has topped the IIFL Wealth Hurun India Rich List 2021? (a) Shiv Nadar (c) Mukesh Ambani

(b) Gautam Adani (d) SP Hinduja

24. Which among the follow ing banks in the only one left under the RBI’s PCA regime? (a) Central Bank of India (b) UCO Bank (c) Bank of Maharashtra (d) Canara Bank

25. The government has recently enhanced the income limit of disabled dependents to be eligible for family pension for life from Ministry of Defence. What is the new limit? (a) less than 20% of the last pay drawn (b) less than 10% of the last pay drawn (c) less than 30% of the last pay drawn (d) less than 40% of the last pay drawn

26. PM Modi inaugurated a special commemorative coin of what value to mark the birth anniversary of ISKCON founder Srila Bhaktivedanta Swami Prabhupada? (a) ` 100 (c) ` 200

(b) ` 125 (d) ` 250

https://sscstudy.com/

20

CUET (UG) Section III : General Test

27. Centre has launched Pradhan Mantri

33. Assertion (A) India was the fifth

Gatishakti scheme for holistic development of infrastructure sector and boosting employment. What is the approx amount that will be invested in the scheme?

largest recipient of FDI inflows in the world in 2021. Reason (R) Received $ 64 billion FDI in 2020, which is an increase of 50%.

(a) ` 50 trillion (c) ` 150 trillion

(b) ` 100 trillion (d) ` 200 trillion

in the incentives given to commercial banks for distribution of coins per bag from September 2021. What w ill be the new incentive per bag? (b) ` 65

(c) ` 75

names of which bank for privatisation in 2021-22? (a) Indian Overseas Bank and Punjab National Bank (b) Bank of India and Canara Bank (c) Central Bank of India and Indian Overseas Bank (d) Canara Bank and Bank of Maharashtra

34. Which Bank has launched an

(d) ` 135

41. LIC Cards Services (LIC CSL) has

initiative called ‘Salaam Dil Sey’ to pay tribute and show gratitude to doctors across the country for their tireless service during the pandemic ?

29. Which of these countries have been inducted as the new member country of the New Development Bank (NDB) set-up by BRICS nation? 1. Bangladesh 2. UAE 3. Uruguay 4. Syria 5. Zimbabwe Codes (a) 1 , 2 and 4 (c) 1, 2 and 3

40. NITI Aayog has recommended the

Code (a) A is true but R is false. (b) Both A and R are true and R is the correct explanation of A. (c) Both A and R are true but R is not the correct explanation of A. (d) Both A and R are false.

28. The RBI has announced an increase

(a) ` 45

(a) World Bank (b) Asian Development Bank (c) International Monetary Fund (d) World Economic Forum

(a) SBI (c) ICICI Bank

launched a contactless prepaid gift card, ‘Shagun’, in collaboration w ith (a) Bank of India (b) ICICI Bank (c) SBI (d) IDBI Bank

(b) HDFC Bank (d) PNB

35. The government has recently

(b) 2, 3 and 4 (d) All of these

30. The SAMRIDH programme has recently been launched by which ministry to boost start-up ecosystem in India?

extended the superannuation age of LIC. Chairman.What is the revised age for the same?

42. Which Indian bank has joined hands

(a) 65 years (c) 62 years

(a) Canara Bank (c) HDFC Bank

w ith NPCI to link its digital wallet Pockets w ith UPI ID?

(b) 70 years (d) 60 years

(b) SBI (d) ICICI Bank

36. In June 2021, Union Minister Nitin

43. TRIFED has collaborated with NITI

(a) Ministry of Electronics & Information Technology (b) Ministry of Statistics and Programme Implementation (c) Ministry of Micro, Small and Medium Enterprises (d) Ministry of Health & Family Welfare

Gadkari launched ‘MSME Prerna’, a first of its kind programme by

Aayog to implement Van Dhan Yojana scheme in the Tribal Aspirational Districts identified by NITI Aayog. How many Tribal Aspirational Districts are there presently?

31. Recently Government of India has

(a) Credila Financial Services Private Limited (b) Satin Creditcare Network Limited (c) HDB Financial Services Limited (d) Centrum Financial Services Limited

(a) Canara Bank (c) Indian Bank

amended Insolvency and Bankruptcy Board of India (Insolvency Resolution Process for Corporate Persons) Regulations, 2016.It w ill be effective from which date ? (a) July 14, 2021 (c) July 22, 2021

(b) SBI (d) PNB

37. Which NBFC has received approval

(a) 29 (c) 39

from RBI to take over PMC bank and re-launch it as a Small Finance Bank?

44. The National Digital Financial Infrastructure (NADI) is a next-generation Digital Financial Infrastructure, being build by body? (a) National Payments Corporation of India (NPCI) (b) State Bank of India (SBI) (c) NITI Aayog (d) Institute for Development and Research in Banking Technology (IDRBT)

38. What is the rank of India in terms of

(b) July 19, 2021 (d) July 24, 2021

recipient of Foreign Direct Investment (FDI) inflows in the year 2020, as per the World Investment Report 2021 by the UNCTAD?

32. Which among these sectors have been included as MSMEs sector by government for purpose of Priority Sector Lending?

(a) 3rd (c) 7th

(a) Retail and Wholesale trade (b) Forestry and logging (c) Fishing and aquaculture (d) Bee-keeping and production of honey and beeswax

(b) 41 (d) 18

45. RBI has excluded Lakshmi Vilas Bank from the Second Schedule of the Reserve Bank of India Act, 1934, follow ing the merger of the Lakshmi Vilas Bank w ith

(b) 5th (d) 6th

39. Which financial body has signed

(a) DBS Bank India Ltd (b) HSBC Bank (c) United Overseas Bank (d) Citibank

$484 million loan w ith Indian government to improve transport connectivity in the Chennai–Kanyakumari Industrial Corridor (CKIC)?

ANSWERS 1. 11. 21. 31. 41.

(c) (c) (d) (a) (d)

2. 12. 22. 32. 42.

(b) (d) (c) (a) (d)

3. 13. 23. 33. 43.

(a) (c) (c) (d) (c)

4. 14. 24. 34. 44.

(a) (a) (a) (b) (d)

5. 15. 25. 35. 45.

(b) (c) (c) (c) (a)

6. 16. 26. 36.

(d) (d) (b) (c)

https://sscstudy.com/

7. 17. 27. 37.

(b) (a) (b) (d)

8. 18. 28. 38.

(d) (b) (b) (b)

9. 19. 29. 39.

(c) (a) (c) (b)

10. 20. 30. 40.

(b) (b) (a) (c)

https://sscstudy.com/

CURRENT AFFAIRS ONELINER

21

Science & Technology 1. The first edition of Indian Ocean Naval

11. India’s largest Electric Vehicle (EV)

Symposium (IONS) Maritime Exercise 2022 (IMEX-22) was held in which state?

charging station inaugurated at

(a) Gujarat (c) Kerala

(b) Goa (d) Maharashtra

2. The Indian Army version of Medium Range Surface to Air Missile (MRSAM) has been developed by DRDO in partnership w ith which country? (a) Israel (c) France

(b) Russia (d) United States

3. Name the joint military training exercise between the Indian army and Uzbekistan which kicked off in March 2022 at Yangiarik. (a) Ex-Shared Destiny (b) Ex-Dustlik (c) Ex-Khanjar (d) Ex-Sea Breeze

4. Which Naval Ship was recently presented the prestigious President’s Colour? (a) INS Valsura (b) INS Gomati (c) INS Brahmaputra (d) INS Betwa

5. Which Naval Command of Indian Navy has conducted offshore security exercise ‘Prasthan’ off the Mumbai coast? (a) Western Naval Command (b) Southern Naval Command (c) Northern Naval Command (d) Eastern Naval Command

6. Which country hosted multilateral exercise MILAN 2022? (a) India (c) The UK

(b) France (d) Malaysia

7. Which nation has successfully tested its new naval air defence system called ‘C-Dome’? (a) Israel (c) Turkey

(b) Iran (d) North Korea

8. The ISRO successfully launched which earth observation satellite, in its first launch of the year 2022? (a) EOS-2 (c) EOS-3

(b) EOS-5 (d) EOS-4

(a) Jaipur (Rajasthan ) (b) Mumbai (Maharashtra) (c) Chennai (Tamil Nadu) (d) Gurugram (Haryana)

21. What is the rank of India in the

(a) IBM (c) Tesla

(b) Alphabet Inc. (d) Meta Inc.

13. Choose the correct statements regarding MPATGM : 1. DRDO successfully flight tested the final deliverable configuration of Man Portable Anti-Tank Guided Missile (MPATGM). 2. The indigenously developed anti-tank missile is a low weight, fire & forget missile and is launched from a man portable launcher, integrated with thermal sight. Codes (a) Only 1 (c) Both 1 and 2

(b) Only 2 (d) Neither 1 nor 2

14. What type of orbit placement has been planned by the NASA for recently launched James Webb Telescope? (a) LEO (c) Geo-synchronous

(b) L2 (d) None of these

15. Which country has topped the “World Talent Ranking Report 2021” published by IMD World Competitive Centre? (a) Switzerland (c) Sweden

(b) Norway (d) Finland

16. Which country has topped the Global Health Security (GHS) Index 2021? (a) Thailand (c) Australia

(b) Finland (d) USA

17. Which organisation has successfully launched a long-range Supersonic Missile Assisted Release of Torpedo (SMART) System from Abdul Kalam Island, off the Odisha coast?

18. Which institution has topped the Atal

(b) DRDL (d) HAL

Ranking of Institutions on Innovation Achievements (ARIIA) 2021?

10. Which country has delivered all the

(a) IIT Bombay (c) IIT Kanpur

contracted 70000 AK-203 assault rifles to India?

19. Isomorphic Labs is a newly launched

(b) Afghanistan (d) Russia

in the world to approve a pill for treating cases of symptomatic COVID-19?

unveiled an artificial intelligence supercomputer ‘AI Research Super Cluster (RSC)’?

intermediate-range ballistic missile Hwasong-12?

(a) Israel (c) Germany

20. Which country has become the first (a) Britain (c) The USA

9. Which country test fired its (b) Japan (d) South Korea

(b) Meta (d) Samsung

12. Which of the follow ing company has

(a) IAF (c) DRDO

(a) China (c) North Korea

(a) Apple (c) Alphabet

(b) IIT Madras (d) IIT Delhi

wholly owned subsidiary of which company?

https://sscstudy.com/

(b) Singapore (d) Australia

Climate Change Performance Index (CCPI) 2022 by Germanwatch? (a) 10th (c) 7th

(b) 12th (d) 15th

22. Name the fourth Scorpene class submarine received by Indian Navy under the Project 75. (a) INS-Vela (c) INS-Vayu

(b) INS-Vagir (d) INS-Veer

23. The Zircon hypersonic cruise missile was successfully test-fired by which country? (a) Egypt (c) Russia

(b) Israel (d) United Arab Emirates

24. The World Health Organisation has designated the coronavirus variant, B.1.1.529 named ………… as ‘Variant of Concern (VOC)’. (a) Alpha (c) Delta

(b) Beta (d) Omicron

25. Consider the follow ing statements : 1. Norway topped the Global Drug Index 2021. 2. India ranked at 18 out of 30 countries with an overall score of 46/100. Of the above statement(s), the correct is/are (a) Both 1 and 2 (c) Only 2

(b) Only 1 (d) None of these

26. The multinational ‘Blue Flag-2021’ air combat exercise was hosted by which country? (a) Turkey (c) Israel

(b) Russia (d) Singapore

27. Which country has recently unveiled the world’s largest hydrogen fuel cell power plant? (a) South Korea (c) Germany

(b) Israel (d) Singapore

28. i-Drone, a drone-based vaccine delivery model for Northeast states, has been developed by which organisation? (a) ISRO (b) Indian Council of Medical Research (c) DRDO (d) Indian Institute of Technology, Madras

https://sscstudy.com/

22

CUET (UG) Section III : General Test

29. The ISRO has decommissioned which

37. How many Indian beaches have been

46. On July 29, 2021 Russia’s module

communications satellite through 11 Re-orbiting manoeuvres after 14-year service?

awarded w ith the Blue Flag Certification by the Foundation for Environment Education in Denmark?

docked w ith international space station. What is the name of this module?

(a) INSAT-2E (b) KALPANA-1 (c) GSAT-29 (d) INSAT-4B

(a) 10

30. Which organisation for the first time successfully demonstrated India’s indigenous technology, Quantum Key Distribution (QKD) link between Prayagraj and Vindhyachal in Uttar Pradesh? (a) DRDL (c) ISRO

(b) DRDO (d) HAL

31. Match the follow ing. (a) Gaofen-5 02

1. North Korea

(b) Tsirkon (Zircon) hypersonic cruise missile

2. US

(c) Hypersonic Air-breathing Weapon Concept (HAWC)

3. Russia

(d) Anti-aircraft missile

4. China

(a) a-3, b-4, c-2, d-1 (c) a-4, b-3, c-2, d-1

(b) a-4, b-2, c-3, d-1 (d) a-4, b-3, c-1, d-2

32. Which among these space-tech startups has become the first private company to formally sign agreement w ith ISRO for using its expertise and facilities? (a) Dhruva Space (b) Agnikul Cosmos (c) Bellatrix Aerospace (d) Skyroot Aerospace

33. i-Drone, a drone-based vaccine delivery model for Northeast states, has been developed by which organisation? (a) ISRO (b) Indian Council of Medical Research (ICMR) (c) DRDO (d) Indian Institute of Technology, Madras

34. Hwasong-8 is a new hypersonic missile, which was tested successfully by which country recently? (a) North Korea (c) Japan

(b) South Korea (d) Taiwan

35. The maiden successful test fight of which new advanced version of the Akash Missile was carried out by DRDO? (a) Akash Prime (c) Akash Fine

(b) Akash Delux (d) Akash A1

36. Name the system launched by Supreme Court for transferring e-authenticated copies from courts to prisons? (a) SAFER (c) WINNER

(b) FASTER (d) GIFT

(b) 8

(c) 12

(d) 9

(a) Chaka (c) Luna

(b) Nauka (d) None of these

38. JIMEX 2021 is the 5th edition of the

47. The Indian Army has named a firing

annual bilateral maritime exercise of India w ith which country?

range at Gulmarg in Jammu & Kashmir after which of these Bollywood actress?

(a) Germany (c) Italy

(a) Priyanka Chopra (b) Vidya Balan (c) Diya Mirza (d) Anushka Sharma

(b) Thailand (d) Japan

39. The digital platform ‘e-Source,’ is being developed by which institution to tackle the problem of e-waste? (a) IIT Madras (c) IIT Kanpur

(b) IIT Delhi (d) IIT Hyderabad

40. Consider the follow ing statements : 1. Rajnath Singh dedicated indigenously built Coast Guard Ship Vigraha to the nation in Chennai. 2. ICGS Vigraha is seventh in the series of Offshore Patrol Vessels. Of the above correct statement(s) is/are (a) Only 1 (c) Both 1 and 2

(b) Only 2 (d) None of these

41. The annual military exercise KAZIND-21 is being conducted between the Indian Army and the Army of (a) Kyrgyzstan (c) Kenya

(b) Kazakhstan (d) Kuwait

42. India’s first indigenously developed Quantum Computer Simulator (QSim) Toolkit has been launched. The toolkit has been developed by which institution? 1. Centre for Development of Advanced Computing (C-DAC) 2. Indian Institute of Technology Roorkee 3. Indian Institute of Science, Bengaluru Codes (a) 1 and 2 (c) 1 and 3

(b) 2 and 3 (d) All of these

43. India’s first green hydrogen electrolyzer manufacturing unit has been unveiled in which city? (a) Bengaluru (c) Chennai

(b) Visakhapatnam (d) Hyderabad

44. What is the name of India’s first Cattle Genomic Chip for the conservation of pure varieties of indigenous cattle breeds? (a) BharGau (c) IndiGau

(b) GauDham (d) GauLok

45. Which Indian ship participated in the Maiden Maritime Partnership Exercise 2021 conducted by the Indian Navy w ith Algerian Navy? (a) INS Talwar (c) INS Teg

(b) INS Tabar (d) INS Trikand

https://sscstudy.com/

48. Choose the correct statements : 1. The Union Defence Minister Rajnath Singh has informed that India’s first Indigenous Aircraft Carrier (IAC-I), is planned to be commissioned by 2022. 2. Once commissioned, the carrier will be rechristened as INS Vikrant, in memory of India’s first aircraft carrier. 3. The IAC-1 caarier is being built at the Cochin Shipyard Limited , Kochi, Kerala, under public-private partnership. Code (a) 1 and 2 (c) 1, 2 and 3

(b) 2 and 3 (d) None of these

49. The Ministry of Defence has inked ` 499 crore contract w ith which entity for

the manufacture and supply of Akash missiles to the Indian Air Force (IAF)? (a) Hindustan Aeronautics Limited (b) Bharat Heavy Electricals Limited (c) Bharat Dynamics Limited (d) Ordnance Factory Board

50. Which space agency has planned to launch the world’s first wooden satellite, WISA Woodsat, into orbit by the end of the year 2021? (a) NASA (c) ISRO

(b) ESA (d) Roscosmos

51. The Ministry of Defence has recently inked contract w ith which company for construction of two Pollution Control Vessels (PCVs) for Indian Coast Guard (ICG)? (a) Garden Reach Shipbuilders & Engineers Ltd. (b) Mazagon Dock Shipbuilders Ltd. (c) Goa Shipyard Ltd. (d) Hindustan Shipyard Ltd.

52. With which of the follow ing naval force, India has conducted a joint exercise in Gulf of Aden? (a) SAARC (c) NATO (d) European Union

(b) SCO

https://sscstudy.com/

23

CURRENT AFFAIRS ONELINER

53. Which space agency has planned to

62. The supercomputer named ‘Simorgh’

69. Russia launched 38 foreign satellites

launch the world’s first wooden satellite, WISA Woodsat, into orbit by the end of the year 2021?

has been developed by which country recently?

from 18 countries into orbit in March 2021. Which rocket system was used for this launch?

(a) France (c) South Korea

(a) NASA (b) European Space Agency (c) ISRO (d) Roscosmos

(a) Soyuz-2.1a (c) Soyuz-2.1v

63. Union cabinet has approved the

virtually addressed the VivaTech 2021. The event has been organised in which city?

decommissioned M-46 Catapult Guns and Tampella Mortars, two of the longest serving artillery gun systems from service. The caliber of M-46 Catapult Guns and Tampella Mortars were ............ and ........... respectively.

(a) Surface to Surface Missile (b) Air to Surface Missile (c) Surface to Air Missile (d) Air to Air Missile

(d) Dubai

(b) Soyuz-2.1b (d) Soyuz-2.1s

70. The Indian Army has

proposal of exporting the indigenouslydeveloped Akash Missile System. The missile comes under which category?

54. Prime Minister Narendra Modi

(a) London (b) Rome (c) Paris

(b) Iran (d) Finland

55. The Conference of BRICS Network

64. The Military Junta of which nation has

Universities 2021, has been organised at which Indian institute?

arrested the President, Prime Minister and Defence Minister?

(a) 190 mm and 190 mm (b) 120 mm and 150 mm (c) 100 mm and 110 mm (d) 130 mm and 160 mm

(a) IIT Kanpur (c) IIT Delhi

(a) Zambia (c) Ethiopia

71. The web portal and mobile app,

(b) IIT Bombay (d) IIT Guwahati

56. The mobile app ‘Namaste Yoga’,

(b) Mali (d) Eritrea

devoted to Yoga, has been launched by which Ministry?

test firing of the 5th generation Python-5 missile in April,2021. The state of the art missile is which type of missile?

(a) Ministry of AYUSH (b) Ministry of Youth Affairs and Sports (c) Ministry of Health and Family Welfare (d) Ministry of Information & Broadcasting

72. The first-ever ‘Hot Air Balloon Wildlife Safari’ of India in a tiger reserve has been launched in which tiger reserve?

news recently, is related to

66. The Nehru Zoological Park is the

(a) Satellite launch (c) Cyclone

first zoo in India to get the ISO 9001:2015 Quality Management Standards Certificate from United Kingdom. The zoo is situated in which city?

(b) Covid variant (d) Food Production

58. CBSE has collaborated w ith which tech giant to introduce Coding and Data Science as new subjects in 2021-22 academic sessions?

59. The sustainable electric

73. The Union cabinet has recently approved new guidelines for DTH broadcasting services, as per which, the DTH services w ill now be issue licences for a period of .......... years.

67. India has signed an agreement

two-wheeler,named ‘PiMo’ has been developed by which institution?

(a) 20 (c) 15

w ith which country for cooperation in India’s first human space mission, Gaganyaan?

(b) IIT Delhi (d) IIT Kharagpur

60. The multilateral military exercises,

(a) Russia (c) United States

‘Steadfast Defender 2021’, has been organised by which group of inter-governmental organisation?

68. NITI Aayog has launched a national

61. Kappa and Delta, which has been in

Departmental Vigilance Officers (DVO) Portal has been launched by which state for easy grievance redressal?

(b) Israel (d) France

(a) Delhi (b) Madhya Pradesh (c) Jammu & Kashmir (d) Uttar Pradesh

75. DRDO has partnered w ith which country for the development of MRSAM missile?

(a) Aushadhi Gyan (b) Poshan Gyan (c) Fitness Gyan (d) Aarogya Gyan

news recently, is related to (b) Covid variant (d) Food production

(b) 10 (d) 30

74. The mobile app ‘Satark Nagrik’ and

digital repository on health and nutrition. What is the name given to this repository?

(b) NATO (c) OPEC (d) BRICS

(a) Satellite launch (c) Cyclone

(a) Sariska Tiger Reserve (b) Satpura Tiger Reserve (c) Bandhavgarh Tiger Reserve (d) Kanha Tiger Reserve

(a) Bengaluru (b) Nagpur (c) Jaipur (d) Hyderabad

(a) Google (b) Microsoft (c) IBM (d) TCS

(a) G-20

(a) Central Public Works Department (b) Directorate of Printing (c) Land & Development Office (d) Directorate of Estates

(a) Air-to-Air Missile (b) Surface-to-Air Missile (c) Air-to-Surface Missile (d) Surface-to-Surface Missile

57. Kappa and Delta, which has been in

(a) IIT Madras (c) IIT Kanpur

‘e-Sampada’ has been launched by which sector?

65. DRDO conducted successful maiden

(a) France (c) Israel

(b) Russia (d) United States

ANSWERS 1. 11. 21. 31. 41. 51. 61. 71.

(b) (d) (a) (c) (b) (c) (b) (d)

2. 12. 22. 32. 42. 52. 62. 72.

(a) (d) (a) (d) (d) (d) (b) (c)

3. 13. 23. 33. 43. 53. 63. 73

(b) (c) (c) (b) (a) (b) (c) (a)

4. 14. 24. 34. 44. 54. 64. 74.

(a) (b) (d) (a) (c) (c) (b) (c)

5. 15. 25. 35. 45. 55. 65. 75.

(a) (a) (a) (a) (b) (b) (a) (c)

6. 16. 26. 36. 46. 56. 66.

(a) (d) (c) (b) (b) (a) (d)

https://sscstudy.com/

7. 17. 27. 37. 47. 57. 67.

(a) (c) (a) (a) (b) (b) (d)

8. 18. 28. 38. 48. 58. 68.

(d) (b) (b) (d) (c) (b) (b)

9. 19. 29. 39. 49. 59. 69.

(c) (c) (d) (a) (c) (a) (a)

10. 20. 30. 40. 50. 60. 70.

(d) (a) (b) (c) (c) (b) (d)

https://sscstudy.com/

24

CUET (UG) Section III : General Test

Sports 1. Who has been roped in as the new President of Badminton Association of India (BAI) for four years from 2022-2026? (a) Jyotiraditya Scindia (b) Kiren Rijiju (c) Himanta Biswa (d) Tarun Gogoi

2. Which player has won the Formula One 2022 Saudi Arabian Grand Prix? (a) Max Verstappen (c) Charles Leclerc

(b) Lewis Hamilton (d) Carlos Sainz Jr.

11. Match the follow ing columns A. Rio Open 2022

1. Dabang Delhi

B. Mexican Open 2022

2. Andrey Rublev

Championship in 2022 held in Himachal Pradesh?

C. Dubai Tennis 3. Rafael Nadal Championships 2022

(a) Chandigarh (b) Ladakh (c) Delhi (d) Indo-Tibetan Border Police

D. PKL 8

19. Name the Indian shuttler who has

(a) a-3, b-4, c-2, d-1 (c) a-4, b-3, c-2, d-1

4. Carlos Alcaraz

(b) a-4, b-2, c-3, d-1 (d) a-4, b-3, c-1, d-2

12. Which cricket team has won the under-19 Asia Cricket Cup 2021 in Dubai?

(a) Saina Nehwal (c) Sania Mirza

(b) PV Sindhu (d) N. Sikki Reddy

20. Who among the follow ing has been

3. PV Sindhu beat which player to lift

(a) Nepal (c) India

women’s singles title at the 2022 Sw iss Open Badminton Tournament?

13. Who among the follow ing has won

(a) Pornpawee Chochuwong (b) Busanan Ongbamrungphan (c) Yeo Jia Min (d) Ratchanok Intanon

the Men's Singles title of Australian Open 2022? (a) Daniil Medvedev (b) Novak Djokovic (c) Alexander Zverev (d) Rafael Nadal

4. Wrist Assured is the autobiography of

14. Name the Indian cricketer who has

(a) Victoria Azarenka (b) Jen Brady (c) Naomi Osaka (d) Simone Biles

which former cricket player?

been named in 2021 ICC Women’s T20I team of the year.

22. Who has become the first Indian man

(a) EAS Prasanna (c) GR Viswanath

(b) Dilip Vengsarkar (d) Sunil Gavaskar

5. Name the Indian cricket player who

(b) Pakistan (d) Sri Lanka

won silver at BWF World Tour Finals 2021.

(a) Mithali Raj (c) Jhulan Goswami

(b) Smriti Mandhana (d) Shafali Verma

has been felicitated w ith the ‘Sports Icon’ award at the Maldives Sports Awards 2022?

15. Which player has won the Tata Steel

(a) Rohit Sharma (b) Virender Sehwag (c) S. Sreesanth (d) Suresh Raina

(a) Magnus Carlsen (b) Hikaru Nakamura (c) Garry Kasparov (d) Fabiano Caruana

6. Which country has topped the medal

16. Consider the follow ing statements

table of Winter Olympics Games 2022 held in Beijing?

regarding Syed Modi International Title 2022 : 1. PV Sindhu clinched the Syed Modi International Title 2022 after beating Malvika Bansod in the women’s singles final at the Babu Banarasi Das Indoor Stadium. 2. It was also Sindhu’s first BWF title since winning the BWF World Championships in 2019, after losing twice in the finals last year—at the Swiss Open and the BWF World Tour Finals.

(a) India (c) Germany

(b) Norway (d) China

7. Who won Mexican Open 2022 men’s singles title? (a) Rafael Nadal (b) Cameron Norrie (c) Daniil Medvedev (d) Stefanos Tsitsipas

8. Sadia Tariq won a gold medal for India in which event? (a) Wushu (c) Boxing

(b) Fencing (d) Weightlifting

9. Which team has won the FIFA Club

Chess Tournament 2022 in Masters section?

Choose the correct statements (a) Only 1 (b) Only 2 (c) Both 1 and 2 (d) None of these

World Cup Championship 2021?

17. Which player has won the women’s

(a) Chelsea (c) Liverpool

single Indian Open (Badminton) 2022?

(b) Manchester City (d) Palmeiras

10. Which country has become the world's top ranked T20 team in the latest ICC T20 Team Rankings? (a) South Africa (c) Australia

(b) India (d) New Zealand

(a) Busanan Ongbamrungphan (b) Ratchanok Intanon (c) Akane Yamaguchi (d) Nozomi Okuhara

18. Which team has won the 9th National Women’s Ice Hockey

https://sscstudy.com/

honoured w ith Sports Journalists’ Federation of India (SJFI) Medal 2021? (a) Ravi Shastri (c) Sunil Gavaskar

(b) Rahul Dravid (d) Anil Kumble

21. Who has been named Time magazine’s 2021 Athlete of the Year?

to w in a silver medal at BWF World Championships? (a) Srikanth Kidambi (b) Sandeep Gupta (c) Varun Kapur (d) B. Sai Praneeth

23. Name the w inner of the Mexico City Grand Prix 2021. (a) Charles Leclerc (c) Sergio Perez

(b) Max Verstappen (d) Lewis Hamilton

24. Which player has won the WTA Finals 2021 tennis title? (a) Garbine Muguruza (b) Simona Halep (c) Victoria Azarenka (d) Karolína Pliskova

25. Who has been appointed as the new permanent CEO of the International Cricket Council (ICC)? (a) Mark Boucher (c) Jay Shah

(b) Dave Richardson (d) Geoff Allardice

26. Which team has won the Syed Mushtaq Ali Trophy 2021-22 in cricket? (a) Karnataka (c) Madhya Pradesh

(b) Tamil Nadu (d) Gujarat

27. Who has become the youngest bowler to take 400 T20 w ickets? (a) Rashid Khan (c) Stuart Broad

(b) Jasprit Bumrah (d) Dale Steyn

28. Which player has won the 2021 Denmark Open men’s single badminton championship? (a) Kento Momota (c) Chen Long

(b) Viktor Axelsen (d) Anders Antonsen

https://sscstudy.com/

25

CURRENT AFFAIRS ONELINER

29. India beat which team to w in the 2021 SAFF Championship? (a) Sri Lanka (c) Malaysia

(b) Nepal (d) Japan

30. How many medals have been won by (b) 31

(c) 52

regarding US Open 2021. 1. Russia’s Daniil Medvedev won the men’s singles title. 2. Leylah Fernandez won the women's singles title. Codes

Singh, who has passed away on June 18, 2021, was associated w ith which sports event?

39. Avani Lekhara has won the first gold

(a) Lewis Hamilton (c) Sergio Perez

medal for India at Tokyo Paralympics 2020 in which event?

48. Name the w inner of women’s singles (a) Sofia Kenin (b) Elise Mertens (c) Katerina Siniaková (d) Barbara Krejcikova

49. CRICURU is India’s first AI-enabled

(b) Javelin throw (d) Long Jump

coaching website, launched by which Indian cricket player? (a) Sourav Ganguly (b) Yuvraj Singh (c) Virender Sehwag (d) Sachin Tendulkar

41. Who has become the first Assamese woman to assure Indian an Olympic medal?

50. Sanjeet Kumar, is an Indian sports

(a) Mirabai Chanu (b) Mary Kom  (c) Lovlina Borgohain (d) Hima Das

33. Jyothi Surekha Vennam has recently won silver medal in which sports event for India at the international platform? (b) Badminton (d) Shooting

Beijing 2022 Winter Olympics and Paralympics?

person, related to which sports category?

42. Name the w inner of the 2021

(a) Weightlifting (c) Sprinting

Wimbledon women’s singles title in Tennis?

51. Pooja Rani has won the gold medal

(a) Sofia Kenin (c) Simona Halep

34. What is the official motto of the (a) Together for a Shared Future (b) Faster, Higher, Stronger – Together (c) Passion lives here (d) With glowing hearts

recently become the youngest-ever Grandmaster in chess history by breaking the previous record held by Sergey Karjakin.

35. Name the Indian para-athlete who

(a) Abhimanyu Mishra (b) Anish Giri (c) Nihal Sarin (d) Samay Raina

(a) Wresling (c) Boxing

group to look into Domestic Cricket, including the compensation package, amid Pandemic. How many members are there in this group?

36. Harvinder Singh has become the

(a) 9 (c) 11

first ever Indian para-athlete to w in a medal in …… event at the Paralympics (b) Archery (d) Badminton

single title at the Italian Open 2021? (a) Novak Djokovic (b) Rafael Nadal (c) Stefanos Tsitsipas (d) Daniil Medvedev

53. Which football team has won the UEFA Champions League 2020-21? (a) Chelsea (b) Manchester City (c) Liverpool (d) West Ham United

(b) 7 (d) 15

45. Who has been named as the Captain

54. Iga Sw iatek has won the women’s

for the Indian Men’s Hockey Team, participating in Tokyo Olympics 2021?

single title at the Italian Open 2021 Tennis tournament. She represents which country?

(a) Rupinder Pal Singh (b) Harmanpreet Singh (c) Manpreet Singh (d) Surender Kumar

37. The National Sports Day is observed in India to mark the birth anniversary of which Indian sports person?

(b) Shooting (d) Sprinting

52. In Tennis, who has won the men’s

44. The BCCI has formed a working

(a) Mariyappan Thangavelu (b) Sharad Kumar (c) Devendra Jhajharia (d) Avani Lekhara

(b) Shooting (d) Boxing

on May 30,2021 for India in which sports event?

(b) Ashleigh Barty (d) Karolina Pliskova

43. Name the chess player who has

has won two medals at the ongoing Tokyo Games in Shooting event?

(b) Max Verstappen (d) Charles Leclerc

title at French Open 2021.

(b) Shooting (d) Badminton

(a) Discuss Throw (c) Shotput

(b) 1 Gold (d) 3 Silver

(a) Shotput (c) Discuss Throw

the French Grand Prix 2021?

claimed gold medal in which event at the ongoing Tokyo Paralympics?

Indian contingent at the 2021 World Archery Championships?

(b) Athletics (d) Shooting

47. Who among the follow ing has won

(b) Javelin Throw (d) Shooting

40. Indian para-athlete Sumit Antil has

32. How many medals have been won by

(a) Archery (c) Tennis

(a) Boxing (c) Weightlifting

(a) Archery (c) Tennis

(b) Only 1 (d) None of these

(a) 3 Bronze (c) 2 Silver

38. Indian para-athlete Mariyappan (a) Discuss Throw (c) High Jump

(d) 28

31. Choose the correct statement

(a) Both 1 and 2 (c) Only 2

46. India’s Asian Gold-medalist Milkha

Thangavelu has claimed a silver in which event at Tokyo 2020 Paralympics?

Indian Shooting team at the 2021 ISSF Junior World Championship? (a) 43

(a) KD Jadhav (b) Dhanraj Pillay (c) Milkha Singh (d) Major Dhyan Chand

(a) Russia (c) Spain

(b) Poland (d) Mexico

ANSWERS 1. 11. 21. 31. 41. 51.

(c) (c) (d) (b) (c) (c)

2. 12. 22. 32. 42. 52.

(a) (c) (a) (d) (b) (d)

3. 13. 23. 33. 43. 53.

(b) (d) (b) (a) (a) (a)

4. 14. 24. 34. 44. 54.

(c) (b) (a) (a) (b) (b)

5. 15. 25. 35. 45.

(d) (a) (d) (d) (c)

6. 16. 26. 36. 46.

(b) (c) (b) (b) (b)

https://sscstudy.com/

7. 17. 27. 37. 47.

(a) (a) (a) (d) (b)

8. 18. 28. 38. 48.

(a) (b) (b) (c) (d)

9. 19. 29. 39. 49.

(a) (b) (b) (b) (c)

10. 20. 30. 40. 50.

(b) (c) (a) (b) (d)

https://sscstudy.com/

26

CUET (UG) Section III : General Test

Awards and Honours 1. Which film has won the Best Picture

Of the above correct statement(s) is/are

17. Which city has received the Swachh

award at the 94th Academy Awards?

(a) Both 1 and 2 (c) Only 2

Survekshan Award 2021 for being the Cleanest City of India?

(a) Belfast (b) CODA (c) The Power of the Dog (d) King Richard

9. How many peoples have been awarded ‘Padma Bhushan’ in Padma Awards 2022?

2. Match the follow ing columns : A. Stockholm Water Prize for 2022

1. Mario Marcel

B. TIME100 Impact Awards 2022

2. Dennis Parnell Sullivan

C. Abel Prize 2022

3. Deepika Padukone

D 2022 Governor of 4. Wilfried Brutsaert . the year award

Codes A B C D (a) 3 4 2 1 (c) 4 3 2 1

(b) Only 1 (d) None of these

(a) 4 (c) 52

(b) 17 (d) 74

10. Who has won the award for the Best National Costume at the Mrs. World 2022 pageant? (a) Suman Rao (c) Vartika Singh

(b) Sanjana Vij (d) Navdeep Kaur

(a) Ahmedabad (c) Mumbai

18. Match the follow ing columns : A. Vihaan Agarwal 1. Tata Literature Live! Lifetime Achievement and Nav Award 2021 Agarwal B. M. Mukundan

B 2 3

C D 3 1 1 2

3. Name the w inner of Miss World 2021

Assam’s highest civilian honour ‘Assam Baibhav’ for the year 2021? (a) Ratan Tata (c) Azim Premji

(b) Adar Poonawalla (d) Narayana Murthy

beauty pageant?

12. Prime Minister Narendra Modi was

(a) Milena Sadowska (b) Karolina Bielawska (c) Izabella Krzan (d) Rozalia Mancewicz

conferred with which country’s highest civilian award, Ngadag Pel gi Khorlo [Order of the Dragon King (Druk Gyalpo)]?

4. Which state has topped under the Best

(a) Vietnam (c) Japan

(b) Bhutan (d) Thailand

State category at the third National Water Awards?

13. Who has won “Woman of the Year

(a) Uttar Pradesh (c) Karnataka

5. Which airport has been adjudged as

(a) Manasi Joshi (b) Koneru Humpy (c) PV Sindhu (d) Anju Bobby George

the ‘Best Airport’ under the General Category at Wings India 2022?

14. Name the Sahitya Akademi and

(b) Madhya Pradesh (d) Gujarat

Award” at World Athletics Awards 2021?

2. Indian Personality of the Year 2021

C. Hema Malini and 3. JCB Prize for Prasoon Joshi Literature 2021 D. Anita Desai

11. Who has been conferred w ith the A (b) 4 (d) 4

(b) Indore (d) Surat

Codes A B C D (a) 3 4 2 1 (c) 4 3 2 1

4. International Children’s Peace Prize 2021

A (b) 4 (d) 4

B 2 3

C D 3 1 1 2

19. Who has been awarded the Best Actor award at the BRICS Film Festival 2021? (a) Vikram Gokhale (b) Jitendra Bhikulal Joshi (c) Kunal Kapoor (d) Dhanush

20. Name the w inner of the 2021 Booker Prize for Fiction. (a) Richard Powers (c) Marion Hänsel

(b) Maggie Shipstead (d) Damon Galgut

21. Name the w inners of the 2021

(a) Rajiv Gandhi International Airport (b) Chhatrapati Shivaji International Airport (c) Kempegowda International Airport (d) Netaji Subhash Chandra Bose International Airport

Padma Shri awardee Assamese poet who has won the 56th Jnanpith Award.

Global Leadership Award, by the US India Business Council (USIBC)?

(a) Akkitham Achuthan Namboothiri (b) Amitav Ghosh (c) Shanka Ghosh (d) Nilmani Phookan

6. Which Indian mathematician was

15. Who has won the 2021 Miss

(a) Shiv Nadar and Mallika Srinivasan (b) N. Chandrasekaran and Jim Taiclet (c) Sundar Pichai and Adena Friedman (d) Adar Poonawalla and Adil Zainulbhai

conferred ‘Ramanujan Prize for Young Mathematicians 2021’? (a) Neena Gupta (b) R. Praggnanandhaa (c) Dr. B. Uma Shankar (d) Dr. Kuntal Ghosh

Universe crown? (a) Adline Castelino (c) Andrea Meza

16. Match the follow ing columns : A. SAIL

1. RedInk Award for ‘Journalist of the Year’ 2020 2. THE Asia Awards 2021

7. Which film won Best Film award at the Dadasaheb Phalke International Film Festival Awards 2022? (a) Shershaah (c) Mimi

(b) 83 (d) Pushpa

8. Consider the follow ing statements : 1. Bill Gates has been conferred with ‘Hilal-e-Pakistan’. 2. Hilal-e-Pakistan is the highest civilian award of the Islamic Republic of Pakistan.

(b) Vartika Singh (d) Harnaaz Sandhu

B. Divya Hedge C. OP Jindal 3. UN Women’s Award for Leadership Commitment Global University D. Danish 4. Golden Peacock Siddiqui Environment Management Award 2021

Codes A B C D (a) 3 4 2 1 (c) 4 3 2 1

A (b) 4 (d) 4

B 2 3

C D 3 1 1 2

https://sscstudy.com/

22. Which programme has won the outstanding drama series at the 2021 Emmy Award? (a) Mare of East town (b) The Crown (c) Ted Lasso (d) Hacks

23. Dr. Firdausi Qadri and Muhammad Amjad Saqib, who were seen in the news recently, are the recipients of which award? (a) Booker Prize (b) Ramon Magsaysay Award (c) Pulitzer Award (d) Abel Prize

24. Name the w inner of Bird Photographer of the Year (BPOTY) 2021? (a) Felipe Foncueva (c) Jonas Classon

(b) Alejandro Prieto (d) Maofeng Shen

https://sscstudy.com/

27

CURRENT AFFAIRS ONELINER

accomplishments in the area of tobacco control

25. Name the w inners of the 2021 Nobel Prize in Physics. (a) Syukuro Manabe, Klaus Hasselmann and Giorgio Parisi (b) Andrea M. Ghez, Jim Peebles and Michel Mayor (c) Reinhard Genzel, Didier Queloz and Kip Thorne (d) Arthur (b) McDonald, Peter Higgs and Adam Riess

(a) Narendra Modi (c) Harsh Vardhan

26. Name the Indian activist who has

34. Who among the follow ing has won

Deendayal Upadhyay Telecom Skill Excellence Award

(b) Smriti Irani (d) Piyush Goyal

(a) Sreenivas Karanam (b) Sreelakshmi Suresh (c) Deepanjali Dalmia (d) Ranveer Allahbadia

33. Who is the w inner of the 2021 International Booker Prize? (a) David Diop (b) Olga Tokarczuk (c) Marieke Lucas Rijneveld (d) Han Kang

been felicitated w ith the prestigious Commonwealth Points of Light award by the UK government recently?

42. Name the Indian film which has won the Best Foreign Language Feature award at Moscow International Film Festival (a) Choked (c) Bulbbul

Pulitzer Prize 2021 in ‘International Reporting category’ for reporting on China’s detention camps for Muslims?

(a) Yasir Nadeem al Wajidi (b) Hucheshwar Gurusidha Mudgal (c) Bharat Bhushan Tyagi (d) Syed Osman Azhar Maqsusi

Hadlee Medal for the 2020-21 season at the New Zealand Cricket Awards? (a) Glenn Turner (b) Ross Taylor (c) Kane Williamson (d) Chris Cairns

35. Who among the follow ing has won the Miss Universe title 2020? (a) Julia Gama (c) Adline Castelino

included in the India-based enterprises announced Best Small Business by United Nations?

(b) Andrea Meza (d) Kimberly Jimenez

44. Name the Indian filmmaker who has been honored with the second highest French civilian honour “Knight of the Order of Arts and Letters”.

36. Who is the w inner of the

(a) Edible Routes Private Limited (b) Oorja Development Solutions India (c) Taru Naturals (d) Zydus Wellness

UNESCO/Guillermo Cano World Press Freedom Prize 2021?

28. Who won the Maharashtra Bhushan

37. Which nation honoured Nobel

Award 2021?

Laureate Amartya Sen w ith its top award in social sciences?

(a) Cheche Lazaro (c) Maria Ressa

(a) Sachin Tendulkar (b) Lata Mangeshkar (c) Asha Bhosle (d) Ratan Tata

(a) Denmark (c) Spain

29. The European Union’s Sakharov Prize 2020 has been awarded to the opposition movement of which country?

(a) Sunil Bohra (c) Shaan Vyas

(b) Rodrigo Duterte (d) Mike Enriquez

bagged “The Best FIFA Women’s Player” for 2020? (a) Ellen White (c) Lucy Bronze

(b) France (d) Germany

Tansen Samman for 2020? (a) Pt. Satish Vyas (b) Pt. Vidyadhar Vyas (c) Pt. Ulhaas Kashalkar (d) Pt. Dalchan Sharma

(a) Louis Theroux (b) Steve Backshall (c) Trevor McDonald (d) David Attenborough

National Award 2021? (a) Sanjay Gupta (b) Deepak Tilak (c) Cyrus Poonawalla (d) Adar Pooonawalla

47. Who has been awarded the Saraswati Samman, 2020?

39. Who has won the title of Sir Garfield

(a) K. Siva Reddy (b) Sharankumar Limbale (c) Vasdev Mohi (d) Sitanshu Yashaschandra

Sobers Award for ICC Male Cricketer of the Decade 2020?

31. Who has been awarded the prestigious Central European University (CEU) Open Society Prize for 2021?

(a) Kumar Sangakkara (b) Ravi Ashwin (c) Virat Kohli (d) Steve Smith

(a) Sharankumar Limbale (b) Rumana Sinha Sehgal (c) KK Shailaja (d) Guneet Monga

48. Which film has won the best film

40. Which film has won the Best Picture

award at the 66th Filmfare Awards for the year 2021?

Award at the 93rd Academy Awards or Oscars 2021?

(a) Thappad (b) Tanhaji : The Unsung Warrior (c) Angrezi Medium (d) Ludo

(a) The Father (b) Judas And The Black Messiah (c) Nomadland (d) Minari

32. Name the Indian who has been awarded w ith the ‘WHO DirectorGeneral Special Recognition Award’ for

(b) Jill Scott (d) Toni Duggan

46. Who has been conferred w ith the

“People’s Advocate” of the UN Climate Change Conference (COP26)?

30. Who won the Lokmanya Tilak

(b) Anurag Kashyap (d) Guneet Monga

45. Name the female player who has

38. Who has been chosen as the

(b) Chile (d) Singapore

(b) Panga (d) Puglya

43. Name the w inner of the Sir Richard

(a) Megha Rajagopalan (b) Alison Killing (c) Christo Buschek (d) All of the above

27. Which amongst the follow ing is not

(a) Belarus (c) Argentina

41. Name the w inner of the Pandit

ANSWERS 1. 11. 21. 31. 41.

(b) (a) (a) (c) (a)

2. 12. 22. 32. 42.

(c) (b) (b) (c) (d)

3. 13. 23. 33. 43.

(b) (d) (b) (a) (c)

4. 14. 24. 34. 44.

(a) (d) (b) (d) (d)

5. 15. 25. 35. 45.

(c) (d) (a) (b) (c)

6. 16. 26. 36. 46.

(a) (c) (d) (c) (a)

https://sscstudy.com/

7. 17. 27. 37. 47.

(b) (b) (d) (c) (b)

8. 18. 28. 38. 48.

(b) (c) (c) (d) (a)

9. 19. 29. 39.

(b) (d) (a) (c)

10. 20. 30. 40.

(d) (d) (c) (c)

https://sscstudy.com/

28

CUET (UG) Section III : General Test

In the News 1. Who has been elected as the new

10. Who among the follow ing has been

19. Indian social worker Sindhutai

Director-General of the International Labour Organisation (ILO)?

appointed as chairman of Pfizer India?

Sapkal, who has passed away recently, was fondly referred as

(a) Gilbert Houngbo (b) Muriel Penicaud (c) Kanayo Nwanze (d) Kang Kyung-wha

11. Who among the follow ing took

2. Ramesh Chandra Lahoti, who has passed away recently, was the former .............. of India? (a) Lok Sabha Speaker (b) Chief Justice (c) Chief Election Commissioner (d) ISRO Chairman

3. Name the creator of the Graphics Interchange Format (GIF), who has passed away recently. (a) Alan Ladd Jr (c) Stephen Wilhite

(b) Eugene Parker (d) Rod Marsh

4. Who has been elected as the fellow of the Royal Society of Edinburgh (RSE) in Scotland? (a) Chanda Kochhar (b) Vandana Luthra (c) Soumya Swaminathan (d) Kiran Mazumdar-Shaw

5. Which country’s former President Shahabuddin Ahmed passed away recently? (a) Pakistan (c) Afghanistan

(b) Saudi Arabia (d) Bangladesh

6. Choose the correct statement(s) regarding Ketanji Jackson. 1. Ketanji Jackson nominated as first Black Woman to US Supreme Court. 2. She would replace liberal Supreme Court Justice Stephen Breyer. Codes (a) Both 1 and 2 (c) Only 2

(a) KV Kamath (b) Rajiv Mehrishi (c) Mukundakam Sharma (d) Pradip Shah

(b) Only 1 (d) None of these

7. KN Raghavan has recently been appointed as the Chairman of which international body? (a) International Rubber Study Group (b) International Steel Group (c) International Dairy Federation (d) International Automotive Task Force

charge as the Director of Vikram Sarabhai Space Centre (VSSC)? (a) Unnikrishnan Nair (b) Badrinath Srinivasan (c) Dileep Sanghani (d) Vikram Dev Dutt

12. Who has been named as the next chief economist of the International Monetary Fund (IMF)? (a) Geoffrey Okamoto (b) Gita Gopinath (c) Pierre-Olivier Gourinchas (d) Kristalina Georgieva

13. Who among the follow ing has been appointed as the head of United Nations Children’s Fund (UNICEF)? (a) Audrey Azoulay (b) Catherine Russell (c) Henrietta H. Fore (d) Rafael Grossi

14. Choose the correct statement regarding Elon Musk. 1. Elon Musk named TIME Magazine’s ‘Person of the Year’ for 2021. 2. Musk is also the founder and CEO of rocket company SpaceX. Codes (a) Both 1 and 2 (c) Only 2

(b) Only 1 (d) None of these

15. Abdalla Hamdok has resigned as the Prime Minister of which country? (a) Rwanda (c) Sudan

(b) Turkey (d) Somalia

16. Who has been appointed as the next President of Federation of Indian Chambers of Commerce and Industry (FICCI)? (a) Soma Sankara Prasad (b) Ramalingam Sudhakar (c) Sanjiv Mehta (d) Arun Kumar Mishra

17. Name the India’s first woman

(a) Women of Peace (b) Iron Lady of India (c) Mother of Orphans (d) Lioness of Maratha

20. Who has been appointed as the next Chief of the Naval Staff to succeed Admiral Karambir Singh? (a) Anil Kumar Chawla (b) R. Hari Kumar (c) Shekhar Sinha (d) Surinder Pal Singh Cheema

21. Keshav Desiraju who has passed away recently had formerly served at which post in the Union Cabinet? (a) Economic Secretary (b) Health Secretary (c) Finance Secretary (d) Agricultural Secretary

22. Consider the follow ing statements : 1. Air Chief Marshal Vivek Ram Chaudhari assumed charge as the Chief of the Air Staff (CAS). 2. Air Marshal Sandeep Singh has been appointed as the new and 46th Vice Chief of the Air Staff (VCAS) of the Indian Air Force (IAF). Of the above correct statement(s) is/are (a) Both 1 and 2 (c) Only 2

(b) Only 1 (d) None of these

23. The 23-year-old Shashvat Nakrani is the youngest self-made individual in Hurun India Rich List 2021. He is the co-founder of which company? (a) Paytm (c) WinZO

(b) PhonePe (d) BharatPe

24. Kamla Bhasin, who has passed away recently, was a well known author and poet, besides being (a) Journalist (b) Women’s rights activist (c) Basketball player (d) Filmmaker

25. Name the mountaineer who has

8. Who has been appointed as India’s

psychiatrist, who passed away recently.

first National Maritime Security Coordinator (NMSC)?

achieved the feat of becoming the fastest Indian to scale two peaks?

(a) G S Lakshmi (b) Sarada Menon (c) Janani Narayanan (d) Vrinda Rathi

(a) Shivangi Pathak (c) Chhanda Gayen

(a) Ravneet Singh (c) G. Ashok Kumar

18. Who has been appointed as the new

26. Who has been appointed as the

(b) Karambir Singh (d) Sunil Lanba

9. Nobel Prize w inner Luc Montagnier passed away recently. He was a/an (a) Astrophysicist (c) Paleontologist

(b) Cardiologist (d) Virologist

Chairman and CEO of Railway Board? (a) Sanjeev Mittal (b) Vinay Kumar Tripathi (c) Naresh Salecha (d) Ravinder Gupta

https://sscstudy.com/

(b) Geeta Samota (d) Premlata Agrawal

Chairman of the Central Board of Direct Taxes (CBDT)? (a) JB Mohapatra (b) Rajesh Damor (c) Pramod Y. Devikar (d) Deepa Rasal

https://sscstudy.com/

CURRENT AFFAIRS ONELINER

27. Which Indian banker has been appointed as the independent director of the Hongkong and Shanghai Banking Corporation (HSBC) Asia? (a) Rajnish Kumar (b) Aditya Puri (c) Arundhati Bhattacharya (d) Anshula Kant

28. Who has been appointed as the Chairperson of Stop TB Partnership Board? (a) Harsh Vardhan (b) Mansukh Mandaviya (c) Kiren Rijiju (d) Dharmendra Pradhan

29. Who has been roped in by the Delhi government as the brand ambassador for its ‘Desh Ke Mentors’ programme? (a) Virat Kohli (c) Ranvir Singh

(b) Sonu Sood (d) Kapil Dev

30. Who has been appointed as the Chairman of the reconstituted Advisory Board for Banking and Financial Frauds (ABBFF) by CVC?

29

37. Name the CEO of NITI Aayog, whose

(a) Ranganath Misra (b) MN Venkatachalliah (c) Arun Mishra (d) S. Rajendra Babu

tenure has been extended by government for one year till June 2022?

47. Name the female IAF officer who has

(a) Arvind Panagariya (b) Rajiv Kumar (c) Amitabh Kant (d) Nripendra Misra

become the first woman flight test engineer of India?

(a) Rashi (c) Brahman

(b) Sudharma (d) Purana

38. The renowned software developer and the creator of the McAfee antivirus software has passed away at the age of 75, due to suicide. Name that software pioneer (a) Kim McAfee (c) Nick McAfee

(b) Charles McAfee (d) John McAfee

39. What was the profession of Raj Kaushal who has passed away recently at the age of 49? (a) Politician (c) Journalist

(b) Sportsman (d) Filmmaker

40. Satya Nadella has been appointed as the Chairman of Microsoft in June, 2021. He replaced whom? (a) Manu Sawhney (c) Gina Haspel

(b) Mark Lowcock (d) John Thompson

(a) Padmavathy Bandopadhyay (b) Mitali Madhumita (c) Aashritha V Olety (d) Sophia Qureshi

48. Who has been appointed as the new Deputy Chief of Naval Staff? (a) Ravneet Singh (c) Satish Soni

(b) G. Ashok Kumar (d) Jagjit Singh Bedi

49. Who has been sworn in as the new Chief Minister of Assam to replace Sarbananda Sonowal? (a) Prafulla Kumar Mahanta (b) Himanta Biswa Sarma (c) Atul Bora (d) Kailash Nath Sarma

50. Name the women mountaineer who

41. Tadang Minu is the first Indian

has recently created historic record of world’s fastest ascent of Everest by a woman

Karnataka's new Chief Minister?

female to be appointed as a member of the Coaches Committee of the International Boxing Association (AIBA). She hails from which State?

(a) Gerlinde Kaltenbrunner (b) Oh Eun-sun (c) Moni Mulepati (d) Tsang Yin-hung

(a) Arun Singh (c) CT Ravi

(a) Manipur (c) Ladakh

51. Who has been appointed as the new

(a) Ajay Banga (b) TM Bhasin (c) Kris Gopalakrishnan (d) NR Narayana Murthy

31. Who has been sworn in as (b) Thaawarchand Gehlot (d) Basavaraj Bommai

(b) Sikkim (d) Arunachal Pradesh

32. Which nation's President has sacked

42. Who has been appointed as the new

the Prime Minister and suspend the Parliament?

Chairman of Microsoft?

(a) Armenia (c) Tunisia

(b) Turkmenistan (d) Indonesia

33. Dr. PK Warrier, who has passed away

(a) Elon Musk (c) Larry Page

(b) Sunder Pichai (d) Satya Nadella

43. Centre has extended the term of

at the age of 100 recently, was a veteran

which Deputy Governors of RBI by two years, till June 2023?

(a) Agricultural Scientist (b) Economist (c) Astrologer (d) Ayurveda practitioner

(a) Michael Patra (b) Mahesh Kumar Jain (c) Rabi Sankar (d) M. Rajeshwar Rao

34. Who has been appointed as the

44. Who has been appointed as the

Chairman of Indian Federation of United Nations Associations (IFUNA)?

President of 76th UN General Assembly (UNGA) for 2021-22?

(a) Justice Sudhanshu Dhulia (b) Justice Rajesh Bindal (c) Justice Mohammad Rafiq (d) Justice Shambhu Nath Srivastava

(a) Peter Mohan (c) Omar Hilale

35. Indian politician Virbhadra Singh, who has passed away was the former Chief Minister of which State?

(b) Munir Akram (d) Abdulla Shahid

45. What was the profession of TM Kalliannan Gounder, who has passed away at the age of 101?

(a) Himachal Pradesh (b) Gujarat (c) Haryana (d) Bihar

(a) Actor (b) Agricultural Scientist (c) Freedom Fighter (d) Sportsperson

36. KV Sampath Kumar, the editor of

46. Who among the follow ing has been

world’s first Sanskrit Daily, has passed away. What is the name of the Sanskrit Daily?

appointed as the new Chairperson of the NHRC of India?

https://sscstudy.com/

Director of the CBI? (a) Subodh Kumar Jaiswal (b) Vishwas Nangare Patil (c) Amulya Patnaik (d) Anil Deshmukh

52. Who among the follow ing has been appointed as the new Chairperson of the National Human Rights Commission (NHRC) of India? (a) Ranganath Misra (b) MN Venkatachalliah (c) Arun Mishra (d) S. Rajendra Babu

53. Noted Indian environmentalist Sunderlal Bahuguna passed away due to COVID-19 on May 21, 2021. He was a prominent leader of which among the follow ing movements? (a) Narmada Bachao Andholan (b) Chipko Movement (c) Save Silent Valley Movement (d) Appiko Movement

54. Jagannath Bidyadhar Mohapatra has been roped in as the new Chairman of which of these organisation? (a) CBDT (c) CBI

(b) NITI Aayog (d) SIDBI

https://sscstudy.com/

30

CUET (UG) Section III : General Test

as the Chief Minister of Kerala for second straight term. He is the .............. CM of the State. (a) 12th

(b) 14th

(c) 11th

(d) 15th

56. Who has been appointed as the new CBI Chief? (a) Rakesh Asthana (c) Subodh Jaiswal

(c) Ramesh Pokhriyal Nishank (d) Bhagat Singh Koshyari

(a) C. Rangarajan (b) M. Narasimham (c) S. Venkitaramanan (d) YV Reddy

55. Pinarayi Vijayan has been sworn in

(b) YC Modi (d) SS Deswal

72. Lou Ottens, the inventor of the audio

64. Justice Nuthalapati Venkata Ramana

cassette tape has passed away. He was from which country?

has been sworn in as the ............. Chief Justice of India (CJI).

(a) Ireland (c) Netherlands

(a) 40th (c) 52nd

73. The Padma Bhushan awardee

(b) 36th (d) 48th

the youngest mayor in India. She is from which State?

has become the first female of the country to summit the Mt Annapurna in April 2021, the tenth highest mountain in the world.

(a) Tamil Nadu (c) Kerala

(a) Gurmayum Devi (c) Priyanka Mohite

(b) Assam (d) Goa

(a) Agriculture (c) Literature

(b) Malavath Purna (d) Santosh Yadav

58. Who has been sworn in as the new

66. GVG Krishnamurty has passed away.

Chief Minister of Tamil Nadu after 2021 Assembly election?

He was the former ............. of India

(a) MK Tamilarasu (c) M. Karunanidhi

Laxman Pai has passed away. He was conferred w ith the prestigious honor for contribution in which field?

65. Name the Indian mountaineer, who

57. Arya Rajendran, has been elected as

59. Name the newly appointed Chief

listed in the 2021 Young Global Leaders (YGLs) forum by World Economic Forum (a) Anushka Sharma (b) Shraddha Kapoor (c) Deepika Padukone (d) Alia Bhatt

75. Name the Indian economist who has

Minister of Union Territory of Puducherry

67. Who has been appointed as the new

(a) V. Vaithilingam (c) N. Rangasamy

Chief Election Commissioner of India?

(b) V. Narayanasamy (d) YS Bharati

(a) Sushil Chandra (c) Navin Chawla

60. Chaudhary Ajit Singh, who has

been appointed as the Assistant Secretary-General and Head of the New York Office of the UNEP (a) Prakash Lohia (b) Arora Akanksha (c) V. Sundramoorthy (d) Ligia Noronha

(b) Rajiv Kumar (d) Nagendra Singh

76. Who has been appointed as the new

68. Who has been appointed as the new

passed away recently was the founder of which political party?

Chairman and Managing Director of SIDBI?

Chairman of Steel Authority of India Limited (SAIL)?

(a) Ambedkar Samaj Party (ASP) (b) Rashtriya Lok Dal (RLD) (c) Bahujan Samaj Party (BSP) (d) Rashtriya Janata Dal (RJD)

(a) Mistu Mahajbin (c) Meeta Baghel

(a) S. Ramann (b) Devendra Kumar Singh (c) V. Satya Venkata Rao (d) Pankaj Jain

(b) Soma Mondal (d) Radha Pyari

69. Who has been appointed as the new

61. Mamata Banerjee has been sworn in

Chairman & CEO of Railway Board?

as the Chief Minister of West Bengal after massive w in in the 2021 Assembly election. This w in is her ............. consecutive w in as the head of the state.

(a) Sanjay Pal (b) Amit Dhawan (c) Suneet Sharma (d) Harish Kumar Sahu

(a) fourth (c) third

70. Justice Hima Kohli has been

(b) second (d) fifth

77. Chandra Nayudu, who has passed away, was first women commentator of which sports? (a) Football (c) Tennis

62. Name the fourth Deputy Governor of the RBI, who has recently been appointed in place of BP Kanungo

(a) Karnataka (c) Ladakh

(a) T. Rabi Sankar (b) GK Chadha (c) DM Nanjundappa (d) TN Srinivasan

71. Name the new Chief Minister of

topped the Forbes list of India’s 10 richest billionaires for the year 2021 (a) Gautam Adani (c) Mukesh Ambani

(b) Delhi (d) Telangana

(b) Shiv Nadar (d) Uday Kotak

79. Name the Deputy Governor of RBI who has retired from his service.

Uttarakhand, who has been appointed to replace the former CM Trivendra Singh Rawat?

considered as the “father of banking reforms in India”, who has passed away due to COVID-19 related illness.

(b) Cricket (d) Hockey

78. Name the Indian billionaire who has

appointed as the first-ever female Chief Justice of the High Court of

63. Name the former Governor of RBI,

(b) Mathematics (d) Painting

74. Which Indian celebrity has been

(a) Home Minister (b) President (c) Election Commissioner (d) Chief of Army Staff

(b) MK Stalin (d) MK Alagiri

(b) Finland (d) Sweden

(a) BP Kanungo (b) Mahesh Kumar Jain (c) Michael Patra (d) Rajeshwar Rao

(a) Tirath Singh Rawat (b) Dhan Singh Rawat

ANSWERS 1. 11. 21. 31. 41. 51. 61. 71.

(a) (a) (b) (d) (d) (a) (c) (a)

2. 12. 22. 32. 42. 52. 62. 72.

(b) (c) (a) (c) (d) (c) (a) (d)

3. 13. 23. 33. 43. 53. 63. 73.

(c) (b) (d) (d) (b) (b) (b) (d)

4. 14. 24. 34. 44. 54. 64. 74.

(d) (a) (b) (d) (d) (a) (d) (c)

5. 15. 25. 35. 45. 55. 65. 75.

(d) (c) (b) (a) (c) (a) (c) (d)

6. 16. 26. 36. 46. 56. 66. 76.

(a) (c) (a) (b) (c) (c) (c) (a)

https://sscstudy.com/

7. 17. 27. 37. 47. 57. 67. 77.

(a) (b) (a) (c) (c) (c) (a) (b)

8. 18. 28. 38. 48. 58. 68. 78.

(c) (b) (b) (d) (a) (b) (b) (c)

9. 19. 29. 39. 49. 59. 69. 79.

(d) (c) (b) (d) (b) (b) (c) (a)

10. 20. 30. 40. 50. 60. 70.

(d) (b) (b) (d) (d) (b) (d)

https://sscstudy.com/

CURRENT AFFAIRS ONELINER

31

Miscellaneous 1. What is the theme of World Theatre

10. South Maubuang in which state, has

Day in the year 2022?

been declared as the first model ODF Plus village of the state?

(a) Theatre and a Culture of Peace (b) Tales of Theatre (c) Theatre of Nations (d) Theatre and Drama in Education

(a) Meghalaya (c) Tripura

(b) Mizoram (d) Assam

(a) Puneet Dalmia (b) Rakeysh Omprakash Mehra (c) Suresh Raina (d) Narotam Sekhsaria

20. What was the theme for World AIDS

11. Which of the follow ing state has got

Day 2021?

‘The Little Book of Joy’?

first-of-its-kind state-level bird atlas in India?

(a) Desmond Tutu (c) Penpa Tsering

(a) Karnataka (c) Tamil Nadu

(a) Communities make the difference (b) End inequalities, End AIDS and End Pandemics (c) On the fast track to end AIDS (d) Global Solidarity and Shared Responsibility

2. Who has authored a children’s book (b) Tenzin Gyatso (d) Both (a) and (b)

(b) Telangana (d) Kerala

3. United Nations General Assembly has

12. Which day has been declared by

proclaimed which day as the International Day to Combat Islamophobia?

Prime Minister Narendra Modi to be observed as ‘National Start-up Day’ in India?

(a) March 14 (c) March 17

(b) March 16 (d) March 15

4. The Dredging Museum named ‘Nikarshan Sadan’ has been inaugurated in, which city? (a) Chennai (c) Kochi

(b) Vishakhapatnam (d) Panaji

5. India’s first Biosafety level-3 containment mobile laboratory has been inaugurated in (a) Jaipur (Rajasthan) (b) Nashik (Maharashtra) (c) Chennai (Tamil Nadu) (d) Gurugram (Haryana)

6. What is the theme of National Science Day 2022?

(a) January 5 (c) January 18

(b) January 16 (d) January 22

13. Which Ministry has come out w ith the pictorial comic book titled ‘India’s Women Unsung Heroes’? (a) Ministry of Culture (b) Ministry of Women & Child Development (c) Ministry of Defence (d) Ministry of Sports

14. On which day World Leprosy Day is observed every year? (a) Last Thursday of January (b) Last Friday of January (c) Last Saturday of January (d) Last Sunday of January

(a) The Future of STI (b) Future of STI : Impact on Education Skills and Work (c) Women in Science (d) Integrated Approach in S&T for Sustainable Future

15. Who among the follow ing is the

7. The book titled ‘A Nation To Protect’

………… as Children’s Word of the Year 2021, based on their recent research.

has been authored by (a) Narayan Rane (b) Priyam Gandhi Mody (c) Mansukh Mandaviya (d) Pranab Mukherjee

8. Match the follow ing columns : A. Maru Mahotsav

1. Telangana

B. Khajuraho Festival 2. Jammu and Kashmir C. Kanchoth festival

3. Madhya Pradesh

D. Medaram Jatara Festival

4. Rajasthan

(a) a-3, b-4, c-2, d-1 (c) a-4, b-3, c-2, d-1

(b) a-4, b-2, c-3, d-1 (d) a-4, b-3, c-1, d-2

9. The government of India has organised the annual Polio National Immunisation Day 2022 on which day? (a) February 26 (c) February 27

(b) February 28 (d) February 25

author of the book titled ‘Operation Khatma’? (a) RC Ganjoo (c) Kiran Bedi

(b) Ashwini Bhatnagar (d) Both (a) and (b)

16. Oxford University Press has declared (a) Coronavirus (c) Anxiety

(b) Isolate (d) Pandemic

17. PM Modi unveiled 216-foot statue of saint Ramanujacharya in which of the follow ing city? (a) Varanasi (c) Hyderabad

(b) Chennai (d) Bengaluru

18. Which Railway Station has been named as the ‘Veerangana Laxmibai Railway Station’? (a) Habibganj Railway Station (b) Bina Junction Railway Station (c) Kanpur Central Railway Station (d) Jhansi Railway Station

19. Who has authored his autobiography titled ‘The Ambuja Story: How a Group of Ordinary Men Created an Extraordinary Company’?

https://sscstudy.com/

21. International Literacy Day is observed every year on (a) September 05 (c) September 07

(b) September 06 (d) September 08

22. What is the theme of the World Suicide Prevention Day (WSPD) 2021? (a) Take a minute, change a life (b) Creating hope through action (c) 40 seconds of action (d) Working Together to Prevent Suicide

23. Nuakhai is a harvest festival, celebrated by the native of which Indian state? (a) Karnataka (c) Andhra Pardseh

(b) Tamil Nadu (d) Odisha

24. Who is the author of the book and ‘My Life in Full: Work, Family, and Our Future’? (a) Nita Ambani (b) Melinda French Gates (c) Indra Nooyi (d) Satya Nadella

25. Who is the author of the book ‘Accelerating India : 7 Years of Modi Government’? (a) KK Shailaja (c) MK Stalin

(b) Pinarayi Vijayan (d) KJ Alphons

26. Which nation will host SCO Defence Ministers’ meet? (a) Afghanistan (c) Uzbekistan

(b) Tajikistan (d) Turkmenistan 

27. Ramappa Temple has become the 39th Indian site to be inscribed in UNESCO’s World Heritage List. It is located in which state? (a) Telangana (c) Tamil Nadu 

(b) Karnataka (d) Kerala

28. Who is the author of the book ‘The Ramayan of Shri Guru Gobind Singh Ji’? (a) Baljit Kaur Tulsi (b) Rohit Baban Deo (c) Sanjay Kishan Kaur (d) Pushpa Virendra Ganediwala

https://sscstudy.com/

32

CUET (UG) Section III : General Test

(a) Sikkim (c) Tamil Nadu

29. Choose the correct statements 1. Armenia’s acting Prime Minister Nikol Pashinyan has won the country’s parliamentary elections 2021, with 54% of the vote. 2. Pashinyan, who is the leader of the Civil Contract party, first came to power in 2018, following the country’s first free and fair election. Code (a) Only 1 (b) Only 2 (c) Both 1 and 2 (d) None of these

Convention Centre’ has been inaugurated in which of these UNESCO World Heritage locations?

Environment Day, PM Modi has launched three E 100 ethanol dispensing stations under a pilot project in which city? (a) Hyderabad (c) Pune

the National Fish Farmers’ Day? (a) July 7 (b) July 8 (c) July 9 (d) July 10

31. Who is the author of the book ‘The

(a) Sanchi (c) Konark

(b) Ahmedabad (d) New Delhi

38. Who is the author of the book

‘COVID-19: Sabhyata ka Sankat aur Samadhan’?

‘‘LANGUAGES OF TRUTH: Essays 2003-2020’’?

(a) Kailash Satyarthi (b) Narendra Modi (c) Tathagata Roy (d) Amish Tripathi

(a) VS Naipaul (b) Salman Rushdie (c) Arundhati Roy (d) Kiran Desai

46. The National Safe Motherhood Day marks the birth anniversary of which Indian political activist? (a) Kasturba Gandhi (b) Kamala Nehru (c) Indira Gandhi (d) Vijaya Lakshmi Pandit

‘Stargazing: The Players in My Life’? (a) Sourav Ganguly (b) Kapil Dev (c) Ravi Shastri (d) Mohammad Azharuddin

(b) Salman Rushdie (d) Amitav Ghosh

32. What is the theme of the 2021 International Yoga Day?

47. PM Narendra Modi released the Hindi translation of the book ‘Odisha Itihaas’. Who is the author of this book? (a) Biren Mitra (b) Harekrushna Mahtab (c) Nilamani Routray (d) Biswanath Das

40. Who is the author of the book

(a) Yoga for Peace (b) Yoga for well-being (c) Yoga at Home and Yoga with Family (d) Yoga for Climate Action

‘Savarkar: A contested Legacy (1924-1966)’? (a) Ashok Desai (c) Bibek Debroy

33. ‘Believe- What Life Taught me’, is the

(b) Hindol Sengupta (d) Vikram Sampath

48. Which country is the sponsor for International Children’s Book Day 2021?

41. What was the theme of the 2021

autobiography of which Indian player?

(a) United States (c) Australia

International Nurses Day?

(a) Gautam Gambhir (b) Shikhar Dhawan (c) Suresh Raina (d) Yuvraj Singh

(a) Nurses: A Voice to Lead – Health for All (b) Nurses: A Voice to lead – Health is a human right (c) Nurses: A Voice to Lead – Nursing the World to Health (d) Nurses: A Voice to Lead – A vision for future healthcare

34. What is the theme of World Day Against Child Labour 2021? (a) Generation Safe & Healthy (b) Protect children from child labour, now more than ever (c) Children shouldn’t work in fields, but on dreams (d) Act now: End child labour!

plant is being set-up at which place? (a) Talcher (c) Ramagundam

(a) Shilpa Shetty (c) Twinkle Khanna

every year on which day? (a) April 5 (c) April 7

every year in India on which day? (a) April 4 (c) April 6

plant is being set up at which place?

been declared as a National Park. Where is it located?

(b) April 5 (d) April 7

52. The Tulip Festival is organised in

(a) Talcher (b) Vijayawada (c) Ramagundam (d) Vindhyachal

36. Raimona Reserve Forest has recently

(b) April 6 (d) April 4

51. National Maritime Day is celebrated

(b) Kalki Koechlin (d) Anushka Sharma

43. India’s biggest floating solar power

(b) Nako (d) Weyan

(b) Vijayawada (d) Vindhyachal

50. The World Health Day is observed

Womb” is a debut book by which Indian celebrity?

the entire population above 18 years has been vaccinated.

(b) New Zealand (d) Denmark

49. India’s biggest floating solar power

42. The book titled “Elephant In The

35. Name the first village in India, where

(b) Khajuraho (d) Hampi

45. Who is the author of the book

39. Who is the author of the book

Nutmeg’s Curse: Parables for a Planet in Crisis’?

(a) Malana (c) Diskit

44. The ‘Maharaja Chhatrasal

37. On the occasion of World

30. In India, which day is observed as

(a) Arundhati Roy (c) Chetan Bhagat

(b) Assam (d) Punjab

which State/UT of India every year? (a) Jammu & Kashmir (b) Kerala (c) Himachal Pradesh (d) Nagaland

ANSWERS 1. 11. 21. 31. 41. 51.

(a) (d) (d) (d) (d) (b)

2. 12. 22. 32. 42. 52.

(d) (b) (b) (b) (b) (a)

3. 13. 23. 33. 43.

(d) (a) (d) (c) (c)

4. 14. 24. 34. 44.

(b) (d) (c) (d) (b)

5. 15. 25. 35. 45.

(b) (d) (d) (d) (a)

6. 16. 26. 36. 46.

(d) (c) (b) (b) (a)

https://sscstudy.com/

7. 17. 27. 37. 47.

(b) (c) (a) (c) (b)

8. 18. 28. 38. 48.

(c) (d) (a) (b) (a)

9. 19. 29. 39. 49.

(c) (d) (c) (c) (c)

10. 20. 30. 40. 50.

(b) (b) (d) (d) (c)

https://sscstudy.com/

GENERAL KNOWLEDGE

3

CHAPTER 01

General Knowledge Important Days Day

First in the World Date

National Youth Day

January 12

First Radio Telescope Satellite launched into Space

Japan

National Tourism Day

January 25

First country to use glass

Egypt and Mesopotamia

Martyrs’ Day

January 30

First country to make map

World Leprosy Eradication Day

January 30

Sumeria (3800 BC) (1500 BC)

National Science Day

February 28

First Space Ship Landed on Mars

Viking-I (July 1976)

International Women’s Day

March 8

World’s first Multipurpose River Valley Project

Tennessee River Valley Project (USA)

World Disabled Day

March 15

First Space Shuttle Launched

Columbia

World Consumer Rights Day

March 15

First rocket to go near the Sun

Helius ‘B’

World Forestry Day

March 21

First country to make Constitution

America

World Day for Water

March 22

First country to start underground metro rail

Britain

World TB Day

March 24

First unmanned mission on Moon

LUNA-9

World Health Day

April 7

First van to carry man on Moon

Apollo - 11

World Heritage Day

April 18 April 22

First country to do Artificial Satellite Experiment

Russia

Earth Day Worker’s Day (International Labour Day)

May 1

Country to give voting right to women

New Zealand

World Red Cross Day

May 8

First country to appoint lokpal

Sweden

National Technology Day

May 11

First country to imposed carbon tax

New Zealand

Anti-tobacco Day

May 31

First clonned animal

Dolly (a sheep)

World Environment Day

June 5

World’s first Atomic Energy Plant

EBR (USA)

World Yoga Day

June 21

World Population Day

July 11

World Senior Citizen’s Day

First in World (Male) Jagmohan Dalmiya

August 8

First Asian to head the International Cricket Council

Teacher’s Day

September 5

First man to climb Mount Everest

International Literacy Day (UNESCO)

September 8

World Ozone Day

September 16

Sherpa Tenzing Norgay and Sir Edmund Hillary (29th May, 1953)

World Tourism Day

September 27

First man to go into space

Major Yuri Gagarin (USSR)

Gandhi Jayanti; International Day of Non-violence World Food Day

October 2 October 16

First man to work in space

Alexei Leonel (Rerssiaj)

First person to give information about planets and their motion around the Sun

Nicolous Copernicus

First man to compile Encyclopaedia

Aspheosis (Athens) Albert P Carey

UN Day

October 24

Children’s Day; Diabetes Day

November 14

Citizen’s Day

November 19

First person to go on both the poles (North and South)

World AIDS Day

December 1

First man to reach North Pole

Robert Peary

Human Rights Day

December 10

First man to reach South Pole

Roald Amundsen

Good Governance Day (Birthday of Atal Bihari Vajpayee)

December 25

First man to climb on Mt Everset without oxygen

Phu Dorji Sherpa

https://sscstudy.com/

https://sscstudy.com/

4

CUET (UG) Section III : General Test

First in World (Female)

Country

Capital

Currency

First woman President of a country

Maria Estela Peron (Argentina)

Myanmar

Nai-pe-da

Kyat

First woman in the world to cross the Strait of Gibralter

Arti Pradhan (India)

Nepal

Kathmandu

Nepalese Rupee

Portugal

Lisbon

Euro

First woman Cosmonaut in space

Valentina Tereshkova (USSR)

Qatar

Doha

Riyal

First female to find asteroid

Caroline and Derschell

Russia

Moscow

Rouble

First non-white female to win Nobel Prize in Literature

Toni Morrison

Saudi Arabia

Riyadh

Rial

South Africa

Rand

First woman to reach Antarctica

Caroline Michaelson

Spain

Pretoria Madrid

First woman to have a space-walk

Svetlana Yevgenyevna Savitskaya

Sri Lanka

Sri Lankan Rupee

First Youngest girl to reached Mount Everest

Purna Malavath

Sri Jayawardene Pura Kotte

Sweden

Stockholm

Krona

Switzerland

Berne

Swiss Franc

Currency

Syria

Damascus

Syrian Pound

Dushanbe

Somoni

Countries, their Capitals and Currencies Country

Capital

Euro

Afghanistan

Kabul

Afghani

Tajikistan

Argentina

Buenos Aires

Peso

Thailand

Bangkok

Baht

Australian Dollar

Tanzania

Dodoma

Shilling

Taka

Turkey

Ankara

Lira

Euro

Turkmenistan

Askhabad

Manat

United Arab Emirates (UAE)

Abu Dhabi

Dirham

Australia Bangladesh Belgium

Canaberra Dhaka Brussels

Bhutan

Thimpu

Ngultrum

Brazil

Brasilia

Real

Brunei

Bandar Seri Begawan Brunei Dollar

United Kingdom (UK)

London

Pound Sterling

United States of America (USA)

Washington D C

Dollar

Uzbekistan

Tashkent Hanoi

Som

Vietnam Yemen

Sana’s

Cambodia

Phnom Penh

Riel

Canada

Ottawa

Canadian Dollar

Chile

Santiago

Peso

China

Beijing

Renmminbi Yuan

Cuba

Havana

Cuban Peso

East Timor

Dili

US Dollar

Egypt

Cairo

Egyptian Pound

Iceland

Reykjavik

Krona

Name of the Country

Name of the Parliament

Name of the Name of the Country Parliament

India

New Delhi

Rupee

Afghanistan

Shora

Australia

Parliament

Indonesia

Jakarta

Rupiah

Bangladesh

Jatiyo Sansad

Bhutan

Tshongdu

Iran

Tehran

Toman

Iraq

Baghdad

Iraqi Dinar

Ireland

Dublin

Euro

Israel

Jerusalem

Shekel

Japan

Tokyo

Yen

Kenya

Nairobi

Shilling

North Korea

Pyongyang

Won

South Korea

Seoul

Won

Kuwait

Kuwait city

Kuwaiti Dinar

Kyrgyzstan

Bishkek

Dong Riyal

Countries and their Parliaments

Brazil

National Congress

Canada

Parliament

China

National People’s Congress

Denmark

Folketing

Egypt

People’s Assembly

France

National Assembly

Germany

Bundestag

India

Sansad

Iran

Majlis

Iraq

National Assembly

Israel

Knesset

Japan

Diet

Maldives

Majlis

Mongolia

Khural

Som

Nepal

Rashtriya Panchayat

Pakistan

National Assembly

Sejm

Russia

Duma

Laos

Vientiane

Kip

Poland

Malaysia

Kuala Lumpur

Ringgit

Spain

Cortes Generales

Sweden

Riksdag

Maldives

Male

Rufiyaa

Saudi Arabia

Majlis Al Shura

Sudan

Majlis Watani

Taiwan

Yuan

Turkey

Grand National Assembly

USA

Congress

UK

Parliament

Mauritius

Port Louis

Mauritius Rupee

Mexico

Mexico City

Mexican Peso

Morocco

Rabat

Dirham

https://sscstudy.com/

https://sscstudy.com/

5

GENERAL KNOWLEDGE

National Emblems of Major Countries Country

Emblem

Country

Emblem

Discovery

Discoverer

Discovery

Discoverer

Solar System

Copernicus

Circumnavigation of World

Magellan

Australia

Kangaroo

Italy

White Lily

Bangladesh

Water Lily

Japan

Chrysanthemum

Planets

Kepler

Mount Everest

Edmund Hillary

Belgium

Lion

Netherlands

Lion

South Pole

Amundsen

Neil Armstrong

Canada

White Lily

New Zealand

Southern Cross, Kiwi, Fern

First person to set foot on the moon

North Pole

Robert Peary

Tasmania Island

Tasman

Chile

Candor and Huemul

Norway

Lion

China

Marco Polo

Cape of the Good Hope

Baurtho Romelodeis

France

Lily

Pakistan

Crescent

Germany

Corn Flower

Spain

Eagle

India

Lioned Capital

United Kingdom

Rose

Iran

Rose

USA

Golden Rod

National Monuments of Some Famous Countries Monument

Monument

Country

Emperial Palace (Tokyo) Japan

Country

Leaning Tower of Pisa

Italy

Eiffel Tower (Paris)

France

Pyramid (Giza)

Egypt

Great Wall of China

China

Opera House (Sydney)

Australia

Primary Name

Kremlin (Moscow)

Russia

Statue of Liberty (New York) USA

Bengal’s Sorrow

River Damodar

Kinder Disk

Denmark

Taj Mahal (Agra)

City of the Golden Gate

San Francisco (USA)

City of the Golden Temple

Amritsar

Sobriquets (Places and Persons) Places Sobriquets

India

Persons

City of Seven Hills/Eternal City

Rome

Sobriquet

City of Skyscrapers

New York (USA)

Adi Kavi

Valmeeki

Garden City

Bangaluru

Anna

C N Annadurai

Gateway of India

Bombay

Frontier Gandhi/Badshah Khan

Abdul Ghaffar Khan

Gift of the Nile

Egypt

Deenbandhu

C F Andrews

Holy Land

Palestine

Deshbandhu

C R Das

Island of Cloves

Zanzibar

Fuhrer

Adolf Hitler

Island of Pearls

Bahrain

Loknayak

Jaiprakash Narayan

Land of the Kangaroo

Australia

Kaviguru

Rabindranath Tagore

Land of the Golden Pagoda

Myanmar (Burma)

Lady with the Lamp

Florence Nightingale

Land of the Midnight Sun

Norway

Land of the Rising Sun

Japan

Land of Thousand Lakes

Finland

Land of White Elephants

Thailand

Pink City

Jaipur

Queen of the Adriatic

Venice (Italy)

Queen of the Arabian Sea

Kochi

Roof of the World

Pamer

Sorrow of China

Hwang Ho

Sugar Bowl of the World

Cuba

Venice of the East

Alappuzha

Venice of the North

Stockholm (Sweden)

White City

Belgrade

World’s Bread Basket

Prairies of North America

Name of Person

Lokamanya

Bal Gangadhar Tilak

Mahamana

Madan Mohan Malaviya

Mahatma

Gandhiji

Man of Blood and Iron

Bismark

Netaji

Subhash Chandra Bose

Panditji

Jawaharlal Nehru

Qaid-e-Azam

Mohmmed Ali Jinnah

Rajaji

C Rajgopalachari

Saint of the Gutters

Mother Teresa

Crematoriums of Famous Persons

Geographical Discoveries

Crematorium

Famous Person

Crematorium Famous Person

Raj Ghat

Mahatma Gandhi

Shanti Van

Vijay Ghat

Lal Bahadur Shastri

Shakti Sthal

Indira Gandhi

Kisan Ghat

Ch Charan Singh

Abhay Ghat

Morarji Desai Jagjivan Ram

Jawaharlal Nehru

Discovery

Discoverer

Discovery

Discoverer

Veer Bhumi

Rajiv Gandhi

Samata Sthal

America

Christopher Columbus

New Foundland

Gobot Sebastian

Ekta Sthal

Giani Zail Singh, Chandra Shekhar

Karma Bhumi Dr. Shankar Dayal Sharma

Sea Route to India via Cape of Good Hope

Vasco Da Gama

Hudson Bay

Henry Hudson

Uday Bhoomi

K R Narayana

Mahaprayan Ghat

https://sscstudy.com/

Dr. Rajendra Prasad

https://sscstudy.com/

6

CUET (UG) Section III : General Test

Largest, Longest and Highest in the World

The largest Fresh Water Lake

Kolleru Lake (Andhra Pradesh)

Speciality

Entity

The biggest River Islands

Majuli, Brahmaputra river (Asom)

Largest Archaepelago

Indonesia

The largest Lake

Wular lake (Jammu and Kashmir)

Largest Bird

Ostrich

The highest Dam

Tehari Dam (Uttarkhand)

Smallest Bird

Humming bird

The highest Waterfall

Longest Canal

Suez Canal

Kunchikal Falls (Karnataka) (455 m, 1493 ft) Bhagirathi and Alaknanda

Highest Capital of the World

Lapaz (Bolivia)

The deepest River Valley

Largest City (Area)

Mount Isa (Australia)

The longest River Bridge

Bhupen Hazarika Setu (Assam)

Largest City (Population)

Tokyo (Japan)

The biggest Conti Lover Bridge

Rabindra Setu or Howrah Bridge (Kolkata)

Largest Continent

Asia

Smallest Continent

Australia

The state with longest Coastline

Gujarat (1600 km)

Largest Country (Area)

Russia

The longest river without Delta

Narmada and Tapti

Smallest Country (Area)

Vetican City

The longest Sea Bridge

Largest Country (Population)

China

Anna Indira Gandhi Bridge (Tamil Nadu) or Pamban Bridge (2.3 km)

Largest Animal

Blue Whale

The largest Artificial Lake

Govind Sagar Bhakhra Nangal (HP)

Largest Desert

Sahara (Africa)

Godavari (1465 km)

Largest Delta

Sunderban (Ganga-Brahmaputra Delta)

The longest River of Southern India The longest Railway Platform

Gorakhpur, 1.3 km (Uttar Pradesh) Grand Trunk Road (Kolkata to Delhi)

Largest Dam

Grand Coolie (USA)

The longest Road

Highest Dam

Zingping-I (China)

The longest Corridor

Largest irrigation project

Lloyd Barrage (Pakistan)

Corridor of Ramnathswami Temple at Rameshwaram ( Tamil Nadu)

Largest Island

Greenland

The highest Road

Largest Sea

Mediterranean Sea

Road at Umling La (in East Ladakh Sector)

Deepest Lake

Baikal

The highest Airport

Leh Airport (Ladakh)

Highest Lake

Titikaka

The largest Desert

Thar (Rajasthan)

Largest Lake

Caspian Sea

Largest Freshwater Lake

Superior Lake (USA & Canada)

Largest Saline Lake

Caspian Sea

The largest Delta

Sunderbans (Paschim Banga)

The state with maximum Forest Area

Madhya Pradesh (11.24%)

The largest Zoo

Arignar Anna Zoological Garden (Tamil Nadu)

The biggest Stadium

Sadar Patel Stadium (Ahemdabad)

The longest National Highway

NH-44 ( Srinagar to Kanyakumari)

The highest Award

Bharat Ratna

Green Land National Park (Denmark)

The highest Gallantry Award

Param Vir Chakra

Coldest Place

Verkhoyansk (Siberia)

The largest Gurudwara

Golden Temple, Amritsar

Hottest Place

Death Velly, (USA)

The largest Cave Temple

Kailash Temple (Ellora, Maharashtra)

Longest Platform

Gorakhpur (UP, India)

The highest Peak

Godwin Austin I, K 2 (8611 m)

Longest Railwayline

Trans-Siberian Railway

The largest Mosque

Jama Masjid (Delhi)

Largest River

Amazon (South America)

The longest Tunnel

Longest River

Nile (Africa)

Chennani-Nashri Tunnel (Jammu and Kashmir)

The largest Auditorium

Sri Shanmukhanand Hall (Mumbai)

Highest Mountain Peak

Mount Everest (8850 m)

Largest Mountain Range

Andes (South America)

Largest Museum

Louvre Museum (Paris)

Largest Ocean

Pacific Ocean

Largest Park

Superlatives (India) (Biggest, Highest, Largest, Longest, Smallest etc.) The longest River

The Ganga (2525 km)

The longest Canal

Indira Gandhi Canal or Rajasthan Canal (Rajasthan) (649 km)

The longest Dam

Hirakud Dam (Odisha) (26 km)

The longest Sea Beach

Marina Beach (Chennai) (13 km)

The highest Lake

Devtal Lake (17745 ft), Garhwal (Uttarakhand)

The largest Saline Water Lake

Chilka Lake (Odisha)

The largest Animal Fair

Sonepur (Bihar)

The largest Cave

Amarnath (Jammu and Kashmir)

The highest Gate Way

Buland Darwaza, Fatehpur Sikri (Uttar Pradesh)

The tallest Statue

Statue of Unity (Gujarat)

The largest Public Sector Bank

State Bank of India

The most Populous City

Mumbai (Maharashtra)

The biggest Church

Saint Cathedral at Old Goa (Goa)

The highest Battlefield

Siachen Glacier (5753 m)

https://sscstudy.com/

https://sscstudy.com/

GENERAL KNOWLEDGE

First in India Newspaper

Bengal Gazette (James Hickey)

Vernacular Daily

Samachar Darpan

Hindi Newspaper

Udant Martand

Telegraph Line

Diamond Harbour to Kolkata

International Telephone Service

Mumbai to London (1851)

Silent Movie

Raja Harish Chandra (Dadasaheb Phalke 1913)

First Cricketer to get Padma Bhushan First Indian to get through ICS

CK Naidu Satyendra Nath Tagore (1869)

First Indian to swim across the English Channel First Indian Cricketer to get Bharat Ratna

Mihir Sen (1958) Sachin Tendulkar

First in India (Female) First Indian Female Chairperson of Indian National Congress

Sarojini Naidu (1925)

Talkie Movie

Alam Ara (Ardeshir Irani-1931)

Aircraft Carriage Warship

INS Vikrant

First Woman Cabinet Minister

Rajkumari Amrit Kaur (1947)

Satellite

Aryabhatta (19th April, 1975)

Vijaya Laxmi Pandit

Satellite dedicated exclusively for Education purposes

EDUSAT

First Woman Chairman of the UN General Assembly First Woman President of India

Pratibha Devi Singh Patil

Successful indigenous launch vehicle

SLV-3

First Woman Speaker of Lok Sabha

Meira Kumar (2009)

Nuclear Reactor

Apsara

Margaret Alva (1962)

Lunar Mission

Chandrayaan-I (October, 2008)

First Woman Deputy Chairman of Rajya Sabha

Hydroelectric Project

Sivasamudram (1902)

First Woman Prime Minister of India

Indira Gandhi

Asian Games

Delhi (1951)

First Woman to reach Antarctica

Meher Moos (1976)

Census

1872

First Woman IAS Officer

Anna Rajam George (1950)

Regular DEcadal Census

1881 Onwards

First Female Chief Justice

Leela Seth (1991)

Biosphere Reserve

Nilgire

First Woman to win the Jnanpith Award

Asha Poorna Devi (1976)

National

Haley National Park (Jim Corbett), 1936

First Woman to get the Bharat Ratna

Indira Gandhi

First Female Nobel Prize Winner

Mother Teresa (1979)

Chairman of UPSC

Ross Barker Ahmedabad

First woman to complete century in World Cup Cricket

Thirush Kamini

First E-court Court exclusively dedicated to women

Malda (WB) Technopark, Thiruvananthapuram

First actress to be nominated to the Rajya Sabha First Woman Chairperson of UPSC

Nargis Dutt

Technology Park

First Foriegn Minister

Sushma Swaraj

First Olympic Winner

P.T. Usha

First in India (Male) First Governor-General of India Last Governor-General of India First and last Indian Governor-General of Free India First Commander-in-Chief of Free India First Field Marshal of India

William Bentinck (1828) Lord Mountbatten C Rajgopalachari

First Indian to go in Space First Indian to climb the Mount Everest without Oxygen First Indian to become the Managing Director of World Bank First Chairman of National Human Rights Commission

Rakesh Sharma Sherpa Ang Dorje

First Indian to get Nobel Prize in Physics First Indian to get Nobel Prize in Literature

CV Raman (1930) Rabindranath Tagore (1913) Dr Amartya Sen Dr Har Govind Khorana

First Indian to get Nobel Prize in Economics First Indian to get Nobel Prize in Medicines (Physiology) First Indian to get Bharat Ratna

First Person to be Honoured with the Jnanpith Award First Person to get Bharat Ratna (Posthumously)

7

Rose Millian Bathew (Kharbuli)

First Female Pilot of Indian Navy

Shivangi Singh

First Female Excavation Engineer

Shivani Meena

Important Books and Authors General KM Kariappa General SHFJ Manekshaw (1971)

Gautam Kaji Rangnath Mishra

Book Panchatantra Kadambari Sursagar Rajtarangini

Author Vishnu Sharma Banabhatta Surdas Kalhana

Buddhacharita Shahnama Mudrarakshasha Arthashashtra

Uttarramcharita

Bhavbhuti

Padmavat

Geetanjali, Gora

Bharat-Bharti

Dr S Radhakrishnan, C Rajgopalachari and Dr CV Raman

Ravindranath Tagore Anamika, Parimal Suryakant Tripathi Nirala Kurukshetra, Ramdhari Urvashi Singh ‘Dinkar’ Chandrakanta Devki Nandan Khatri Mother Maxim Gorki War and Peace Leo Tolstoy

G Sankara Kurup (Malayalam)

Ignited Minds

Long Walk to Freedom Satanik Verses

Lal Bahadur Shastri Half a life

https://sscstudy.com/

A P J Abdul Kalam V S Naipaul

Chidambara Malgudi Days, Guide Devadasa, Charitrahin Mein Kemph The Insider

Ashvaghosha Firdausi Vishakhadatta Chanakya (Kautilya) Mallik Mohammad Jayasi Maithilisharan Gupta Sumitranandan Pant R K Narayan Sharatchandra Chattopadhyay Adolf Hitler P V Narsimha Rao Nelson Mandela Salman Rushdie

https://sscstudy.com/

8

CUET (UG) Section III : General Test

United Nations Organisation (UNO) Ä

Ä

Ä

Ä Ä

Ä

The United Nations (UN) is a world organisation formed in 24th October, 1945. It came into existence after World War II, when the leaders of the world, including American President Roosevelt and British Prime Minister Churchill, decided to create a world organisation that would help to ensure peace. The original membership of 51 nations has grown to 193 members. The 193rd member being the newly created South Sudan. The United Nations Headquarters is in New York City. The UN also has offices in Nairobi (Kenya), Geneva (Switzerland) and Vienna (Austria). Six official languages are spoken and used in documents at the United Nations: Arabic, Chinese, English, French, Russian and Spanish, but the working languages are English and French only. The General Assembly is the main place for discussions and policy making in the United Nations. The Security Council has primary responsibility for the maintenance of international peace and security. The Security Council is made up of 15 members. There are five permanent members of the Secutiry Council-China, France, Russia, United Kingdom and USA and 10 non-permanent members elected for 2 years terms starting on 1st January. UN and its Specialized Agencies Organization

Year of Establishment

Headquarters

United Nations (UN)

Oct 24, 1945

New York

International Atomic Energy Agency (IAEA)

July 29, 1957

Vienna (Austria)

Food and Agriculture Organization (FAO)

Oct 16, 1945

Rome (Italy)

United Nations Educational, Scientific and Cultural Organization (UNESCO)

Nov 4, 1946

Paris (France)

World Health Organization (WHO)

July 22, 1946

Geneva (Switzerland)

International Labour Organization (ILO)

April 11, 1919

Geneva (Switzerland)

International Finance Corporation (IFC)

July, 1956

Washington D C

International Civil Aviation Organization (ICAO)

Dec, 1944

Montreal (Canada)

Universal Postal Union (UPU)

July 1, 1875

Berne (Switzerland)

World Meteorological Organization ( WMO)

1951

Geneva (Switzerland)

International Maritime Organization (IMO)

March, 1948

London (Britain)

World Intellectual Property Organization ( WIPO)

1967

Geneva (Switzerland)

Some Other International Organizations Organization

Year of Headquarters Establishment

No. of Member States

Arab League

1945

Cairo (Egypt)

22

African Union

1963

Addis Ababa (Ethiopia)

53

Asia-Pacific Economic Cooperation (APEC)

1989

Singapore

21

Asian Development Bank (ADB)

1966

Manila (Philippines)

59

Association of South-East Asian Nations (ASEAN)

1967

Jakarta (Indonesia)

10

Commonwealth

1931

London (Britain)

54

European Union (EU)

1991

Brussels (Belgium)

27

Commonwealth of Independent States (CIS)

1991

Kirava (Belarus)

12

G-8 (Group of Eight)

1975



8

INTERPOL

1923

Lyon (France)

184

North Atlantic Treaty Organization (NATO)

1949

Brussels (Belgium)

30

Organization of Petroleum Exporting Countries (OPEC)

1960

Vienna (Austria)

13

Non-Aligned Movement (NAM)

1961

(First Conference at Belgrade) Secretariat at Kathmandu (Nepal)

South Asian Association for Regional Cooperation (SAARC)

1985

Kathmandu (Nepal)

https://sscstudy.com/

120 8

https://sscstudy.com/

GENERAL KNOWLEDGE

Indian Defence Ä

Ä

Ä

Ranks of Commissioned Officers

The President of India is the supreme commander of the Indian Defence System.

Army

Air Force

Navy

General

Air Chief Marshal

Admiral

The whole administrative control of the Armed forces lies in the Ministry of Defence.

Lt. General

Air Marshal

Vice Admiral

Major General

Air Vice Marshal

Rear Admiral

Indian Defence System has been divided into three services-Army, Navy and Air Force.

Brigadier

Air Commodore

Commodore

Colonel

Group Captain

Captain

Lt. Colonel

Wing Commander

Commander

Major

Squadron Leader

Lt Commander

Captain

Flt Lieutenant

Lieutenant

Lieutenant

Flying Officer

Sub-Lieutenant

Indian Army Ä

The Indian Army is organized into seven commands Command

Headquarters

Western Command

Chandigarh

Indian Defence Training Institutions

Kolkata

Training Institution

Place

Estd. in

Northern Command

56 APO

Rashtriya Indian Military College (RIMC)

Dehradun

1922

Southern Command

Pune

Army Cadet College (ACC)

Dehradun

1929

Indian Military Academy (IMA)

Dehradun

1932

National Defence Academy (NDA)

Khadakwasla, Pune 1941

Eastern Command

Central Command

Lucknow

Army Training Command

Shimla

South Western Command

Jaipur

Indian Air Force Ä

It is organized into seven commands Command

Headquarters

Western Command

New Delhi

Central Command

Allahabad

Eastern Command

Shillong

South Western Command

Gandhi Nagar

Training Command

Bangaluru

Maintenance Command

Nagpur

Southern Command

Thiruvananthapuram

High Altitude Warfare School (HAWS)

Gulmarg

1948

National Defence College (NDC)

New Delhi

1960

Officers Training Academy (OTA)

Chennai

1963

Counter Insurgency and Jungle Warfare School

Vairengte (Mizoram)

1970

College of Defence Management

Secunderabad (Andhra Pradesh)

1970

College of Combat/Army War College

Mhow (Madhya Pradesh)

1971

Army School of Physical Training (ASPT)

Pune

1978

Army Air Defence College (AADC)

Gopalpur (Odisha)

1989

Indian Para-Military Forces Forces

Indian Navy Ä

9

It is organized into three commands

Estd. in

Indo-Tibetan Border Police

1962

National Security Guards

1984

Central Industrial Security Force

1969

Assam Rifles

1835

Command

Headquarters

Border Security Force

1965

Eastern Command

Vishakhapatnam

Central Reserve Police Force

1939

Southern Command

Cochin

National Cadet Corps

1948

Mumbai

Intelligence Bureau

1920

Central Bureau of Investigation

1953

Western Command Ä

Each command is headed by Vice Admiral.

Ä

The Navy is headed by the “Chief of the Naval Staff” of the rank of Admiral.

Defence Research and Development Organisation (DRDO)

Ä

Each command is headed by Vice Admiral.

Ä

INS Shakti was India’s first indigenously built submarine.

It was established in 1958 to provide a solid base to the national security system.

Ä

INS Kadamba is India’s largest naval base situated at Karwar, Karnataka.

Ä

It was commissioned in 2005 under the project ‘Seabird’.

Prithvi Surface to surface missile with a short range of 150 km with 1000 kg warhead and 250 km with 500 kg warhead. (Prithvi-I Army version, Prithvi-II-Air Version, Prithvi-III-Naval.)

https://sscstudy.com/

https://sscstudy.com/

10

CUET (UG) Section III : General Test

Agni Ä

Ä

Surface to surface intermediate range ballistic missile (IRBM) with a range of 1000 km-2500 km. Angi V- Surface to surface Intercontinental Ballistic Missile (ICBM) with a range of 5000-6000 km.

Awards, Honours and Prizes Nobel Prize Ä

Ä

Akash Surface to surface missile with a range of 25 km.

Ä

Nag Third generation ‘fire and forget’ anti-tank missile with a range of 4 km.

Ä

3Trishul Surface to air missile with a range of 500m to 9 km. It is being developed for all the three services.

It was instituted by the inventor of dynamite, Alfred Bernard Nobel (1833-96). The award is given on December 10, which is the death anniversary of its founder. Nobel Prize is given every year to those eminent persons who have made pioneering achievements in the field of Physics, Chemistry, Medicine, Peace, Literature and Economics. Apart from Economics, all other categories have been given since 1901. Economics Nobel Prize was instituted in 1967 and was first given in 1969. Indian Nobel Prize Winners

MBT-Arjun

Name

Field

India’s Main Battle Tank (MBT) has been designed and developed by DRDO. It has Global Positioning System (GPS) to facilitate to find its position.

Rabindra Nath Tagore

Literature

1913

Dr. CV Raman

Physics

1930

Dr. Hargovind Khurana

Medicine

1968

Mother Teresa

Peace

1979

Dr. S Chandrashekhar

Physics

1983

Lakshya It is the Pilotless Target Aircraft (PTA).

Year

Dr. Amartya Sen

Economics

1999

Nishant

VS Naipaul

Literature

2001

It is the Remotely Piloted Vehicle (RPV) for the survelliance purpose.

Venkataraman Ramakrishnan

Chemistry

2009

Kailash Satyarthi

Peace

2014

Tejas

Abhijeet Banerjee

Economics

2019

Smallest, light weight, single-engine, single-seat, multi-role, fourth generation combat aircraft—the Light Combat Aircraft (LCA).

Pulitzer Prize Ä

It was instituted in 1917 and named after the US Publisher Joseph Pulitzer. It is conferred annually in the USA for accomplishments in journalism, literature and music.

Pinaca

Ä

It is a lethal ground based multibarrel Rocket Launcher weapon system. It has a range of 39 km and has a capability to fire up to 12 rockets within seconds.

Magsaysay Awards Ä

Brahmos It is a cruise missile jointly developed by India and Russia.

Ä

India’s Atomic Research Ä

Ä

Ä

India’s journey to atomic energy research started with the establishment of the Atomic Energy Commission on August 10, 1948, under the chairmanship of Dr Homi J Bhabha. Subsequently, the Department of Atomic Energy (DAE) was established in 1954 for implementation of atomic energy programmes. Bhabha Atomic Research Centre (BARC) was established in 1957 at Trombay (Maharashtra). It is India’s largest atomic research centre. Apsara (India’s first atomic reactor), Cirus, Zerlina, Dhruva, Purnima I and II and Kamini (India's first fast breeder nuclear reactor, at Kalpakkam) are the BARC’S atomic reactors.

Ä

They were instituted in 1957 and named after Ramon Magsaysay, the late President of Philippines. This award is given annually on August 31, for outstanding contributions to public service, community leadership, journalism, literature and creative arts and international understanding. First Indian to get this award was Vinobha Bhave.

Man Booker Prize Ä

Ä

Ä

It is the highest literary award given to the authors of British, Irish and Commonwealth countries. It was instituted in 1968 by the Booker Company and the British Publishers Association along the lines of Pulitzer Prize of US. Booker Prize has been renamed as Man Booker Prize.

Oscar Awards Ä

These awards were instituted in 1929 and conferred annually by the Academy of Motion Pictures in USA.

https://sscstudy.com/

https://sscstudy.com/

GENERAL KNOWLEDGE

Ä

Ä

These are considered the most prestigious awards in the cinema world. The first Indian to get an Oscar was Bhanu Athaiya for the movie ‘Gandhi’. Satyajit Ray was awarded Oscar for lifetime achievements in cinema in 1992.

Gallantry Awards Param Vir Chakra is the highest decoration of valour award. It is the most conspicuous act of bravery or some act of valour or self-sacrifice in the presence of the enemy, whether on land, at sea or in the air. The medal is made of bronze. Mahavir Chakra is the second highest gallantry award for acts of conspicuous gallantry in the presence of the enemy whether on land, at sea or in the air. The medal is made of standard silver. Vir Chakra is awarded for acts of gallantry in the presence of enemy, whether on land, at sea or in the air. The medal is made of standard silver.

Bharat Ratna Ä

Ä

Ä

Ä

Ä

It is the highest civilian award of India. It is presented by the Government of India. It is presented for exceptional public service and rarest achievements in the field of art, literature, sport and science. It was instituted in 1954 and the first recepient were C Rajagopalachari, Dr Radhakrishnan and CV Raman. Padma Vibhushan is the second highest civilian award for distinguished services in any field including Government service. Padma Bhushan and Padma Shree are the other important civilian awards.

Ashok Chakra This is awarded for valour, courageous action or sacrifice, away from the battlefield. It is highest military award during peacetime. Kirti Chakra The decoration is awarded for conspicuous gallantry. It is made of standard silver and is circular in shape. The obverse and the reverse are exactly the same as in Ashoka Chakra. Shaurya Chakra The decoration is awarded for an act of gallantry during peacetime.

Bharatiya Jnanpith Award Ä

Ä

Sahitya Akademy Awards It was instituted in 1955 and is given for any exclusive writing in any of the 22 languages including English literature during last 5 years. These awards are given to the Indian scientists for their exceptional performance.

Ä

Arjuna Awards Ä

Ä

These were instituted in 1961 and given by Sports Ministry, Government of India. These are given for the special achievements in different types of sports.

Ä

These were instituted in 1985 and given by Sports Ministry, Government of India. These are given to sports coaches.

Ä

Ä

Rajiv Gandhi Khel Ratna (Major Dhayan Chand Khel Ratna Award) Ä

Ä

Ä

Ä

The National Film Awards are the most prominent film award ceremony in India, established in 1954 and it is administered, along with the international film festival of India and the Indian Panorama, by the Indian government’s Directorate of Film Festivals since 1973. Due to the national scale of the National Film Awards, it is considered to be the equivalent of the American Academy Awards.

Filmfare Awards

Rajiv Gandhi Khel Ratna Award formerly known as Major Dhyan Chand Khel Ratna Award is the highest sporting honour of India. This award was instituted in 1991-92. It is awarded annually by the ministry of Youth Affairs and Sport.

Ä

Ä

Dadasaheb Phalke Award Ä

Established in 2nd October, 1994, on the occasion of the 125th birthday anniversary of Mahatma Gandhi, carries a cash prize of ` 1 crore. Indian Government instituted this annual prize to encourage and promote the significance of Gandhian values over the world.

National Film Awards

Dronacharya Awards Ä

Instituted in 22nd May, 1961, carries a cash prize of ` 5 lakh, a citation and a bronze replica of Vagdevi (Saraswati). This award is given for the best literary writing by an Indian citizen in a language listed in Eighth Schedule of the Indian Constitution.

Gandhi Peace Prize Ä

Shanti Swaroop Bhatnagar Awards

11

Dadasaheb Phalke is known as the Father of Indian Cinema. The highest National Film Award is named after him in 1969. Mrs Devika Rani Roerich was the first person to receive Dadasaheb Phalke Award in 1969. It was instituted in 1962 and is presented for commendable display by the players.

The filmfare awards are presented annually by the times group to honour both artistic and technical excellence of professionals in the Hindi language film industry of India. They were initially referred to as the Clare Awards after the editor of the Times of India, Clare Mendonca.

Sangeet Natak Akademi Puraskar (Akademi Award) Ä

Ä

Awarded by the Sangeet Natak Akademi India’s National Academy of Music, Dance and Drama. It is the highest Indian recognition given to practicing artists in the categories of music, dance, theatre, other traditional/folk/tribal/dance/music/theatre and Puppetry and contribution/scholarship in performing arts.

https://sscstudy.com/

https://sscstudy.com/

12

CUET (UG) Section III : General Test

Musical Instruments and Instrumentalists

Lalit Kala Akademi Ratna Ä

Ä

Instituted in 1955 by the Government of India is an honour for the fine arts given to eminent artists for their lifetime achievements in the field of visual arts. It is awarded by the Lalit Kala Akademi, India’s National Academy of Art, it is the highest honour in the fine arts conferred by the Government of India. Classical Dancers of India

Instruments

Instrumentalists

Stringed Instruments Been

Asad Ali Khan, Zia Moin-ud-din Khan

Santoor

Shiv Kumar Sharma

Sarod

Buddhadev Dasgupta, Ali Akbar Khan, Amjad Ali khan, Bahadur Khan, Sharan Rani, Zarin S Sharma

Sarangi

Ustad Binda Khan

Dance

Dancer

Sitar

Bharatnatyam

Bala Saraswati, CV Chandrasekhar, Leela Samson, Mrinalini Sarabhai, Padma Subramanyam, Rukmini Devi, Sanyukta Panigrahi, Sonal Mansingh, Yamini Krishnamurti

Ravi Shankar, Hara Shankar Bhattacharya, Nikhil, Banerjee, Vilayat Khan, Mustaq Ali Khan

Surb Ahar

Sajjad Hussain, Annapurna

Veena

Doraiswamy Iyengar, Chittibabu, Emani Sankara Shastri, Dhanammal, S Bala Chandran, KR Kumaraswamy

Violin

Gajanan Rao Joshi, MS Gopal Krishnan, TN Krishnan, Baluswamy, Dikshitar, Dwaran Venkataswamy Naidu Lalyuli G Jayaraman, Mysore T Chowdiah, VG Jog

Kathak

Bharti Gupta, Birju Maharaj, Damayanti Joshi, Durga Das, Gopi Krishna, Kumudini Lakhia, Sambhu Maharaj, Sitara Devi

Kuchipudi

Josyula Seetharamaiah, Vempathi Chinna Sathyam

Manipuri

Guru Bipin Sinha, Jhaveri Sisters, Nayana Jhaveri, Nirmala Mehta, Savita Mehta

Odissi

Debaprasad Das, Dhirendra Nath Patnaik, Indrani Rahman, Kelucharan Mahapatra, Priyambada Mohanty

Kathakali

Mohiniattam

Mrinalini Sarabhai, Guru Shankaran, Namboodripad, Thottam Shankaran, Kutti Nayyar, Shankar Kurup, KC Pannikar, TT Ram Kulti Protima Devi, Sanyukta Panigrahi, Sonal Mansingh, Pankaj Charan Das, Kelucharan Mahapatra, Madhvi Mudgal, etc.

State and Folk Dances

Wind Instruments Flute

TR Mahalingam, N Ramani, Hari Prasad Chaurasia, Pannalal Ghosh

Nadaswaran

Sheikh Chinna Moula, Neeruswamy Pillai, Rajaratanam Pillai

Shehnai

Bismillah Khan

Percussion (Strikting Thumping) Instruments Mridangam

Palghat Mani Iyer, Karaikudi R Mani, Palghat Raghu

Pakhawag

Pt Ayodhya Prasad, Gopal Das, Babu Ram Shanker Pagaldas

Andhra Pradesh

Kuchipudi, Ghantamardala, Ottam Thedal

Asom

Bihu, Bichhua, Natpuja, Maharas, Kaligopal, Bagurumba, Naga dance, Khel Gopal, Tabal Chongli, Canoe, Jhumura Hobjanai

Tabla

Zakir Hussain, Nikhil Ghosh, Kishan Maharaj, Alla Rakha Khan, Pandit Samta Prasad, Kumar Bose, Latif Khan

Bihar

Jata-Jatin, Bakho-Bakhain, Panwariya, Sama-Chakwa, Bidesia

Kanjira

Pudukkotai Dakshinamurthi Pillai

Gujarat

Garba, Dandiya Ras, Tippani Juriun, Bhavai

Haryana

Jhumar, Phag, Daph, Dhamal, Loor, Gugga, Khor, Gagor

Himachal Pradesh

Jhora, Jhali, Chharhi, Dhaman, Chhapeli, Mahasu, Nati, Dangi

Anthropological Survey of India, 1945

Kolkata

¡

Archaeological Survey of India, 1861

New Delhi

¡

Asiatic Society, 1784 (Sir William Jones)

Kolkata

¡

Indira Gandhi National Centre for Arts, 1985

New Delhi

¡

Lalit Kala Akademi (National Academy of Fine Arts), 1954

New Delhi

¡

National Archives of India, 1981

New Delhi

¡

National School of Drama, 1959

New Delhi

Odissi (Classical), Savari, Ghumara, Painka, Munari, Chhau

¡

Sahitya Academy, 1954

New Delhi

¡

Sangeet Natak Academy, 1953

New Delhi

Kathi, Gambhira, Dhali, Jatra, Baul, Marasia, Mahal, Keertan etc

¡

Library of Tibetan Works and Archives

Dharmashala

¡

Science City

Kolkata

¡

Victorial Memorial Hall

Kolkata

¡

Birla Industrial and Tech Museum

Kolkata Leh

Rauf, Hikat, Mandjas, Kud Dandi Nach, Damali

Karnataka

Yakshagan, Huttari, Suggi, Kunitha, Karga, Lambi

Kerala

Kathakali (Classical), Ottam Thulal, Mohiniattam, Kaikottikali

Maharashtra

Lavani, Nakata, Koli, Lezim, Gafa, Dahikala Dasavtar or Bohada

Paschim Banga

Headquarters

¡

Jammu & Kashmir

Odisha

Cultural Institutions Institutions

Punjab

Bhangra, Giddha, Daff, Dhaman etc

Rajasthan

Ghumar, Chakri, Ganagor, Jhulan Leela, Jhuma, Suisini, Ghapal, Kalbeliya

Tamil Nadu

Bharatnatyam, Kumi, Kolattam, Kavadi

¡

Central Institute of Buddhist Studies

Uttar Pradesh

Nautanki, Raslila, Kajri, Jhora, Chappeli, Jaita

¡

Nava Nalanda Mahavihara

Nalanda (Birla)

Uttarakhand

Garhwali, Kumayuni, Kajari, Jhora, Raslila, Chappeli

¡

National Gallery of Modern Art

New Delhi

https://sscstudy.com/

https://sscstudy.com/

GENERAL KNOWLEDGE

Cricket World Cup

Sports Olympics Ä

Ä

Ä

Ä

Ä

Ä

Ä

Ä

Olympics games were started in 776 BC on Mount Olympia in the honour of Greek God ‘Zeus’. The modern Olympic Games were started in Athens the capital of Greece on 6th April, 1896 with great efforts made by French nobleman, Baron Pierre de Coubertin. The Olympic Games are organised after every four years. The Olympic Flag is made up of White Silk and contains five interwined rings as the Olympic Emblem. The five interlaced rings are arranged in 3-2 pattern on a white background, with the blue ring to the extreme left, followed by yellow, black, green and red, in the same order. Blue for Europe, Black for Africa, Red for Americas (North and South America), Yellow for Asia and Green for Oceania (Australia and New Zealand). The official Olympic Motto is ‘‘Citius, Altius, Fortius’’, a Latin phrase meaning Swifter, Higher, Stronger. The Head Office of International Olympic Committee (IOC) is at Lausanne (Switzerland).

Commonwealth Games Ä

Ä

Ä

Ä

Ä

Ä

The first Commonwealth Games were held in 1930 in Hamilton, Canada. Since 1930, the games have been conducted every four years except for 1942 and 1946 due to World War II. The Commonwealth Games Federation (CGF) is the organization which is responsible for the direction and control of the Commonwealth Games. There are currently 54 members of the Commonwealth of Nations and 71 teams participated in the games. 21st Commonwealth Games of 2018 were held at Goldcoast city, Queens land (Australia). 22nd commonwealth Games of 2022 will be held at Berminghom (England)

The Asian Games Ä

Ä

Ä

Ä

13

The Asian Games, also called the Asiad, are a multi-sport event held every four years among athletes from all over Asia. The games are regulated by the Olympic Council of Asia (OCA), under the supervision of the International Olympic Committee (IOC). 18th Asian Games of 2018 were held at Jakarta (Indonesia). 19th Asian Games of 2022 will be held in Hangzhou, China.

The first Cricket World Cup was organised in England in 1975. A separate women’s Cricket World Cup has been held every four years since 1973.

Hockey World Cup The first Hockey World Cup was organised in Barcelona (Spain) in 1971. Women’s Hockey World Cup has been held since 1974. The 14th Hockey World Cup were be held in the Bhubaneswar, Odisha (India) in 2018. 15th Hockey World Cup (Men’s) will be held in Odisha (India) in 2023.

Football World Cup Ä

Ä

Ä

The Football World Cup is organised by FIFA (Federation of International Football Association). The World Cup is called ‘Jules Rimet Cup’ named after the name of FIFA President Jules Rimet. The first Football World Cup was organised in Uruguay in 1930. In 1942 and 1946, the Football World Cup was not played due to World War II. Brazil is the only nation to have participated in every World Cup so far. The 2014, 2018 and 2022 Football World Cup scheduled to be held at Brazil, Russia and Qatar respectively. Important Sport Terms

Sport

Terms

Basketball

Dunk, Front court, Lay up, Held ball, Pivot, Rebound, Steal

Cricket

Bye, Draw, Googly, Topspin, Over throw, Duck, Hit wicket

Football

Bend dribble, Dissent, Dummy, Feint, Free kick, Header, Red card, Throwins

Hockey

Bully, Striking, Circle, Post back

Chess

Castle, Diagonaes, Files, Pawns, Peices, Promote, Gambit, Pawn

Boxing

Jab, laying on Knock, Second out habbit punch, Upper cut

Badminton

Loab, Let, Drive, Drop, Love

Polo

Chuker, Bunker

Baseball

Diamond, Home run, Put out, Strike, Ant-rubber.

Rifle Shooting

Target, Muzzle fulb, Bulls eye

Wrestling

Half, Nelson, Hold Sager, Rebuts

Golf

Fore some, Stymie, T, put hole, Caddy, Nib lick, Iron, The green, Bunkeer

Billiards

Jigger, Pot, Break pot, In off, Cans, Bolting, Long, Hazard, Cue

Swimming

Breast Stroke, Twist, Butterfly, Crawl, Spring Board

Volley Ball

Antennae, Attack hit, Libero, Service, Set up, Blocking, Dribbling

Lawn Tennis

Advantage, Ace, Dence, Volley, Foot Foult, Smash, Grand-Slam, Slice, Love

Table Tennis

End line, Flat hit, Foil, Service, Phnholder grip, Reverse, Top-spin, Couter-hitting, Let

https://sscstudy.com/

https://sscstudy.com/

14

CUET (UG) Section III : General Test

Nickname of Players

Number of Players (in each team)

Sports

Player

Nickname

Players

Nickname

Major Dhyanchand

Hockey ka Jadugar

Rahul Dravid

The wall

Milkha Singh

Flying Sikh

Harbhajan Singh

Bhajji, The Turbanator

PT Usha

Payyoli Express, Golden Girl

Javagal Srinath

Mysore Express

Shoaib Aktar

Rawalpindi Express

Paes and Bhupati Indian Express

Golf

Several individuals compete simultaneously

Badminton, Tennis and Table Tennis

1 or 2 (Singles and Doubles respectively)

Gymnastic

Several individuals compete simultaneously

Important Books of Ancient India Books

Important Cups and Trophies

Authors

Sport

Cup and Trophy

Mudrarakshas

Vishakha Dutta

Cricket

Irani Trophy, Duleep Trophy, Ranji Trophy, Vizzy Trophy, Asia Cup, Deodhar Trophy, CK Naidu Trophy, Cooch-Behar Trophy

Mahabhasya

Patanjali

Ashtadhyayi

Panini

Vikramankadevacharita

Bilhan

Ramcharit

Sandhyakar Nandi

Football

Durand Cup, Nizam Gold Cup, Rovers Cup, Sanjay Gold Cup, Santosh Trophy, Subroto Mukherjee Cup, Vittal Trophy, Nehru Gold Cup

Prabandha Kosh

Rajashekhara

Agha Khan Cup, Azlan Shah Cup, Nehru Trophy, Dhyanchand Trophy, Beighton Cup, Scindia Gold Cup, Modi Gold Cup, Indira Gandhi Gold Cup, Rangaswami Cup, Khan Abdul Gaffar Cup.

Arthashastra

Chanakya (Kautilya)

Nitishastra

Sukra

Brihat-Katha Kosha

Harisena

Canada Cup, Muthian Gold Cup, Ryder Cup, Walker Cup

Kamasutra

Vatsayana

Svapna-Vasavdatta

Bhasa

Lawn Tennis Davis Cup, Hamlet Cup, Australian Open, French Open, Wimbledon, US open

Buddacharita

Asvaghosh

Fu-Kuo-Ki

Fa-hien

Badminton

Thomas Cup (men), Uber Cup (women), Narang Cup

Kiratarjuniyam

Bharavi

Boxing

Aspy Adjania Trophy

Panchatantra

Vishnusharma

Rowing

Wellington Trophy

Surya Siddhanta

Aryabhatta

Bridge

Ruia Trophy

Pancha Siddhantika

Varahamihir

Polo

Ezra Cup, Winchestor Cup, Radha Mohan Cup

Bagvatgita

Ved Vyas

Gita-Govinda

Jayadeva

Country

National Sport Country

National Sport

Australia

Cricket

India

Hockey

Canada

Lacrose

Japan

Sumo

Organisation

Place

Founder

Year

Calcutta

Raja Rammohan Roy

1815

Hockey

Golf

Table Tennis Corbillion Cup (women), Jayalaxmi Cup (women), Swaythling Cup (men)

Important National Sports

Important Socio-Religious Organisations

China

Table Tennis

Malaysia

Badminton

Atmiya Sabha

England

Cricket

Scotland

Rugby/Football

Brahmo Samaj

Calcutta

Raja Rammohan Roy

1828

Spain

Bull Fighting

USA

Baseball

Brahmo Samaj of India

Calcutta

Keshab Chandra Sen

1866

Sadharan Brahmo Samaj

Calcutta

Anand Mohan Bose & Shivnath Shastri

1878

Dharma Sabha

Calcutta

Radhakant Deb

1829

Tattvabodhini Sabha

Calcutta

Devendranath Tagore

1839

Paramhansa Mandali

Bombay

Dadoba Pandurang, Gopalhari Deshmukh

1849

Radhaswami Movement

Agra

Tulsi Ram (Shiv Dayal Saheb)

1861

Number of Players in Some Population Sports Sports

Number of Players (in each team)

Hockey, Football and Cricket

11

Water Polo

7

Kho Kho

9

Babaddi

7

Billiards/Snooker

1

Boxing/Chess

1

Bridge

2

Prarthana Samaj

Bombay

Atmaram Pandurang

1867

Netball

7

Arya Samaj

Bombay

Dayanand Saraswati

1875

Volleyball

6

New York

5

Madan H P Blavatsky & Colonel H S Olcott

1875

Basketball

Theosophical Society

https://sscstudy.com/

https://sscstudy.com/

GENERAL KNOWLEDGE

Organisation

Place

Founder

Year

Deccan Education Society

Poona

G G Agarkar

1884

Indian National Social Conference

Bombay

M G Ranade

1887

Dev Samaj

Lahore

Shivnarayan Agnihotri

1887

Ramkrishna Mission

Belur

Swami Vivekanand

1897

Servant of India Society

Bombay

Social Service League

Bombay

G K Gokhale

Major Rivers of the World

1905

N M Joshi

1911

River

Origin

Nile

Victoria lake

Amazon

Andes (Peru)

Yangtze

Tibetan Kiang Plateau

Mississippi Missouri

Itaska Lake (USA)

Yenisei

Tannu-Ola Mountains

Huang Ho

Kunlun Mountains

Ob

Altai Mountains, Russia

Congo

Lualaba and Luapula rivers North East China

Seva Samiti

Allahabad

H N Kunjru

1914

Amur

Rahnumai Mazdyasan Sabha

Bombay

Dadabhai Naoroji, Naoroji Fardonji, S S Bengalee

1851

Lena

Baikal Mountains

Mekong

Tibetan Highlands

Niger

Guinea

Major Mountain Ranges Range Andes Himalayan, Karakoram and Hindu kush Rockies Great Dividing Range Atlas Western Ghats Caucasus Alaska Alps

Location South America South Central Asia

Length (km) 7200 5000

North America East Australia North-West Africa Western India Europe USA Europe

4800 3600 1930 1610 1200 1130 1050

Major Mountain Peaks Mountain Peak

Location

Mountain Peak

Location

Mt Everest (highest in the world)

Nepal-Tibet

Tirich Mir

Pakistan

K2 (Godwin Austin)

India (POK)

Aconcagua

Argentina

Dhaulagiri

Nepal

Cotopaxi

Ecuador

Annapurna

Nepal

Kilimanjaro

Tanzania

Gurla Mandhata

Tibet

Famous Plateaus of the World Plateau

Situation

Tibetan Plateau

Between Himalayas and Kunlun Mountains

Deccan Plateau

Southern India

Arabian Plateau

South-West Asia

Plateau of Brazil

Central-Eastern South America

Plateau of Mexico

Mexico

Plateau of Colombia

USA

Plateau of Madagascar

Madagascar

Plateau of Alaska

North-West North America

Plateau of Bolivia

Andes Mountains

Great Basin Plateau

South of Colombia Plateau, USA

Colorado Plateau

South of Great Basin Plateau, USA

15

Important Straits of the World Strait

Water Bodies joined

Area

Bab-elMandeb

Red Sea and Arabian Sea

Arabia and Africa

Bering

Arctic Ocean and Bering Sea

Bosphorus Black Sea and Marmara Sea

Alaska and Asia Turkey

Dover

North Sea and Atlantic Ocean

England and Europe

Florida

Gulf of Mexico and Atlantic Ocean

Florida and Bahamas Islands

Gibralter

Mediterranean Sea and Atlantic Ocean

Spain and Africa

Malacca

Java Sea and Bay of Bengal

India and Indonesia

Palk

Bay of Bengal and Indian Ocean

India and Sri Lanka

Megellan

South Pacific and South Atlantic Ocean

Chile

Sunda

Java Sea and Indian Ocean

Indonesia

Important Lakes of the World Lake

Location

Lake

Caspian

Asia

Tanganyika

Africa

Superior

Canada and USA

Baikal

Russia

Victoria

Africa

Great Bear

Canada

Huron

Canada and USA

Aral

Kazakshtan

Michigan

USA

Great Slave

Canada

List of Waterfalls

https://sscstudy.com/

Waterfall

Location

Angel Falls

Venezuela

Tugela Falls

South Africa

Monge

Norway

Yosemite

United States

Catarata Yumbilla

Peru

Location

https://sscstudy.com/

16

CUET (UG) Section III : General Test

Riverside Cities

Important International Boundary Lines

Town

River

Town

River

Akyab (Myanmar)

Irrawaddy

Baghdad (Iraq)

Tigris

Basara (Iraq)

Tigris and Euphrates

Belgrade

Danube

Name

In Between

Radclife Line (1947)

India and Pakistan (Indo-Pak)

Mac Mohan (1914)

India and China (Indo-China) Pakistan and Afghanistan

Bristol (UK)

Avon

Durand Line (1896)

Budapest (Hungary) Danube

Cairo (Egypt)

Nile

Hindenburg Line

Germany and Poland

Canton

Glasgow (Scotland) Clyde

Maginot Line

France and Germany

Hamburg (Germany) Elbe

Jamshedpur

Subarnarekha

Oder Neisse Line

Germany and Poland

Kabul

Kabul

Khartoum (Sudan)

Nile

Siegfried Line

Fortification between Germany and France

Karachi

Indus

Lahore

Ravi

38th Parallel Line

North and South Korea

Lisbon (Portugal)

Tangus

London (UK)

Thames

Lucknow

Gomti

Montreal (Canada)

Ottawa

49th Parallel Line

USA and Canada

Nanking

Yang-tse-Kiang New Castle(UK)

24th Parallel Line

Pakistan claims that it is the boundary between India and Pakistan in Rann of Kuchch

17th Parallel Line

North Vietnam and South Vietnam

Berlin (Germany)

Spree Si-Kiang

Tyre

New Orleans (USA) Mississippi

New York(USA)

Hudson

Paris (France)

Seine

Philadelphia (USA)

Delaware

Rome (Italy)

Tiber

Shanghai

Yang-tse-Kiang

Srinagar

Jhelum

Warsaw (Poland)

Vistula

Yangon (Myanmar)

Irawady

Highest Peaks of India Highest Peak

Great Deserts

Height (in m)

State

MT K2

8611

PoK (India)

Kanchenjunga

8598

Sikkim

Name

Country/Region

Nanda Devi

7817

Uttarakhand

Sahara (Libyan, Nubian)

North Africa

Saltoro Kangri

7742

Jammu and Kashmir

Australian (Gibson, Simpson), Victorian Great Sandy)

Australia

Kangto

7090

Arunachal Pradesh

Reo Purgyil

6816

Himachal Pradesh

Saramati

3841

Nagaland

Arabian (Rub al Khali, An-Nafud)

Arabia

Dast-e-Lut (Barren Desert)

Iran

Dast-e-Kavir (Salt Desert)

Iran

Desierto de Sechura

Peru

Sandakphu

3636

Paschim Banga

Atacama

North Chile

Khayang

3114

Manipur

Patagonia

Argentina

Anaimudi

2695

Kerala

Kalahari

Botswana

Dodda Betta

2636

Tamil Nadu

Namib

Namibia

Important Indian Towns on Rivers

Major Islands of the World Rank 1. 2. 3. 4. 5. 6.

Name

Area (km2 )3

Country

Greenland New Guinea Borneo Madagascar Baffin Island Sumatra

2130800 785753 748168 587713 507451 443066

Dernmark Indonesia Indonesia, Malaysia Madagascar Canada Indonesia

Famous Grasslands of the World Grassland Steppe Pustaz Prairie Pampas Weld Downs Canterbury

Country Eurasia Hungary USA Argentina and Uruguay (South America) South Africa Australia New Zealand

Town

River

Jamshedpur

Subarnrekha

Town Delhi

River Yamuna

Kanpur

Ganga

Surat

Tapti

Ferozpur

Sutlej

Allahabad

At the confluence of the Ganga and Yamuna

Varanasi

Ganga

Haridwar

Ganga

Badrinath

Alaknanda

Ludhiana

Sutlej

Srinagar

Jhelum

Ayodhya

Saryu

Ahmedabad

Sabarmati

Patna

Ganga

Kota

Chambal

Jabalpur

Narmada

Panji

Mandavi

Ujjain

Kshipra

Guwahati

Brahmaputra

Kolkata

Hooghly

Cuttack

Mahanadi

Hyderabad

Musi

Nasik

Godavari

Lucknow

Gomti

https://sscstudy.com/

https://sscstudy.com/

GENERAL KNOWLEDGE

Important Rivers of India Name

Originates from

Falls into

Name

Originates from

Falls into

Ganges

Combined Sources

Bay of Bengal

Brahmaputra

Near Mansarovar Lake

Bay of Bengal

Sutlej

Mansarovar Rakas Lakes

Chenab

Narmada

Amarkantak

Gulf of Khambat

Indus

Near Mansarovar Lake

Arabian Sea

Tapti

Gulf of Khambat

Ravi

Kullu Hills near Rohtang Pass

Chenab

Betul District in Madhya Pradesh

Beas

Near Rohtang Pass

Sutlej

Mahanadi

Raipur District in Chhattisgarh Bay of Bengal

Jhelum

Verinag in Kashmir

Chenab

Luni

Aravallis

Yamuna

Yamunotri

Ganga

Ghaggar

Himalayas

Near Fatehabad

Chambal

Singar Chouri Peak, Vindhyan Yamuna escarpment

Sabarmati

Aravallis

Gulf of Khambat

Krishna

Western Ghats

Bay of Bengal

Ghaghara

Matsatung Glacier

Ganga

Godavari

Nasik district in Maharashtra

Bay of Bengal

Kosi

Near Gosain Dham Peak

Ganga

Caurery

Vindhyanchal

Yamuna

Brahmagir Range of Western Ghats

Bay of Bengal

Betwa Son

Amarkantak

Ganga

Tungabhadra

Western Ghats

Krishna

Rann of Kuchchh

Important Sanctuaries and National Parks Name

Location

Reserve For One Horned rhinoceros, gaur, elephant, leopard and wild buffalo

Sonai Rupa Sanctuary

Asom Asom

¡

Namdapha Sanctuary

Arunachal Pradesh

Elephant, panther, sambhar, tiger, cheetal and king cobra

¡

Gautam Buddha Sanctuary

Bihar

Tiger, leopard, sambhar, cheetal and barking deer

¡

Achanakmar Sanctuary

Chhattisgarh

Tiger, boar, cheetal, sambhar and bison

¡

Valvadore National Park

Gujarat

Wolf and black buck

¡

Wild Ass Sanctuary

Gujarat

Wild ass, wolf, nilgai and chinkara

¡

Gir Forest

Gujarat

India’s biggest wildlife sanctuary famous for Gir lions

¡

Dachigam Sanctuary

Jammu and Kashmir

Kashmiri stag

¡

Banerghatta National Park

Karnataka

Elephant, cheetal, deer and grey partridge and green pigeon

¡

Bhadra Sanctuary

Karnataka

Elephant, cheetal, panther, sambhar and wild boar

¡

Bandipur Sanctuary

Karnataka

Elephant, tiger, panther, sambhar, deer and birds

¡

Dandeli Sanctuary

Karnataka and Tamil Nadu

Tiger, panther, elephant, cheetal, sambhar and wild boar

¡

Tungabhadra Sanctuary

Karnataka

Panther, cheetal, sloth bear and four-horned antelope

¡

Nagarhole National Park

Karnataka

Nagarhole National Park

¡

Pachmarhi Sanctuary

Karnataka

Tiger, panther, boar, sambar, nilgai and barking deer

¡

Gandhi Sagar Sanctuary

Madhya Pradesh

Cheetal, sambhar, chinkara and wild birds

¡

Bandhavgarh National Park

Madhya Pradesh

Tiger, panther, cheetal, nilgai and wild boar

¡

Simlipal Sanctuary

Madhya Pradesh

Elephant, tiger, leopard, gaur and cheetal

¡

Ghana Bird Sanctuary

¡

Kanchenzunga National Park

Rajasthan Sikkim

Snow leopard, musk deer and Himalayan boar

¡

Vedanthangal Bird Sanctuary

Tamil Nadu

Important bird sanctuary

¡

Chandraprabha Sanctuary

Uttar Pradesh

Gir lions, cheetal and sambhar

¡

Dudhwa National Park

Uttar Pradesh

Tiger, panther, sambar, cheetal, nilgai and barking deer

¡

Corbett National Park

Uttarakhand

Tiger, leopard, elephant and sambhar (named in memory of Jim Corbett)

¡

Jaldapara Sanctuary

Paschim Banga

Rhinoceros

¡

Sunderban Tiger Reserve

Paschim Banga

Tiger, deer, wild boar, crocodile and Gangetic dolphin.

¡

Kaziranga National Park

¡

Elephant, sambhar, wild boar and one-horned rhinoceros

Water birds, black-buck, cheetal and sambar

https://sscstudy.com/

17

https://sscstudy.com/

18

CUET (UG) Section III : General Test

Practice Questions 1. Browning of paper in old books is caused by (a) frequent use (c) collection of dust

(b) lack of aeration (d) oxidation of cellulose

2. Who was popularly known as Africa's Gandhi? (a) Mir Karzai (c) Firoz Gandhi

(a) December 24 (c) October 25

(d) 1995

6. The Central Food Technological Research Institute (CFTRI) is located at (b) Ernakulam (d) Dehradun

(b) France (d) Great Britain (b) Mumbai (d) Bengaluru

10. Who had played key role in the formation of Lokpal bill in India? (a) Baba Amte (b) Medha Patekar (c) Vipin Hazarika (d) Anna Hajare

(b) Bar Code Reader (d) OMR

12. Which one of the following is an ‘Air-to-air’ missile? (a) Astra (c) Agni

(b) Prithvi (d) Akash

(b) Football (d) Table Tennis (Women)

(a) Orissa (c) Rajasthan

(b) Andhra Pradesh (d) Gujarat

23. Who discovered America? (a) Vasco-Da-Gama (c) Captain Cook

(b) Columbus (d) Amundsen

24. The famous painting ‘Monalisa’ was the creation of (a) Michael-Angelo (c) Piccasso

(b) Leonardo-Da-Vinci (d) Van Gogh

(b) Chennai (d) Delhi

(a) Chandini (c) Mridula Rajiv

(b) Bula Choudhary (d) Priya Shanbhag

26. Who said that “India’s soul lives in villages”? (a) Vinoba Bhave (c) Nehru

(b) Jayaprakash Narayan (d) Gandhiji

27. The headquarters of WTO is at (b) Doha (d) Geneva

28. Which state is called the ‘Rice Bowl’ of India?

(b) dark box (d) black box

(a) Andhra Pradesh (c) Kerala

14. National Library, the largest in India, is located in (a) Kolkata (c) Mumbai

(b) K-15 Sagarika (d) Agni

22. Odissi classical dance originated in

(a) New York (c) Uruguay

13. Flight-recorder is technically called (a) altitude meter (c) blind box

(b) Mexico (d) Thailand

25. The first woman to swim across seven important seas is

11. Which of the following devices is generally used to check multiple choice questions? (a) MICR (c) OCR

(b) September 25 (d) November 25

20. Which one of the following is a ‘Surface to Air missile’?

(a) Hockey (c) Kabaddi

9. The Indian Institute of Science is located at (a) Kolkata (c) Chennai

(b) Karnataka (d) Madhya Pradesh

21. Nehru Trophy is associated with

(b) Florida (d) New Jercy

8. Name the country where the first Industrial Revolution took place (a) Germany (c) America

(a) Yugoslavia (c) Japan

(a) Trishul (c) Brahmos

7. Head office of coca cola company is in (a) Atlanta (c) New York

(b) Arms and the Man (d) Murder in the Cathedral

19. Yen is the currency of

5. In which year Colombia hosted the NAM summit

(a) Nagpur (c) Mysore

(d) bosie

18. National Consumer Rights Day is observed in India on

(b) Shih Huang-ti (d) Confucius (c) 1994

(c) bye

16. Bernard Shaw is the author of the drama

(a) Bihar (c) Maharashtra

4. The Great Wall of China was built by

(b) 1997

(b) leg bye

17. Gol Gumbaz, the largest dome in India, is located in the State of

(a) Secretariat (b) Human Rights Council (c) Trusteeship Council (d) Economic and Social Council

(a) 1996

(a) drive

(a) Twelfth Night (c) Death of a Salesman

(b) Nelson Mandela (d) MK Gandhi

3. Name the body which has replaced the UN Commission on Human Rights.

(a) Li-tai-pu (c) Lao-tze

15. In cricket, a run taken when the ball passes the batsman without touching his bat or body is called

(b) Tamil Nadu (d) Karnataka

29. Flight Recorder is technically called (a) Dark box (c) Black box

https://sscstudy.com/

(b) Blind box (d) Altitude meter

https://sscstudy.com/

GENERAL KNOWLEDGE

30. Which of the following countries has entered into an agreement with India for the supply of Advanced Jet Trainer (HAWK)? (a) Russia (c) England

(b) USA (d) France

(a) Morarjee Desai (c) Sachchidananda Sinha

(b) Karnataka (d) Tamil Nadu

34. The 16 years old school boy, Arjun Vajpai who became the youngest Indian to successfully climb the world’s highest peak, Mount Everest, on May 22, 2010, is from which state? (a) (b) (c) (d)

Uttar Pradesh Madhya Pradesh National Capital Territory of Delhi Uttarakhand (b) Agni (d) Astra

(b) Indonesia (d) Russia

(a) Ravishankar (c) Mamta Shankar

(b) Canada (d) None of these

(a) Sri Sri Ravishankar (c) Swami Agnivesh

(b) Japan (d) South Korea (b) Earthquake (d) Over commercialisation

(b) Himalayas (d) Arabian Sea

(b) Uday Shankar (d) Tanushree Shankar

(b) Swami Ramdev (d) Rajneesh

54. The organization ‘World Wide Fund for Nature’ (WWF) was established in 1961 at (a) France (c) Switzerland

(b) UK (d) Spain

55. A popular dance form of Jammu and Kashmir is

(b) Egypt (d) Kazakhstan

(a) Garba (c) Chakri

42. Who one among the following Ex-Chief Minister did not go to the jail in Scamsor Corruption Charges? (b) Lalu Prasad (d) Madhu Koda

43. World famous book ‘Das Capital’ was written by (b) Karl Marx (d) None of these

(b) Verghese Kurien (d) J V Narlikar

53. Who, among the following is the propagator of ‘Art of Living’?

41. ‘Sabawoon’ is the news agency of

(a) Angels (c) R.C. Dutta

(b) Boxing (d) Badminton

52. Anushka Shankar is the daughter of which famous Indian?

40. The Holy City Kedarnath devastated due to

(a) Om Prakash Chautala (c) Mayawati

(a) Wrestling (c) Shooting

(a) Indian Ocean (c) Antarctica

39. ‘Xinhua’ is the news agency of

(a) Turkey (c) Afghanistan

(b) 6 (d) 7

51. India’s Permanent Research Station ‘Dakshin Gangotri’ is located at

38. ‘Reuters’ is the news agency of

(a) Cloud burst (c) Bombing

48. How many countries are permanent member of the Security Council of United Nations Organization?

(a) K Rangarajan (c) M S Swaminathan

37. ‘Antara’ is the news agency of

(a) North Korea (c) China

(b) Swimming (d) Badminton

50. The name of the White Revolution is associated with

(a) Dr. Rajendra Prasad (b) Jawaharlal Nehru (c) R K Shanmukham Chetty (d) Dr. Ambedkar

(a) Britain (c) France

(a) Cricket (c) Lawn Tennis

49. Mary Kom belongs to which sports?

36. Who presented the first Union Budget of Independent India?

(a) Saudi Arab (c) Japan

(b) John Mathai (d) Dr. Ambedkar

47. The term ‘Grand Slam’ is associated with which one of the following game?

(a) 4 (c) 5

35. Which one of the following is an ‘Air-to-Air’ missile? (a) Prithvi (c) Akash

(b) M S Dhoni (d) Shikhar Dhawan

46. Who was the first Railway Minister of Independent India?

(b) Football (d) Badminton

33. Kathakali classical dance originated in (a) Kerala (c) Rajasthan

(a) Virat Kohli (c) Ravindra Jadeja

(a) Railway track for freight movement (b) Road link for freight movement (c) A business hub of Uttar Pradesh (d) None of the above

(b) China (d) Yugoslavia

32. Santosh Trophy is associated with (a) Hockey (c) Basketball

44. India won Champions Trophy 2013 under Captainship of

45. What is called Dedicated Freight Corridor?

31. Yuan is the currency of (a) Japan (c) Italy

19

(b) Chhau (d) Bihu

56. ‘Otis’ is name associated with (a) telegraph (c) computer

(b) submarine (d) lift

57. The name of the eminent activist associated with Lokpal Bill is (a) Medha Patkar (c) Sonia Gandhi

https://sscstudy.com/

(b) Anna Hazare (d) Baba Amte

https://sscstudy.com/

20

CUET (UG) Section III : General Test

58. Zubin Mehta, the Indian-born maestro is a (a) vocalist (c) musician

72. Which city is called the ‘City of Nawabs’?

(b) painter (d) poet

(a) Delhi

59. The dance form popular in North East India is (a) Lai Haroba (c) Roof

(a) Kathak (c) Kuchipudi

61. Among the following wonders of the world, the one that is still surviving is (b) Colossus of Rhodes (d) Statue of Zeus at

(a) P T Usha (c) Mahesh Bhupati

(b) lawn tennis (d) All of these

(a) Padma Shri (c) Param Vir Chakra

(b) corner (d) penalty corner

65. Which of the following university is an Open University? (b) Mumbai University (d) Nalanda University

66. Which of the following countries is not a permanent member of UN Security Council? (b) UK (d) Spain

67. ‘Green Peace International’ (Netherlands) was founded in (a) 1961

(b) 1971

(c) 1984

(d) 1974

68. ‘World Watch Institute’ is located in (a) Rome (c) Geneva

(b) Paris (d) Washington

(a) Social Welfare (b) Corporate Affairs (c) Home Affairs (d) Information and Broadcasting

70. ‘Berlin’ is situated on the bank of river (b) Volga (d) Dnieper

71. Which of the following is a ‘Fire and forget’ anti-tank missile? (a) Trishul (c) Akash

(b) Agni (d) Nag

(b) Asha Bhonsle (d) Mrinal Sen

(b) Sania Mirza (d) Saina Nehwal

(b) Arjuna Award (d) Ashok Chakra

79. The temple situated near Pushkar Lake in Rajasthan is related to (a) Lord Ganesh (c) Lord Mahesh

(b) Lord Vishnu (d) Lord Brahma

80. If someone is injured in an accident and broken his knee joint then he or she needs to consult (a) Orthopedic (c) Oncologist

(b) Paediatrician (d) Urologist

81. If you have the option of buying a refrigerator with 1 star, 3 star, 5 star rating given by BEE, then which one you will prefer? (a) 3 star (c) 1 star

(b) 5 star (d) does not matter

82. Name the city which hosts the literary festival (largest in Asia Pacific)

69. Dada Saheb Phalke Award is given by the Ministry of

(a) Denube (c) Spree

(b) Jamini Roy (d) M F Hussain

78. Which of the following award is given to recognise outstanding achievement in sports?

64. The term not associated with football is

(a) USA (c) France

(a) K L Saigal (c) Manjit Bawa

77. Who among the following is an Olympic Medal winner?

63. ‘Court’ is the playing area for

(a) Calcutta University (c) IGNOU

75. Which of the following name is not associated with painting?

(a) Amitabh Bachchan (c) Pran

(a) Sarat Chandra Chattopadhyay (b) Vikram Seth (c) Munshi Premchand (d) Rabindranath Tagore

(a) forward (c) pass

(b) Kathakali (d) Bharat Natyam

76. Who among the following is not a Dada Saheb Phalke Award winner?

62. ‘Devdas’ is written by

(a) badminton (c) basketball

(b) Easter (d) Diwali

74. Which of the following is a classical dance from North India?

Centre for Railway Internal Services Centre for Railway Information Systems Centre for Rail and Indian Stations Centre for Railway International Services

(a) Pyramids of Egypt (c) Pharos of Alexandria Olympia

(d) Lucknow

73. Which of the following festivals is associated with full moon day? (a) Id-ul-Fitr (c) Holi

(b) Parhaun (d) Bhangra

60. CRIS stands for (a) (b) (c) (d)

(b) Amritsar (c) Agra

(a) Jaipur (c) Kolkata

(b) Lucknow (d) New Delhi

83. Sri Pranab Mukherjee was elected as MP from which of the following places before becoming the President of India? (a) Howrah (c) Malda

(b) Jangipur (d) Raiganj

84. Who won the Best Actor Award for the film Paan Singh Tomar? (a) Irrfan Khan (b) Aamir Khan (c) Farhan Akhtar (d) Ranbir Kapoor

https://sscstudy.com/

https://sscstudy.com/

GENERAL KNOWLEDGE

85. Hugo Chavez was the President of which country? (a) Cuba (c) Argentina

(b) Venezuela (d) Brazil

86. ‘Cue ball, break shot, pool table’ are terms associated with which of the following games? (a) Billiards (c) Golf

(b) Squash (d) Cricket

(a) India (c) England

(b) Three (d) Five

92. In which year Border Security Force (BSF) was established? (b) 1950 (d) 1965

93. Which of the following Fighter Groups are include in Air Force? (a) Mig-21 Veriant (b) Mig-29 S, Mig-25 S, Mig-27 S (c) Jaguar (d) All of the above (b) 1972 (d) 1977

(a) Herat (c) Kabul

(b) Bhaskar-I (d) Insat

105. Which is the currency of Japan? (b) Yuan (d) Rial

106. Find the most suitable answer. A tree always has (a) branches (c) fruit

(b) leaves (d) root

107. Which of the following two stations ‘Samjhota Express’ is running? (a) Delhi - Islamabad (c) Howrah - Dhaka

(b) Candla - Krachi (d) Attari - Amritsar

108. Which of the following place is famous for ‘Rock Garden’? (a) Bengaluru (c) Ahmedabad

(b) Chandigarh (d) Lucknow

(a) 2 (c) 10

(b) 7 (d) 15

(a) (b) (c) (d)

Palace of Chandragupta Maurya at Patna Ashokan Pillar at Allahabad Ashokan Pillar at Delhi Lion Capital of Ashoka at Sarnath

111. How many spokes are in Indian National Flag ?

96. Ustad Amjad Ali is associated with

(a) 24 (c) 26

(b) Tabla (d) Sitar

97. Which of the following folk dance is represent by Mridalini Sarabhai? (b) Kathak (d) Manipuri17.

98. Who is the first Indian to climb Mount Everest? (b) KD Yadav (d) Tenjing Norge

(b) Kandhar (d) Multan

110. The state emblem of India has been adopted from

95. Which is the first Indian satellite to be launched?

(a) Navog Sherpa (c) Bachendri Pal

(b) Tigris (d) None of these

109. How many language are used on a ten rupee note?

94. In which year Shatabadi Express was started?

(a) Bharatnatyam (c) Kuchipuri

(b) Surdas (d) Tulsidas

103. ‘Baghdad’ is loated on the bank of

(a) Yen (c) Rupaih

91. How many commands are made in Indian Navy?

(a) Sarod (c) Violin

(b) Virendra Sehwag (d) Sunil Gavaskar

104. Which is the capital of Afghanistan?

(a) New Delhi - Howrah (b) New Delhi - Chennai (c) New Delhi - Mumbai (d) New Delhi - Hyderabad

(a) Aryabhatt (c) Rohini

(a) Kalidas (c) Banbhatt (a) Vistula (c) Eupherates

90. Which of the following two terminal stations August Kranti Rajdhani Express is running?

(a) 1969 (c) 1988

(b) Anna George Malhotra (d) Karnelia Sorabji

102. Who is the writer of Abhigyanshakuntalam?

(b) Dnieper (d) Po

(a) Jammu Tawi Express (b) Jammu Kanniyakumari Express (c) Himsagar Express (d) Vivek Express

(a) 1947 (c) 1960

(b) Mumbai (d) Chennai

100. Who is the first woman to become an IPS Officer?

(a) Sachin Tendulkar (c) Rahul Dravid

89. Name the train which covers longest distance in India.

(a) Two (c) Four

(a) Kolkata (c) New Delhi

101. Who is the first Indian cricketer to make a triple century in Test Cricket?

(b) Sri Lanka (d) South Africa

88. On which river’s bank ‘Rome’ is located ? (a) Don (c) Tiber

99. First Automated Teller Machine (ATM) is started in which of the following city?

(a) Ujjwala Rai (c) Kiran Bedi

87. Who won the Champions Trophy, 2013?

21

(b) 23 (d) 22

112. The National Anthem of India was adopted by the Constituent Assembly on (a) 26 Jan, 1950 (c) 24 Jan, 1950

(b) 15 Aug, 1947 (d) 26 Nov, 1949

113. The National Calendar was adopted on (a) 26 Jan, 1950 (c) 15 Aug, 1947

https://sscstudy.com/

(b) 26 Nov, 1949 (d) 22 March, 1957

https://sscstudy.com/

22

CUET (UG) Section III : General Test

114. The National Song was composed by

129. First Indian woman to summit Mount Everest was

(a) Rabindranath Tagore (b) Maithilisharan Gupta (c) Bankim Chandra Chatterji (d) Ramdhari Singh ‘Dinkar’

(a) Santosh Yadav (c) Bula Choudhary

130. Who was the first woman to conquer Everest twice ? (a) Arati Saha (c) Bachhendri Pal

115. Which is the National Flower of India ? (a) Rose (c) Lilli

(b) Lotus (d) Jasmine

(a) Bula Choudhary (c) Bachhendri Pal

(b) Elephant (d) Rhinoceros

(a) Bula Choudhary (c) Arati Saha

(b) 1 Vaishakha (d) 1 Magh (b) Banyan (d) Neem

(a) Madeleine Albright (c) Vijayalakshmi Pandit

119. The National Fruit of the country is (a) Banana (c) Mango

(a) Vijayalakshmi Pandit (c) Annie Beasant

(a) John Adams (c) Thomas Jefferson

(b) Anonsheh Ansari (d) Valentina Tereshkova

125. Who was the first Indian to reach space (a) Mihir Sen (b) Rakesh Sharma (c) Major Somnath Sharma (d) S Mukherjee

126. Which was the first artificial satellite launched into orbit ? (a) Explorer 1 (c) Sputnik 1

(b) Vostok 1 (d) Apollo 8

(b) Richard Nixon (d) None of these

139. Who was the first Indian batsman to complete 10000 runs ? (a) Lala Amarnath (c) Sachin Tendulkar

(b) Sunil Gavaskar (d) Rahul Dravid

140. Who became the first Indian cricketer to score a triple century ? (a) Sunil Gavaskar (c) Sachin Tendulkar

(a) K D Jadhav (c) Sushil Kumar

(b) Apollo 11 (d) Explorer 1

128. First Asian woman to swim across the English Channel was (a) Bula Choudhary (b) Arati Saha (c) Santosh Yadav (d) None of the above

(b) Richard Nixon (d) None of these

(b) Mohammad Azharuddin (d) Virendra Sehwag

141. Who became the first Indian to win an individual Olympic Gold ?

127. First manned space vehicle was (a) Vastok 1 (c) Apollo 8

137. The first US President to be assassinated was

138. The first and only US President to resign from office was

(b) Yuri Gagarian (d) None of these

124. First woman of the world in space (a) Dr Sally K Ride (c) Marie Collins

(b) Sucheta Kriplani (d) Vijayalakshmi Pandit

(a) George Washington (b) Thomas Jefferson (c) William Henry Harrison (d) John Adams (a) Abraham Lincoln (c) John Adams

(b) 1928 (d) 1948

123. The first man to land on the Moon was (a) Neil Armstrong (c) Valentina Tereshkova

(a) Annie Beasant (c) Meira Kumar

136. Who was the first US President to die in office ?

(b) Cricket (d) Hockey

122. India won the first Olympic Hockey Gold at Amsterdam in (a) 1932 (c) 1936

(b) Sarojini Naidu (d) Sucheta Kriplani

135. Who became the first woman Lok Sabha speaker in India ?

(b) Eagle (d) Peacock

121. Which of the following is the National Sports of India ? (a) Football (c) Kabaddi

(b) Kim Campbell (d) None of these

134. First woman Chief Minister of a state in India was

(b) Coconut (d) Apple

120. The National Bird of India is (a) Duck (c) Cuckoo

(b) Bachhendri Pal (d) None of these

133. First woman to became the President of UN General Assembly was

118. Which is the National Tree of India ? (a) Pipal (c) Sal

(b) Arati Saha (d) None of these

132. Who was the first Indian woman to cross English Channel twice ?

117. The National Calendar begins on (a) 1 Chaitra (c) 1 Asadha

(b) Junko Tabei (d) Santosh Yadav

131. First Indian woman to swim across seven seas of the world was

116. The National Animal of India is (a) Lion (c) Tiger

(b) Bachhendri Pal (d) None of these

(b) Abhinav Bindra (d) Karnam Malleswari

142. Who is the first Indian woman medal winner in Olympics ? (a) S Vijayalakshmi (b) Kamaljit Sandhu (c) Karnam Malleswari (d) None of the above

https://sscstudy.com/

https://sscstudy.com/

GENERAL KNOWLEDGE

143. The first Indian to cross the English Channel was (a) Mihir Sen (c) Arati Saha

(b) Santosh Yadav (d) None of these

(a) The Hindu (c) Samachar Darpan

(b) Bengal Gazettear (d) Uddant Martand

145. The first post-office in India was established at (a) Kolkata (c) Mumbai

(b) Delhi (d) Allahabad

(a) 1855 (b) 1853 (c) 1857 (d) 1852

147. In India, Radio Transmission Service was started at Kolkata and Mumbai in (b) 1866

(c) 1933

(d) 1874

148. The first Television Centre of India was established in 1959 at (a) Kolkata (c) New Delhi

(b) Bangaluru (d) Chennai

(b) 15 Aug, 1986 (d) 15 Aug, 1974 (b) Rohini (d) Insat 2A

151. First indigenously built Indian satellite was (a) Aryabhatta (c) Insat 1A

(b) Insat 2A (d) Rohini

152. The first hydel plant in India was established at (a) Darjeeling (c) Asom

(b) Arunachal Pradesh (d) Uttar Pradesh

153. The first indigenously designed and built Indian Missile was (a) Nag

(b) Trishul

(c) Prithvi

(d) Agni

154. The first underground nuclear testing of India was carried in (a) 1971

(b) 1974

(c) 1977

(d) 1978

155. The first test-tube baby of India was (a) Baby Harsha (c) Baby Pinki

(b) Norway (d) All of these

(c) Canada

(a) Red Cliff Line (c) Maginot Line

(b) Mac Mohan Line (d) Durand Line

(b) UK (d) Both ‘a’ and ‘b’

(a) France and Germany (b) North Korea and South Korea (c) USA and Canada (d) None of the above (a) North Korea and South Korea (b) France and Germany (c) USA and Canada (d) Germany and Poland

167. ‘17th Parallel’ is the boundary between (a) North Vietnam and South Vietnam (b) USA and Canada (c) USA and Mexico (d) Peru and Chile

168. The boundary line between West Germany and Poland is (a) Hindenberg Line (c) 38th Parallel Line

(b) 17th Parallel Line (d) Maginot Line

169. Who is popularly known as ‘Rajaji’ ? (a) C Rajgopalachari (c) Raja Ramanna

(b) C F Andrews (d) None of these

(b) Richard Golden (d) None of these

(b) Bismark (d) None of these

172. The official document of UK is known as

(d) Belgium

159. ‘White Lily’ is the National Symbol of (a) Italy (c) France

(b) UK (d) None of these

164. Which of the following is the international boundary between India and Pakistan ?

(a) Sardar Patel (c) Hitler

(b) Sudan (d) All of these (b) France

(a) Canada (c) Chile

171. Who of the following is known as ‘Blood and Iron Man’ ?

158. ‘Lily’ is the National Symbol of (a) UK

(b) Double headed eagle (d) None of these

163. ‘Mapple Bird’ is the National Symbol of

(a) Sir Walter Scott (c) Tenzing Norge

157. ‘Eagle’ is the National Symbol of (a) Spain (c) Germany

162. The National Symbol of Russia is

170. Who of the following is popularly known as ‘Tiger of Snow’ ?

(b) Baby Khatoon (d) Baby Julu

156. ‘Lion’ is the National Symbol of (a) Belgium (c) Netherlands

(b) Lily (d) Moon and Stars

166. ‘38th Parallel’ is the boundary between

150. Which was the first Indian satellite launched ? (a) Aryabhatta (c) Insat 1A

(a) Rose (c) Eagle

165. ‘49th Parallel’ is the boundary between

149. The coloured programme on television was started on (a) 15 Aug, 1980 (c) 15 Aug, 1982

(b) France (d) All of these

161. The National Symbol of Iran is

(a) Eagle (c) Mapple Bird

146. The first postal stamp in India was issued in

(a) 1927

160. ‘Water Lily’ is the National Symbol of (a) UK (c) Bangladesh

144. Which was the first newspaper published in India ?

23

(a) Blue Book (c) Yellow Book

(b) Green Book (d) Orange Book

173. ‘Yellow Book’ is the official document of (a) UK (c) France

https://sscstudy.com/

(b) Germany (d) Italy

https://sscstudy.com/

24

CUET (UG) Section III : General Test

176. ‘Anand Van’ is associated with

174. ‘Orange Book’ is the official document of (a) Germany (c) Portugal

(a) Vinoba Bhave (b) Baba Amte (c) Indira Gandhi (d) Mahatma Gandhi

(b) Italy (d) Netherlands

175. The place ‘Nirmal Hriday’ is associated with (a) Rabindranath Tagore (b) Mother Teresa (c) Guru Nanak (d) Baba Amte

177. ‘Phoenix Farm’ is associated with (a) Mahatma Gandhi (c) Rabindranath Tagore

(b) J L Nehru (d) None of these

ANSWERS 1. 11. 21. 31. 41. 51. 61. 71. 81. 91. 101. 111. 121. 131. 141. 151. 161. 171.

(d) (d) (b) (b) (c) (c) (a) (d) (b) (b) (b) (a) (d) (a) (b) (b) (a) (b)

2. 12. 22. 32. 42. 52. 62. 72. 82. 92. 102. 112. 122. 132. 142. 152. 162. 172.

(b) (a) (a) (b) (c) (a) (a) (d) (a) (d) (a) (c) (b) (a) (c) (a) (b) (a)

3. 13. 23. 33. 43. 53. 63. 73. 83. 93. 103. 113. 123. 133. 143. 153. 163. 173.

(b) (d) (b) (a) (b) (a) (d) (c) (b) (d) (b) (d) (a) (c) (a) (c) (a) (c)

4. 14. 24. 34. 44. 54. 64. 74. 84. 94. 104. 114. 124. 134. 144. 154. 164. 174.

(b) (a) (b) (a) (b) (c) (d) (a) (a) (c) (c) (c) (d) (d) (b) (b) (a) (d)

5. 15. 25. 35. 45. 55. 65. 75. 85. 95. 105. 115. 125. 135. 145. 155. 165. 175.

(d) (c) (b) (c) (a) (c) (c) (a) (b) (a) (a) (b) (b) (c) (a) (a) (c) (b)

6. 16. 26. 36. 46. 56. 66. 76. 86. 96. 106. 116. 126. 136. 146. 156. 166. 176.

(c) (b) (c) (c) (b) (d) (d) (a) (a) (a) (d) (c) (c) (c) (d) (d) (a) (b)

https://sscstudy.com/

7. 17. 27. 37. 47. 57. 67. 77. 87. 97. 107. 117. 127. 137. 147. 157. 167. 177.

(a) (b) (d) (b) (c) (b) (b) (d) (a) (a) (d) (a) (a) (a) (a) (d) (a) (a)

8. 18. 28. 38. 48. 58. 68. 78. 88. 98. 108. 118. 128. 138. 148. 158. 168.

(d) (a) (a) (a) (c) (c) (d) (b) (c) (d) (b) (b) (b) (b) (c) (b) (a)

9. 19. 29. 39. 49. 59. 69. 79. 89. 99. 109. 119. 129. 139. 149. 159. 169.

(d) (c) (c) (c) (b) (a) (d) (d) (d) (b) (d) (c) (b) (b) (c) (d) (a)

10. 20. 30. 40. 50. 60. 70. 80. 90. 100. 110. 120. 130. 140. 150. 160. 170.

(d) (a) (b) (a) (b) (b) (c) (a) (c) (c) (d) (d) (d) (d) (a) (c) (c)

https://sscstudy.com/

GENERAL KNOWLEDGE

25

CHAPTER 02

General Science Inventions and Discoveries Invention

Year

Inventor

Country

Artificial Heart

1957

Willem Kolff

Netherlands

Atomic Bomb

1945

J Robert Oppenheimer USA

Atomic Theory

1803

Dalton

Britain

Automatic Rifle

1918

John Browning

USA

Ball-Point Pen

1888

John J Loud

USA

Barometer

1644

Evangelista Torricelli

Italy

Invention

Year

Inventor

Country

Proton

1919

Rutherford

N Zealand

Quantum Theory

1900

Plank

Germany

Radar

1922

AH Taylor and Leo C Young

USA

Radiocarbon dating

1947

Libby

USA

Relativity Theory

1905

Einstein

Germany

Stethoscope

1819

Laennec

France

Telegraph Code

1837

Samuel FB Morse

USA

Telescope

1608

Hans Lippershey

Netherlands

Television (Mechanical)

1926

John Logie Baird

Britain

Transformer

1831

Michael Faraday

Britain

Transistor

1948

Bardeen, Shockley and Brattain

USA

Bicyle

183940

Kirkpatrick Macmillan

Britain

Bifocal Lens

1780

Benjamin Franklin

USA

Bunsen Burner

1855

R Willhelm von Bunsen Germany

Calculus

1670

Newton

Britain

Cloning, DNA

1973

Boyer, Cohen

USA

Diesel Engine

1895

Rudolf Diesel

Germany

Wireless (telegraphy) 1896

G Marconi

Italy

Dynamo

1832

Hypolite Pixii

France

X-ray

WK Roentgen

Germany

Electron

1897

Thomson J

Britain

Fountain Pen

1884

Lewis E Waterman

USA

Galvanometer

1834

Andre-Marie Ampere

France

Gramophone

1878

Thomas Alva Edison

USA

Hydrogen Bomb

1952

Edward Teller

USA

Intelligence Testing

1905

Simon Binet

France

Laser

1960

Theodore Maiman

USA

Lift (Mechanical)

1852

Elisha G Otis

USA

Lighting Conductor

1752

Benjamin Franklin

USA

Logarithms

1614

Napier

Britain

Microphone

1876

Graham Bell

USA

Movie Projector

1893

Thomas Edison

USA

Pacemaker

1952

Zoll

USA

Pasteurization

1867

Louis Pasteur

France

Periodic Table

1869

Mendeleyev

Russia

Pistol, Revolver

1836

Colt

USA

Printing Press

1455

Johann Gutenberg

Germany

1895

Milestones in Medicine Discovery/ Invention

Year

Discoverer/ Inventor

Country

Aspirin

1889

Dreser

Germany

Ayurveda

2000-1000 BC —

India

Bacteria

1683

Leeuwenhock

Netherlands

Biochemistry

1648

Jan Baptista Van Belgium Helmont

Blood Plasma storage (Blood bank)

1940

Drew

USA

Blood Transfusion 1625

Jean-Baptiste Denys

France

Chemotherapy

1493-1541

Paracelsus

Switzerland

Circulation of blood

1628

William Harvey

Britain

Heart Transplant Surgery

1967

Christian Barnard

S Africa

https://sscstudy.com/

https://sscstudy.com/

26

CUET (UG) Section III : General Test

Discovery/ Invention

Year

Penicillin

1928

Discoverer/ Inventor

Country

Alexander Fleming

Britain

Rabies Vaccine

1860

Louis Pasteur

France

Rh-factor

1940

Karl Landsteiner

USA

Vaccination

1796

Edward Jenner

Britain

Vaccine, Polio

1954

Jonas Salk

USA

Vaccine, Polio-oral 1960

Albert Sabin

USA

Vaccine, Rabies

1885

Louis Pasteur

France

Yoga

200-100 BC

Patanjali

India

Unit of Measurements

Instrument

Uses

Fathometer

It measures the depth of the ocean.

Hydrometer

It measures the specific gravity of liquids.

Hygrometer

It measures humidity in air.

Lactometer

It determines the purity of milk.

Microphone

It converts the sound waves into electrical vibrations and to magnify the sound.

Microscope

It is used to obtain magnified view of small objects.

Periscope

It is used to view objects above sea level (used in sub-marines).

Pyrometer

It measures very high temperature.

Radar

It is used for detecting the direction and range of an approaching plane by means of radio microwaves.

Seismograph

It measures the intensity of earthquake shocks.

Stethoscope

An instrument which is used by the doctors to hear and analyse heart and lung sounds.

Quantity

Unit (SI)

Quantity

Unit (SI)

Length

Metre

Weight

Newton or Kilogram

Time

Second

Impulse

Newton-second

Mass

Kilogram

Heat

Joule

Telescope

It views distant objects in space.

Thermometer

This instrument is used for the measurement of temperatures.

Voltmeter

It measures the electric potential difference between two points.

Area

Square metre

Temperature

Kelvin

Volume

Cubic metre

Absolute temperature

Kelvin

Velocity

Metre/second

Resistance

Ohm

Acceleration

Metre/second square

Electric current

Ampere

Ores of Metals Name of the Elements

Density

Kilogram/metre Electromotive force cube

Volt

Momentum

Kilogram metre/second

Electric energy

Kilo watt hour

Work

Joule

Electric power

Kilo watt or watt

Energy

Joule

Charge

Coulomb

Force

Newton

Luminous flux

Candela

Pressure

Pascal or Newton/sq metre

Intensity of sound

Decibel

Frequency

Hertz

Power of lens

Dioptre

Power

Watt

Depth of sea

Fathom

Important Scientific Instruments

Ore

Chemical Formulae

Aluminium (Al)

(a) Bauxite (b) Kryolite

Al 2O 3 ⋅ 2H2O Na 3 AlF6

Iron (Fe)

(a) Haematite (b) Magnetite (c) Iron Pyrite

Fe 2O 3 Fe 3O 4 FeS2

Copper (Cu)

(a) Copper Pyrite (b) Copper Glance

CuFeS2 Cu2S

Zinc (Zn)

(a) Zinc Blende (b) Calamine

ZnS ZnCO 3

Sodium (Na)

(a) Rock Salt (b) Sodium Carbonate

NaCl Na 2CO 3

Lead (Pb)

(a) Galena (b) Anglesite

PbS PbCl 2

Mercury (Hg)

(a) Cinnabar (b) Calomel

HgS Hg 2Cl 2

Instrument

Uses

Altimeter

It measures altitudes and is used in aircrafts.

Ammeter

It measures strength of electric current (in amperes).

Anemometer

It measures force and velocity of wind.

Audiometer

It measures intensity of sound.

Common Name

Audiophone

Common and Chemical Names of Some Compounds Chemical Name

Chemical Formulae

It is used for improving imperfect sense of hearing.

Dry Ice

Solid Carbondioxide

CO 2

Barometer

It measures atmospheric pressure.

Bleaching Powder

Calcium Oxychloride

CaOCl 2

Binocular

It is used to view distant objects.

Nausadar

Ammonium Chloride

NH4Cl

Dynamo

It converts mechanical energy into electrical energy.

Caustic Soda

Sodium Hydroxide

NaOH

https://sscstudy.com/

https://sscstudy.com/

GENERAL KNOWLEDGE

Common Name

Chemical Name

Potash Alum

Potassium K 2SO 4 ⋅ Al 2 (SO 4 )3 Aluminium Sulphate ⋅ 24H2O

Epsom

Magnesium Sulphate

Some Important Facts of Human Body

Chemical Formulae

MgSO 4 ⋅ 7H 2 O

27

Number of cells in body

75 trillion

Longest bone

Femur (thigh bone)

Smallest bone

Ear ossicle, stapes

Blood volume

6.8 litres (in 70 kg body) 120/80 mm Hg

Quick Lime

Calcium Oxide

CaO

Plaster of Paris

Calcium Sulphate

1 (CaSO 4 ) H 2 O 2

Normal BP Life span of RBC

120 days 3-4 days O Rh-ve

Gypsum

Calcium Sulphate

CaSO 4 ⋅ 2H 2 O

Life span of WBC

Green Vitriol

Ferrous Sulphate

FeSO 4 ⋅ 7H 2 O

Universal blood donor

Blue Vitriol

Copper Sulphate

CaSO 4 ⋅ 5H 2 O

Universal blood recipient

AB 70 kg

White Vitriol

Zinc Sulphate

ZnSO 4 ⋅ 7H 2 O

Average body weight

Marsh Gas

Methane

CH 4

Normal body temperature

98.4° F or 37°C

Vinegar

Acetic Acid

CH 3 COOH

Dental formula

Baking Powder

Sodium Bicarbonate NaHCO 3

adult : 2123/2123 = 32; child : 2120/2120 = 22 milk teeth

Washing Soda

Sodium Carbonate

Na 2 CO 3 ⋅ 10H 2 O

Gestation period

9 months (253-266 days)

Largest gland

Liver

Magnesia

Magnesium Oxide

MgO

Largest muscle in the body

Gluteus maximus (Buttock muscle)

Laughing Gas

Nitrous Oxide

N2O

Longest cell

Neuron (nerve cell)

Chloroform

Tricholoro Methane

CHCl 3

Menstrual cycle

28 days

Vermelium

Mercuric Sulphide

HgS

Minimum distance for proper vision

25 cm

Borax

Borax

Na 2 B 4 O 7 ⋅ 10H 2 O

pH of urine

6.0

Alcohol

Ethyl Alcohol

C 2 H 5 OH

Sugar

Sucrose

C12 H 22 O11

Heavy Water

Duterium Oxide

D2O

Sand

Silicon Oxide

SiO 2

Nutrition Ä

Carbohydrates Ä

Some Important Alloys Name

Composition

Ä

Use

Ä

Brass

Cu (60 to 80%), Zn (40 to 20%)

For making household utencils

Bronze

Cu (75 to 90%), Sn (25 to 10%)

For making coins, idols, utencils

German Silver

Cu (60%), Zn (25%), Ni (15%)

For making utencils

Duraliumin

Al (95%), Cu (4%), Mg (0 ⋅ 5%), Mn ( 0 ⋅ 5%)

In aircraft manufacturing

Fe (98%), C (2%)

For making nails, screws, bridges

Steel Stainless Steel

Constitutes 3 elements : Carbon, Hydrogen and Oxygen. Main source of energy providers. 1 gm, gives 17 kg of energy (or 4.1 kcal of energy).

Fats Ä

Ä

Ä

Provides twice the energy of carbohydrates (1 g provides 37 kJ of energy). An enzyme called Lipase digests fats. It breaks down into fatty acids and glycerol. Daily requirement : 50 g.

Proteins Ä Ä

Fe (82%) Cr, Ni (18%) For making cooking utencils, knives

An alloy is a mixture of two or more metals.

The components of food are : Carbohydrates, Fats, Proteins, Minerals, Vitamins, Water and Roughage.

Ä Ä

Made up of Carbon, Hydrogen, Oxygen and Nitrogen. Important for growth and repair of the body. Made up of amino acids. Kwashiorkar and Marasmus are the diseases which occur due to deficiency of protein.

https://sscstudy.com/

https://sscstudy.com/

28

CUET (UG) Section III : General Test

Minerals Types of Minerals Mineral Element Source

Significance

Effect of Deficiency

Macro Element Calcium (Ca)#

Milk, cereals, cheese, green vegetables

Required for formation of teeth and bones, blood clotting, functions of nerves and muscles

Weak teeth and bones; retarded body growth

Phosphorus (P)

Milk, meat, cereals

Required for formation of teeth and bones and acid-base balance; component of ATP, DNA, RNA

Weak teeth and bones; retarded body growth and physiology

Sulphur (S)

Many proteins of food

Component of many amino acids

Disturbed protein metabolism.

Potassium (K)

Meat, milk, cereals, fruits and vegetables

Required for acid-base balance, water regulation and function of nerves

Low blood pressure, weak muscles; risk of paralysis

Chlorine (Cl)

Table salt

Required for acid base balance; component of gastric juice

Loss of appetite; muscle cramps

Sodium (Na)

Table salt

Required for acid-base and water balance and nervous functions

Low blood pressure, Loss of appetite; muscle cramps

Cereals, green vegetables

Cofactor of many enzymes of glycoly- sis and a number of other metabolic reactions dependent upon ATP

Irregularities of metabolism principally affecting nervous functions

Iron (Fe)

Meat, eggs, cereals, green vegetables

Component of haemoglobin and cytochromes

Anaemia, weakness and weak immunity

Iodine (I)

Milk, Cheese, Sea food, iodized salt

Important component of thyroxine hormone

Goitre, Cretinism

Fluorine (F)

Drinking water, tea, sea food

Maintenance of bones and teeth

Weak teeth, larger amount causes mottling of teeth

Zinc (Zn)

Cereals, milk, eggs, meat, sea food

Cofactor of digestive and many other enzymes

Retarded growth, anaemia, rough skin, weak immunity and fertility

Copper (Cu)

Meat, dry fruits, pods, Cofactor of cytochrome oxidase enzyme. Necessary green vegetables, sea food for iron metabolism and development of blood vessels and connective tissues

Anaemia, weak blood vessels and connective tissues

Dry fruits, cereals, tea, fruits Cofactor of some enzymes of urea synthesis and and green vegetables transfer of phosphate group

Irregular growth of bones, cartilages and connective tissues

Magnesium (Mg)

Micro Element

Manganese (Mn)

# The salf of Ca required by our body is Ca 3 (PO 4 )2 .

Vitamins Necessary for normal growth, good health, good vision, proper digestion of the body etc. They do not provide energy to our body. Vitamins can be divided into two categories Ä Water-soluble Vitamin B-complex, Vitamin C. Ä Fat-soluble Vitamin A, Vitamin D, Vitamin E, Vitamin K. Types of Vitamins Vitamin

Chemical Name

Property

Deficiency Disease

A

Retinol

General health giving vitamin, can be stored in liver

Night blindness

B1

Thiamine

For growth, carbohydrate metabolism, functioning of heart

Beri-Beri

B2

Riboflavin

For keeping skin and mouth healthy

Cheilosis

C

Ascorbic Acid

For keeping teeth, gums and joints healthy. Gets destroyed on heating

Scurvy

D

Calciferol

For normal bones and teeth, can be stored in liver

Rickets

E

Tocopherol

For normal reproduction, removes scars and wrinkles

Sterility

K

Phylloquinone

For normal clotting of blood

Haemophilia

https://sscstudy.com/

https://sscstudy.com/

GENERAL KNOWLEDGE

Water Important in digestion, transportation, excretion and to regulate body temperature (body contains 65% water).

Roughage Fibrous material present in the cell wall of plants. Mainly contains cellulose. It doesn't provide energy but only helps in retaining water in the body.

Ä Ä Ä

Blood Blood is a fluid connective tissue. Its quantity is 6.8 litres in man and 500 ml less in woman. Constitutes 6–8% of body weight and has a pH of 7.4.

Ä Ä

Ä

Communicable or Infectious Diseases Diseases Caused by Virus Disease

Pathogen responsible

Smallpox

Variola virus

Chickenpox

Varicella virus

Common cold

Rhino virus

Influenza flu

Orthomixo virus

Measles (Rubella)

Measles virus (paramyxo virus)

Mumps

Mumps virus (paramyxo virus)

Poliomylities

Polio virus (entero virus)

Rabies (Hydrophobia)

Rabies virus (rhabdo virus)

Dengue fever or Breakbone fever

Dengue virus (arbo virus)

Acquired Immuno Deficiency Syndrome (AIDS)

Human T-cell Leukemia virus (HTLV-III); called LAV (retro virus)

Types of Blood Groups

Diseases Caused by Bacteria

Blood Group

Can donate to

Can receive from

A

A, AB

B

Disease

Pathogen responsible

A, O

Diphtheria

Irregular rod (Corynebacterium diptheriae)

B, AB

B, O

Pneumonia

Diplococcus pneumoniae

AB

AB

A, B, AB, O

Tuberculosis

Irregular rod (mycobacterium tuberculosis)

O

A, B, AB, O

O

Plague or Bubonic plague

Short rod, Yersinia pestis

Tetanus or lockjaw

Clostridium tetani

Typhoid or enteric fever

Salmonella typhi

Cholera

Vibrio cholerae

RH Factor ●





It is a blood antigen found in RBC. A person can be Rh+ or Rh– depending upon the presence of Rh factor in RBC. Blood transfusion technique was developed by James Blundell.

Syphilis

Spiral-shaped organism (Treponema pallidum)

Leprosy

Mycobacterium leprae

Botulism

Clostridium botulinum

Diseases Caused by Protozoans

Nervous System Ä

Ä

29

The nerves, the brain and the spinal cord constitute the nervous sytem. Nervous system controls and regulates the activities of all the other systems of the body.

Brain

Disease

Pathogen responsible

Malaria

Plasmodium

Sleeping sickness (Trypanosomiasis)

Trypanosoma brucei

Kala-azar or black sickness

Leishmania donovani

Common Diseases and Affected Body Part

Brain is the main organ of the nervous system. It consists of cerebrum, cerebellum and medulla oblongata.

Disease

Affected Part

AIDS

Immune system of the body

Cerebrum

Arthritis

Joints

It controls the voluntary actions and is the seat of intelligence.

Asthma

Bronchial muscles

Cataract

Eyes

Conjunctivitis

Eyes

Glaucoma

Eyes

Diabetes Mellitus

Pancreas and blood

Dermatitis

Skin

Cerebellum It controls the voluntary actions and is the seat of intelligence.

Medulla Oblongata Lowest part of the brain and is connected with the spinal cord. It controls the involuntary actions.

Diphtheria

Throat

Dementila

Brain

https://sscstudy.com/

https://sscstudy.com/

30

CUET (UG) Section III : General Test

National Laboratories and Research Institutions

Disease

Affected Part

Eczema

Skin

Name

Place

Goitre

Thyroid

Central Leather Research Institute

Chennai, T.N.

Hepatitis

Liver

Central Mining Research Station

Dhanbad, Jharkhand

Jaundice

Liver

Indian Institute of Science

Bengaluru, Karnataka

Malaria

Spleen

Indian Space Research Organisation

Bengaluru, Karnataka

Meningitis

Brain

All India Institute of Medical Sciences

New Delhi

Paralysis

Nerves and limbs

Polio

Legs

Nuclear/Atomic Research Centre

Pyarrhoea

Teeth

Name of the centre

Place

Pneumonia

Lungs

Bhabha Atomic Research Centre

Trombay, near Mumbai, MS

Typhoid

Intestines

Saha Institute of Nuclear Physics

Kolkata, Paschim Banga

Tuberculosis

Lungs Tonsils gland in throat

Tata Institute of Fundamental Research

Mumbai, Maharashtra

Tonsillitis

Environmental Science Components of Ecosystem

Ecology Ä

Ä

Ä

Ecology is the study of interaction between living organisms and their environment. The term ‘ecology’ was coined by Ernst Haeckel in 1886. Odum (1971) defined ecology as ‘study of structure and function of nature’.

Ecosystem The term ecosystem was given by A G Tansley (1935). Ä Interaction between living and non-living components is called ecosystem. There are two types of ecosystems : Ä

I. Natural Ecosystem These are operated by themselves under natural conditions without any major interference by man. These are of two types (i) Terrestrial eg, grassland, forest, desert ecosystem, etc. (ii) Aquatic It is further of two types :

(a) Fresh water Which may be lotic (running water as springs stream or river) or lentic (standing water as lake, ponds, ditch, swamp, etc). (b) Marine Ocean and sea.

II. Artificial Ecosystem These are maintained artificially by man. It is also known as man made ecosystem. eg, crop land ecosystem, aquarium.

Ecosystem has two components

I. Abiotic Component Non-living portion of an ecosystem can be divided into three parts (i) Climate, e.g. light, wind, temperature, water etc. (ii) Inorganic substances These includes various elements, e.g. sulphur, nitrogen, calcium, phosphorus. (iii) Organic compounds, e.g. proteins, carbohydrates, lipids.

II. Biotic Component These includes all living organisms on the basis of mode of nutrition biotic components are of following types : (i) Autotrophs These include all photosynthesizing plants these are self food producing members and prepare their food in the presence of sunlight, chlorophyll, CO2 and water. (ii) Heterotrophs These include organism, which cannot prepare their own food material and they obtain food from autotrophs. These are of two types on the nature of food. (a) Herbivores Obtain food from plants. (b) Carnivores Obtain food from various herbivores. They further are of two types : Ä Lower carnivores (obtain food from herbivores) Ä Higher carnivores (obtain food from carnivores) (iii) Decomposers These are micro-organisms include bacteria and fungi, etc. They cause breakdown of dead and decaying material into simple organic and inorganic components. These are further used by the autotrophs.

https://sscstudy.com/

https://sscstudy.com/

GENERAL KNOWLEDGE

Food Chain Ä

Ä

The process of transfer of energy from producer through a series of organism to consumers is known as food chain. e.g.,

Grass

eaten by

(producer)

Ä

Rabbit (primary consumer)

eaten by

Ä

In nature, there are two types of food chains are present : (i) Grazing food chain starts with green plants,

Ä

Ä

Ä

Food Web Ä

Ä

Various food chains are interconnected with each other forming an interlocked system, which is known as food web. Food web are very important in maintaining equilibrium of ecosystem.

Ä

Ä

Food web

It is straight single pathway

It consists of number of interconnected food chains

Members of higher trophic level feed upon a single type of organisms of lower trophic level

Members of higher trophic level can feed as a member of alternative organisms of power trophic level

It increases adaptability and competitiveness of the organisms

It increases adaptability and competitiveness of the organisms.

Ä

Ä

Ä

Ä

Ä Ä

Ä Ä

Ä

Ecological Pyramids The producers and consumers can be arranged into several feeding or nutritional groups, each is known as trophic level. Trophic structure of an ecosystem can be express by means of ecological pyramids. The concept of ecological pyramid was given by Charles Elton in 1927. Each food level is also called trophic level.

Ä

Ä

Ä

I. Pyramid of Number In this, number of individuals at each trophic level is shown. Pyramid of number is upright in case of grassland and pond ecosystem, intermediate in forest ecosystem and inverted in parasitic food chain.

II. Pyramid of Biomass Ä

In this, the biomass of each trophic level is shown in pyramid.

It is due to excess of different gases, smoke, particulate matters, chemicals, allergens, etc. Important primary air pollutants are CO, SO2 , NO2 , HF, etc. Secondary air pollutants are O3 , Peroxy Acetyl Nitrate (PAN), etc. Most important gaseous air pollutant is SO2. SO 2 is produced due to combustion of fossil fuels. Excess of sulphides is the cause of SO2 injury in plants. SO 2 above 1 ppm affects human beings in terms of irritation of eyes, respiratory tract, bronchitis asthma etc. Maximum SO2 pollution in India occurs in Kolkata. Yellowing and blacking of Taj Mahal in Agra is due to SO2 released by Mathura refinery. This type of pollution is called ‘Stone cancer’. Increase in global temperature mainly due to CO2 concentration is called Green-house effect. The mixture of SO2 and NO2 with water is called acid rain. Major air pollution disaster in India occurred in 1984 at Bhopal due to release of MIC (Methyl Isocyanate) from insectiside factory of union carbide.

II. Water Pollution

Types of Ecological Pyramids These are of three types :

Odum defined the term ‘pollution’. Undesirable change in physical, chemical and biological properties of air, water and soil, which directly or indirectly affect human beings, is called pollution. Pollution causing agents or substances are called pollutants.

I. Air Pollution Ä

Differences between Food Chain and Food Web Food chain

Energy content or productivity of different trophic level is shown in pyramid of energy. Pyramid of energy is always upright in all ecosystems.

Pollution Ä

which are eaten by herbivores, which in turn eaten by carnivores. (ii) Detritus food chain starts with dead organic matter acted upon by decomposers.

Pyramid of biomass may be upright in grassland and forest ecosystem whereas it invert in pond ecosystem.

III. Pyramid of Energy

Hawk (secondary consumer)

31

Ä

Ä

Ä Ä

It is mainly due to addition of sewage, detergents, domestic wastes, infectious agents, industrial wastes, minerals, etc, in the ordinary water. Degree of water pollution is measure in terms of BOD (Biological Oxygen Demand). If BOD is less than 1500 mg/L, it indicates less pollution. If BOD is 1500-4000 mg/L, it indicates medium pollution. If BOD is more than 4000 mg/L, it indicates high pollution. Mercury (Hg) is very persistent effluent. Cadmium (Cd) accumulation in liver, kidney and thyroid causes itai-itai (ouch-ouch) disease.

https://sscstudy.com/

https://sscstudy.com/

32

CUET (UG) Section III : General Test

III. Soil Pollution Ä

Ä Ä

Ä

Ä

Toxicity of soil increases as a result of chemical wastes, which has adverse effect on human beings. Excessive use of nitrogen fertilizers has increased level of nitrates in soil, which is responsible for blue-baby syndrome. Major cause of soil pollution is disposal of plastics. Plastic articles are non-biodegradable, ie, they cannot be decomposed. Biological pathogens play important role in soil pollution.

Ä

V. Radioactive Pollution Ä Ä

Ä

Ä

IV. Noise Pollution Ä

Ä

Unwanted sound is called noise it is measured in term of decibles (dB). Generally, sound above 80 dB is termed as noise.

Kanpur is the most noise polluted city in UP. Large green plants, which are planted in high noise pollution zone are known as green mufflers as they have the capacity to absorb sound waves.

Ä

Chief sources are nuclear explosions. Sr-90 accumulates in bone marrow and causes cancer (leukaemia). Cs137 accumulates in muscles and causes muscular pain. Radioactive disaster was first occurred in 1945 in Hiroshima and Nagasaki (Japan), when USA was dropped nuclear bombs during Second World War. Recently in 1986, nuclear accident occurred at Chernobyl (USSR).

https://sscstudy.com/

https://sscstudy.com/

GENERAL KNOWLEDGE

33

Practice Questions 1. Myopia is a disease connected with (a) lungs (c) ears

15. Decibel is a term connected with

(b) brain (d) eyes

(a) air

2. The weight of a body acts through the centre of (a) gravity (c) both (a) and (b)

(b) mass (d) buoyancy

(a) 36000 km (c) 30000 km

(a) photochemical reaction (b) physical change (c) endothermic reaction (d) exothermic reaction

5. The splitting of white light into its components is due to (b) dispersion (d) refraction

(a) oil has a higher viscosity (b) water has a higher viscosity (c) oil has a higher surface tension (d) water has a higher surface tension

(c) white

(d) grey

11. The principle due to which circular patches of light is seen under a tree during day time, is similar to that of image formation by a (b) pinhole (d) convex lens

(a) Isoprene (c) Butadiene

(d) 1

(b) Styrene (d) Ethylene

(b) Neon (d) Oxygen

23. A nuclear reaction must be balanced in terms of (b) energy and weight (d) mass and energy (b) IR rays (d) Cosmic rays

25. A micron is equal to (a) 01 . mm (c) 0.001 mm

(b) 0.01 mm (d) 0.0001 mm

26. Persons suffering from myopia are advised to use (a) convex lens (c) plano-convex lens

(a) Communication (c) Musical instruments

(a) magnetic North-South direction (b) polarity of a magnet (c) strength of a magnet (d) direction of magnetic field

(b) concave lens (d) plano-concave lens

(b) Weaving (d) Food industry

28. Which of the following are most penetrating?

13. In which medium, velocity of sound is maximum? (d) Polymer

14. Decibel is used to measure the intensity of (b) sound (d) heat

(c) 2

27. Optic fibres are mainly used for which of the following?

12. A compass needle cannot be used to detect

(a) magnetic field (c) light

(b) 45° with horizontal (d) 15° with horizontal

21. Natural rubber is the polymer of

(a) UV rays (c) X-rays

(b) common balance (d) balance wheel

(c) Water

(b) Iron (d) Silver

24. For taking the clinical photograph of bones, we ues

10. The quantity ‘weight’ is measured by

(b) Air

(b) 4

(a) mess and volume (c) number of electrons

(a) self-demagnetization (b) demagnetization due to heating (c) Earth’s magnetic field (d) effect of other magnets

(a) Metals

(a) 60° with horizontal (c) 30° with horizontal

(a) Helium (c) Argon

9. Magnetic keepers are used to protect magnets from

(a) concave lens (c) photographic camera

(a) Copper (c) Aluminium

22. In addition to hydrogen, the other abundant element present on Sun’s surface is

(b) cathode rays (d) β-rays

(a) beam balance (c) spring balance

18. The best conductor of electricity among the following is

(a) 3

8. Radiations which is not emitted during radioactivity is (a) γ-rays (c) α-rays

(b) 42000 km (d) None of these

20. The minimum number of geostationary satellites needed for uninterrupted global coverage is

7. An object which absorbs all colours and reflects none appears (b) black

(b) Concave mirror (d) Parabolic mirror

19. The angle in which a cricket ball should be hit to travel maximum horizontal distance is

6. A small drop of oil spreads over water because

(a) blue

(d) soil

17. The height of a geo-stationary satellite from the Earth’s surface is approximately

(b) pulley system (d) class II lever

4. Combustion of a candle is a/an

(a) transmission (c) reflection

(c) sound

16. Which type of mirror is used in the head lights of vehicles? (a) Plane mirror (c) Convex mirror

3. A wheel-barrow is an example of (a) class III lever (c) class I lever

(b) water

(a) α-rays (c) γ-rays

(b) β-rays (d) X-rays

29. Old-written material, which cannot be read easily, can be read by (a) cosmic rays (c) infrared rays

https://sscstudy.com/

(b) ultraviolet rays (d) None of these

https://sscstudy.com/

34

CUET (UG) Section III : General Test

39. A bomb is dropped at a point from a moving aeroplane. The pilot observes that

30. Cryogenic engines find applications in (a) rocket technology (b) frost-free refrigerators (c) sub-marine propulsion (d) researches in superconductivity

31. The best colour(s) for a sun umbrella will be (a) (b) (c) (d)

black black on top and white on the inside White on top and black on the inside printed with all the seven colours of rainbow

40. Rise of mercury in a barometer indicates

32. Which of the following laws is explained by the statement that matter can neither be created for destroyed? (a) Law of Conservation of Energy (b) Le Chatelier's principle (c) Law of Conservation of Mass (d) Law of Osmosis (b) kerosene (d) diesel

34. Which of the following is the correct combination of the inventors and the inventions? (a) Galileo Galilei - Transistors (b) Elisha Thomson -Windmill (c) JL Baird - Television (d) Sir Frank Whittle - Laser

(a) Vikram Sarabhai (b) Homi J Bhabha (c) MGK Menon (d) Raja Ramanna

(a) ocean (c) atmosphere

(b) sun (d) space

(a) frequency and wavelength (b) frequency and speed (c) wavelength and speed (d) frequency, wavelength and speed

43. Which of the following are the discoveries of Einstein? Radioactivity and photoelectric effect Radioactivity and theory of relativity Theory of relativity and photoelectric effect X-rays and photoelectric effect

44. The nuclides having the same number of neutrons but different number of protons or mass number are known as (a) isobars (c) isotopes

(b) isoelectronic (d) isotones

45. Why is there a spluttering sound when water is sprinkled on hot oil?

36. Match the following Names

(a) (b) (c) (d)

Contribution 1. Theory of numbers

B. S Chandrasekhar 2. Elementary particles statistics

Boiling point of the water is higher than that of oil Boiling point of the oil is higher than that of water Oil and water do not mix Oil and water combine to form a compound

C. S Ramanujam

3. Astronomy

D. Aryabhatta

4. Theory of black holes

46. Which falls down faster in a vacuum-a feather, a wooden ball or a steel ball?

D 3 1 3 2

47. A light year is a unit of

Codes A (a) 2 (b) 4 (c) 2 (d) 3

B 1 3 4 1

C 4 2 1 4

(a) A wooden ball (b) A feather (c) A steel ball (d) All will fall down at the same speed in a vacuum because there will be no air resistance

37. What happens to the level of mercury in the barometer tube when it is taken down a coal mine? It (a) falls (c) rises

(b) remains unaltered (d) rises and then falls

38. In a barometer, mercury is preferred over water because (a) (b) (c) (d)

(b) storm (d) Both (b) and (c)

41. The source of our energy is basically the

(a) (b) (c) (d)

35. Who among the following laid the foundation of nuclear science in the country?

A. SN Bose

(a) fair weather (c) rain

42. When light passes from air into glass it experiences change in

33. Aviation fuel for jet aeroplanes consists of purified (a) petrol (c) gasoline

(a) the bomb traverses a curved path and falls some distance behind that point (b) the bomb traverses a curved path and falls some distance ahead (c) the bomb drops vertically downward (d) the bomb remains stationary in the air for some time

mercury is a good conductor of heat mercury shines and therefore its level can be read easily mercury has high density and low vapour pressure mercury is available in pure form

(a) time (b) distance (c) speed of light (d) intensity of light

48. The technique of calculating the age of fossil organisms is (a) Radiocarbon dating (b) Electroporation (c) Counting the annual rings (d) Micro manipulation

https://sscstudy.com/

https://sscstudy.com/

GENERAL KNOWLEDGE

(c) this can damage the appliances (d) this can damage the domestic wiring due to overheating

49. A dynamo is used to convert (a) mechanical energy into electrical energy (b) electrical energy into mechanical energy (c) electrical energy into magnetic energy (d) magnetic energy into mechanical energy

58. A ship rises as it enters the sea from a river because the

50. Shaving mirror is (a) convex (c) plane

(b) concave (d) parabolic

51. Transfer of heat energy from the sun to the moon takes place by (a) radiation only (b) radiation and conduction (c) radiation and convection (d) radiation, conduction and convection

1. sky is clear 3. wind is blowing

Cusec

1. Pressure

B.

Byte

2. Intensity of earthquakes

C.

Richter 3. Rate of flow

D.

Bar

Codes A (a) 2 (b) 3 (c) 3 (d) 4

B 4 4 2 1

(a) Only 1 (c) 2 and 3

(a) Uranium-238 (c) Thorium-236

(a) (b) (c) (d)

(a) sound waves are directly transmitted (b) electrical energy is converted into sound waves directly (c) sound waves are converted into electrical energy and then reconverted into sound after transmission (d) amplification is not required at all

54. The blue colour of the water in the sea is due to (a) absorption of other colours by water molecules (b) reflection of the blue light by the impurities in sea water (c) reflection of blue sky by sea water and scattering of blue light by water molecules (d) None of the above

55. A lighted candle gets extinguished when covered with a tumbler because of adequate supply of air inadequate supply of air presence of non-luminous matter None of the above

conversion of neutrons into protons conversion of chemical energy into heat energy conversion of chemical energy into nuclear energy conversion of mass into energy

62. Which of the following is used as a moderator in nuclear reactor? (a) Ordinary water (c) Thorium

(b) Radium (d) Graphite

63. Electron was first identified by (a) JJ Thomson (c) J Kepler

(b) Daniel Rutherford (d) James Chadwick

64. Kinetic energy depends (a) (b) (c) (d)

on the mass of the moving body the velocity or speed of the moving body the pressure of the moving body both mass and velocity of the moving body

65. Match the following Device

56. A weather balloon is not fully inflated on the ground because (a) it cannot rise up in the air when fully inflated (b) it cannot withstand the outside pressure if fully inflated (c) if the balloon is fully inflated, it may not remain stable in a storm (d) the air inside the balloon expands as it rises and may burst

57. One should not connect a number of electric appliances to the same power socket because (a) the appliances will not get full voltage (b) the appliances will not get proper current

(b) Neptunium-239 (d) Plutonium-239

61. In an atomic explosion, enormous energy is released which is due to the

D 1 1 4 3

53. Microphone is a device in which

(a) (b) (c) (d)

(b) 1 and 3 (d) Only 2

60. Which of the following is good nuclear fuel ?

4. Computer memory

C 3 2 1 2

2. sky is cloudy

Which of the statements given above is/are correct?

List II

A.

(a) water in sea is hard water (b) larger quantity of water in the sea pushes the ship upwards (c) density of sea water is lower than that of the river water (d) density of sea water is higher than that of the river water

59. Consider the following statements The fog during winter season is more on the day when

52. Match the following List I

35

Conversion

A. Battery charger

1. Electrical energy into mechanical energy

B. Battery

2. Mechanical energy into electrical energy

C. Washing machine

3. Chemical energy into electrical energy

D. Dynamo

4. Electrical energy into chemical energy

Codes A (a) 4 (b) 4 (c) 1 (d) 3

https://sscstudy.com/

B 3 3 2 4

C 1 2 3 1

D 2 1 4 2

https://sscstudy.com/

36

CUET (UG) Section III : General Test

66. If you swim one km, how many miles do you swim? (a) 0.5

(b) 0.62

(c) 0.84

(d) 1.6

67. The temperature of a body is an indicator of (a) (b) (c) (d)

the total energy of the molecules of the body the average energy of the molecules of the body the total velocity of the molecules of the body the average kinetic energy of the molecules of the body

68. Why are two blankets warmer than one? (a) Two blankets have more wool and hence, provide greater warmth (b) Two blankets enclose air which does not allow the cold to penetrate (c) Two blankets compress the air in between the body and the blankets and this compression produces that (d) None of the above

69. Tiredness is felt on the top of the mountains because of (a) (b) (c) (d)

high temperature low temperature high pressure outside the body low pressure outside the body

(b) Henry Becquerel (d) Einstein

(b) nitrogen (d) helium

(b) Magnesium fluoride (d) Carbon tetrachloride

76. Radioactive materials should be kept in the container made of

(a) salt (c) citric acid

(b) vinegar (d) lactic acid

(b) candela (d) curie

83. The gas used for artificial ripening of green fruit is (a) acetylene (c) ethane

(b) ethylene (d) carbon dioxide

84. Glass is (b) crystalline solid (d) None of the above

(b) 100

(c) 77

(d) 65

(b) Tungsten (d) Zinc

87. The process of covering water pipes made of iron with zinc layer to prevent corrosion is called (a) Zinc plating (c) Vulcanization

(b) Alloy formation (d) Galvanization

88. The particle required to continue the chain process of Uranium fission is (b) Proton (d) Positron

89. Which of the following is used to increase the anti-knocking property of petrol? (a) Tetramethyl lead (c) Trimethyl lead

(b) Tetraethyl lead (d) Triethyl lead

(b) Nitric acid (d) Nitrogen dioxide

(d) Al

(a) Methane (c) Propane

(b) Ethane (d) Butane

92. Which one of the following elements is used in the manufacture of fertilizers? (a) Fluorine (c) Lead

(b) Potassium (d) Aluminium

93. Which atom is not found in proteins? (a) Cartbon (c) Phosphorus

(b) Nitrogen (d) Sulphur

94. The chemical used in a photographic plate is

78. Brass is an alloy of (a) Cu and Zn (c) Cu, Al and Zn

(a) angstrom (c) fermi

91. Which of the following is the major constituent of LPG?

75. Which among the following is a covalent compound?

77. Sweat contains water and ..... .

82. The unit of radioactivity is

(a) Ammonia (c) Nitrogen

(b) Dalton (d) Pascal

(c) Fe

(b) talc (d) serpentine

90. The chief source for the production of nitrogeneous fertilizers is

74. 'Atomic theory' of matter was given by

(b) Steel

(a) asbestos (c) gypsum

(a) Electron (c) Neutron

73. In deep-sea diving, divers use a mixture of gases consisting of oxygen and

(a) Pb

(b) tincture iodine (d) 100% ethyl alcohol

81. Cosmetic powders are prepared from

(a) Copper (c) Nichrome

(a) conc. HNO3 and conc. HCI (b) conc. HNO3 and conc. H2SO4 (c) dil, HCI and dil. HNO3 (d) conc. HCI and conc. HNO3

(a) Calcium chloride (c) Sodium chloride

(a) methylated spirit (c) 95% ethyl alcohol

86. An electric iron has a heating element made of

72. Aquaregia is a mixture of the following in the ratio 1 : 3 by volume

(a) Avogadro (c) Newton

80. Rectified spirit is

(a) 0

71. Radioactivity was discovered by

(a) hydrogen (c) argon

(b) Polypropylene (d) Polyester

85. The percentage of lead in lead pencil is

oil is very light of the diffusion of oil through the wick of the surface tension phenomenon of the capillary action phenomenon

(a) Rutherford (c) Roentgen

(a) Polyethylene (c) Polyamide

(a) super cooled liquid (c) liquid crystal

70. Oil rises up the wick in a lamp because (a) (b) (c) (d)

79. Nylon is made up of

(b) Al and Mg (d) Cu and Sn

(a) Barium chloride (c) Silver nitrate

https://sscstudy.com/

(b) Silver bromide (d) Silver chloride

https://sscstudy.com/

GENERAL KNOWLEDGE

109. Which one of the following fertilizers has more nitrogen content?

95. The element used for making solar cells is (a) Magnesium (c) Calcium

(b) Sodium (d) Silicon

96. Which of the following is used for wrapping of fractured bones? (a) White cement (c) Zinc oxide

(b) White lead (d) Plaster of Paris

97. Carbon, diamond and graphite are together called (a) isomers (c) isomorphs

(a) Urea (c) Potassium Nitrate

110. Dialysis is used for people with defective kidneys. It involves the process of (a) adsorption (c) electropheresis

(b) Silver (d) Copper

List I

B. Most abundant metal 2. Lead C. Most malleable metal 3. Aluminium D. Poorest conductor of heat

(b) tungsten (d) titanium (b) 25% (d) 35%

102. The metallic constituents of hard water are (a) (b) (c) (d)

magnesium, calcium and tin iron, tin and calcium calcium, magnesium and iron magnesium, tin and iron

104. The variety of coal in which the deposit contains recognisable traces of the original plant material is (b) lignite (d) bitumen

105. The highest grade of coal whose surface is shining and has the highest calorific value, is (b) Lignite (d) Peat

Salts of silicates Esters of heavy fatty acids Sodium or potassium salts of heavier fatty acids Mixture of glycerol and alcohols

107. In vulcanisation, natural rubber is heated with (a) Carbon (c) Silicon

(b) Coal (d) Carbon black

D 4 1 1 2

114. Of the following metals which one used to pollute the air of a big city? (a) Copper (c) Lead

(b) Chromium (d) Cadmium

115. The element present in the largest amount in rocks and minerals is (a) carbon (c) hydrogen (a) Mercury (c) Lithium

(b) silicon (d) aluminium (b) Silver (d) Lead

117. The most malleable metal is (a) Silver (c) Aluminium

(b) Gold (d) Sodium

118. Balloons are filled with (a) helium (c) nitrogen

(b) Sulphur (d) Phosphorus

108. Which of the following is used as a filler in rubber tyres? (a) Graphite (c) Coke

C 2 4 4 1

116. Which of the following is the lightest metal?

106. What are soaps? (a) (b) (c) (d)

B 3 3 2 3

(a) it exists as a solid, a liquid or a gas (b) it contains hydrogen and oxygen (c) it contains two different elements joined by chemical bonds (d) it cannot be split up into simpler substances by chemical means

(a) metel alloys (b) ceramic oxides (c) inorganic polymers (d) pure rare earth metals

(a) Bituminous (c) Anthracite

Codes A (a) 1 (b) 2 (c) 3 (d) 4

4. Silver

113. Water is a compound because

103. The high-temperature superconductors are

(a) anthracite (c) peat

List II

A. Best conductor of heat 1. Gold and electricity

(b) radium (d) uranium

101. The average salinity of sea water is (a) 20% (c) 30%

(b) manganse oxide (d) iron oxide

112. Match the following

100. The element found on the surface of the moon is (a) tin (c) tantalum

(b) osmosis (d) diffusion

(a) zinc oxide (c) iron sulphate

99. An element that does not occur in nature but can be produced artificially is (a) thorium (c) plutonium

(b) Ammonium Nitrate (d) Ammonium Phosphate

111. The substance coated on plastic tape-recorder tapes is

(b) allotropes (d) isotopes

98. Which of the following metals was first discovered by man? (a) Bronze (c) Iron

37

(b) oxygen (d) argon

119. Which of the following is used in beauty parlours for hair setting? (a) Chlorine (c) Phosphorus

https://sscstudy.com/

(b) Sulphur (d) Silicon

https://sscstudy.com/

38

CUET (UG) Section III : General Test

(b) lack of oxygen in the surroundings of sigrees (c) accumulation of smoke and dust over the sigrees (d) interrupted release of moisture from the material

120. Which of the following is the lightest gas? (a) Nitrogen (c) Ammonia

(b) Hydrogen (d) Carbon dioxide

121. Match the following

132. de Vries proposed

A.

Bronze

1. Lead, antimony, tin

B.

Brass

2. Copper, zinc, nickel

C.

German silver

3. Copper, zinc

D.

Type metal

4. Copper, tin

Codes A (a) 2 (b) 2 (c) 3 (d) 4

B 1 1 2 3

C 4 3 1 2

(a) Theory of Speciation (b) Theory of Crossing Over (c) Theory of Natural Selection (d) Theory of Mutation

Composition

Alloy

133. Haemoglobin in the blood is a complex protein rich in (a) iron (c) copper

D 3 4 4 1

134. Vitamin-C is also known as (a) amino acid (c) acetic acid

diamond is the hardest, lamp black is the softest diamond is the hardest, coke is the softest coke is the hardest, graphite is the softest diamond is the hardest, graphite is the softest

123. Which one of the following is the basis of modern periodic table? (a) Atomic mass (c) Atomic size

(b) Atomic number (d) Atomic volume

124. Which is/are the important raw material(s) required in cement industry? (a) Limestone (c) Clay

(b) Gypsum and clay (d) Limestone and clay

125. Glass is made of the mixture of (a) quartz and mica (c) sand and silicates

(b) sand and salt (d) None of these

(b) Iron (d) Lead

127. The quantum theory was first enunciated by (a) Niel Bohr (c) Max Planck

(b) Albert Einstein (d) Max Born

128. The noble gases do not react with any other atom because (a) (b) (c) (d)

they are heavy elements they are stable compounds their outermost orbits contain eight electrons None of the above

136. Clitellum is found in (a) leech (c) nereis

(b) snail (d) earthworm

137. Plant genetic material in ‘Gene-Bank’ is preserved at- 196°C in liquid nitrogen as (a) pre-mature seed (high moisture) (b) ripe fruit (c) seedling and meristem (d) mature seed (dry)

138. Which is not an anasthetic agent in surgical operations? (b) Acetone (d) Ether

139. Insulin (a) constricts blood vessels (b) stimulates lactation (c) increases blood sugar (d) decreases blood sugar

140. Animals having open circulatory system possesses (a) pseudocoel (c) coelenteron

(b) haemocoel (d) spongocoel

141. Flowers emit fragrance to (a) purify air (c) attract insects (a) pure water (c) salt water

(b) Calcium bicarbonate (d) Sodium bicarbonate

130. Which of the following is the basic characteristic of a living organism? (a) Ability to breathe (c) Ability to move

(b) Corm of arvi (d) Tuber of potato

(b) drive away flies (d) All of the above

142. Brine refers to

129. Washing soda is the common name for (a) Calcium carbonate (c) Sodium carbonate

(a) Bulb of onion (c) Tuber of sweet-potato

(a) Nitrous oxide (c) Chloroform

126. Which one of the following metals has the highest density? (a) Gold (c) Platinum

(b) ascorbic acid (d) azelaic acid

135. Which of the following is not a stem modification?

122. Among the various allotropes of carbon (a) (b) (c) (d)

(b) silver (d) gold

(b) Ability to reproduce (d) Ability to eat

131. The open ‘Sigrees’ or coal stoves often require fanning to sustain burning because of (a) tendency of carbon dioxide to form a layer along with dust smoke

(b) starch water (d) sweet water

143. Haemoglobin contains the metal (a) Copper (c) Iron

(b) Molybdenum (d) Magnesium

144. Which of the following prevents bleeding of gums? (a) Nicotinamide (c) Ascorbic acid

(b) Thiamine (d) Vitamin B

145. The locomotion of insects to the source of light is called (a) phototactic (c) phototropic

https://sscstudy.com/

(b) thermotactic (d) hydrotropic

https://sscstudy.com/

GENERAL KNOWLEDGE

146. Rabies is a (a) helminthic disease (c) bacterial disease

162. Water from soil enters into the root hairs owing to (b) viral disease (d) protozoan disease

147. Tuberculosis is transmitted through (a) droplet transmission (c) contaminated water

(b) acellular (d) multicellular

(b) calcium (d) albumin (b) Michael Faraday (d) William Harvey (b) Edward Jenner (d) SA Wakesman (b) Skin cancer (d) Bone cancer

(b) arsenic (d) asbestos

(a) eubacteria (c) protozoa

(b) cyanobacteria (d) fungi

172. The vitamin which is very liable and easily destroyed during cooking as well as storage is

(a) Vitamin A (c) Vitamin C

(b) 260 (d) 360

(b) Vitamin C (d) Vitamin K (b) Brain (d) Kidney

(b) Fat (d) Nucleic acid

(a) Vitamin A (c) Vitamin C

(b) Vitamin B (d) Vitamin D

(b) Vitamin B 6 (d) Vitamin K

173. The compound used in anti-malarial drug is (a) aspirin (c) isoprene

(b) Liver (d) Pancreas

161. Which one of the following is known as the ‘immovable property’ in the cell? (a) Carbohydrate (c) Protein

(b) Tse-tse fly (d) mosquito

(b) Algae (d) Bacteria

160. The largest organ of human body is (a) Heart (c) Liver

168. Plague is transmitted by

171. Which vitamin is provided by sunlight to the body?

159. Name the vitamin not found in any animal food. (a) Vitamin BI2 (c) Vitamin D

(b) Jonas E Salk (d) James Simpson

(b) Ghee (d) Fresh Vegetables

158. Bile is secreted by (a) Gall bladder (c) Bile duct

(b) Lead (d) Zinc

170. Blue-green algae are included in the group

157. The total number of bones in the human body is (a) 206 (c) 306

(a) JH Gibbon (c) Robert Edwards

(a) mercury (c) cadmium

156. Penicillin is isolated from (a) Fungi (c) Virus

(b) Regeneration (d) Mutation

169. Itai-itai disease is due to poisoning caused by

155. Which of the following is a good source of vitamin ‘E’? (a) Meat (c) Yellow Yolk

(a) Mercury (c) Cadmium

(a) House fly (c) rat flea

154. The radioactive Strontium-90 causes (a) Brain cancer (c) Lung cancer

(b) 100-120 days (d) 150-200 days

167. The first effective vaccine against polio was prepared by

153. Insulin was discovered by (a) F Banting (c) Ronald Ross

(b) Vitamin C (d) Vitamin E

166. Minamata disease was caused by

152. Penicillin was discovered by (a) Charles Goodyear (c) Six Alexander Fleming

(a) 100-200 days (c) 160-180 days

(a) Aestivation (c) Hibernation

151. Too much consumption of tea or coffee can result in deficiency of (a) vitamin B12 (c) vitamin C

(a) Vitamin B (c) Vitamin K

165. Dormancy period of animals during winter season is called

(b) Apple (d) Strawberry

150. Viruses are (a) cellular (c) unicellular

(b) Capillary pressure (d) Osmotic pressure

164. The average life span of red blood corpuscles is about

(b) monocyte (d) histiocyte

149. Which of the following is a parthenocarpic fruit? (a) Banana (c) Mulberry

(a) Atmospheric pressure (c) Root pressure

163. The vitamin necessary for coagulation of blood is

(b) blood transfusion (d) sexual contact

148. Antibodies are mainly synthesised from (a) megakaryocyte (c) lymphocyte

39

(b) neoprene (d) chloroquin

174. Which of the following tests helps in diagnosis of cancer? (a) X-ray (c) Blood test

(b) Urine test (d) Biopsy test

175. Which of the following vitamins is synthesised in the body by intestinal bacteria? (a) Vitamin B1 (c) Vitamin D

(b) Vitamin B 4 (d) Vitamin K

176. Which acid is produced when milk gets sour? (a) Acetic acid (c) Lactic acid

https://sscstudy.com/

(b) Tartaric acid (d) Butyric acid

https://sscstudy.com/

40

CUET (UG) Section III : General Test

177. Which of the following vitamins can be synthesised by our body? 1. Vitamin B6 3. Vitamin E

2. Vitamin D 4. Vitamin K

(a) 1 and 2 (c) 2 and 4

(a) rickets (c) xerophthalmia

(b) 2 and 3 (d) 1 and 4

(a) small pox vaccination (b) BCG injection (c) inoculation against cholera (d) None of these

179. Match the following List II

A. BCG Vaccine 1. Malaria 3. Tuberculosis

D. Penicillin

4. Rabies

Codes A (a) 3 (b) 3 (c) 4 (d) 4

B 4 4 3 3

C 1 2 1 2

D 2 1 2 1

(b) decreases (d) fluctuates

C.

Islets of Langerhans

3. At the base of the brain

D.

Gall bladder

4. Part of the pancreas

B 2 3 1 2

C 4 4 2 4

D 1 1 3 3

1. application of mathematics to the study of living things 2. application of the knowledge of the living world characteristics to the world of machines 3. study of relation of an organism to its environment 4. study of laws of life (c) Only 3

(b) tuberculosis (d) cholera

189. Calcium content is maximum in (b) maize (d) wheat

(b) Turmeric (d) Serpentina

191. Acid rain is formed due to contribution from the following pair of gases

183. Bionics is the science of

(d) 1 and 4

184. Salt is used in our food because (a) it makes the food tasty (b) we sweat and lose salt (c) it helps in production of hydrochloric acid required to digest the food (d) it makes water absorb food particles better

185. The vitamin which is generally excreted by humans in urine is (a) vitamin A (c) vitamin C

2. By the side of the kidneys

(a) Tulsi (c) Garlic

kills the microorganisms present in them helps to improve their taste and flavour drains moisture to inhibit growth of microorganisms mixes well with the fruits

(b) Only 2

Pituitary gland

190. Which of the following has been found useful in keeping cholesterol level down?

(b) 2 and 3 (d) 1, 2 and 3

182. Sugar syrup is used for the preservation of fruits because it

(a) 1 and 2

B.

(a) bajra (c) sorghum

181. During sleep a man's blood pressure

(a) (b) (c) (d)

1. Below the right portion of the liver

(a) kwashiorkar (c) diphtheria

1. Tse tse fly-Sleeping sickness 2. Female-anopheles mosquito-Malaria 3. Sand fly-Kala azar

(a) increases (c) remains constant

List II

Adrenal gland

188. A girl ate sweets while fanning the flies away. Due to this, she suffered from a disease diagnosed as

180. Which of the following are correctly matched?

(a) 1 and 2 (c) 1 and 3

List I A.

Codes A (a) 3 (b) 2 (c) 4 (d) 1

B. BPL Vaccine 2. Sore throat C. Chloroquin

(b) scurvy (d) None of these

187. Match the following

178. The most important requirement for the people of a flood hit area is

List I

186. A person who lives exclusively on milk, egg and bread is likey to become a victim of

(b) vitamin D (d) None of these

(a) Methane and Ozone (b) Oxygen and Nitrous oxide (c) Methane and Sulphur dioxide (d) Nitrogen oxides and Sulphur dioxide

192. Most serious environmental effect posed by hazardous waste is (a) air pollution (b) contamination of ground water (c) increased use of land for landfils (d) destruction of habitat

193. Which of the following is not a primary contribute to the greenhouse effect? (a) Carbon dioxide (c) Argon

(b) Carbon mono oxide (d) Methane gas

194. The increase in the concentration of CO2 in our environment in last fifty years, since 1960 is about (a) 20%

(b) 10%

(c) 14%

(d) 6%

195. How the biological oxygen demand gets affected with the increased presence of organic matter in water? (a) The oxygen demand increases (b) The oxygen demand decreases (c) The oxygen demand remain unchanged (d) None of the above

https://sscstudy.com/

https://sscstudy.com/

GENERAL KNOWLEDGE

196. The headquarters of International Whaling Commission is located at (a) New York (c) Perth

197. Increased defoliation in plants is caused by (a) ozone depletion (c) global warming

(b) acid rains (d) ground pollution

198. National Biodiversity Authority is located at (a) Shimla (c) Chennai

(b) Hyderabad (d) Kolkata

199. According to World Health Organisation which is the most polluted city in the world? (a) Los Angeles, California (b) Mexico City, Mexico (c) New Delhi, India (d) Shanghai, China (b) composting (d) incineration

(b) Methane (d) Ozone

202. Milankovitch cycles (a) refer to shifts in the temperature of surface water in the middle latitudes of Pacific Ocean (b) are changes in Earth’s rotation and orbit around the Sun that may trigger climate variation (c) describe the timing of the Northern lights in the thermosphere (d) describe upwelling and downwelling in the ocean

203. Keeling’s reports from Mauna Loa demonstrated (a) an increase in Carbon dioxide from the 1950 to present (b) that sediments deposited on the seafloor can yield dues about past climates (c) Carbon dioxide gradient decreases from urban to rural (d) the presence of El-Nino

204. Kyoto is to carbon dioxide as montreal is to (a) living modified organisms (b) persistent organic pollutants (c) chlorofluoro carbons (d) ozone

205. Which of the following terms best applies to the practice of environ- mental science? (a) Highly specialised (c) Abstract

(b) Integrative (d) Theoretical

206. Salinisation occurs when the irrigation water accumulate in the soil evaporates, leaving behind salts and minerals. What are the effects of Salinisation on the irrigated land? (a) It greatly increases the crop production (b) It makes some soils impermeable (c) It raises the water table (d) It fills the air spaces in the soil

208. Consider the following 1. Photosynthesis 2. Respiration 3. Decay of organic matter 4. Volcanic action Which of the above add carbon dioxide to the carbon cycle on the Earth? (a) 1 and 4 (c) 2, 3 and 4

(a) Indian Wild Buffalo (c) Indian Wild Boar

201. Which among the following greenhouse gas has the highest radiative forces? (a) Carbon dioxide (c) Chlorofluoro carbons

(b) Botanical Garden (d) Wildlife Sanctuary

(b) 2 and 3 (d) All of these

209. A sandy and saline area in the natural habitat of an Indian animal species. The animal has no predators in that area but it’s existen is threatened due to the destruction of it’s habitat. Which one of the following could be that animal?

200. The best way to dispose plant waste is (a) burning (c) dumping

207. Which one of the following is not a site for in-site method of conservation of flora? (a) Biosphere Reserve (c) National Park

(b) Kyoto (d) Cambridge

41

(b) Indian Wild Ass (d) Indian Gazelle

210. The Himalayan Range is very rich in species diversity. Which one among the following is the most appropriate reason for this phenomena? (a) It has a high rainfall that supports luxuriant vegetation (b) It is a confluence of different bio geographical zones (c) Exotic and invasive species have not been it introduced in this region (d) It has less human interference

211. Consider the following 1. Carbon dioxide 2. Oxides of Nitrogen 3. Oxides of Sulphur Which of the above is/are the emission(s) from coal combustion zone at thermal power stations? (a) Only 1 (c) 1 and 3

(b) 2 and 3 (d) 2 and 3

212. The formation of ozone hole in the Antarctica region has been a cause of concern, what could be the reason for the formation of this hole? (a) Presence of prominent tropospheric turbulence and inflow of chlorofluoro carbons (b) Presence of prominent polar front and stratospheric clouds and inflow of chlorofluoro carbons. (c) Absence of polar front and stratospheric clouds and inflow of methane and chloro flouro carbons (d) increased temperature at polar region due to global warming

213. Regarding carbon credits, which one of the following statements is not correct? (a) The carbon credit system was ratified in conjunction with the Kyoto protocol (b) Carbon credits are awarded to countries or group that have reduced green house gases below their emission quota. (c) The goal of the carbon credit system is to limit the increase of carbon dioxide emission. (d) Carbon credits are traded at a price fined from time to time by United Nations Environment Programme

https://sscstudy.com/

https://sscstudy.com/

42

CUET (UG) Section III : General Test

214. Mon-863 is a variety of maize. It was in the news from which one of the following reasons? (a) It is genetically modified dwarf variety, which is resistant to drought (b) It is genetically modified variety, which is pest resistant (c) It is genetically modified variety with ten times higher protein content than regular maize crop (d) It is a genetically modified variety used exclusively for bio-fuel production

(d) Ministry of Rural Development

219. Which one of the following is also known as Top slip? (a) Simlipal National Park (b) Periyar Wildlife Sanctuary (c) Manjira Wildlife Sanctuary (d) Indira Gandhi Wildlife Sanctuary and National Park

220. Which one of the following is located in Bastar region? (a) Bandhavgarh National Park (b) Randeli Sanctuary (c) Rajaji National Park (d) Indravati National Park

215. Where was the world summit on sustainable development (Rio-10) held? (a) Davos (c) Johannesburg

(b) Nova Seotia (d) Shanghai

216. Out of all the Biosphere reserves in India, four have been recognised on the world network by UNESCO. Which one of the following is not one of them? (a) Gulf of Mannar (c) Nanda Devi

(b) Kangchenjunga (d) Sunderbans

217. Which one among the following has the maximum numbers of National Park? (a) Andaman and Nicobar Islands (b) Arunachal Pradesh (c) Asom (d) Meghalaya

221. Which one of the following country is the first country in the world to propose a carbon tax for its people to address global warming? (a) Australia (c) Japan

(b) Germany (d) New Zealand

222. Amongst the following Indian states, which one has minimum total forest cover? (a) Sikkim (c) Haryana

(b) Goa (d) Kerala

223. Genetic engineering approval commission, whose permission is required for cultivation of any genetically modified crop such as Bt-Cotton in India, is under Union Ministry of

218. Which one of the following Union Ministries is implementing the biodiesel mission (as nodal ministry)?

(a) Agriculture (b) Environment and forest (c) Commerce and industry (d) Rural development

(a) Ministry of Agriculture (b) Ministry of Science and Technology (c) Ministry of New and Renewable Energy

ANSWERS 1. 11. 21. 31. 41. 51. 61. 71. 81. 91. 101. 111. 121. 131. 141. 151. 161. 171. 181. 191. 201. 211. 221.

(d) (b) (a) (c) (b) (a) (d) (b) (b) (d) (d) (d) (d) (a) (c) (a) (a) (d) (d) (d) (b) (d) (d)

2. 12. 22. 32. 42. 52. 62. 72. 82. 92. 102. 112. 122. 132. 142. 152. 162. 172. 182. 192. 202. 212. 222.

(a) (c) (a) (c) (c) (b) (d) (a) (d) (b) (c) (d) (d) (d) (c) (c) (b) (c) (c) (b) (b) (b) (c)

3. 13. 23. 33. 43. 53. 63. 73. 83. 93. 103. 113. 123. 133. 143. 153. 163. 173. 183. 193. 203. 213. 223.

(d) (a) (d) (b) (c) (c) (a) (d) (b) (c) (b) (c) (b) (a) (c) (a) (c) (d) (a) (c) (b) (d) (b)

4. 14. 24. 34. 44. 54. 64. 74. 84. 94. 104. 114. 124. 134. 144. 154. 164. 174. 184. 194. 204. 214.

(d) (b) (c) (c) (d) (c) (d) (b) (a) (b) (c) (c) (d) (b) (c) (d) (b) (d) (c) (c) (c) (b)

5. 15. 25. 35. 45. 55. 65. 75. 85. 95. 105. 115. 125. 135. 145. 155. 165. 175. 185. 195. 205. 215.

(b) (c) (c) (b) (b) (b) (a) (d) (a) (b) (c) (b) (c) (c) (a) (d) (c) (d) (c) (a) (b) (c)

6. 16. 26. 36. 46. 56. 66. 76. 86. 96. 106. 116. 126. 136. 146. 156. 166. 176. 186. 196. 206. 216.

(d) (b) (b) (c) (d) (d) (b) (a) (c) (d) (c) (c) (c) (d) (b) (a) (a) (c) (b) (d) (d) (b)

https://sscstudy.com/

7. 17. 27. 37. 47. 57. 67. 77. 87. 97. 107. 117. 127. 137. 147. 157. 167. 177. 187. 197. 207. 217.

(b) (a) (a) (c) (b) (d) (d) (a) (d) (b) (b) (b) (c) (d) (a) (a) (b) (c) (b) (a) (b) (a)

8. 18. 28. 38. 48. 58. 68. 78. 88. 98. 108. 118. 128. 138. 148. 158. 168. 178. 188. 198. 208. 218. 228.

(b) (d) (c) (c) (a) (d) (b) (a) (c) (d) (d) (a) (c) (b) (c) (a) (c) (c) (d) (c) (c) (d) ()

9. 19. 29. 39. 49. 59. 69. 79. 89. 99. 109. 119. 129. 139. 149. 159. 169. 179. 189. 199. 209. 219. 229.

(a) (b) (c) (b) (a) (a) (d) (c) (b) (c) (a) (b) (c) (d) (a) (c) (c) (a) (b) (b) (b) (d) ()

10. 20. 30. 40. 50. 60. 70. 80. 90. 100. 110. 120. 130. 140. 150. 160. 170. 180. 190. 200. 210. 220. 230.

(c) (c) (a) (a) (b) (d) (d) (c) (a) (d) (b) (b) (c) (b) (b) (c) (b) (d) (c) (b) (b) (d) ()

https://sscstudy.com/

GENERAL KNOWLEDGE

43

CHAPTER 03

Computer Awareness The word ‘computer’ has been originated from the Latin word ‘compute’ which means to calculate. Computer is an electronic device which receives data, processes it and gives meaningful result. To know the work of computer, some important terms are as follows Ä Data The word data has been derived from Latin word ‘datum’ means fact. It is a collection of facts and figures which are not in directly usable form. Ä Processing This is an act of performing some actions to convert it into usable form. Ä Information It is processed data or final result which is in directly usable form and conveys some meaningful results. Ä Instruction It is a type of action to be taken on data. Ä Basic Operations Input, storing, processing, output, control.

Components of a Computer Input Unit Ä Ä

Ä

It accepts data and instructions from the user. It converts the accepted instructions into the machine language. It gives converted instructions to CPU for processing.

Central Processing Unit (CPU) It performs all processing work of computer. Ä It is also known as brain of computer. Ä It controls all the parts of computer system. Central Processing Unit (CPU) has three components Ä

Control Unit It gives command to transfer data from the input devices to memory and from memory to arithmetic logic unit. It transfers the results from memory to output unit.

Memory It temporarily/permanently holds the data received from input device. It holds the data being processed and intermediate results being generated. It holds the system software and the application software in use. Memory is mainly classified into two types (a) Random Access Memory (RAM) All active programs and data are stored in RAM so that they may readily available and easily accessed by the CPU. (b) Read Only Memory (ROM) The data stored on ROM cannot be changed. It is non-volatile which means that it can keep its contents even without a power source.

Output Unit Ä Ä

Ä

It accepts processed data from CPU. It converts results from machine language into simple language. It displays results to user.

Software It is a set of instructions that directs the computer to process information. It can be classified into three types.

System Software

Arithmetic and Logic Unit It performs all arithmetic and logic operations. It consists of number of small locations, termed as registers. It has two parts (a) Arithmetic Section All complex arithmetic expressions are performed by repetitive combination of these operations. (b) Logic Section It performs all logic operations like comparing, selecting and merging of data.

It coordinates the operation of the various hardware components of the computer. e.g. DOS, Windows 95/98/2000/ME and UNIX.

Application Software It is a set of program designed for specific uses or applications e.g. MS Word, MS Excel, etc.

https://sscstudy.com/

https://sscstudy.com/

44

CUET (UG) Section III : General Test

Utilities Software

Year

Inventor

Invention

Utility programs are those that are very often requested by many applications programs. It is used to debug the software errors, correct the data through the editor, sort data, etc, e.g. Pkzip, Antivirus software, etc.

1642

Blaise Pascal

First Calculating Machine

1671

Gottfried Von Leibnitz

Calculator (Modified Pascal’s machine)

1801

Joseph Jacquard

Card of holes for weaving patterns

1823-34

Charles Babbage

Difference Engine and Analytical Engine

1880

Herman Hollerith

Tabulating machine using punch cards

Bits, Bytes and Words Unit

Size

Description

Bit

One binary digit

Stores either a binary 0 or 1

Byte

Eight bits

One character

Lady Ada Lovelace

Binary Number System

Word

16 to 64 bits

One character

1930

MARK-I

Kilobyte (KB)

1 thousand bytes

About one page of double-spaced text

Howard Aitken and Grace Hopper (IBM)

1937-38

Megabyte (MB)

1 million bytes

About the size of a short book

Dr John Vircent Atanassoff

ABC (Atanasoff-Berry-Computer)

Gigabyte (GB)

1 billion bytes

1000 short books

1946

JP Eckert and JW Mauchly

ENIAC (Electronic Numerical Integrated And Calculator)

Terabyte (TB)

1 trillion bytes

An entire library

1940s

1 quadrillion bytes

Just about all the libraries in the US

H Goldshine, AW Burks and John Von Neumann

Stored Program Concept

Petabyte (PB)

1947-49

John Von Neumann

EDSAC (Electronic Delayed Storage Automatic Computer)

1950

Moor School in USA

EDVAC (Electronic Discrete Variable Automatic Computer)

1951

Eckert and JW Mauchly

UNIVAC-I (Universal Automatic Computer)

Historical Overview of Computer Year

Inventor

Invention

16th Century

China

Abacus

1617

John Napier

Napier’s Bones

Computer Generations Generation

Year

Switching Device

Storage Device

Speed

Operating System

Language

Application

First

1949-54

Vacuum Tubes/ Electronic Valves

Acoustic delay lines and later magnetic drum, 1KB memory,

333 micro seconds

Mainly batch

Machine and assembly languages, Simple monitors

Mostly scientific later simple business systems

Second

1954-64

Transistors

Magnetic core, main memory, tape and disk peripheral memory, 100 KB main memory

10 micro seconds

Multi-bag remaining, Time sharing

High level languages, Fortran, Cobol, Algol, Batch, Operating system.

Extensive business applications, Engineering design optimisation scientific research

Third

1964-80

Integrated Circuits

High speed magnetic cores Large disks 100 MB, 1 MB main memory

100 nano seconds

Real time, Time sharing

Fortran IV, Cobol 68 PI/I, Time sharing Operating system,

Database management systems, On-line systems

Fourth

1980-till now

Large scale integrated circuits. Microprocessors (LSI)

Semi-conductor memory, Winchester disk, 10 MB main memory, 1000 MB disks

300 nano seconds

Time sharing, networks

Fortran 77, Pascal, ADA, Cobol-74

Personal computers, Distributed systems Integrated CAD/ CAM Real time control, Graphics oriented systems

https://sscstudy.com/

https://sscstudy.com/

GENERAL KNOWLEDGE

Types of Computer Based on Uses Computers can be classified based on their uses in following ways :

Analog Computer It measures pressure, temperature, length, etc, of physical quantities and converts into its numerical values. They are mainly used for scientific and engineering purposes.

Digital Computer It is used in counting of numbers, words or other special symbols. It is also used in fuel control of automobiles and banking processes.

Hybrid Computer

45

A modern computer has the following hardwares : (i) Motherboard (ii) Electric Supply (iii) Video Display Controller (iv) Removable Media Devices (v) Secondary Storage (vi) Sound Card (vii) Peripherals

Input Devices We give unprocessed data to computer with the help of input devices and get output after the data has been processed. We can understand that output and the input devices may be hand operated or automated. The act of processing is mainly regulated by CPU.

It is the combined form of analog and digital computer. Informations are continuously converted into data that are received in the form of input. Output may be in form of analog and digital. They are used in factories and machines.

Hand Operated Input Devices

Based on Sizes

Concept Keyboard

Computers can be classified based on their size in following ways

Concept keyboard is related to a special function or item. The labels of picture or text may reside above its buttons.

Micro Computer It is the smallest computer in which ALU and CPU are based on the same chip.

Personal Computer It is single user system used generally in business.

Mini Computer It is more powerful than micro computer and can be used by many users at the same time. It processes data very fast.

Mainframe Computer They are giant in size and of highly storage capacity. They can process an ample amount of data very rapidly. They are used in banks, large organizations and public sectors.

Super Computer They are capable of processing the huge amount of data in fraction of seconds. They are used in very complex calculations.

Hardware All the components of computer, parts and devices that we can touch are known as Hardware. Computer is composed of several physical parts of the computer hardware on which a number of softwares are installed to do the desired work of operating system and operator.

Keyboard and Mouse are the most commonly used hand operated input devices. Some of the hand operated input devices along with keyboard and mouse are as follows :

Track Ball It is used as an alternate of mouse. The user has to move its ball in order to utilise it that causes the pointer to move on the screen.

Joystick It is very popular among the people fond of computer games, but today different types of game controllers are replacing them gradually. Joystick is used to control the machine such as crane in the process of building constructions.

Digital Camera Digital camera can take the images and also records the videos. The images and videos taken by digital cameras are recorded in files. These files can get copied into the computer that can be further edited for many purposes.

Microphone Microphone is used to input the sound. It can be used to enter text in any word processing application with voice recognition software in the computer.

Touch Screen There is a network of light rays in touch sensitive visual display unit or in screen that recognises the touch. Touch screen is used in many mobile phones. They are often used in cash machines and shopping centres.

https://sscstudy.com/

https://sscstudy.com/

46

CUET (UG) Section III : General Test

Video Digital

Projector

Video digital takes the images from video camera or television and stores it into the computer for further use. The video sequences taken by video digital are mainly used in multimedia presentation.

Projector is that hardware device which projects an image on the flat surface like computer screen. These devices are generally used in meetings, seminars and presentations because each person can easily see the image projected by it sitting in a big room.

Scanner Scanner is used to convert an image into its digital form. It is similar to photocopier but it makes digital copy of an image instead of its hard copy. It is used to scan the text with the software optical character recognition.

Graphic Tablet There is a flat pad on the graphic tablet upon which a user writes with the help of a special pen. Pictures can also be drawn on the screen using that pen in this way.

Keyboard Keyboard is the most commonly used input device in computer. There are three types of keys on the keyboard (a) Alphanumeric keys (b) Special keys (c) Function keys Alphanumeric keys include the whole alphabets from A to Z and digits from 0 to 9 and different characters such as space, / × ` ; ! @ # $ % ^ & − + = \ { } [ ]. Special Keys include ENTER, Backspace, Delete, Shift, Capslock, Ctrl, Alt, etc. Keys from F1 to F12 in the keyboard are known as Function Keys.

Sound Card It is also known as Sound Board or Audio Card. Sound card is an expansion card or unified circuit that enables computer to produce sound through speaker or headphone.

Speaker It is the device which presents the sound produced during any programme. This is connected to sound card and gives the sound produced by card in the form of output.

Video Card It is also known as Graphics Card, Video Board or Video Controller. This is an internal circuit board that allows the image displaying devices like monitor to display the images.

Internet Ä

Mouse

Ä

It is the most popular point and draw input device which is connected to a terminal or microcomputer through a cable wire.

Ä

Output Devices The means through which computer gives output are known as Output Devices. Some of the most popular output devices are as follows : 1. Monitor 2. Printer 3. Projector 4. Sound Card 5. Speaker 6. Video Card

Ä

Ä

Ä

Monitor Video display screen fixed in hard layer and computer is also known as Monitor or Video Display Terminal. Monitor means only that devices in which there is no any electronic device other than the device required for showing the image.

Ä

Ä

Ä

Printer Printer is that external hardware device which gives the output in printed format after receiving data from computer. This is such an operated external device used with computer that prints text, image or photo.

Ä

Ä

An Internet is a global connection of computers linked together by cables and telephones and makes the communication possible among them. For such communication, the address of the destination and a safe means of moving data are required. The rules governing the sending and receiving of data on the Internet are implemented in two parts as Transmission Control Protocol (TCP) and Internet Protocol (IP). Transmission Control Protocol divides the data into little data packets. The rule of Internet Protocol here is to put destination addressing information on such packets. Hosts are in general, individual machines at a particular location. Domains are general category that a computer on the internet belongs to. E-mail is mainly used for sending electronic piece of text. Use net and news groups are set up by people sharing common interests ranging from current topics to cultural heritages. Telnet is a program that allows an Internet host computer to become a terminal of another host on the Internet. Gopher displays a set of resources on the Internet in the form of menus or lists of items.

https://sscstudy.com/

https://sscstudy.com/

GENERAL KNOWLEDGE

Ä

Ä

Ä

Ä

Ä

Ä

Ä

Ä

Ä

Ä

Hypertext Markup Language (HTML) is a language to create documents for world wide web. Standard Generalised Markup Language (SGML) is used to define the structure and manage the content of a digital document. World Wide Web (www) is a giant collection of hypertext pages on the Internet. Hyper Text Transfer Protocol (HTTP) is used to manage all the linkings between one hypertext document and another. On the Internet, a home page may be related to a single person, a specific subject, or to a corporation. Uniform Resource Locator (URL) is a way to avail of Internet resources. Browser is an application program which is used for exploring Internet resources. File Transfer Protocol (FTP) is used to access a remote Internet host. Modem is a device allowing a computer to transmit information over a telephone line. It acts as a modulator and demodulator while the process of transmitting and receiving. Wide Area Information Service (WAIS) is a type of service to access text databases or libraries on the Internet.

Some antivirus programs are capable of ‘On the Fly’ scanning of received e-mails along with scanning of the open files. This process is known as ‘On-Access’ scanning. Antivirus software does not alter the efficiency of the host software for the communication of viruses. These softwares need to be updated regularly so that the new viruses can be detected. The loss incurred by the viruses can also be negated by the regular backup of data on other mediums. If any data get misplaced due to this virus then we can retrack it by backup. Important Facts Related to Computer Virus ●

● ●









Computer Virus A computer virus is a computer program that can copy itself and infect a computer. A virus can spread from one computer to another when its host is taken to the target computer, for instance because a user sent it over a network or the Internet, or carried it on a removable medium such as a floppy disk, CD, DVD or USB drive. Viruses can increase their chances of spreading to other computers by infecting files on a network file system or a file system that is accessed by another computer.

Antivirus Software and Other Preventive Measures Antivirus software can destroy the viruses by finding them. The antivirus software uses two common methods in order to detect viruses. 1. By using the Virus Signature Definitions list 2. By using Heuristics Algorithm

47





The Creeper virus was first detected on ARPANET, the forerunner of the Internet, in the early 1970s. The Reaper Program was created to delete creeper. The actual term ‘virus’ was first used in David Gerrold’s 1972 novel, when HARLIE was one. A program called ‘‘Elk Cloner’’ was the first computer virus to appear ‘‘in the wild’’ that is, outside the single computer or lab where it was created. It attached itself to the APPLE DOS 3.3 operating system and spread via floppy disk. The first PC virus in the wild was a boot sector virus ‘‘Brain’’ created in 1986. With the spread of personal computer, traditional computer viruses emerged in 1980. Bulletin board-driven software sharing contributed directly to the spread of Trojan horse programs. Shareware and boot leg software were equally common vectors for viruses on BBS. Macro Viruses have become common since the mid 1990. Most of these viruses are written in the scripting languages for Microsoft Programs such as word and Excel and spread throughout Microsoft Office by infecting documents and spreadsheets. Viruses that spread using cross site scripting were first reported in 2002 and were academically demonstrated in 2005. There have been multiple instances of the cross site scripting viruses in the wild, exploiting websites such as My Space and Yahoo. The most popular five computer virus came into light so far as follows : (i) I Love You (2000) It spreads through e-mails. (ii) Code Red (2001) It affects windows server. (iii) Nimada (2001) It is the most rampant virus. (iv) Melissa (1999) It spreads through Word Macro and e-mails. (v) Sasser (2004) It spreads by attaching to port 445 on the network windows system.

https://sscstudy.com/

https://sscstudy.com/

48

CUET (UG) Section III : General Test

Practice Questions 1. Assembly language is

13. Which of the following displays programs that are currently running? A single click on one of the program icons will bring the window up?

(a) Machine language (b) High-level programming language (c) Low-level programming language (d) Language for assembling computers

(a) Menu bar (c) Title bar

2. Which of the following is not one of the syntax rules? (a) The order in which you list the function’s arguments (b) The precedence of the arguments (c) Whether or not the function has arguments (d) Properly spelling the function’s name

3. The binary system is a number system to the base (a) 2 (c) 8

(b) 4 (d) 10

(b) Printer (d) Interpreter

(b) Input (d) Memory

6. Which of the following is hardware and not software? (a) Excel (c) Operating System

(b) Printer driver (d) CPU

(b) Loader (d) Assembler

10. In Excel, this is a prerecorded formula that provides a shortcut for complex calculations (b) Data Series (d) Field

(a) the desktop (b) the clipboard (c) microsoft word (d) paste

(d) #

(a) data into charts and vice-versa (b) charts into charts (c) data into charts (d) charts into most advanced charts

18. Excel can display upto ....... fields of a data form. (b) 32 (d) 128

(b) exclamation (!) (d) ampersand (&)

20. Which menu option can be used to split windows into two?

(a) auto formatting (c) changing fonts

(b) View > Windows > Split (d) View > Split

(b) applying styles (d) All of these

22. Which one is the example of spreadsheet package? (a) Visicalc (c) Ada

11. The speed at which the monitor accepts data is called

12. The copy command saves to

(c) =

21. Data can be arranged in a worksheet in an easy to understand manner using

(b) Data Series (d) Text

(b) interlacing (d) scanning

(b) View sheet (d) Chart sheet

17. Using the chart wizard, we can quickly and easily turn our

(a) Format > Windows (c) Windows > Split

9. This is not a function category in Excel

(a) bandwidth (c) response time

(b) @

(a) apostrophe (’) (c) hash (#)

8. To be able to “boot”, the computer must have a(n)

(a) Value (c) Function

(a) View chart (c) Embedded chart

19. A numeric value can be treated as a label value if it proceeds with

(a) read and write information (b) only read information (c) only write information (d) read, write and rewrite information

(a) Logical (c) Financial

(b) Data Consolidation (d) Data Filter

15. When we create a chart on a separate sheet in the same workbook, it is called a

(a) 16 (c) 64

7. On a CD-RW, you can

(a) Compiler (c) Operating System

(a) Data Validation (c) Data Form

(a) $

5. Which contents are lost when the computer turns off? (a) Storage (c) Output

14. ……… is a process of summarising data from more than one source.

16. In order to tell Excel that we are entering a formula in cell we must begin the with an operator such as

4. Which of the following is not an example of hardware? (a) Scanner (c) Monitor

(b) Task bar (d) Status bar

(b) Unify (d) Snowball

23. Legends are used in (a) Chart Wizard (c) Auto Sum

(b) Function Wizard (d) Validation

24. Which is the comparison operator? (a) = (c) *

(b) / (d) None of these

25. From which tool bar we can change chart type? (a) Chart Tool Bar (c) Formula Bar

https://sscstudy.com/

(b) Formatting Tool Bar (d) Clipboard Bar

https://sscstudy.com/

49

GENERAL KNOWLEDGE

26. Which sign is used to specify a cell range? (a) : (colon) (c) * (asterisk)

35. Which protocol provides e-mail facility among different hosts?

(b) / (slash) (d) - (hyphen)

27. In which tool bar will you find Auto Sum short button? (a) Formatting Bar (c) Clipboard Bar

(a) SMTP (c) TELNET

36. 192.9.200.153 is an (a) Ethernet Address (b) IP Address (c) Computer Address (d) Any of these

(b) Standard Bar (d) Formula Bar

28. In which table, we find the pivot table command? (a) Tools (c) Insert

(b) Data (d) Window

37. The storage that supplements the primary internal storage of a computer is known as

29. Which menu keeps the Goal seek command? (a) Data (c) Insert

(a) secondary storage (b) primary storage (c) back-end storage (d) background storage

(b) Tools (d) Format

30. By which option, we can show and print the gridlines? (a) Page Setup (c) Auto Format

38. A browser is a

(b) Format (d) Style

(a) line that delineates the edge of a programme window (b) software programme especially designed for viewing web pages on the internet (c) tool for resizing the window (d) device used for modifying the window

31. What are ‘Laptops’? (a) Computers used in clinical laboratories (b) A computer manufactured by Compaq (c) A computer having voice recognition system (d) Lightweight computers, small enough to fit in a small suitcase

39. A folder A. is an organizational tool used to store files B. is synonymous with directory, but folder is the newer preferred term C. contains data (a) Only A (b) Only B (c) Only C (d) A and C

32. The basic architecture of computer was developed by (a) John Von Neumann (c) Blaise Pascal

(b) FTP (d) SNMP

(b) Charles Babbage (d) Garden Moore

33. What does CPU stand for? (a) Central Processing Unit (b) Central Process Unit (c) Central Printing Unit (d) Central Peripheral Unit

40. The part of a computer system containing the circuitry that does the adding, subtracting, multiplying, dividing and comparing, called as

34. Ethernet, LAN, Token Ring and Token Bus are types of (a) WAN (b) communication channels (c) LAN (d) physical medium

(a) Arithmetic Logic Unit (b) Control (c) Memory (d) CPU

ANSWERS 1. 11. 21. 31.

(c) (a) (d) (d)

2. 12. 22. 32.

(b) (b) (b) (a)

3. 13. 23. 33.

(a) (d) (c) (a)

4. 14. 24. 34.

(d) (b) (d) (b)

5. 15. 25. 35.

(d) (d) (b) (a)

6. 16. 26. 36.

(d) (c) (a) (a)

https://sscstudy.com/

7. 17. 27. 37.

(d) (c) (d) (c)

8. 18. 28. 38.

(c) (b) (b) (b)

9. 19. 29. 39.

(b) (a) (b) (d)

10. 20. 30. 40.

(c) (c) (d) (a)

https://sscstudy.com/

3

GENERAL MENTAL ABILITY

CHAPTER 01

Analogy ‘Analogy’ means ‘similarity’, i.e. having similar features. Analogy test is an inference of similarity between two relations. In it, words are used between which specific logical relations exist. It is a comparison between two elements that are usually thought to be different, but have something in common. Questions on analogy test the ability of a candidate to understand the relationship between two given elements and apply the same relationship to find the one asked in the question. Analogy can be of four types from which questions are generally asked in competitive exams.

1. Word Analogy In questions based on word analogy, two words are given related to each other in some way. The candidate has to find that relationship and apply the same to choose the correct alternative which is either in the form of a word or a pair of words. Some of the important word relationships are as following SN

Types of Relationship

Examples

1.

Synonymous relationship

Slim : Thin, Meet : Contact

2.

Opposite relationship

Poor : Rich, Never : Always

3.

Cause and effect relationship

Shoot : Kill, Fast : Hunger

4.

Game and place of playing Tennis : Court, Boxing : Ring

5.

Worker & Tool

Writer : Pen, Hunter : Gun

6.

Tool & Object

Pen : Paper, Eraser : Paper

7.

Whole & Part

Car : Wheel

8.

Part & Whole

Handle : Chair

9.

Word and Intensity

Hot : Warm

10.

Classification

Cow : Mammal, Ice : Water

11.

Function

Surgeon : Operates

12.

Male and female

Bull : Cow, Brother : Sister

13.

Sequential

Morning : Afternoon, Nine : Ten

14.

Finished Product & Raw Bread : Flour, Fabric : Yarn Material

SN

Types of Relationship

Examples

15.

Quantity & Unit

Mass : Kilogram, Length : Metre

16.

Utility relationship

Chair : Sleeping

17.

Comparative

Black : Coal, Red : Blood

18.

Animal and Sound

Snake : Hiss, Duck : Quack

19.

Place relationship

Delhi : Red fort, Punjab : Ludhiana

20.

Purpose relationship

Sandpaper : Abrasion

21.

Specialist & Subject

Heart : Cardiologist, Skin : Dermatologist

22.

Individual and Young ones Man : Child, Cow : Calf

23.

Country-Capital

India : New Delhi, Japan : Tokyo

24.

Country-Currency

Iraq : Dinar, Russia : Rouble

25.

Instrument & Measurement

Barometer : Pressure, Hygrometer : Humidity

26.

Individual & Group

Sailor : Crew, Cattle : Herd

27.

Individual & Dwelling

Dog : Kennel, Horse : Stable

28.

Study & Topic

Ornithology : Birds, Seismology : Earthquakes

29.

Worker & Working place

Chef : Kitchen, Teacher : School

30.

Worker & Product

Farmer : Crop, Poet : Poem, Chef : Food

Sitting,

Bed

:

Ex. 1 Find out the missing word from the given alternatives, which bears the same relationship to the third/fourth word, as the first two bear Cattle : Herd :: Sheep : ? (a) Flock

(b) Swarm

(c) Shoal

(d) Mob

Sol. (a) Herd is a group of cattle. Similarly, flock is a collection of sheep.

Ex. 2 Cat is related to mouse in the same way as Bird is related to …… . (a) Worm (c) Animal

(b) Snake (d) Crow

Sol. (a) In this case, cat feeds on Mouse. Similarly, bird feeds on worm.

https://sscstudy.com/

https://sscstudy.com/

4

CUET (UG) Section III : General Test

Ex. 3 Find a pair similar to ‘Arrow : Bow’ from the following. (a) Football : Hand (c) Bullet : Rifle

Ex. 5 Out of the four given alternatives, choose that alternative as your answer which is similar to the given words. L B W : Slip : Cover

(b) Salad : Knife (d) Smoke : Water

(a) Dence (c) Corner

Sol. (c) As, arrow is released with the help of bow, similarly a bullet is released with the help of rifle.

Sol. (d) LBW, slip and cover, all are related to cricket. Similarly, chinaman is a term used in cricket.

Ex. 4 Select the alternative that does not have a similar relationship as the given pair. Forward : Backward (a) Hope : Despair (c) Anger : Wrath

(b) Dribble (d) Chinaman

Ex. 6 Find the similarity in the following. Elephant, Camel, Buffalo, Giraffe (a) The milk produced by all of them cannot be consumed by people (b) All of them have horns (c) None of them are mammals (d) The young ones of all of them are called calf

(b) Love : Hate (d) Light : Dark

Sol. (c) In the given pair both the words are opposite to each other. But in pair (c) i.e. Anger : Wrath, both the words have similar meaning.

Sol. (d) The young ones of elephant, camel, buffalo and giraffe are called calf.

2. Letter Analogy In letter or alphabet analogy, both of question and answer pairs consists of letters or alphabets. The candidate has to examine the question pair and find the relationship on the basis of which either a group of letters or a pair of group of letters is to be find out. To solve these questions the candidate has to remember the positional values of letters in English alphabetical order. Forward

1

2

3

4

5

6

7

8

9

10

11

12

13

14

15

16

17

18

19

20

21

22

23

24

25

26

Alphabet

A

B

C

D

E

F

G

H

I

J

K

L

M

N

O

P

Q

R

S

T

U

V

W

X

Y

Z

Backward 26

25

24

23

22

21

20

19

18

17

16

15

14

13

12

11

10

9

8

7

6

5

4

3

2

1

Ex. 7 Complete the second pair in the same way as first pair CD : GH :: LM : ? (a) DC

(b) EG

(c) AB

(d) IK

Sol. (c) As, in the first term i.e CD, the letters are consecutive and the second letter comes after the first letter and so is the term GH. Similarly, LM and AB follows the same pattern.

Ex. 8 Choose the pair from the given alternatives which is similar to the given pair BCD : EFG :: ? : ? (a) L M N : O P Q (c) A B C : M N O

Similarly, L M N

+3 +3 +3

E

F

+3 +3 +3

O

G

P Q

3. Number Analogy In this analogy, either a number or a group of numbers are related to each other in the first pair according to a certain pattern. The candidates has to find that pattern and choose the related number or a pair of numbers.

Ex. 9 Choose the number from the given alternatives which will complete the second pair in the same way as first pair. 3 : 11 :: 7 : ? (a) 22

(b) 29

(c) 18

Similarly, 72 + 2 = 51

Ex. 10 Find the set among the four sets which is like the given set ( 13 : 20 : 27). (a) (3 : 11 : 18) (c) (18 : 27 : 72)

Sol. (b) As, 13

(b) (18 : 25 : 32) (d) (7 : 14 : 28) 20

+7

27

Similarly, 18

+7

25 +7

32 +7

4. Mixed Analogy

(b) P R S : S T V (d) X W V : Z Y Z

Sol. (a) As, B C D

Sol. (d) As, 32 + 2 = 11.

(d) 51

In this type of analogy, letters and numbers are given, following a certain relationship. The relationship may be based on the position of alphabets or letters, sum or product of the position of alphabet or letters etc. The candidate is required to understand the relationship and complete the analogy.

Ex. 11 Complete the second pair in the same way as first NEWS : 14, 5, 23, 19 : : PAPER : ? (a) 16, 5, 16, 1, 18 (c) 16, 1, 16, 5, 18

(b) 18, 5, 16, 1, 16 (d) 32, 2, 32, 10, 36

Sol. (a) Here, each letter is assigned its positional value in forward order So, PAPER → 16, 1, 16, 5, 18

https://sscstudy.com/

https://sscstudy.com/

GENERAL MENTAL ABILITY

5

Practice Questions 1. Goitre is related to Iodine as Anaemia is related to (a) Vitamin

(b) Blood

(c) Iron

(d) Weakness

2. Needle is related to Thread as Pen is related to (a) Ink

(b) Cap

(c) Paper

(d) Wood

3. If H2 O : Hydrogen, then KOH : …… (a) Cobalt (c) Potassium

14. Ohm : Watt : Ampere (a) Electricity (b) Volt (a) Finger

(b) Lungs

16. Nose : Eyes : Ears

questions, there is a certain relationship between two given words on the left side of :: and one word is given on the right side of :: while another words is to be found from the given alternatives having the same relation with this word as the words of the given pair bear.

17. Dhoni : Yuvraj : Dravid

(b) Anesthesia (d) Burn (d) Legs

6. Contemporary : Historic :: … : Ancient (a) Past

(b) Classic

(c) Modern

(d) Future

(c) Farmer

(d) Crop

7. Blunt : Sharp :: Sow : (a) Reap

(b) Seeds

8. Circle : Circumference :: Square : (a) Sides

(b) Area

(c) Perimeter (d) Diagonal

Directions (Q. Nos. 9-11) The following questions consist of two words each that have a certain relationship to each other followed by four lettered pair of words. Choose the pair having the same relationship.

Directions (Q. Nos. 19-25) In these questions, there is a certain relationship between the two terms to the left of :: and the same relationship exists between the two terms to the right. One of the terms either to the left or to the right is missing. Find out this term. 19. KMF : LLH :: RMS : ? (a) SLR (a) BKPR

(b) Hen : Mew (d) Sheep : Bleat

(a) C H M R (a) STRUIF

as the given pair. Inside : Outside (b) Sun : Star (d) White : Black

Directions (Q. Nos. 13-15) Out of the four given alternatives, choose that alternative as your answer which is similar to the given words. 13. Pen : Pencil : Rubber (b) Cell

(b) MNCC

(c) HRYY

(d) HMNC

(c) Pillow

(b) AFKP

(c) YTOJ

(d) ZUPK

(b) MLKJ

(c) PLHD

(d) XVTR

(b) D N I S

(c) H D K P

(d) C F M S

24. FLOWER : REWOLF :: FRUITS : ?

(b) Yard : Alley (d) Vein : Artery

12. Select the alternative that has a different relationship

(a) Day : Night (c) Light : Dark

(d) SUS

23. A F K P : B G L Q :: ? : D I N S

11. Brook : River (a) Pen : Paper (c) Path : Highway

(c) SSU

22. JLNP : OMKI :: SUWY : ? (a) FGHI

(a) Flies : Squeak (c) Fox : Snout

(b) SLU

20. FJUL : BOQQ :: LHRX : ?

(a) AEIM

(b) Socks : Feet (d) Legs : Shoes

10. Ass : Bray

(a) Page

(a) They are means of entertainment (b) They are means of mass media (c) All are public undertakings (d) They give word wide news

21. EJOT : VQLG :: BGLQ : ?

9. Hands : Gloves (a) Bank : Fog (c) Fish : Water

(b) Athlete (d) Singers

18. Cinema : Press : Television

5. Bird : Wings :: Fish : ? (c) Fins

(d) Kidney

(a) They are internal part of human body (b) They are not the external part of human body (c) They are parts of the body below waist (d) They are parts of the body above neck (a) Cricketers (c) Politicians

4. Consolation : Grief :: Sedative : …

(b) Head

(c) Heart

15. Ear : Nose : Lips

Directions (Q. Nos. 4-8) In each of the following

(a) Gills

(d) Light

Directions (Q. Nos. 16-18) Find out the common feature among the given words and pick the alternative that mentions the properties common to the given words.

(b) Phosphorous (d) Krypton

(a) Chloroform (c) Pain

(c) Hour

(d) TV

(b) STUIRF

(c) STIURF

(d) STUIFR

25. B J N T : C I O S :: D H P V : ? (a) E G Q U (c) E L P V

(b) E L Q W (d) E I O U

Directions (Q. Nos. 26-30) Each of these questions consists of a pair of letters or groups having a certain relationship followed by alternatives. Select the pair similarly related to the given pair. 26. BC : FG (a) PQ : ST (c) AD : PQ

https://sscstudy.com/

(b) HI : LM (d) JK : LM

https://sscstudy.com/

6

CUET (UG) Section III : General Test

27. MN : OP

42. 7 : 24

(a) AD : GH (c) QR : ST

(a) 30 : 100 (c) 19 : 58

(b) AB : PQ (d) RS : TV

28. CEH : IKN (a) ACD : FHJ (c) EFH : KMN

Directions (Q. Nos. 43-45) In each of the following questions, choose that set of numbers from the four alternatives, which is similar to the given set.

(b) CDF : IJK (d) OQT : UWZ

43. (2, 14, 16)

29. LLAMS : SMALL (a) SRENID : DINERS (c) BARK : KRAB

31. 18 : 30 :: 36 : ?

(a) (26, 32, 42) (c) (24, 16, 0) (a) 353 (c) 606

46. C : 16 :: F : ?

(b) 62

(c) 64

(d) 66

(b) 465

(c) 546

(d) 3125

(b) 336

(c) 343

(a) 30 (c) 49

(b) 216

(a) 7×19 (c) 14×15

48. MK : (c) 222

(d) 240

(b) 90

(c) 92

(d) 100

(b) 81

(c) 90

(d) 99

36. 7 : 56 :: 9 : ? (a) 63

49.

(b) 132 (d) 140

50.

(d) 4

E 22 B : :: :? H 19 I 25 18 17 (d) 19

(b)

51. MLO : 121114 :: GJL : ? (a) 192022 (a) 36

(b) 125 : 5 (d) 729 : 81

40. 5 : 35 (b) 9 : 45 (d) 3 : 24 (b) 9 : 243 (d) 5 : 75

(c) 160813

(d) 2218175

(b) 38

(c) 27

(d) 24

53. STBP : 1920216 :: MNGO : ? (a) 1314715

(b) 1413715

(c) 5173141

(d) 3114715

(c) 35

(d) 18

(c) 20

(d) 17

54. X M A E : 16 :: V T N G : ? (a) 21

55.

41. 8 : 256

(b) 060911

52. GREAT : 25 :: NUMBER : ?

39. 27 : 9

(a) 7 : 343 (c) 10 : 500

(b) 3

(a)

consists of a pair of numbers that have a certain relationship to each other followed by four other pairs of numbers given as alternatives. Select the pair having the same relation.

(a) 7 : 77 (c) 11 : 55

T X : 2 :: :? J H

24 17 3 (c) 7

(b) 225 (d) 292

Directions (Q. Nos. 39-42) Each of these questions

(a) 64 : 8 (c) 135 : 15

100 81 81 (d) 100

(b)

(a) 2 23 (c) 7

38. 48 : 122 :: 168 : ? (a) 215 (c) 290

169 :: JH : ? 121

100 64 64 (c) 120

37. 42 : 56 :: 110 : ? (a) 18 (c) 136

(b) 5×17 (d) 6×18

(a)

35. 42 : 56 :: 72 : ? (a) 81

(b) 40 (d) 50

47. M×N : 13×14 :: F×R : ?

(d) 350

34. 68 : 130 :: ? : 350 (a) 210

(b) 329 (d) 520

Directions (Q. Nos. 46-55) In each of these questions, find out the missing term from the given alternatives.

33. 6 : 222 :: 7 : ? (a) 210

(b) (34, 24, 14) (d) (42, 34, 16)

45. (525, 813, 714)

32. 3 : 243 :: 5 : ? (a) 425

(b) (2, 9, 16) (d) (4, 16, 18)

44. (32, 24, 8) (b) AMONG : OMNAG (d) ENTRY : RNYET

Directions (Q. Nos. 31-38) In these questions, there is a certain relationship between two given numbers on the left side of :: and one number is given on the right side of :: while another number is to be found from the given alternatives. Find the missing term. (a) 54

(a) (2, 7, 8) (c) (3, 21, 24)

(b) CHART : TRACH (d) TREE : EERT

30. LOWER : WORLE (a) GLAZE : AGELZ (c) WORDS : ROSWD

(b) 23 : 72 (d) 11 : 43

G ×Q Q (a) 25

https://sscstudy.com/

(b) 17

: 20 ::

B×V V

:?

(b) 27

https://sscstudy.com/

ANSWERS 1. 11. 21. 31. 41. 51.

(c) (c) (c) (d) (c) (b)

2. 12. 22. 32. 42. 52.

(a) (b) (d) (d) (b) (a)

3. 13. 23. 33. 43. 53.

(c) (a) (a) (d) (a) (a)

4. 14. 24. 34. 44. 54.

5. 15. 25. 35. 45. 55.

(c) (b) (c) (c) (c) (d)

6. 16. 26. 36. 46.

(c) (a) (a) (b) (c) (a)

7. 17. 27. 37. 47.

(c) (d) (b) (c) (c)

(a) (a) (c) (b) (d)

8. 18. 28. 38. 48.

9. 19. 29. 39. 49.

(c) (b) (d) (c) (a)

10. 20. 30. 40. 50.

(d) (b) (a) (d) (b)

(d) (d) (c) (a) (b)

Hints & Solutions Similarly, S U W

Y

X

1. (c) Goitre is caused by the deficiency of

13. (a) As, Pen, Pencil and Rubber are

Iodine. Similarly, Anaemia is caused by the deficiency of Iron.

stationary items. Similarly, page is also a stationary item

–1 –1

2. (a) Thread is required by the Needle to

14. (b) As, Ohm, Watt, Ampere are

–1

function. Similarly, Ink is required by the Pen to function. H2O → Hydrogen 3. (c) As,

measurement units of electricity. Similarly, volt is the measurement unit of electricity.

Here, ‘H’ signifies Hydrogen. Similarly, KOH → Potassium

V

T

R

–1

23. (a) As, A

15. (a) Ear, Nose and Lips all are external parts of the human body and so is the finger.

16. (d) ‘Nose’, ‘Eyes’ and ‘Ears’ are the parts of human body above neck.

F K

P

Similarly, C H M R +1 +1 +1 +1

+1 +1 +1 +1

B G L

D

Q

I

N

S

24. (c) As, F L O W E R

R EW O L F

F R U I T S

S T I U R F

17. (a) It is clear that the common feature, Here, ‘K’ signifies Potassium.

4. (c) As, cosolation soothe the grief, Similarly,

among Dhoni, Yuvraj and Dravid is that they are cricketers.

sedative alleviate the pain.

18. (b) All are means of mass media.

5. (c) As, bird uses wings to fly. Similarly, fish

19. (b) As, K M F

uses fins to swim.

6. (c) Contemporary and historic are opposite to each other, similarly modern and ancient are opposite to each other.

L

8. (c) As, the boundary of the circle is known

Gloves are worn. Similarly, Legs are the parts of the body where Shoes are worn.

10. (d) Bray is the sound produced by Ass. Similarly, Bleat is the sound produced by Sheep.

11. (c) As, River is bigger form of brook. Similarly, Highway is bigger form of path.

12. (b) In the given pair, both the words are opposite to each other. But in pair (b) both the words, i.e. sun and star are similar in nature.

U

L

Similarly,

U

L +5

B O Q Q Similarly, L –4

as circumference, similary the boundary to the square is known as perimeter.

9. (d) Hands are the parts of the body where

S

+5 –4

+2

+1 –1

H

J

–4

7. (a) As, blunt and sharp are opposite to each other, similarly sow and reap are opposite to each other.

L

20. (d) As, F

Similarly, R M S

+2

+1 –1

H R +5 –4

H M N

As, B

25. (a)

X

C

C

O

S

Similarly, D H

P

V

E

26. (b) As, B

So, B G L Q → Y T O J

+4

22. (d) As, N

T

+1 –1

I

+1 –1

alphabet which occupies the same place as the letter in reverse English alphabetical order.

L

N

+5

21. (c) Here, each letter is replaced by an

J

J

+1 –1

P

O

M

–1 –1 –1 –1

https://sscstudy.com/

K

I

C +4

+1

G Q

–1

U

Similarly, H +4

I +4

F

G

L

M

27. (c) As, M

N

Similarly, Q

R

+2

O

+2

P

+2

S

+2

T

https://sscstudy.com/

8

CUET (UG) Section III : General Test

28. (d) As, C E H

Similarly, O Q

+6 +6 +6

I

K

T

+6 +6 +6

N

39. (d) As, 27 : 9 ⇒ 33 : 32 3

similarly, 729 : 81 ⇒ 9 : 9

U W Z

40. (a) The first number is multiplied by the next prime number to obtain the second number. As, 5 : 5 × 7 . Similarly, 7 : 7 × 11

29. (a) The second group is a meaningful word formed by reversing the order of the letters of the first group.

41. (c) As, 8 : 256 ⇒ 8 :

30. (c)

50. (b) Here, each letter is assigned its positional value in reverse English alphabetical order. E 22 As, = H 19 B 25 Similarly, = 18 I

2

As, L

O

W

E

R Similarly, 10 : 500 ⇒

W

O

R

L

E

Similarly, W

O

R

D

S

83 2

10 :

10 2

−1 → 11 12 L  −1 15 O  → 14 i.e. MLO : 121114

42. (b) As, 7 : 24 ⇒ 7 : (7 × 3) + 3

31. (d) As, 18

O

:

30

S

W

:

Similarly, 36

(18×2)–6

32. (d) As, 3

As, 2

D

−1 → 09 10 J 

: 3125

: 243 Similarly, 5 ×34

7, (7 3 + 7 ) = 350

–8

0 –16

Similarly, F is the 6th letter of the English alphabet and ( 6 + 1)2 = 49 :

Similarly, 9

90

9×(9+1)

37. (b) As, 42 = 7 × 6 and 56 = 7 × ( 6 + 2 )

the English alphabets, respectively. So, M × N corresponds to 13 × 14. Similarly, F and R are 6th and 18th letters of the English alphabet, respectively. So, F × R corresponds to 6 × 18.

English alphabet, respectively and

(72–1) Similarly, 168

122 (7+4)2 +1

:

290

(132–1) (13+4)2 +1

P

Similarly, M N G O

19 20 2 16

13 14 7 15

T

B

As, X

M

A

E

24

13

1

5

1

5

2+4 1+3

= 11 × (10 + 2 ) = 11 × 12 = 132 :

53. (a) Here, each letter is represented by its position in English alphabetical order.

47. (d) M and N are 13th and 14th letters of

48. (a) M and K are 13th and 11th letters of

As, 48

52. (a) As, GREAT has 5 letters, it is represented by square of 5 i.e 25. similarly, NUMBER has 6 letters. So, it is represented by square of 6 i.e. 36

54. (d)

Similarly, 110 = 11 × 10 So, the required number

38. (c)

−1 12 L  → 11 GJL : 060911

As, S

alphabet and ( 3 + 1) = 16

So, missing number = 9 × 10 = 90

7×(7+1)

–16

16



2

72 = 8 × 9

56

8 Similarly, 24

24

46. (c) C is the 3rd letter of the English

35. (b) As, 42 = 6 × 7: 56 = 7 × 8,

:

As, 32

Similarly, 606 = 6 + 0 + 6 = 12

Similarly, 222 = 6 3 + 6 : 350 = 7 3 + 7

36. (c) As, 7

44. (c)

813 ⇒ 8 + 1 + 3 = 12 714 ⇒ 7 + 1 + 4 = 12

34. (c) As, 68 = 43 + 4:130 = 53 + 5

Similarly,

×8

45. (c) As, 525 ⇒ 5 + 2 + 5 = 12

33. (d) As, 6, ( 63 + 6) = 222 Similarly,

24

×7 ×8

–8

×54

21

×7

66

(36×2)–6

16 Similarly, 3

14

−1 07 G  → 06

Similarly,

Similarly, 23 : 72 ⇒ 23 : (23 × 3) + 3

43. (c) R

−1 13 M  → 12

51. (b) As, 3

13 2 112

=

169 121

Similarly, J and H are 10th and 8th letters of the 10 2 100 English alphabet, respectively and 2 = . 64 8

49. (b) T and J are 20th and 10th letters of the English alphabet, respectively and 20 ÷10 = 2 . Similarly, X and H are 24th and 8th letters of the English alphabet, respectively and 24 ÷ 8 = 3.

https://sscstudy.com/

6 + 4 + 1 + 5 =16 Similarly, V N G T 22

20

14

2+2 2+0 1+4

7 7

4 + 2 + 5 + 7 =18

55. (a) Here, each letter is assigned its position value in reverse English alphabetical series. G × Q 20 × 10 As, = = 20 Q 10 B × V 25 × 5 Similarly, = = 25 V 5

https://sscstudy.com/

9

General Mental Ability

CHAPTER 02

Classification Classification means to group various objects on the basis of a certain common quality that they possess and spot the odd one out. In this test, questions are designed to test the candidate’s ability of classifying given objects and find the one that doesn’t belong to the group. In this type of questions, all the items except one, follow a certain rule or pattern or they possess some common quality or characteristics between them and one which is odd does not possess the common quality or characteristics. The candidate has to find that odd item. Classification can be of three types from which questions are generally asked in competitive exams, i.e. 1. Word Classification

2. Alphabet Classification

3. Number Classification

(a) B D

(b) I K

(c) P N

(d) S U

Sol. (c) Here, B P

+2

D, I

+2

–2

N, S

+2

K U

All except PN follows the same pattern. So, PN is the odd one.

Ex. 4 Choose the odd letter group. (a) BDGK B

(b) JLOS

(c) NPSW

(d) MORU

D

J

K

G

L

S

O

;

In this classification, either words or pairs of words are given and the candidate has to find out the basis of similarity between them so as to separate the odd word or pair of words.

Ex. 1 Choose the odd one out. (b) Magnetic Storage (d) Petabyte

Sol. (d) Except petabyte, all others are storage divices, but petabyte is a unit of measurement used to measure the data.

Ex. 2 Choose the odd pair of words. (a) Cow : Calf (c) Lion : Cub

Ex. 3 Choose the group of letters which is different from others.

Sol. (d) Here,

1. Word Classification

(a) Optical disc (c) Solid State Drive

group and only then one can be able to solve such questions.

(b) Dog : Bitch (d) Tortoise : Turtle

Sol. (b) Except in Dog : Bitch, all other pairs have the second word as the young one of the first. In Dog:Bitch, second is the female of the first.

+2

N

+3

P +2

+4

W

S +3

+2

+4

;

M

+3

O +2

+4

U

R +3

+3

All except MORU, follows the same pattern. So, MORU is the odd one.

3. Number Classification In this type, certain numbers are given either in simple group or pairs, out of which all except one share some common property and hence are alike while one is different. This number is to be chosen as the answer.

Ex. 5 Choose the number which is different from others in the group. (a) 12

(b) 25

(c) 37

(d) 49

Sol. (c) Clearly, 37 is the only prime number in the group.

2. Alphabet Classification

Ex. 6 Choose the number/pair/group which is different from others.

(Letter Classification) Sometimes we are given a group of alphabets and we are asked to find the odd letter group out of them. The candidate must understand the relationship within each

(a) 50-66

(b) 22-38

(c) 64-80

(d) 63-77

Sol. (d) Clearly, in each of the pairs except (d), the second number is 16 more than the first.

https://sscstudy.com/

https://sscstudy.com/

10

General Mental Ability

Practice Questions Directions (Q. Nos. 1-16) In each of these questions, four words have been given out of which three are alike in some manner while the fourth one is different. Choose out the odd one. 1. (a) Pear

(b) Apple

2. (a) Football

(b) Volleyball (c) Cricket

3. (a) Tortoise (c) Snake

4. (a) Violet (c) Green

5. (a) Tea (c) Rubber

6. (a) Bajra (c) Rice

(c) Guava

(d) Orange (d) Chess

9. (a) Dagger (c) Knife

10. (a) King (c) Bishop

11. (a) Feeling (c) Anxiety

12. (a) Aluminum (c) Copper

13. (a) Silent valley (c) Indus valley

14. (a) Spade (c) Axe

15. (a) Garlic (c) Ginger

16. (a) Manure (c) Ammonia

(c) Table : Wood

(c) Shallow : Deep

(b) Cinchona (d) Chalk

23. (a) Sky : Cloud

(b) Mustard (d) Wheat

24. (a) Tree : Stem

(c) Cupboard : Almirah (c) Chair : Sofa

25. (a) Shirt : Dress (c) Mango : Fruit

26. (a) Pen - Ink (c) Bottle - Tonic

27. (a) Bulb - Filament (c) Leaf - Flower

(b) Hammer (d) Sword

(b) Wrong : Right (d) Genuine : Real

(b) Sulphur & Phosphorous : Match stick (c) Hydrogen & Oxygen : Water (d) Magnesium & Silver : Stainless steel (c) Food : Hunger

(b) Quintals (d) Kilometres

(b) Pages : Note book (d) Students : Class

21. (a) Nitrogen & oxygen : Air

22. (a) Water : Thirst

(b) Talent : Education (d) Air : Suffocation (b) Purse : Wallet (d) Chair : Stool (b) Face: Eye (d) Plant : Flower (b) Boy : Girl (d) Table : Furniture (b) Pot - Water (d) Cylinder - Air (b) Pencil - Lead (d) Fruit - Seed

28. Which among the following is a mismatched pair of religion practiced and its holy book?

(b) Queen (d) Minister

(a) Islam : Quran (c) Jainism : Upanishad

(b) Joy (d) Anger

(b) Sikhism : Guru Granth Saheb (d) Christianity : Bible

Directions (Q.Nos. 29-43) In each of these questions, some groups of letters are given, all of which except one, share a common similarity while one is different. Choose the odd one out.

(b) Iron (d) Brass (b) Silicon valley (d) Damodar valley (b) Knife (d) Blacksmith

29. (a) A

(b) O

(c) U

(d) Y

30. (a) GE

(b) MK

(c) WU

(d) QN

31. (a) AE

(b) AI

(c) IO

(d) EI

32. (a) ABD

(b) FGI

(c) LMO

(d) STU

(b) Chilli (d) Potato

33. (a) BDW

(b) DFU

(c) FHS

(d) GIQ

34. (a) VRT

(b) RMP

(c) YUW

(d) FBD

(b) Nitrogen (d) Urea

35. (a) DEB

(b) HIF

(c) NOL

(d) RTP

36. (a) HJN

(b) JLP

(c) PRU

(d) QSW

37. (a) CHG

(b) LMM

(c) BBC

(d) HEG

38. 39. 40. 41.

(b) OUT (b) K O M (b) B X C Y

(c) BED (c) R V T (c) D V E W (b) B O D Q (d) E S G T (b) U W X V (d) C E F D

(d) LOT (d) W Y X (d) M P O N

Directions (Q. Nos. 17-27) In these questions, four pairs of words are given, out of which the words in three pairs bear a certain common relationship. Choose the pair in which the words are differently related. 17. (a) Shoe : Leather

20. (a) Dim : Bright

(b) Blue (d) White

(b) Chatrapati Shivaji Terminus (c) Taj Mahal (d) Sun Temple (c) Grams

(c) Letters : Word

(b) Duck (d) Whale

7. (a) Rashtrapati Bhavan

8. (a) Tonnes

19. (a) Principal : School

(b) Iron : Axe (d) Jewellery : Gold

18. (a) Saturn : Planet (b) Sun : Star (c) Milky Way : Constellation (d) Titan : Satellite

(a) RAT (a) D H F (a) A Y B Z (a) C P B O (c) A N H U 42. (a) P R S Q (c) L O N M

43. (a) M O N J K (c) G M R G S

https://sscstudy.com/

(b) A C A Z V (d) M E T E I

https://sscstudy.com/

GENERAL MENTAL ABILITY

Directions (Q. Nos. 44-52) In each of the following questions, four numbers are given. Out of these, three are alike in a certain way but the rest one is different. Choose the one that is different from the rest four. 44. 45. 46. 47. 48.

Directions (Q.Nos. 53-60) Choose the odd numeral pair/group in each of the following questions. 53. (a) 34 : 43

(b) 27

(c) 29

(d) 37

(a) 29

(b) 53

(c) 85

(d) 125

(a) 37

(b) 45

(c) 49

(d) 65

55. (a) 12 : 72

(a) 35

(b) 49

(c) 50

(d) 63

(c) 60 : 74

54. (a) 12 : 96

(b) 168 (d) 380

56. (a) 18 : 108

49. (a) 126

(b) 217 (d) 513

57. (a) 62 : 37

(b) 17 (d) 37

58. (a) 34 : 12

51. (a) 4867

(b) 5555

59. (a) 7 : 18

(c) 6243

(d) 6655

(c) 23

52. (a) 272

(b) 210

(c) 240

(d) 304

(b) 13:117 (d) 16 : 128

(c) 15 : 120

(a) 120 (c) 290

50. (a) 13

(b) 55 : 62 (d) 83 : 92

(c) 62 : 71

(a) 17

(c) 345

11

(b) 24 : 36 (d) 84 : 96 (b) 23 : 138 (d) 26 : 156

(c) 20 : 80

(b) 74 : 40 (d) 103 : 78

(c) 85 : 60

(b) 43 : 30 (d) 62 : 19

(c) 52 : 21

(b) 9 : 26 (d) 13 : 42

(c) 11 : 36

60. (a) 133 : 98

(b) 150 : 115 (d) 188 : 153

(c) 182 : 140

ANSWERS 1. 11. 21. 31. 41. 51.

(d) (a) (d) (b) (d) (d)

2. 12. 22. 32. 42. 52.

(d) (d) (b) (d) (c) (c)

3. 13. 23. 33. 43. 53.

(d) (b) (a) (d) (a) (b)

4. 14. 24. 34. 44. 54.

(d) (d) (c) (b) (b) (b)

5. 15. 25. 35. 45. 55.

(d) (b) (b) (d) (d) (c)

6. 16. 26. 36. 46. 56.

(b) (b) (a) (c) (c) (c)

7. 17. 27. 37. 47. 57.

(a) (b) (c) (d) (c) (b)

8. 18. 28. 38. 48. 58.

(d) (c) (c) (b) (d) (d)

9. 19. 29. 39. 49. 59.

(b) (a) (d) (d) (c) (c)

10. 20. 30. 40. 50. 60.

(d) (d) (d) (d) (c) (c)

Hints & Solutions 1. (d) Orange is the only citrus fruit in the

11. (a) Except feeling, all others denote

group.

various types of feelings.

2. (d) Except chess, all others are outdoor

12. (d) Only brass is an alloy and all others

games.

are metal. Hence, brass is different.

3. (d) Except whale, all others lay eggs.

13. (b) Except silicon valley all others real

4. (d) All except white, are the colours of the

places, while bangalore is known as silicon valley.

rainbow.

5. (d) All except chalk, are obtained from plants.

14. (d) All except Blacksmith, are tools.

6. (b) All except Mustard, are food grains

15. (b) All except Chilli, are modified stems.

while Mustard is an oilseed.

7. (a) Except Rashtrapati Bhavan, all others

16. (b) All except Nitrogen, are used as fertilisers.

are declared as word heritage site by UNESCO.

17. (b) In all pairs except Iron : Axe, first is a product made from the second.

8. (d) All except kilometers are units to

18. (c) Saturn is a planet, sun is a star and titan is a satellite, but milky way is not a constellation it is a galaxy.

measure weights.

9. (b) All except Hammer, are sharp-edged and have a cutting action.

10. (d) All except Minister, are chessmen.

19. (a) Except (a), in all other pairs second word is the collection of first i.e. a note book is

https://sscstudy.com/

a collection of pages, a word is a collection of letters and a class is a collection of students.

20. (d) In all pairs except Genuine : Real, the two words are antonyms of each other. 21. (d) The major components of air are Nitrogen & Oxygen, of match stick are Sulphur & Phosphorous and of water are Hydrogen & Oxygen. But Stainless Steel does not contain Magnesium & Silver. 22. (b) In all other except Talent : Education, lack of first causes the second. 23. (a) In all pairs except Sky : Cloud, the two words denote things which serve the same purpose. 24. (c) In all pairs except Chair : Sofa, second is a part of the first. 25. (b) In all pairs except Boy : Girl, second denotes the class to which the first belongs. 26. (a) Except Pen-Ink in all other, the second is kept in the first

https://sscstudy.com/

12

CUET (UG) Section III : General Test

27. (c) In all pairs except Leaf-Flower, the second is contained in the first.

40. (d) Here,

51. (d) Here, +1

28. (c) The holy book of Jainism is Agam Literature. Hence, Jainism : Upanished, is odd one out.

A

Y

B

Z

B

+1

29. (d) All letters except Y are vowels.

X

D

V

C

Y

6243 ⇒ 6 + 2 + 4 + 3 = 15 i.e divisible 5 6655 ⇒ 6 + 6 + 5 + 5 = 22 i.e not divisible by 5

+1 +1

30. (d) Here,

4867 ⇒ 4 + 8 + 6 + 7 = 25 i.e divisible by 5 5555 ⇒ 5 + 5 + 5 + 5 = 20 i.e divisible by 5

+1

E

All except 6655 follow the same pattern. Hence, 6655 is the odd one.

–2

W

M

P

O

N

52. (c) Here,

−3

−2 −2 −2 G → E, M → K, W → U, Q → N

+1

+2

272 ⇒ 2 + 7 + 2 = 11 i.e prime number 210 ⇒ 2 + 1 + 0 = 3 i.e prime number

All except QN follow the same pattern. All except MPON follow the same pattern.

So, QN is the odd one.

31. (b) Except AI, all other groups contain two consecutive vowels of the English alphabet. +1

+2

+1

+2

+2

+1

+1

32. (d) Here, A → B → D, F → G → I, +1

41. (d) Here, number of alphabets between alternate letters is similar –1

L → M → O, S → T → U C

all except STU follow the same pattern. So, STU is the odd one +2

P

33. (d) Here, B → D ←→ W, opposite

+2

opposite

B

+2

O

B

–1

opposite

+2

240 ⇒ 2 + 4 + 0 = 6 i.e composite number 304 ⇒ 3 + 0 + 4 = 7 i.e prime number

So, MPON is the odd one.

O

A

F → H ←→ S,

N

53. (b) In all other pairs, second number is 9 more than the first except 55 : 62.

Q

54. (b) Here,

+2 +7

D → F ←→ U,

D

H

All except 240 follow the same pattern. Hence, 240 is the odd one.

+1

U

E

+7

S

:

12

G

T

96

13

: 120

16

×8

+2

15

: 117 ×9

: 128

+2

G → I → Q

Hence, ESGT is the odd one.

All excpet GIQ follow the same pattern. So, GIQ is the odd one. +4 +5 +2 +2 34.(b) Here, V ← R → T, R ← M → P,

42. (c) Here, P

R

+2 +2 D Y ← U → W, F ← B → +4

+4

All except RMP follow the same pattern. So, RMP is the odd one. +1

+1

−3

L

−3

All except RTP follow the same pattern. So, RTP is the odd one. +4

+2

+2

+4

36. (c) Here, H → J → N, J → L → P, +3

+4

+2 +2 S → W P → R → U, Q →

S

Q

U

W

O

–1

+1

X

V

N

M

C

E

+1

F

D

+1

All except LONM follow the same pattern. Hence, LONM is the odd one.

43. (a) Except MONJK, in all other alternatives atleast one letter is repeated.

44. (b) Each of the numbers except 27, is a prime number.

All except PRU follow the same pattern. So, PRU is the odd one.

45. (d) 125 is the only number in the group

37. (d) HEG is the only group of letters that contains a vowel while others consists consonants only.

46. (c) 49 is the only perfect square in the

38. (b) OUT is the only group of letters that

divisible by 7.

contains two vowels, while other consists one vowel.

48. (d) Each of the numbers except 380, is

−2

+4 39. (d) Here, D → H → F, −2

+4 K → O → M, −2

+4 R → V → T, +2

−1

W → Y → X All except WYX follow the same pattern. So, WYX is the odd one

×8

all except 13 : 117 follows the same pattern. Hence, 13 : 117 is the odd one

55. (c) The HCF of two numbers in all other pairs is 12 except 60 : 74.

+1

+1

35. (d) Here, D → E → B, H → I → F, +1

+1

+1

−3

+2 −4 T → P N → O → L , R →

×8

which is a perfect cube. group.

47. (c) Each of the numbers except 50 is

either one less or one more than the square of a certain number.

56. (c) In all pairs except 20 : 80, second number is 6 times the first. 57. (b) In all pairs except 74 : 40, the difference between the two numbers is 25. 58. (d) Here 34 : 12 ⇒ 3 + 4 + 1 + 2 = 10 43 : 30 ⇒ 4 + 3 + 3 + 0 = 10 52 : 21 ⇒ 5 + 2 + 2 + 1 = 10 62 : 19 ⇒ 6 + 2 + 1 + 9 = 18 All except 62 : 19 follow the same pattern. Hence, 62 : 19 is the odd one.

59. (c) 7

: 18

(7×4)–10

13

: 42

(13×4)–10

9

:

26

(9×4)–10

11 :

36

(11×4)–8

49. (c) Each of the numbers except 345, is one more than the cube of a certain number.

50. (c) Each of the given numbers is a prime number. But the number obtained on reversing the digits of each of the numbers except 23, is also a prime number.

https://sscstudy.com/

All except 11 : 36 follow the same patter. Hence, 11:36 is the odd one.

60. (c) In all pairs except 182 : 140, first number is 35 more than the second.

https://sscstudy.com/

GENERAL MENTAL ABILITY

13

CHAPTER 03

Series A series is a sequence of letters or numbers or a combination of both obtained by some particular predefined rule. The candidate is required to study the given series, identify the pattern followed and complete the given series with the most suitable alternative or find the wrong term in the series. The following types of questions are generally asked in exams related to series.

2. Letter Series In this type, a series of letters, either single or in groups is given. The terms of the series form a definite pattern or sequence as regards the positions of letters in the English alphabet. The candidate is required to find out that pattern and apply it to find the missing term.

Ex. 4 Find the next term in the series A, C, F, J, O, ? (a) T

1. Number Series In these questions, a number series is given following a particular sequence or rule. The candidate has to either complete the series or find the wrong term given at a specific place in the series.

Ex. 1 Find the missing term in the series 5, 10, 15, 20, 25, ? (a) 30 (c) 35

A

15

+5

+5

20 +5

25 +5

(b) 42

+5

(c) 43

7 +4

13 +6

21 +8

31

+10

+2

+2

(d) 44

A

+1

B

+1

C

+1

D

+1

E

I

+1

J

+1

K

+1

L

+1

M

Q

+1

R

+1

S

+1

T

+1

U

Ex. 6 Find the next term in the series BMO, EOQ, HQS, ? (a) KSU (c) SOV

43

(b) LMN (d) SOW

Sol. (a) The pattern of the series is,

+12 +2

M

Ex. 3 Find the wrong term in the series 4, 12, 30, 68, 146, 302, 622 (a) 12

(b) 30

(c) 146

O

×2+4

×2+6

30

68

146

+2 +2

E O Q

+3 +2 +2

H Q S

+3 +2 +2

K S U

3. Alpha Numeric Series 304

12

+3

(d) 302

Sol. (d) The pattern of the series is, 4

+6

(b) EMV (d) EMU

B +2

+5

+4

U

Ex. 5 Find the next term in the series AIQ, BJR, CKS, DLT, ?

30

Sol. (c) The pattern of the series is, 3

+3

O

Sol. (d) The pattern of the series is,

Ex. 2 Find the missing (?) in the series 3, 7, 13, 21, 31, ? (a) 41

(d) V

J

F

C +2

Sol. (a) The pattern of the series is, 10

(c) S

(a) ENU (c) ENV

(b) 40 (d) 45 5

(b) U

Sol. (b) The pattern of the series is,

302

622

×2+8 ×2+10 ×2+12 ×2+14

This series is a combination of letter series and number series which move according to a set pattern. The candidate has to find the pattern for both to get the next term of the series

https://sscstudy.com/

https://sscstudy.com/

14

CUET (UG) Section III : General Test

Ex. 7 Find the next term in the series 3 F, 6G, 11 I, 18 L ? (a) 21 O (c) 25 P

5. Conditional Letter, Number and Symbol Sequence

(b) 25 N (d) 27 P

In this type of questions, a jumbled sequence of some letters, numbers and / or symbols are given. The candidate is required to find the total number of a particular number / letter/ symbol in the sequence applying certain condition.

Sol. (d) The pattern of the series is, 3 F

+3 +1

+5

6

+2

G

11 I

+7

18

+3

+9 +4

L

27 P

Ex. 8 Choose the missing term.

Ex. 11 How many such symbols are there in the following arrangement each of which is immediately preceded by a number but not immediately followed by a consonant?

P 3 C, R 5 F, T 8 I, V 12 L, ? (a) X 17 O (c) X 12 T

(b) L 12 S (d) I 17 O

Sol. (a) The pattern of the series is,

B 5 R 1 @ EK 4 F 7  D A M 2 P 3 % 9 H I W 8 * 6 U I $ V Q #

P

+2

R

+2

T

+2

V

+2

X

3

+2

5

+3

8

+4

12

+5

17

(a) None

C

+3

F

+3

I

+3

L

+3

O

Sol. (d) Let us see

(b) One

(c) Two

(d) Three





4. Continuous Pattern Series In this type, a group of letters usually given in small letters are repeated in a systematic way to establish a series. However, some letters are missing from the series. These missing terms are then given in a proper sequence as one of the alternatives. The candidate is required to choose this alternative as the answer.

Ex. 9 Find the missing letters in the series mc __ m __ a __ ca __ ca __ c __ mc (a) acmmma (b) camcam (c) aaccmm (d) acmmca

B5R1@EK4F7©DAM2P3%9HIW8★6UJ$VQ#







✓ = Condition fulfilled ✗ = Condition not fulfilled Clearly, there are three such symbols.

Ex. 12 How many 7’s immediately preceded by 6 but not immediately followed by 4 are there in the following series? 74276436753578437672406743 (a) One

(b) Two

(c) Four

Sol. (a) The series is m c a / m c a / m c a / m c a / m c a / mc. So, the missing letters are ‘acmmma'.









74276436753578437672406743

Ex. 10 Find the missing letters in the series ✓

ba–cb–b–bab– (a) acbb (c) bcaa

(d) Six

Sol. (b) Let us see

(b) bacc (d) cabb

Sol. (b) The series is babc / babc / babc. So, the missing letters are ‘bacc’.

✓ = Condition fulfilled ✗ = Condition not fulfilled Clearly, there are two such 7’s

https://sscstudy.com/



https://sscstudy.com/

GENERAL MENTAL ABILITY

15

Practice Questions Directions (Q.Nos. 1–10) In the following questions, a number series is given with one term missing. Choose the correct alternative that will continue the pattern. 1. 6, 9, 12, 15, 18, ? (a) 21

(b) 20

(c) 19

(d) 22

(c) 43

(d) 54

2. 13, 14, 18, 27, ?, 68, 104 (a) 36

(b) 41

3. 6, 11, 21, 36, 56, ? (a) 42

(b) 51

(c) 81

(d) 91

(b) 191 (b) 72

(c) 161

(d) 169

(c) 136

(d) 86

7. 325, 259, 204, 160, 127, 105, ? (a) 94

(b) 96

(c) 98

(d) 100

(c) 64

(d) 48

(c) 150

(d) 160

(c) 15

(d) 18

8. 82, 70, 76, 64, 70, 58, ? (a) 52

(b) 76

9. 1, 5, 14, 30, 55, 91, ? (a) 130

(b) 140

10. 3, 15, 4, 16, 5, 17, 6, ?, 7 (a) 12

(b) 13

Directions (Q.Nos. 11–18) In the following questions, one term in the number series is wrong. Find out the wrong term. 11. 3, 10, 27, 4, 16, 64, 5, 25, 125 (a) 3

(b) 4

(c) 10

(d) 27

12. 5, 27, 61, 122, 213, 340, 509 (a) 27 (c) 122 (b) 10 (b) 124

(c) 96 (c) 123 (b) 22 (d) 70 (b) 24 (d) 567

(a) E (c) G

(b) F (d) H

22. E, J, ?, T, Y, D (a) B

(b) O

(c) F

(d) J

(b) DM

(c) GH

(d) LM

(b) IV

(c) JQ

(d) KP

(c) LVR

(d) LWP

23. AI, BJ, CK, ? (a) DL

24. AZ, CX, FU, ? (a) IR

25. PMT, OOS, NQR, MSQ, ? (a) LUP

(b) LVP

26. BCA, FGE, ?, NOM, RSQ, VWU (a) IJH (c) KJI

(b) KLJ (d) JKI

27. ADGJ, CFIL, EHKN, ? (a) FILO (c) DGJM

(b) HKNQ (d) GJMP

28. CBDA, GFHE, KJLI, ? (b) MNOP (d) ONPM

(d) 382

alpha-numeric series is given with one or more terms missing. Choose the missing term. 29. 2B, 4C, 8E, 14H, ?

(d) 129

(a) 16 K

(b) 20 I

(c) 20 L

(a) 9 (c) 12

(b) 10 (d) 19

(a) BX17 (c) CY17

(b) BY17 (d) CY18

32. C4X, F9U, I16R, ?

17. 1, 3, 12, 25, 48 (a) 3 (c) 25

(b) Q (d) S

31. KM5, IP8, GS11, EV14,?

16. 2, 6, 24, 96, 285, 568, 567 (a) 6 (c) 285

(b) J, H (d) K, I

30. 2, A, 9, B, 6, C, 13, D, ?

15. 15, 16, 22, 29, 45, 70 (a) 16 (c) 45

(a) J, G (c) K, H

Directions (Q.Nos. 29-36) In the following questions, a

14. 125, 126, 124, 127, 123, 129 (a) 126

19. T, R, P, N, L, ?, ?

(a) NOPM (c) PMNO

(b) 61 (d) 509

13. 4, 10, 22, 46, 96, 190, 382 (a) 4

Directions (Q. Nos. 19-28) In the following questions, the terms of an alphabet series are given with one or more terms missing. Choose the missing terms.

21. A, I, P, V, A, E, ?

6. 10, 14, 28, 32, 64, 68, ? (a) 132

(b) 12 (d) 15

(a) O (c) R

(b) 48 (d) 72

5. 5, 11, 23, 47, 95, ? (a) 190

(a) 11 (c) 17

20. H, I, K, N, ?

4. 1, 1, 8, 4, 27, 9, ?, 16 (a) 32 (c) 64

18. 1, 5, 5, 9, 7, 11, 11, 15, 12, 17

(b) 12 (d) 48

(a) K 25 P (c) L 25 O

https://sscstudy.com/

(b) L 25 P (d) L 27 P

(d) 22L

https://sscstudy.com/

16

CUET (UG) Section III : General Test

33. 2 Z 5, 7 Y 7, 14 X 9, 23 W 11, 34 V 13, ? (a) 27 U 24 (c) 47 U 15

45. a _ bccb _ ca _ cca _ baab_c

(b) 45 U 15 (d) 47 V 14

(a) ababc (c) accab

34. N 5 V, K 7 T, ?, E 14 P, B 19 N (a) H 9 R (c) H 10 R

46. How many 4’s are there in the series which comes between two 5’s ? 344545421454574545

(b) H 10 Q (d) I 10 R

(a) 4 (c) 6

35. Q1F, S 2 E, U 6 D, W 21 C, ? (a) Y 44 B (c) Y 88 B

(b) Y 66 B (d) Z 88 B

immediately preceded by 6 but not immediately followed by 8? 348761567849675

(b) 36 J 21 (d) 48 J 23

(a) 1 (c) 3

Directions (Q.Nos. 37-45) In the following letter series, some letters are missing which are given in that order as one of the alternatives. Choose the correct alternative.

times 1, 3 and 7 have appeared together, 7 being in the middle and 1 and 3 on either side of 7? 2931737771331738 571377173906

(b) aabaa (d) baabb

(a) 3 (c) 5

38. a _ ba _ b _ b _ a _b (a) abaab (c) aabba

(b) abbab (d) bbabb

(a) acab (c) cbab

sequence of numbers each of which is immediately followed by an odd number as well as immediately preceded by an even number? 86768932753422355228119

(b) bcbb (d) ccab

(a) One (c) Five

40. ab _ d _ aaba _ na _ bad na _ b (a) andaa (c) badna

(b) babda (d) dbanb

mstnpzxnpnpqynpranpst (a) 2 (c) 4

(b) acba (d) caba

are immediately preceded by N and immediately followed by U? ABCDKNLJMNKSTRZNKUANKUBWXNKLS

(b) ababb (d) babba

(a) 6

43. ac_cab_baca_aaa_aba (b) aacb (d) bcbb

(b) 2

(c) 3

(d) 4

52. Which of the following is the 10th to the right of the 19th from the right end of the below arrangement? F4T2E%MP5W9@LQR6UH3Z7*AT B8 V#G$YD

44. abca_bcaab_ca_bbc_a (a) ccaa (c) abac

(b) 3 (d) 5

51. In the following series, how many Ks are there which

42. _ a _ b _ abaa _ bab _ abb

(a) aabc (c) babb

(b) Three (d) None of these

50. How many times n comes before p in the given series?

41. a _ bbc _ aab _ cca _ bbcc (a) bacb (c) abba

(b) 4 (d) None of these

49. How many even numbers are there in the following

39. bca _ b _ aabc _ a _ caa

(a) aaabb (c) babab

(b) 2 (d) 4

48. In the following series of numbers, find out how many

37. ab _ _ baa _ _ ab _ (a) aaaaa (c) aabab

(b) 5 (d) 3

47. How many 7’s are there in the given series which are

36. 2 A 11, 4 D 13, 12 G 17, ? (a) 36 I 19 (c) 48 J 21

(b) abcaa (d) bacaa

(b) bbaa (d) abba

(a) M

(b) T

(c) #

(d) 2

ANSWERS 1. (a)

2. (c)

3. (c)

4. (c)

5. (b)

6. (c)

7. (a)

8. (c)

9. (b)

10. (d)

11. (c)

12. (a)

13. (c)

14. (d)

15. (b)

16. (b)

17. (c)

18. (b)

19. (b)

20. (c)

21. (d)

22. (b)

23. (a)

24. (c)

25. (a)

26. (d)

27. (d)

28. (d)

29. (d)

30. (b)

31. (c)

32. (c)

33. (c)

34. (c)

35. (c)

36. (d)

37. (b)

38. (d)

39. (a)

40. (a)

41. (b)

42. (d)

43. (a)

44. (c)

45. (a)

46. (d)

47. (a)

48. (a)

49. (d)

50. (d)

51. (b)

52. (b)

https://sscstudy.com/

https://sscstudy.com/

Hints & Solutions 1. (a) The pattern of the series is, 9

6

12

+3

15

+3

11. (c) The correct sequence is

18

+3

+3

14

18

+1

+4

(1)2

27 +9

(2)2

43

+16

(3)2

68 +25

3. (c) The pattern of the series is, 11

6

21

+5

+10

36 +15

+5

+20

+5

5

27

61

122

213

340 509

23–3

33–3

43–3

53–3

63–3

73–3

81

A

4

10 +6

22 +12

46 +48

+24

E 190 382

96

+96

So, 96 is the wrong term.

13 12 23 22 33 32 43 42

14. (d) The pattern of the series is,

23

47

A 125

191

95

126

124

127

×2+1

×2+1

×2+1

×2+1

14

28

+4

32

×2

15. (b) The pattern of the series is, 68

+4

136

259

–66

204 –55

+11

160 –44

+11

–33

+11

–11

70 –12

76 +6

64 –12

70 +6

64

–12

+6

5 +4

22

14 +9

30 +16

32

+25

42

52

+1

+36

62

72

+1

+9

+16

+25

22

32

42

52

+3

E

H

+5

J

O

+5

T

+5

+5

Y

D

+1

+1

B

+1

+1

J

C K

+1

D

+1

L

+2

+3

C

–2

–3

X

F U

+4

J

–4

Q

–1

+2

–1

T

O

O

S

–1

N

+2

Q

–1

R

–1

+2

–1

M

S

Q

–1

+2

–1

L

U

P

26. (d) The pattern of the series is, F

+4

J

+4

N

+4

R

+4

V

C

G

+4

K

+4

O

+4

S

+4

W

16. (b) The pattern of the series is,

A

+4

E

+4

I

+4

M

+4

Q

+4

U

25

6

×6–6

24 ×5–5

96

×4–4

285 568 567

×3–3

×2–2

27. (d) The pattern of the series is,

×1–1

So, 24 is the wrong term.

1

3

12

25

48

12–02

22–12

42–22

62–32

82–42

A

+2

C

+2

E

+2

D

+2

F

+2

H

+2

J

G

+2

I

+2

K

+2

M

J

+2

L

+2

N

+2

P

G

28. (d) The pattern of the series is, C

+4

G

+4

K

+4

O

B

+4

F

+4

J

+4

N

So, 25 is the wrong term.

D

+4

H

+4

L

+4

P

18. (b) The pattern of the series is,

A

+4

E

+4

I

+4

M

+4

+1

+4

+5

V

+4

+1

+1

+6

P

+4

+49

3 15 4 16 5 17 6 18 7 +1

+7

I

B

140

10. ((a)) The pattern of the series is, +1

+4

27 91

55

+4

25. (a) The pattern of the series is,

17. (c) The pattern of the series is,

9. (b) The pattern of the series is, 1

70

45

12

2

58

29

+1

+11

8. (c) The pattern of the series is, 82

22

N

So, 22 is the wrong term.

94

105 –22

16

15

×2

127

+11

Z

M

20

7. (a) The pattern of the series is, 325

128

–1

P

64 ×2

+4

129

So, 129 is the wrong term.

6. (c) The pattern of the series is, 10

123

+3

K

24. (c) The pattern of the series is,

+1

–1 ×2+1

+5

A

5. (b) The pattern of the series is, 11

+8

I

+1

H

23. (a) The pattern of the series is,

+192

1 1 8 4 27 9 64 16

J

22. (b) The pattern of the series is,

94

+5

L

–2

21. (d) The pattern of the series is,

83–3

13. (c) The pattern of the series is,

4. (c) The pattern of the series is,

5

+2

I

A

+25

+5

+1

H

So, 27 is the wrong term.

56

–2

N

20. (c) The pattern of the series is,

24

(6)2

–2

P

–2

+36

(4)2 (5)2

–2

R

2

12. (a) The pattern of the series is,

104

–2

T

So, 10 is wrong and must be replaced by 3 i.e. 9.

+3

2. (c) The pattern of the series is, 13

19. (b) The pattern of the series is,

3, 3 2, 3 3, 4, 4 2, 4 3, 5, 5 2, 5 3

21

1

+2

5 5 9 +4

+2

+4

+2

13 7 11 11 15 12 17 +4

+2

So, 12 is the wrong term.

https://sscstudy.com/

29. (d) The pattern of the series is, 2 B

+2 +1

4 C

+4 +2

8 E

+6 +3

14 H

+8 +4

22 L

R

https://sscstudy.com/

18

CUET (UG) Section III : General Test

30. (b) The pattern of the series is, +1

+1

2 A 9 B +7

38. (d) The series is a b b /a b b / a b b / a b b. Thus, the pattern ‘abb’ is repeated.

+1

+7

b c a a. Thus, the pattern ‘bcaa’ is repeated.

–3

–2

+3

M

P

+3

5

–2

I

8

–2

G

+3

S

+3

11

V

+3

C

+3

Y

+3

14

17

32. (c) The pattern of the series is, C

+3

4(2)2

F

+3

9(32)

X

–3

U

+3

I

L

16(42) –3

25 52

–3

R

O

2 Z

5

+5

7

–1

Y

+2

7

+7 –1

14 X

+2

9

+9

23

–1

+11 –1

W

+2

11

34

+13

+2

13

5

V

–3 +2

–2

K 7

T

–3 +3

H 10

–2

R

–3 +4

–2

–3

E 14

+5

–2

P

42. (d) The series is baa / bba/ baa / bba / baa / bb. Thus, the pattern baa/bba is repeated.

Q

×1+1

1 F

–1

S

2 E

+2

×2+2 –1

+2

U

6

×3+3

D

–1

W

21 C

a c a / c a b / a b a/ c a b / a a a / c a b / a.

50. (d) m s t n p z x n p n p q y n p r a n p s t Clearly, required number of n = 5 times

U

/aabcc.

15

Thus, the pattern move in a cyclic order and in each group, the middle letter occurs only once.

×4+4 –1

2 A 11

+3 +2

4 D 13

×3 +3 +4

12 G 17

×4 +3 +6

51. (b) ✗





ABCDKNLJMNKSTRZNKUANKUBWXNKLS

B

46. (d)

19

✗ ✗

N





Y

88 B





3 4 4 5 4 5 4 2 1 4 5 4 5 7 4 5 4 5 ✓

+2

Even number Even number Odd number Preceded Middle Followed So, total number of such even numbers = Four =

45. (a) The series is aabcc / bbcaa / ccabb





.P = Condition fulfilled ✗ = Condition not fulfilled ∴ Required 4 = 3 times

47. (a) ✗

36. (d) The pattern of the series is, ×2

8676 8 93275342 2 35522 8 119 Required pattern

43. (a) The series is

35. (c) The pattern of the series is, +2

49. (d) Clearly, the given sequence of numbers is as follows

abc /aabc/aabbc/aabbc/a.

34. (c) The pattern of the series is, N

bbcc. Thus, the pattern ‘aabbcc’ is repeated.

47

–1

V

+2

Therefore, total number of such patterns = 3

41. (b) The series is aabbcc / aabbcc / aa

44. (c) The series is

33. (c) The pattern of the series is,

1/ 3 7 1/ 3 Preceding middle following

=

abadna / ab. Thus, the pattern ‘abadna’ is repeated.

–2

E

+3

∴Required pattern

40. (a) The series is abadna / abadna/

31. (c) The pattern of the series is, K

293173777133173857 1377173906

39. (a) The series is b c a a / b c a a / b c a a/

6 C 13 D 10

–3

48. (a) Clearly, the given number series is as follows



48

3 4 8 7 6 1 5 6 7 8 4 9 6 7 5

52. (b) 1 2 3 4 5 6 7 8 9 10 11 12 13 14 F 4  T 2E%MP 5 W 9 @ L 15 16 17 18 19 20 21 22 23 24 25 26 Q R 6 U H 3 Z 7 * A T B 27 28 29 30 31 32 33 8 V # G $ Y D Total terms = 33 ∴19th term from right = ( 33 + 1 − 19)

= 34 − 19

J 23

37. (b) The series is ab a / ab a / a b a / ab a. So, the missing letters are aabaa



P = Condition fulfilled ✗ = Condition not fulfilled ∴ Required 7 = 1 time

https://sscstudy.com/



.P = Condition fulfilled ✗ = Condition not fulfilled ∴Required number of K = 2 times

= 15 th from left = Q ∴ 10th term to the right of 19th term = 10th term to the right of Q = (15 + 10)th = 25th term from left = T

https://sscstudy.com/

GENERAL MENTAL ABILITY

19

CHAPTER 04

Coding and Decoding Coding is a method of transmitting information to someone using some suitable codes, so that it is not understood by others. Decoding is the method of finding the actual meaning of those codes. In questions based on coding-decoding, a word is coded in a particular way and the candidates are asked to code the other word in the same way. There are mainly four types of questions which are asked.

Ex. 3 In a certain coding system, ‘SHEEP’ is written as ‘GAXXR’ and ‘BLEAT’ is written as ‘HPXTN’. How can ‘SLATE’ be written in that same coding system? (a) GPTNX (c) GPXN

(b) GPTXN (d) PTGXN

Sol. (a) In both the words ‘SHEEP’ and ‘BLEAT’, the letter E is common and code for E is X. Hence, using direct letter coding method, we have

1. Letter Coding In this category, certain alphabets are coded as certain other alphabets. The candidate is required to understand the pattern and solve the problems based on that pattern.

Ex. 1 In a certain code language ‘MIGHT’ is written as ‘GHMTI’, then how will ‘EARTH’ be written in that code? (a) RTEHA (c) RTAEH Sol. (a) As,

Similarly,

(b) RTEAH (d) RETHA

G

and B

H

A

L

H ∴ S P L

E

X

E

X

A

T

E

X

A

T

T

N

P

R

T

N

E

X

G P

Ex. 4 If in a certain code language ‘NOTION’ is written as ‘MLGRLM’, then how will ‘VECTOR’ be written in that language? (a) EVXLGI

(b) EVGXIL (c) EVXGIL (d) EVXGLI

Sol. (d) As, 13 12 7 18 12 13

14 15 20 9 15 14

M L G R L M

N O T I O N Opposite letter

M I G H T

G H M T I

Opposite letter

1 2 3 4 5 (position)

3 4 1 5 2 (new position)

Opposite letter Opposite letter Opposite letter

E A R T H

R T E H A

1 2 3 4 5 (position)

3 4 1 5 2 (new position)

Opposite letter

22 5

(b) OBNF

Sol. (b) As,

C

O

(c) BOFN L

Similarly, N

P

M

A

M

E

+1 +1 +1 +1

O

B

N

F

E V X G L I Opposite letter Opposite letter Opposite letter Opposite letter Opposite letter

(d) EMAE

Opposite letter

D

+1 +1 +1 +1

D

5 22 24 7 12 9

3 20 15 18

Similarly, V E C T O R

Ex. 2 In a certain code language ‘COLD’ is coded as ‘DPME’, then how will ‘NAME’ be written in that language? (a) OBME

S

E

2. Number/Symbol Coding In this category, certain alphabets are coded as certain numbers /symbols. The candidate is required to understand the pattern and solve the problem based on that pattern.

https://sscstudy.com/

https://sscstudy.com/

20

CUET (UG) Section III : General Test

Ex. 5 If in a certain code language ‘PUT’ is written as ‘16, 21, 20’, then how will ‘BAT’ be written in that language? (a) 2, 4, 20 (c) 4, 5, 21

(b) 5, 2, 11 (d) 2, 1, 20

Sol. (d) Here, each letter assigned its positioned value in English alphabetical series. PUT = 16, 21, 20 and BAT = 2, 1, 20 (b) 47 (d) 49

English alphabetical order and then the word is coded as the sum of the position of the alphabets in English alphabetical series. X = 24 (positional value in English alphabet) BE = 2 + 5 = 7 Similarly, RING = 18 + 9 + 14 + 7 = 48

Ex. 7 In a certain code, the following numbers are coded by assigning signs 2

3










>

=


= < (a) 63181 (c) 62781

Ex. 9 If ‘Orange’ is called ‘Butter’, ‘Butter’ is called ‘Soap’, ‘Soap’ is called ‘Ink’ and ‘Ink’ is called ‘Red’, then what is used for washing clothes? Sol. (c) We use ‘Soap’ to wash our clothes. But here

Sol. (c) Here, each letter is coded as its position in

1

In this category, a series of words is given and each word of this series is substituted with another word. The candidate is required to answer the question in the substituted code language.

(a) Red (c) Ink

Ex. 6 If X = 24 and BE = 7, then RING = ? (a) 41 (c) 48

3. Substitution Coding

(a) 6 (c) 3

Sol. (c) We have,

(b) !@%$# (d) !@%#$

Similarly

R E S E A R C H

S C A R E

$ # ! # % $ & @

!

& % $ #

(b) 2 (d) 1 go

to

school

1 2 5

...(i)

study

in

school

1 4 6

...(ii)

run

to

school

1 3 5

...(iii)

Now, from Eq. (iii), to → 5, school → 1 run → 3 ∴

https://sscstudy.com/

https://sscstudy.com/

GENERAL MENTAL ABILITY

21

Practice Questions 1. If ‘FISH’ is written as ‘EHRG in a certain code, then (a) ITMFKD (c) KVOHMF

(b) ITNFKD (d) TIMFKD

2. If SUMMER is coded as RUNNER, then code for WINTER will be (a) SUITER (c) WALKER

then how is ‘LEADER’ written in that code?

(b) SULQFLSDO (d) MRFKZFMXI

(b) TOOT (d) TONT

6. If EHFNRQ is the code for BECKON, then which word has the code QDFWXULQ?

Which word would be written as CQFZE? (b) BRAED (d) BRADE

(a) FLKBN (c) LKNBF

(b) MWEO (d) MWIO

LBBIHOD, then which word would be coded as SLTMFNB? (b) TKULGMC (d) TMUNGOC

‘BDFGJK’ and ‘CHILDREN’ is written as ‘MOXQUFGJ’ then how is ‘REPRINT’ written in that same code?

How is COMPUTER written in the same code? (b) ETUPMOCR (d) MOCPETUR

‘CBDWLQJWYQCL’ and ‘OPERATION’ as ‘CXFBWYQCL’. How would ‘SEPERATION’ be coded? (b) JFQYWBCXQL (d) QCLYWBFXJE

18. If the letters in PRABA are coded as 27595 and THILAK is coded as 368451, then how can BHARATI be coded? (b) 9675538 (d) 9567538

19. If NOIDA is written as 39658, then how will INDIA be written? (a) 36568 (c) 63569

10. In a certain code, KAVERI is coded as VAKIRE. How

(b) FGBUXJK (d) BGFXJK

17. In a code language ‘ORGANISATION’ is written as

(a) 9567568 (c) 9657538

9. In a certain code, INACTIVE is coded as VITCANIE.

(b) LBNKF (d) LNBKF

16. If in a code language ‘PARENT’ is written as

(a) EJXEBYQCL (c) JFXFBWYQCL

8. If in a certain language, MACHINE is coded as

(a) PMOCRETU (c) UTEPMOCR

(a) MWED (c) MWOE

(a) FGBFXJK (c) FGBFXGD

(b) NACUTIRN (d) NACTURIN

7. In a certain code, SWITCH is written as TVJSDG.

(a) RKSLEMA (c) RMSNEOA

(b) SEPTEMBER (d) NOVEMBER

how will VOTES be written in that code?

‘DONE’ and ‘CORD’ is coded as ‘DOSE’, then how would you code SONS?

(a) BARED (c) BREAD

(a) AUGUST (c) OCTOBER

15. In a certain code, STOVE is written as FNBLK, then

5. If ‘GOLD’ is written as ‘HOME’, ‘COME’ is coded as

(a) NCAUTIRN (c) NATCRIUN

13. In a certain code, DECEMBER is written as,

MOVE as ZMWE. Following the same rule of coding, what should be the code for the word OVER?

code for PRINCIPAL?

(a) TPOT (c) TOOS

(b) TCAEHER (d) TAEECHR

14. In a certain code, FIRE is written as QHOE and

(b) NHCGGU (d) OHDGHU

4. If the code for MOTHER is JRQKBU, then what is the (a) MRFKZLMXI (c) MUFQZLMDI

(a) TAECEHR (c) TAECHER

ERMBCEDE. Which word will be written as ERMBVENO in that code?

(b) VIOUER (d) SUFFER

3. If in a certain code, ‘BASIC’ is written as ‘DDULE’, (a) NGCFGT (c) OGDFHT

12. If in a certain language, CONDEMN is written as CNODMEN, then how will TEACHER be written in that code?

how would JUNGLE be written in that code?

(b) 63568 (d) 65368

20. If ‘FLARE’ is coded as 21, 15,26, 9, 22, then how

is MYSORE written in that code?

would ‘BREIF’ be coded in the same language?

(a) EROSYM

(b) SYMROE

(c) SYMERO

(d) None of these

(a) 25, 9, 22, 21, 18 (c) 13, 19, 11, 37, 5

11. If SPIDER is written as PSDIRE in a certain code,

21. If ‘LINGER’ is ‘123456’ and ‘FORCE’ is ‘56789’ then

then how could COMMON be written in that code?

‘FIERCE’ will be

(a) OCOMMO (c) OCMOMN

(a) 345667 (c) 345677

(b) OCMMNO (d) OCMMON

(b) 5, 37, 11, 19, 13 (d) 25, 9, 22, 18, 21

https://sscstudy.com/

(b) 456678 (d) Cannot be determined

https://sscstudy.com/

22

CUET (UG) Section III : General Test

(a) Well (c) Sky

22. If B = 2, BAG = 10, then BOX = ? (a) 36 (c) 41

(b) 39 (d) 52

33. ‘If ‘dust’ is called ‘air’, ‘air’ is called ‘fire’, ‘fire’ is called ‘water’, ‘water’ is called ‘colour’, ‘colour’ is called ‘rain’ and ‘rain’ is called ‘dust’, then where do fish live?

23. If OWL = 50 and N = 14, then TIME is (a) 45 (c) 43

(b) 47 (d) 49

(a) Fire (c) Colour

24. If CAT =12, then MAN = ? (a) 14 (c) 16

animals who crawl are called ‘flying’, those living in water are called ‘snakes’ and those which fly in the sky are called ‘hunters’, then what will a lizard be called?

25. If ASHA equal 79, then VINAY BHUSHAN = ? (b) 200 (d) 180

(a) Swimmers (c) Flying

26. If MOBILITY is coded as 46293927, then EXAMINATION is coded as (a) 45038401854 (c) 57159413955

‘white’ is called ‘red’, ‘red’ is called ‘black’, then what is the colour of clear sky? (a) Blue (c) White

27. If ARC is written as $@* and HIT is #&%, then CHAIR is

and ‘he is great’ = ‘bo cp dq', then what represents ‘he is god’?

28. In a certain code language ‘SAFER’ is written as

(a) cp er bo (c) dq bo cp

‘5@3#2’ and ‘RIDE’ is written as ‘2©%#’ how would ‘FEDS’ be written in that code?

Sarurpur’, mhi cinto keepi tsi oind’ means ‘her first love is literature’ and ‘oind geit tsi cinto pki’ means ‘literature collection is her hobby’, which word would mean ‘literature’?

29. If in a certain code language ‘STAR’ is written as ‘5 $ * 2’, ‘TORE’ is written as $ 3 2 @, then how will ‘OATS’ be written in that language?

(a) cinto (c) oind

(b) 3 * $ 5 (d) 3 5 * $

30. If in a certain code language ‘GONE’ is written as ‘5 @

means ‘I like oranges’ and 267 means ‘oranges are sour’. Which digit represents sour? (a) 9 (c) 6

(b) * 9 © 6 (d) * @ 56

31. If ‘sky’ is ‘star’, ‘star’ is ‘cloud’, ‘cloud’ is ‘earth’, ‘earth’

(b) baoli (d) geit

38. In a certain code, 592 means ‘grapes are sweet’, 374

© 9’ and ‘’SEAL is written as ‘6 9 % *’, then how will ‘LOGS’ be written in that language? (a) * 9 © 6 (c) * @ 65

(b) an bo cp (d) an bo dq

37. If ‘ cinto baoli tsi nzro’ means ‘her village is

(b) 3 @ % 5 (d) 3 # % 2

(a) 3 * 5 $ (c) 3 $ * 5

(b) Green (d) Black

36. If ‘god is great’= ‘cp an bo’, ‘great help done’ = ‘er cp fs’

(b) #*$&% (d) *#$&%

(a) 3 # © 5 (c) 3 # % 5

(b) Snakes (d) Hunters

35. If ‘blue’ is called ‘green’, ‘green’ is called ‘white’,

(b) 56149512965 (d) 67250623076

(a) #*&$@ (c) *#$&@

(b) Water (d) Dust

34. If the animals which can walk are called ‘swimmers’,

(b) 24 (d) 15

(a) 211 (c) 144

(b) Island (d) Air

(b) 5 (d) Cannot be determined

39. In a certain code language, ‘134’ means ‘good and tasty’,

is ‘tree’ and ‘tree’ is ‘book’ , then where do the birds fly?

‘478’ means ‘see good pictures’ and ‘729’ means ‘pictures are faint’. Which of the following digits stands for ‘see’ ?

(a) Cloud (c) Star

(a) 9 (c) 1

(b) Sky (d) None of these

32. If ‘sand’ is called ‘air’ , ‘air’ is called ‘plateau’, plateau’

(b) 2 (d) 8

40. If VOTER = 41352, HEATER = 743654, TEASER = 645834, which number represents S?

is called ‘well’, ‘well’ is called ‘island’ and ‘island’ is called ‘sky’, then from where will a woman draw water?

(a) 1 (c) 8

(b) 5 (d) 7

ANSWERS 1. 11. 21. 31.

(a) (b) (d) (c)

2. 12. 22. 32.

(b) (a) (c) (b)

3. 13. 23. 33.

(b) (d) (b) (c)

4. 14. 24. 34.

(c) (b) (a) (c)

5. 15. 25. 35.

(b) (b) (d) (b)

6. 16. 26. 36.

(d) (a) (b) (d)

https://sscstudy.com/

7. 17. 27. 37.

(c) (c) (c) (c)

8. 18. 28. 38.

(b) (c) (c) (c)

9. 19. 29. 39.

(b) (b) (b) (d)

10. 20. 30. 40.

(c) (d) (d) (c)

https://sscstudy.com/

23

GENERAL MENTAL ABILITY

Hints & Solutions F

1. (a) As,

J

H R –1

S

7. (c) As,

–1

G

–1

T

E

–1

M F

T

S

U M M E

Similarly,

R

I

N T

I

V

K U L

S

I

E

L

–1

Similarly,

R

E

I

N A C T

V

I

E and

O V

G M C

–1 +1 –1 +1

T M F

R

N O M O V Z

E

R

M W E

O

E

M W E

D

–1

15. (b) As,

S

T

O V

E

F

N B L

K

N B

9. (b) As,

C

D D U L L

+1

S

+2 +3 +2 +3 +2

Similarly,

T

H O

B I

I

Q H O E

E

–1 +1 –1 +1

+1

–1

O U E R

B A

E

N

B

Similarly,

R

14. (b) As, F

+1

I

C H

L

+1 +1

–1

3. (b) As,

E

+1 –1

C Q F Z –1

V E M B

R M B V E

E

E A D

R

8. (b) As, M A

Similarly, N O

C H

S D G

+1 –1

K D

+1 +1

W

T

+1 –1 +1 –1

J

B

R U N N E R

Similarly,

I

V

–1

I

–1

W

+1 –1

U N G L –1

2. (b) As,

H

–1

–1

E

Similarly,

S

I

–1

I

V

E

T C A N

I

E

Similarly,

V O

T E

L

N K F

S

E

A D E

R

Similarly,

+2 +3 +2 +3 +2 +3

C O M P

U

T E

R

B

16. (a) As, P A R E N T

N H C G G U

4. (c) As, M

O

–3

T

+3

J

R

Similarly,

P

H

–3

Q

R

I

–3 +3

E

+3

R

–3

E

K

B

N C

I

10. (c) As, K A V E R I

U P A L

V

Similarly, L M D

step forward and each vowel remains unchanged. D , C

O L

+1 +1

+1

H O M and C

D O S S T

E

R E

M Y

S O R

S

M E R O

Similarly,

E

Y

–3

B E

+1

P E

Similarly,

I

D E

R E

P R

I

N T

F

B F

X

J K

G

O R G A N I S A T I O N

S

D I

R E

C B D W L Q J W Y Q C L

C O M M O

N

O P E R A T I O N C X F B W Y Q C L

12. (a) As, C O N D E M N

O N

Similarly, S E P E R A T

I O N

S

+1 +1

O O

Similarly,

–3

T E

A C H E R

–3

T A E C E

13. (d) As, D E C E M B E R U L –3

U R

–3

I

18. (c) As, P

R

A B A and T

H

I

L

A

K

2

7

5 9

3

6

8

4

5

1

H R

C K O N

–3

J F X F B W Y Q C L

C N O D M E N

T

–3 –3

N A C T

X Q U F G J

D R E N

R

O C M M N O

E

–3 –3

M O

L

and

+1

Similarly, Q D F W X –3

I

17. (c) As, 11. (b) As, S P

H F N R Q

–3

–3

I

C H

E

D O N

+1

6. (d) As,

K

K

and

D

+1

+1

O M

+1

E

O R

Similarly,

A

D F G J

I

5. (b) Here, each consonant is move(a) one

G

B

U P M O C R

–3 +3 –3 +3 –3 +3 –3

M U F Q Z

As,

T

+3

Q

Similarly,

5

B H

A R A

T

I

9

5 7

3

8

–3

N

E R

M B C E

D E

https://sscstudy.com/

6

5

https://sscstudy.com/

24

CUET (UG) Section III : General Test

19. (b) As,

N O

I

25. (d) As,

D A

30. (d) As, A

Similarly,

3

9

6 5

8

I

N

D I

A

S

H A

Position in reverse ⇒ 26, 8, 19, 26 = 79 English alphabet

G O N E and

S E

A L

5 @ c 9

6

% *

Similarly,

Similarly, 6

3

5

6

V I

20. (d) Here, each letter is coded by its position in reverse English alphabetical order As,

F

L

A R

Position in reverse English ⇒ 5+18+13+26+2+25+19+6+ 8+19+26+13 = 180 alphabet

Position in reverse English alphabet

26. (b) As, M

Similarly,

B R

E

I

25, 9, 22, 18, 21

21. (d) As, L

I

N G E

R

1

2

3 4

6

F

O R C E

5

6

Similarly,

7 8

O

B

I

13 15

2

9 12

1+3 1+5

2

9 1+2 9 2+0 2+5

2

9

F

5

9

F → 5

4

I → 2 E → 5 / 9

6

Similarly, E X A M I

L

I

9 20 25

3

N A T

Y

T

9

2

I

7

O N

5 24 1 13 9 14 1 20 9 15 14 5 2+4 1 1+3 9 1+4 1 2+0 9 1+5 1+4

5 6 1

5 1 2 9

4 9

31. (c) Birds fly in the sky and here sky is called star. So, Birds fly in star. 32. (b) We know that, water is drawn from well and here well is called island. So, woman will draw water from island. 33. (c) Fish lives in water and here water is called colour. So, fish lives in colour. 34. (c) We know that lizard crawls and animals who crawl are called flying so, lizard is called flying. 35. (b) Colour of clear sky is blue and here blue is called green. So, colour of clear sky is green. 36. (d) god is great he is great

A R C and H

I

E → 5 / 9

$ @ *

I

bo cp dq

∴he is god = dq bo an ⇒ an bo dq

37. (c)

# & %

C H A

er cp fs

Here, he = dq, is = bo, god = an

T

her village is sarurpar

Similarly,

cp an bo

great help done

6 5

C → 8 Hence, code for ‘FIERCE’ cannot be determined

* @ 5 6

27. (c) As,

R → 6 / 7

G S

N A Y B H U S HA N

E

21, 15, 26, 9, 22

O

L

8

9

R

her first love is literature

22. (c) As, B = 2 (positional value in English

cinto baoli tsi nzro mhi cinto keepi tsi oind

literature collection is her hobby

oind geit tsi cinto pki

* # $ & @

alphabet) B

A G

2 +1 +7 = 10

Similarly,

B O

∴Code for literature is ‘oind’

28. (c) As, S A F E R and R

I

D E

X

5 @ 3 # 2 2+15+24 = 41

Similarly,

F

2 © % #

E

D S

3 #

% 5

39. (d)

(positional value in English alphabet) O W L = 15 + 23 + 12 = 50 Similarly, T I M E = 20 + 9 + 13 + 5 = 47

24. (a) As, C A

29. (b) As, S T A R and T O R E

T

5 $ * 2

3 +1 +20 = 24, 24÷2=12 N

Similarly,

$

O A

T S

3

$ 5

grapes are sweet

3 7 4

I Iike oranges

2 6 7

oranges are sour

∴6 = sour

23. (b) N = 14

Similarly, M A

38. (c) 5 9 2

3

2 @

good and tasty

1 3 4

see good pictures

4 7 8

pictures are faint

7 2 9

∴Code for see ⇒ 8

40. (c) Given, V O T E R = 4 1 3 5 2 H E A T E R= 7 4 3 6 5 4 T E A S E R= 6 4 5 8 3 4

13+1+14 = 28, 28÷2= 14

*

https://sscstudy.com/

∴S = 8

https://sscstudy.com/

GENERAL MENTAL ABILITY

25

CHAPTER 05

Alphabet Test and Jumbling As the name says, this test is related to the English alphabets. Questions asked in this test are based on the alphabetical order of letters, formation of words either meaningful or meaningless various arrangements of letters etc. to which the candidates have to be familiar with.

(a) 7 (c) 8

(b) 11 (d) 9

Sol. (b) Here, 1 2 3 4 5 6 7 8 9 10 11 12 13 14 15 16 17 18 19 20 21 22 23 24 25 26

1. Finding a Particular Letter in English Alphabetical Order

A B C D E F G H I J K L M N O P Q R S T U V W X Y Z From left

In this type of questions, the questions are asked to find out the letter to the left or right of another letter in the English alphabetical order. Questions in other format may also be asked based on the arrangement of alphabetical order. Important Rules 1. nth letter to the left of mth letter from right end = (m + n)th letter from the right end. 2. nth letter to the right of mth letter from left end = (m + n)th letter from the left end. 3. nth letter to the left of mth letter from left end = (m − n)th letter from the left end. 4. nth letter to the right of mth letter from right end = (m − n)th letter from the right end.

Ex. 1 Find the 11th letter to the left of 20th letter from left in the English alphabet. (a) D (c) K

Ex. 2 How many letters are there between 8th letter from left and 7th letter from right in the English alphabet?

(b) J (d) I

11 letters 7th

8th

From right

So, there are 11 letters between 8th letter from left and 7th letter from right.

2. Letter-Word Problems In this type, a word is given and it is asked to find the different letter pairs having as many letters between them as in English alphabet. Also, in some questions, a word is given and then asked how many letters remains same at their position, if they are arranged in ascending or descending order. Also, some questions are asked based on letter rearrangement.

Ex. 3 How many such pairs of letters are there in the word CORPORATE each of which has as many letters between them in the word as in the English alphabet both forward and backward? (a) None (c) Two

(b) One (d) Four

Sol. (d)

Sol. (d) Here,

C O R P O R A T E

1 2 3 4 5 6 7 8 9 10 11 12 13 14 15 16 17 18 19 20 21 22 23 24 25 26

A B C D E F G H I J K L M N O P Q R S T U V W X Y Z To the left 11th From left

20th

Hence, 11th letter to the left of 20th letter from left is I. Alternate Method In English alphabet, 11th letter to the left of 20th letter from left = (20 − 11)th letter from left = 9th letter from left = I

There are four such pairs.

Ex. 4 If the first and third letters in the word NECESSARY were interchanged, also the fourth and sixth letters, and the seventh and the ninth letters, which of the following would be the seventh letter from the left? (a) A (c) R

https://sscstudy.com/

(b) Y (d) E

https://sscstudy.com/

26

CUET (UG) Section III : General Test

Sol. (b) On interchanging the positions of the letters, N E C E S S A R Y

(a) S (c) O

The new letter sequence will be CENSSEYRA. So, the seventh letter from the left is Y.

Ex. 5 If the letters in the word FATHER are written in alphabetical order, then how many letters will not change their position? (a) None (c) Two

(b) One (d) Three

Sol. (b) Word

F

A

T

H

E

R

In alphabetical order A

E

F

H

R

T

Sol. (b) ‘LESS’ is the word which is formed by using first, fourth, seventh and eleventh letters of the word ‘SUPERFLUOUS’. First letter of this word is ‘L’.

Ex. 9 Choose one word out of the given alternatives, which cannot be formed from the letters of the word ‘CONSULTATION’. (b) NATION (d) STATION

Sol. (c) Carefully looking at the words, we find that the word ‘CONSULTATION’ does not contain the letter ‘E’. So, the word ‘SALUTE’ cannot be formed.

3. Alphabetical Order of Words Arranging words in alphabetical order means to arrange them in the order in which they appear in the dictionary, i.e. according to the order of letters with which they begin. To arrange the words as per dictionary follow the given steps. (a) Take the first alphabet of each word and arrange the words in the order in which these alphabets appear in English alphabetical series. (b) If more than one word begin with the same alphabet the such words should be arranged in the order of second alphabet in the word and so on

Ex. 6 Arrange the words in the alphabetical order and tick the one that comes at the third place. (b) Remove (d) Revive

Sol. (d) The arrangement of words would be, Remove, Retrospect, Revive, Rigour,. Clearly, Revive at the third place.

Ex. 7 Arrange the given words according to English dictionary. 1. Episode 2. Epistle 3. Episcope 4. Epigraph (a) 2, 3, 1, 4 (c) 3, 2, 1, 4

(b) L (d) E

(a) CONSTANT (c) SALUTE

There is only one such alphabet.

(a) Rigour (c) Retrospect

Ex. 8 If it is possible to form a word with the first, fourth, seventh and eleventh letters of the word ‘SUPERFLUOUS’. Write the first letter of that word, otherwise X is the answer.

(b) 4, 3, 1, 2 (d) 1, 2, 3, 4

Sol. (b) The correct sequence of words would be Epigraph, Episcope, Episode, Epistle i.e. 4, 3, 1, 2

4. Word Formation In this type, letters from some positions of a given word are selected and then arranged to form another meaningful word. Sometimes words are given in the options and the candidate has to find out the word that can or cannot be formed from the given one.

Ex. 10 Find the word which can be formed by using the letters of the given word — RATIONALISATION. (a) NATIONALIST (c) REALISATION

(b) NATIONALISTICS (d) SITUATION

Sol. (a) From the given word, ‘NATIONALIST’ can be formed because all the letters of this word are present in the given word.

5. Jumbling In this type of questions, a set of English letters is given in a jumbled order. The candidate is required to arrange these letters to form a meaningful word. Note Always try to place the letters according to the numbers provided in options rather than doing it on the basis of your vocabulary knowledge.

Ex. 11 Select the combination of numbers that forms a meaningful word. P N O A C L M I 1

2

3

(a) 2, 7, 8, 6, 4, 3, 1, 5 (c) 7, 1, 8, 5, 6, 2, 4, 3

Sol. (d)

4

5 6

7

8

(b) 4, 7, 5, 2, 6, 8, 1, 3 (d) 5, 3, 7, 1, 6, 4, 8, 2

C O M P L A I N 5

3 7 1 6 4 8 2

Clearly, the given letters, when arranged in the order of ‘5, 3, 7, 1, 6, 4, 8, 2,’ form the word ‘COMPLAIN’.

Ex. 12 Rearrange the letters A, R, T, Y and D to form a meaningful word and select from the given alternatives, the word which is almost opposite in meaning to the word so formed. (a) Dirty (c) Quick

(b) Quiet (d) Queek

Sol. (c) The word that can be formed by rearranging the given letters is ‘Tardy’ which means ‘Sluggish’ and opposite of this word is ‘Quick’.

https://sscstudy.com/

https://sscstudy.com/

GENERAL MENTAL ABILITY

27

Practice Questions 1. In the English alphabet, which letter is 15th from the right? (a) D (c) O

(a) One (c) Three

(b) M (d) L

2. Which letter is exactly midway between G and Q in English alphabet? (a) K (c) N

(b) L (d) J

(b) Two (d) More than three

13. KLING (a) Three (c) Five

(b) Four (d) Six

14. CHILDREN

3. Which letter will be fifth to the right of letter which is fourth to the left of I in English alphabet? (a) O (c) J

12. PRISON

(a) Two (c) Four

(b) Three (d) Five

15. PRODUCTION

(b) N (d) K

4. Which letters are exactly in the middle of the sixth letter from the left and fourteenth letter from the right end in English alphabet?

(a) Nil (c) Two

(b) One (d) Three

16. If the positions of the first and the sixth letters in the

letter from left in the English alphabet?

word DISTRIBUTE are interchanged. Similarly, the positions of the second and the seventh, the third and the eighth and so on. Which of the following letters will be the fifth from left after interchanging the positions?

(a) X (c) P

(a) E (c) S

(a) MN (c) PM

(b) IJ (d) OL

5. Which letter will be the 3rd letter to the right of 19th (b) U (d) V

6. In the English alphabet, which letter will be 8th letter to the left of 16th letter from your left? (a) H (c) I

(b) X (d) Y

7. If the order of the English alphabet is reversed, then which letter would be exactly in the middle? (a) L (c) N

8. If the English alphabet is written in the reverse order, which will be the fifth letter to the left of the ninth letter from the right? (a) P (c) D

(b) N (d) W

from the given alphabet, then which of the following will be the tenth letter from the right end? (b) D (d) H

10. If A interchanges position with B, similarly C and D interchange positions and so as E and F and so on upto Y and Z, then which of the following will be seventh from the left? (a) A

(b) F

(c) L

(d) H

Directions (Q. Nos. 11-15) How many pairs of letters are there in the given word each of which has as many letters between them in the word as in the English alphabet? 11. CREATIVE (a) One (c) Three

(b) Two (d) Four

word DOCUMENTATION are interchanged, and likewise, the positions of the fourth and seventh letters, the second and sixth letters is also interchanged. Which of the following will be eleventh letter from the right end? (b) I (d) U

Directions (Q. Nos. 18 and 19) If the given words are written in alphabetical order, then find the number of letters that will not change their place. 18. S T O N E D

9. If every alternate letter starting from B is deleted (a) G (c) Q

17. If the positions of the third and tenth letters of the

(a) C (c) T

(b) M (d) None of these

(b) I (d) T

(a) None (c) Two

(b) One (d) Three

19. ENGLISH (a) One (c) Three

(b) Two (d) None

Directions (Q Nos. 20 and 21) Arrange the words in alphabetical order and tick the one that comes at the second place. 20. (a) Plane (c) Plenty

21. (a) Cathedral (c) Caterpillar

(b) Plain (d) Player (b) Catenation (d) Category

22. Which of the following words will come at the fourth place according to the dictionary? (a) Converse (c) Curator

https://sscstudy.com/

(b) Current (d) Cushion

https://sscstudy.com/

28

CUET (UG) Section III : General Test

23. Which word will come at third place in the English (a) Radical (c) Racket

(a) None (c) Two

(b) Radiate (d) Radius

24. Arrange the following word in alphabetical order and

(a) Single (c) Sinister

26. 1. Protein 4. Property

3. Send

(a) 1, 2, 3, 4, 5 (c) 2, 5, 4, 1, 3

3. Proverb

(b) 2, 1, 4, 3, 5 (d) 3, 4, 5, 2, 1

snaps’ are rearranged in the alphabetical order, then what will be the middle word? (b) Sample (d) She

Directions (Q. Nos. 28-30) Find the word that can be formed from the letters of the given word. (b) AGMARK (d) MAGIC

38. 1 G

2 3 4 5 6 I C O D N (b) 4, 3, 2, 6, 5, 1 (d) 3, 4, 5, 2, 6, 1

1 2 3 4 5 6 (a) 2, 3, 1, 6, 4, 5 (c) 4, 5, 2, 3, 1, 6

(b) MANTLE (d) ASSURE

1 (b) ENTERTAIN (d) ENTRANCE

2

3

4

5

(a) 1, 2, 3, 4, 5, 6 (c) 5, 1, 6, 4, 3, 2

41. R T A N U E

words, select the word which cannot be formed using the letters of given word.

1 2 3 4 5

31. INFLATIONARY

6 (b) 3, 2, 4, 6, 1, 5 (d) 4, 6, 5, 2, 3, 1

42. When we arrange the letters of the word ‘VRAKIE’ in

(b) FAULTY (d) RATION

correct order, we get the name of a river. Fifth letter from left in that name, is

32. DETERMINATION (a) NATION (c) TERMINATE

(a) 1, 3, 2, 6, 4, 5 (c) 4, 3, 2, 5, 1, 6

6 (b) 4, 1, 6, 2, 3, 5 (d) 6, 1, 2, 3, 5, 4

Directions (Q.Nos. 31 and 32) From the given alternative

(a) FLAIR (c) NATIONAL

(b) 3, 2, 4, 5, 6, 1 (d) 6, 3, 4, 5, 2, 1

40. E M I H T R

30. ENVIRONMENT (a) MOVEMENT (c) EMINENT

Directions (Q. Nos. 38-41) Letters of the words given below have been jumbled up and you are required to construct the words. Each letter has been numbered and each word is followed by four options. Choose the option which gives the correct order of the letters as indicated by the numbers to form words.

39. V A R S T E

29. MEASUREMENT (a) MASTER (c) SUMMIT

(b) R (d) X

(a) 2, 1, 4, 3, 6, 5 (c) 6, 5, 2, 3, 1, 4

28. PRAGMATIC (a) GUITAR (c) GAME

(b) E (d) No word can be formed

(a) T (c) E

27. If the words in the sentence, ‘She showed several sample

(a) Snaps (c) Several

(a) R (c) V

seventh and eleventh letters of the word INTERPRETATION, then which of the following will be third letter of that word, otherwise X is your answer?

(b) 2, 1, 5, 4, 3 (d) 2, 5, 4, 1, 3

2. Problem 5. Project

meaningful word what will be the fifth letter from the right?

37. If it is possible to form a word with the first, fourth,

5. Seller

(a) 1, 2, 4, 5, 3 (c) 2, 1, 4, 5, 3

(b) T (d) D

36. If the letters, ERVSECI can be rearranged to form a

as they occur in the dictionary and then choose the correct sequence. 2. Seldom

35. Rearrange the letters AYDOT to form a meaningful (a) A (c) Y

(b) Singer (d) Simple

Directions (Q. Nos. 25 and 26) Arrange the given words

4. Selfish

(b) One (d) Three

word. The last letter of this word is the answer.

find the third word? Singer, Single, Sinister, Simple

25. 1. Select

34. How many meaningful words can be formed from the letters ADRW, using each letter only once in each word?

dictionary?

(a) R (c) I

(b) DETERMINE (d) DIET

(b) E (d) K

43. If a meaningful word can be formed by rearranging

33. How many meaningful three letter words can be formed with the letters, W, N and O using each letter only once in each word?

the letters ‘USCALA’. The first letter of the word so formed is, the answer. If no such word can be formed the answer is X.

(a) None (c) Two

(a) C (c) A

(b) One (d) Three

https://sscstudy.com/

(b) S (d) L

https://sscstudy.com/

29

GENERAL MENTAL ABILITY

44. What is the last letter of the word which is formed by

46. DAXEPN

rearranging the following letters? C P A O E C K (a bird) (a) E (c) P

(a) INCREASE (c) STILL

(b) K (d) C

47. HRADTE (a) DECREASE (b) LOSS (c) REDUCTION

45. The letters of the word ‘NUMKIPP’ are disorder. If they are arranged in proper order, the name of a vegetable is formed. What is the last letter of the word so formed? (a) K (c) N

(b) REDUCE (d) DECREASE

48. DNHBEI (a) FRONT (c) BACK

(b) M (d) P

Directions (Q. Nos. 46-48) In each of these questions,

(d) SCARCITY (b) SIDE (d) LAST

49. Unscramble the letters ‘CCITRKE’ to form an English word and find the fifth letter of the unscrambled word?

jumbled letters of a meaningful word are given. You are required to rearrange these letters and select from the given alternatives, the word which is almost similar in meaning to the rearranged word.

(a) C (c) E

(b) K (d) I

ANSWERS 1. 11. 21. 31. 41.

(d) (c) (b) (b) (c)

2. 12. 22. 32. 42.

3. 13. 23. 33. 43.

(b) (d) (d) (b) (a)

(c) (d) (b) (d) (a)

4. 14. 24. 34. 44.

(b) (c) (a) (c) (b)

5. 15. 25. 35. 45.

(d) (d) (c) (c) (c)

6. 16. 26. 36. 46.

(a) (a) (c) (a) (a)

7. 17. 27. 37. 47.

(d) (c) (d) (d) (d)

8. 18. 28. 38. 48.

9. 19. 29. 39. 49.

(b) (a) (d) (d) (c)

10. 20. 30. 40.

(a) (a) (a) (c) (b)

(d) (a) (c) (b)

Hints & Solutions 1. (d)

7. (d) The new letter series obtained on

ABCDEFGHIJKLMNOPQRSTUVWXYZ

reversing the order of the English alphabet is ZYXWVUTSRQPONMLKJIHGFE DCBA Since, the series has an even number of letters (i.e. 26). There is no such letter which lies exactly in the middle.

15th from right

2. (b) Letters between G and Q G H I J K L M N O P Q Middle letter Here, L is midway between G and Q.

8. (b) The new alphabet series is ZYXWVUTSRQPONMLKJIHGFE DCBA The ninth letter from the right is I. The fifth letter to the left of I is N.

3. (c) Fourth letter to the left of I = E ∴Fifth letter to the right of E = J

4. (b) Sixth letter from left = F Fourteenth letter from right = M Letters between F and M = G

H

I

J

K

Letters exactly in the middle

5. (d) 19th letter from left = S 3rd letter to the right of S = V

6. (a) 16th letter from the left = P 8th letter to the left of P = H

L

12. (d)

P

R

I

S

O

N

So, PS, RN, RO, ON, four pairs are present.

13. (d) K L

N G

I

So, KI, IG, LN, KL, KG, KN, i.e. six pairs are present.

14. (c) C H I L D R E N

9. (a) The new alphabet series is

So, EI, HI, IN, HN, i.e. four pairs are present.

ACEGIKMOQSUWY The tenth letter from the right is G.

15. (d) P R O D U C T

10. (d) The new alphabet series is

So, PI, RU and ON, i.e. three pairs are present.

BADCFEHGJILKNMPORQTSVU XWZY Clearly, the seventh letter from the left is H.

16. (a) The new letter sequence is IBUTEDISTR as shown below 1 2

11. (c)

I

3

4

5

6

7

8

9 10

D I S T R I B U T E

C R E A T

I

V E

So, CE, AE and TV, three pairs are present. So, the fifth letter from the left is E.

https://sscstudy.com/

O N

https://sscstudy.com/

30

CUET (UG) Section III : General Test

17. (c) D O C U M E N T A T I O N 1

2

3

4

5

7

6

8

9

10 11 12 13

The new letter sequence is DETNMOUTACION. So, the eleventh letter from the right is T.

25. (c) Seldom, Select, Selfish, Seller, Send i.e. 2, 1, 4, 5, 3. 26. (c) Problem, Project, Property, Protein, Proverb i.e. 2, 5, 4, 1, 3 27. (d) The correct alphabetical order is Sample, Several, She, Showed, Snaps.

18. (a) Word

S

T

O

N

E

D

Alphabetical order

D

E

N

O

S

T

There is no such letter.

19. (a) Word

E

N

G

L

I

S

H

Alphabetical order

E

G

H

I

L

N

S

20. (a) Plain, Plane, Player, Plenty Clearly, plane is at second place.

21. (b) Category, Catenation, Caterpillar, Cathedral Clearly, catenation is at second place.

22. (d) Converse, Curator, Current, Cushion Clearly, cushion is at fourth place. 23. (b) Racket, Radar, Radiate, Radical, Clearly, Radiate is at third place. 24. (a) Simple → Singer → Single → Sinister Clearly, Single is at third place.

28. (d) All letters of the word MAGIC are

37. (d) Since, 1st, 4th, 7th and 11th letters are I, E, R and T, respectively. hence, two meaningful words RITE and TIRE can be formed. ∴ Required answer = X

38. (d) CODING → (3, 4, 5, 2, 6, 1) 39. (c) STARVE → (4, 5, 2, 3, 1, 6) 40. (b) HERMIT → (4, 1, 6, 2, 3, 5)

present in the given word.

41. (c) NATURE → (4, 3, 2, 5, 1, 6)

29. (a) All letters of the word MASTER are present may be only once but this word can be formed.

42. (a) After arranging the given letters

30. (c) All letters of th word EMINENT are present in the given word.

43. (a) The correct word is CASUAL and first letter is ‘C’.

31. (b) FAULTY cannot be formed using the letters of the given word, as it contains U.

44. (b) The name of the bird is ‘PEACOCK’

32. (b) DETERMINE cannot be formed using the letters of the given word, as it contains three E.

45. (c) The name of that vegetable would be

33. (d) Three meaningful words can be

46. (a) The word is ‘EXPAND’ and the

formed using W, N and O i.e WON, OWN and NOW

34. (c) Two meaningful words can be formed viz. DRAW and WARD

35. (c) Meaningful word = TODAY ∴

Last letter = Y

36. (a) Meaningful word = SERVICE ∴Fifth letter from right = R

https://sscstudy.com/

in proper sequence, we get the river name KAVERI and its fifth letter from left is ‘R’.

and the last letter is ‘K’.

‘PUMPKIN’ and last letter of the word is ‘N’. meaning is ‘INCREASE’.

47. (d)The word is ‘DEARTH’ and the meaning is ‘SCARCITY’.

48. (c) The word is ‘BEHIND’ and the meaning is ‘BACK’.

49. (b) Word formed is ‘CRICKET’ and the fifth letter of this word is ‘K’.

https://sscstudy.com/

GENERAL MENTAL ABILITY

31

CHAPTER 06

Mathematical Operations Mathematical operation can be defined as the simplification of expression containing numbers and different mathematical signs. While simplifying a mathematical problem, one must follow ‘VBODMAS’ rule. The order of various operations is the same as the order of letters in ‘VBODMAS’ from left to right.

Sol. (c) Converting alphabets into mathematical symbols, we get 18 × 12 ÷ 4 + 5 − 6 = 18 ×

2. Interchanging of Signs and Numbers

V → Viniculum, (—) B → Bracket, ( ), { }, [ ] O → of D → Divide, ( ÷ ) M → Multiply, ( × ) A → Addition ( + ) S → Subtraction (−)

In this type of questions, the given equation becomes correct and fully balanced when either two signs of the equation or both the numbers and signs of the equation are interchanged . The candidate is required to find the correct pair of signs and numbers from the given alternatives.

Ex. 3 If ‘−’ means ‘+’, ‘+’ means ‘−’, ‘×’ means ‘÷’ and ‘÷’ means ‘×’, then which of the following equation is correct?

The various types of questions that are asked in competitive exams are as follows

(a) 30 − 5 + 4 ÷ 10 × 5 = 62 (c) 30 + 5 − 4 ÷ 10 × 5 = 28

1. Problem Solving by Substitution

Ex. 1 If ‘+ ’ means ‘× ’, ‘−’ means ‘÷’, ‘×’ means ‘+’ and ‘÷’ means ‘−’; compute the value of the expression 17 + 6 × 13 ÷ 8 = ? Sol. (b) Given

17 + 6 × 13 ÷ 8 ↓ ↓ ↓ 17 × 6 + 13 − 8 = 102 + 13 − 8 = 115 − 8 = 107

(b) 81 (d) 24



LHS = 30 × 5 − 4 ÷ 10 + 5 ↓ ↓ ↓ ↓ ÷ + × − = 30 ÷ 5 + 4 × 10 − 5 = 6 + 4 × 10 − 5 = 6 + 40 − 5 = (6 + 40) − 5 = 41 = RHS (using VBODMAS rule) LHS = RHS

Ex. 4 Which of the given interchanges in signs would make the given equation correct? 3 ÷ 5 × 8 + 2 − 10 = 13

(b) 107 (d) 109

(a) + and − (c) ÷ and −

Ex. 2 If P denotes ÷, Q denotes ×, R denotes + and S denotes –, then what is the value of 18 Q 12 P 4 R 5 S 6=? (a) 64 (c) 53

(b) 30 + 5 ÷ 4 − 10 × 5 = 22 (d) 30 × 5 − 4 ÷ 10 + 5 = 41

Sol. (d) From option (d),

In this type, there are substitutes for various mathematical symbols or the numerals and questions are asked related to calculation of an expression or choosing the correct equation.

(a) 100 (c) 110

12 + 5 − 6 = 53 4

(b) × and ÷ (d) ÷ and +

Sol. (d) On interchanging + and − as in option (a) the result is 24 + 8 ≠ 13 3 ÷ 5 × 8 − 2 + 10 = 5 On interchanging symbols × and ÷ as given in option (b), we get 5 3 × 5 ÷ 8 + 2 − 10 = 3 × + 2 − 10 ≠ 13 8

https://sscstudy.com/

https://sscstudy.com/

32

CUET (UG) Section III : General Test

On interchanging ÷ and − as given in option (c), we get 2 3 − 5 × 8 + 2 ÷ 10 = 3 − 5 × 8 + 10 2 = 3 − 40 + ≠ 13 10 On interchanging ÷ and + as given in option (d), we get 8 3 + 5 × 8 ÷ 2 − 10 = 3 + 5 × − 10 2 = 3 + 20 − 10 = 13

3. Balancing the Equation In this type of questions, the signs given in one of the alternatives are required to fill up the blank spaces for the signs in order to balance the given equation.

Ex. 5 Select the correct set of symbols 44 4 7 5 = 82 (a) ×, −, ÷ (c) +, −, ÷

⇒ ⇒ ⇒

11 × 7 + 5 = 82 77 + 5 = 82 82 = 82

4. Trick Based Mathematical Operations In this type of questions, there is some logic behind the given expressions. The candidate is required to find that logic and then solve the questions accordingly.

Ex. 6 If 9 × 5 × 2 = 529 and 4 × 7 × 2 = 724, then 3× 9× 8=? (a) 983 (c) 938

(b) 839 (d) 893

Sol. (a) As, 9×5×2=5

(b) +, ÷, − (d) ÷, ×, +

Similarly,

2

9 and

3×9×8=9

8

4×7×2=7

2

4

3

Sol. (d) From option (d), we get 44 ÷ 4 × 7 + 5 = 82

Practice Questions 1. If the mathematical operator ‘÷ ’ means ‘ ×’ , ‘ + ’ means

then the value of 10 R 192 P 48 S 48 P 96 Q 1 is

‘ − ’ , ‘ × ’ means ‘+’ and ‘−’ means ‘÷ ’, then 25 + 18 − 3 × 7 ÷ 3 = ? (a) 25 (c) 19

(a) 10 (c) 8

(b) 21 (d) 40

(b) 376 (d) 488

3. If ‘+’ ‘means ‘multiplication’, ‘−’ means ‘division’, ‘×’ means ‘subtraction’ and ‘÷’ means ‘addition’, then 9 + 8 ÷ 8 − 4 × 9 is (b) 11 (d) 56

4. If ‘×’ stands for +, ÷ stands for ‘−’, ‘−’ stands for ‘×’ and ‘ +’ stands for ‘÷’ then find the value of following equation. 54 ÷ 16 − 3 × 6 + 2 = ? (a) 9 (c) 8

(b) 12 (d) 15

5. If P means ‘÷ ’, R means ‘× ’, Q means ‘+’ and S means ‘− ’, then 48 P 8 Q 6 R 9 S 31 = ? (a) 60

(b) 29

(c) 31

(d) 54

then 18 C 14 A 6 B 16 D 4 = ? (b) 238

9. If ‘−’ stands for division + stands for subtraction ÷ stands for multiplication ‘×’ stands for addition, then which one of the following equations is correct ? (a) 70 − 2 + 4 ÷ 5 × 6 = 44 (c) 70 − 2 + 4 ÷ 5 × 6 = 341

(b) 70 − 2 + 4 ÷ 5 × 6 = 21 (d) 70 − 2 + 4 ÷ 5 × 6 = 36

10. If ‘−’ stands for ‘÷’ ‘+’ stands for ‘×’, ‘÷’ for ‘−’ × for ‘+’, which one of the following equations is correct ? (a) 30 − 6 + 5 × 4 ÷ 2 = 27 (c) 30 × 6 ÷ 5 − 4 + 2 = 32

(b) 30 + 6 − 5 ÷ 4 × 2 = 30 (d) 30 ÷ 6 × 5 + 4 − 2 = 40

Directions (Q.Nos. 11-16) Which of the following alternatives would make the equation correct? 11. 42 ÷ 4 + 2 − 3 × 5 = 29

6. If A means ‘+’ B means ‘−’ C means ‘× ’ and D means ÷,

(a) 254

‘÷’, which of the following statement is correct? (a) 36 R 4 S 8 Q 7 P 4 = 10 (b) 16 R 12 P 49 S 7 Q 9 = 200 (c) 32 S 8 R 9 = 160 Q 12 R 12 (d) 8 R 8 P 8 S 8 Q 8 = 57

find the value of 26 + 74 − 4 × 5 ÷ 2.

(a) 65 (c) 26

(b) 9 (d) 7

8. If P denotes ‘+’, Q denotes ‘−’, R denotes ‘×’ and S denotes

2. If ‘+’ stands for ‘× ’, ‘−’ for ‘÷’ , ‘ ×’ for ‘− ’ and ‘÷’ for ‘+’, (a) 220 (c) 478

7. If P means ‘÷’, Q means ‘+’, R means ‘−’ and S means ‘×’,

(c) 188

(d) 258

(a) + and × (c) − and ×

(b) + and − (d) ÷ and +

12. 2 × 3 + 6 − 12 ÷ 4 = 17 (a) × and + (c) + and ÷

https://sscstudy.com/

(b) + and − (d) − and ÷

https://sscstudy.com/

33

GENERAL MENTAL ABILITY

13. 12 ÷ 2 − 6 × 3 + 8 = 16

21. 6 5 4 = 34

(a) ÷ and + (c) × and +

(a) − and + (c) ÷ and ×

(b) − and + (d) ÷ and ×

22. 27 3 19 10 = 90

14. 5 + 6 ÷ 3 − 12 × 2 = 17 (a) ÷ and × (c) + and ÷

(a) × , − , ÷ (c) + , − , ÷

(b) + and × (d) + and −

(a) 3, 9 (c) 8, 9

(a) −, × and × (c) × , − and ÷

(b) 3, 8 (d) 3, 5

(a) × ÷ + (c) ÷ × +

(b) + and ÷ (d) 3 and 9

and ‘−’ and ‘=’ to get a balanced equation out of (27 # 15 # 2) # 10 # 4. Choose the right sequence from below.

equation 17 ÷ 7 − 27 + 7 × 37, then its value will be (c) 27

(d) 37

(a) + ÷ = − (c) + − ÷ =

Directions (Q. Nos. 18 and 19) On making interchanges in signs and numbers, which of the equations would be correct?

(a) 270 (c) 596

and 5.

(a) 676 (c) 647

19. Given interchanges : Signs − and × and numbers 3

(b) 476 (d) 764

28. If 2463 = 36 and 5552 = 30, then 6732 = ?

and 6.

(a) 32 (c) 34

(b) 3 − 6 × 8 = 10 (d) 3 × 6 − 4 = 33

Directions (Q. Nos. 20-24) If the following equations has to be balance, then the signs of which of the following options will be used? 20. 65 40 11 = 36 (a) − and + (c) ÷ and +

(b) 845 (d) 659

27. If 5 × 3 × 9 = 395 and 9 × 7 × 5 = 759, then 7 × 6 × 4 = ?

(b) 5 × 4 + 20 = 85 (d) 5 × 4 + 20 = 95

(a) 6 − 3 × 2 = 9 (c) 6 × 3 − 4 = 15

(b) − + = ÷ (d) + = ÷ −

26. If 4 × 6 × 2 = 351 and 3 × 9 × 8 = 287, then 9 × 5 × 6 = ?

18. Given interchanges : Signs + and × and numbers 4 (a) 5 × 4 + 20 = 40 (c) 5 × 4 + 20 = 104

(b) + ÷ × (d) + × ÷

25. Replace # sign with the mathematical operators ‘+’, ‘÷’

17. If ‘+’ and ‘÷’, ‘×’ and ‘−’ are interchanged in the (b) 17

(b)−, × and ÷ (d) × , ÷ and −

24. 8 * 8 * 1 * 7 = 8

16. (18 ÷ 9) + 3 × 5 = 45 (a) × and ÷ (c) 18 and 5

(b) + , ÷, − (d) × , + , −

23. 7 4 8 2 = 24

15. 8 × 20 ÷ 3 + 9 − 5 = 38

(a) 7

(b) × and + (d) × and −

(b) 36 (d) 39

29. If 2 + 6 + 9 = 926 and 1 + 8 + 2 = 218 then 4+3+1=? (a) 314 (c) 143

(b) 341 (d) 431

30. If 2 = 6, 4 = 12 and 8 = 24, then 10 = ?

(b) × and ÷ (d) + and ×

(a) 30 (c) 15

(b) 20 (d) 2

ANSWERS 1. 11. 21.

(d) (d) (b)

2. 12. 22.

(c) (a) (d)

3. 13. 23.

(a) (b) (c)

4. 14. 24.

(a) (a) (c)

5. 15. 25.

(b) (d) (c)

6. 16. 26.

(a) (b) (b)

https://sscstudy.com/

7. 17. 27.

(b) (a) (c)

8. 18. 28.

(d) (c) (a)

9. 19. 29.

(b) (b) (c)

10. 20. 30.

(a) (a) (a)

https://sscstudy.com/

Hints & Solutions 1. (d) Given, 25 + 18 − 3 × 7 ÷ 3 = ? ↓ ↓ ↓ = 25 − 18 ÷ 3 + 7 × 3 = 25 − 6 + 7 × 3 = 25 − 6 + 21 = 19 + 21 = 40

LHS = 42 + 2 − 3 × 5 = 42 + 2 − 15 = 44 − 15 = 29 = RHS

12. (a) On interchanging × and + , we get Given expression = 2 + 3 × 6 − 12 ÷ 4 = 2 + 3 × 6 − 3 = 2 + 18 − 3 = 17

2. (c) Using the correct symbols, we have given expression 37 − 5+ 2 2 = 13 × 37 − 5 + 2 = 481 − 5 + 2 = 478

= 26 × 74 ÷ 4 − 5 + 2 = 26 ×

3. (a) Given, 9 + 8 ÷ 8 −4 × 9 ↓ ↓ ↓ ↓ 9 × 8 + 8 ÷ 4 −9 = 9 × 8 + 2 − 9 = 72 + 2 − 9 = 74 − 9 = 65

4. (a) Using the correct symbols, we have given expression 54 − 16 × 3 + 6 ÷ 2 ⇒ 54 − 48 + 3 ⇒ 57 − 48 = 9

5. (b) Given, 48 P 8 Q 6 R 9 S 31 ↓ ↓ ↓ ↓ 48 ÷ 8 + 6 × 9 − 31 48 + 6 × 9 − 31 8 = 6 + 54 − 31 = 60 − 31 = 29

=

13. (b) On interchanging − and + , we get Given expression = 12 ÷2 + 6 × 3 − 8 = 6 + 6 × 3 − 8 = 6 + 18 − 8 = 16

14. (a) On interchanging ÷ and ×, we get the

7. (b) Given, 10 R 192 P 48 S 48 P 96 Q 1 ↓ ↓ ↓ ↓ ↓ = 10 − 192 ÷ 48 ×48 ÷ 96 +1 48 = 10 − 4 × +1 96 1 = 10 − 4 × + 1 2 = 10 − 2 + 1 = 8 + 1 = 9

we get the statement as 70 ÷ 2 − 4 × 5 + 6 ⇒ 35 − 20 + 6 ⇒ 41 − 20 = 21 = RHS

10. (a) Using the proper notation in option (a), we get the statement as 30 ÷ 6 × 5 + 4 − 2 ⇒ 5 × 5 + 2 ⇒ 25 + 2 = 27 = RHS

11. (d) From option (d), 42 ÷ 4 + 2 − 3 × 5 = 29 ↓ ↓ 42 + 4 ÷ 2 − 3 × 5 = 29

⇒ ⇒ ⇒ ⇒

(27 # 15 # 2) # 10 # 4 (27 + 15 − 2 ) ÷10 = 4 ( 42 − 2 ) ÷ 10 = 4 40 ÷ 10 = 4 4=4

15. (d) On interchanging 3 and 5, we get the

and

16. (b) On interchangng + and ÷ we get the

3×9×8=2

↓ ↓ ↓ ↓ 17 + 7 × 27 ÷ 7 − 37 27 = 17 + 7 × − 37 7 = 17 + 27 − 37 = 44 − 37 = 7

1

8

7

9×5×6=8

4

5

27. (c) As, 5 × 3 × 9 = 3

9

5

and

9×7×5=7

5

9

Similarly,

7×6×4=6

4

–1 –1 –1

Similarly,

–1 –1 –1

equation as, (18 + 9) ÷ 3 × 5 ⇒ 27 ÷ 3 × 5 ⇒ 9 × 5 = 45 = RHS

17. (a) Given, 17 ÷ 7 − 27 + 7 × 37

5

–1 –1 –1

equation as 8 × 20 ÷ 5 + 9 − 3 ⇒ 8 × 4 + 6 ⇒ 32 + 6 = 38 = RHS

7

18. (c) On interchanging + and × and 4 and 5 in option (c), we get the equation as 4 + 5 × 20 = 104 = RHS

19. (b) On interchanging − and × and 3 and 6 in option (b) we get the equation as 6 × 3 − 8 = 10 ⇒ 18 − 8 = 10 = RHS

28. (a) As, 2463 = (2 + 4 + 6) × 3 and Similarly ,

= 12 × 3 = 36 5552 = ( 5 + 5 + 5) × 2 = 15 × 2 = 30 6732 = ( 6 + 7 + 3) × 2 = 32

29. (c) As, 2 + 6 + 9 = 9

2

6

and

1+8+2=2

1

8

Similarly,

4+3+1=1

4

3

20. (a) From option (a) 65 − 40 + 11 = 76 − 40 = 36 = RHS

we get the statement as 8 × 8 + 8 ÷ 8 − 8

9. (b) Using the proper notations in option (b),

25. (c) From option (c),

26. (b) As, 4 × 6 × 2 = 3

8. (d) Using the proper notations in option (d), = 8 × 8 + 1 − 8 = 64 + 1 − 8 = 65 − 8 = 57

8 ÷ 8 × 1+ 7 ⇒ 1× 1+ 7 ⇒ 1 + 7 = 8 = RHS Hence, option (c) is correct.

equation as, 5 + 6 × 3 − 12 ÷ 2 ⇒ 5 + 18 − 6 ⇒ 23 − 6 = 17 = RHS

6. (a) Using the correct symbols, we have given expression 18 × 14 + 6 − 16 ÷ 4 ⇒ 252 + 6 − 4 ⇒ 252 + 2 = 254

24. (c) From option (c),

21. (b) From option (b), 6 × 5 + 4 ⇒ 30 + 4 = 34 = RHS

22. (d) From option (d), ⇒ ⇒ ⇒

27 × 3 + 19 − 10 = 90 81 + 19 − 10 = 90 100 − 10 = 90 90 = 90

23. (c) From option (c), 7 × 4 − 8 ÷2 ⇒ 28 − 4 = 24 = RHS Hence, option (c) is correct.

https://sscstudy.com/

30. (a) As, 2 = 6 , 4 = 12 and 8 = 24 ×3

Similarly, 10 = 30 ×3

×3

×3

https://sscstudy.com/

GENERAL MENTAL ABILITY

35

CHAPTER 07

Direction Sense Test Direction sense test deals with a sort of direction puzzle. A successive follow up of directions is formulated and the candidate is required to find the final direction or the distance between two points.

The different types of questions which are asked in exams related to direction sense test are as given below.

To solve the questions based on direction sense test the candidates must have knowledge of some important concepts, which are given below.

In this type, questions are based on final direction of a person/object or direction of a person with respect to starting point.

1. The four main directions are North (N), South (S), East (E) and West (W) and four cardinal directions are North-East (NE), North-West (NW), South-East (SE) and South-West (SW). Left

N-W

N

Right

Right

N-E E

Right

Left

S-W

Right

S

Ex. 1 Deepa moved a distance of 75 m towards the North. She then turned to the left and walking for about 25 m, turned left again and walked 80 m. Finally, she turned to the right. In which direction was she moving finally? (a) North (c) West

Left

W

1. Finding Out the Final Direction

(b) South (d) South-West

Sol. (c) According to the given information,

S-E

Left

2. Pythagoras theorem, AC2 = AB2 + BC2 2

S

Perpendicular

A

Hy po

te nu

se

Base

360°

45° 90°

270°

C

135°

225° 180°

D

A (starting point)

Clearly, she is moving towards West.

Ex. 2 A man is facing West. He turns 45° in the clockwise direction and then another 180° in the same direction and then 270° in the anti-clockwise direction. Which direction is he facing now? (a) South (c) West

(b) North-West (d) South-West

Sol. (d) According to the given information,

45° Clockwise

E

(end point)

4. Movement in the direction of clock is called clockwise movement and in the direction opposite to that of clock is called anti-clockwise movement 325°

75 m

2

3. In the morning, sunrises in the East, the shadow of person or object falls in the West. In the evening, the sunset in the West, the shadow of a person or object falls in the East.

B

80 m

E

W

AC = AB + BC

i.e.

C 25 m B

N

Anti90° clockwise 135°

360°

B

N

270°

N-W

180° 45°

A

325°

W O

E

S-W

S-E S

270° 180°

N-E

D

225°

C

Clearly, he is facing South-West now.

https://sscstudy.com/

https://sscstudy.com/

36

CUET (UG) Section III : General Test

Ex. 3 If S-E becomes West, N-E becomes South and so on. What will North become? (a) North-East (c) South-East

(b) North-West (d) South-West

Sol. (c) The directions are as follow N (S-E)

N-W (E)

Ex. 6 One day, Ravi left home and cycled 10 km Southwards, turned right and cycled 5 km and turned right and cycled 10 km and turned left and cycled 10 km. How many kilometres will he have to cycle to reach his home straight.

N-E (S) E (S-W)

W (N-E) S-W (N)

(a) 10 km (c) 20 km

S-E (W)

S (N-W)

Ex. 4 Early morning after sunrise Sangeeta went for a walk. Her brother, Ramesh, who was coming towards, her from opposite direction, saw that Sangeeta’s shadow had fallen to his right. From which direction was Ramesh coming?

Sol. (b) The direction will be as follows.

W

Ramesh E

Sun

Sangeeta

Ex. 5 In a clock at 12 : 30, hour needle is in North direction while minute needle is in South direction. In which direction would be minute needle at 12 : 45? (b) South-East (d) East

(end point)

10

Hour hand minute hand

West 9 8 7

6 5 South

2 3 Eest 4

10 West 9 8

S

(a) 5 km, West (c) 7 km, East

(b) 7 km, West (d) 5 km, North-East

Sol. (d) According to the given information, B N

11

Minute hand

N-W

6 km

3 km

C

2

N-E

W

10 km

12 1 Hour hand

7

E

Ex. 7 Kunal walks 10 km towards North. From there, he walks 6 km towards South. Then, he walks 3 km towards East. How far and in which direction is he with reference to his starting point?

At 12 : 45 North

12 1

W

C 5 km B

and minute hand is in South direction

11

10 km

10 km

Sol. (c) The hour hand of the clock is North direction At 12 : 30 North

N

A (home)

In this type of questions, the candidate have to determine both the distance and direction.

Clearly, Ramesh was coming from North.

(a) North-West (c) West

10 km D

3. Finding the Direction and Distance

S Sangeeta shadow

E

Thus, his distance from initial position A = AE = AD + DE = BC + DE = (5 + 10) km = 15 km

(b) North (d) West

N

(b) 15 km (d) 25 km

Sol. (b) According to the given information,

Clearly, North will become South-East.

(a) South (c) East

If there is no straight distance, then distance is calculated using Pythagoras theorem, i.e. In this type of questions, the candidate have to find the final distance between the starting and final points or distance between two points / persons / things.

E

S-W

D (end point)

S-E S

3 Eest

A

4

(starting point)

6 5 South

Then,

As, it is clear from the above diagram that minute hand is in West direction.

AC = (AB − BC)

= (10 − 6) = 4 km Kunal’s distance from starting point A AD = AC2 + CD2

2. Finding the Total Distance

2

The starting and end points are marked using left and right turns and the distance is calculated between them.

2

= 4 + 3 = 5 km Also, D is to the North-East of A.

https://sscstudy.com/

(Pythagoras theorem)

https://sscstudy.com/

GENERAL MENTAL ABILITY

4. Questions Based on Map In these questions, a map of a journey of a man from his starting point to end point is given. On the basis of this map the candidates are required to determine the position of starting point with respect to end point, position of end point with respect to starting point or the position of other place with respect to starting or end point.

Ex. 9 The shortest distance between Shruti’s school and her home is (a) 14 km (c) 13 km

Shruti’s School Temple Shruti’s home 90° Left

7 km 90° Right 5 km

5 km East

On the basis of above map, answer the questions given below.

(b) 12 km (d) 10 km

Sol. (Ex. Nos. 8 and 9) According to given information, N-W

N N-E

W S-W

Directions (Ex. Nos. 8 and 9) Analyse the following information and answer the questions based on it. The map of Shruti’s journey from her home to school is given below

Temple C

S

A Shruti’s Home

7 km

D

7 km

E

Shruti’s School 5 km

5 km

E S-E

5 km

B

8. (b) From the given diagram, Shruti’s school is in North-East direction with respect to her home. 9. (c) From the given diagram, BE = CD = 7 km Then, AE = 5 + 7 = 12 km and DE = CB = 5 km ∴ Shortest distance, AD = (AE)2 + (ED)2 = (12)2 + (5)2

Ex. 8 The direction of Shruti’s school with respect to her home is (a) North (c) South-West

37

(b) North-East (d) East

=

144 + 25

=

169

= 13 km

Practice Questions 1. A boy goes in South direction, then he turns towards left and travels for some distance. After that he turns right and moves certain distance. At last he turns left and travel again for some distance. Now, in which direction is he moving? (a) South (c) East

(b) West (d) North

walked 12 ft, then I turned towards right and walked 6 ft, After that I walked 6 ft in South direction and at last I walked 6 ft in the West. Then, in which direction am I standing from the original point? (b) East (d) North

3. Ram is to the South of Aishwarya and to the West of Rani. If Priyanka is to the South of Ram, then in which direction is Priyanka with respect to Rani? (a) South (c) South-West

(b) North-East (d) North

want to go the temple. The road to the right leads me towards the coffee house while straight road leads to the college. In which direction is the temple? (b) East

(c) South

(a) North-East (c) South-East

(b) West (d) North-West

direction, and then 135º in anti-clockwise direction and then again 90º in clockwise direction. Which direction is she facing now? (a) South-East (c) North-East

(b) South (d) South-West

7. A person walks away from his house at 8:00 am and observes his shadow to his right. Then, he turns towards his left and then again towards his right. Which direction is he facing now? (a) West (c) North

(b) South (d) East

8. One day, during Sunset, two friends Sudhir and Amit

4. There are four roads. I have come from the South and

(a) North

then 180º in anti-clockwise direction. What direction am I facing now?

6. Rama is facing East. He turns 90º in clockwise

2. I was facing East from where I turned to my left and

(a) West (c) South

5. I am facing North. I turn 135º in clockwise direction,

(d) West

were talking, facing each other. If Amit’s shadow was on his right then in which direction is Aman facing? If Aman faces the direction opposite of what Sudhir is facing? (a) North (c) East

https://sscstudy.com/

(b) West (d) South

https://sscstudy.com/

38

CUET (UG) Section III : General Test

9. The time in a clock is quarter past twelve. If the hour hand points to the East, then towards which direction the minute hand is pointing? (a) South-West (c) West

(b) South (d) North

(b) West

(c) East

(a) 35 m, East (c) 40 m, East

(b) 35 m, North (d) 60 m, East

19. Starting from a point P, Rohan walked 20 m towards

10. A direction pole was situated on the road crossing. Due to an accident, the pole turned in such a manner that the pointer which was showing East, started showing South. Sita, a traveller went to the wrong direction thinking it to be West. In what direction actually she was travelling? (a) North

walked 15 m. At what distance is he from his starting point and in which direction?

(d) South

South. He turned left and walked 30 m. He then turned left and wallked 20 m. He again turned left and walked 40 m and reached point Q. How far and in which direction is the point Q from the point P? (a) 20 m, West (c) 10 m, West

(b) 10 m, East (d) 10 m, North

20. Mohan left for his office in his car. He drove 15 km

10 yards. Then, turn to your left and walk 10 yards and now turn 45° to your right and go straight to cover 50 yards. Now, in what direction are you with respect to the starting point?

towards North and then 10 km towards West. He then turned to the South and covered 5 km. Further he turned to the East and moved 8 km. Finally, he turned right and drove 10 km. How far and in which direction is he from his starting point?

(a) North-East (c) South-East

(a) 2 km, West (c) 3 km, North

11. While facing East you turn to your left and walk

(b) North (d) North-West

12. A house faces North. A man coming out of his house walked straight for 10 m, turned left and walked 25 m. He then turned right and walked 5 m and again turned right and walked 25 m. How far is he from his house? (a) 15 m (c) 60 m

(b) 55 m (d) 65 m

13. A cyclist goes 30 km to North and then turning East he goes 40 km. Again, he turns to his right and goes 20 km. After this, he turns to his right and goes 40 km. How far is he from his starting point? (a) 6 km (c) 25 km

(b) 10 km (d) 40 km

14. A boat moves from port towards East. After sailing for 9 miles, it turns towards right and covers another 12 miles. If it wants to go back to the port, what is the shortest distance now from its position? (a) 21 miles (c) 18 miles

(b) 20 miles (d) 15 miles

21. Ramesh walks 10 m towards South. Turning to the left, he walks 20 m and then moves to his right. After moving a distance of 20 m, he turns to the right and then walks 20 m. Finally, he turns to the right and moves a distance of 10 m. How far and in which direction is he from the starting point? (a) 10 m, North (c) 20 m, North

place Q. From there, he turns towards North-West and reaches place R. Then, he turns towards South-West and walks to a place S. From there, he turns towards North-West and finally reaches place T. Which of the following figures shows the movement of Raju? R

T

(2)

(a) 6 m (c) 8 km

R

S P

R S

S

Q

(4)

P

P

T

Directions (Q. Nos. 23 and 24) In the below diagram, the position of seven points are shown. G

(b) 14 km (d) 10 km

F

A

17. A and B starts walking from same point. A goes North and covers 3 km, then turns right and covers 4 km. B goes West and covers 5 km. Then, turns right and covers 3 km. How far apart are they from each other? (a) 10 km (c) 8 km

Q

T

Q (3)

East and further travels 4 km due North. How far is he from the starting point?

R

P

rides 20 km, again turns left and rides 20 km. How far is he from his starting point?

16. A man travels 4 km due North, then travels 6 km due

T

Q S

(1)

(b) 10 km (d) 20 2 km

(b) 20 m, South (d) 10 m, South

22. Raju starts from a place P towards North and reaches

15. A cyclist rides 40 km to the East, turns North and (a) 0 km (c) 15 km

(b) 5 km, East (d) 6 km, South

(b) 9 km (d) 5 km

18. Rohit walked 25 m towards South. Then he turned to his left and walked 20 m. He then turned to his left and walked 25 m. He again turned to his right and

E B

D C

Here, Point A is 11 m North of point B. Point C is 11 m East of point B. Point D is 6 m North of point C. Point E is 7 m West of point D. Point F is 8 m North of point E. Point G is 4 m West of point F. On the basis of the above information, answer the questions asked.

https://sscstudy.com/

https://sscstudy.com/

39

GENERAL MENTAL ABILITY

Directions (Q. Nos. 27 and 28) Study the following map carefully and answer the questions asked.

23. How far is point F form point A? (b) 4 m (d) 5 m

Agarwal Av

Jain Oil Company HQ

Directions (Q. Nos. 25-26) In the following map the position of 5 checkposts A, B, C, D and E are shown. Study the map carefully and answer the questions asked.

Rock AV

Checkpost E

Checkpost D

Garh

PVS Mall

In the map =1 km

Metro Plaza

P Plaza St Arihant Sharma Publications Sweets Building

West st

Road

Dun Rd

Checkpost B

O

Lena Bank Building

M

Delhi

(b) 5 m, North (d) 4 m, South

Josh St

(a) 3 m, North (c) 4 m, North

N Gupta St

Rd

N

Raj St

TP St

A?

Plaza Way

24. How far and in which direction is point G from point

Jain Nagar Rd

(a) 43 m (c) 3 m

Greater Av

Checkpost A

27. Astha is in Arihant Publications Building and can see Sharma sweets in her front which direction is she facing?

Checkpost C

(a) East (c) North

25. If a checkpost A is in the East direction of checkpost C, then in which direction is the checkpost B which respect to chckpost A? (a) North-West (c) South-East

28. Yatharth starts from location ‘P’ and proceeds as

(b) North-East (d) South-West

follows right onto plaza st heading West, second right heading North, first left heading West and stops as location ‘N’. Where is location ‘O’ in relation to his current location?

26. Find the shortest distance between checkpost E and checkpost A. (a) 17 km (c) 18 km

(b) West (d) South

(a) South-East (c) North

(b) 13 km (d) 20 km

(b) South-West (d) East

ANSWERS 1. 11. 21.

(c) (d) (b)

2. 12. 22.

(d) (a) (a)

3. 13. 23.

(c) (b) (d)

4. 14. 24.

(d) (d) (a)

5. 15. 25.

(d) (d) (a)

6. 16. 26.

(a) (d) (b)

https://sscstudy.com/

7. 17. 27.

(b) (b) (a)

8. 18. 28.

(a) (a) (a)

9. 19.

(b) (c)

10. 20.

(a) (a)

https://sscstudy.com/

40

CUET (UG) Section III : General Test

Hints & Solutions 1. (c) The direction diagram is as follows, Starting point

N

N-W

N

W S-W

12. (a) According to the question,

Hence, now Rama is facing in South-East direction.

Clearly, he is moving towards East direction.

2. (b) The direction diagram is as follows,

is towards his right at 8:00 am that means the person is walking towards South direction. 6 ft

12 ft

House

N

Clearly, he is facing South direction.

8. (a) According to the question, at Sunset

Clearly, I am towards North from my initial position.

3. (c) According to the question, Aishwarya N-W Rani

Ram

N

N-E

S-W

S

9. (b) Time quarter past twelve means that the time is 12 : 15.

S-E

Original position

Priyanka

12

2

College

N Coffee W house

Right

9

E

6

N-E

45° E

S

S-E

Hence, now I am facing in North-West direction.

6. (a) Clockwise turn = 90º + 90º = 180º Anti-clockwise turn = 135º Difference =180º – 135º = 45° clockwise

A

25 m

W

E

10 m

S

House

13. (b) According to the question, 40 km

A

B N

20 km

30 km D

W

C

40 km

Starting position

E S

Final position O

∴Required distance, OD = OA – AD = OA – BC = 30 – 20 = 10 km

14. (d) According to the question, Port O

9 miles

A N 12 miles W

W

N

E S

B

E

S

N E

S

= 180º – 135º = 45º anti-clockwise. (anti-clockwise) (clockwise)

S

2

3

Hence, when hour hand is pointing towards East, then the minute hand is pointing towards South.

Clearly, temple is towards West.

W

1 4

W

5. (d) Difference in degrees

E

W

12

5

10. (a)

N

11

5

S

Starting point

10

7

4

8

9

6

3 7

4. (d) According to the question,

8

1

10

Temple

After shifting N

11

Clearly, Priyanka is towards South-West of Rani.

Left

Amit’s shadow is towards his right that means he is facing North. Now, Sudhir is standing in front of him that means Sudhir is facing South. Now, Aman faces the direction opposite of Sudhir. ∴ Aman is facing North direction.

E

W

B

∴ Required distance = OD = OA+AD = OA+BC = 10+5 = 15 m

E S

S

N

O

W

Initial position

D

5m

N

E

W

6 ft Final position

25 m

C

7. (b) According to the question, the shadow

6 ft

S-W

S-E

S

S

N-W

E

45°

E

W

Then, turned his left in the direction of West and walked 10 yards. Now, he turned 45° to his right and walked 50 yards straight in the same direction. Now, the direction of the man with respect to his starting point is North-West.

N-E

The, pointer which was showing West started showing South. Hence, the pointer turned 90° clockwise. Now, Sita went to the direction thinking it as West. The original direction will be +90° clockwise i.e. North direction.

∴ Required distance, OB = OA 2 + AB 2 [using Pythagoras theorem] = 9 2 + 12 2 = 225 = 15 miles

15. (d) According to the question, C

20 km

11. (d) Let A be the initial position of the man. He initially faced East and then turned his left in the direction of North and walked 10 yards. 45° 10 yards C

B 10 yards A

https://sscstudy.com/

N

20 km W Starting point O

D 50 yards

B

D 40 km

A

In ∆OCD, OD = OA − AD = OA − BC = 40 − 20 = 20 km and CD = AB = 20 km. Now, required distance, OC = CD 2 + OD 2 = 20 2 + 20 2 =

800 = 20 2 km

E S

https://sscstudy.com/

41

GENERAL MENTAL ABILITY

16. (d) According to the question,

B 5 km C

C Final position

A

N

4 km W

6 km B

4 km

Final E position

D

OD = AB = 6 km CD = BC + BD = BC + OA = 4 + 4 = 8 km Now, required distance

S

O Starting point

B’s Final position 4 km A’s Final G C D position 3 km 3 km O Starting point N E

B

W

E

20 m

C 15 m

R

B

= 144 + 25 = 169

S

= 13 km

This situation is traced by option (a). Sol. (Q. Nos. 23 and 24) According to the information, the direction diagram is as given below.

27. (a) According to the question, N

S

W

B

N W

Arihant Publications Building

23. (d) According to the question, the direction diagram is as given below. G

W

4m

F

E

Sharma Sweets

28. (a) According to the question, N-W

N

O

A

and

S

Clearly, Sharma Sweets is in East direction of Arihant Publications Building, so Astha is facing towards East direction.

3m S

We can see that point ‘Q’ is to the West of point ‘P’. Now, Required distance = PQ = QC − PC = QC − AB = 40 − 30 = 10 m ∴ Q is 10 m to the West of P.

E

E S

N

C

20 m

30 m

= (12)2 + (5)2 E

P

19. (c) According to the question,

A

= (MA)2 + (EM)2

W

∴ Required distance, OD=OC+CD = BA + CD = 20 + 15 = 35 m Also, point D is to the East of point O. ∴ He is 35 m towards East from his starting point.

Q Final position

= 5 × 1 = 5 km

N

S

E

A Checkpost

Checkpost M (Let)

= 12 × 1 = 12 km

Q

N

D Final position W

Checkpost C

∴Required shortest distance, EA

T

Starting point P C 20 m

Checkpost E

22. (a) According to the question, the

18. (a) According to the question,

20 m

Checkpost D

Distance between checkpost E and checkpost M = Distance between checkpost D and checkpost C = 5 × o

∴ Required distance, GD = GC + CD = EO + CD = 5 + 4 = 9 km

A

26. (b) According to the figure,

Point E is to the South of Point O. Now, Required distance = OE = OA + AE = OA + BC − ED = 10 + 20 − 10 = 20 m Hence, Ramesh is 20 m towards, South from his starting point.

S

25 m

25. (a) Since the checkpost A is in the East direction of checkpost C, so the checkpost B is in North-West direction with respect to checkpost A.

Distance between checkpost M and checkpost A = 12 × o

S

C

direction diagram is as given below.

O Starting point 25 m

24. (a) Point G is in the North direction from point A and AG = 3 m [from above solution]

N 20 m

Final E position 10 m D 20 m

17. (b) According to the question,

W

E

9 + 16 = 5 m

Starting point

O 10 m A

6 2 + 8 2 = 100 = 10 km

5 km

W

AG 2 + GF 2

21. (b) According to the question,

OC = OD 2 + CD 2

E

N 15 km

AF = =

A

Point E is to the West of point O. Now, required distance, EO = AB − CD = 10 − 8 = 2 km Hence, he is 2 km towards West from his starting point.

In ∆OCD, and ⇒

=

10 km D 8 km 10 km

E S

Starting point O



20. (a) According to the question,

AG = BG − AB = (CD + EF ) − ( AB ) = ( 6 + 8) − 11 = 3 m GF = 4 m

https://sscstudy.com/

P

N N-E

W

E

S-W

S-E

S

Clearly, location O is in South-East direction in relation to his current location i.e., N.

https://sscstudy.com/

42

CUET (UG) Section III : General Test

CHAPTER 08

Blood Relations Blood relation between two individuals is defined as a relation between them by the virtue of their birth. Here, a chain of relationship is given in the form of information and the success of candidate depends upon his/her knowledge of blood relations. These relationships can be used to solve various types of problems related to blood relations.

Important Relations

Relation

Relation name

Mother’s or father’s daughter

Sister

Son’s wife

Daughter - in-law

Husband’s or wife’s sister

Sister-in-law

Daughter’s husband

Son-in-law

Husband’s or wife’s brother

Brother-in-law

Brother’s son

Nephew

Brother’s daughter

Niece

Relation name

Sister’s husband

Brother-in-law

Father’s father

Grandfather

Brother’s wife

Sister-in-law

Fother’s mother

Grandmother

Father’s brother

Uncle

Grandson’s or Granddaughter’s daughter or son

Great granddaughter or son

Father’s sister

Aunt

Children of uncle or aunt

Cousin

Wife of uncle

Aunt

Husband of aunt

Uncle

Relations from paternal side Relation

There are three types of questions, which are asked in competitive exams.

1. Jumbled-up Descriptions

Relations from Maternal Side Relation

Relation name

Mother’s father

Maternal grandfather

Mother’s mother

Maternal grandmother

Mother’s brother

Uncle

Mother’s sister

Aunt

Children of maternal uncle or Aunt Cousin

In this type, a round about description is given in the form of certain small relationships and candidate is required to analyse the whole chain of relations and then find out the direct relationship between the concerned persons.

Ex. 1 Pointing towards a person in the photograph, Anjali said, ‘He is the only son of the father of my sister’s brother’. How is that person related to Anjali?

Wife of maternal uncle

Aunt

(a) Mother (c) Maternal uncle

Husband of maternal aunt

Uncle

Sol. (d) The relations may be analysed as Sister’s

Other Relations Relation

Relation name

Grandfather’s or Grandmother’s son

Father or Uncle

Grandfather’s or Grandmother’s only son

Father

(b) Father (d) Brother

brother-Brother, Brother’s father-Father. Father’s son-Brother. So, the person in the photograph is Anjali’s brother as shown in the figure given below (+) Father Brother

Grandfather’s or Grandmother’s daughter Aunt Mother’s or father’s son

Brother

Anjali

https://sscstudy.com/

Sister (–)

Brother (son) (+)

(

(+)

Male

(–)

Female

)

https://sscstudy.com/

GENERAL MENTAL ABILITY

Ex. 2 Kitty said, ‘‘Uthara is one of the two daughters of my mother’s brother’s wife’’. How is Kitty’s mother related to Uthara’s sister? (a) Maternal Aunt (c) Grandmother

3. Coded Relations In this type, the relationships are represented by certain specific codes or symbols such as + ,− ,÷ ,× ,$, @ etc. Candidate has to analyse the codes to determine the required relationship.

(b) Mother (d) Sister

Sol. (a) According to the question, Brother Mother

Ex. 5 If ‘P+Q’ means ‘P is the father of Q’, ‘P×Q’ means ‘P is the brother of Q’, ‘P-Q’ means ‘P is the mother of Q’, then which of the following is definitely true about C−A+B?

Wife

Maternal Aunt Sister

Kitty

Daughter

Uthara

(a) B is the son of A (c) B is the father of A

From the above, it is clear that Kitty’s mother is the maternal aunt of Uthara’s sister.

C

A (+) (Father)

Ex. 3 If ‘A’ is the son of ‘B’ and is the father of ‘C’, how is ‘B’ related to ‘C’?

B

(b) Son (d) Grandchild

Clearly, A is the son of C.

Ex. 6 If ‘P × Q’ means P is mother of Q.

Sol. (c) According to the given information,



B

Father



Grandparent

Son (+)A



(a) W ÷ T ÷ J (c) J × T × W

C

(b) Brother (d) Uncle

Sol. (d) According to the given information, Brothers

Brother (Sisters) (–)C (–)D Son (+)

B(+)

Uncle

(

(+)

Male

(–)

Female

(b) J ÷ T × W (d) None of these

Sol. (d) From option (a),

Ex. 4 A and B are brothers. C and D are sisters. A ‘s son is D’s brother. How is B related to C ?

A(+)

P + Q’ means P is sister of Q. P ÷ Q’ means P is father of Q. P − Q’ means P is brother of Q.

Then, which of the following means J is paternal grandfather of W?

From the above it is clear that, B is the grandparent of C.

(a) Father (c) Grandfather

(–) (Mother)

(Son)

In this type, mutual blood relations of more than two persons are mentioned. Candidate has to analyse the information and find the required relation.

Clearly, B is uncle of C.

(b) A is the son of C (d) C is the mother of B

Sol. (b) According to the given information,

2. Relation Puzzle

(a) Father (c) Grandparent

43

)

W ÷ T ⇒ W is the father of T T ÷ J ⇒ T is the father of J So, W is the grandfather of J. From option (b), J ÷ T ⇒ J is the father of T T × W ⇒ T is the mother of W. So, J is the maternal grandfather of W From option (c), J × T ⇒ J is the mother of T T × W ⇒ T is the mother of W So, J is the maternal grandmother of W. So, all the given relations are false.

https://sscstudy.com/

https://sscstudy.com/

44

CUET (UG) Section III : General Test

Practice Questions 1. Introducing Reena, Monika said, ‘She is the only

11. Showing a man, a woman said, “His brother’s father is

daughter of my father’s only daughter.’ How is Monika related to Reena?

the only son of my grandfather”. How is the woman related to the man?

(a) Aunt (c) Mother

(a) Aunt (c) Daughter

(b) Niece (d) Cousin

2. Pointing to a man, a woman said, ‘His mother is the

(b) Sister (d) Mother

12. Mathew told his friend Shyam, pointing to a

only daughter of my mother.’ How is the woman related to the mother?

photograph, “Her father is the only son of my mother”. The photograph is of whom?

(a) Mother (c) Sister

(a) Mathew’s niece (c) Mathew’s daughter

(b) Daughter (d) Grandmother

3. Pointing to Ketan, Namrata said, ‘He is the son of my father’s only son’. How is Ketan’s mother related to Namrata? (a) Daughter (c) Sister

(b) Aunt (d) Sister-in-law

4. Pointing to a man on the stage, Rashi said, ‘He is the brother of the daughter of the wife of my husband’. How is the man on the stage related to Rashi? (a) Son (c) Cousin

(b) Husband (d) Nephew

5. How is Sameer related to Akbar if Sameer introduced Akbar as his maternal grandmother’s only son’s son? (a) Brother (c) Maternal Uncle

(b) Son (d) Cousin

(b) Brother (d) Nephew

7. Malini said, ‘‘Rohit is my maternal aunt’s mother’s only son’s son’’. How is Malini related to Rohit? (b) Mother (d) Daughter

8. Ram said, ‘‘Sita is my paternal great grandfather’s only son’s only daughter-in-law’’. How is Sita related to Ram? (a) Maternal Aunt (c) Mother

(a) His son’s (c) His father’s

(b) His nephew (d) His own

14. Bina is the daughter of Mohan who is the only son-in-law of Meena. Meena has only one child. Kiran is the granddaughter of Meena. How is Kiran Related to Bina? (a) Sister (c) Maternal aunt

(b) Daughter (d) Mother

(a) Daughter (c) Grandmother

(b) Granddaughter (d) Mother

16. P’s father is Q’s son. M is the paternal uncle of P and N is the brother of Q. How is N related to M? (a) Brother (c) Cousin

(b) Nephew (d) None of these

17. Q is the brother of R. P is the sister of Q. T is the brother of S. S is the daughter of R. How is P related to T? (a) Uncle (c) Brother

(b) Aunt (d) Mother

18. M is the son of N. O is the father of N. P is the father of M. How is N related to P?

(b) Paternal Aunt (d) Sister

9. Pointing to a photograph, a person tells his friend, ‘She is the granddaughter of the elder brother of my father’. How is the girl in the photograph related to this man? (a) Niece (c) Aunt

brother or sister but that man’s father is my father’s son”. At whose photograph was the person looking at?

daughter-in-law of K. How is E related to K?

younger of the two brothers of the daughter of my father’s wife.’ How is the boy playing football related to Deepak?

(a) Cousin (c) Sister

13. Looking at a photograph, a person said, “I have no

15. E is the daughter of P who is the husband of the only

6. Deepak said to Nitin, ‘That boy playing football is the

(a) Son (c) Cousin

(b) Mathew’s mother (d) Mathew’s sister

(b) Sister (d) Sister-in-law

(a) Wife (c) Father

19. P’s father Q, is B’s paternal uncle and A’ s husband M, is P’s only paternal uncle. How is A related to B? (a) Cousin (c) Mother

10. Pointing to a gentleman, Deepak said, ‘His only

(b) Husband (d) Mother

(b) Aunt (d) Data inadequate

20. Q is the son of P, X is the daughter of Q,R is the

brother is the father of my daughter’s father’. How is the gentleman related to Deepak?

aunty (Bua) of X and L is the son of R, then what is L to P?

(a) Grandfather (c) Brother-in-law

(a) Grandson (c) Daughter

(b) Father (d) Uncle

https://sscstudy.com/

(b) Grand daughter (d) Nephew

https://sscstudy.com/

GENERAL MENTAL ABILITY

24. If,

21. If ‘P$Q’ means ‘P is father of Q’, ‘P # Q’ means ‘P is

‘P ÷ Q’ means ‘P is sister of Q’ ‘P × Q’ means ‘P is brother of Q’ ‘P − Q’ means ‘P is mother of Q’ ‘P + Q’ means ‘P is father of Q’ Then, which of the following means ‘ M is maternal uncle of T’ ?

mother of Q’, ‘P * Q’ means ‘P is sister of Q’, then how is D related to N in N# A $ B * D?

• • • •

(a) Nephew (b) Grandson (c) Grand daughter (d) Data inadequate

(a) M ÷ K + T (b) M × K + T (c) M × K − T (d) M ÷ K − T

22. If ‘P × Q’ means ‘P is the daughter of Q’, ‘P + Q’ means ‘P is the father of Q’, ‘ P ÷ Q, means ‘P is the mother of Q’ and ‘P − Q’ means ‘P is the brother of Q’, then in the expression A ÷ B + C − E × F, how is A related to F? (a) Mother (c) Daughter-in-law

45

(b) Aunt (d) None of these

25. If

‘P × Q’ means ‘P is wife of Q’ ‘P ÷ Q’ means ‘P is father of Q’ ‘P + Q’ means ‘P is son of Q’ ‘P − Q’ means ‘P is sister of Q’ Then which of the following represent’s S is mother of T?

• • • •

23. ‘P−Q’ means ‘P is the mother of Q’, ‘P × Q’ means ‘P is the father of Q’ and ‘P+Q’ means ‘P is the daughter of Q’. Now, if M − N × T + Z, then which of the following is not true? (a) T is N’s daughter (b) N is wife of Z (c) M is mother-in-law of Z (d) T is grand daughter of M

(a) S × M ÷ H − T (c) M × S ÷ H − T

(b) S × M + H − T (d) M × S ÷ H + T

ANSWERS 1. 11. 21.

2. 12. 22.

(c) (b) (d)

(a) (c) (d)

3. 13. 23.

(d) (a) (b)

4. 14. 24.

(a) (a) (c)

5. 15. 25.

6. 16.

(d) (b) (a)

7. 17.

(b) (d)

(a) (b)

8. 18.

(c) (a)

9. 19.

(a) (c)

10. 20.

(d) (a)

Hints & Solutions 1. (c) Monika’s fathers’s only daughter-Monika. So, Reena is Monika’s daughter, i.e. Monika is Reena’s mother.

brother- Deepak’s younger brother. So, the boy is Deepak’s brother.

7. (a) According to the question,

2. (a) Only daughter of my mother- Myself. So, 3. (d) Namrata’s father’s only son- Namrata’s brother. So, Ketan is the son of Namrata’s brother. Thus, Ketan’s mother is the wife of Namrata’s brother, i.e. Namrata’s sister-in -law.

Only s Maternal Grand mother

the woman is man’s mother.

father-Deepak’s father. So, the man is the brother of Deepak’s father, i.e. Deepak’s uncle.

Mother Son Rohit

Malini

4. (a) Wife of Rashi’s husband-Rashi Brother of daughter-Son So, the man on the stage is Rashi’s son.

10. (d) Father of Deepak’s daughter’s

on

Maternal aunt

Clearly, Rohit is the cousin of Malini.

8. (c) According to the question,

al at er n M

Grand mother

Father

Only son Brother

Son

Sameer

Akbar

Cousin Clearly, Akbar is the cousin of Sameer.

6. (b) Father’s wife- Mother, Mother’s daughter-Sister, Deepak’s sister’s younger

Only son

13. (a) Here, person’s father’s only son is the person himself and the man in the photograph is the son of that person. So, the person is looking at his son’s photograph. 14. (a) According to the question,

Great grandfather

Daughter-in-law Father Grand father

Sita

Wife Father Ram

er oth

M

son Only law in-

Child

Mohan Daughter (–)Bina

Clearly, Sita is the mother of Ram.

Meena

Daughter Sister

Kiran

Clearly, Kiran is the sister of Bina.

https://sscstudy.com/

Grand daughter

Mother

11. (b) Here, only son of woman’s grandfather is the father of woman. Also man’s brother’s father is the father of that woman. Now, it is clear that the woman is the sister of that man. 12. (c) Here only son of Mathew’s mother means Mathew himself. Now, it is clear that the photograph is of Mathew’s daughter.

Cousin

5. (d) According to the question,

Mother

9. (a) Brother of father-uncle Uncle’s granddaughter-Daughter of uncle’s son-Daughter of Cousin-Niece

https://sscstudy.com/

46

CUET (UG) Section III : General Test

Grand daughter

15. (b) According to the question, K

19. (c) According to the question,

Daughter-in-law

P+

Couple

A(–)

Brother M(+)

(–)

Father

B

ot

M

(+)

P

16. (d) According to the question,

?(+)

Son

N(+)

Brother

Grandson Sister

(+) Q

Uncle (–)

Father

Daughter

Clearly, P is aunt of T.

So, option (c) gives the stated relation.

Father

T(+)

Sister

D B(–) D is either granddaughter or grandson of N.

22. (d) According to the question,

18. (a) According to the question,

A (–)

S(–)

Mother (–)

N

Son

Married Couple

P(+)

B(+)

(+)M

Clearly, N is the wife of P.

Couple

F

Father

Father

(+)

C

25. (a) From option (a), S × M → S is wife of M M ÷ H → M is father of H H − T → H is sister of T

Mother-in-law

O(+) Father

Father Brother (b) M × K + T = M ← K ← T

M is the maternal uncle of T.

A(+)

Aunt

M is the aunt of T

Brother (c) M × K − T = M ← K ← T

Mother

R

Brother

(+) father

Mother

17. (b) According to the question,

S(–)

24. (c) From option

M is the paternal uncle of T

N(–)

Brother Q(+)

T–

(a) M ÷ K + T = M ← K ← T

21. (d) According to the question,

Clearly, N is uncle of M.

(–)

(Father)

( − ) sister

L(+)

X

Z

Clearly, N is husband of Z

Son

Now, it is clear that L is the grandson of P.

P

Sister P(–)

R(–)

Daughter

M(+)

Couple

N

20. (a) According to the question.

Clearly, E is the granddaughter of K.

Son

(Mother)

P

Clearly, A is mother of B

E(–)

Brother Q(+)

M (–)

Mother

r he

Daughter

Q(+)

Father

Husband

23. (b) According to the question,

(–)

Daughter Brother

E

Couple

Father T

Sister

(–)

A is mother-in-law of F.

https://sscstudy.com/

M(+)

Clearly, S is mother of T.

H(–)

https://sscstudy.com/

GENERAL MENTAL ABILITY

47

CHAPTER 09

Logical Sequence of Words The arrangement of words in an order based on reality and facts that are universally accepted is called logical sequence of words. In this type of questions, a group of interrelated words are given in any order. The candidate is required to find out the proper logical arrangement of those words.

Directions (Ex. Nos. 1-3) In each of the following questions arrange the words in a meaningful/logical order and then select the appropriate sequence from the alternatives given below each group of words.

Ex. 1 1. Birth 3. Marriage

2. Death 4. Education

(a) 1, 2, 3, 4 (c) 1, 4, 3, 2

Ex. 2 1. Phrase 3. Word

2. Letter 4. Sentence

(a) 1, 2, 3, 4 (c) 2, 3, 1, 4

(b) 1, 3, 2, 4 (d) 2, 3, 4, 1

Sol. (c) A group of letter makes a word. A group of words makes a phrase. A group of phrases makes a sentence. So, the correct order becomes 2, 3, 1, 4.

Ex. 3 1. Consultation 2. Illness 3. Doctor 4. Treatment 5. Recovery (a) 2, 3, 1, 4,5 (c) 4, 3, 1, 2, 5

(b) 4, 2, 3, 1 (d) 3, 1, 2, 4

(b) 2, 3, 4, 1, 5 (d) 5, 1, 4, 3, 2

Sol. (c) Clearly, the given words when arranged in

Sol. (a) Clearly, Illness occurs first.

order of various events as they occur in man’s life, form the sequence, Birth → Education → Marriage → Death (a) (d) (c) (b) So, the correct order becomes 1, 4, 3, 2.

Then, one goes to a doctor and after consultation with him, undergoes treatment to finally attain recovery. So, the correct order becomes 2, 3, 1, 4, 5.

Practice Questions Directions (Q. Nos. 1-20) In each of the following questions arrange the words in a meaningful/logical order and then select the appropriate sequence from the alternatives given below each group of words. 1. 1. Honey

2. Flower

(a) 1, 3, 4, 2 (c) 2, 3, 1, 4

2. 1. Butterfly

(a) 2, 4, 1, 3 (c) 4, 2, 1, 3

4. Wax

(b) 2, 1, 4, 2 (d) 4, 3, 2, 1

2. Cocoon

(a) 1, 3, 4, 2 (c) 2, 4, 1, 3

3. 1. Weaving

3. Bee

3. Egg

4. Worm

(b) 1, 4, 3, 2 (d) 3, 4, 2, 1

2. Cotton

3. Cloth 4. Thread (b) 2, 4, 3, 1 (d) 3, 1, 4, 2

4. 1. Phrase

2. Letter

(a) 1, 2, 3, 4 (c) 2, 3, 1, 4

5. 1. District

2. Village 3. State

(a) 2, 1, 4, 3 (c) 2, 4, 1, 3

6. 1. Plant

(a) 2, 1, 3, 4 (c) 1, 2, 3, 4

https://sscstudy.com/

4. Sentence

4. Block

(b) 2, 3, 4, 1 (d) 3, 2, 1, 4

2. Fruit

(a) 3, 2, 4, 1 (c) 3, 1, 4, 2

7. 1. Twilight

3. Word (b) 1, 3, 4, 4 (d) 2, 3, 4, 1

3. Seed

4. Flower

(b) 3, 1, 2, 4 (d) 3, 2, 1, 4

2. Dawn

3. Noon (b) 2, 3, 1, 4 (d) 1, 3, 2, 4

4. Night

https://sscstudy.com/

48

CUET (UG) Section III : General Test

8. 1. Pupa

2. Larva

3. Moth

14. 1. Punishment

4. Eggs (b) 4, 1, 2, 3 (d) 4, 3, 1, 2

(a) 5, 1, 2, 3, 4 (c) 4, 3, 5, 1, 2

2. Child

3. Milk (a) 1, 5, 2, 4, 3 (c) 2, 4, 3, 1, 5

(b) 2, 4, 1, 3, 5 (d) 3, 2, 1, 5, 4

3. Day

16. 1. Frog

4. Earn

5. Grass (a) 1, 3, 5, 2, 4 (c) 5, 3, 1, 4, 2

(b) 1, 3, 2, 5, 4 (d) 1, 3, 5, 2, 4

2. Night 4. Sleep

(a) 3, 2, 4, 5, 1 (c) 3, 2, 1, 4, 5

13. 1. Table

4. Think

2. Door

5. Sing

4. Room

(a) 1, 4, 2, 5, 3 (c) 2, 4, 3, 5, 1

19. 1. Index

(b) 4, 2, 1, 5, 3 (d) 1, 3, 2, 4, 5

2. Tree

4. Delivery

2. Appreciation

3. Money

5. Switch on

3. Envelope

(b) 3, 2, 1, 5, 4 (d) 2, 3, 1, 4, 5

18. 1. Compose

(b) 3, 5, 1, 4, 2 (d) 3, 5, 2, 1, 4

(a) 5, 1, 2, 4, 3 (c) 1, 2, 3, 5, 4

(b) 3, 4, 2, 5, 1 (d) 5, 3, 4, 2, 1

17. 1. Postbox 2. Letter 5. Clearance

(a) 1, 3, 5, 2, 4 (c) 3, 5, 1, 2, 4

3. Lock

2. Eagle

3. Grasshopper 4. Snake

5. Work

12. 1. Key

(b) 2, 1, 3, 4, 5 (d) 5, 2, 1, 3, 4

2. Job

(a) 1, 2, 3, 4, 5 (c) 1, 3, 5, 4, 2

3. Cutting

5. Food

(a) 1, 2, 3, 4, 5 (c) 3, 1, 2, 5, 4

5. Apply

11. 1. Tired

2. Market

4. Cooking

5. Smile

3. Examination

4. Crime

(b) 4, 3, 5, 2, 1 (d) 2, 3, 1, 4, 5

15. 1. Vegetable

4. Cry

10. 1. Study

3. Arrest

5. Judgement

(a) 4, 2, 1, 3 (c) 4, 3, 2, 1

9. 1. Mother

2. Prison

(b) 4, 1, 5, 2, 3 (d) 5, 4, 2, 1, 3

2. Contents

3. Title

4. Chapters

5. Introduction

3. Wood

(a) 3, 2, 5, 1, 4 (c) 5, 1, 4, 2, 3

4. Seed

5. Plant (a) 4, 5, 3, 2, 1 (b) 4, 5, 2, 3, 1 (c) 1, 3, 2, 4, 5 (d) 1, 2, 3, 4, 5

(b) 2, 3, 5, 4, 1 (d) 3, 2, 5, 4, 1

20. 1. Windows

2. Walls

5. Roof

6. Room

(a) 4, 5, 3, 2, 1, 6 (c) 4, 2, 1, 5, 3, 6

3. Floor

4. Foundation

(b) 4, 3, 5, 6, 2, 1 (d) 4, 1, 5, 6, 2, 3

ANSWERS 1. 11.

(c) (c)

2. 12.

(d) (d)

3. 13.

(a) (b)

4. 14.

(c) (c)

5. 15.

(c) (b)

6. 16.

(c) (c)

https://sscstudy.com/

7. 17.

(b) (b)

8. 18.

(a) (b)

9. 19.

(b) (a)

10. 20.

(d) (c)

https://sscstudy.com/

Hints & Solutions 1. (c) Meaningful order of events,

8. (a) Meaningful order of events,

15. (b) Meaningful order of events,

Flower → Bee → Honey → Wax i.e. 2, 3, 1, 4

Eggs → Larva → Pupa → Moth i.e. 4, 2, 1, 3

2. (d) Meaningful order of events,

9. (b) Meaningful order of events,

Market → Vegetable → Cutting → Cooking → Food i.e. 2, 1, 3, 4, 5

Egg → Worm → Cocoon → Butterfly i.e. 3, 4, 2, 1

3. (a) Meaningful order of events Cotton → Thread → Weaving → Cloth i.e. 2, 4, 1, 3

4. (c) Meaningful order of events, Letter → Word → Phrase → Sentence i.e 2, 3, 1, 4

5. (c) Meaningful order of events, Village → Block → District → State i.e. 2, 4, 1, 3

Child → Cry → Mother → Milk → Smile i.e. 2, 4, 1, 3, 5

10. (d) Meaningful order of events, Study → Examination → Apply → Job → Earn i.e. 1, 3, 5, 2, 4

11. (c) Meaningful order of events,

16. (c) Meaningful order of events, Grass → Grasshopper → Frog → Snake → Eagle i.e. 5, 3, 1, 4, 2

17. (b) Meaningful order of events, Envelope → Letter → Postbox → Clearance → Delivery i.e. 3, 2, 1, 5, 4

Day → Work → Tired → Night → Sleep i.e. 3, 5, 1, 2, 4

18. (b) Meaningful order of events,

12. (d) Meaningful order of events,

Think → Compose → Sing → Appreciation → Money i.e. 4, 1, 5, 2, 3

Key → Lock → Door → Room → Switch on i.e. 1, 3, 2, 4, 5

6. (c) Meaningful order of events,

13. (b) Meaningful order of events,

Seed → Plant → Flower → Fruit i.e. 3, 1, 4, 2

Seed → Plant → Tree → Wood → Table i.e. 4, 5, 2, 3, 1

7. (b) Meaningful order of events,

14. (c) Meaningful order of events,

Dawn → Noon → Twilight → Night i.e. 2, 3, 1, 4

Crime → Arrest → Judgment → Punishment → Prison i.e. 4, 3, 5, 1, 2

https://sscstudy.com/

19. (a) Meaningful order of events, Title → Contents → Introduction → Index → Chapter i.e. 3, 2, 5, 1, 4

20. (c) Meaningful order of events, Foundation → Walls → Windows → Roof → Floor → Room i.e. 4, 2, 1, 5, 3,6

https://sscstudy.com/

GENERAL MENTAL ABILITY

50

CHAPTER 10

Inserting the Missing Characters Inserting the missing character is filling-up of the empty spaces in letter and number puzzles given in pictorial forms. In such type of questions, some figures are given bearing certain characters, numbers or a combination of both which follow a particular rule for their placement at different positions of the figure. The candidate is required to find this pattern and insert the missing character or number in the figure based on the method used.

Sol. (b) In the first column, 12 × (18 ÷ 3) = 72 In the third column, 16 × (32 ÷ 4) = 128 Let the missing number be x. Then, in the second column, we have 14 × (24 ÷ x) = 112 ⇒ 24 ÷ x = 8 or x = 3

Ex. 4 Find the missing alphabet in the figure. ? P

Ex. 1 Select the missing number from given responses. 729

81

512

64

H J

L

?

49

(a) U (c) T 9

(a) 444

8

(c) 343

Sol. (c) As, (d) 373

2 12

81 × 9 = 729 and 8 × 8 = 64 64 × 8 = 512 7 × 7 = 49 ⇒ 49 × 7 = 343

Similarly,

5

+8

20

L → T

P

G N

F

9

22

21

27

15

(b) 3

K

(c) 4

(a) M (c) Q

9 × 2 = 18

Ex. 3 Find the missing number in the figure. 18

24

32

12

14

16

3

?

4

72

112

128 (c) 4

(d) 5

E ?

(d) 18

Sol. (d) As, 7 × 2 = 14 and 5 × 2 = 10

(b) 3

16

10 7

(a) 2

+8

Ex. 5 Find the missing character in the figure.

? 14

Similarly,

10

So, missing character = T

Ex. 2 Select the missing number from the given responses.

(a) 2

+8

B → J H → P

Sol. (c) As, 9 × 9 = 81 ⇒ ⇒ Same as,

(b) V (d) X 2

7

(b) 515

B

J

(b) P (d) S

Sol. (b) If we put A = 1, B = 2, Y = 25, Z = 26, we have F + P = 6 + 16 = 22 G + N = 7 + 14 = 21 J + E = 10 + 5 = 15 Since, K = 11 so value corresponding to missing letter = (27 − 11 ) = 16. So, the missing letter is the 16th letter of the English alphabet which is P.

https://sscstudy.com/

https://sscstudy.com/

GENERAL MENTAL ABILITY

Practice Questions Directions (Q. Nos. 1-27) In each of the following questions a set of figures carrying certain characters is given. Assuming that the characters in each set follow a similar pattern, find the missing character in each case. 4

5

4

1.

1 10 3 2 14 4 3

(a) 54

11.

(c) 36

(a) 9

9 5 1

(b) 6

3.

(d) 24

(a) 38

2 7 ?

(b) 36

(d) 14 6 ?

6 30 5

21

1

(a) 29

(c) 27

5.

14.

25

2 3 4

? 9 5

4

(a) 9 (a) 10

(b) 16

(c) 25

(d) 20

3 3

(a) 12

4

5

6

(c) 16

3

7.

6

5 10 ? 11

13

9

4

(b) 12

9

7

8

(a) 13

5 9

8.

(a) 105

9.

9

49

(b) 81 5

2

3

(a) 32 (c) 30

?

1

(d) 100 5

4

3

(a) 5

18.

?

26 4

81

(c) 121

28

17.

2

(b) 36 (d) 28

6

4

(a) 10

https://sscstudy.com/

(d) 13

1

4

5

4

2

5

2

2

3

49

64

?

(c) 169 4

10

6

12

96

48

16

152

?

(b) 104

64 36

6

(a) 110

(d) 7

25

4 16

3

(c) 9

(c) 8

(b) 196

?

4 6

(b) 11

(a) 125

16.

5

?

16 13 12

15.

(d)10

4

282

250

12

?

(b) 8

341

27 20 18

4

3

398

(b) 232 (d) 268

6. 2

7

466

(a) 298 (c) 350

(d) 33

42

(b) 21 (d) 16

13

(b) 31

11

84 ? 14

13.

9

7

12

54 42

17 5

6 ?

(b) 10 (d) 19

(a) 8 (c) 14

25 ?

8

15

4

8

9

(d) 40

(d) 29

10

27

12.

7

(c) 42

4.

9

5

(a) 8 (c) 15

(c) 15

8 32 4

(c) 23

5 3

4 3 8

2.

11 12 14 17 ?

(b) 19 4

6

(b) 18

6 7 9 12 16

3 4 6 9 13

(a) 21 5

?

3

2

10.

(d) 144

(c) 112 7

6

9

2

8

4

4

3

?

36

42

26

(b) 2

(d) 124

(c) 3

(d) 4

10

8

18

6

20

?

30

10

30

9

39

13

(b) 12

(c) 14

(d) 8

51

https://sscstudy.com/

52

CUET (UG) Section III : General Test

19.

6 2

2 444 8

9

7

4

4

2

4 ? 1

2

(a) 218 (c) 330

8 63 2

23.

1

A

1

6

B

D

P

K

G

V

C

?

(a) N (c) M

3

(b) K (d) O

24.

(b) 342 (d) 356

20.

A

D

G

?

M

J

(a) N (c) R ?

25.

G Q

(a) A (c) C

(a) H and 8

L

CX

B

J

Q

E

?

2

10

17

?

8

(b) 6 and U

(c) H and 6

(d) 5 and H

26.

(b) Z (d) Y

22.

?

(b) DW (d) DE

B

V

BY

(a) DK (c) DL

(b) Q (d) P

21.

AZ

W

(a) T

P R

D ?

1618

420

(b) U

(c) B

(d) Q

T

27.

Q N

(a) F (c) H

K

?

E

(b) G (d) L

(a)

13 p

(b)

A

22

I

12

?

26

E

18

O

?

15 6

(c)

U 6

(d)

6 U

ANSWERS 1. 11. 21.

(b) (a) (a)

2. 12. 22.

(b) (b) (c)

3. 13. 23.

(c) (b) (b)

4. 14. 24.

(a) (b) (b)

5. 15. 25.

(b) (c) (d)

6. 16. 26.

(d) (b) (a)

https://sscstudy.com/

7. 17. 27.

(d) (b) (c)

8. 18.

(d) (a)

9. 19.

(b) (c)

10. 20.

(a) (d)

https://sscstudy.com/

Hints & Solutions 1. (b) Here, number inside the circle is equal to the sum of the numbers around it. i.e. 4 + 3 + 2 + 1 = 10 ⇒ 5 + 4 + 3 + 2 = 14 Similarly, 4 + 5 + 6 + 3 = 18

2. (b) Here, the sum of numbers in each row = 15 i.e. 4 + 9 + 2 = 15; 3 + 5 + 7 = 15 Similarly, 8 + 1 + ? = 15 ⇒ ? = 15 − 9 ∴ ? =6

As, in 1st figure 4 + 5 + 8 + 3 20 ⇒ = =5 4 4 In 2nd figure 9 + 15 + 12 + 4 40 = = 10 ⇒ 4 4 Similarly, in 3rd figure 8 + 6 + 11 + 7 32 ⇒ = =8 4 4

12. (b) The pattern is as follows,

8 × 4 = 32 ; 6 × 5 = 30 Similarly, 6 × 7 = 42

As, 9 × 3 = 27 ; 9 × 6 = 54; 14 × 3 = 42 and 14 × 6 = 84 Similarly, 7 × 3 = 21 7 × 6 = 42

4. (a) Here, the pattern is as follows,

13. (b) Here, the lower number is twice the

3. (c) Here, the pattern is as follows,

1 + 4 = 5; 5 + 4 = 9; 9 + 4 = 13 13 + 4 = 17 ; 17 + 4 = 21; 21 + 4 = 25 25 + 4 = 29

5. (b) Here, the pattern is as follows 2 → 2 2 = 4 ; 3 → 3 2 = 9; 4 → 4 2 = 16 ; 5 → 5 2 = 25

6. (d) Here, the pattern is as follows, 2×3 6 4 × 3 12 = =3 ⇒ = =6 2 2 2 2 5 × 4 20 Similarly, = = 10 2 2

7. (d) Here, the pattern is as follows, 8 + 4 − 3 = 9; 5 + 6 − 4 = 7 Similarly, 9+ 3− 5 = 7

8. (d) Here, the numbers in each circle are squares of consecutive numbers i.e. 2 2 = 4, 3 2 = 9 and

4 2 = 16 and

5 2 = 25,

6 2 = 36 and 7 2 = 49 8 2 = 64, 9 2 = 81

Similarly, and

2

10 = 100

difference of the first two numbers. In 1st figure ⇒ ( 466 − 341) × 2 = 125 × 2 = 250 Similarly, in 2nd figure ⇒ ( 398 − 282 ) × 2 = 116 × 2 = 232

14. (b) Here, the pattern is as follows, In 1st figure ⇒ (27 + 18) − (12 + 13) = 45 − 25 = 20 In 2nd figure ⇒ (16 + 12 ) − ( 6 + 9) = 28 − 15 = 13 Similarly, in 3rd figure ⇒ (10 + 11) − ( 5 + 4) = 21 − 9 = 12

15. (c) Here, the pattern is as follows, In 1st column ⇒ (1 + 4 + 2 )2 = 7 2 = 49 In 2nd column ⇒ ( 4 + 2 + 2 )2 = ( 8)2 = 64 Similarly, in 3rd column ⇒

2

( 5 + 3 + 5) = 169

16. (b) Here, the pattern is as follows, In 1st row ⇒

4 2 = 16 = 10 + 6

In 2nd row ⇒ 12 2 = 144 = 96 + 48

9. (b) Here, the pattern is as follows,

Similarly, in 3rd row

( 5 + 2 + 4 + 3) × 2 = 28 ( 6 + 1 + 2 + 4) × 2 = 26 Similarly, ( 5 + 3 + 6 + 4) × 2 = 36



10. (a) Here, the pattern is as follows, In 1st figure ⇒ 3 + 1 = 4, 4 + 2 = 6, 6 + 3 = 9, 9 + 4 = 13 In 2nd figure ⇒ 6 + 1 = 7, 7 + 2 = 9; 9 + 3 = 12 , 12 + 4 = 16 Similarly, in 3rd figure ⇒ 11 + 1 = 12 ; 12 + 2 = 14; 14 + 3 = 17 and 17 + 4 = 21

16 2 = 256 = 152 + ? ? = 256 − 152 ⇒ ? = 104

17. (b) Here, the pattern is as follows, In 1st column ⇒ (7 + 2 ) × 4 = 9 × 4 = 36 In 2nd column ⇒ ( 6 + 8) × 3 = 14 × 3 = 42 Similarly, in 3rd column ( 9 + 4) × ? = 26 ⇒ ? = 2

18. (a) Here, the pattern is as follows

11. (a) Here, the number in the middle is equal to the average of numbers at the corners.

Similarly,

10 + 8 = 18 ; 18 ÷ 3 = 6 30 + 9 = 39 ; 39 ÷ 3 = 13 20 + ? = 30 ; ? = 30 − 20 = 10

https://sscstudy.com/

19. (c) Here, the pattern is as follows, In 1st figure ⇒

3 2 1 1 2 3 2 8 4 → 4 8 2

3 2 1 1 2 3 6 2 9 → 9 2 6

926 − 482 = 444 In 2nd figure ⇒ 187 − 124 = 63 Similarly, in 3rd figure ⇒

642 − 312 = 330

20. (d) According to English alphabetical series, 1

4

7

10

13

16

+3

+3 +3 +3 +3 A → D → G → J → M → P

21. (a) According to English alphabetical series, 2

+5

7

12

+5

+5

17

+5

22

+5

1

B → G → L → Q → V → A

22. (c) According to English alphabetical series, 23

−3

20

−3

17

−3

14

−3

11

−3

8

−3

5

W → T → Q → N → K → H → E

23. (b) According to English alphabetical series, 1

2

4

+2

7

+4

11 +5

+3 +1 A → B → D → G → K → 16

22

3

+8

11

+7 +6 P → V → C → K

24. (b) Here, the sum of position of alphabets in each quadrant is 27 A + Z = 1 + 26 = 27 B + Y = 2 + 25 = 27 C + X = 3 + 24 = 27 Similarly,D + W = 4 + 23 = 27 ⇒ DW 25. (d) Here, in each box letters of English alphabetical series and their corresponding positions are given, As, B = 2, J = 10 , Q = 17 Similarly, E = 5 and H = 8 26. (a) Here, the number is obtained from the position of alphabets in English alphabetical series, As, P R ⇒ 1 6 1 8

Similarly,

D

T



4

2 0

27. (c) Here, the vowels appear in zigzag form from top to bottom and numbers at their respective positions in reverse English alphabetical series. ∴

? ?

=

U 6

https://sscstudy.com/

54

CUET (UG) Section III : General Test

CHAPTER 11

Ranking Test The process of determining the position of a person/thing on the basis of comparison or relative position of other person/thing is called ranking.

Sol. (c) According to the question,

There are two types of questions which are asked 1. Based on Comparison 2. Based on Position (Top/Left or Right/Bottom)

1. Based on Comparison In this type of questions, comparison of different objects based on some factors like ages, marks, size, height etc is given. The candidate is required to arrange the data in ascending or descending order and then answer the related question.

Ex. 1 Ram is taller than Mohan and Sohan, while Sohan is taller than Shyam. Also, Mohan is taller than Sohan. Who amongst the following is the shortest? (a) Shyam (c) Ram

(b) Mohan (d) Sohan

Sol. (a) According to the question, Ram > Mohan and Ram > Sohan Sohan > Shyam Mohan > Sohan

Ram Mohan Sohan Shyam

Arranging the above data, we get Ram > Mohan > Sohan > Shyam Clearly, Shyam is the shortest.

Ex. 2 In a group of five friends, Rohit is taller than Swati. Also, Manoj is shorter than Swati. Sumit is taller than Rohit while Ashish is shortest. Who amongst them is the tallest? (a) Swati (c) Sumit

(b) Rohit (d) Manoj

Rohit

Swati Manoj Sumit Ashish

Rohit > Swati; Manoj < Swati or Swati > Manoj; Sumit > Rohit and Ashish is shortest. Arranging the above data, we get Sumit > Rohit > Swati > Manoj> Ashish Clearly, Sumit is the tallest.

2. Based on Position (Top/Left or Right/Bottom) In this type of questions, the position of person (s) from either of the two ends of a row is given. The candidate is required to find the total number of persons in the row or number of persons to the top/left or right/bottom of a particular person etc. While solving this type of questions, keep the following points in mind.

1. Total number of persons in a row = (Rank of a person from left end/top + Rank of that person from right end/ bottom) − 1 2. Rank of a person from right end/bottom = (Total number of persons in row) − (Rank of that person from left end/top) + 1 3. Rank of a person from top/left end = (Total number of persons in row) – (Rank of that person from right end/bottom) + 1 4. In case of interchanging of positions (a) Total number of persons = [Initial position of 1st person + Interchanged position of 2nd person] − 1 (b) New position of 2nd person = [Difference in the two positions of 1st person] + [Initial position of 2nd person]

https://sscstudy.com/

https://sscstudy.com/

GENERAL MENTAL ABILITY

Ex. 3 In a class of 45 students rank of Ayush from the top is 15, then rank of Ayush from bottom is (a) 30 (c) 31

(b) 32 (d) 35

Sol. (c) Here, total number of students = 45 Rank from top = 15 ∴ Rank of Ayush from bottom = (Total number of persons in the class) – (Rank of Ayush from top) + 1 = 45 − 15 + 1 = 31

Alternate Method

Sol. (c) Position of Sandeep from left (L) = 10 Position of Sandeep from right (R) = 16 ∴ Total number of people in the row = (L + R) − 1 = 10 + 16 − 1 = 25

Ex. 5 In a row of students, Anil is 7th from left, while Sunil is 18th from right. Both of them interchanged their positions such that Anil becomes 21st from left. What will be the total number of students in the class? (a) 38 (c) 31

Sol. (a) 15th

(b) 33 (d) 30 7th Anil

18th Sunil

45 Students

Total number of students = 45 Given, the rank of Ayush from top is 15, it means there are 45 − 15 = 30 students below Ayush in the class. ∴ The rank of Ayush from bottom = 30 + 1 = 31

Ex. 4 In a row, position of Sandeep is 10th from left and 16th from right. How many people are there in the row? (a) 30 (c) 25

55

(b) 26 (d) 16

Sunil

Anil 21st

∴ Anil moves (21–7) = 14 places ahead and changed his place with Sunil. ∴ Sunil will also move 14 places ahead. ∴ Sunil's new position from right end = 18 + 14 = 32 We can also calculate the total number of students = (Anil's initial position) + (Sunil's new position) −1 = 7 + 32 − 1 = 38

Practice Questions 1. A pole is taller than a giraffe which is taller than a tree. A signal is shorter than a pole but taller than a building which is taller than a giraffe. Who is the shortest? (a) Signal (c) Tree

(b) Giraffe (d) Building

2. Mohan is taller than Rohan but shorter than Farhan. Kannan is shorter than Mohan but taller than Rohan, Shankar is taller than Rohan and Farhan. Who is the tallest? (a) Mohan (c) Shankar

(b) Farhan (d) Kannan

3. Sameer is older than Milon but not as old as Arjun, Priya is older than Sam but not as old as Milon. Who amongst them is the oldest?

5. If Anita is taller than Surjit but shorter than Kusum and Surjit is just as tall as Kalpana but taller than Vanita, then Kalpana is (a) just as tall as Anita (b) taller than Kusum (c) shorter than Anita (d) shorter than Surjit

6. In a group of five districts Akbarpur is smaller than Fatehpur, Dhanbad is bigger than Palamu, and Barabanki is bigger than Fatehpur but not as big as Palamu. Which district is the biggest? (a) Akabarpur (c) Palamu

(b) Fatehpur (d) Dhanbad

7. Five friends A, B, C, D and E are sitting in a row in

Neha but after Pooja. Shivani finished before Anju but after Neha. Who won the race?

decreasing order of their weight from left to right. A is heavier than only C, while B is heavier than both E and A. D is heavier than B. Who amongst the following is sitting exactly in centre between them?

(a) Sonam (c) Neha

(a) C (c) E

(a) Milon (c) Priya

(b) Sameer (d) Arjun

4. Five girls took part in a race. Sonam finished before

(b) Pooja (d) Shivani

https://sscstudy.com/

(b) A (d) B

https://sscstudy.com/

56

CUET (UG) Section III : General Test

Directions (Q. Nos. 8-10) Consider the following information and answer questions based on it. Suresh is taller than Ramesh who is shorter than Rakesh. Jinesh is taller than the shortest person but shorter than Rakesh. Pritesh is taller than Suresh but shorter than Jinesh.

15. If you are 9th person in a queue starting from one end and 11th from another end, what is the number of persons in the queue? (a) 20 (c) 21

16. Rajesh’s rank is 12th from top and 30th from bottom among the children who passed the annual examination. If 16 children failed, then find the total number of children who appeared for the examination.

8. Who is the shortest? (a) Rakesh (c) Pritesh

(b) Ramesh (d) Suresh

(a) 57 (c) 68

9. If Jayesh who is taller than Suresh joins the group, who will be fourth, if they are arranged in a descending order? (a) Jayesh (c) Pritesh

he was 47th from the bottom among those who passed. 3 students could not appear for the exam. 1 student failed. What is the total number of students?

10. Which statement, among the following is correct?

(a) 60 (c) 59

(a) Suresh is taller than Jinesh (b) Pritesh is shorter than Rakesh (c) Jinesh is taller than Rakesh (d) Ramesh is taller than Pritesh

positions left, then she becomes 6th from left. Find her previous position in the queue from right. (a) 5 (c) 9

and 27th from the last. How many girls are there in the queue? (b) 43 (d) 36

is 25th from left side. When they interchange their place, then Mohan is at 22nd place from right. Find Sohan's new position from left.

What is Sachin's position from left end? (b) 28th (d) 24th

(a) 37th (c) 35th

13. In the name list of 48 students, Rohit’s rank is 41st (c) 7th

Deepak is 18th from right. When Kamal and Deepak interchange their position, then Kamal is 25th from left. Find the total number of students in the queue.

(d) 9th

14. In a row of trees, one tree is 7th from either end of the row. How many trees are there in the row? (a) 11 (c) 14

(b) 38th (d) 36th

20. In a students queue, Kamal is 12th from left and

from bottom. Find his rank from top. (b) 8th

(b) 2 (d) 3

19. In a queue, Mohan is 10th from right side and Sohan

12. In a row of 50 people, Sachin is 24th from right end.

(a) 12th

(b) 62 (d) 61

18. In a queue of 17 people, when Seema shifts 3

11. In a queue of girls, Shama is 11th from the starting

(a) 30th (c) 27th

(b) 33 (d) 45

17. In an examination, Rahul got 11th rank from top and

(b) Jinesh (d) Cannot be determined

(a) 37 (c) 35

(b) 19 (d) 18

(a) 40 (c) 42

(b) 15 (d) 13

(b) 41 (d) 43

ANSWERS 1. 11.

(c) (a)

2. 12.

(c) (c)

3. 13.

(d) (b)

4. 14.

(b) (d)

5. 15.

(c) (b)

6. 16.

(d) (a)

https://sscstudy.com/

7. 17.

(c) (d)

8. 18.

(b) (c)

9. 19.

(d) (a)

10. 20.

(b) (c)

https://sscstudy.com/

Hints & Solutions 1. (c) According to the question,

7. (c) According to the question,

16. (a) Total number of students who passed

Pole > Giraffe > Tree K (i) Pole > Signal > Building > Giraffe K (ii) From (i) and (ii), we get Pole > Signal > Building > Giraffe > Tree Clearly, tree is the shortest.

A is heavier than only C, that means A is at 4th place and C is at 5th place from left. A > C; B > E, A or B > E > A > C Also, D>B ⇒ D > B > E > A > C 1 2 3 4 5 Clearly, E is sitting exactly in centre between them. Sol. (Q. Nos. 8-10) According to the question, Suresh > Ramesh K (i) Rakesh > Ramesh K (ii) Rakesh > Jinesh K (iii) Jinesh> Pritesh > Suresh K (iv) From (i), (ii), (iii) and (iv), we get Rakesh > Jinesh > Pritesh > Suresh > Ramesh

= (12 + 30) − 1 = 41 Number of students who failed = 16 ∴Total number of students who appeared for the exam = 41 + 16 = 57

2. (c) According to the question, Ferhan > Mohan > Rohan K (i) Mohan > Kannan > Rohan K (ii) Shankhar > Rohan , Ferhan K (iii) From (i), (ii) and (iii), we get Shankhar>Ferhan>Mohan> Kannan>Rohan Clearly, Shankhar is the tallest.

3. (d) According to the question, Arjun > Sameer> Milon and Milon > Priya > Sam Arranging in meaningful order, we get Arjun > Sameer > Milon > Priya > Sam Clearly, Arjun is the oldest.

4. (b) According to the question, Pooja > Sonam > Neha and Neha > Shivani > Anju Arranging the data in meaningful order, we get Pooja > Sonam> Neha > Shivani > Anju Clearly, Pooja won the race.

5. (c) According to the question, Kusum>Anita>Surjit and Surjit = Kalpana > Vanita Arranging in meaningful order, we get Kusum> Anita > Surjit = Kalpana > Vanita Clearly, Kalpana is shorter than Anita.

6. (d) According to the question, Barabanki >Fatehpur > Akbarpur and Dhanbad > Palamu and Palamu > Barabanki Arranging the data in meaningful order, we get Dhanbad > Palamu > Barabanki > Fatehpur> Akbarpur Clearly, Dhanbad district is biggest amongst them.

8. (b) Clearly, Ramesh is the shortest. 9. (d) Rakesh > Jinesh > Pritesh > Suresh > Ramesh Jayesh Hence, it cannot be determined that who is the fourth.

17. (d) Total number of students who passed = (11 + 47 ) − 1 = 57 Now, total number of students = 57 + 3 + 1 = 61

18. (c) Seema shifts 3 positions towards left. So, in order to find her original position we have to shift her 3 places towards right. ∴Seema’s original position from left = 6+ 3 = 9 Also, total number of people = 17 ∴Total number of people = (Position from left + Position from right) −1 ∴ 17 = ( 9 + R ) − 1 ⇒ R = 9 ∴Seema is 9th from the right. 19. (a)

25th Sohan

10th Mohan

10. (b) Clearly, Pritesh is shorter than Rakesh. 11. (a) We know that, Total number of girls in queue = (Rank of girl from starting + Rank of girl from last) −1 = 11 + 27 − 1 = 37

12. (c) Total number of people = ( Position from left + Position from right ) −1 ⇒ 50 = (24 + L ) − 1 ⇒ L = 51 − 24 = 27 ∴ Sachin is 27th from left end.

13. (b) Total number of students = ( Rank from top + Rank from bottom ) −1 48 = ( T + 41) − 1 ⇒ T = 8 ∴Rohit is 8th from top.

14. (d) Here, position of tree from both the end is 7. ∴ Total number of trees = 7 + 7 − 1 = 13

15. (b) Total number of persons = ( Position from left end + Position from right end ) −1 = ( 9 + 11) − 1 = 19

https://sscstudy.com/

Mohan

Sohan

22nd ∴ Mohan moves (22 −10) = 12 places ahead and changed his place with Sohan. ∴Sohan will also move 12 places ahead. ∴Sohan's new position from left end = 25 + 12 = 37th

20. (c)

12th Kamal Deepak

18th Deepak Kamal 25th

∴ Total number of students = ( Deepak's initial position + Kamal’s new position ) −1 = 18 + 25 − 1 = 42

https://sscstudy.com/

58

CUET (UG) Section III : General Test

CHAPTER 12

Sitting Arrangement Sitting arrangement refers to, how a group of persons or objects allocated their positions in a particular manner. In these questions, a group of persons or objects with some predefined conditions is given, on the basis of these conditions candidates are required to arrange persons objects in an order as given in the question. Based on the order of sitting, questions can be divided into following types. 1. Linear arrangement. 2. Circular arrangement 3. Square/Rectangular arrangement

3. M is not at the centre. 4. K is to the immediate right of N and immediate left of L.

Ex. 2 The student standing to the immediate left of O is (a) L (a) N

Right

One is required to identify the exact position of the objects and their positions with respect to one another based on the information given.

(b) T

(c) Q

(c) P

(a) K

P

T

S

Q

(b) P

(c) N

(Middle)

(d) J

M

O

J

P

N

K

L

Right

2. (c) M is standing to the immediate left of O. 3. (c) P is at the centre of the row. 4. (c) N is standing fifth from the left end of the row.

2. Circular Arrangement In this type of arrangement, objects/persons are placed around a circle facing the centre. A

D

(d) R

Centre

Left

C Right

B

Ex. 5 Six girls are sitting in a circle facing towards the centre. Bindu is to the left of Viji. Rekha is in between Bindu and Mumtaz. Jessa is in between Viji and Nirmala. Who is to the left of Mumtaz?

R

(Left)

(d) K

Sol. (Ex. Nos. 2-4) According to the question,

Sol. (a) Arrangement according to the question is as follows.

(d) P

Ex. 4 Who stands fifth from the left of the row?

Ex. 1 Five friends P, Q, R, S and T are sitting in a row facing North. Here, S is between T and Q and Q is to the immediate left of R. P is to the immediate left of T. Who is in the middle? (a) S

(b) J

Left

In linear arrangement, person/objects are required to be placed in proper order in a straight line.

(c) M

Ex. 3 Who is at the centre of the row?

1. Linear Arrangement

Left

(b) J

(Right) Facing North

(a) Rekha

∴ S is in the middle of the row.

(b) Nirmala

(c) Viji

Sol. (b) According to the question,

Directions (Ex. Nos. 2-4) Consider the following information and answer questions based on it.

Mumtaz Rekha

Nirmala

Seven students J, K, L, M, N, O and P are standing in a row, in random order, from left to right, such that 1. P, O, K and N do not stand on any of the extreme ends. 2. J is to the immediate left of P and immediate right of O.

Jessa

Bindu Viji

Hence, Nirmala is to the left of Mumtaz.

https://sscstudy.com/

(d) Bindu

https://sscstudy.com/

GENERAL MENTAL ABILITY

Ex. 6 A, B, C and D are playing cards. A and C, and B and D are partners. D is to the right of C. The face of C is towards West. Find the direction that D is facing.

3. Square/Rectangular Arrangement In this type of arrangement, four objects/persons are placed around a square or a rectangular shaped table facing the centre. A

(a) West

(b) East

D

C

(c) South

(d) North

Sol. (c) Arrangement according to the question is as follows.

D Right

Right

59

A

N C

Left

E

W

Left

S

B

Clearly, D is facing South.

B

Practice Questions 1. Five friends are sitting in a row facing South. Here, Mohan is between Balu and Raju and Raju is to the immediate right of Praveen and Amith is to the right of Balu. Who is in extreme right end? (a) Amith (c) Praveen

(b) Balu (d) Mohan

2. Five students Priyanka, Mary, Sunil, Asha and Ryan are standing in a line. Priyanka and Sunil are standing ahead of Ryan. Sunil is standing between Ryan and Asha. Between Sunil and Mary, Ryan is standing. Who is the first in the line? (a) Asha (c) Ryan

facing South. Black coloured bus is standing in the immediate right of Red. Green colour is between Blue and Yellow. Yellow colour is between Black and Green. Which coloured bus is standing in the middle? (b) Blue (d) Green

facing East. C is between A and E. B is just to the right of E but left of D. F is not at the right end. Who is at the right end? (b) B (d) C

5. There are five different houses, A to E, in a row. A is to the right of B and E is to the left of C and right of A, B is to the right of D. Which of the houses is in the middle? (a) A (c) C

(b) B (d) D

in the centre and A and B are at the ends. C is sitting on the left of A. Then, who is sitting on the right of B? (b) D (d) F

(b) B (d) C

Directions (Q. Nos. 8-10) Read the following information and answer the following questions. Five girls are sitting in front, facing you. Ruhi and Manali respectively are sitting to the right and left of Urja. Dhwani is sitting between Manali and Tanya. (a) 1 (c) 3

(b) 2 (d) 4

9. Who is sitting in the middle ? (a) Dhwani (c) Manali

(b) Urja (d) Tanya

10. Who is sitting to the right of Tanya? (b) Manali (d) Urja

Directions (Q. Nos. 11-13) Use the following passage for the questions based on it. There are six flats on a floor in two rows. Out of these, three are North facing and the other three are South facing flats. The flats are to be allotted amongst Arun, Biswajyot, Chitra, Derek, Evan and Fatima. Biswajyot gets a North facing flat and is not next to Derek. Derek and Fatima get diagonally opposite flats. Chitra is next to Fatima and gets a South facing flat. Evan gets a North facing flat. 11. Except Derek and Fatima, which other pair is

6. A, B, C, D, E and F are sitting in a row. E and F are

(a) A (c) E

(a) E (c) A

(a) Ruhi (c) Dhwani

4. Six friends A, B, C, D, E and F are sitting in a row

(a) D (c) E

is on the left of B, but to the right of C. If A is on the left of D. Who is sitting in the middle?

8. How many girls are there between Dhwani and Ruhi ?

(b) Sunil (d) Priyanka

3. Five different coloured buses are standing in a row

(a) Yellow (c) Black

7. Five boys are sitting in a row. A is on the right of B, E

diagonally opposite to each other? (a) Arun and Biswajyot (b) Arun and Chitra (c) Evan and Derek (d) Evan and Chitra

https://sscstudy.com/

https://sscstudy.com/

60

CUET (UG) Section III : General Test

12. Which of the following pairs is exactly opposite to each other? (a) Derek and Evan (c) Evan and Chitra

(b) Fatima and Chitra (d) Evan and Arun

13. Which of the following combination gets South facing flats? (a) Arun, Chitra, and Fatima (b) Chitra, Biswajyot and Derek (c) Evan, Arun and Fatima (d) Derek, Arun and Biswajyot

14. Five persons are sitting facing centre of a circle. Pramod is sitting to the right of Ranjan. Raju is sitting between Brejesh and Naveen. Raju is to the left of Brejesh and Ranjan is to right of Brejesh. Who is sitting to the left of Naveen? (a) Pramod

(b) Raju

(c) Brejesh

(d) Rajan

Directions (Q. Nos. 19-21) Read the following passage and answer the questions based on it. (i) Vinay, Vinod, Vikas, Vasu, Vaibhav, Varun and Vishal are sitting around a circle facing at the centre. (ii) Vasu is the third to the right of Vikas. (iii) Varun is second to the left of Vinay and second to the right of Vishal. (iv) Vaibhav is third to the right of Vinod and is not an immediate neighbour of Varun. 19. Who is second to the right of Vinod? (a) Vaibhav (c) Vishal

20. Who is third to the left of Varun? (a) Vaibhav (c) Vishal

15. Six persons are sitting in a circle facing the centre of the circle. Parikh is between Babita and Narendra. Asha is between Chitra and Pankaj. Chitra is to the immediate left of Babita. Who is to the immediate right of Babita? (a) Parikh (c) Narendra

(a) Vinay

facing Hemant who is sitting to the left of Arvind and right of Sanjay. Suman is to the right of Arvind. Manoj is facing Arvind. Who is now sitting to the right of Manoj? (b) Ajay (d) None of these

17. A, B, C, D, E, F and G are playing cards sitting in a circle. F is 2nd to the right of G. B is neighbour of F but not of E. E, the neighbour of C, is 4th to the right of G. D is between E and A.

(c) Vaibhav

(d) Varun

rectangular table. Radha is to the right of Sheela. Mahima is to the left of Seeta. Which of the persons given in the options are sitting opposite to each other? (a) Sheela-Seeta (c) Radha-Sheela

(b) Radha-Seeta (d) Mahima-Radha

Directions (Q. Nos. 23-25) Consider the following information and answer the questions based on it. Six animals - Horse, Cow, Pig, Dog, Donkey and Goat are tied to a pole each, in a circle, facing each other, in random order. 1. Goat is to the immediate right of Pig. 2. Cow is not tied next to either Donkey or Dog. 3. If the animals mark the vertices of a hexagon, then the Horse is diagonally opposite to the Pig. 23. The Cow is tied to the immediate left of

Who is fourth to the left of G?

(a) Goat (c) Horse

(a) D (b) E (c) C (d) Cannot be determined

(b) Pig (d) Dog

24. Which pair is tied next to each other? (a) Horse and Goat (d) Goat and Dog

18. Four persons M, N, O and P are playing cards. M is on the right of N and P is on left of O. Then which of the following are partners? (a) P and O (c) M and N

(b) Vishal

22. Radha, Sheela, Mahima and Seeta are sitting along a

16. Six persons are sitting around a circular table. Ajay is

(i) (ii) (iii) (iv)

(b) Vasu (d) Vikas

21. Who is to the immediate right of Vikas?

(b) Pankaj (d) Chitra

(a) Arvind (c) Suman

(b) Vasu (d) Vikas

(b) Pig and Cow (d) Donkey and Dog

25. Which animal is tied to the immediate left of Pig? (a) Goat (c) Dog

(b) M and P (d) N and P

(b) Donkey (d) Cannot be determined

ANSWERS 1. 11. 21.

(a) (a) (c)

2. 12. 22.

(d) (c) (b)

3. 13. 23.

(a) (a) (c)

4. 14. 24.

(a) (a) (d)

5. 15. 25.

(a) (a) (d)

6. 16.

(b) (d)

https://sscstudy.com/

7. 17.

(b) (c)

8. 18.

(b) (d)

9. 19.

(c) (d)

10. 20.

(c) (a)

https://sscstudy.com/

Hints & Solutions 1. (a) According to the question, sitting

Sol. (Q. Nos. 11-13) According to the

18. (d) According to the question, the

arrangement will be

question,

arrangement is as follows, South

Praveen

Raju

Mohan

Balu

Left Fatima

Chitra Arun Or

Thus, Amith is at extreme right end.

Biswajyot Evan Derek

Priyanka Asha Sunil Ryan Mary

N

11. (a) Except Derek and Fatima, Arun and Biswajyot are diagonally opposite to each other.

3. (a) According to the question, we have following standing arrangement of buses

Vikas

13. (a) Arun, Chitra and Fatima get South facing flats.

Raju

following sitting arrangement.

Vasu

East Right

∴D is at the extreme right.

5. (a) According to the question, the arrangement is as follows, B

A

E

19. (d) Vikas is second to the right of Vinod. 20. (a) Vaibhav is third to the left of Varun. Pramod

Ranjan

15. (a) According to the question, the

Hence, House A is in the middle.

6. (b) According to the question, the D

E/F F/E C Hence, D is on the right of B.

A

7. (b) According to the question, the D

Sol. (Q. Nos. 8-10) According to the question, Urja Manali Dhwani Tanya

Asha

Chitra

16. (d) According to the question, the arrangement is as follows, Manoj

Ajay

Sanjay

Suman

Hemant

Arvind

Hence, Sanjay is on the right of Manoj.

17. (c) According to the question, the

8. (b) Two girls are sitting between Dhwani

D

E

C

and Ruhi.

10. (c) Dhwani is sitting to the right of Tanya.

Radha

Seeta

Babita

arrangement is as follows,

9. (c) Manali is sitting in the middle.

arrangement is as follows,

Parikh

arrangement is as follows, C E B A Hence, B is in the middle.

22. (b) According to the question, the Mahima

Narendra

Hence, Parikh is to the right of Babita.

arrangement is as follows,

21. (c) Vaibhav is to the immediate right of Vikas.

arrangement is as follows,

Pankaj

C

Vinod Varun

Naveen

Hence, Pramod is sitting on the left of Naveen.

F A C E B D West Left

Vishal

14. (a) According to the question, Brijesh

4. (a) According to the question, we have the

Vinay

Vaibhav

Hence, yellow coloured bus is standing in the middle.

Ruhi

Sol. (Q. Nos. 19-21) According to the question,

to each other.

Red

Black

Yellow

Left

Green

Right

B

Clearly, N and P are partners.

12. (c) Evan and Chitra are exactly opposite

Hence, Priyanka, is the first in the line.

D

M

O Derek Evan Biswajyot

North

2. (d) According to the question,

Blue

P

Arun Chitra Fatima

F

A G

B

Hence, C is fourth to the left of G.

https://sscstudy.com/

Sheela

Clearly, Radha and Seeta are sitting opposite to each other.

Sol. (Q. Nos. 23-25) According to the question, Donkey/Dog

Horse

Donkey/Dog

Amith

Right

Cow

Pig

Goat

23. (c) Cow is tied to the immediate left of Horse. 24. (d) Donkey and Dog are tied next to each other. 25. (d) Either Dog of Donkey is tied to the immediate left of Pig.

https://sscstudy.com/

62

CUET (UG) Section III : General Test

CHAPTER 13

Puzzle Test Puzzle refers to something which is baffling or confusing that is to be solved. In such questions based on Puzzle, some raw information is given about the persons or items that are to be arranged in a particular sequence or order. For this candidates are required to study and analyse the jumbled information given in the question. And finally link them together to form a diagrammatic representation to answer the questions. The question on puzzle test can be based on classification, placing arrangement, comparison, sequential order, based on family, etc.

Ex. 1 Read the given information carefully and answer the question based on it. Six books are labelled A, B, C, D, E and F are placed side by side. Books B, C, E and F have green covers while others have yellow covers. Books A, B and D are new while the rest are old volumes. Books A, B and C are law reports while the rest are medical extracts. Which two books are old medical extracts and have green covers? (a) B and C

(b) E and F

(c) C and F

(d) D and F

Sol. (b) According to the given information, the arrangement is as follows

Books

Colour of cover

Volume

Law/Medical

D

Yellow

New

Medical

E

Green

Old

Medical

F

Green

Old

Medical

It can be seen from the above table that, E and F are old medical extracts and have green covers. Hence, option (b) is correct.

Ex. 2 Read the given information carefully and answer the question based on it. In a group of five persons A, B, C, D and E, one person is a professor, one is a doctor and one is a lawyer. A and D are unmarried females who don’t work. There is a married couple in the group in which E is the husband, B is the brother of A and is neither doctor nor a lawyer. Who is professor? (a) B (c) A

(b) C (d) Data inadequate

Sol. (a) According to the question, (Doctor/Lawyer) E

(+)

Married couple

←→ C ( −) (Doctor / Lawyer) Brother

Books

Colour of cover

Volume

Law/Medical

A

Yellow

New

Law

B

Green

New

Law

C

Green

Old

Law

(Professor) B ( + ) ←→

A( − ) (Unmarried) D( − ) (Unmarried)

A and D are workless. B is neither doctor nor lawyer. Hence, it is clear that B must be professor.

https://sscstudy.com/

https://sscstudy.com/

GENERAL MENTAL ABILITY

63

Practice Questions 1. A ate grapes and pineapple, B ate grapes and oranges, C ate oranges, pineapple and apple, D ate grapes, apple and pineapple. After taking fruits, B and C fell sick. In the light of the above facts, it can be said that the cause of sickness was (a) apple (c) grapes

(b) pineapple (d) oranges

S and T (not necessarily in that order). Each one of them comes from a different city. Further it is given that, I. B and C do not belong to Q. II. B and E do not belong to P and R. III. A and C do not belong to R, S and T. IV. D and E do not belong to Q and T. Which one of the following statements is not correct? (b) D belongs to R (d) B belongs to S

3. Six friends James, Deepak, Hari, Rahul, Vikram and Imran went for a picnic where they decided to play kabaddi. They divided themselves into two different teams with Deepak, Rahul and Vikram in Team-1 and rest in Team-2. Deepak and Imran were fat, but others were thin. Rahul and James were cheating, but other played the game fairly. Who was/were the friend/friends who played the game fairly from the Team-2? (a) Imran and James (c) Hari and Imran

(b) Imran alone (d) Hari alone

4. Five friends, Ramesh, Jim, Manish, Nilesh, Sagar went for interviews in five different companies A, B, C, D and E, but not in the same order, wearing different ties viz. Green, Red, Blue, Yellow and Orange but not in the same order. Nilesh did not go to company B and was not wearing a Green or Orange colour tie. The person who went to company B wearing a Blue colour tie and the person who went to company E was wearing a Yellow colour tie. Manish and Sagar did not go to the company B. Jim went to company D and was wearing a Red coloured tie. Who is working in company B? (a) Sagar (c) Nilesh

(a) Bhopal and Lucknow (c) Bhopal and Shimla

(b) Lucknow and Shimla (d) Bhopal and Delhi

6. There are four boxes, J, K, L and M in which four

2. A, B, C, D and E belong to five different cities P, Q, R,

(a) C belongs to P (c) A belongs to Q

(iii) The person who belongs to Delhi is sitting to the left of the person who belongs to Bhopal. (iv) P is sitting opposite to C. A and B belong to which place ?

(b) Ramesh (d) Manish

5. A, B, C and D are sitting around a circular table and discussing about their home town. (i) A is sitting opposite to the person who belongs to Shimla. (ii) B is sitting to the right of the person who belongs to Lucknow.

types of fruits are stored viz. Litchi, Apple, Grapes and Mango. Boxes are arranged in a certain manner from top to bottom. There are two boxes between K and L. The box in which Grapes are stored is above L, but not immediate above. The Box in which Apple is stored is immediate below M, but not stored in Box L. Litchi box is above the Mango box, but not immediate above Apple box. In which of the following box Litchi is stored? (a) J

(b) M

(c) K

(d) L

7. In a four day period–Monday through Thursday, each of the following temporary office workers worked only one day, each on a different day. Jai was scheduled to work on Monday but he traded with Raj, who was originally scheduled to work on Wednesday. Farid traded with Kajal, who was originally scheduled to work on Thursday. Finally, Jai traded with Kajal. After all the switching was done, who worked on Tuesday? (a) Jai (c) Raj

(b) Farid (d) Kajal

8. A, B, C, D, E, F and G are members of a family consisting of 4 adults and 3 children, two of whom, F and G are girls, A and D are brothers and A is a doctor. E is an engineer married to one of the brothers and has two children. B is married to D and G is their child. Who is C? (a) G’s father (c) E’s daughter

(b) F’s father (d) A’s son

9. In a family of six, there are three men A, B and C and three women R, S and T. The six persons are Architect, Lawyer, CA, Professor, Doctor and Engineer by profession, but not in the same order. There are two married couples and two unmarried persons. C is not R’s husband. The Doctor is married to a Lawyer. R’s grandfather is a Professor. B is neither A’s son nor he is an Architect or a Professor. The Lawyer is T’s daughter-in-law. C is T’s son and the Engineer’s father. A is married to the CA. Which among the following is the correct pair of married couples? (a) AS and CR (c) AT and CS

https://sscstudy.com/

(b) AC and SR (d) AT and RB

https://sscstudy.com/

64

CUET (UG) Section III : General Test

10. Eight students A, B, C, D, E, F, G and H are going to college in two cars and following are conditions. There are four students in each car. A is in the same car in which D is sitting but H is not in the same car. B and C are not in the car in which D is sitting. F is sitting with A and E. Four students sitting in the same car are (a) A, B, G, H (c) B, C, G, H

(b) B, D, F, G (d) A, C, D, E

11. P, Q, R, S, T, U are 6 members of a family in which there are two married couples. T, a teacher, is married to a doctor who is mother of R and U. Q, the lawyer, is married to P. P has one son and one grandson. Of the two married ladies one is a housewife. There is also one student and one male engineer in the family. Which of the following is true about the grand-daughter of the family? (a) She is a lawyer (c) She is a student

(b) She is an engineer (d) She is a doctor

15. White house is owned by (a) Q (c) S

Directions (Q. Nos. 16-18) Consider the following information and answer the questions based on it. The likes and professions of five friends Rosy, Mary, Lily, Andy and Daisy are given. 1. Rosy likes yellow and is a student. 2. The librarian likes green. 3. Mary and Andy like purple and blue respectively and neither of them is a teacher. 4. Lily likes brown and the inspector likes blue. 5. One of the five is a principal. 16. Which colour does the teacher like? (a) Blue

(c) Purple

(d) Green

(a) Daisy

(b) Andy

(c) Mary

(d) Lily

18. Which among the following is the wrong pair?

ground. A and B belong to Ruby House, while the rest belong to Emerald House. D and F are tall, while others are short. C and D are wearing glasses, while others are not wearing. Which girl of Emerald House who is tall and wearing glasses? (b) C (d) A

Directions (Q. Nos. 13-15) Consider the following information and answer questions based on it. P, Q, R and S are four friends who pursue teaching, law, banking, cooking and own red, blue, white and yellow house, in a random order. An individual owns only one house and pursues only one profession. 1. P owns a red house and is not a banker. 2. The owner of blue house is lawyer. 3. The colour of S’s house is neither yellow nor white. 4. R is a teacher.

(a) Andy-Inspector (c) Daisy-Green

(b) Purple-Principal (d) Rosy-Teacher

Directions (Q. Nos. 19-21) Study the following information carefully and answer the questions given below it. 1. There are six members in a family in which there are two married couples. 2. Sandhya, a lawyer, is married to the engineer and is mother of Charu and Suraj. 3. Bhuvanesh, the teacher, is married Aruna. 4. Aruna has one son and grandson. 5. Of the two married ladies one is a housewife. 6. There is also one student and one male doctor in the family. 19. How is Aruna related to Charu? (a) Sister (c) Grandfather

(b) Mother (d) Grandmother

20. Who among the following is the housewife?

13. The owner of the blue house is (a) S (c) Q

(b) Brown

17. Who is the principal?

12. Six students A, B, C, D, E, F are sitting on the

(a) B (c) D

(b) R (d) Cannot be determined

(a) Charu (c) Sandhiya

(b) R (d) Cannot be determined

(b) Aruna (d) None of the above

21. Which of the following is true about the

14. Q is a (a) Lawyer (c) Cook

grand-daughter in the familty?

(b) Banker (d) Teacher

(a) She is a doctor (c) She is a student

(b) She is a teacher (d) Data inadequate

ANSWERS 1. 11. 21.

(d) (c) (c)

2. 12.

(d) (c)

3. 13.

(d) (a)

4. 14.

(b) (b)

5. 15.

(c) (d)

6. 16.

(c) (b)

https://sscstudy.com/

7. 17.

(a) (c)

8. 18.

(d) (d)

9. 19.

(c) (d)

10. 20.

(c) (b)

https://sscstudy.com/

Hints & Solutions Hence, A and B belong to Bhopal and Shimla, respectively.

1. (d) Fruit Person

Apple Pineapple Grapes Oranges

A B C D

û û ü ü

ü û ü ü

ü ü û ü

û ü ü û

Since, B and C fell sick, so the cause of sickness is oranges which is eaten by both of them.

6. (c)

7. (a) Day

Originally

Monday

Jai

Raj

Tuesday

Farid

Kajal

Jai

Wednesday

Raj

Jai

Kajal

Thursday

Kajal

Farid

Farid

table can be drawn

P

Q

R

S

T

A

û

ü

û

û

û

B

û

û

û

û

ü

C

ü

û

û

û

û

D

û

û

ü

û

û

E

û

û

û

ü

û

Person

Fruit Litchi Grapes Apple Mango

∴ Litchi is stored in Box K.

2. (d) From the given information, following City

Box K M J L

Red

8. (d) Family diagram is as follows

Banking

White/Yellow

R

Teacher

Yellow/White

S

Lawyer

Blue

(–) Married

F

James

Team-2

Thin

Cheating

Deepak

Team-1

Fat

Fairly

Hari

Team-2

Thin

Fairly

Rahul

Team-1

Thin

Cheating

Vikram

Team-1

Thin

Fairly

Imran

Team-2

Fat

Fairly

Hence, Hari played fairly from Team-2 and he is thin.

4. (b) According to the question, the arrangement is as follows

(–)

Colour of tie Blue Red Green/Orange Yellow Green/Orange

5. (c) According to the question, A (Bhopal)

D

13. (a) S is the owner of blue house.

(+)

C

(+) Married

14. (b) Q is a banker.

(–)

B

(+)

(–)

(–)

G

= Male

15. (d) White house is owned by either Q or R.

= Female

Sol. (Q. Nos. 16-18) According to the question, Person

9. (c) The family diagram can be drawn as (+) Married

A (Professor)

(–)

T (CA)

(+)

C (Doctor)

Married

S (Lawyer)(–)

(+) B (Engineer)

(Architect) R(–)

Colour

Profession

Rosy

Yellow

Student

Mary

Purple

Principal

Lily

Brown

Teacher

Andy

Blue

Inspector

Daisy

Green

Librarian

16. (b) Teacher likes brown colour.

Hence, the correct pair of married couples are AT and CS.

17. (c) Mary is the principal.

10. (c) By given conditions, there are two groups

Sol. (Q. Nos. 19-21) According to the question,

11. (c) According to the information, the arrangement is as follows Couple Q+

P–

18. (d) Rosy is not a teacher, she is a student.

(+)

Bhuvanesh (Teacher)

Housewife Son Lawyer

Couple

T+ Teacher

R (+, –) Engineer/Student

Married couple

Aruna (–) (Housewife)

Daughter-in-law

Son

S– Doctor

(–)

Sandhye (Lawyer)

Children

C/P (Delhi)

B (Shimla)

Brother

(Doctor)

of students which are A, D, E, F and B, C, G, H.

Hence, Ramesh works in company B.

(Lucknow) C/P

A

Son

Playing Nature

Company B D A/C E A/C

(+)

(Engineer) E

ther

Size

Profession House colour Cooking

Grandfa

Team

Person Q

So, C is A’s son.

Friend

Sol. (Q. Nos. 13-15) According to the question, the arrangement is as follows P

3. (d) According to the question, the

Person Ramesh Jim Manish Nilesh Sagar

Raj

Hence, Jai worked on Tuesday.

Now, it is clear from the above table that B does not belong to S. So, statement (d) is incorrect. arrangement is as follows

Finally

12. (c) We have total six student sitting on the ground Ruby House → A, B Emerald House → C, D, E, F; Tall → D, F Short → A, B, C, E Wearing glasses → C, D Wearing no glasses → A, B, E, F We have to find out girl of Emerald House who is tall and wearing glasses. So, we have to find out the common students in the given condition. Therefore, D is the only student who is wearing glasses, tall and belongs to Emerald Houses.

U (+, –) Engineer/Student

So, she is a student is correct and either R or U be a student.

https://sscstudy.com/

Mother

Charu

Son

Married couple

Suraj

(+)

(Engineer)

Female – Student Male – Doctor

19. (d) Aruna is the grandmother of Charu. 20. (b) Aruna is the housewife. 21. (c) The grand-daughter of the family is the student.

https://sscstudy.com/

66

CUET (UG) Section III : General Test

CHAPTER 14

Venn Diagram Venn diagram is the diagrammatic representation of a particular group of objects. In it, different figures are drawn to represent relationship between two or more objects. The figures used are generally circular but sometimes square, triangle and rectangles are also used. In these Venn diagrams, the basic concept of set, subset, disjoint set etc. are used. (1) An object is a subset of another object if former is a part of latter and this relation is shown by two concentric circles. e.g. Chair, furniture.

Here, a group of three items are shown by three circles and their relation can be as follows. (a) Three items belong to three different groups A

C

B

(b) One item belongs to the class of second and second belongs to the class of third.

A B C

Chair

(c) Two separate items belong to the class of third.

Furniture

(2) An object is said to have an intersection with another object when two objects share something in common. e.g. Surgeon, Males

A

B C

Surgeon

Males

(d) Two items with some common relationship belong to the class of third.

(3) Two objects are said to be disjoint when neither one is a subset of another nor they share anything in common. e.g. Furniture, Car.

A

B C

Furniture

Car

(e) Two items are partly related to the third and are themselves independent of each other. The questions related to Venn diagram are based on the following types

1. Identification of Relation Based on

A

C

B

(f) Three items are partly related to each other.

Venn Diagram In this type, geometrical figures in the form of circles are given that represent different classes. These different classes share some common relationship or sometimes they are completely disjoint too.

https://sscstudy.com/

A

B C

https://sscstudy.com/

GENERAL MENTAL ABILITY

(g) One item belongs to the class of second while third item is different from the two.

67

Ex. 6 Tea, Coffee, Beverages Sol. 2. (b) Iron is a metal but chlorine is a non-metal.

B C

A

Iron

(h) One item belongs to the class of second and the third item is partly related to these two.

Chlorine

Metal

3. (d) Some houses and some bridges are made of bricks.

B C

A

House

Brick

(i) One item belongs to the class of second and third is partly related to second.

Bridge

A

B C

4. (c) Potato and cabbage are entirely different. But both are vegetables.

(j) Two items are partly related to each other and the third item is entirely different from the two.

Vegetables Potato

A

Cabbage

C

B

Ex. 1 Which of the following diagrams represents pens and pencils ?

5. (b) All thieves are criminals. But judge is different. Judge

(a)

(b)

(c)

(d)

Thief

Sol. (c) Pens and Pencils both are used for writing but Criminal

both are entirely different. Pens

6. (c) Tea and coffee are entirely different. But both

Pencils

belong to the class of Beverages.

Directions (Ex. Nos. 2-6) In the following questions, three classes are given. Which of the following figures will represents the best relationship amongst them?

Tea

Coffee

Beverages

(a)

(b)

(c)

Ex. 2 Metal, Iron, Chlorine Ex. 3 Brick, House, Bridge Ex. 4 Vegetables, Potato, Cabbage Ex. 5 Judge, Thief, Criminal

(d)

2. Analysis Based Venn Diagram In this type, generally a venn diagram is given. Each geometrical figure in the diagram represents a certain class. The candidate is required to analyse all the figures carefully and answer the questions.

https://sscstudy.com/

https://sscstudy.com/

68

CUET (UG) Section III : General Test

Directions (Ex. Nos. 7 and 8) In the following diagram, three classes of population are represented by three figures. The triangle represents school teachers, the square represents the married persons and the circle represents the persons living in joint families. F

27 16 37 (c) 16

D

28 16 38 (d) 16

(a)

(b)

Sol. (d) Number of people who are train travelers = 50 + 100 + 50 + 50 + 100 + 10 + 20

C

B

E

Ex. 9 The ratio of total number of train travelers to the people who do not travel by train is

= 380

A

Number of people who do not travel by train

Ex. 7 Married persons living in joint families but not working as school teachers are represented by (a) C

(b) F

(c) D

represented by the region common to the square and the circle, i.e. D and B. But according to the given conditions, the persons should not be school teachers. So, B is to be excluded. Hence, the required condition is denoted by region D.

Ex. 8 School teachers who are neither married nor do live in joint families are represented by (b) C

= 160

(d) A

Sol. (c) Married persons living in joint families are

(a) F

= 100 + 20 + 40

(c) B

(d) A

380 160 38 = 16

∴ Required ratio =

Ex. 10 How many people who travel by train also travel by car or bus but not cycle? (a) 50 (c) 80

(b) 70 (d) 100

Sol. (a) School teachers are represented by the triangle.

Sol. (c) Number of people who travel by train also

But according to the given conditions, persons are neither married nor do they live in joint families. So, the region should not be a part of either the square or the circle. Such a region is F.

travel by car or bus but not cycle = 10 + 20 + 50 = 80

Directions (Ex. Nos. 9-11) Study the following diagram and answer questions based on it. 100 100

50

20 40

10 50 50

100

Car Bus

Ex. 11 Which two modes of transport used by people are mutually exclusive? (a) Car—Bus (b) Car—Cycle (c) Bus—Cycle (d) Cannot be determined

Train

Sol. (b) From the diagram it is clear that car → ‘

Cycle

and cycle ‘

20

https://sscstudy.com/

’ are mutually exclusive.



https://sscstudy.com/

GENERAL MENTAL ABILITY

69

Practice Questions 1. Which of the following diagrams best depicts the relation among Red, Pink and Colour?

9. Topic, Book, Chapter 10. Clothes, Saree, Dhoti 11. Forests, Earth, Mountains 12. Which of the following diagrams best depicts the

(a)

(b)

(c)

relation among Liquids, Milk and River water?

(d)

2. Identify the diagram that best represents the relationship among the classes given below. Book, Pen, Pencil

(a)

(b)

(c)

(d)

Directions (Q. Nos. 13 and 14) Study the following diagram carefully and answer the following questions. (a)

(b)

(c)

(d) 2

Directions (Q. Nos. 3-6) In the following questions, three

7 3 6

classes are given. Out of the four figures that follows, you have to find the figure that best represents the relationship amongst the three given classes.

4

1

5

13. Find the number that lies inside all the figures. (a) 3

(b) 6

(c) 4

(d) 7

14. Find the number that lies only inside the triangle. (a)

(b)

(c)

(a) 1 (c) 7

(d)

3. Elephants, Wolves, Animals

(b) 4 (d) There is no such number

Directions (Q. Nos. 15-17) In the following figure, triangle represents Doctors, Square represents Young persons and circle represents Rural peoples. Study the diagram carefully and answer the given questions.

4. Tea, Coffee, Drinks 5. Boys, Students, Athletes 6. Police, Thief, Criminal

Doctors

7. Which of the the following figures represents the

A

relationship among Doctor, Man and Actor ?

B C E

(a)

(b)

(c)

D

F

Young G

Rural

(d)

15. Find the portion that represents young Doctors that 8. Identify the diagram that best depicts the relation among Classroom, Black board and School?

live in rural areas. (a) F

(b) G

(c) E

(d) C

16. Find the portion that represents doctors living in rural area but are not young. (a) E (a)

(b)

(c)

(b) A+B

(c) B

(d) D+C

(d)

17. Find the portion that represents people living in rural

Directions (Q. Nos. 9-11) In the following questions,

area who are neither Doctors nor young.

three classes are given. Out of the four figures that follows, you have to find the figure that best depicts the relation amongst the three given classes.

(a) B (c) C

(a)

(b)

(c)

(d)

(b) E (d) F

Directions (Q. Nos. 18-20) In the following figure, rectangle represents males, triangle represents educated people, square represents public servants and circle represents people living in urban area. Study the diagram carefully and answer the given questions.

https://sscstudy.com/

https://sscstudy.com/

70

CUET (UG) Section III : General Test

Urban

3 12

Public servants

10

7

6

13

4

14

43

5

28

12

(c) 11

(d) 9

24. How many persons like reading?

19. Out of the following options, how many people live in urban area who are neither public servants nor educated or males? (a) 3

(b) 5

(c) 6

(c) 6

(b) 154

(c) 176

(d) 117

cycling?

(d) 10

(a) 12

(b) 31

(c) 44

(d) 28

26. How many persons like reading and cycling but not

males are living in urban area? (b) 5

(a) 170

25. How many persons like trekking and reading but not

20. Out of the following options, how many educated (a) 4

44

36

males do not live in urban area? (b) 4

26

12

Educated

18. Out of the following options, how many educated (a) 10

31

Males

11

trekking?

(d) 10

(a) 31

Directions (Q. Nos. 21-23) Study the following diagram and answer questions based on it. The diagram represents the likes of kids of a class.

(b) 12

(c) 26

(d) 28

Directions (Q. Nos. 27-29) Study the following diagram and answer questions based on it. 5

13 16

10

13

Jupiter

11

4

2

12 14

20

15

10

5 3

Mercury

6

14

12

Mars

5

21. The ratio of kids who like only Mars to those who like Tennis players

all the three is (a)

7 5

(b)

7 8

(c)

7 6

(d)

14 13

Hockey players Chess players

22. What is the difference between the kids who like Mercury and Jupiter? (a) 6

(b) 8

(c) 10

27. The number of Non-Chess players is how much more

(d) 12

than the players who play both Chess and Tennis? (a) 5 (c) It is the same

23. How many people who like Mars like Jupiter also? (a) 45

(b) 29

(c) 26

(d) 13

Directions (Q. Nos. 24-26) Study the diagram given

28. The number of Tennis players who are not Hockey players is

below and answer the questions based on it.

(a) 10

∆ represents persons who like reading represents persons who like cycling represents persons who like trekking

(b) 10 (d) Cannot be determined

(b) 50

(c) 55

(d) 35

29. The ratio of Hockey players to Chess players is (a)

9 18

(b)

10 18

(c)

11 18

(d)

15 18

ANSWERS 1. 11. 21.

(c) (c) (d)

2. 12. 22.

(c) (a) (a)

3. 13. 23.

(c) (a) (b)

4. 14. 24.

(c) (a) (a)

5. 15. 25.

(a) (d) (d)

6. 16. 26.

(b) (c) (a)

https://sscstudy.com/

7. 17. 27.

(d) (b) (c)

8. 18. 28.

(a) (c) (d)

9. 19. 29.

(a) (a) (c)

10. 20.

(d) (a)

https://sscstudy.com/

General Mental Ability

71

Hints & Solutions 1. (c) Red and Pink both belong to the group of colour. But they both are entirely different.

14. (a) The number that lies only inside the triangle is 1.

Man Actor

Doctor Red Pink

8. (a) Blackboard is a part of classroom and

Colour

classroom is a part of school.

2. (c) Book, Pen and Pencil, all the three belong to stationery items but they are entirely different.

Black board

Book

Classroom

Pen

School

Pencil

9. (a) A book has chapters and chapter has 3. (c) Elephants and Wolves are entirely

topics.

different. But, they both belong to the class of animals.

Wolves

Animals

Book

10. (d) Dhoti and Saree are entirely different

4. (c) Tea and Coffee are entirely different. But

but both belongs to the group of clothes.

both belong to the class of drinks.

Clothes Dhoti

Tea

Saree

Coffee Drinks

11. (c) Mountains and Forests are part of 5. (a) Some boys can be students and some boys can be Athletes. Also, some athletes can be students.

Earth. But, Some mountains are forested and some forests are mountainous. Earth

Students

Mountains

Forests

Boys

are not thieves. Also, police is entirely different. Criminals

17. (b) The region ‘E’ is present in circle only and hence is the required answer. 18. (c) Only 11 educated males do not live in urban area. 19. (a) Only 3 persons live in urban area who are neither public servants nor educated or males.

21. (d) Number of kids who like only Mars = 14 Number of kids who like all the three = 13 14 ∴ Required ratio = 13 22. (a) Number of kids who like Mercury = 10 + 12 + 13 = 35 Number of kids who like Jupiter = 16 + 13 = 29 ∴Required difference = 35 − 29 = 6

23. (b) Number of kids who like Mars like Jupiter also = 13 + 16 = 29 24. (a) Number of persons who like Reading = 43 + 31 + 12 + 28 + 12 + 44 = 170

25. (d) Number of persons who like Trekking and Reading but not Cycling = 28 26. (a) Number of persons who like Reading and Cycling but not Trekking = 31

Athletes

6. (b) All thieves are criminals, but all criminals

16. (c) The region ‘B’ is common to triangle and circle only. Hence, ‘B’ represents Doctors living in rural area but are not young.

20. (a) Only 4 educated males are living in urban area.

Topic Chapter

Elephant

15. (d) The region ‘C’ is common to all the figures. Hence, ‘C’ represents young Doctors that live in rural areas.

12. (a) Milk and River water both belong to the group of liquids. But they both are entirely different.

27. (c) Number of players who play both Chess and Tennis = 11 + 10 + 14 = 35 Number of Non-Chess players = 35 Clearly, the number of Non-Chess players are same the number of players who play both Chess and Tennis. 28. (d) The number of Tennis players who are not Hockey players = 5 + 11 + 14 + 5 = 35 29. (c) Number of Hockey players

Thieves Milk

Police

7. (d) Doctors and Actors are entirely different. But some doctors and actors are men.

River water

Liquid

13. (a) The number common to triangle, circle and rectangle is 3.

https://sscstudy.com/

= 5 + 2 + 3 + 10 + 15 + 20 = 55 Number of Chess players = 2 + 13 + 11 + 4 + 15 + 6 + 14 + 12 + 3 + 10 = 90 55 11 = ∴Required ratio = 90 18

https://sscstudy.com/

72

CUET (UG) Section III : General Test

CHAPTER 15

Cube and Dice Cube A cube is a three dimensional solid object bounded by six square faces Face

Number of smaller cubes with 1 side painted = Number of central cubes Number of cubes with no side painted = Number of nucleus or Inner central cubes.

Ex. 1 How many smaller cubes of 1 cm side can be formed with a solid cube of 3 cm side ?

Edge

(a) 3

Corner

=

A cube has 6 faces, 8 corners and 12 edges. If a larger cube is cut into smaller cubes of equal volume so that each edge is divided into n parts, then A A



B C

B B

A A

B B

A

A

B

A

B

C

B

A

B

A

A

n=

(d) 27

Volume of bigger cube (3)3 = (3)3 = 27 = Volume of smaller cube ( 1 )3

Ex. 2 A cube is coloured red on all its faces. It is then cut into 64 smaller cubes of equal size. The smaller cubes so obtained are separated. How many cubes have no faces coloured? (a) 24 (c) 8

A

Sol. (c) Number of smaller cubes = n 3 =64 ⇒ n = 3 64 = 4

B

(b) 16 (d) 10

Now, cubes with no painted surface are inner central cubes or nucleus cubes. ∴ Number of nucleus cubes = (n − 2)3 = (4 − 2)3 = 8

A

Number of smaller cubes = ( n )3 where,

(c) 9

B C

B

(b) 6

Sol. (d) Required number of smaller cubes

Ex. 3 How many cubes are there in this diagram?

Edge of larger cube Edge of smaller cube



Number of corner cubes (exist at each corner) = A = 8



Number of middle cubes (exist at middle of each edge) = B = 12 ( n − 2)



Number of central cubes (exist at centre of each face) = C = 6 ( n − 2)2



Number of nucleus cube/Inner Central Cube (hidden and exist at centre of large cube)

(a) 16 (c) 10

(b) 12 (d) 8

Sol. (b)

1 1

2 2

3 3

4 4

3

= D = ( n − 2)

12 12

Note When a large cube is painted with some colour and then divided into smaller cubes of equal size, then after separation, number of smaller cubes so obtained are as follows,

11

Number of smaller cubes with 3 sides painted = Number of corner cubes Number of smaller cubes with 2 sides painted = Number of middle cubes

10

5

11 9 9

8 8

6 7

4 5 6 7

So, there are 12 cubes in the given figure, which are illustrated above.

https://sscstudy.com/

https://sscstudy.com/

GENERAL MENTAL ABILITY

Dice

73

upper left

A dice is a three dimensional figure with 6 square faces.

right

front

back

lower

upper back

There are two types of dice. (a) When the common digit is on different faces When the common digit is on different faces, then list these digits in clockwise direction starting from common digit and then compare them. e.g. 4 2 3

1

6

lower

Here, Upper face is opposite to Lower face. Front face is opposite to Back face, and Right face is opposite to Left face.

Ex. 4 What number will come at the opposite face of 1 in the following dice ?

5

2

3 (i)

(a) 4

(ii)

(b) 5

(c) 3

Ex. 5 When the following figure is folded to form a cube, then which number will be opposite to 1?

(a) 6

1 6 4 5

3

(b) 4

(c) 5

Sol. (b)

1 6 4 5

2

Spread out Form of a Dice The spread out form of a dice is shown on the basis of the front side of the dice.

(d) 2

the remaining face containing number ‘4’ will be opposite to ‘1’.

2

Here, uncommon digits in each dice are opposite to each other. So, 4 is opposite to 6 and 3/2 is opposite to 1/5

6

Sol. (a) The adjacent faces of ‘1’ are 2, 3, 5 and 6. So,

3

(d) 3 upper

3

left

front

right

lower back

We know that upper face is opposite to lower face. So, 1 is opposite to 4.

Ex. 6 On the basis of the given two positions of single dice, find the letter at the face opposite to the face having letter A.

Upper

Right

Left

1

1

Here, 1 is common in both position. So move in, clockwise direction starting from 1. We get, 1 → 2 → 3 1→ 5→ 4 So, 3 is opposite to 4 2 is opposite to 5 and 1 is opposite to 6 (b) When two digits are common e.g. 2 2 3

right

front

1

5

4

left

Front

D

A

Lower

B

C

E

C

Back

(I)

(a) B

back

upper left

front lower

(d) D

From Position II,

right

Clearly, E is opposite to A.

https://sscstudy.com/

C

C

B

D

Opposite

lower

From Position I,

back

right

(c) E

Opposite

front

(b) C

Sol. (c) Common letter = C (on different face)

upper left

(II)

Common

One can unfold a dice in many ways, some of which are given below.

A

E

https://sscstudy.com/

74

CUET (UG) Section III : General Test

Practice Questions 1. How many cubes are there in the given figure?

10. Find the digit at the face opposite to the face having digit 2 in the dice given below. 1 2

(a) 3

(b) 5

(c) 7

(d) 13

2. Count the number of blocks in the given figure.

(a) 3, 4

6

(b) 3, 5

(c) 4, 5

11. Two positions of a dice are given. Based on them, find out which number is opposite to number 4 in that dice? 6

6 3

2

(a) 1 (a) 7 (c) 13

(b) 11 (d) 15

(c) 3

(c) 18

digit at the face opposite to the face having digit 2.

(d) 24

4. If a cube of 12 cm side is divided into smaller cubes of 3 cm side, then find the total number of inner central cubes? (a) 18

(b) 9

(c) 8

(d) 4

12. From the given two positions of a single dice, find the 6

2

side 1 cm. What will be the total number of middle cubes? (b) 12

4

3

(b) 2

3. A cube of 3 cm side is divided into smaller cubes of (a) 6

(d) 3, 4, 5

3

6

(a) 4

1

4

(b) 5

(c) 3

(d) 1

13. Which number is on the face opposite to face having number 1? 2

5

2

5

(d) 81

1

3

6

4

1

5. A cube having 24 cm side is divided into 64 smaller cubes of equal volume. Find the edge of smaller cubes? (b) 6 cm (d) 4 cm

(b) 3

(c) 4

14. Which number is on the face opposite to face having number 3?

Directions (Q.Nos. 6-8) Read the given information carefully and answer the questions that follow. All the surfaces of a cube of side 15 cm are painted with red colour and then it is cut into smaller cubes of side 3 cm. The smaller cubes so obtained are separated. 6. How many smaller cubes are there having two

3 6

(a) 2

2

(c) 32

V

(d) 64

faces painted with red colour? (c) 4

U

(i)

(ii)

n Gree Orange

(a) 2 (c) 5

1

(i)

(b) 6/2 (d) 3

(d) I

16. From the given four positions of a single dice, find the

digit 4 in the given dice. 4

(c) J

colour at the face opposite to the face having green.

(d) 2

9. Find the digit at the face opposite to the face having 5

J

I

(b) U

Re d

(b) 0

S J

(a) S

(a) 8

(d) 6

letter at the face opposite to the face having letter V.

(d) 54

8. How many smaller cubes are there having 4 or more

6

15. From the given two postions of a single dice, find the

surfaces painted with red colour? (b) 10

3

1

(c) 4

7. How many smaller cubes are there having only three (a) 8

5

(a) Orange (c) Silver

https://sscstudy.com/

White Orange

(ii)

g Oran Violet

e

(iii)

(b) Red (d) Violet

r Silve Green

(iv)

Re d

(c) 36

4

(b) 3

Gree n

(b) 24

2

3

5 1

surfaces painted with red colour? (a) 8

(d) 5

Re d

(a) 5 cm (c) 3 cm

(a) 2

https://sscstudy.com/

GENERAL MENTAL ABILITY

19. Choose the cube which will be formed on folding the

17. Four positions of dice are given below. Which letter

question figure. Question figure

will be opposite to D? B

(i)

C

E A

C C

A

(iii)

(b) B

D E

D

(c) A

(iv)

B

C

(d) C

T

(a) E

(ii)

75

18. When the following figure is folded to form a dice,

O K L M U

then which number will be opposite to 5? 3

(b) 2

Answer figures L

K T

(a) 1

1 2 4 6

5

(c) 3

(d) 4

M

L

(a)

K K

T

(b)

O U

K

(c)

L

(d)

ANSWERS 1. 11.

(c) (b)

2. 12.

(a) (d)

3. 13.

(b) (b)

4. 14.

(c) (c)

5. 15.

(b) (a)

6. 16.

(c) (d)

7. 17.

8. 18.

(a) (c)

(b) (c)

9. 19.

10.

(d) (a)

(d)

Hints & Solutions 1. (c) Number of cubes in upper row = 2 Number of cubes at the bottom = 5 ∴ Total cubes = 2 + 5 = 7

2. (a) Number of blocks at the top = 3 Number of blocks at bottom = 4 ∴ Total blocks = 3 + 4 = 7

5 + 4 = 9; 5 + 1 = 6 1 + 4 = 5 [not equal to 7] Hence, it is a standard dice. Now, in a standard dice, sum of opposite faces is 7. Hence, 4 is opposite to 3.

Clearly, violet is opposite to green.

17. (c) Dice can be shown as, B D C A

10. (d) Here, the sum of adjacent faces

3. (b) Number of middle cubes = 12 ( n − 2 ) 3 = 12  − 2  = 12 × 1 = 12 1 

4. (c) Total number of inner central cubes 3

12 = (n − 2 )3 =  − 2  = (2 )3 = 8  3 

2 + 1 = 3, 2 + 6 = 8, 1 + 6 = 7 Hence, it is a general dice. Digits at the opposite face of 2 = 3, 4 or 5.

11. (b) In the given two figures, two numbers are common, i.e. 3 and 6. So, remaining two numbers are opposite to each other . Hence, 2 is opposite to 4.

12. (d) In both the positions, 6 is common.

5. (b) We know that,

6. (c) We know that Side of bigger cube 15 = =5 Side of smaller cube 3 Smaller cubes having 2 surfaces painted = Number of middle cubes = 12 ( n − 2 ) = 12 ( 5 − 2 ) = 36 n=

7. (a) Smaller cubes having 3 surface painted = Number of corrner cubes = 8

8. (b) There is no such cube with 4 or more faces painted.

9. (d) Here, the sum of adjacent faces

6

3

4

?

So, from the unfolded dice, it is clear that the letter opposite to D is A.

18. (c) Taking 2 as front face, we get 1 as upper face and 4 as lower face. Similarly, 3 is left face, 5 is right face and 6 is the back face. Hence, 3 is opposite to 5. 19. (a) The unfolded dice can be represented as

1

Moving in clockwise direction from 6, we get 6 → 2 → 3 6 → 1 → 4 Hence, 1 is opposite to 2.

13. (b) The numbers 2, 4, 5 and 6 are on the surfaces adjacent to 1. Hence, 3 is opposite to 1.

14. (c) Figure (i), (iii) and (iv) show that numbers 6, 1, 5 and 2 appears on the surfaces adjacent to the number 3. Hence, 4 will be opposite to number 3. 15. (a) Here J is common in both the positions. Hence, letter ‘S’ is at the face opposite to the face having letter V. 16. (d) From the given four positions of a single dice, faces adjacent to face having green colour = Orange, red, white, silver

https://sscstudy.com/

O Opposite

K Opposite

T

n =3 64 = 4 Edge of bigger cube Also, n= Edge of smaller cube 24 ⇒ Edge of smaller cube = = 6 cm 4 Sol. (Q. Nos. 6-8) ⇒

6

2

Number of smaller cubes = n3 = 64

E

L M

U

Opposite

Here, O and L are opposite, K and M are opposite and T and U are opposite. In answer figure (b), K and M are shown adjacent to each other. In answer figure (c), T and U are shown adjacent to each other and in answer figure (d), O and L are shown adjacent to each other. According to the question figure these are opposite to each other, so these alternatives are not possible. Only the cube given in answer figure (a) can be formed because K, L and T can be on adjacent faces.

https://sscstudy.com/

76

CUET (UG) Section III : General Test

CHAPTER 16

Non-Verbal Reasoning Non-Verbal reasoning tests candidates thinking ability, ability to identify similarity and dissimilarity and come to a conclusion quickly on the basis of given figures, patterns and symbols. The following topics are covered in this chapter. 1. Analogy 2. Classification 3. Series 4. Embedded figure 5. Counting of Figures 6. Figure completion 7. Paper cutting and folding 8. Mirror & Water Image.

Sol. (c) First design of first pair of problem figures is inverted and then placed on the original figure to get the second figure. Similar concept is used in second pair.

Ex. 2 Find the figure from the set of answer figures that will replace the question mark ‘?’. Problem figures

? Answer figure

1. Analogy Analogy refers to correspondence or similarity in relationship. When a figure exhibits some kind of relationship with another figure on some basis then the two figures are said to be analogous to each other. In questions based on analogy, two sets of figures namely problem figures and answer figures are given. The set of problem figures consist of two parts which bear certain relationship between them. The second part comprises of one figure and a sign of ‘?’. The candidate is asked to select one figure from the set of answer figures to replace ‘?’ but maintaining similar relationship as depicted between the first two figures.

Ex. 1 Find the figure from the set of answers figures that will replace the question mark‘?’. Problem figures

(a)

(b)

(c)

(d)

Sol. (c) The figure rotates through 180° anti-clockwise. On following the same pattern, the answer figure (c) will replace the question mark ‘?’.

2. Classification Classification is the process of putting things or objects into a group and then finding the different object or thing that does not belong to that group based on some common properties. In questions based on classification, the candidate has to find an odd figure from a set of four figures based on some common properties.

Ex. 3 Find the odd figure from the set of given figures. ?

Answer figures (a)

(b)

(c)

(d)

Sol. (b) Except figure (b), in all other figures the main (a)

(b)

(c)

(d)

figure is divided into four parts while in second figure two parts are there.

https://sscstudy.com/

https://sscstudy.com/

77

GENERAL MENTAL ABILITY

Ex. 4 Find the odd figure from the set of given figures.

Ex. 7 Find the answer figure in which the question figure is embedded. Question figure

+ + × + + + + × + + × + × + (a) (b) (c) (d) +

Answer figures

+ +

(a)

Sol. (b) Figure (b) has 4 plus signs and other have 3.

(b)

(c)

(d)

Sol. (c) The question figure is embedded in answer Fig.

3. Series

(c)

Series is a continuous sequence of figures following a certain defined pattern. In this type of questions, a series of figures is given as problem figures and the candidate is asked to select one figures from the set of answer figures which will continue the given sequence.

Ex. 5 Find the figure that will continue the set of problem figures.

Ex. 8 Find the answer figure which is embedded in the question figure. Question figure Answer figure

Problem figures

(a)

(b)

(c)

(d)

Sol. (b) The answer figure (b) is embedded in the

Answer figures

question figure.

(a)

(b)

(c)

(d)

Sol. (b) In each step, a pin is added on the left hand side of existing pin/pins. The head of the pin is in a direction opposite to adjacent pin.

Ex. 6 Find the figure that will continue the set of problem figures. Problem figures

?

(b)

(c)

Counting of figures is simply the realisation of different geometrical figures from a complex one. In questions based on counting of figures the candidate is asked to count straight lines, triangles, squares, circles etc. from a given figure.

Ex. 9 How many squares are there in the figure given below?

Answer figures

(a)

5. Counting of Figures

(d)

Sol. (d) In each successive problem figure, black dot moves three steps clockwise and white circle moves two steps clockwise.

(a) 4 (c) 6

(b) 5 (d) 7

Sol. (b) There are 5 squares in the given figure

4. Embedded Figures

A

A figure is said to be an embedded figure when that figure is completely contained by another figure.

H

Questions based on embedded figures comprise of a question figure and four answer figures and it is asked to find to correct answer figure embedded in given figure or the correct answer figure in which the given question figure is embedded.

D

E

I G

B F

C

viz. ABCD, AEIH, EBFI, FICG, and HIGD.

https://sscstudy.com/

https://sscstudy.com/

78

CUET (UG) Section III : General Test

Ex. 10 How many triangles are there in the figure given below?

Ex. 12 Choose the answer figure that will complete the question figure. Question figure

(a) 20 (c) 18

?

(b) 27 (d) 29

Sol. (a) Naming the figure,

(a)

(b)

(c)

(d)

Sol. (b) Clearly, option (b) completes the original figure

B

A

Answer figure

which looks like the figure given below

C

I

J D

H

M

K

L G

E

F

Smallest triangles are ∆ABI, ∆BCJ, ∆CDJ, ∆DEK, ∆EFK, ∆FGL, ∆GHL, ∆HAI = 8 Small triangles formed with two triangles ∆BCD, ∆DEF, ∆FGH, ∆HAB = 4 Large triangles formed with two triangles and one square are ∆ACM, ∆CEM, ∆EGM, ∆GAM = 4 Largest triangles ∆ACE, ∆AGE, ∆GAC, ∆GEC = 4 ∴ Total triangles = 8 + 4 + 4 + 4 = 20

Ex. 11 How many circles are there in the figure given below?

(a) 4 (c) 9

(b) 8 (d) 10 2

3

5

6

Question figure

?

Answer figure

(a)

(b)

(c)

(d)

Sol. (a) Clearly, option (a) completes the question figure which looks like the figure given below.

7. Paper Cutting and Folding

Sol. (b) 1

Ex. 13 Choose the answer figure that will complete the question figure.

4

Paper folding and cutting problems are based on a transparent sheet which is folded along a dotted line or folded and cut (or punched) in a particular manner.

7

In questions based on paper folding, a transparent sheet having a certain design and a dotted line is given on it. The candidate is required to identify the design that would appear on the sheet when it is folded along the dotted line.

8

Clearly, total number of circles = 8

6. Figure Completion Figure completion is a process to find out the missing part of an incomplete figure to complete it.

In questions based on paper cutting, few figures are given showing the way in which a piece of paper is to be folded and then cut to form a particular section.

In this type of questions, a figure is given and a part of 1 it is missing, generally th part of the figure and the 4 candidate is asked to find that missing part from the given set of answer figures.

Ex. 14 In the given question, a square transparent sheet with a pattern is given. Find out from amongst the four alternatives as to how the pattern would appear when the transparent sheet is folded along the dotted line.

https://sscstudy.com/

https://sscstudy.com/

79

GENERAL MENTAL ABILITY

Transparent sheet

Object

Answer figures

(a)

(b)

(c)

Water

(d)

Sol. (c) The transparent sheet will appear like option figure (c) Water Image

Directions (Ex. Nos. 15 and 16) In the following questions a set of three figure (X), (Y) and (Z) have been given showing a sequence in which paper is folded and finally cut from a particular section. These figures are followed by a set of answer figures marked (a), (b), (c) and (d) showing the design which the paper actually acquires when it is unfolded. You need to select the answer figure which is closest to the unfolded piece of paper.

Ex. 15 Question figures

Note If the mirror is placed along the horizontal line than the mirror image of the object would be same as water image.

Ex. 17 Find the mirror image of the question figure when the mirror is placed along the line MN. Question figure

Answer figures

M

(a)

N

Answer figures

(b)

(c)

(d)

Sol. (c) Option figure (c) is the correct mirror image of question figure.

X

Y

Z

(a)

(b)

(c)

(d)

Sol. (d)

Ex. 18 If a mirror is placed on the line XY, then which of the answer figure is the right image of the given figure?

Ex. 16 Question figures

X

Y

Z

Question figure

Answer figures

(a)

(b)

(c)

Answer figures

X

(d)

Sol. (c)

(a)

Y

(b)

(c)

(d)

Sol. (a) Option figure (a) is the correct mirror image of question figure

8. Mirror and Water Image

Object

Question figure Answer figures ×××× ×××× ×××× ××××

–––– –––– –––– ––––

–––– –––– –––– ––––

In mirror image, the left hand side and right hand side of an object interchange their places, while top and bottom remains constant.

Ex. 19 Find the Water image of the question figure.

– – – – ×××× – – – – ×××× – – – – ×××× – – – – ××××

×××× ×××× ×××× ××××

Mirror image is the reflection of an object into the mirror, and water image of an object is its reflection into the water.

(a)

Mirror Image

(b)

×××× ×××× ×××× ××××

–––– –––– –––– ––––

(c)

–––– –––– –––– –––– ×××× ×××× ×××× ××××

(d)

Sol. (d) option figure (d) is the correct water image of question figure. –––– –––– –––– ––––

Mirror

In water image, the top and bottom of an object interchange their places, while the left hand side and right hand side remains constant.

https://sscstudy.com/

×××× ×××× ×××× ××××

https://sscstudy.com/

80

CUET (UG) Section III : General Test

Practice Questions Directions (Q. Nos. 1-5) Find a

Answer figures

12. Question figures

figure from the set of answer figures that will replace the question mark (?).

?

1. Question figures

(a) ?

(b)

(c)

(d)

Directions (Q. Nos. 6-10) In the following questions select the figure which is different from the other three responses.

Answer figures

Answer figures

(a)

(b)

(c)

(d)

13. Question figures 6. (a)

(b)

(c)

?

(d)

(a)

2. Question figures

(b)

(c)

(d)

Answer figures

7. ?

(a)

Answer figures

(b)

(c)

(d)

8.

(a)

(b)

(c)

(b)

(c)

?

(d)

?

Answer figures (a)

(b)

(c)

(d)

10.

Answer figures

(c)

(d)

9.

3. Question figures

(b)

14. Question figures

(a)

(d)

(a)

(a)

(b)

(c)

(d)

15. Question figures (a)

(a)

(b)

(c)

(d)

4. Question figures ?

(b)

(c)

(d)

Directions (Q. Nos. 11-15) Find the figure form the set of answer figure that will follow the pattern of question figure and will replace question mark(?).

(a)

(b)

(c)

(d)

Directions (Q. Nos. 16-18) In the following questions from the given answer figures, select the one in which the question figure is hidden/embedded.

?

(b)

Answer figures

11. Question figures

Answer figures

(a)

?

(c)

(d)

Answer figures

16. Question figure

5. Question figures ?

(a)

(b)

(c)

https://sscstudy.com/

(d)

https://sscstudy.com/

GENERAL MENTAL ABILITY

Answer figures

21. What is the minimum number of

81

Answer figures

straight lines required to make the figure? (a)

(b)

(c)

(a)

(d)

(b)

(c)

(d)

(c)

(d)

(c)

(d)

(c)

(d)

(c)

(d)

27. Question figure

17. Question figure

(a) 20 (c) 11

(b) 15 (d) 7

Directions (Q. Nos. 22 and 23) Find the total number of triangles in the given figures.

Answer figures

Answer figures

22.

(a)

(b)

(c)

(d)

(a) 9 (c) 11

18. Question figure

(b) 10 (d) 12

(a)

(b)

28. Question figure

23.

Answer figures (a) 5 (c) 9 (a)

(b)

(c)

(d)

(b) 12 (d) 10

Answer figures

24. What is the number of circles in the following figure?

Directions (Q. Nos. 19 and 20) (a)

Which of the answer figure is hidden oblique line embedded in the question figure? 19. Question figure

(b)

29. Question figure (a) 8 (c) 12

(b) 10 (d) 15

25. Find the total number of squares

?

in the given figure. Answer figures

(a)

(b)

Answer figures

(c)

(d)

(a) 10 (c) 12

(b) 14 (d) 16

Directions (Q. Nos. 26-30) In the following questions, find the answer figure that will complete the pattern in the question figure.

20. Question figure

(a)

(b)

30. Question figure

?

26. Question figure Answer figures

Answer figures

(a)

(b)

(c)

(d)

?

(a)

https://sscstudy.com/

(b)

https://sscstudy.com/

82

CUET (UG) Section III : General Test

Directions (Q. Nos. 31 and 32) In each of the following questions, a figure marked as transparent sheet is given and followed by four answer figures, one out of these four options resembles the figure which is obtained by folding transparent sheet along the dotted line. Find the answer from these figures.

Answer figures

33. X

Y

Z

(a)

(b)

(c)

(d)

(c)

(d)

38. Question figure (a)

(b)

(c)

M

(d)

N

34.

31. Transparent sheet X

Y

Z

Answer figures Answer figures (a)

(b)

(c)

(d)

35. (a)

(b)

(c)

(a)

(b)

Directions (Q. Nos. 39 and 40)

(d) X

Y

Z

Find the water image of the given question figure.

32. Transparent Sheet

39. Question figure (a)

Answer figures

(a)

(b)

(b)

(c)

(d)

Direction (Q. Nos. 36-38) Find the mirror image of the question figure if the mirror is placed along the line MN. (c)

Answer figures

36. Question figure

(d)

M

Directions (Q. Nos. 33-35) In each (a)

of the following questions, a set of three figure (X), (Y) and (Z) has been given, showing a sequence in which a paper is folded and finally cut at a particular section. Below these figures a set of answer figures marked (a), (b), (c) and (d) showing the design which the paper actually acquires when it is unfolded are also given. You have to select the answer figure which is closest to the unfolded piece of paper.

(b)

(c)

(d)

(c)

(d)

N

40. Question figure

Answer figures

(a)

(b)

(c)

(d)

Answer figures

37. Question figure M APPROACH

(a)

N

(b)

ANSWERS 1. 11. 21. 31.

(c) (c) (c) (c)

2. 12. 22. 32.

(d) (b) (d) (a)

3. 13. 23. 33.

(c) (b) (d) (c)

4. 14. 24. 34.

(d) (b) (c) (b)

5. 15. 25. 35.

(b) (c) (b) (d)

6. 16. 26. 36.

(b) (b) (d) (a)

https://sscstudy.com/

7. 17. 27. 37.

(b) (b) (c) (c)

8. 18. 28. 38.

(c) (b) (d) (d)

9. 19. 29. 39.

(c) (c) (b) (c)

10. 20. 30. 40.

(c) (c) (b) (d)

https://sscstudy.com/

GENERAL MENTAL ABILITY

83

Hints & Solutions 1. (c) From first figure to second the main figure is rotated 90º clockwise. So, answer figure (c) will replace the ‘?’.

2. (d) From figure first to second, the figure inside the main figure rotates 90º clockwise and moves to the sides. So, answer figure (c) will replace the ‘?’

3. (c) From first figure to second the main figure is divided into six parts. So, option figure (c) will replace the ‘?’.

4. (d) From first figure to second the main figure is divided into two equal parts. So, option figure (d) will replace the ‘?’

12. (b) In each subsequent figure one line segment is added. So, option (b) figure will follow the same pattern.

21. (c) There are 5 horizontal lines and 6 vertical lines. ∴ Number of lines = 5 + 6 = 11

13. (b) The inner line segments move in anti-clockwise direction. So, option (b) figure will replace symbol ‘?’.

22. (d)

14. (b) In each subsequent figure. The design rotates clockwise (or anti-clockwise) through 90° along with the addition of a line segment. Hence, option (b) is the correct choice as it follows the corresponding pattern. 15. (c) In each step the elements are moving as follows

5. (b) From first figure to second, the outermost figure is removed and the second figure from inside becomes the outermost figure. So, option figure (b) will replace the ‘?’

6. (b) Except in figure (b) in all others the design has been divided vertically.

7. (b) In the given question option (b) is

two exactly opposite triangles are shaded. In option (c) shaded figure is not opposite to another one.

E F C

D

B

Number of triangles=12 viz. ∆AFB, ∆AFE, ∆FEC, ∆FDC, ∆BFD, ∆BFC, ∆AFC, ∆ABD, ∆ADC, ∆ABE, ∆CBE and ∆ABC

23. (d)

A

Following the above pattern option figure (c) will replace the question mark(?).

16. (b) Clearly, the question figure is embedded in answer figure (b) as shown below

B C D E

F

Number of triangles=10, viz. ∆ABC, ∆ACD, ∆ADE, ∆AEF, ∆ABD, ∆ACE, ∆ADF, ∆ABE, ∆ACF and ∆ABF

different of all other three options because in every option lines are meeting in shape except figure (b).

8. (c) Except in figure (c), in all other figures,

A

17. (b) Clearly, question figure is embedded in answer figure (b) as shown below

24. (c) 1

4

7

10

9. (c) Except in figure (c), in all other figures

2

5

8

11

the left and right designs consist of similar figures.

3

6

9

12

In figure (c), the left and right designs consist of different figure, so option (c) is correct.

18. (b) Clearly, question figure is embedded in answer figure (b) as shown below

Thus, there are all 12 circles in the figure.

25. (b)

10. (c) Except in figure (c), in all other figures, the inner design consists of less number of sides than that of the outer design. In option (c) the inner design have more sides that outer design.

19. (c) Clearly, the answer figure (c) is

11. (c) In each step the elements are moving

embedded after inverting in question figure as shown below.

A E

as follows.

20. (c) Clearly, the answer figure (c) is embedded in question figure as shown below

Following above pattern option figure (c) will replace the question mark(?).

https://sscstudy.com/

I M

B

C

F

G

J

K

N

O

D H L

P Number of squares=14 viz. ABFE, BCGF, CDHG, EFJI, FGKJ, GHLK, IJNM, JKON, KLPO, ACKI, BDLJ, EGOM, FHPN and ADPM.

26. (d) Clearly, option (d) figure will complete the pattern of the given figure.

https://sscstudy.com/

84

CUET (UG) Section III : General Test

27. (c) Clearly, option (c) figure will complete the pattern of the given figure

31. (c) The transparent sheet will appear like option figure (c)

37. (c) The mirror image would be like

32. (a) The transparent sheet will appear like option figure (a). 33. (c) 38. (d) The mirror image would be like

28. (d) Clearly, option (d) figure will complete the pattern of the given figure.

34. (b) 39. (c) The water image of the question figure would be

29. (b) Clearly, option (b) figure will complete the pattern of the given figure.

35. (d)

40. (d) The water image of the question figure would be.

30. (b) Clearly, option (b) figure will complete the pattern of the given figure.

36. (a) The mirror image would be like

https://sscstudy.com/

https://sscstudy.com/

CHAPTER 01

Number System Numbers are everywhere in our daily life for counting, measuring, labeling, ordering (sequencing) and coding. In the most popular, the Hindu-Arabic system, we use the symbol 0, 1, 2, 3, 4, 5, 6, 7, 8 and 9. These symbols are called digits. Out of these ten digits, 0 is called an insignificant digit whereas the other are called significant digits.

Number

Descending Order

A mathematical symbol represented by a set of digits called a number. Every digit has a face value which equals the value of the digit itself, irrespective of its place in the numeral. Each digit in a number or numeral has a place value besides its face value. We can express a number in two ways i.e., in words and in figures. e.g., If a number figure is 3290, then we can write this number in words as ‘three thousand two hundred and ninety’.

A number is said to be in descending order, if they are arranged with greatest number to smallest number. e.g., 13, 9,7, 5, 2 are in descending order.

Successor A number (say b) is said to be a successor of another number (say a) if a + 1 = b

Predecessor A number (say a) is said to be a predecessor of another number (say b ) if b − 1 = a

Face Value and Place Value of the Digits in a Number

Type of Numbers

Face Value (Real Value) In a number, the face value of a digit is the value of the digit itself. The place of the digit is not considered in the number. e.g., In the number 43857, the face value of 4 is 4, the face value of 3 is 3 and so on.

Place Value (Local Value) In a number, the place value of a digit changes according to the change of its place. Place value of units digit = (Unit digit) × 1 Place value of tens digit = (Tens digit) × 10 Place value of hundreds digit = (Hundreds digit) × 100 Place value of thousands digit = (Thousands digit) × 1000 and so on. e.g., In the number 48379, the place value of 4 is 4 × 10000 i.e., 40000, the place value of 8 is 8 × 1000 i.e., 8000 and so on.

Ascending Order A number is said to be in ascending order, if they are arranged with smallest number to greatest number. e.g., 2, 5, 7, 9, 13 are in ascending order.

Natural Numbers Counting number is known as natural number. e.g., 1, 2, 3, 4, ..., ∞ Whole Numbers Natural number including zero (0) is known as whole numbers. e.g., 0, 1, 2, 3, ..., ∞ Integers The set of positive and negative whole numbers including 0 is known as integers. e.g., ..., − 3, − 2 , − 1, 0, 1, 2, 3, ... p Rational Numbers The numbers of the form , where q q ≠ 0 and p and q is integer is known as rational number. 5 2 5 0 1 3 e.g., , − , − , , , etc. 1 1 7 1 5 5 Irrational Numbers The number, which is non-repeating and non-terminating, is called irrational number. e.g., 3 , 5 , 2 etc. Real Numbers All rational and irrational numbers are known as real numbers. 1 1 12 e.g., 5, 7, 3, , , , 3, 5 etc. 2 5 17 π is an irrational number.

https://sscstudy.com/

https://sscstudy.com/

4

CUET (UG) Section III : General Test

Even Numbers The number, which is multiple of or divisible by 2, is called even number. e.g., 2, 4, 6, 8, ... Odd Numbers The number, which is neither multiple of 2 nor divisible by 2, is called odd number. e.g., 1, 3, 5, 7, ... Prime Numbers The number, which has only two factors i.e., 1 and itself, is called prime number. e.g., 2, 3, 5, 7, 11, 13, ... 2 is only even number which is prime. Composite Numbers The number, which has more than two factors, is known as composite number. e.g., 4, 8, 9, 10, ... 4 is composite number because it is divisible by 1, 2 and 4. Consecutive Numbers These are series of numbers differing by 1 in ascending or descending order. e.g., 12, 13, 14, 15, ... Similarly, example of consecutive even numbers are 4, 6, 8, 10, ..., 22, 24, 26, 28 ... and so on. e.g. of consecutive prime numbers are 7, 11, 13, 17, ... Decimal Numbers A collection of digits (0, 1, 2, 3, ... ,9) after a point (called the decimal point) is called a decimal fraction. A number containing a decimal point is called a decimal number. Every decimal fraction represents a fraction. These fractions have denominators with powers of 10. As 4 6 7 35.467 = ( 3 × 101 ) + ( 5 × 100 ) + 1 + 2 + 3 10 10 10 4 6 7 = ( 3 × 10) + ( 5 × 1) + + + 10 100 1000 4 6 7 = 30 + 5 + + + 10 100 1000 400 + 60 + 7 467 35467 = 35 + = 35 + = 1000 1000 1000 Perfect Square Numbers The numbers whose square root can be determined, are called perfect square numbers. e.g., 4, 36, 225, … etc. Two Digits Numbers If unit place digit is y and ten digit is x, then the two digits numbers can be expressed as 10x + y. e.g., 87 = 10 × 8 + 7

Divisibility A number (dividend) is said to be divisible by another number (called divisor) when the quotient is a natural number and the remainder is zero. In other words, we can say that when a number (dividend) is divisible by another number (divisor), the dividend can be expressed

as multiple of the divisor. The relation so obtained can be given by Dividend = Divisor × Quotient Here, remainder equals zero.

Divisibility Tests Divisibility by 2 An number is divisible by 2, when its unit’s digit is either even or zero. e.g., The numbers 4, 6, 8, 10, 24, 28, 322, 4886 etc., are divisible by 2. 3, 5, 7, 11, 13, 221, 995, 4637 etc., are not divisible by 2. The rule emanates form the fact that any number multiplied by 2 gives product whose unit’s digit is either even or zero. Divisibility by 3 A number is divisible by 3 when the sum of its digits is divisible by 3. 426 ⇒ 4 + 2 + 6 = 12, which is divisible by 3. Hence, 426 is also divisible by 3. 8349 ⇒ 8 + 3 + 4 + 9 = 24, which is divisible by 3. Hence, 8349 is also divisible by 3. 7493 ⇒ 7 + 4 + 9 + 3 = 23. 23 is not divisible by 3. Hence, 7943 is also not divisible by 3. Note This rule comes from the fact that the multiples of 3 are numbers, the sum of whose digits is divisible by 3.

3 × 6 = 18; 1 + 8 = 9 9 is divisible by 3 3 × 27 = 81; 8+1= 9 3 × 14 = 42; 4+ 2= 6 3 × 220 = 660; 6 + 6 + 0 = 12 3 × 12547 = 37641; 3 + 7 + 6 + 4 + 1 = 21 and so on. In the above examples 9, 6, 12 and 21 all are divisible by 3. Divisibility by 4 A number is divisible by 4, when the number formed by its two extreme right digits is either divisible by 4 or both these digits are zero. e.g., 324, 5632, 3500 , 4320 and 89412 are divisible by 4 as they satisfy the above mentioned conditions. 323, 5131, 3510 and 54645 are not divisible by 4 as 23, 31, 10 and 45 respectively, are not divisible by 4 nor is the condition of double zero fulfilled. Note This rule is derived from the fact that 100 or multiples of 100 are always divisible by 4. So, if any such two digit number which is divisible by 4, is added to the multiples of 100, the sum will also be divisible by 4.

Divisibility by 5 A number is divisible by 5, when its unit’s digit is either 5 or zero. e.g., 2145, 630, 735 and 4500 are divisible by 5 as they have either 5 or 0 at their unit’s place. 1546, 243, 11, 19647 are not divisible by 5 as they do not satisfy the above condition.

https://sscstudy.com/

https://sscstudy.com/

NUMERICAL ABILITY

Divisibility by 6 A number is divisible by 6, when it is divisible by 2 as well as 3. This rule emanates from the fact that 2 and 3 are the two factors or sub-multiples of 6. Hence, both the conditions i.e., for divisibility by 2 as well as divisibility by 3 must be satisfied simultaneously. Alternatively, for a number to be divisible by 6 it must have either zero or an even digit at the unit’s place and also simultaneously the sum of its digits must be divisible by 3. 72 (i) Its unit’s digit i.e., 2 is an even number and hence it is divisible by 2. (ii) Also, the sum of its digits equals 9 ( = 7 + 2), which is divisible by 3. (iii) Therefore, 72 is divisible by 6. 2247 (i) Its unit’s digit is an odd number and hence it is not divisible by 2. (ii) The sum of its digits equals 15 ( = 2 + 2 + 4 + 7), which is divisible by 3. (iii) We see that both the conditions i.e., divisibility by 2 and divisibility by 3 are not satisfied simultaneously. Therefore, 2247 is not divisible by 6, even though the number is divisibly by 3. Divisibility by 7 A number is divisible by 7, when the difference between twice the digit at ones and the number formed by other digits is either zero or a multiple of 7. e.g., 658 is divisible by 7 because 65 − 2 × 8 = 65 − 16 = 49 As 49 is divisible by 7, the number 658 is also divisible by 7. Divisibility by 8 A number is divisible by 8, when the number formed by its three extreme right digits is divisible by 8 or when these last three digits are zeros. 3648 Since, 648 is divisible by 8, 3648 is also divisible by 8. 11600 Since, 600 is divisible by 8, 11600 is also divisibly by 8. 216000 As, the last three digits of the given number are zeros, 216000 is divisible by 8. 21700 Since, 700 is not divisible by 8, 21700 is also not divisible by 8. Divisibility by 9 A number is divisible by 9, when the sum of its digits is divisible by 9. 39537 Sum of the digits = 3 + 9 + 5 + 3 + 7 = 27. Since, 27 is divisible by 9, 39537 is also divisible by 9. Divisibility by 10 A number is divisible by 10, when it has zero at its unit’s place. e.g., The numbers 150, 7250, 1900, 35450 etc., are divisible by 10. 8564, 7509, 29005 etc., are not divisible by 10.

5

Divisibility by 11 A number is divisible by 11 when the difference between when the sum of the digits at odd places and the sum of the digits at even places is either 0 or divisible by 11. 9851833 Difference = Sum of digits at odd places − Sum of digits at even places = ( 9 + 5 + 8 + 4) − ( 8 + 1 + 3 + 3) = 26 − 15 = 11 So, the given number is divisible by 11. 602613 Difference = ( 6 + 2 + 1) − ( 0 + 6 + 3) = 9 − 9 = 0 So, the given number is divisible by 11. Divisibility by 12 A number is divisible by 12, when it is divisible by both the numbers 3 and 4. This is so because 4 and 3 are the two factors or sub-multiples of 12. Therefore, conditions of divisibility by 4 as well as 3 must be satisfied simultaneously. Alternatively, for a number to be divisible by 12, its last two digits must be either zero or the number formed by them, must be divisible by 4 and at the same time the sum of all the digits of the number must be divisible by 3. 9612 (i) 12 is divisible by 4. So, the number is divisible by 4. (ii) 9 + 6 + 1 + 2 = 18, which is divisible by 3. Hence, 9612 is divisible by 3. (iii) Therefore, 9612 is divisible by 12. 7623 (i) Its last two digits are neither zeros nor is 23 divisible by 4. So, the number is not divisible by 4. (ii) 7 + 6 + 2 + 3 = 18, which is divisible 3. So, the number is divisible by 3. (iii) We find that conditions for divisibility by 4 and 3 are not satisfied simultaneously. Therefore, 7623 is not divisible by 12 even though it is divisibly by 3.

To Find the Unit’s Place Digit of a Given Exponential In case of 0, 1, 5, 6 The unit’s place digit is 0, 1, 5, 6 respectively. In case of 4, 9 (a) if power is odd → The unit’s place digit is 4 and 9 respectively. (b) if power is even → The unit’s place digit is 6 and 1 respectively. In case of 2, 3, 7, 8 See the following example. To find the unit’s place digit of (134647)553 . Step I 553 ÷ 4 gives 1 as remainder this remainder is taken as new power. Step II (134647)553 = (134647)1 = 71 = 7. ∴ The unit’s place digit is 7. On dividing it, the remainder obtained is zero, take 4 as new power instead of zero. e.g., (134647)552 Sol. (134647)552 = (134647)0 = 70 = 74 = 2401 ∴ The unit’s place digit is 1.

https://sscstudy.com/

https://sscstudy.com/

6

CUET (UG) Section III : General Test

Important Tips/Formulae n ( n + 1) 2 Sum of the first n even natural numbers = n( n + 1) Sum of the first n odd natural numbers = n2 n( n + 1)( 2n + 1) Sum of the squares of first n natural numbers = 6 2 n ( n 1 ) +  Sum of the cubes of first n natural numbers =    2  nth term of the series a, a + d, a + 2d, a + 3d , ... = a + ( n − 1) d Where a is the first term and d is common difference. Sum to the n terms of the series a, a + d, a + 2d ... n n = [2a + ( n − 1) d ] or = [ a + l ] 2 2 Where l = last term Dividend = (Divisor × Quotient) + Remainder Sum of the first n natural numbers =

Solved Examples 1. Find the unit digit in the expression

55725 + 735810 + 22853. (a) 5

(b) 6

(c) 7

(d) 4

725

Sol. (b) The unit digit of 55

is 5. is 9 (Remainder is 2 ∴ (3)2 = 9) 73 22853 is 2 ∴ The unit a digit of the expression = 5 + 9 + 2 = 16 i.e., 6 5810

2. How many integers lie between 300 and 1000,

which are exactly divisible by 13? (a) 50

(b) 54

(c) 53

(d) 52

Sol. (c) The numbers lying between 300 and 1000, which are exactly divisible by 13, are

312, 325, 338, ..., 988 Here, a = 312, cd = 13 Let n be the numbers of terms. ∴ tn = a + (n − 1) d, 988 = 312 + (n − 1) × 13 676 ⇒ = n −1 13 ⇒ n = 52 + 1 = 53

Alternate 1000 300 − 13 13 1 12 = 76 − 23 13 13 = 76 − 23 = 53

Required number =

3. Find the sum of all natural numbers from 75 to 97. (a) 2008 (c) 1895

(b) 1985 (d) 1978 74 (74 + 1) = 37 × 75 = 2775 Sol. (d) 1 + 2 + 3 + ... + 74 = 2 97 (97 + 1) 97 × 98 and 1 + 2 + 3 + K + 97 = = = 4753 2 2 ∴ 75 + 76 + 77 + 78 + K + 97 = 4753 − 2775 = 1978

4. A number, when divided by 114, leaves remainder

21. If the same number is divided by 19, then the remainder will be (a) 1 (c) 7

(b) 2 (d) 17

Sol. (b) Let given number = (14K + 2) = (19 × 6K ) + 19 + 2 = 19 (6K + 1) + 2 Hence, the required remainder is 2.

Practice Questions 1. A number of three digits when divided by 2, 5, 9,

6. The sum of the squares of 3 consecutive positive

11 leaves remainder 1 in each case. The number is

numbers is 365. The sum of the numbers is

(a) 981

(a) 30

(b) 983

(c) 991

(d) 997

2. The sum of a two digit number and the number

(b) 33

(c) 36

(d) 45

7. In a division sum, the divisor is 10 times the

obtained by reversing its digits is a square number. How many such numbers are there?

quotient and 5 times the remainder. If the remainder is 46, then the dividend

(a) 5

(a) 4236

(b) 6

(c) 7

(d) 8

3. When a number is divided by 24, the remainder is

(b) 4306

is half of 78, then

(a) 3

(a) 6

(c) 6

(d) 8

4. The unit’s digit in the product 7 71 × 663 × 3 65 is (a) 1

(b) 2

(c) 3

5. The units digit of the expression

256251 + 36528 + 7354 is (a) 6 (c) 4

(b) 5 (d) 0

(d) 4

(d) 5336

8. If x is a number midway between 10 and 16 and y

16. The remainder when the same number is divided by 12 is (b) 4

(c) 4336

y is equal to x

(b) 5

(c) 4

(d) 3

9. The difference between the largest 3 digit number

and the smallest 2 digit number is (a) 989

(b) 899

(c) 998

(d) 988

10. What largest number of four digits is exactly

divisible by 88? (a) 9988 (c) 9768

https://sscstudy.com/

(b) 8888 (d) 9944

https://sscstudy.com/

7

NUMERICAL ABILITY

11. In 337 337 , the unit digit is occupied by (a) 1

(b) 3

(c) 7

x ( x + 1) (2x + 1) , then 6 2 2 2 2 1 + 3 + 5 + K + 19 is equal to

23. If 12 + 22 + 32 + ... + x2 =

(d) 9

12. When a number is divided by 121, the remainder is

25. If the same number is divided by 11, the remainder will be (a) 3

(b) 4

(c) 6

(a) 1330

(c) 10

(a) 9899

1  1   1 −  1 −   4  3 1 − 1  1 − 1  , is     99   100

(a)

2 99

(b)

1 25

(c)

1 50

(d)

(a) (b) (c) (d)

(c) 12

(a) 5

(c) 20

(a) 775

(b) 468

(c) 921

(a) 2

(c) 27

(d) 30

(a) 11

(c) 0

(c) 6336

125

× 553

3703

828

× 4532

(a) 4 (c) 0

(d) 9

2 of the other part, then the longer part 3 (in m) is (a) 34

(b) 56

2 3

(c) 85

(d) 51

be (a) 1

(d) 6363

(b) 3

(c) 7

(d) 9

32. The sum of all natural numbers between 100 and

200 which are multiples of 3 is

is

(a) 5000

(b) 2 (d) 5

(b) 4950

(c) 4980

(d) 4900

33. A number divided by 899 gives a remainder of 63.

If the number is divided by 29, the remainder will be

22. Find the number of divisors of 1420. (a) 14 (c) 13

(c) 3

31. The unit’s place digit in the product (3127)173 will

21. The unit digit of the expression 813

(d) 39

part is

divisible by 9 is (b) 6633

(c) 42

(b) 5

(d) 12

20. The sum of all integers between 200 and 400 (a) 3366

(b) 32

30. An 85 m long rod is divided into two parts. If one

candidate took 12 as divisor, instead of 21. The quotient obtained by him was 35. The correct quotient is (b) 20

(d) 6

interchange places and the new number is added to the original number, the resulting number will be divisible by

19. In a question on division with zero remainder, a

(a) 13

(c) 7

29. A number consists of two digits. If the digits

(d) 875

each divisible by 3 is 72. What is the largest among them? (b) 24

(b) 3

as many rupees as the total collection was ` 3042. The number of members present in the party was

18. The sum of three consecutive natural numbers

(a) 21

(d) 2

28. Each member of a picnic party contributed twice

(d) 10

17. The sum of odd numbers from 10 to 60 is

(c) 4

remainder is 39. On dividing the same number by 17. What will be the remainder? (a) 5

(d) 30

(b) 3

27. On dividing a certain number by 357, the

to make it exactly divisible by 45? (b) 80

Last digit is 4 Last digit is 8 Last digit is 2 Last two digits are zero

(268 × 539 × 826 × 102)

1 100

16. Which of the least number should be added at 1000 (a) 35

(d) 9889

26. Find the unit digit in the product of

quotient and 3 times the remainder. If the remainder is 4, the dividend is (b) 40

(c) 9989

where n is a positive integer?

1  ... 5

15. In a division problem, the divison is 4 times the

(a) 36

(b) 8999

25. What can be said about the expansion of 212 n − 64 n,

(d) 12

14. The simplified value of 1 − 

(d) 2500

and the smallest 3 digit number is

are 14 more than twice the number of heads. The number of cows is (b) 7

(c) 2485

24. The difference between the largest 4 digit number

(d) 25

13. In a group of cows and hens, the number of legs

(a) 5

(b) 2100

(b) 15 (d) 12

(a) 2

(b) 5

(c) 13

(d) 28

ANSWERS 1. 11. 21. 31.

(c) (c) (c) (c)

2. 12. 22. 32.

(d) (a) (d) (b)

3. 13. 23. 33.

(b) (b) (a) (b)

4. (d) 14. (c) 24. (a)

5. (d) 15. (b) 25. (d)

6. (b) 16. (a) 26. (c)

https://sscstudy.com/

7. (d) 17. (d) 27. (a)

8. (d) 18. (c) 28. (d)

9. (a) 19. (b) 29. (a)

10. (d) 20. (b) 30. (d)

https://sscstudy.com/

8

CUET (UG) Section III : General Test

Hints & Solutions 1. LCM of 2, 5, 9, 11 = 990 Required number = 990 + 1 = 991 2. Let the two digit number be 10x + y. By given condition, 10x + y + 10 y + x = z 2 (Square number) 11(x + y) = z 2 z2 x+ y = 11 Putting the all values of x and y, which sum is 11, we find 8 such numbers. 3. When 16 is divided by 12 the remainder is 4. So, the required remainder is 4. 4. Unit digit of [771 × 663 × 365 ] = Unit digit of [74×17+ 3 × 6 × 34×16+ 1 ] = Unit digit of [73 × 6 × 31 ] = Unit digit of [343 × 6 × 3] [Q 73 = 343] = Unit digit of [3 × 6 × 3] = Unit digit of [54] = 4 5. Unit digit of (256251 + 36528 + 7354 ) = Unit digit of [5 + 6 + (3)54 ] = Unit digit of [5 + 6 + (34 )13 × 32] = Unit digit of [5 + 6 + 1 × 9] = Unit digit of [20] = 0 6. Let the three consecutive numbers be x, x + 1 and x + 2 respectively. According to question, x2 + (x + 1)2 + (x + 2)2 = 365 x2 + x2 + 1 + 2x + x2 + 4 + 4x = 365 3x2 + 6x + 5 − 365 = 0 x2 + 2x − 120 = 0 On solving, x = 10 ∴ Sum of numbers = 10 + 11 + 12 = 33 7. Let remainder = 2x = 46 46 x= = 23 2 ∴ Quotient = 23, Remainder = 46, Divisor = 230 QDividend = Divisor × Remainde r + Quotient

= 230 × 23 + 46 = 5336 8. Value of x between 10 and 16 will be 11, 12, 13, 14, and 15. According to question, 78 y= = 39 2 y 39 Then, = x 13 (Let x = 13 because 39 divided by 13) y =3 x 9. Largest three digit’s number = 999 Smallest two digit’s number = 10 Difference = 999 − 10 = 989 10. Largest number of four digits = 9999 88 ) 9999 ( 113 88 119 88 319 264 55 ∴ Required number = 9999 − 55 = 9944 11. The given number is 337337. The unit’s place digit is 7, so by applying the method, we divide the power by 4 i.e., 4 ) 37 ( 84 32 17 16 1 The remainder is 1. So, the new power is 1. ∴ (337)337 = (337)1 Hence, the unit’s place digit is 7. 12. By Shortcut : 25 ÷ 11 gives 3 as remainder 13. Let there be x cows and y hens. Then, (4x + 2 y) − 14 = 2 (x + y) 4x + 2 y −14 = 2x − 2 y 4x − 2x = 14 2x = 14 14 =7 x= 2

https://sscstudy.com/

1  1  1 1   14. 1 −  1 −  1 −  K 1 −    3  4  5 99 1   1 −   100 2 3 4 5 98 99 = × × × × ... × × 3 4 5 6 99 100 2 1 = = 100 50 15. Let divison = d , then d = 4 q = 3r Since r = 4, then d = 3 × 4 = 12 4 ×3 and q = =3 4 ∴ N = dq + r = 12 × 3 + 4 = 40 16. 45)1000(22 90 100 90 10 So, the required number = 45 − 10 = 35 17. Sum to first n odd numbers  59 + 1 =   2 

2

{Last odd number = 59} = (30)2 = 900 and sum to odd numbers from 1 to 10 2 2  9 + 1  10 2 =  =   = (5) = 25  2  2 ∴ The required odd numbers = 900 − 25 = 875 18. Let the three consecutive number divisible by 3 be x, (x + 3), (x + 6), then x + (x + 3) + (x + 6) = 72 3x + 9 = 72 3x = 63 x = 21 So, the largest number must be x + 6 = 21 + 6 = 27 19. The number is 35 × 12 = 420 Now, correct quotient = 420 ÷ 21 = 20 20. The numbers are 207, 210, 225, ..., 396 Tn = a + (n − 1) d 396 = 207 + (n − 1) × 9 396 − 207 ⇒ n= +1 9

https://sscstudy.com/

NUMERICAL ABILITY



n = 22 n S n = (a + l) 2 22 (207 + 396) = 2 = 6633

21. The unit digit in 125813 = 5 The unit digit in (553)3703 = Divide 3703 by 4 the remainder is 0 = 0 The unit digit in (4532)828 = Divide 828 by 4 the remainder is 0 = 0 Hence, unit digit will be =5 ×0 ×0 =0 22. 1420 = 22 × 51 × 711 ∴ Number of divisors = (2 + 1) (1 + 1) (1 + 1) = 12 23. 12 + 22 + 32 + ...+ 192 19 × 20 × 39 = = 2470 6 22 + 42 + K + 182 = 1140 = 22(12 + 22 + K + 92) 4 × 9 × 10 × 19 = 1140 = 6 2 2 2 ∴ 1 + 3 + 5 + K + 192 = 2470 − 1140 = 1330 24. Largest four digit’s number = 9999 Smallest three digit’s number = 100 Difference = 9999 − 100 = 9899 25. 212n − 64n = (212)n − (64 )n = (4096)n − (1296)n

= (4096 − 1296) [(4096)n − 1 + (4096)n − 2(1296) + … + (1296)n − 1 ] = 2800 (k) Hence, last two digits are always be zero. 26. Product of unit digits of the number = 8 × 9 × 6 × 2 = 864 ∴ Required digit = 4 27. Let the given number be (357 k + 39). Then, (357k + 39) = (17 × 21 k) + (17 × 2) + 5 = 17 × (21k + 2) + 5 ∴ Required remainder = 5 28. Let the number of members of a picnic = x ∴ Contribution of each member = 2x ∴ 2x × x = 3042 3042 ⇒ x2 = = 1521 2 ∴ x = 39 29. Let the unit’s place digit = x and ten’s place digit = y number = 10 y + x ∴ and new number = 10x + y Sum of number = 10 y + x + 10x + y = 11 (x + y) This shows that the sum of numbers is divisible by 11. 30. Let the length of first part = x m ∴ The length of another part = (85 − x) m

https://sscstudy.com/

9

According to the question, 2 x = (85 − x) 3 ⇒ 3x = 170 − 2x 170 = 34 x= ⇒ 5 ∴ The length of first part = 34 m and the length of second part = 85 − 34 = 51 m 31. Unit’s place digit in (3127)173 = unit’s place digit in (7)173 = 7172+ 1 = (7)1 = unit’s place digit in 1 × 7 = 7 32. Multiple of 3 between 100 and 200 are 102, 105, ......, 198. Let the number of numbers is n. ∴ 198 = 102 + (n − 1) ⋅ 3 198 − 102 ⇒ (n − 1) = 3 96 = = 32 3 ⇒ n = 33 33 ∴ Required sum = (102 + 198) 2 33 = × 300 2 = 4950 33. If a number x is divided by 899 gives a remainder 63 and quotient y. ∴ x = y × 899 + 63 ⇒ x = (31 y + 2) × 29 + 5 It is clear from above that if x is divided by 29, then the remainder is 5.

https://sscstudy.com/

10

CUET (UG) Section III : General Test

CHAPTER 02

HCF and LCM HCF of any given set of number is the greatest factor common to them and LCM of two or more numbers is the smallest number which is common multiple of the given number.

HCF (Highest Common Factor)

LCM (Least Common Multiple)

A common factor of two or more numbers is a number which divides each of them exactly. e.g., 2 is a common factor of 18 and 24. It is also known as GCD (Greatest Common Divisor) or GCM (Greatest Common Multiple)

The least number which is exactly divisible by each one of the given numbers is called their LCM.

Methods of Finding HCF Factorisation Method Break the given numbers into prime factors and then find the product of common prime factors with least powers gives the HCF.

Methods of Finding LCM Factorisation Method Break the given numbers into their prime factors and then find the product of highest powers of all the factors, which occur in the given number and this product is the required LCM. e.g., Find the LCM of 24, 45 and 60. 45 = 3 × 3 × 5 = 32 × 5 60 = 2 × 2 × 3 × 5 = 22 × 3 × 5 LCM = 23 × 32 × 5 = 8 × 9 × 5 = 360 The product of the divisors and the undivided numbers is the required LCM.

e.g., Find the HCF of 24, 45 and 60. Sol.

24 = 2 × 2 × 2 × 3 45 = 3 × 3 × 5 60 = 2 × 2 × 3 × 5 HCF of 24, 45, 60 = 3

Division Method Divide the larger number by smaller number and then divide the divisor by the remainder and repeat this process till the remainder becomes zero and in the last divisor is the require HCF. e.g., Find the HCF of 13281 and 15844. Sol.

24 = 2 × 2 × 2 × 3 = 23 × 3

Sol.

13281 ) 15844 ( 1 13281 2563 ) 13281 ( 5 12815 466 ) 2563 ( 5 2330 233 ) 466 ( 2 466 ×

Division Method Write down the given numbers in a line, separating them by commas and then divide by anyone of the prime numbers which exactly divides atleast two of the given numbers. Now, write down the quotients and the undivided numbers in the line below the first and repeat this process untill you get a line of numbers which are prime to one-another. In the last the product of all the divisors and the numbers in the last line is the required LCM. e.g., Find the LCM of 24, 45 and 60. Sol.

2 24, 45, 60 2 12, 45, 30

HCF = 233 ∴ If the successive divisions continue until remainder of 1 is obtained this means that the original numbers have no common divisor.

https://sscstudy.com/

3

6, 45, 15

5

2, 15, 5

2

2, 3, 1

3

1, 3, 1 1, 1, 1

LCM = 2 × 2 × 2 × 3 × 3 × 5 = 360

https://sscstudy.com/

NUMERICAL ABILITY

Now, LCM

Important Tips/Formulae Product of two numbers = (their HCF) × (their LCM) Co-prime Two numbers are coprime if their HCF is 1. HCF and LCM of fractions HCF of numerators (i) HCF = LCM of denominators LCM of numerators (ii) LCM = HCF of denominators The greatest number that will divide x, y, z leaving remainders a, b and c respectively is given by HCF of ( x − a ), ( y − b ) and ( z − c). The least number which when divided by x, y and z leaves the same remainder R in each case is given by LCM of [( x, y, z) + R]. The greatest number that will divide x, y and z leaving the same remainder in each case is given by HCF of| x − y |, | y − z| and | z − x |.

1. Among how many children may 429 mangoes and (b) 123 (d) 160

Sol. (a) HCF (429, 715) 429 )715( 1 429 286 )429( 1 286 143 )286( 2 286 × ∴The number of children must be 143.

4 38 5 (c) 132

and

3, 15, 10

5

1, 5, 10

2

1, 1, 2 1, 1, 1

LCM = 2 × 2 × 3 × 5 × 2 = 120 HCF = 35 = 5 × 7 49 = 7 × 7 21 = 3 × 7 HCF = 7 ∴ 7 Required HCF = 120

3. Two numbers are in the ratio 5 : 7. If their HCF is

4, find the numbers. (b) 20, 28 (d) 30, 42

Sol. (b) Let the required numbers be 5x and 7x.

24 and 36 leaves the remainders 7, 25 and 27 respectively. (a) 140 (c) 130

(b) 125 (d) 135

Sol. (d) Here 16 − 7, 24 − 15, 36 − 27 = 9, 9, 9 Required number = LCM of (16, 24, 36) − 9 = 144 − 9 = 135

7 120 7 (d) 142

2 16, 24, 36

(b)

Sol. (b) HCF of

6, 15, 20

3

4. Find the least number which when divided by 16,

35 49 21 and . , 12 30 40

(a)

2

Then, their HCF is x. So, x=4 ∴ The numbers are (5 × 4) and (7 × 4) i.e. 20 and 28.

also 715 oranges be equally divided ?

2. Find the HCF of

2 12, 30, 40

(a) 15, 21 (c) 25, 35

Solved Examples (a) 143 (c) 152

11

35 49 , 12 30

21 HCF of 35, 49 and 21 = 40 LCM of 12, 30 and 40

2

8, 12, 18

3

4, 6, 9

2

4, 2, 3

2

2, 1, 3

3

1, 1, 3 1, 1, 1

Practice Questions 1. HCF of the reciprocals of the fractions

3 9 15 , and 4 10 16

is (a)

3 15

(b)

3 80

(c)

1 15

(d)

2 45

2. The LCM of two numbers is 48. The numbers are

in the ratio 2 : 3. The sum of the numbers is (a) 28

(b) 32

(c) 40

(d) 64

3. Which is the smallest number of five digits which

is divided by 41? (a) 10045 (c) 10041

(b) 10004 (d) 10025

4. Which of the following number has the highest

divisor? (a) 99 (c) 176

https://sscstudy.com/

(b) 101 (d) 182

https://sscstudy.com/

12

CUET (UG) Section III : General Test

5. In a morning walk, three persons step off together.

Their steps measure 80 cm, 85 cm and 90 cm respectively. What is the minimum distance each should walk so that all can cover the same distance in complete steps? (a) 12220 cm (c) 12240 cm

(b) 12230 cm (d) 12250 cm

15. If the LCM of three numbers is 9570, then their

HCF will be (a) 11 (c) 19

16. The product of the LCM and HCF of two numbers

is 24. The difference of the two numbers is 2. Find the numbers.

6. Mayank, Manoj and Ankit begin to jog around a

circular stadium. They complete their revolutions in 425, 565 and 635, respectively. After how many seconds will they be together at the starting point? (a) 520 s (c) 380 s

(b) 504 s (d) 480 s

(a) 8 and 6 (c) 12 and 4 (a) 314 (c) 35 (a) 4−81 (c) 47

(a)

5 36

(b)

10. The LCM of 1 54 20 (c) 3

(a) 80 paise (c) 40 paise

(b) 5490 (d) 5095

5 10 25 is , , 6 18 36 25 6

(c)

25 36

(d)

4641 respectively. If one of the numbers lies between 200 and 300, the two numbers are

5 18

(a) 273, 357 (c) 215, 314

237 to leave the same remainder in each case.

10 27 (d) None of these (b)

(b) 14 : 3

(a) 21 (c) 35

(c) 12 : 2

intervals of 3, 5, 7, 8 and 10 s. Find after what interval they will again toll together. How many times does they toll together in one hour?

(d) 12 : 1

3780 respectively. If one of them is 540, then the second one is (b) 126 (d) 112

(a) 14 min, 3 times (c) 14 min, 4 times

the ceiling of a room 15 m 17 cm long and 9 m 2 cm broad is (b) 814

(c) 902

(d) 738

(b) 12 min, 4 times (d) 12 min, 3 times

23. The LCM of (16 − x2 ) and ( x2 + x − 6) is (a) (x − 3)(x + 3) (4 − x2 ) (b) 4 (4 − x2 ) (x + 3) (c) (4 − x2 ) (x − 3) (d) (16 − x2 ) (x − 2) (x + 3)

13. The least number of square tiles required to pave

(a) 656

(b) 30 (d) 40

22. Five bells first begin to toll together and then at

12. The HCF and LCM of two numbers are 18 and

(a) 142 (c) 118

(b) 210, 340 (d) 210, 252

21. Find the largest number which divides 62, 132 and

11. Find the ratio between LCM and HCF of 5, 15 and 20. (a) 8 : 1

(b) 60 paise (d) 20 paise

20. The HCF and LCM of two numbers are 21 and

1 5 2 4 is , , , 3 6 9 27

(a)

(b) 45 (d) 412

19. HCF of ` 1.20, ` 3.40, ` 4.80

8. The LCM of 22, 54, 108, 135 is

9. The HCF of

(b) 38 (d) 39

18. Find out of LCM of 45 , 4 −18 , 412 and 4 7

(b) 56 (d) 64

(a) 5940 (c) 5405

(b) 8 and 10 (d) 6 and 4

17. Find out the HCF of 3 8 , 35 , 3 9 and 314

7. The HCF of 1056, 1584, 2178 is (a) 66 (c) 62

(b) 12 (d) 21

24. The LCM of two numbers is ( a + b) and their HCF is

P ( a − b). If one of the numbers is P, then the other number is

14. What is the largest number which can divide 1356,

Pa b

1868 and 2764 leaving 12 as remainder in each case ?

(a)

(a) 64 (c) 156

(c) a 2 − b2

(b) 124 (d) 260

(b) Pab (d)

(a + b) P (a − b)

ANSWERS 1. (d) 11. (d) 21. (c)

2. (c) 12. (b) 22. (c)

3. (b) 13. (b) 23. (d)

4. (c) 14. (a) 24. (c)

5. (c) 15. (a)

6. (b) 16. (d)

https://sscstudy.com/

7. (a) 17. (c)

8. (a) 18. (d)

9. (a) 19. (d)

10. (c) 20. (a)

https://sscstudy.com/

Hints & Solutions 1. Required HCF 4 10 16 and = HCF of , 3 9 15 HCF of (4,10 and 16) 2 = = LCM of (3, 9 and 15) 45 2. Let the two numbers be 2x, 3x respectively. LCM of numbers = 48 2 × 3x = 48 48 x= =8 6 ∴Sum of two numbers = 2 ×8 + 3 ×8 = 16 + 24 = 40 3. Smallest number of 5 digits = 10000 When 10000 is divided by 41, remainder = 37 Hence, required number = 10000 + (41 − 37) = 10004 4.

99 = 3 × 3 × 11 101 = 101 176 = 2 × 2 × 2 × 2 × 11 182 = 2 × 7 × 13 Hence, 176 has the highest divisors.

5. Minimum distance each should walk = LCM of 80 cm, 85 cm, 90 cm 5 2

80, 85, 90 16, 17, 18 8, 17, 9

Minimum distance = 5 × 2 × 8 × 17 × 9 = 12240 cm 6. Required time = LCM of 42, 56 and 635 2 42, 56, 63 3 21, 28, 63 3

7, 28, 21

7

7, 28, 7

4

1, 4, 1 1, 1, 1

∴ Required time = 2 × 3 × 3 × 7 × 4 = 504 s 8. The LCM of 22, 54, 108, 135 2

22, 54, 108, 135

3

11, 27, 54, 135

9

11, 9, 11, 1,

18, 45 2,

5

= 2 × 3 × 9 × 11 × 5 × 2 = 5940

5 10 25 , , 6 18 36 HCF of 5, 10, 25 5 = = LCM of 6, 18, 36 36 1 5 2 4 10. The LCM of , , , 3 6 9 27 LCM of 1, 5, 2, 4 = HCF of 3, 6, 9, 27 20 = 3 9. HCF of

11. LCM of 5, 15, 20 = 60 HCF of 5, 15, 20 = 5 60 The required ratio = = 12 : 1 5 12. Product of 1st × 2nd number = HCF × LCM ⇒ 540 × 2nd number = 18 × 3780 18 × 3780 = 126 ⇒ 2nd number= 540 13. Side of the each square tiles = HCF of 1517 cm and 902 cm = 41 cm Required numbers of tiles 1517 × 902 = = 814 41 × 41 14. Required number = HCF of (1356 − 12), (1868 − 12), (2764 − 12) = HCF of (1344, 1856, 2752) 1856)2752(1 1856 896 )1856(2 1792 64 )896(14 64 256 256 × HCF of 2752 and 1856 is 64. Now, HCF of 64 and LCM 1344 64)1344(21 128 64 64 × HCF = 64 15. We know that LCM of the given numbers is always divisible by their HCF. So, 11 is the HCF. 16. Let the numbers be x and y. Then, x − y = 2

https://sscstudy.com/

⇒ y= x−2 LCM × HCF = x × y = 24 ⇒ x (x − 2) = 24 ⇒ x2 − 2x − 24 = 0 ⇒ (x − 6) (x + 4) = 0 ⇒ x=6 The numbers are 6 and 4 respectively. 17. HCF = Highest common factor = 35 So, 35 the highest common factor in 38 , 35 , 39 , 314. 18. LCM − Least Common Factor So, 412 is least common factor in 45 , 4−81, 412 and 4. 19. ` 1.20 = 120 paise ` 3.40 = 340 paise ` 4.80 = 480 paise So, HCF of 120, 340, 480 = 20 × 6, 20 × 17, 20 × 24 HCF = 20 paise (highest common factor) 20. Since, the HCF of two numbers is 21, hence let the numbers be 21x and 21 y respectively. We know that product of two numbers = HCF × LCM 21x × 21 y = 21 × 4641 ∴ x y = 221 = (13 × 17) because x and y are co-prime. Therefore, the numbers are (21 × 13 , 21 × 17) ⇒ (273 , 357) 21. Required number = HCF of (132 − 62), (237 − 132) and (237 − 62) = HCF of 70, 105 and 175 = 35 22. Required time interval = LCM of 3, 5, 7, 8 and 10 = 840 s = 14 min Number of times they will toll together in one hour 60 = = 4 times. 14 (ignoring the fraction part) 23. (16 − x2) = (4 − x) (4 + x) (x2 + x − 6) = (x + 3) (x − 2) LCM of (16 − x2) and (x2 + x − 6) = (16 − x2) (x − 2) (x + 3)

https://sscstudy.com/

14

CUET (UG) Section III : General Test

CHAPTER 03

Square Root and Cube Root ‘‘Square and Cubes are the basic operations with equal importance as the binary operations. To cater such problems you must have the conceptual clarity regarding the same’’.

Square

The square root of 49 is written as 49.

The square of any number is the number multiplied by itself. e.g., 22, 32, 42, ..., n 2 22 = 2 × 2 = 4

Thus, 49 = 7 × 7 = 7

42 = 4 × 4 = 16 n2 = n × n In 22, 32, 42, ... , n 2, figure 2, 3, 4, ..., n are called the base and at the top of a number tells you to square it. The above statements can be expressed by saying that the square of 2 is 4 or two squared is 4 and so on.

Properties of Square It cannot be a negative number. It cannot have odd number of zeros at its end. It cannot end with 2, 3, 7 or 8.

Square root of a number can be learnt with the help of given below. 81 = 9

100 = 10

121 = 11

144 = 12

169 = 13

196 = 14

225 = 15

256 = 16

289 = 17

324 = 18

361 = 19

400 = 20

441 = 21

484 = 22

529 = 23

576 = 24

525 = 25

676 = 26

729 = 27

784 = 28

841 = 29

900 = 30 and so on .... Unit Digit in Square Number

Perfect Square A natural number is called a perfect square or a square number. If it is the square of some natural number. n = m 2, for some natural number m, then n is said to be a perfect square.

Number

Unit Digit

(...1) 2

…1

(...6) 2

…6

(...2) 2

…4

(...7) 2

…9

(...3) 2

…9

(...8) 2

…4

(...4) 2

…6

(...9) 2

…1

…5

2

…0

(...5)

e.g., 4 is a perfect square of 2.9 is a perfect square of 3.

2

Unit Digit

(...0)

How to Calculate the Square Root?

Note

(i) Squares of even numbers are always even. (ii) Squares of odd numbers are always odd.

There are two methods to calculate the square root.

Prime Factorisation Method

Square Root The square root of a number is that factor of the number which, when multiplied by itself, will give that number. The above statement can be expressed by if a is the square root of ‘b’, then ‘b’ is the square of ‘a’. The square root of a number is indicated by the sign or 2 .

This method has the following steps. Step I Express the given number as the product of prime factors. Step II Keep these factors in pairs. Step III Take the product of these prime factors taking one out of every pair of the same primes. This product gives us the square root of the given number.

https://sscstudy.com/

https://sscstudy.com/

NUMERICAL ABILITY

Step IV Bring down the next period, which is 69. Now, trial divisor is 13 × 2 = 26 and trial dividend is 1869. So, we take 267 as dividend and 7 as quotient. The remainder is 0 now. Step V The process (processes like III and IV) goes on till all the periods (pairs) come to an end and we get remainder as 0 (zero).

e.g. Find the square root of 1089. Sol. Prime factors of 1089 = 11 × 11 × 3 × 3 11 11 3 3

1089 99 9 3 1

Hence, the required square root = 137

1089 = 11 × 11 × 3 × 3 ⇒ Now, taking one number from each pair and multiplying them, we get

e.g. What is the square root of 151321? Sol.

389 15 13 21 9 68 613 544 769 6921 6921 × 3

1089 = 11 × 3 = 33

e.g. Find the square root of 1024. Sol. Prime factors of 1024 = 2× 2× 2× 2× 2× 2× 2× 2× 2× 2 ⇒

1024 =

15

2× 2× 2× 2× 2× 2× 2× 2× 2× 2

Now, taking one number from each pair and multiplying them, we get 1024 = 2 × 2 × 2 × 2 × 2 = 32



Required square root = 389

Note Division method should be applied, when the given number is so large that it is very difficult to find its square root by the prime factorisation method.

2

1024

2

512

Square of Two Digits

2

256

2

128

2

64

2

32

Let AB is 2 digit number, then Step I B2 Step II 2( A × B) Step III A2 = A2 + 2 AB + B2

2

16

2

8

2

4

2

2

e.g. (16)2 Sol. Here, 1 = A and 6 = B. Then, Step I (6)2 = 36 Step II 2(1 × 6) = 12 Step III (1)2 = 1 = 1 + 12 + 36 = 2 + 1 5 + 3 6 = 256

Division Method

e.g. (23) 2

The steps of this method can be easily understood with the help of following examples e.g. Find the square root of 18769.

Sol. Here, 2 = A and 3 = B. Then, Step I (3)2 = 9 Step II 2(2 × 3) = 12 Step III (2)2 = 4 = 4 + 12 + 9 = 5 + 1 2 + 9 = 529

Sol. Step I In the given number, mark off the digits in pairs starting from the unit digit. Each pair and the remaining one digit (if any), is called a period. Step II Now, on subtracting, we get 0 (zero) as remainder. Step III Bring down the next period, i.e., 87. Now, the trial divisor is 1 × 2 = 2 and trial dividend is 87. So, we take 23 as divisor and put 3 as quotient. The remainder is 18 now. 1 23 267

137 1 87 69 1 87 69 1869 1869 ×

e.g. ( 26)2 Sol. Here, 2 = A and 6 = B. Then, Step I (6)2 = 36 Step II 2(2 × 6) = 24 Step III (2)2 = 4 = 4 + 24 + 36 = 6 + 27 + 3 6 = 676

e.g. ( 47)2 Sol. Here, 4 = A and 7 = B. Then, Step I (7)2 = 49 Step II 2(4 × 7) = 56 Step III (4)2 = 16 = 16 + 56 + 49 = 22 + 6 0 + 4 9 = 2209

Square of a Number Ending in 5 Step I Multiply the number formed after deleting 5 at the unit’s place with the number one higher than it.

https://sscstudy.com/

https://sscstudy.com/

16

CUET (UG) Section III : General Test

Step II Annex 25 on the right side of the product and you will get the square of the given number. e.g. (15)2 Sol. Step I Step II

1× 2 = 2 = 25 ⇒ (15)2 = 225

e.g.

2704 is equal to 81

Sol.

2704 = 81

2704 52 = 81 9

e.g. ( 35)2 Sol. Step I Step II

5

3 × 4 = 12 = 25

102

⇒ (35)2 = 1225

Square of Decimal Numbers Step I Find the square of the number ignoring the decimal point. Step II Put the decimal point in such a way that the number of decimal places in the square is twice of that in the original number. Step III The square of a decimal number will lie between the square of its integral part and the square of the number one higher than the integral part. e.g. ( 3.6)2 = ( 36)2 = 1296 = 12.96 e.g. e.g.

( 4.5)2 = ( 45)2 = 2025 = 20.25 (10.5)2 = (105)2 = 11025 = 110.25

Step I Multiply the number formed after leaving 25 by a number 5 suffixed to it. Step II Annex 625 to the right side of the product. e.g. (125)2 = 1 × 15 = 15 hence, (125)2 = 15625 e.g. ( 225)2 = 2 × 25 = 50 hence, ( 225)2 = 50625 e.g. ( 325)2 = 3 × 35 = 105 hence, ( 325)2 = 105625

How to Calculate the Square Root of Decimal Numbers? If in a given decimal number, the number of digits after decimal are not even, then we put a 0 (zero) at the extreme right. Now, we mark off the periods and try to calculate the square root applying the division method mentioned above. e.g. Find the square root of 147.1369. 1 22 241 242 3

Note Sometimes, numerator and denominator are not a complete square. In these types of cases, it is better to convert the given fraction into decimal fraction to find the square root.

Cube The cube of any number is the number multiplied by itself and by itself again. e.g. 23 , 53 , ..., n3 A small figure 3 at the top of given number tells you to cube it. 23 is usually read as ‘two cubed’ or the ‘cube of 2’ and so on. The cube of 5 is 5 × 5 × 5 = 125

Perfect Cube

Square of a Number Ending in 25

Sol.

52 2704 25 204 204 ×

12.13 1 47 . 13 69 1 47 44 313 241 7269 7269 ×

A natural number is said to be a perfect cube, if it is the cube of some natural number. A natural number n is a perfect cube, if n = m3 , where m is a natural number. Numbers such as 1, 8, 27, 64, ... are called perfect cube. Note Cube of a number is the triple product obtained on multiplying the number by itself.

e.g. 93 = 9 × 9 × 9 = 729 113 = 11 × 11 × 11 = 1331 123 = 12 × 12 × 12 = 1728 and so on.... Cube of a number can be learnt through this below 03 = 0 3

13 = 1

23 = 8

3 = 27

4 = 64

53 = 125

63 = 216 93 = 729 123 = 1728

73 = 343 103 = 1000

83 = 512 113 = 1331

133 = 2197

143 = 2744

3

3

15 = 3375

16 = 4096

173 = 4913

183 = 5832

193 = 6859

203 = 8000

213 = 9261

223 = 10648

233 = 12167

3

24 = 13824

3

3

25 = 15625

Cube Root The cube root of a number multiplied by itself and by itself again gives the number. A natural number m is the cube root of a number n if n = m3 .

∴ Required square root = 12.13

How to Calculate the Square Root of a Fraction? To find square root of a fraction, we have to find the square roots of numerators and denominators, separately.

The above statement can be expressed by if m is the cube root of n, then n is the cube of m. e.g., 125 is the cube of 5 and therefore 5 is the cube root of 125.

https://sscstudy.com/

https://sscstudy.com/

17

NUMERICAL ABILITY

2. If 1369 = 37; find the value of

How to Calculate the Cube Root? Prime Factorisation Method

13.69 + 01369 . + 0001369 . + 000001369 .

This method has following steps Step I Express the given number as the product of prime factors. Step II Keep these factors in a group of three. Step III Take the product of these prime factors picking one out of every group (group of three) of the same primes. This product gives us the cube root of given number. e.g., Find the cube root of 9261.

(a) 4.1207 (c) 4.1107

(b) 4.1109 (d) 3.8506

Sol. (c) We have, 1369 1369 + 100 10000 1369 1369 + + 1000000 100000000 37 37 37 37 = + + + 10 100 1000 10000

Given expression =

Sol. Prime factors of 9261 = (3 × 3 × 3) × (7 × 7 × 7) 9261 = 3 3 × 3 × 3 × 7 × 7 × 7 Now, taking one number from each group of these,

= 3.7 + 0.37 + 0.037 + 0.0037 = 4.1107

3

3 3 3 7 7 7

we get

3

9261 3087 1029 343 49 7 1

a = b



to 8958 so that the sum is a perfect square. (a) 65 (c) 69

9261 = 3 × 7 = 21

4. Find 3 1 −

b

a b

(a)

The value of x ±



x ± K∞

e.g., 20 − 20 − 20 − 20 K ∞ is equal to

4 5

Sol. (b)

Sol. (c)

x ⋅ x ⋅ x ⋅ x K ∞ If the root goes upto ∞ in multiplication,

Solved Examples

Sol. (b)

(b) 288 = 128

3 2

(b) 3

(a) 1.8

the answer is x itself.

1. Find the value of

8958 81 858 736 122

5 6

(c)

288 144 12 3 = = = 8 2 128 64

(d) 2

Sol. (c) ∴ ⇒

https://sscstudy.com/

3 4

1 . 21 . (c) 1.5

(d) 2.2

64 121 0.064 + 1.21 = 3 + 1000 100 3 64 121 =3 + 100 1000 4 11 = + = 0. 4 + 1.1 = 1. 5 10 10

(a) 11 (c) 12 1 2

(d)

216 − 91 3 125 = 216 216 5× 5× 5 5 =3 = 6× 6× 6 6

(b) 1.3 3

7 8

6. Find the value of n, if 3 n = 729

288 . 128 (c)

94

91 . 216

5. Evaluate 3 0064 . +

Sol. 20 = 4 × 5. Since, the sign is ( − ), then the required answer is the smaller factor i.e., 4.

2 3

9 9 184 4 188

∴Required number = (95) 2 − 8958 = 9025 − 8958 = 67

Suppose ‘a’ and ‘b’ are consecutive factors of x where b > a. If there sign is (+) in the expression, the answer is b i.e., bigger factor and if there sign is (–), the answer is a i.e., the smaller factor.

(a)

(b) 63 (d) 67

Sol. (d)

Important Tips/Formulae ab =

3. Find the smallest number which should be added

(b) 10 (d) 13 3n = 729 3n = 36 3n = (36 )2 = 312 n = 12

https://sscstudy.com/

18

CUET (UG) Section III : General Test

Practice Questions 16. If a = 0 . 1039, then the value of

1. Evaluate 129 + 216 + 68 + 169 (a) 13

(b) 15

2. The simplified value of (a) 12

(c) 9

4 a2 − 4 a + 1 + 3 a is

(d) 12

112 576 256 is × × 196 12 8

(b) 8

(c) 16

(a) 0.1039

(a) 99967 (c) 0.03

(d) 0.09

5.

(a) 31

(a) 5

(a) 6

(b) 0.02 (d) None of these

(b) 0.084 (d) 0.84

(b) 3

2 3

(b) 3

1 2

(c) 6

(c) 9

(b) 8

(c) 10

(b) 16

(c) 16.6

(d) 11

23.

(d) 3

(c) 13

(d) 18.2

(a) 1000

(c) 3

(d) 6

(b) 2 3

(c) 4

(d) 1

2

p ( p + 3 p + 3) + 1 = ?

(b) 999

(c) 1002

(d) 998

26. By what least number, 3600 be divided to get a (a) 9

(b) 50

(c) 300

(d) 450

27. By what least number, 675 be multiplied to obtain

a number which is a perfect cube?

12 . 25 + 0 . 1225 + 0 . 001225 is

(a) 5

(b) 388.5 (d) 3.885

(b) 6

(c) 7

(d) 8

28. The largest four-digit number which is a perfect

14. If 5 = 2. 24 and 6 = 2. 45, then the value of

is

2 5 + 3 6

cube is (a) 9999 (c) 8000

(b) 9261 (d) None of these 4

29. The value of 2 4 + 3 64 + 28 is

(b) 1.57 (d) 1.75

(a) 12

1296 x , then find the value of x. 15. If = 2.25 x (c) 8

(b) 4

perfect cube?

13. Given that 1225 = 35, then the value of

(b) 9

(b) 1500 (d) 700

25. If p = 999, then (d) 25

(b) 75.69 (d) 210.25

(a) 1.37 (c) 1.73

(d) 5

6860 so that 19 be the cube root of the result from this subtraction? (a) 3

126.15. What is the number ?

(a) 0.3885 (c) 38.85

5

24. What least number should be subtracted from

12. Three-fifth of the square of a certain number is (a) 14.5 (c) 145

(c)

7. 2 (?) 3 , then find the value of (?) = 3 0729 . (2) 3 (a) 5

(d) 12

13 x , then x is equal to = 144 12 (b) 12

(b) 25

(a) 1200 (c) 3600

10. The value of 248 + 52 + 144 is

(a) 10

(d) 11

one of 3, 4, 5, 6, 8 is

4 n = 1024, then the value of n is

(a) 1

(c) 1

22. The least perfect square number divisible by each

1.21 × 0.9 is 8. The value of 1.1 × 0.11

11. If 1 +

(b) 7

(a) 125

(a) 0.048 (c) 0.48

(a) 14

(d) 20

21. Find the value of 5 ⋅ 5 ⋅ 5 ⋅ 5 ⋅ K ⋅ ∞ .

0.441 =? 0625 .

(a) 5

(c) 7

20. The value of 6 + 6 + 6 + 6 + K is

004 . =?

9. If

(b) 16

from 1850 makes it a perfect square. (b) 119 (d) None of these

(a) 2

(d) 99999

19. Find the least number which when subtracted

14161 = ?

(a) 0.002 (c) 0.2

7.

(c) 99856

perfect square is (b) 1900501 (d) 2204701

(a) 129 (c) 121

6.

(b) 99764

18. The least number to be added to 269 to make it a

4. 1499 × 1499 = ? (a) 19501 (c) 2247001

(d) 2.1039

perfect square is

is 3. The value of 0000729 . (b) 0.3

(c) 1.1039

17. The largest number of five digits when it is a

(d) 32

6

(a) 0.027

(b) 0.2078

(d) 6

(b) 16

(c) 18

(d) 24

30. The value of 3 (216) −3 ÷ (343) −2 is (a)

36 156

https://sscstudy.com/

(b)

17 54

(c)

49 216

(d)

49 412

https://sscstudy.com/

ANSWERS 1. (d) 11. (d) 21. (d)

2. (d) 12. (a) 22. (c)

3. (b) 13. (d) 23. (b)

4. (c) 14. (c) 24. (d)

5. (b) 15. (b) 25. (a)

6. (c) 16. (c) 26. (d)

7. (d) 17. (c) 27. (a)

8. (b) 18. (d) 28. (b)

9. (c) 19. (c) 29. (a)

10. (b) 20. (d) 30. (c)

Hints & Solutions = 129 + 216 + 68 + 13 = 129 + 216 + 81 = 129 + 216 + 9 = 129 + 225 = 129 + 15 = 144 = 12 2.

3.

112 576 256 × × 12 8 196 112 24 16 = × × 14 12 8 = 8 × 2 × 2 = 32 6

? = 1499 × 1499 ⇒

12.

? = 2247001



? = 119 ? = 0.04

6. ⇒ 7. ⇒ ⇒ ⇒

13.

? = 0.2 0.441 ?= 0.625 441 ?= 625 21 ?= 25 ? = 0.84

121 × 9 = 9 =3 = 11 × 11 1.1 × 0.11 4n = 1024

⇒ ⇒

n (4) 2



n (4) 2

14.

= 1024

n =5 2 n = 10

248 + 52 + 144 =

248 + 52 + 12

= 1 − 2a + 3a =1 + a = 1 + 0.1039 = 1.1039

3 3 61 1 626

5 2 5 = + 6 3 6 2× 2 = + 3× 2

9 99 99 9 99 61 3899 3756 143

316

∴Required number = 99999 − 143 = 99856

1225 1000000

18. As we know that 162 = 256 and 172 = 289 Hence, 289 − 269 (Given number) = 20 So, 20 is the required number to be added to make given number a perfect square.

5 6

2 + 5 2 + 2.24 = 2.45 6 4.24 = = 1.73 2.45

15.

= (1 − 2a )2 + 3a

12 . 25 + 0 . 1225 + 0 . 001225

2 + 3

4a 2 − 4a + 1 + 3a = (1)2 − 2 × 2a + (2a )2 + 3a

3 2 x = 126 . 15 5 126.15 × 5 x2 = = 42.05 × 5 3 x = 210 . 25 x = 14.5

=

= 45

16.

17. Largest number of 5 digits is 99999.

x = 13 + 12 = 25

1225 1225 = + + 100 10000 35 35 35 = + + 10 100 1000 = 3.5 + 0.35 + 0.035 = 3.885

1.21 × 0.9



10.



? = 14161

5.

9.

Alternate Method

0.000729 = (0.3) = 0.3

⇒ x2 = 2. 25 × 2 × 2 × 3 × 3 ⇒ x2 = 2. 25 × 36 ⇒ x = 2. 25 × 36 ⇒ x = 1.5 × 6 ⇒ x=9

248 + 64

= 248 + 8 = 256 = 16 13 x 11. 1+ = 144 12 x 169 = ⇒ 1+ 144 144 x 169 ⇒ = −1 144 144 25 = 144 25 ⇒ x= × 144 = 25 144

6

6

4.

8.

=

129 + 216 + 68 + 169

1.

1296 x = x 2.25 ⇒ x2 = 2.25 × 1296 ⇒ x2 = 2. 25 × 2 ×2 × 2 × 2 × 3 × 3 × 3 × 3

https://sscstudy.com/

20.

6+ 6+ 6+ 6+K∞ 6 has two factors 2 and 3. Since, here is (+ ve ) sign, so that bigger factor is the answer i.e., 3 is the answer.

21. Clearly, 5 is answer Since, root goes upto ∞ 5 ⋅ 5 ⋅ 5 ⋅ 5 K ∞ = 5 (itself)

https://sscstudy.com/

20

CUET (UG) Section III : General Test

22. By Hit and Trial As we know that 3600 is the only given option which is a perfect square too. And again it is divisible by all the given numbers 3, 4, 5, 6, 8 Hence, 3600 is the required number. 7.2 (?)3 23. 3 = 3 0.729 (2) 3

⇒ 3



⇒ ⇒ ⇒ ⇒

7.2 (?) = 8 729 1000

7.2 × 8 9 ×9 ×9 3 10 × 10 × 10 7.2 × 8 (?)3 = 9 / 10 7.2 × 8 × 10 3 (?) = 9 (?)3 = 64 ? = 3 64 = 4 (?)3 =

24. Q (19)3 = 6859 ∴ Required number = 6860 − 6859 = 1

25.

3

∴ 675 = 52 × 33 To get perfect cube, it must be multiplied by 5.

p ( p2 + 3 p + 3 ) + 1

= { p3 + 3 p2 + 3 p +

1 1}3

1

= [( p + 1)3 ]3 = p + 1 = 999 + 1 = 1000 26. Prime factorisation of 3600 is 2 3600 2 1800 2 900 2 450 3 225 3 75 5 25 5 5 1

∴ 3600 = 23 × 2 × 32 × 52 To get perfect cube, it must be divided by 2 × 32 × 52 = 450 27. Prime factorisation of 675 is 5 5 3 3 3

675 135 27 9 3 1

https://sscstudy.com/

28. By given option, 9261 is clearly a perfect cube. 24 + 3 64 + 4 28

29.

= 22 + 4 + 4 24 × 24 =4 + 4 + 4 = 12 30.

3

(216)−3 ÷ (343)−2 = 3 =3

(343)2 (216)3

=3

(73 )2 (63 )3

=3

73 × 73 6 × 63 × 63 3

=

7×7 6 ×6 ×6

=

49 216

(216)−3 (343)−2

https://sscstudy.com/

NUMERICAL ABILITY

21

CHAPTER 04

Indices and Surds The term indices refer to the power to which a number is raised. Thus, n2 is a number with an index of 2. It prefers the phrase ‘n’ to the power of ‘2’. Term surd is not often used, instead roots are used, occassionally you will be asked to given an answer to surd form, this implies that you should provide the answer in the terms of constant and square roots instead of working out an imprecise decimal approximation.

Indices

Surds

Let a is a real number and m is a positive integer, then a × a × a × a × ... m times = a m where m is known as indices to the base a.

Let a is a rational number and m is a positive integer,

Rules of Indices Let a and b be two real numbers and m and n are two positive integers, then am am × an = am + n = am − n an m am  a ( a m )n = a m × n   = m  b b 1 m m m −m ( ab) = a × b a = m a ( a )0 = 1 e.g. If 4a = 5, 5b = 6, 6c = 7, 7d = 8, then find the value of abcd. ...(i) 4a = 5 ...(ii) 5b = 6 ...(iii) 6c = 7 ...(iv) 7d = 8 On putting value of 5 from Eq. (i) in Eq. (ii), we get (4a )b = 6 [Q (am )n = amn] ⇒ 4ab = 6 ab c Likewise (4 ) = 7 ⇒ 4abc = 7 abc d and (4 ) = 8 ⇒ 4abcd = 8 ⇒ (22 ) abcd = 23 ⇒ 22abcd = 23 Since, there exist equality between them with same base, then their power must be equal. 3 2abcd = 3 ⇒ abcd = = 15 . 2

Sol. We have,

1

1

then if a m be a irrational number, then a m is known as 1 surds of power m. i.e. m = m a = mth root of a. a

Rules of Surds Let a is a rational number and m and n are two positive integers, then a ma m m = ( n a )n = a ab = m a ⋅ m b b mb m n ( m a )n = m a n a = mn a

Similarity If the product of two surds is rational, then each of the two surds is called a rationalizing factor of the other. 1 Rationalizing factor of = a a 1 Rationalizing factor of =am b a± b 1 Rationalizing factor of = am b a± b

Addition and Subtraction of Surds For addition and subtraction of surds, first make the possible factors of the terms, then add or subtract the equivalent surds. e.g. Find the value of 80 + 3 245 − 125. 80 = 16 × 5 = 4 5

Sol.

3 245 = 3 49 × 5 = 21 5 125 =

25 × 5 = 5 5

∴ 80 + 3 245 − 125 = 4 5 + 21 5 − 5 5 = 20 5

https://sscstudy.com/

https://sscstudy.com/

22

CUET (UG) Section III : General Test

Multiplication and Division of Surds

Sol. (c)

For multiplication and division of surds, first make the denominates of the powers, same as we did while arranging them in increasing or decreasing order (or in comparison). Then, multiply or divide as usual.



5 = 5 = 5 = (125) , 6 = 6 = (6) 4 = 41/3 = 42/ 6 = (16)1/ 6 ∴ Required product = (125 × 6 × 16)1/ 6 = (12000)1/ 6 1/ 6 6

1/ 6

1/ 6

3

e.g., Divide 12 × 41/ 3 by 3 2. Sol.

12 × 41/3 4 × 42/ 6 4 × (16)1/ 6 = = 3 × 21/ 2 1 × 2 3/ 6 (8)1/ 6 16 = 4    8

3

−6

(a) − 2

= 4(2)1/ 6 = 4 6 2

3

 3  3 Sol. (c)    

−6

 5  5



 3    5

1

2 x−1

(c) − 1

3 =    5

3 −6

Sol. (b)

, then x is equal to



(d) 1

3 =    5

x = 3, then find the value of x. (b) 64 5+

3

x =3

5+

3

x=9

3

(c) 125

(d) 27

x = 4 ⇒ x = (4)3 = 64 ⇒ x = 64 4

4

3 6 5. Simplify  6 29  ×  3 29  .



2x − 1

(c) 1







(b) 210 (d) 216

(a) 215 (c) 212 4

3 6  6 3  Sol. (d)  29  ×  29     

4

4

4

4

1 1  1 1      =  ( 6 29 )3  ×  (3 29 ) 6  =  (29 )18  ×  (29 )18         

153 × 212 2. The value of the expression will be 352 × 34 (b) 0

3



2x − 1

= 34m

m2 − 2m − 4m = 0 ⇒ m (m − 6) = 0 m = 0, 6

(a) 216

⇒ 2x − 1 = − 3 ⇒ 2x = − 3 + 1 −2 ∴ x= = −1 2

(a) 2

2 − 2m

3m

4. If 5 +

(b) 2

(b) 2, 4 (d) 5, 1

 3m2 m Sol. (a)  2m  = 81  3  2 ⇒ 3 m − 2m = 81m ⇒ ⇒

1/ 6

3 =    5

 5  5

(a) 0, 6 (c) 1, 4



Solved Examples 3 3 1. If    

1

2

Sol. LCM of 2, 6 and 3 = 6 3/ 6

53 × 33 × 72 × 32 34 = 4 =1 52 × 72 × 34 3

=

3. If [3 m ÷ (3 m )2 ] m = 81, then find the value of m.

e.g. Find the product of 5 , 6 6 and 3 4. 1/ 2

153 × 212 (5 × 3)3 × (7 × 3)2 = (5 × 7)2 × 34 352 × 34

4

= (22 )4 × (22 )4 = (22 × 22 )4 = 216

(d) 3

Practice Questions 1. What should come in place of the question mark (?)

(a) 9/2 (c) 7/2

2.

(a) 40 (c) 20

(b) 6 (d) 4

1 1 1 1 1 − + − + 3− 8 8− 7 7− 6 6− 5 5−2 is equal to (a) 5 (c) 1

(b) 3 (d) 0

(a) 21 (c) 3969

(b) 63 (d) None of these

(b) 16 (d) 100

5. If 6440 soldiers were asked to stand in rows to

form a perfect square, it was found that 40 soldiers were left out. What was the number of soldiers in each row? (a) 40 (c) 64

3. In the equation ( x / 21) × ( x / 189) = 1, which of the

following number will replace both the x?

4. The number of boys raised ` 400 for a relief fund,

each boy giving as many 25 paise coin as there were boys. The number of boys was

in the following equation? 317.5 ÷ 313/ 2 × 31− 3 = ( 31) ?

(b) 80 (d) 60

6. The value of 5 + 11 + 19 + 29 + (a) 3 (c) 7

https://sscstudy.com/

(b) 9 (d) 5

49 is

https://sscstudy.com/

23

NUMERICAL ABILITY

7. Each member of picnic party contributed twice as

many rupees as the total number of members and the total collection was ` 3042. The number of members present in the party was (a) 2

(b) 32

(c) 40

(d) 39

1 17. If x = 7 − 4 3, then the value of  x +  is 

(a) 3 3 (c) 14 + 8 3

18. (16) 0.16 × (16) 0. 04 × (2) 0.2 is equal to (a) 1

8. If cube root of 175616 is 56, then the value of 3

is equal to 175616 . + 0175616 . + 0000175616 . 3

3

(a) 0.168 (c) 6.216

3 5

(b) 3

2 5

(b) 2

(b) 8.526



20. The value of 2 + 2 +



(c) 3n + 1

(d) 3n − 16

has 7936 as one of its factors is equal to (b) 246016

(a) 2 (c) 2 +

(c) 61504

(d) 240616

for a picnic. The money contributed by each student was equal to the cube of the total number of students. If the total collected amount was ` 29791, find the total number of students. (b) 27

(c) 31

(d) 34

12. 17 3.5 × 17 7. 3 ÷ 174.2 = 172 , then find the value of (?). (a) 6.5

13. If  2 

(b) 7.2

21. Simplify



2 3

(b) 1.4

(a) 26



(a) 4

3 2

(c) − 49

16. Which is greater (b)

4

3

(c) 1

3

6

4 , 6, 15,

6

6

12

(c) 15

(d) 0

(a) 16

(b) 17.5

9 × 3 × (27) 3 × (81)4 5

3

(c) 8.5

(d) 13

= 27, then n = ?

(b) 2

(c) 1

(d) 0

25. Which of the two is greater 2

200

or 3

(a) 2 3

(b)

?

(b) 2300 (d) Can’t say

26. If A = 5 + 2 6, then the value of

(d) 49

(c) − 3 3

(d) 1. 4

300

= 5x + 2 , then the value of x is

(b) 5

(c) 1

(b) 2

(a) 3200 (c) Both are equal

1 49

(b)

15. If 5 5 × 53 ÷ 5

(a)

(d) 1

1

23. The value of x is257.5 × 52.5 ÷ 125 1.5 = 5x

is equal to

1 49

(a) −

(c) 3

1

( x 3)2 × x4 22. If = x p , then the value of p is x10

(a) 3 (b) 0

1  14.  −   343 

(a) 0.14

24. If

= 2 , then x is equal to

 

1

(6 . 25) 2 × (0 . 0144) 2 + 1 (0 . 027) 3 × (81) 4

n

x

(d) 8.876

1 1  +  is 2+ 2 2 − 2

1

(d) 15.8

2

2

(a) 2

3

(c) 6.6

(c) 8.426

(b) 2 − 2 (d) 2 2

2

11. Each student of class 10 contributed some money

(a) 15

(d) 16

8 + 2 32 − 3 128 + 4 50 is (a) 8.484

10. The least number which is a perfect square and (a) 12.008

(c) 4

19. Given 2 = 1.414, then the value of

(b) 62.16 (d) 6.116

3 ⋅ 9 n + 1 + 9 ⋅ 32 n − 1 is 9. The value of 9 ⋅ 32 n − 6 ⋅ 9 n − 1 (a) 3

x

(b) 8 3 (d) 14

3

(c)

A +

1 A

is

(d) 7

2

 1+ 2   1− 2  27. The value of   +  is  5 + 3  5 − 3

(d) − 6

245?

(a) 5 + 6 (c) 5 − 6

(d) 12 245

(b) 2 5 + 6 (d) 2 5 − 3 6

ANSWERS 1. (b) 11. (c) 21. (d)

2. (a) 12. (c) 22. (d)

3. (b) 13. (d) 23. (d)

4. (a) 14. (d) 24. (a)

5. (b) 15. (a) 25. (a)

6. (a) 16. (b) 26. (a)

7. (d) 17. (d) 27. (c)

8. (c) 18. (b)

9. (a) 19. (a)

10. (b) 20. (a)

Hints & Solutions 317.5 ÷ 31

1.

3

2

× 31−3 = ( 31 )?

⇒ 317.5 ÷ 311.5 × 31−3 = ( 31 )? 317.5 −1.5 −3 = ( 31 )? ? ? ⇒ 313 = (31) 2 ⇒ = 3 ⇒ ? = 6 2



2.

1 = 3− 8 9 1 − ∴ − 8 8 = (3 + 8 ) − (

https://sscstudy.com/

1 9+ 8 × = ( 9 + 8) − 8 9+ 8 1 1 1 1 − + + 6− 5 5 −2 − 7 7− 6 8 + 7 ) + ( 7 + 6 ) − ( 6 + 5 ) + ( 5 + 2)

https://sscstudy.com/

24

CUET (UG) Section III : General Test

=3 + 8 − 8 − 7 + 7 + 6 − 6 − 5 + 5 + 2 =5 x x 3. Q × =1 21 189 ⇒ x2 = 21 × 189 x = 21 × 21 × 3 × 3 = 21 × 3 = 63 4. Let the number of boys = x According to question, 25 2 x = 400 ⇒ x2 = 1600 ⇒ x = 40 100

12. 173.5 × 177.3 ÷ 174.2 = 17? 173.5 + 7.3 ⇒ = 17? 17 4.2 ⇒ 1710. 8 − 4.2 = 17? ⇒ 176. 6 = 17? Since, base are same. ∴ ? = 6.6

⇒ ⇒

∴Number of soldiers in a row = 6400 = 80 5+

6.

11 +

19 +

2

1

22 −

2 3

3 ⋅ 9n + 1 + 9 ⋅ 32n − 1 9 ⋅ 32n − 6 ⋅ 9n − 1 3 ⋅ 32n + 2 + 32 ⋅ 32n − 1 = 2 2n 3 ⋅ 3 − 3 × 32n − 2 × 2 32n + 3 + 32n + 1 32n + 1 [9 + 1] = 2n − 1 2n + 2 2n − 1 − 2 ⋅3 3 3 [27 − 2] 3 32 × 10 18 = = =3 5 25 5 =

11. Total collected amount = ` 29791 ∴Total number of students = 3 29791 = 3 31 × 31 × 31 = 31

2

= (− 343)3

15. 5 5 × 53 ÷ 5



3 2

1 × 52

5



= 5x + 2 × 53

3 − 5 2

1

= 5 + 11 + 5 = 5 + 16 = 5 + 4 = 9 =3

9.

= 2x 2 = 2x ⇒ x = 1

/ = (− 73 )23 = (− 7)2 = 49

= 5 + 11 + 25

3 8. Q 175616 = 56 Now, 3 175.616 + 3 0.175616 + 3 0.000175616 175616 3 175616 =3 + 1000 1000000 175616 +3 1000000000 56 56 56 = + + 10 100 1000 = 5.6 + 0.56 + 0.056 = 6.216

×2 1

1   14.  −   343

= 5 + 11 + 19 + 6

7. Let the number of members = x According to question, 2x2 = 3042 ⇒ x2 = 1521 ⇒ x = 39 × 39 = 39

= 2x ( 2 ) = 2x 2

29 + 7

= 5 + 11 + 19 + 36

= 2x

2× 2

⇒ ⇒

= 5x + 2

3

⇒ 5 × 5 2 × 53 × 5 2 = 5x + 2 1+

⇒ ⇒ ⇒ ∴ 16.

3

5

1 3 +3 + 2 2

= 5x + 2 54 + 2 = 5x + 2 x+ 2 =6 x=4

4 , 3 6 , 6 15 , 12 245 LCM of 3, 3, 6, 12 = 12



6

3

4 = 12 44 = 12 256

3

6 = 12 64 = 12 1296

15 = 12 152 = 12 225

245 = 12 245 = 12 245 Clearly, 3 6 is greater. 12

17.

x= 7 −4 3 1 (Conjugate) =7+4 3 ∴ x 1 ∴ x + = 7 − 4 3 + 7 + 4 3 = 14 x

18. (16)0.16 × (16)0. 04 × (2)0. 2 = (2)0. 64 × (2)0.16 × (2)0. 20 = (2)1. 00 = 2 19.

1

21.

1

(6 . 25) 2 × (0 . 0144) 2 + 1 1

1

(0 . 027)3 × (81) 4

2

2  13. Given,  2   

5. Total number of soldiers = 6440 and number of left soldiers = 40 ∴Number of soldiers in square = 6440 − 40 = 6400

4 + 2 2 + 2 − 2 −2 − 2 2 4 = =2 2 =

8 + 2 32 − 3 128 + 4 50

= 2 2 + 8 2 − 24 2 + 20 2 =6 2 = 6 × 1.414 = 8.484 1 1 20. 2 + 2 + + 2+ 2 2 −2 2− 2 2+ 2 =2 + 2 + + −2 2

https://sscstudy.com/

2 . 5 × 0 . 12 + 1 1 . 3 13 = = 0 .3 × 3 0 .9 9 13 = = 1 .4 9 (x3 )2 × x4 x6 + 4 22. = x p ⇒ 10 = x p 10 x x ⇒ x p = 1 = x0 ⇒ x p = x0 ⇒ p = 0 =

. 23. 257.5 × 525 ÷ 1251.5 = 5x 15 25 ⇒ 5 × 5 . ÷ 54.5 = 5x 1 ⇒ 515 × 2 = 5x 5 ⇒ 5x = 513 ⇒ x = 13 9n × 35 × (27)3 24. = 27 3 × (81)4 (32)n × 35 × (33 )3 ⇒ = 33 3 × (34 )4 32n × 35 × 39 = 33 ⇒ 3 × 316 32n × 314 ⇒ = 33 317

⇒ 32n × 314 = 317 × 33 ⇒ 32n =

320 314

⇒ 32n = 36 ⇒ 2n = 6 ⇒ n = 3 25. In these types of questions, either the base or the exponent is made same. 2300 = (23 )100 = 8100 3200 = (32)100 = 9100 Now, clearly 9100 > 8100 So, 3200 > 2300 26. A = 5 + 2 6 = ( 2 + 3 )2 = 3+ 2 1 = 3− 2 ∴ A 1 So, A + A = 3+ 2+ 3− 2 =2 3 1+ 2 1− 2 27. + 5+ 3 5− 3 5 − 3 + 10 − 6 + 5 − 10 + 3− 6 ( 5 + 3) ( 5 − 3) 2 5 −2 6 = 5− 6 = 2 =

https://sscstudy.com/

NUMERICAL ABILITY

25

CHAPTER 05

Simplification In mathematics, we normally use four mathematical operations + (addition), − (subtraction), × (multiplication) and ÷ (division). Now, we will study about these operations in detail.

Addition

Multiplication

Adding the objects of different sets means finding the total number of objects that are being considered. e.g., ‘‘2 marbles and 3 marbles make 5 marbles,’’ you can write 2 + 3 = 5 on the board or on paper. When you say “2 pencils and 3 pencils make 5 pencils”, you could again write 2 + 3 = 5 and so on.

It is a mathematical concept which indicates that how many times a number is added to itself, e.g., if 2 is added 4 times, then 2 + 2 + 2 + 2 = 8. This can be done by multiplication also i.e., 2 × 4 = 8 In other words, we can say that, the basic idea of multiplication is repeated addition. The number to be repeated is called the multiplicand and the number which indicates how many times the multiplicand is to be repeated is called the multiplier. The sum of the repetition obtained is called the product. e.g., (i) 5 × 3 = 5 + 5 + 5 = 15 (ii) 649 × 3 = 649 + 649 + 649 = 1947

Addition of Fractional Numbers If any two or more than two fractional numbers are given P in the form of , then we can do the sum in the following Q ways. 2P 5P Numerator = P + 2P + 5P 8P P (i) = = + + Q Denominator = Q Q Q Q LCM LCM LCM P× + 2P × + 12P × P 2P 12P Q R S (ii) + = + LCM of Q, R and S Q R S P

Q T Q T  +S = (P + S ) +  +  R U R U

Subtraction The process of subtraction is the reverse of addition. In subtraction we take away or remove some number of objects from the given objects to make it smaller.

Subtraction of Fractional Numbers

General Method of Multiplication 48 We all know the general method of × 23 multiplication, i.e., how multiplication is done. 144 let us now derive a general formula. 96 The actual multiplication is done in the following 1104 way. I. 48 ↓ 23 8 × 3 = 24 we write ‘4’ and carry ‘2’ 4 8 II. ( 4 × 3) + ( 2 × 8) = 28

2 3

We subtract the fractional numbers in the following ways R P (i) If the fraction is subtracted from , Q Q P R P−R then − = Q Q Q Q −T T Q (ii) P −S = (P − S ) + R R R

Add carry ‘2’ of step I = 28 + 2 = 30 We write ‘0’ at the ten’s place and carry ‘3’. III. 4 8 b 2 3 4× 2= 8 Add carry 3 of step II = 8 + 3 = 11 IV. The resultant product is 1104.

https://sscstudy.com/

https://sscstudy.com/

26

CUET (UG) Section III : General Test

Multiplying the Fractional Numbers While multiplying two or more than two fractions, multiply numerator with numerator and multiply denominator with denominator. P R P× R (i) × = Q S Q× S P R T PRT (ii) × × = Q S U QSU 1 5 13 65 1 5 4 20 (ii) 2 × 4 = × = e.g., (i) × = 3 2 3 6 2 3 7 21 5 4 9 180 90 30 (iii) × × = = = 42 21 7 2 3 7

Following are the steps which are used to solve the questions Step I First of all read and understand the relation which is given for the symbol properly. Step II Keeping the given relation in your mind, arrange the numbers according to it. Step III At last, find the value of the expression using original symbols. e.g. if a * b = a 2 + b2, then 5 * 3 = 52 + 32 = 25 + 9 = 34

Solved Examples 1. Add the numbers 5936, 87569, 75.003, 7.8753,

Division

7135.990.

Division is the method of finding how many times one given number called the divisor is contained in another given number called the dividend. The number expressing this, is called the quotient and the excess of the dividend over the product of the divisor and the quotient is called the remainder. e.g., 265 ÷ 48 Dividend Divisor

48) 265 ( 5 Quotient 240 25 Remainder

(a) 100732.8638 (c) 100723.8683

Sol. (c)

(b) 100723.8863 (d) 100732.8683

5936.0000 87569.0000 75.0030 7.8753 + 7135.9900 Sum = 100723.8683

2. Simplify 1856 − 3287 + 5432 − 679. (a) 3122 (c) 3322

Formulae for Division Based Problems Dividend = Divisor × Quotient + Remainder

(b) 2233 (d) 4500

Sol. (c) Simplified value = 1856 + 5432 − (3287 + 679) = 7288 − 3966 = 3322

Divisor) Dividend (Quotient Remainder Dividend − Remainder Divisor = Quotient Quotient =

How to Solve the Questions?

3. Find the multiplication of the numbers

5384 and 7329. (a) 39429336 (c) 39898736

Dividend − Remainder Divisor

Sol. (b)

Other Mathematical Operations Sometime we face some other operations but actually these are not new. e.g., if we assume the symbol * as +, then 5* 3 = 5 + 3 = 8 It means that our basic mathematical operations come in the form of other symbols but questions are solved by taking the original symbols. The symbols given in the question can be *, δ , π , £ etc.

(b) 39459336 (d) 38459336

5384 × 7329 48456 10768× 16152×× 37688 × × × 39459336

4. If m ⊕ n = m 3 − n2 + mn, find 2 ⊕ 3. (a) 1 (c) 5

(b) −1 (d) −7

3 2 Sol. (c) Q m ⊕ n = m − n + mn



https://sscstudy.com/

2 ⊕ 3 = 2 3 − 32 + 2 × 3 = 8 − 9 + 6 = 5

https://sscstudy.com/

27

NUMERICAL ABILITY

Practice Questions 3 1 − 2 1 ÷ 1 of 1 1    3 004 . 4 =? 2 2 1. of 1 1 1 003 . + of 3 5 9 (a) 1

(b) 5

(c)

8. Which of the following will come in place of both the

1 5

(d)

questions marks (?) in the following equation? 128 ÷ 16 × ? − 7 × 2 =1 72 − 8 × 6 + ? 2

1 2

(a) 17 (c) 18

2. The value of

1 1 1 1 is + + + ... + 100 + 99 2 +1 3+ 2 4+ 3 (a) 1 (c) 99

(a)

9. If a number is decreased by 4 and divided by 6 the

result is 8. What would be the result if 2 is subtracted from the number and then it is divided by 5?

(b) 9 (d) 99 − 1

(a) 9

1 1 2 1 + − 3 4 5 2 3. Simplify 4 2 3 3 1 of − of 5 4 4 3 37 78

(b)

37 13

74 78

(d)

74 13

11. How many

Mathematics book and two pencils for ` 165. Rahim went to the same market and bought another copy of the same book and ten pencils of the same brand for ` 169. The price of each pencil was

1 5

2 5

(d) 10

(b) 10101 (d) 3

1 1 are in ? 8 2 (b) 4 (d) 16

of the number 3 in 12345 is (a) 299 (b) 297 (c) 298 (d) None of the above

13. When 121012 is divided by 12, the remainder is (a) 0 (b) 2 (c) 3 (d) 4

(b) 26 (d) 22

1+

(c) 11

12. The difference of the place value and the face value

(b) ` 1 (d) ` 2

heads : number of feet = 12 : 35, find out the number of hens, if the number of heads alone is 48. 1

1 5

(a) 8 (c) 2

5. A man has some hens and cows. If the number of

6. 1 +

(b) 10

(a) 111 (c) 1001001

4. Ram went to a market and bought one copy of a

(a) 28 (c) 24

2 3

10. If 567567567 is divided by 567, the quotient is (c)

(a) ` 0.50 (c) ` 0.75

(b) 16 (d) 3

is equal to

1 4

14. A chocolate has 12 equal pieces. Manju gave th of it

(a) 11/6 (c) 15/6

1 1 to Anju, rd of it to Sujata and th of it to Fiza. The 3 6 number of pieces of chocolate left with Manju is

(b) 13/6 (d) None of these

7. In the following, which is the greatest number? (a) [(2 + 2) 2] 2 (c) (4) 2

(b) ( 2 + 2 + 2) 2 (d) ( 2 × 2 × 2) 2

(a) 1 (c) 3

(b) 2 (d) 4

ANSWERS 1. 11.

(b) (b)

2. 12.

(b) (b)

3. 13.

(a) (d)

4. 14.

(a) (c)

5.

(b)

6.

(a)

https://sscstudy.com/

7.

(a)

8.

(d)

9.

(d)

10.

(c)

https://sscstudy.com/

28

CUET (UG) Section III : General Test

Hints & Solutions 1 1 1  1 3 − 2  ÷ of 1  3  0.04 2 2 4 1. of 1 1 1 0.03 + of 3 5 9  10 5 5 − ÷  0.04  3 2 8 = × 1 1 0.03 + 3 45 5 8 × 4 6 5 = × 3 1+ 1 3 45 4 4 3 = × 3 16 45 4 4 45 = × × =5 3 3 16 1 2 −1 2. Q × = 2 −1 2 +1 2 −1 Similarly, 1 1 + +... 2 +1 3+ 2 1 + 100 + 99 = ( 2 − 1) + ( 3 − 2 ) + ....+ ( 100 − 99 ) = 100 − 1 = 10 − 1 = 9 1 1 2 1  + − 3 4 5 2  3. 2 3 3 4 1 of − of 3 4 4 5 1 1 4 − 5  + 3 4  10  = 3 5 4 3 × − × 4 3 5 4 1 1 1 + − 3 4  10  = 5 3 − 4 5 1 1 − 37 20 37 = 3 40 = × = 13 120 13 78 20

4. According to the question, Cost price of → (1 book + 2 pencils) = ` 165 …(i) Cost price of → (1 book + 10 pencils) = ` 169 …(ii) On subtracting Eq. (i) from Eq. (ii), we get 8 pencils = ` 4 4 1 pencil = ` = ` 0.50 ∴ 8 5. Suppose, number of heads = 12K and number of feet = 35 K Then, 12K = 48 ⇒ K = 4 ∴ Number of feet = 35 × 4 = 140 Again, suppose number of hens =x and number of cows = y ∴ x + y = 48 …(i) and 2x + 4 y = 140 ⇒ 2x + 4 (48 − x) = 140 [from Eq. (i)] ⇒ 2x + 192 − 4x = 140 ⇒ 2x = 52 ⇒ x = 26 1 6. Expression = 1 + 1 1+ 5 1 5 =1 + =1 + 5+1 6 5 6 + 5 11 = = 6 6 [(2 + 2)2] 2 = (16) 2 = 256 (2 + 2 + 2)2 = (6)2 = 36 (4)2 = 16 (2 × 2 × 2)2 = (8)2 = 64 Therefore, 256 > 64 > 36 > 16 ∴ [(2 + 2)2] 2 is the greatest. 128 ÷ 16 × ? − 7 × 2 8. =1 72 − 8 × 6 + ?2 8 × ? − 14 ⇒ =1 49 − 48 + ?2 2 ⇒ 1 + ? = 8 × ? − 14 ⇒ ?2 − 8 × ? + 15 = 0 7.

https://sscstudy.com/

On putting ‘x’ in place of question mark ‘?’, we get x2 − 8x + 15 = 0 (x − 3) (x − 5) = 0 ∴ x = 3 or 5 9. Let the number be N. According to the question, N −4 =8 6 ⇒ N = (8 × 6) + 4 ⇒ N = 52 N − 2 52 − 2 50 Now, = = = 10 5 5 5 567567567 10. Clearly, = 1001001 567 1 1 11. Required number of in 8 2 1 8 = 2 = =4 1 2 8 12. The place value of the given number is 300 and the face value of that number is 3. So, the required difference is 300 − 3 = 297 13.

12)121012(10084 12 101 96 52 48 Hence, when 121012 is divided by 12, then remainder is 4.

14. The number of pieces of chocolate left with Manju  1 1 1 =1 −  + +   4 3 6  3 + 4 + 2 =1 −    12  9 12 − 9 3 =1 − = = 12 12 12 Hence, number of pieces of chocolate left with Manju is 3.

https://sscstudy.com/

NUMERICAL ABILITY

29

CHAPTER 06

Average An average or arithmetic mean of given data is the sum of the given observations divided by number of observation.

Average The average of a number of quantities belongs to some kind is their sum divided by the number of those quantities. In other words an average of a given observation or data is a number which is found on dividing the sum of observations or data by the number of observations or data given. Sum of observations Average = Number of observations

Points to be Remember The quantities whose average is to be determined, should be in the same unit. Average of the given observations is x. (i) If each observation is increased by a, then new average = x + a (ii) If each observation is decreased by a, then new average = x − a (iii) If each observation is multiplied by a, then new average = ax x (iv) If each observation is divided by a, then new average = a

Important Tips/Formulae Average of two groups are a and b, respectively. If number of items in first and second group are x and y respectively, then ax + by Average of both the groups = x+ y The average weight/age of group of n person is x. If a person of y kg/yr is replaced by another person of z kg/yr, then z − y New average weight/age = x +    n  The average of marks of n students was calculated as x. But it was later found that the marks of one student had been wrongly entered as a instead of b and of another as c instead of d. The correct average Sum of correct marks −Sum of wrongly entered marks  =x+  n   The average of consecutive natural numbers upto n =

n+ 1 2

The average of square of consecutive natural numbers upto n ( n + 1)( 2n + 1) = 6 The average of cubes of consecutive natural numbers upto n  n + 1  = n    2  

2

n + 1 2 n+ 1 The average of consecutive odd numbers upto n = 2 The average of first n consecutive even numbers = ( n + 1) The average of first n consecutive odd numbers = n A man covers a distance d at x km/h and returns back to the starting point at y km/h. Then, 2xy km/h Average speed during whole journey = ( x + y) The average of consecutive even numbers upto n =

https://sscstudy.com/

https://sscstudy.com/

30

CUET (UG) Section III : General Test

Solved Examples 1. If average of given numbers 3, 5, 6, 7, 9, x is 7, then

find the value of x. (a) 12

(b) 10 (c) 14 3+ 5+ 6+ 7+ 9+ x Sol. (a) Average = 6 30 + x i.e., 7= 6 ⇒ 30 + x = 42 ⇒ x = 12

(d) 11

100 students of an examination, by mistake enters 68, instead of 86 and obtained the average as 58; the actual average marks of those students is (b) 57.82

(c) 58.81

(d) 57.28

Sol. (a) Actual total marks of 100 students = 5800 + (86 − 68) = 5818 5818 = 58.18 ∴ Required actual average = 100

3. The average weight of 8 persons increased by

1.5 kg when a person weighing 65 kg is replaced by a new person. What could be the weight of new person? (a) 75 kg

(b) 82 kg

Alternate Method Weight of new person = number of persons × increment of average + weight of the person replaced = 8 × 1.5 + 65 = 77 kg

4. The average of 9 numbers is 30. The average of

2. A tabulator while calculating the average marks of

(a) 58.18

⇒ 8x − 65 + w = 8x + 12 ⇒ w = 77 kg

(c) 77 kg

(d) 70 kg

Sol. (c) Let the average weight of 8 persons be x and the weight of the new person be w kg. Sum of the weight of 8 persons = 8x By given condition, new sum = 8x − 65 + w 8x − 65 + w ∴ New average = = x + 1.5 8

first 5 numbers is 25 and that of the last 3 numbers is 35. What is the 6th number? (a) 42

(b) 40 (c) 39 Sum of observations Sol. (b) Average = Number of observations

(d) 41



Total sum = 9 × 30 = 270 Sum of first five = 5 × 25 = 125 Sum of last three = 3 × 35 = 105 6th number = 270 − (125 + 105) = 40 ∴

5. A library has an average number of 510 visitors on

Sunday and 240 on other days. The average number of visitors per day in a month of 30 days beginning with Sunday is (a) 290 (c) 295

(b) 285 (d) 300

Sol. (b) A month beginning with Sunday will have = 5 Sundays total number of other days = 30 − 5 = 25 days 510 × 5 + 25 × 240 ∴Average number of visitor = 30 2250 + 6000 = = 285 visitors 30

Practice Questions 1. The average of 10, 12, 16, 20, p and 26 is 17. Find

the value of p. (a) 17

(b) 18

(c) 15

(d) 16

2. The average of 11 results is 60 marks. If the

average of first six results is 59 marks and that of the last six is 62 marks, then the sixth result contains (a) 65 marks (c) 60 marks

There are 12 officers having an average wage of ` 14000, while the average wage of the remaining persons is ` 5600, the number of the workers is that factory is (b) 252

(c) 240

(d) 230

4. The average of 5 numbers is 20 and 4 of the

numbers are 10, 15, 20 and 25. If the number are arranged in ascending order, then the average of the last three i (a) 25

(b) 18.75

(c) 24

examination is 73 and that of boys is 71. The average score for the whole class is 71.8. Find the percentage of girls. (a) 40% (c) 55%

(b) 50% (d) 60%

6. The average of three numbers is 135. The largest

number is 195 and the difference between the other two is 20. The smallest number is

(b) 66 marks (d) 61 marks

3. The average wage of workers in a factory is ` 6000.

(a) 242

5. In a class, the average score of girls in an

(d) 22.33

(a) 65 (c) 105

(b) 95 (d) 115

7. The average of 6 numbers is 30. If first four

numbers average is 25 and last three numbers average is 35, then fourth number will be (a) 25 (c) 35

(b) 30 (d) 40

8. In the first 10 overs of a cricket game, the run rate

was only 3.2. What should be the run rate in the remaining 40 overs to reach the target of 282 runs? (a) 6.25

https://sscstudy.com/

(b) 6.5

(c) 6.75

(d) 7

https://sscstudy.com/

31

NUMERICAL ABILITY

9. A grocer has a sale of ` 6435, ` 6927, ` 6855, ` 7230 and ` 6562 for 5 consecutive months. How much

17. A motorist travels to a place 150 km away at an

average speed of 50 km/h and returns at 30 km/h. His average speed for the whole journey (in km/h) is

sale must he have in the sixth month, so that he gets an average sale of ` 6500? (a) ` 4991

(b) ` 5991

(c) ` 6001

(d) ` 6991

(a) 35

(b) 37

(c) 37.5

(d) 40

18. Three years ago, the average age of a family of 5

10. The population of a town increased from 175000 to

262500 in a decade. The average per cent increase of population per year is

members was 17 yr. A baby having been born, the average age of the family is the same today. The present age of the baby is

(a) 4.37%

(a) 2 yr

(b) 5%

(c) 6%

(d) 8.75%

11. The average of the even numbers from 1 to 30 is (a) 15

(b) 17

(c) 19

(c) 20

(a) 35 yr

(c) M − N

(d) 2M − N

14. The average of marks of a student in 7 subjects is

(a) 90

(b) 90 (d) None of these

(b) 3 : 4 (d) None of these

16. The average of 50 numbers is 38. If two numbers,

(a) 60

(c) 37.5

(d) 100

(b) 55

(c) 65

(d) 50

average of another eight numbers is 81. What is the combined average of all numbers together? (a) 66

(b) 60.5

(c) 68.5

(d) 64

23. The average of first five multiple of 3 is (a) 3

(b) 9

(c) 12

(d) 15

3 1 1 1 24. Find the average of four numbers 2 , 5 , 4 , 8 . 4 3 6 2 3 16 5 (c) 16 3

3 16 16 (d) 3 3

(a) 5

namely 45 and 55 are discarded, the average of remaining numbers is (b) 37

(c) 85

22. The average of five numbers is 42 while the

age of the boys in the class is 16.4 yr while that of the girls is 15.4 yr. What is the ratio of boys to girls in the class ?

(a) 36.5

(b) 110

average is 30 runs. How many runs must he make in his next innings so as to raise his average to 32?

15. The average age of a class is 15.8 yr. The average

(a) 1 : 2 (c) 3 : 5

(d) 50 yr

21. A cricketer has completed 14 innings and his

75. His average in 6 subjects excluding Science is 72. How many marks did he get in Science? (a) 72 (c) 93

(c) 30 yr

of first five observations is 58 and that of last five is 56, the sixth observation is

N, then the other number is (b) 2M

(b) 40 yr

20. The average of 11 observations is 60. If the average

(d) 22

13. The average of two numbers is M. If one number is (a) 2N

(d) 1.5 yr

15 yr. Average age of P, Q and R today is 20 yr. How old will R be after 10 yr?

highest of these numbers will be (b) 18

(c) 3 yr

19. Five years ago, the average age of P and Q was

(d) 16

12. The average of 5 consecutive numbers is 18. The (a) 24

(b) 2.4 yr

(d) 37.52

(b) 3

ANSWERS 1. (b) 11. (d) 21. (a)

2. (b) 12. (c) 22. (a)

3. (b) 13. (d) 23. (b)

4. (a) 14. (c) 24. (a)

5. (a) 15. (d)

6. (b) 16. (c)

7. (a) 17. (c)

8. (a) 18. (a)

9. (a) 19. (c)

10. (b) 20. (a)

Hints & Solutions 12 × 14000 + (n − 12) × 5600 ⇒ 6000n n = 168000 + 5600n − 67200 ⇒ 400n = 100800 ∴ n = 252

1. Average of 10, 12, 16, 20, p and 26 = 17 10 + 12 + 16 + 20 + p + 26 = 17 ⇒ 6 ⇒ 84 + p = 102 ⇒ p = 18

6000 =

2. Required perimeter = 6 × 59 + 6 × 62 − 11 × 60 = 6 [59 + 62 − 110] = 6 [121 − 110] = 6 × 11 = 66 3. Let the total number of employee in factory be n, then According to question,

4. Sum of 5 numbers = 20 × 5 = 100 Since, 4 of the number are 10, 15, 20 and 25, therefore fifth number = 100 − (10 + 15 + 20 + 25) = 100 − 70 = 30 Now, arranging in ascending order, last three numbers are 20, 25 and 30.

https://sscstudy.com/

https://sscstudy.com/

32

CUET (UG) Section III : General Test

∴ Required average 20 + 25 + 30 75 = = = 25 3 3 5. Let the number of girls and boys are x, y respectively. By given condition, 73x + 71 y = 71.8 (x + y) ⇒ 1. 2x = 0. 8 y 2 x= y 3 ∴ Percentage of girls x = × 100% (x + y) 2 y 3 = × 100% = 40% 2   y + y 3  6. Let smallest number = x Second number = x + 20 ∴ According to question, Average of three numbers = 135 x + x + 20 + 195 = 135 3 ⇒ 2x = 135 × 3 − (20 + 195) ⇒ 2x = 405 − 215 190 x= = 95 2 7. Sum of 6 numbers = 6 × 30 = 180 Sum of first 4 numbers = 4 × 25 = 100 Sum of last 3 numbers = 3 × 35 = 105 Hence, fourth number = (100 + 105) − 180 = 205 − 180 = 25 8. Total run in 10 overs = 3. 2 × 10 = 32 runs Remaining run in 40 overs = 282 − 32 = 250 runs ∴Required run rate in 40 overs 250 = 6 .25 = 40 9. Let total sell will be in 6 months =`x Total amount Average = Number of months 6435 + 6927 + 6855 + 7230 + 6562 + x 6500 = 6 ⇒ 6500 × 6 = 34009 + x ∴ x = 39000 − 34009 = ` 4991 10. Population increased in a decade = 262500 − 175000 = 87500

The average per cent increase of population 87500 = × 100 175000 × 10 8750 = = 5% 1750 11. The average of even numbers from 1 to n is n + 2 30 + 2 = = 2 2 32 = = 16 2 12. The four consecutive numbers are x, x + 1, x + 2, x + 3, x + 4 x + (x + 1) + (x + 2) + (x + 3) + (x + 4) = 18 5 5x + 10 = 18 ⇒ 5 ⇒ 5x = 90 − 10 80 x= ⇒ 5 ∴ x = 16 The highest number = x + 4 = 16 + 4 = 20 N +x 13. Given, M = , 2 where x is the other number. Then, x = 2 M − N ⇒

14. Average Sum of numbers in all subjects = Number of subjects ∴ Sum of numbers in 7 subjects = 75 × 7 = 525 Sum of numbers in 6 subjects (excluding Science) = 72 × 6 = 432 ∴ Marks in Science = 525 − 432 = 93 15. Boys 16.4

Girls 15.8

15.4 0.4 0.6 Required ratio = 2 : 3 16. Average of remaining numbers (38 × 50) − (45 + 55) = 48 1800 = = 37.5 48 17. Average speed Total distance = Total time 300 = 150 150 + 30 50 300 = 37.5 km/h = 8

https://sscstudy.com/

18. Total age of family three years ago = 17 × 5 = 85 yr Total age of family today = 85 + 5 × 3 = 100 yr Let the age of the baby be x yr. 100 + x = 17 ∴ 6 ⇒ x = 2 yr 19. Total age of P and Q today = (15 × 2 + 5 × 2) = 40 yr Total age of P , Q and R today = 20 × 3 = 60 yr Therefore, present age of R = 60 − 40 = 20 yr Age of R after 10 yr = 20 + 10 = 30 yr 20. Sum of 11 observation = 60 × 11 = 660 Sum of first 5 observation = 5 × 58 = 290 Sum of last 5 observation = 5 × 56 = 280 ∴6th observation = 660 − (290 + 280) = 90 21. Total runs scored in 14 innings = 30 × 14 = 420 Suppose he makes x runs in his next inning. 420 + x Then, = 32 15 ∴ x = 32 × 15 − 420 = 60 runs 22. Sum of five numbers = 42 × 5 = 210 Sum of eight numbers = 81 × 8 = 648 ∴ Average of all numbers 210 + 648 858 = = = 66 13 13 23. First five multiple of 3 = 3, 6, 9, 12, 15 3 + 6 + 9 + 12 + 15 Average = 5 45 = =9 5 24. Sum = (2 + 5 + 4 + 8) +  3 1 1 1  + + +   4 3 6 2 7 3 = 19 + = 20 + 4 4 3 20 + 4 Average = 4 3 20 3 3 = + =5+ =5 16 4 16 16

https://sscstudy.com/

NUMERICAL ABILITY

33

CHAPTER 07

Ratio and Proportion The number of times one quantity contains another quantity of the same kind is called ratio of the two unit.

Ratio

Proportion

It is a tool to compare two or more numbers of same quantities. Or The ratio of two quantities in the same units is the fraction that one quantity is of the other. a Thus, the ratio a to b is the fraction written as a : b. b

The equality of two ratios is called proportion. Let a , b, c and d are four quantities, then the proportional are a : b :: c : d.

Note In the ratio a : b, the first term a is antecedent and second term b is consequent.

Properties of Ratios (i) The value of a ratio remains unchanged, if each one of its terms is multiplied or divided by a same non-zero number. (ii) a 2 : b2 is the duplicate ratio of a : b. (iii) a3 : b3 is the triplicate ratio of a : b. (iv)

Note In the proportion a : b :: c : d , a and d are extreme values and b and c are mean values. i.e., Product of means = Product of extreme

Properties of Proportion (i) If x is the third proportional to a , b, then a : b :: b : x. (ii) Mean proportional between a and b is ab. a c a+b c+d (iii) If = , then = b d a−b c−d a−b c−d and (Componendo and Dividendo) = a+b c+d e.g., Divide ` 1024 among, A , B and C so find the value of A and B. Sol. Sum of the ratios = 4 + 5 + 7 = 16

9 First part =  1024 ×  = ` 576  16  7 and second part =  1024 ×  = ` 448  16 

a : b is the sub-duplicate ratio of a : b.



(v) If a : b and c : d are two ratios, then the compounded ratio is ac : bd.

Important Tips/Formulae a a+ x a−x If = 1, = 1 and =1 b a+ x a−x a c e If = = = ... = k b d f a + c + e + ... Then, =k b + d + f + ... If A : B and B : C are given, then A : B : C is given by A : B

e.g., If A : B = 5 : 4 and B : C = 3 : 2, then A : B : C = ? Sol.

3 : 2 = 5× 3: 3× 4: 4×2 = 15 : 12 : 8 If

B : C i.e.,

5 : 4

a N1 b N2 c N3 , = , then a : b : c: d is given by = , = b D1 c D2 d D3 N1 N2 N3 : D1 N2 N3 : D1 D2 N3 : D1 D2 D3

A ×B :B ×B :B ×C

https://sscstudy.com/

https://sscstudy.com/

34

CUET (UG) Section III : General Test

Solved Examples 1. If

a 3 b 5 c 7 = , = , = , then find a : b : c : d. b 4 c 6 d 8

(a) 105 : 120 : 135 : 180 (c) 105 : 140 : 158 : 212

(b) 105 : 140 : 168 : 192 (d) 115 : 140 : 168 : 182

= N1 N 2N3 : D1 N 2N3 : D1 D2N3 : D1 D2D3 = 3 × 5 × 7 : 4 × 5 × 7 : 4 × 6 × 7 : 4 × 6 × 8 = 105 : 140 : 168 : 192

2. What is the ratio of 80 paise and ` 5? (b) 4 : 35

(c) 4 : 25

(d) 6 : 35

Sol. (c) ` 5 = 5 × 100 paise Their ratio =

7+ x 3 = 11 + x 4

⇒ 28 + 4x = 33 + 3x ⇒ 4x − 3x = 33 − 28 ⇒ x = 5

5. If x : y = 3 : 1, then find the ratio of x 3 − y 3 : x 3 + y 3.

Sol. (b) a : b : c : d

(a) 2 : 25

Sol. (d) Let the required number is x, then

80 = 4 : 25 500

(a) 12 : 11 (c) 13 : 14

(b) 13 : 21 (d) 21 : 42 x = 3 k; y = k

Sol. (c) Let 3

3

Then,

x − y = 27 k3 − k3 = 26 k3

and

x3 + y3 = 27 k3 + k3 = 28 k3

Their ratio =

3. Two natural numbers are in the ratio 3 : 5 and

26 k3 28 k3

= 13 : 14

6. If two numbers are in the ratio of 5 : 8 and if 9 be

their product is 2160. The smaller of the numbers is

added to each, the ratio becomes 8 : 11. Find the lower number.

(a) 36

(a) 10

(b) 24

(c) 18

(d) 12

Sol. (a) Let the natural numbers are 3x and 5x, then 3x × 5x = 2160 ⇒15x2 = 2160 ⇒ x2 = 144 ⇒ x = 12 Hence, smaller = 3x = 3 × 12 = 36

7 : 11 so as to make it equal to 3 : 4? (b) 7.5

(c) 6.5

(c) 12

(d) 15

Sol. (d) Let the numbers are 5 x and 8 x. 5x + 9 8 = 8x + 9 11 64x + 72 = 55x + 99 9x = 27

By given condition,

4. What must be added to each term of the ratio (a) 8

(b) 13

(d) 5

⇒ ⇒ ⇒

x=3

Hence,

lower number = 5 × 3 = 15

Practice Questions 1. The least ratio among 10 : 18, 7 : 21, 12 : 16, 8 : 20

is (a) 12 : 16 (c) 10 : 18

3 persons in the ratio 3 : 4 : 7, so that the second person receives ` 12 only?

(b) 8 : 20 (d) 7 : 21

2. If x be the mean proportion between ( x − 2) and

( x − 3), then the value of x is (a) 6 (c) 6/5

is (b) 30 (d) 24

4. The ratio of land and water in the whole world is

1 : 2 . If this ratio in the Northern hemisphere be 2 : 3, then the ratio of land and water in the Southern hemisphere is (a) 4 : 7 (c) 3 : 4

(b) 4 : 11 (d) 4 : 3

5. A certain number is divided into two parts such

that 5 times the first part added to 11 times, the second part makes 7 times the whole. The ratio of the first part to the second part is (a) 2 : 1 (c) 1 : 2

(b) 5 : 11 (d) 2 : 3

(a) ` 21 (c) ` 9

(b) ` 32 (d) ` 42

7. In a certain examination, the number of those who

(b) 5 (d) 5/6

3. The fourth proportional of the numbers 12, 16, 18 (a) 28 (c) 20

6. What sum of money is to be divided among

passed was 4 times the number of those who failed. If there had been 35 fewer candidates and 9 more had failed, the ratio of passed and failed candidates would have been 2 : 1, then the total number of candidates was (a) 135 (c) 145

(b) 155 (d) 150

8. The monthly income of H and W is in the ratio 4 : 3

and the expenditure is in the ratio 3 : 2. If each of them saves ` 600 per month, the income of W, in rupees is (a) 1200 (c) 1800

9. If

(b) 2400 (d) 9000

a 2 b 4 = and = , then ( a + b) :( b + c) = ? b 3 c 5

(a) 3 : 4 (c) 5 : 9

https://sscstudy.com/

(b) 4 : 5 (d) 20 : 27

https://sscstudy.com/

35

NUMERICAL ABILITY

10. Ratio between the monthly incomes of A and B is

20. If there are ` 495 in a bag in denominations of ` 1,

9 : 8 and the ratio between their expenditures is 8 : 7. If they save ` 500 each, find A’s monthly income.

50 paisa and 25 paisa coins, which are in the ratio 1 : 8 : 16. How many 50 paisa coins are there in the bag?

(a) ` 3500 (c) ` 4500

(a) 50 (c) 440

(b) ` 4000 (d) ` 5000

11. If x : y = 3 : 4, then the value of 7 (a) 25

7 (b) 23

5x − 2 y . 7x + 2y

7 (c) 29

21. The monthly salary of A, B, C are in the ratio of

2 : 3 : 5. If C’s monthly salary is ` 1200 more than that of A, then B ’s annual salary is

7 (d) 17

(a) ` 14400 (c) ` 1200

1 1 1 1 , then the value of x is 12. If : = : 5 x x 1.25 (a) 1.5

(b) 2

(c) 2.5

(a) 1 : 50 (c) 1 : 40 (a) 1 : 3 (c) 2 : 3

(c) 18

(d) 25

then the value of c is

1 (a) 2

a+ b (b) c+ d

a−b (c) c−d

(d)

(a) 6 (c) 8

25. If

a2 + b2 15. If a : b = c : d, then the value of 2 is c + d2

1 of A = 75% of B = 06 . of C, then A : B : C is 3 (b) 5 : 9 : 4 (d) 9 : 4 : 5

26. A certain sum of money is divided between P and

d2

1 1 : 5 . If P gets ` 180 less than Q, 2 2 then the square of Q is

Q in the ratio of 3

6 leaps of the cat, but 4 leaps of the dog are equal to 5 leaps of the cat. Compare the speeds of the dog and the cat. (b) 9 : 25 (d) 25 : 24

(a) ` 315 (c) ` 630

(b) ` 495 (d) ` 810

27. Four numbers in the ratio 1 : 3 : 4 : 7 add upto give

a sum of 105. Find the value of the biggest number.

17. What should be subtracted from each of the

numbers 54, 71, 75 and 99, so that the remainders are in continued proportion? (a) 9 (c) 4

(b) 7 (d) 14

(a) 4 : 5 : 9 (c) 9 : 5 : 4

b2

16. A dog pursues a cat and takes 5 leaps for every

(a) 15 : 22 (c) 25 : 21

(b) 3 : 2 (d) 1 : 2

24. If a + b : b + c : c + a = 6 : 7 : 8 and a + b + c = 14 ,

25 paise coins respectively. If the total value is ` 35, how many coins of each type are there? (b) 16

(b) 2 : 35 (d) 3 : 25

23. If p : q = 3 : 4 and q : r = 8 : 9, then p : r is

(b) na : mb (d) md : nc

14. A bag contains an equal number of ` 1, 50 paise and

(a) 20

(b) ` 24000 (d) ` 2000

22. The ratio of 40 m and 2 km is (d) 3.5

ma + nc is equal to 13. If a : b = c : d, then mb + nd (a) m : n (c) a : b

(b) 220 (d) None of these

(a) 42

(b) 35

(c) 49

(d) 63

28. What number must be taken from each term of the

fraction 27/35 that it may become 2 : 3?

(b) 7 (d) 3

(a) 9

18. Divide 170 into three parts such that the first part

(b) 10

(c) 7

(d) 11

29. The present ratio of ages of A and B is 4 : 5. 18 yr

is 10 more than the second and its ratio with third part is 2 : 5.

ago, this ratio was 11 : 16. Find the sum of total of their present ages.

(a) 22 : 35 : 113 (c) 40 : 30 : 100

(a) 90 yr

(b) 35 : 55 : 80 (d) 35 : 65 : 70

19. Ratio of boys to the girls in a class is 5 : 4. Which of

(c) 110 yr

(d) 80 yr

30. If two numbers are in the ratio of 5 : 8 and if 9

added to each, the ratio becomes 8 : 11. Find the sum of the numbers.

the following cannot be the number of students in the class? (a) 45 (c) 108

(b) 105 yr

(a) 39

(b) 49

(c) 64

(d) 19

(b) 72 (d) 98

ANSWERS 1. (c) 11. (c) 21. (a)

2. (c) 12. (c) 22. (a)

3. (d) 13. (c) 23. (c)

4. (b) 14. (a) 24. (a)

5. (a) 15. (d) 25. (d)

6. (d) 16. (d) 26. (b)

https://sscstudy.com/

7. (b) 17. (d) 27. (c)

8. (c) 18. (c) 28. (d)

9. (d) 19. (d) 29. (a)

10. (c) 20. (c) 30. (a)

https://sscstudy.com/

36

CUET (UG) Section III : General Test

Hints & Solutions 1.

10 2 7 1 = = 0.22 , = = 0.33 , 18 9 21 3 12 3 8 2 = = 0.75, = = 0 .4 16 4 20 5 Thus, 10:18 is the least among the given ratios.

2. Since, x is mean proportion of (x − 2) and (x − 3). ∴ x2 = (x − 2)(x − 3) ⇒ x2 = x2 − 5 x + 6 ⇒ 5x = 6 6 x= ⇒ 5 3. Let x be the fourth proportion of 12, 16 and 18, then 16 × 18 x= ⇒ x = 24 12 4. Let the whole world having 30 equal parts the fraction of the 1 land to the world = × 30 = 10 3 parts and the fraction of the water to 2 the world = × 30 = 20 parts 3 Q Northern hemisphere carries 15 parts of a whole, hence the fraction of land at Northern hemisphere 2 = 15 × = 6 parts 5 and the fraction of water at Northern hemisphere 3 = 15 × = 9 parts 5 ∴Ratio of land and water in Southern hemisphere = (10 − 6) : (20 − 9) = 4 : 11 5. Let first and second part of the number be x and y, respectively. Then, 5x + 11 y = 7(x + y) ⇒ 11 y − 7 y = 7x − 5x ⇒ 4 y = 2x ∴ x : y = 2 :1 6. Let required money be ` x. Then,



4 x = 12 14 12 × 14 x= = ` 42 4

7. Let number of failed and passed candidates be x and 4x, respectively. Therefore, total number of candidates was 5x. According to the question, if total number of students had been 5x − 35, then 4x − 35 − 9 2 = x+9 1 ⇒ 4x − 44 = 2(x + 9) ⇒ 4x − 2x = 18 + 44 ⇒ 2x = 62 ∴ x = 31 Thus, total number of candidates was 31 × 5 i.e., 155. 8. Let the monthly income of H and W be `4x, ` 3x respectively and the expenditure be ` 3 y,` 2 y respectively. By given condition, ...(i) 4x − 3 y = 600 ...(ii) 3x − 2 y = 600 From Eqs. (i) and (ii), x = 600 ∴Monthly income of W = 600 × 3 = ` 1800 a 2 b 4 9. = , = b 3 c 5 ⇒ a : b : c = 8 : 12 : 15 Let a = 8k, b = 12k and c = 15k a + b 8k + 12k 20 = = ∴ b + c 12k + 15k 27 10. Let A’s monthly income = ` 9x and B ’s monthly income = ` 8x According to question, Ratio between their expenditures = 8 : 7 9x − 500 8 = ⇒ 8x − 500 7 6x − 3500 = 64x − 4000 x = 500 ∴ A’s monthly income = 9 × 500 = ` 4500 x 3 4x 11. Q = ⇒ y= y 4 3  4x 5x − 2    3 5x − 2 y ∴ = 7x + 2 y  4x 7x + 2    3 8x 5x − 3 = 15x − 8x = 7x = 7 = 8x 21x + 8x 29x 29 7x + 3

https://sscstudy.com/

12.

1 1 1 × = 2 5 1. 25 x

x2 = 5 × 1.25 x = 6.25 = 2.5 a c 13. Let = =k b d Then a = bk, c = dk a ma + nc mbk + ndk =k= = ∴ b mb + nd mb + nd ⇒

14. Number of coins of each type Total value = Sum of value of each type of coin 35 = = 20 1 + 0.5 + 0.25 a c 15. = = k; a = bk, c = dk b d  a 2 + b2 b2(k2 + 1) b2 = =  2  d 2(k2 + 1) d 2  c + d2  5 ×5 16. 5 leaps of dog = leaps of cat 4 ∴ Speed of dog : Speed of cat 25 = : 6 = 25 : 24 4 17. Let number x be subtracted from each of the numbers, then 54 − x 75 − x = 71 − x 99 − x For x = 3, this relation is correct. 18. Let first and third part be 2x and 5x, then second part be 2x − 10. (2x) + (2x − 10) + 5x = 170 ⇒ x = 20 Therefore, three parts are 40, 30 and 100. 19. The total number of students should be divisible by (5 + 4) = 9 20. Ratio of number of coins = 1 : 8 : 16 Ratio of value of each type of coins = 1 : 4 : 4 ∴ Value of 50 paisa coins 4 = × 495 = 220 9 Therefore, number of coins of 50 paisa = 220 × 2 = 440 21. Let the monthly salary of A , B and C be 2x, 3x and 5x. 5x = 2 x + 1200 ⇒ x = ` 400

https://sscstudy.com/

NUMERICAL ABILITY

So, annual salary of B = 3 × 400 × 12 = ` 14400 22. 24 m = 2 × 1000 = 2000 m So that, ratio of 40 m and 2000 m = 40 : 2000 = 1 : 50 23. p : q = 3 : 4 and q : r = 8 : 9 p 3 q 8 i.e., = and = q 4 r 9 On multiplying these two p p q 3 8 2 = × = × = r q r 4 9 3 ∴ p: r = 2 :3 24. Given a + b : b + c : c + a = 6 : 7 : 8 Let a + b = 6k, b + c = 7k and c + a = 8k ∴ 2(a + b + c) = 6k + 7k + 8k 21 ⇒ a + b+ c= k 2 21 ⇒ 14 = k 2 28 4 k= = ⇒ 21 3 Therefore, c = (a + b + c) − (a + b)

=

21 9k 9 4 k − 6k = = × 2 2 2 3 c=6

∴ 1 25. of A = 75% of B = 0.6 of C 3 1 75 × B ⇒ ×A= = 0.6 × C 3 100 A 3B 3C = = ⇒ 3 4 5 9 5 Now, A = B and C = B 4 4 Then, ratio of 9 5 A : B :C = B : B : B 4 4 Required ratio = 9 : 4 : 5

26. The ratio of P ’s share and Q ’s share 1 1 7 11 = 3 :5 = : = 7 : 11 2 2 2 2 By given condition, 11x − 7x = 180 ⇒ x = 45 ∴ The share of Q = 11x = 11 × 45 = ` 495 27. Let the numbers are x, 3x, 4x and 7x. By given condition, x + 3x + 4x + 7x = 105

https://sscstudy.com/

37

⇒ 15x = 105 ⇒ x=7 ∴ Biggest number = 7 × 7 = 49 27 − x 2 28. Let x be taken, then = 35 − x 3 ⇒ 70 − 2x = 81 − 3x ⇒ x = 11 29. Let the present ages of A and B are 4x and 5x, respectively. 4x − 18 11 By given condition, = 5x − 18 16 ⇒ ⇒

64x − 18 × 16 = 55x − 18 × 11 9x = 18(16 − 11)

⇒ x = 10 ∴The sum of their present ages = 40 + 50 = 90 yr 30. Let the numbers are 5x and 8x. By given condition, 5x + 9 8 = 8x + 9 11 ⇒ 64x + 72 = 55x + 99 ⇒ 9x = 27 ⇒ x=3 Hence, the numbers are 5 × 3, 8 × 3 i.e., 15 and 24. The sum of the numbers = 15 + 24 = 39

https://sscstudy.com/

38

CUET (UG) Section III : General Test

CHAPTER 08

Percentange Percentage is a method of expressing fractions or parts of any quantity into a equal form. It is expressed in terms of hundredths.

Per cent The word per cent means per hundred. Thus 21 per cent means 21 part out of 100 parts, which can also be 21 written as . Therefore, per cent is a fraction whose 100 denominator is 100 and the numerator of this fraction is called the rate per cent. 21 So, = 21%. Here 21 per cent is the rate. The sign for 100 per cent is %. (i) To convert a fraction or a decimal or a whole number into a per cent, multiply if by 100. (ii) To convert a per cent to a fraction or a decimal, divided it by 100 and delete the % sign. Also, any fraction or decimal can be converted into its equivalent percentage by multiplying with 100%.

1 100 1 2% = 50 1 4% = 25 1 5% = 20 1 1 6 %= 4 16 1 1 8 %= 3 12 1 10% = 10 1% =

1 1 12 % = 2 8 2 1 16 % = 3 6 1 20% = 5 1 25% = 4 1 1 33 % = 3 3 1 50% = 2 1 100% = 1

4 25 16 64% = 25 2 40% = 5 3 60% = 5 4 80% = 5 1 3 37 % = 2 8 1 7 87 % = 2 8

16% =

1 5 83 % = 3 6

1 4 %= 3 3 2 2 66 % = 3 3 2 8% = 25

133

Note Every number is 100% of itself.

Important Tips/Formulae If A’s income is x% more than that of B,B ’ s income is less than 100 ⋅ x  that of A by  %.  100 + x  If A’s income is x% less than that of B, B’s income is more than 100 ⋅ x  that of A by  %.  100 − x 

Suppose the price of a certain commodity increases by R% and expenditure on this commodity remains the same. Then, R reduction percentage in consumption =  × 100 %   100 + R Suppose the price of commodity decreases by R% and expenditure on this commodity remains the same. Increase R percentage in consumption =  × 100 %   100 − R

https://sscstudy.com/

https://sscstudy.com/

NUMERICAL ABILITY

39

Solved Examples 1. 60% of a number is 24 less than 3/4th of that

number. Find the number (a) 150

(b) 180

(c) 160

(d) 175

Sol. (c) Let the required number be x.

1 (b) 11 % 9

(c) 13%

(b) 13% 1 (d) 13 % 23

Sol. (d) Suppose the price of cooking gas = ` 100 Increased price = ` 115 ⇒

2. If A earns 10% more than B’ s, then how many per 1 (a) 11 % 3

how many per cent should a family reduce its consumption so as not to exceed its budget on cooking gas? (a) 14% (c) 17%

Then, by given condition 3 60% of x = x − 24 4 60 3x 3x 3x  ×x= − 24 ⇒  −  = 24 ⇒  4 100 4 5 ⇒ 15x − 12x = 480 ⇒ x = 160

cent less does B earn than A ?

3. If the price of the cooking gas increased by 15%, by

1 (d) 12 % 2

 Sol. (b) Required percentage = 

R × 100 %  100 − R  10 = × 100 100 − 10 1 10 = × 100 = 11 % 9 90

⇒ ⇒

` 115, he should reduce ` 15 15 ` 1, he should reduce ` 115 1 15 × 100 = 13 % ` 100 he should reduce ` 23 115

Alternate Method Reduction percentage in consumption R   = × 100 %   (100 + R ) 15 1 = × 100 = 13 % 115 23

Practice Questions 1. The length of a rectangle is increased by 40% and

7. The cost of an article worth ` 100 is increased by

its breadth decreased by 30%. The change in the area of the rectangle is

10% first and again increased by 10%. The total increase in rupees is

(a) 2% increase (c) 10% increase

(a) 20

(b) 2% decrease (d) 10% decrease

government aided school is 3 : 2. 20% of the boys and 25% of the girls are not scholarship holders. The percentage of students who are scholarship holders is? (b) 48%

(c) 60%

(d) 78%

3. The side of a square increases by 10%, then find,

by what per cent, does its area increase? (a) 27

(b) 30

(c) 19

(d) 21

1 2

4. A number increased by 137 % and the increment (b) 22

(c) 24

(d) 25

5. A’s salary is 20% less than B’s salary. Then, B’s

salary is more than A’s salary by

1 (a) 33 % 3 (c) 20%

(a) 3.3

(b) 3

(c) 2.9

(d) 2.7

9. In an examination, 1100 boys and 900 girls

appeared. 50% of the boys and 40% of the girls passed the examination. The percentge of candidates who failed is (a) 45 (c) 50

(b) 45.5 (d) 54.5

10. When the price of cloth was reduced by 25%, the

(a) 5% increase (c) 10% increase (a) 400

(b) 5% decrease (d) 10% decrease

(b) 600

(c) 250

(d) 500

12. 23% of 8040 + 42% of 545 = ? % of 3000

female and 60% of the male voted for me. The percentage of votes I got was (b) 42 (d) 52

excess of 5% on one side and 2% deficit on the other. Then, the error per cent in the area is

11. 20% of (50% of 5000) is

2 (b) 16 % 3 (d) 25%

6. In an office 40% of the staff is female, 40% of the

(a) 24 (c) 50

(d) 121

quantity of cloth sold increased by 20%. What was the effect on gross receipt of the shop?

is 33. The number is (a) 27

(c) 110

8. In measuring the sides of a rectangle, there is an

2. The ratio of the number of boys and girls in a

(a) 70%

(b) 21

(a) 56.17 (c) 69.27

(b) 63.54 (d) 71.04

13. ?% of 8745 = 5159. 55 (a) 47 (c) 54

https://sscstudy.com/

(b) 49 (d) None of these

https://sscstudy.com/

40

CUET (UG) Section III : General Test

14. If 16% of x is same as 12% of 48, then x =? (a) 24

(b) 36

(c) 32

22. If x earns 25% less than y, what per cent more does

y earn, then x?

(d) 40

(a) 25%

15. 80% of 1450 is (a) 1160

(b) 1235

(c) 1045

(d) 1250

(b) 2

(c) 40

1 2

(d) 4

number is

25 is 150, then the value of x is 17. If x% of 2 (a) 1000

(b) 1200

(c) 1400

(a) 16

(b) ` 150

(c) ` 67

(a) 350 kg (c) 240 kg

(d) ` 1.50

(c) 50

(b) 12

(c) 15

(a) 25% (d) 20

(c) ` 8

(c) 18%

(d) 15%

455 failed. How many students appeared for the examination?

purchaser to obtain 3 kg more for ` 120. The original price of sugar per kg is (b) ` 12

(b) 20%

26. In an examination, 35% of the students passed and

21. A reduction of 20% in the price of sugar enable a

(a) ` 15

(b) 250 kg (d) 450 kg

expenses on petrol should not increase. One must reduce travel by

(d) 40

20. What per cent of 400 is 60? (a) 6

(d) 28

25. The price of petrol went up by 25% . In order that

number, then the number is (b) 60

(c) 20

the quantity of the ore required is

19. If 35% of a number is 12 less than 50% of that (a) 80

(b) 24

24. An ore contains 26% copper. To get 91 kg of copper

(d) 1500

18. 10% of 15% of 20% of ` 500 is (a) ` 225

2 (d) 16 % 3

23. A number increased by 137 % gives 33, then the

16. 0.008 is what per cent of 0.2? (a) 0.4

1 (c) 33 % 3

(b) 15%

(a) 490 (c) 1300

(d) ` 10

(b) 700 (d) 845

ANSWERS 1. (b) 11. (d) 21. (d)

2. (d) 12. (c) 22. (c)

3. (d) 13. (d) 23. (b)

4. (c) 14. (b) 24. (a)

5. (d) 15. (a) 25. (b)

6. (d) 16. (d) 26. (b)

7. (b) 17. (b)

8. (c) 18. (d)

9. (d) 19. (a)

10. (d) 20. (c)

Hints & Solutions 1. Here, x = 40, y = 30 Required change in the area of rectangle xy   = x − y − %  100 40 × 30  = 40 − 30 − %  100  = −2% Where negative sign shows the decrement. 2. Let number of boys and girls in the school be 3x and 2x, respectively making 5x as the total number of students. Now, number of students who are scholarship holders = 80% of 3x + 75% of 2x = 2 . 4x + 1 . 5x = 3. 9x Therefore, required percentage 3. 9x = × 100% 5x = 78%

3. Required percentage increment  (10)2 x2   = 2x +  = 2 × 10 +  100  100   = 20 + 1 = 21 % 4. Let the number be x. Then, 137.5% of x = 33 33 × 100 ∴ x= = 24 137. 5 5. Required percentage 20 = × 100% 100 − 20 20 = × 100% = 25% 80 6. Let the total number of employes =x Number of female employes 4x = 40% of x = 10 and number of male employes 6x = 10

https://sscstudy.com/

∴ Number of my votes 4x 6x + 60% of = 40% of 10 10 16x 36x 52x = + = 100 100 100 ∴Required percentage = 52% 7. x = 10, y = 10 ∴Total increase 10 × 10   = 10 + 10 + % = 21% 100   8. x = 5%, y = − 2% ∴ Required error per cent in the xy   area = x + y +  100  5 ×2 =5 −2 − = 2 . 9% 100 9. Number of total candidates = 1100 + 900 = 2000 Number of boys passed the examination = 50% of 1100 = 550 Number of girls passed the examination

https://sscstudy.com/

NUMERICAL ABILITY 900 × 40 = 360 100 Total number of candidates passed the examination = 550 + 360 = 910 Total number of candidates failed the examination = 2000 − 910 = 1090 ∴ Required percentage 1090 = × 100 = 54. 5% 2000 = 40% of 900 =

10. x = − 25%, y = 20% ∴ Required effect 25 × 20   = −25 + 20 − %  100  = − 10% or 10% decrease 11. 20% of (50% of 5000)  50  = 20% of  × 5000  100  = 20% of 2500 20 × 2500 = = 500 100 12. ?% of 3000 = 23% of 8040 + 42% of 545 3000 ⇒ ?× 100 42 23 = 8040 × + 545 × 100 100 3000 ?× = 1849.2 + 228 .9 ⇒ 100 ⇒ ? × 30 = 2078 .1 2078 .1 ⇒ ?= 30 ⇒ ? = 69.27 13. ?% of 8745 = 5159.55 ? ⇒ 8745 × = 5159.55 100 515955 ⇒ ?= = 59 8745

14. 16% of x = 12% of 48 48 48 × 12 ⇒ x× = 100 100 48 × 12 = 3 × 12 = 36 ∴x= 16 80 15. × 1450 = 1160 100 16. Let x% of 0.2 = 0.008 x ⇒ × 0.2 = 0.008 100 0.008 × 100 ⇒x=4 ⇒x= 0.2 x 25 17. × = 150 100 2 150 × 100 × 2 ⇒ x= ⇒ x = 1200 25 18. 10% of 15% of 20% of ` 500 10 15 20 = × × × 500 = `1.50 100 100 100 19. Let the number be x. Then, 35% of x = 50% of x − 12 x 35 ×x= × 50 − 12 ⇒ 100 100 ⇒ 35x = 50x − 1200 ⇒ x = 80 20. Let x% of 400 is 60 x i.e., × 400 = 60 100 60 × 100 = 15 ⇒ x = 15 ⇒x= 400 21. Let the original price be ` x per kg. Reduced price = ` (80% of x) 80 4x ×x=` =` 100 5 120 120 − =3 ⇒ 4x x 5 120 × 5 120 − =3 4x x

https://sscstudy.com/

⇒ ⇒

41

3x = (150 − 120) = 30 x = ` 10 per kg

22. Let y earns a % more than x.  25  a% =  × 100 %  100 − 25   25  a%= × 100 %  75  100 a% = % 3 1 ⇒ a % = 33 % 3 23. Let the number be x, then according to question x + 37.5% of x = 33 3 x + × x = 33 8 3  ⇒ x 1 +  = 33  8 11 x× = 33 ⇒ x = 24 8 24. Since 26% ≡ 91 kg 91 Hence, 100% = × 100 26 100% = 350 kg So, 350 kg ore is required. 25. One must reduce travel by x %.  25  Then x% =  × 100  %  100 + 25   25  x% =  × 100 %  125  x% = 20% 26. Total number of failed students is 445 ≡ (100 − 35)% 445 ≡ 65% So, the total number of students appeared for the examination is 455 given by 100% = × 100 65 91 = × 100 = 700 student 13

https://sscstudy.com/

42

CUET (UG) Section III : General Test

CHAPTER 09

Profit, Loss and Discount On sell of an article when a person received greater amount, then the cost price of an article is known as profit and when received less amount, then the price of an article is known as loss. The following basic terms are very useful to solve the problems on profit and loss. Cost Price (CP) The cost price of an article is the price at which the article is bought. Selling Price (SP) The selling price of an article is the price at which the article is sold. Profit or Gain If selling price of an article is more than its cost price, there is profit or gain. Loss If selling price of an article is less than its cost price (CP), there is loss. In fact, Profit = SP − CP; Loss = CP − SP Discount Discount is a term used during a business. It is the rebate given to the buyer by the seller to increase the sale. Discount is always given on marked price or printed price of the article. i.e.,

Discount = Marked/Print Price − Selling Price

Important Tips/Formulae SP − CP Profit Profit percentage = × 100 = × 100 CP CP CP − SP Loss Loss percentage = × 100 = × 100 CP CP SP × 100 SP × 100 (P = profit, L = loss) = CP = (100 + P %) (100 − L %)

A person sells goods at a profit of x%. Had he sold it for ` X more, y% would have been gained. X Then, CP is given by = ` × 100 y−x A person sells goods at a loss of x%. Had he sold it for ` X more, he would have been gained y%. X Then, CP is given by = ` × 100 y+x

CP (100 + P %) CP(100 − L %) = 100 100 Discount Discount percentage = × 100 Marked price

SP =

When there is a gain of x% and a loss of y% the net effect is

If two successive discount of x% and y% is given on an article, 100 − x   100 − y  then Selling Price = Marked Price ×   ×   100   100  xy  Equivalent discount =  x + y −   100  If a false weight is used during selling of an article, then Gain per Error cent = × 100% True value − Error Here, Error = True weight − False weight

given by. xy  Net effect =  x − y − %  100  If CP of x articles is equal to SP of y articles where x > y. x−y Then, profit percentage = × 100 y If CP of x articles is equal to SP of y articles where x < y. y−x Then, loss percentage = × 100 y

https://sscstudy.com/

https://sscstudy.com/

NUMERICAL ABILITY

43

Solved Examples 1. If selling price of an article is 8/5 times its cost

price, the profit per cent on it will be? (a) 50%

(b) 60%

(c) 65%

(d) 70%

8 CP 5 5 SP = 8 ⇒ Profit = 3 unit 3 Profit percentage = × 100 = 60% 5

Sol. (b) SP = ⇒ ⇒

2. A chair is sold for ` 705 at a gain of 6%. Find its

cost price. (a) ` 666

(b) ` 670

(c) ` 665

5 53

(d) ` 680

Sol. (c) Let the CP be ` x. SP = x + x ×

Then But ⇒ ⇒ ∴

(± a × ± b)  Effective change =  ± a ± b + %   100

SP = ` 705 53x = 705 50 5 705 × 50 x= = 665 53 53 5 CP = ` 665 53

a = 20% (+ ve for increase/profit) b = − 20% (− ve for decrease/profit) (− 20 × 20)  = − 4 = 4% loss =  20 − 20 +   100

5. A chair listed at ` 350 is available at successive

3. If the cost price of 9 pens is equal to the selling

price of 11 pens, then what is gain or loss percentage?

(c) 18

2 % 11

Then, marked price is 20% higher than CP 120 × 100 ∴ Marked Price = 120% of CP = = 120 100 Now, 20% discount is given on marked price, then (100 − Discount) × Marked Price SP = 100 (100 − 20) × 120 80 × 120 = = = 96 100 100 ∴ Loss = CP − SP = 100 − 96 = 4 Loss 4 Loss percentage = × 100 = × 100 = 4% CP 100

Alternate Method

6 53x = 100 50

(a) 19%

Sol. (c) Let CP be ` 100.

discounts of 25% and 10%. The selling price of the chair is (a) ` 240.25

(d) ` 230.25

 ± a ± b + (± a × ± b)  − ve for discount     100 + ve for increase a = − 25 ; b = − 10 − 25 × − 10    = − 25 − 10 +      100

then CP of 11 pens = ` 11 SP of 11 pens = CP of 9 pens = ` 9 Then, loss = ` 2 200 2 % × 100 = ∴ Loss percentage = 11 11 2 = 18 % 11

= − 35 + 2.5 = − 32.5 = 32.5% effective discount (100 − discount) × CP SP = 100 (100 − 32.5) × 350 67.5 × 350 ∴ SP = = = ` 236.25 100 100

6. On selling an article for ` 264 a man loses 4%. In

order to gain 12%. Find the selling price of the article.

Alternate Method CP of 9 pens = SP of 11 pens ( y) Clearly, x < y 11 − 9 Then, loss per cent = × 100 11 2 2 = × 100 = 18 % 11 11

(a) ` 300

(b) ` 310

(c) ` 295

(d) ` 308

Sol. (d) Let CP be ` x.

4. The marked price is 20% higher than cost price. A

96 × x = 264 100

Then,

96% of x = 264 ⇒



264 × 100  x =   = 275   96



CP = ` 275, Gain = 12%

discount of 20% is given on the marked price. By this type of sale, there is (b) 4% gain (d) 2% loss

(c) ` 236.25

Sol. (c) Effective discount of 2 successive discounts

8 (b) 17 % 3 2 (d) 19 % 3

Sol. (c) Let CP of each pen be ` 1,

(a) No loss no gain (c) 4% loss

(b) ` 242.25

SP = 112% of ` 275 = 275 × ∴

https://sscstudy.com/

SP = ` 308

112 = 308 100

https://sscstudy.com/

44

CUET (UG) Section III : General Test

Practice Questions 1. If the loss on an article is 5% and its cost price is ` 90, find the selling price. (a) ` 95.50 (c) ` 85

(b) ` 85.50 (d) ` 95

(a) 25 (c) 15

2. A defective TV costing ` 5000 is being sold at a loss of 50%. If the price is further reduced by 50%, then its selling price is (a) ` 1225 (c) ` 1025

(b) ` 1250 (d) ` 1200

(b) ` 2.25 (d) ` 2.50

4. A businessman marks his goods at such price that after allowing a discount of 15%, he makes a profit of 20%. The marked price (in `) of an article having cost price ` 170 is (a) 236

(b) 220

(c) 240

(d) 204

5. Find the rate of discount when marked price is ` 250 and selling price is ` 235. (a) 6.0%

(b) 7.0%

(c) 6.5%

(d) 5.0%

6. The list price of a watch is ` 160. After two successive discounts, it is sold for ` 122.40. If the first discount is 10%, what is the rate of the second discount? (a) 13%

(b) 18%

(c) 16%

(d) 15%

1 1 7. If the discount sales reduce from 3 % to 3 %, what 2 3 difference does it make to purchases for ` 12102? (a) ` 20.17 (c) ` 15.35

(b) ` 16.35 (d) ` 17.35

8. Hundred apples were bought for ` 300. Out of these four were rotten and the rest were sold at ` 50 per dozen. Then, the net profit is (a) ` 150

(b) ` 100

(c) ` 50

(d) ` 125

9. Rekha purchased a scooter for ` 20000 and sold it for ` 22000. The percentage of profit is (a) 15 (c) 10

(b) 12 (d) 20

(b) ` 31500 (d) ` 27000

11. An article listed at ` 26580 is sold at a discount of 10%. Due to festival season the shopkeeper allows a further discount of 5%. Find the selling price of the article. (a) ` 22750.00 (c) ` 22725.90

13. A shopkeeper purchases 10 pieces of certain items for ` 8.00 and sells them at 8 pieces for ` 10.00. The profit percentage of the shopkeeper is

(b) ` 22825.00 (d) ` 23922.00

(b) 56.25 (d) 26.25

14. The marked price is 10% higher than the cost price. A discount of 10% is given on the marked price. In this kind of sale, the seller (a) losses 1.5% (c) gains 1%

(b) bears no loss, makes no gain (d) losses 1%

15. If an article is sold at a gain of 6% instead of at a loss of 6%, then the seller gets ` 6 more. The cost price of the article is (a) ` 106 (c) ` 94

(b) ` 50 (d) ` 100

16. The selling price of 20 articles is equal to the cost price of 22 articles. The gain percentage is (a) 12% (c) 10%

(b) 9% (d) 11%

17. A man gains 10% by selling an article for a certain price. If he sells it at double the price, then the profit made is (a) 120% (c) 40%

(b) 20% (d) 100%

18. The cost price of a book is ` 300. The shopkeeper wants to gain 20% after allowing a discount of 10% on the marked price. Then, the marked price of the book must be (a) ` 360 (c) ` 400

(b) ` 336 (d) ` 396

19. The printed price of a book is ` 60, but the seller allows successive discounts of 20% and 30%. The net sale price is subject to a sales tax of 5%. The net sale price is (a) ` 36.28 (c) ` 36.60

(b) ` 33.60 (d) ` 35.28

3 20. A dealer sold of his articles at a gain of 24% and the 4 remaining at the cost price. Percentage of gain in the whole transaction is

10. The cost price of a refrigerator is ` 28000. The shopkeeper offers a discount of 20% on it and loses 10%. The marked price of the refrigerator is (a) ` 28000 (c) ` 25200

(b) 30 (d) 20

(a) 56.50 (c) 25.50

3. A grocer buys 10 dozens of eggs at ` 18 per dozen from the wholesale market. Out of these, 10 eggs were found broken and had to be thrown away. At what price per egg should he sell them so as to make a profit of 10%, if he spent ` 24 on transportation? (a) ` 2.00 (c) ` 2.04

12. The cost price of 50 cups is equal to the sale price of 40 cups. The percentage of profit in the transaction is

(a) 15 (c) 24

(b) 18 (d) 32

21. The marked price of a watch is ` 1600. The shopkeeper gives successive discounts of 10% and x% to the customer. If the customer pays ` 1224 for the watch, the value of x is (a) 5

(b) 10

(c) 15

(d) 20

22. If the cost price is 95% of the selling price, what is the profit per cent? (a) 4

https://sscstudy.com/

(b) 4.75

(c) 5

(d) 5.26

https://sscstudy.com/

45

NUMERICAL ABILITY

23. The difference between a discount of 35% and two successive discounts of 20% on a certain bill was ` 22. The amount of the bill was (a) ` 200

(b) ` 220

(c) ` 1100

(d) ` 2200

24. If I purchased 11 books for ` 100 and sold 10 books for ` 110, the percentage of profit per book sold is (a) 10

(b) 11.5

(c) 17.3

(d) 21

25. A cloth merchant sold half of his cloth at 40% profit, half of remaining at 40% loss and the rest was sold at the cost price. In the total transaction, his gain or loss will be (a) 20% gain (c) 10% gain

(b) 25% loss (d) 15% loss

26. A bookseller sells a book on 10% profit. If he purchase this book on 4% loss and sells ` 6 more, then he earns 18 3 % profit. Find the cost price of the book. 4 (a) ` 130

(b) ` 140

(c) ` 150

(d) ` 160

27. Ravi purchased two toffees in one rupee of some quantity and sells it five toffees in one rupee. Find the loss per cent. (a) 120

(b) 90

(c) 30

(d) 60

28. The percentage profit earned by selling an article for ` 1920 is equal in the percentage loss incurred by selling the same article for ` 1280. At what price should the article he sold to make 25% profit? (a) ` 2000 (b) ` 2200 (d) Data inadequate (c) ` 24000 29. If 5 lemons are bought for ` 16, then the selling price of a lemon at 25% profit will be (a) ` 5

(b) ` 4

(c) ` 6

(d) ` 8

30. Alfred buys an old scooter for ` 4700 and spends ` 800 on its repairs. If he sells the scooter for ` 5800, his gain per cent is 4 (a) 4 7

5 (b) 5 11

(c) 10

(d) 12

31. A vendor bought toffees at 6 for a rupee. How many for a rupee must be sell to gain 20%? (a) 3

(b) 4

(c) 5

(d) 6

32. It cost ` 1 to photocopy a sheet of paper. However, 2% discount is allowed on all photocopies done after first 1000 sheets. How much will it cost to photocopy 5000 sheets of paper? (a) ` 3920

(b) ` 3980

(c) ` 4900

(d) ` 4920

33. When the selling price of an article is ` 280, the loss percentage is 20%. What is the loss or gain percentage, if the selling price is increased to ` 380? 4 %, profit 7 50 (c) %, profit 3 (a) 8

5 % , loss 7 9 (d) 4 % , profit 11 (b) 4

(b) ` 14.50 (d) ` 17.20

(a) ` 420 (c) ` 500

(b) ` 550 (d) ` 400

36. If the cost price of 36 books is equal to the selling price of 30 books, then the gain percentage is (a) 20%

(b) 16

4 % 6

(c) 16%

(d) 8

2 % 6

37. A man buys 6 dozen eggs for ` 10.80 and 12 eggs are found rotten and the rest are sold at 5 eggs per rupee. Find his gain or loss percentage. 1 (a) 11 % , gain 9 1 (c) 9 % , gain 11

1 (b) 11 %, loss 9 1 (d) 9 %, loss 11

38. Sunanda is making a profit of 25% on his selling price, what is her actual profit percentage? (a)

100 3

(b) 20

(c) 25

(d) 30

39. If a commission of 10% given on the marked price of an article, the gain is 25%. Find the gain per cent, if commission is increased to 20%. (a) 11

1 9

(b) 12

(c)

100 3

(d)

50 3

40. The cost price of an article is 40% of the selling price. The per cent that the selling price is of cost price is (a) 250%

(b) 240%

(c) 60%

(d) 40%

41. A shopkeeper has certain number of eggs of which 5% are found to be broken. He sells 93% of the remainder and still has 266 eggs left. How many eggs did he originally have? (a) 3800 (c) 4200

(b) 4000 (d) None of these

42. A dishonest dealer marks his goods 20% above the cost price. He also makes a profit by using a false weight of 900 g in place of 1 kg while buying or selling. Find the percentage profit earned by the shopkeeper. (a) 20

(b) 12

(c) 42

(d) 46.6

43. Mukul bought 80 kg of rice for ` 1200 and sold it at a loss of as much money as he received for 20 kg rice. At what price per kg did he sell the rice? (a) ` 12 per kg (c) ` 8 per kg

(b) ` 10 per kg (d) ` 11 per kg

44. A furniture shop allows 20% discount on the marked price of each item what price must be marked on a table costing ` 560, so as to make a profit of 25%? (a) ` 800 (c) ` 700

34. A man buys 25 chairs for ` 375 and sells them at a prof it equal to the selling price of 5 chairs. What is the selling price of one chair? (a) ` 18.75 (c) ` 15.20

35. If a man reduces the selling price of a fan from ` 400 to ` 380, his loss increases from x% to ( x + 4) %. What is the cost price of the fan?

(b) ` 825 (d) ` 875

45. A shopkeeper allows a discount of 12.5% on the marked price of a certain article and makes a profit of 20%. If the article costs the shopkeeper ` 210, what price must be marked on the article ? (a) ` 280 (c) ` 300

https://sscstudy.com/

(b) ` 288 (d) None of these

https://sscstudy.com/

46

CUET (UG) Section III : General Test

ANSWERS 1. 11. 21. 31. 41.

(b) (c) (c) (c) (b)

2. 12. 22. 32. 42.

(b) (a) (d) (d) (d)

3. 13. 23. 33. 43.

(c) (b) (d) (a) (a)

4. 14. 24. 34. 44.

(c) (d) (d) (a) (d)

5. 15. 25. 35. 45.

(a) (b) (c) (c) (b)

6. 16. 26. 36.

(d) (c) (c) (a)

7. 17. 27. 37.

(a) (a) (d) (a)

8. 18. 28. 38.

(b) (c) (a) (a)

9. 19. 29. 39.

(c) (d) (b) (a)

10. 20. 30. 40.

(b) (b) (b) (a)

Hints & Solutions 1. Required SP = (100 − 5)% of 90 = 95% of 90 = ` 85.50 2. Selling price of the TV = 50% of 50% of 5000 = ` 1250 3. Total cost price of eggs including transportation cost = 10 × 18 + 24 = 180 + 24 = ` 204 To get 10% profit, their SP = 110% of 204 = ` 224.40 But this is the SP of 110 eggs as 10 eggs were broken. Hence, sale price per egg 224.40 = ` 2.04 = 110 4. Sale price of the article = (100 + 20)% of 170 = ` 204 Now, (100 − 15)% of MP = SP 100 MP = 204 × = ` 240 ⇒ 85 5. Rate of discount 250 − 235 = × 100% 250 15 = × 100% = 6% 250 6. Let the rate of the second discount be r %. Then, (100 − 10)% of (100 = r )% of 160 = 122.40 90 100 − r ⇒ × × 160 = 122.40 100 100 122.40 × 100 × 100 ⇒ 100 − r = 90 × 160 = 85 ⇒ r = 15% 7. Difference is discount 1 1 = 3 % −3 % 3 2

7 10 %− % 2 3 1 = % 6 ∴ Required difference 1 = % of 12102 = ` 20.17 6 =

8. Cost price of 96 apples (8 dozen) = `300 [Q 4 apples were rotten.] Selling price of 96 apples (8 dozen) = 8 × 50 = `400 ∴Required gain = 400 − 300 = ` 100 9. Profit on selling the scooter = 22000 − 20000 = ` 2000 ∴ Required profit percentage 2000 × 100 = = 10% 20000 10. Let the MRP of refrigerator be ` x , then (100 − 20)% of x = 90% of 28000 80% of x = 90% of 28000 ⇒ 80 90 ×x= × 28000 ⇒ 100 100 28000 × 90 ⇒ x= 80 ∴ x = `31500 11. Equivalent discount of 10% and 5% 10 × 5 = 10 + 5 − 100 = 15 − 0. 5 = 14. 5% ∴ Required selling price = (100 − 14.5)% of 26580 26580 × 85.5 100 = ` 22725.90 =

https://sscstudy.com/

12. Required percentage gain x− y = × 100 y 50 − 40 = × 100 40 10 = × 100 = 25% 40 13. Cost price of 1 peice of article = ` 0. 8

Selling price of 1 peice of article 10 = = ` 1.25 8 Profit while selling on a peice of article = 1. 25 − 0. 8 = `0.45 ∴ Required gain percentage 0.45 × 100 = 0.8 = 56.25 % 14. Here, x = 10, y = 10 ∴ Resultant gain or loss xy   = x − y − %  100 10 × 10  = 10 − 10 − %  100  = − 1% Thus, seller losses 1%. 15. Let CP of the article be ` x, then 106% of x − 94% of x = 6 ⇒ ∴

12% of x = 6 6 × 100 x= 12 = ` 50

16. Here, x = 20 and y = 22, then y−x × 100% x 22 − 20 = × 100% 20 = 10%

∴Per cent gain =

https://sscstudy.com/

NUMERICAL ABILITY

17. Let CP of the article be ` x. Then, SP = 110% of x = ` 1.1x If SP be double i.e., 2.2 x, then 2. 2x − x Profit per cent = × 100% x = 120% 18.

CP = ` 300



SP = 120% of 300 = ` 360 100 MP = 360 × 90 = ` 400

19. After discounts, the sale price of the book = 80% of 70% of 60 = ` 33.60 But there is sale tax of 5%, therefore required SP = 105% of 33.60 = ` 35.28 20. Let the CP of articles = ` x By given condition, 3 124 x SP of articles = x × + 4 100 4 472 x =` 400 ∴ Gain percentage 472 x−x = 400 × 100 x 72 = × 100 = 18% 400 21. Marked price = ` 1600 Equivalent discount 10 × x   = 10 + x −  100  9x   = 10 + %  10  and selling price = ` 1224 Q Percentage discount 1600 − 1224  × 100 % =   1600 9x    376  %= × 100 % ⇒ 10 +  1600  10  ⇒

9x = 23 . 5 − 10 100 13 . 5 × 10 x= = 15% 9

22. Required profit per cent  100 − 95 = × 100 %   95 = 5 . 26%

23. Let the amount of the bill be ` x . Equivalent discount of two successive discounts of 20% 20 × 20   = 20 + 20 − % 100   = 36% According to question, 36% of x − 35% of x = ` 22 x × 36 x × 35 x − = 22 = 22 100 100 100 ⇒ x = ` 2200 100 24. CP of 1 book = ` 11 110 SP of 1 book = ` = ` 11 10 ∴Percentage profit SP − CP = × 100 CP 100 11 − 11 × 100 = 100 / 11 = 21% 25. Let CP of cloth be ` 100, then SP of half of his cloth at 40% profit 140 = ` 70 = 50 × 100 1 SP of th of his cloth at 40% loss 4 60 = ` 15 = 25 × 100 1 SP of remain th cloth = ` 25 4 ∴Total SP = ` (70 + 15 + 25) = ` 110 ∴ Required profit 110 − 100 = × 100 = 10% 100 26. Suppose, cost of book = ` 100 ∴ First selling price of the book = 100 + 10 = ` 110 ∴ Reduced cost price of the book 96 = 100 × = ` 96 100 ∴ Second selling price of the book 96 ×  75 = 100 +  100  4 = 96 ×

475 = ` 114 4 × 100

Q Difference of selling price = ` 114 − ` 110 = ` 4

https://sscstudy.com/

47

Q When ` 4 get more, cost price of the book = ` 100 Q When ` 6 get more, cost price 100 of the book = × 6 = ` 150] 4 27. Q Cost price of 2 toffees = ` 1 1 ∴ Cost price of 1 toffee = ` 2 Q Selling price of 5 toffees = ` 1 1 ∴Selling price of 1 toffee = ` 5 1 1 − ∴ Loss per cent = 2 5 × 100 1 2 (5 − 2) × 2 × 100 10 × 1 3 ×2 = × 100 = 60% 10 =

28. Let cost price of article = ` x Profit = 1920 − x 1280 + x = Loss Profit = Loss Q ∴ 1920 − x = 1280 + x 3200 = 2x x = ` 1600 125 Profit on 25 % = 1600 × 100 = ` 2000 29. Cost price of 5 lemons = ` 16 16 Cost price of 1 lemon = ` 5 Selling price of 1 lemon on 25% 16 125 16 5 profit = × = × =`4 5 100 5 4 30. Total cost price of scooter = 4700 + 800 = ` 5500 Selling price of scooter = ` 5800 Profit = 5800 − 5500 = ` 300 Profit × 100 Profit per cent = Cost price 300 × 100 60 = = 5500 11 5 =5 % 11 1 31. Cost price of 1 toffee = ` 6 Selling price of 1 toffee on 20% 1 120 1 6 1 Profit = × = × = 6 100 6 5 5 Hence, vender selling 5 toffees in a rupee.

https://sscstudy.com/

48

CUET (UG) Section III : General Test

32. Cost of first 1000 sheets = ` 1000 Remaining sheets = 5000 − 1000 = 4000 Given, 2% discount on remaining sheets 98 = 4000 × = 40 × 98 100 = ` 3920 Total cost = 1000 + 3920 = `4920 33. The cost price of an article at a 280 × 100 loss of 20% = = ` 350 80 New SP of an article = ` 380 ∴ Profit = 380 − 350 = ` 30 Hence, profit per cent 30 = × 100 350 4 =8 % 7 34. SP of 25 chairs = CP of 25 chairs + Profit SP of 25 chairs = CP of 25 chairs + SP of 5 chairs (Q Profit = SP of 5 chairs) ⇒ SP of 20 chairs = CP of 25 chairs ⇒ SP of 20 chairs = ` 375 ⇒ SP of one chair = ` 18.75 35. Difference of SP = Difference of loss/profit ⇒ 400 − 380 = [(x + 4) − (x)]% 20 = 4% 20 ∴ × 100 100% = 4 = ` 500 36. If the CP of m book is equal to SP of n books, then gain percentage

m−n × 100 n 36 − 30 = × 100 30 = 20% =

37. CP of 60 eggs = ` 10.80 (as 12 eggs were found rotten) SP of 60 eggs = ` 12.00 gain = ` 1.20 ∴ Gain per cent 1.2 100 % = × 100 = 10.8 9 1 = 11 % 9 38. Let the selling price = ` 100. If profit is ` 25 on SP, the cost price = ` 75 Actual profit per cent Profit = × 100 CP 25 = × 100 75 100 % = 3 39. Gain per cent = (100 + % first profit) 100 − % 2nd discount  − 100  100 − % 1st discount   100 − 20 = (100 + 25)   − 100  100 − 10 80 = 125 × − 100 90 1 100 = = 11 % 9 9 40. The CP will be ` 40, if the SP is ` 100. Percentage of SP to CP 100 = × 100 = 250% 40 41. Let the shopkeeper has x eggs.

https://sscstudy.com/

 100 − 5  100 − 93 ∴x×  = 266    100   100  19 7 × = 266 20 100 ⇒ x = 4000 ⇒ x×

42. The dealer uses a false weight in buying as well as selling. It means that he buys 1100 g and sells 900 g in place of 1 kg. Now, let the CP of 1100 g be ` 1000. Marked price of 1100 g of goods = ` 1200 (20% up), and this is the SP of 900 g of goods. CP of 900 g = ` 818.18 Profit = ` 381.82 and profit per cent 381.82 = × 100 = 46.6% 818.18 43. SP of 80 kg of rice = CP of 80 kg − loss SP of 80 kg rice = 1200 − SP of 20 kg of rice SP of 100 kg of rice = ` 1200 ∴ SP of 1 kg of rice = ` 12 44. CP of the table = ` 560 SP of the table at a profit of 25% = ` 700 Since SP is arrived after a discount of 20% on marked price 700 = ` 875 ∴ Marked price = 0.8 45. Cost price of the article = ` 210 Selling price at a profit of 20% = ` 252 ∴ MP × 0.875 = ` 252 252 MP = ` = ` 288 ⇒ 0.875

https://sscstudy.com/

NUMERICAL ABILITY

49

CHAPTER 10

Simple and Compound Interest Interest is the money paid by the borrower to the lender for the use of money but the interest calculated at principal borrowed known as simple interest and the interest calculated after addition of principal and interest known as compound interest.

Simple Interest Interest is the amount paid by the borrower to the lender for using his money. When the interest is calculated uniformly on the amount throghout the loan period, then that is simple interest. The amount on which simple interest is calculated is known as principal. PTR SI = 100 Where, SI → Simple Interest T → Time P → Principal on which interest is calculated R → Rate Amount ( A) = P + SI

Compound Interest It is the interest calculated on a sum of money which includes principal and interest calculated for previous year. The SI and CI for first year is same and for second and subsequent years differ by an amount which is arrived by calculating interest on interest for previous years.

When Interest added half yearly and time, rate is given in yearly, then time becomes double and rate becomes half. When time and rate is given in yearly and interest added quarterly, then rate becomes 1/4 of yearly rate and times becomes four times of yearly. The following table will illustrate the conceptual working of simple interest and compound interest. Rate of interest per annum is 10% For the year

Simple Interest

Compound Interest

Principal

SI

1000

100

1000

100

2

1000

100

1000 + 100 = 1100

110

3

1000

100

1100 + 110 = 1210

121

1

Principal

CI

On the basis of above calculation It is clear that (i) Simple interest for each year is constant. (ii) Compound interest calculated for each year includes—simple interest on principal and simple interest on interest calculated for previous year.

Important Tips/Formulae Let principal = ` P, Rate = R % per annum, Time = n yr (i) When interest is compounded annually, then R  Amount = P  1 +   100 

R R R Amount = P  1 + 1   1 + 2   1 + 3   100   100   100 

n

Difference between CI and SI for 2 yr = Difference between CI and SI for 3 yr =

(ii) When interest is annually and rate of interest be R1 % during 1st year, R2 % during 2nd year, R3 % during 3rd year.

P × r2 ( 300 + r ) ( 100 )

3

P × r2 2

( 100 )

(iii) When interest is half-yearly Amount 2n R  = P ×  1 +   2 × 100  (iv) When interest is quarterly. 4n R  Amount = P ×  1 +   4 × 100 

https://sscstudy.com/

https://sscstudy.com/

50

CUET (UG) Section III : General Test

Solved Examples 1. A sum doubles in 20 yr at simple interest. How

much is the rate? (a) 5% per annum (c) 5.5% per annum

(b) 8% per annum (d) 8.5% per annum

Sol. (a) Let sum = P

3. The simple interest on a certain sum of money for

1 2 1 yr at 8 % per annum is ` 560. Then, find the sum. 3 3 (a) ` 2880

(b) ` 2800

(c) ` 8880

(d) ` 8280

Sol. (a) Let the sum be ` x. Then,

Then, amount = 2P, SI = 2P − P = P SI × 100 R= ∴ P ×T P × 100 = P × 20

25 1 7 7 × × = 560 ⇒ x × = 560 3 100 3 36 560 × 36 x= = 2880 ⇒ x = ` 2880 7

x× ⇒

4. Find the compound interest on ` 10000 in 2 yr at

4% per annum (p.a.), the interest being compounded half-yearly.

R = 5% per annum

2. Find simple interest on ` 4800 at 8% per annum

(a) ` 836

(b) ` 824.32

(c) ` 324

(d) ` 868

Sol. (b) Principal = ` 10000, Rate = 2% per half-year

for 10 months. (a) ` 400

(b) ` 420

(c) ` 320

(d) ` 510

Time = 2 yr = 4 half-year 4  2   Amount = 10000 ×  1 + ∴    100    51 51 51 51 = 10000 × × × × 50 50 50 50 = ` 10824.32 CI = ` 10824.32 − 10000 = ` 824.32 ∴

Sol. (c) P = ` 4800, R = 8% per annum, 10 5 yr = yr 12 6 P × R × T 4800 × 8 × 5 = ` 320 = ∴ SI = 100 6 × 100 T =

Practice Questions 1. If a sum of money at simple interest doubles in

5 yr, it will become 4 times in how many years? (a) 18 (c) 15

(b) 16 (d) 20

2. A certain sum of money becomes ` 2250 at the end

of 2 yr and becomes ` 2625 at the end of 5 yr. If the person receives only simple interest, then the rate of interest is (a) 6.25% (c) 8%

6. A sum of ` 8448 is to be divided between A and B

who are respectively 18 and 19 yr old, in such a way that if their shares be invested at 6.25% per annum compound interest, they will receive equal amounts on attaining the age of 21 yr. The present share of A is (a) ` 4225 (c) ` 4096

(b) ` 4352 (d) ` 4000

7. On a certain sum of money, the difference between

(b) 5% (d) 6.5%

for 4 yr. Had it been put at 3% higher rate it would have fetched ` 1440 more. Find the sum.

the compound interest for a year, payable half-yearly, and the simple interest for a year is ` 180. If the rate of interest in both the cases is 10%, then the sum is

(a) ` 14400 (c) ` 10000

(a) ` 60000 (c) ` 62000

3. A sum was put at simple interest at a certain rate

(b) ` 11000 (d) ` 12000

4. If the compound interest on a certain sum for 2 yr

at 4% is ` 102, what would be the simple interest at the same rate for 2 yr? (a) ` 75 (c) ` 100

(b) ` 100.50 (d) ` 98

5. A sum of money amounts to ` 2240 at 4% per

annum simple interest in 3 yr. The interest on the same sum for 6 months at 3.5% per annum is (a) ` 30 (c) ` 35

(b) ` 50 (d) ` 150

(b) ` 72000 (d) ` 54000

8. In a certain time, the ratio of a certain principal

and the simple interest obtained from it are in the ratio 10 : 3 at 10% interest per annum. The number of years the money was invested is (a) 1 (c) 5

(b) 3 (d) 7

9. Find the simple interest due after 120 days for

` 4800 at 10%. (a) ` 157.80 (c) ` 156.01

https://sscstudy.com/

(b) ` 157 (d) ` 124.93

https://sscstudy.com/

51

NUMERICAL ABILITY

10. Find the rate of interest at which ` 100 becomes

18. At what rate per cent compound interest, will ` 400

amount to ` 441 in 2 yr?

` 200 in 10 yr.

(a) 10% (c) 8%

(a) 4% (c) 6%

(b) 15% (d) 17%

11. Rina borrows a loan of ` 1200 at simple interest

19. Find the simple interest on ` 1600 at 6% per

annum for 146 days.

and the number of years is equal to the rate per cent per annum and she give ` 432 as interest. Find the rate of interest. (a) (b) (c) (d)

(a) ` 42.50 (c) ` 38.40

interest to obtain a total amount of ` 12710. For how many years did he invest the sum? (a) 6 (c) 5

12. A man took loan from a bank at the rate of 12%

par annum simple interest. After 3yr he had to pay ` 5400 interest only for the period. The principal amount borrowed by him was (b) ` 10000 (d) ` 20000

(b) 8 (d) 4

21. The simple interest on ` 2000 for 7 months at

5 paise per rupee per month is (a) ` 700 (c) ` 350

(b) ` 70 (d) ` 305

22. A certain sum is invested at simple interest. If it

13. A sum of money at simple interest amount to ` 815 in 3 yr and to ` 854 in 4 yr. The sum is (a) ` 650 (b) ` 690 (c) ` 698 (d) ` 700

trebles in 10 yr, what is the rate of interest? (a) 18% per annum (c) 22% per annum

to ` 4320 in 2 1 yr and to ` 4752 in 4 yr. The rate of 2 interest per annum is

an interest of ` 1350 is to be earned in 5 yr, then the amount of money deposited is (b) ` 4500 (d) ` 4800

(b) 20% per annum (d) 25% per annum

23. A certain sum of money at simple interest amounts

14. The rate of interest is 6% per annum in a bank. If

(a) ` 13500 (c) ` 5400

(b) ` 30.60 (d) ` 40

20. Arun invests ` 10250 at 4% per annum simple

3.6% 6% 18% Cannot be determined

(a) ` 2000 (c) ` 15000

(b) 5% (d) 3%

1 (b) 8 % 2

(a) 8%

1 (c) 7 % 2

(d) 9%

24. A lent be ` 5000 to B for 2 yr and ` 3000 to C for

15. A man borrows ` 50000 at 4% compound interest

per annum. Then, the total amount of money he has to pay after 2 yr is

4 yr on simple interest at the same rate of interest and received ` 2200 in all from both as interest. The rate of interest per annum is

(a) ` 54080 (c) ` 54000

(a) 8% (c) 10%

(b) ` 50480 (d) ` 54800

16. Sudha borrowed ` 400 from her friend at the rate

25. The simple interest on ` 7380 from 11 May, 1987 to

of 12% per annum for 2 1 yr. The interest and the 2 amount paid by her were (a) ` 140, ` 540 (c) ` 125, ` 525

(b) ` 130, ` 530 (d) ` 120, ` 520

11 September, 1987 at 5% per annum. (a) ` 123 (c) ` 200

(b) ` 103 (d) ` 223

1 1 of his capital at 8%, at 9% and 3 4 the remainder at 10%. If his annual income is ` 1180, the whole capital is

26. A man invested

17. The compound interest on a sum at the rate of 5%

for 2 yr is ` 512.50. The sum is (a) ` 5200 (c) ` 5000

(b) 9% (d) 12%

(a) ` 9000 (c) ` 15000

(b) ` 4800 (d) ` 5500

(b) ` 12000 (d) ` 13500

ANSWERS 1. (c) 11. (b) 21. (a)

2. (a) 12. (c) 22. (b)

3. (d) 13. (c) 23. (a)

4. (c) 14. (b) 24. (c)

5. (c) 15. (a) 25. (a)

6. (b) 16. (d) 26. (b)

https://sscstudy.com/

7. (b) 17. (c)

8. (b) 18. (b)

9. (a) 19. (c)

10. (a) 20. (a)

https://sscstudy.com/

52

CUET (UG) Section III : General Test

Hints & Solutions 1. Here, n1 = 2, t1 = 5, n2 = 4, t2 = ? t n −1 5 1 Using, 1 = 1 ⇒ = t2 n2 − 1 t2 3 ⇒

t2 = 15 yr

3. Let the principal be ` P and rate of interest r % per annum. Then, according to question, P × (r + 3) × 4 P × r × 4 − = 1440 100 100 ⇒ 4P + 12P − 4 Pr = 1440 × 100 ⇒ 12 P = 1440 × 100 ⇒ P = 120 × 100 ⇒ P = ` 12000 4. Let the principal be ` P. According to question, 4   102 + P = P 1 +   100

2

2

 26 102 + P = P    25 P × 676 102 + P = ⇒ 625 676P − 625P ⇒ 102 = 625 ⇒ 51P = 108 × 625 P = `1250 Required single interest 1250 × 4 × 2 = = `100 100 ⇒

5. If the sum be ` P, then P ×4 ×3 2240 − P = 100 12P 2240 = +P ⇒ 100 112 ⇒ P 2240 = 100 2240 × 100 ∴ P= = ` 2000 112 Now, required interest, 7 1 1 Prt SI = = 2000 × × × 2 2 100 100 = ` 35 6. Let shares of A and B are ` x and ` (8448 − x), respectively. Amount got by A after 3 yr = Amount got by B after 2 yr 3 6. 25  x 1 +   100  6. 25  = (8448 − x) 1 +   100 

4

⇒ ⇒ ⇒ ⇒ ⇒ ∴

x =1 + 8448 − x x =1 + 8448 − x

6. 25 100 1 17 = 16 16

16x = 17 (8448 − x) 16x = 143616 − 17x 33x = 143616 x = ` 4352

7. Let the sum be ` P. Compound interest 2   41 5  = P 1 + ×P  − 1 =   100   400 and simple interest P × 10 × 1 1 = = ×P 100 10 Given, CI − SI = ` 180 41 1 ×P− × P = 180 100 10 P = 180 × 400 = ` 72000 8. Let time = t yr ∴ Q

P = 10x and SI = 3x Prt SI = 100 10x × 10 × t 3x = 100 t = 3 yr

9. Given, principal = ` 4800 Rate = 10%, Time = 120 days P × R×T Simple interest = 100 4800 × 10 × 120 = 100 × 365 = ` 157. 8 10. Principal = ` 100, Time = 10 yr Amount = ` 200, Simple interest = 200 − 100 = ` 100 SI × 100 100 × 100 Rate = = 10% = 100 × 10 P ×T 11. Given, time = rate of interest = r 1200 × r × r Then, 432 = 100 432 × 100 2 ⇒ r = 1200 ⇒ ⇒ ∴

r 2 = 36 r = 36 r = 6%

https://sscstudy.com/

12. Let amount of loan = ` x Simple interest Amount × Rate × Time 100 x × 12 × 3 5400 = 100 5400 × 100 x= 36 x = ` 15000

=

13. Let amount = ` x Let interest rate = r % Simple interest in 3 yr, x × r ×3 ...(i) ⇒ 815 − x = 100 Simple interest in 4 yr x × r ×4 ...(ii) ⇒ 854 − x = 100 On dividing Eq. (i) by Eq. (ii), 815 − x 3 = ⇒ 854 − x 4 ⇒ 3260 − 4x = 2562 − 3x ∴ x = ` 698 14. Given, Rate = 6% per annum, Time = 5 yr Interest = ` 1350 Let amount = ` x Then, Simple Interest

=

Amount × Rate × Time 100

x×6 ×5 100 1350 × 100 ⇒ =x 30 ⇒ x = 45 × 100 = ` 4500 ⇒ 1350 =

15. Given, Rate = 4% Amount = ` 50000 Time = 2 yr Then, the total amount of money 2 4   = 50000 1 +   100 26 26 = 50000 × × 25 25 = 80 × 676 = ` 54080 400 × 5 × 12 16. SI = = ` 120 2 × 100 A = P + SI = (400 + 120) = ` 520

https://sscstudy.com/

NUMERICAL ABILITY

2   R  17. CI = P 1 +  − 1  100   2   5  ⇒ 12.50 = P 1 +  − 1  100   512 .50 × 400 P= ∴ 41 = ` 5000

18. P = ` 400, A = ` 441, n = 2 yr R=? n R   A = ? 1 +   100 ⇒

R   441 = 400 1 +   100



441  R  = 1 +  400  100



R   21    = 1 +   20  100

2

⇒ ⇒ ⇒ ⇒

2

2

2

21  R  = 1 +  20  100 R 21 = −1 100 20 R 1 = 100 20 R = 5%

19. Here, P = ` 1600, R = 6 % per 146 annum and T = yr 365 P × R ×T ∴ SI = 100 6 146  = 1600 × ×   100 365

192 5 = ` 38.40 =

20. P = ` 10250, R = 4% per annum, SI = ` (12710 − 10250) = ` 2460 100 × SI Now, T = P×R 100 × 2460 = 10250 × 4 = 6 yr ∴ T = 6 yr 21. SI on ` 1 for 1 yr = (5 × 12) paise = 60 paise SI on ` 100 for 1 yr  60  = × 100  100  = ` 60 Now, P = ` 2000, R = 60 % 7 yr and T = 12  2000 × 60 × 7 SI =  ∴   100 × 12  = ` 700 22. Let the sum be ` x. Then, SI = ` (3x − x) = ` 2x x × R × 10 ∴ 2x = 100 ∴ R = 20 % per annum 24. Suppose rate of interest per annum = r% According to the question, 5000 × 2 × r 3000 × 4 × r + = 2200 100 100

https://sscstudy.com/

53

⇒ 100 r + 120 r = 2200 ⇒ 220 r = 2200 2200 ⇒ r= 220 = 10% 7380 × 5 × 1 = ` 123 100 × 3 4 1   = yr T = 4 months =  12 3 

25. SI =

26. Let the capital be ` x. x Then, Ist capital = ` 3 x IInd capital = ` 4   x x  IIIrd capital = ` x −  +     3 4  5x =` 12 8 9 x  x  × 1 +  × × 1 ∴ ×  3 100   3 100   5x 10  + × × 1 = 1180  12 100  ⇒

2x 3x x + + = 1180 75 100 24



16x + 18x + 25x = 600 × 1180

⇒ 59x = 600 × 1180 ⇒

x=

600 × 1180 59

= 12000 ∴ Capital = ` 12000

https://sscstudy.com/

54

CUET (UG) Section III : General Test

CHAPTER 11

Mixture and Alligation When two or more than two pure substances are mixed in a certain ratio, they create a mixture. Here, we shall confine ourselves to mostly homogeneous mixtures in view of the question commonly asked.

Mixture The new product obtained by mixing two or more ingredients in a certain ratio is called a mixture of those particular ingredients.

Mean Price

e.g. A mixture of a certain quantity of milk with 16 L of water is worth ` 0.75 per litre. If pure milk be worth ` 2.25 per litre, how much milk is there in the mixture? Sol. According to the rule of alligation,

The cost price of a unit quantity of the mixture is called the mean price.

Water (0)

Rules of Mixture or Alligation

Mean price 0.75

This rule of mixture is used to determine the mean price of the mixture, when the prices of the individual items being mixed together and the proportion in which they are being mixed are given. When two ingredients at given prices are known, then the ratio in which these two are mixed to obtain a mixture of known price is given by Amount of cheaper Cost price of dearer – Mean price = Amount of dearer Mean price – Cost price of cheaper

2.25 – 0.75 = 1.5

(D-M)

CP of dearer (per unit) (D) Mean price (M)

(M -C)

Hence, Amount of cheaper : Amount of dearer = (D − M ) : (M − C)

0.75 – 0 = 0.75



Water : Milk = 1.5 : 0.75 = 2 : 1 1 Clearly, quantity of milk = of water 2 1 = × 16 2 = 8L

It is also known as rule of alligation. It can also be expressed, as CP of cheaper (per unit) (C)

Milk (2.25)

Keep in Mind! The value of the mixture is always higher than the lowest value and lower than the highest value of the items being mixed. Rule of mixture or alligation is applied for rate, ratio, percentage value, speed, prices etc. and not for absolute values. In other words, whenever per cent, per hour, per km, per kg etc. are being compared, we can use this method.

https://sscstudy.com/

https://sscstudy.com/

NUMERICAL ABILITY

55

Solved Examples 1. Tea worth ` 126 per kg and ` 135 per kg are mixed

with a third variety in the ratio 1 : 1 : 2 . If the mixture is worth ` 153 per kg. The price of the third variety per kg will be (a) ` 169.5 (c) ` 175.5

Ratio in which they are mixed 8 13 9 65 – 63 91

(b) ` 170.0 (d) ` 180.0

2 2 13 2 2 1 91 = = = × = = 2: 7 : 1 91 1 91 13 7 13

Sol. (c) Let the common ratio be x. Price of 3rd variety of tea = ` p Now, by given condition, 126 × x + 135 × x + p × 2x = 153 (x + x + 2x) ⇒ 126x + 135x + p × 2x = 153 × 4x ⇒ 261x + p × 2x = 612x ⇒ p × 2x = 612x − 261x 351x p= ∴ 2x ⇒ p = ` 175.50

3. A mixture of milk and water is such that the

quantity of milk is 3 that of water. The proportion 5 of milk in the mixture is (a) 1/8

2. Two vessels A and B contain milk and water mixed

in the ratio 8 : 5 and 5 : 2 , respectively. The ratio in which these two mixtures be mixed to get a new 3 mixture containing 69 % milk is 13 (a) 2 : 7 (c) 5 : 2

(b) 3 : 5 (d) 5 : 7

Sol. (a) Total ratio of milk in vessel A =

13

5 7 9 – 8 13 13

8 13

(b) 1/2 (c) 3/8 3 Sol. (d) Milk = water ⇒ 5 milk = 3 water 5 Total mixture = 5 + 3 = 8 units 5 Portion of milk in mixture = 8

4. Two solutions of 90% and 97% purity are mixed,

resulting in 21 L of mixture of 94% purity. The quantity of the second solution in the resulting mixture, in litres, is (a) 15

5 Total ratio of milk in vessel B = 7 3 69 13 Ratio of milk in new mixture = 100 900 9 = = 13 × 100 13

(d) 5/8

(b) 12

(c) 9

(d) 6

Sol. (b) By mixture method, 90%

97% 94%

3% 4% of second solution in 21L of mixture ∴Quantity 4 = × 21 = 12 L 7

Practice Questions 1. A mixture of 20 kg of spirit and water contains

10% water. After adding a certain amount of water, the weight of the new mixture is 25 kg. What is the percentage of water in the new mixture? (a) 18 (c) 12.5

(b) 28 (d) 15

2. 60 kg of a certain variety of rice at ` 32 per kg is

mixed with 48 kg of another variety of rice and the mixture is sold at the average price of ` 28 per kg. If there be no profit or loss due to the new sale price, then the price of the second variety of rice is (a) ` 25.60 per kg (c) ` 23 per kg

(b) ` 25 per kg (d) ` 30 per kg

3. By mixing two different quantities of pulses in the

ratio 2 : 3 and selling the mixture at the rate of ` 22 per kg a shopkeeper makes a profit of 10%. If the cost of the smaller quantity be ` 14 per kg, then the cost per kg of the larger quantity is (a) ` 23 (b) ` 24 (c) ` 25 (d) None of the above

4. A drum contain 20 L of a paint. From this 2 L of

paint is taken out and replaced by 2 L of oil. Again 2 L of this mixture is replaced by 2 L of oil. If the operation is performed once again, then final ratio of the paint and the oil in the drum would be (a) 729 : 271 (c) 3 : 7

https://sscstudy.com/

(b) 172 : 279 (d) 217 : 972

https://sscstudy.com/

56

CUET (UG) Section III : General Test

5. Several litres of acid were drawn off a 54 L vessel

10. Atul bought 30 kg of rice at ` 8.50 per kg and 20 kg

full of acid and an equal amount of water added. Again the same volume of the mixture was drawn off and replaced by water. As a result, the vessel contained 24 L of pure acid. How much of the acid was draw off initially? (a) 12 L (c) 18 L

(b) 16 L (d) 24 L

of rice at ` 8.00 per kg. If he has to make a 20% profit, at approximately, what rate per kg should he sell the rice? (a) ` 10.00 (c) ` 8.50

11. Raghav buys milk at a certain price and after

mixing it with water sells it again at the same price. How many of water he mixes in a litres of milk, if he makes a profit of 20%?

6. There are three containers of equal capacity. The

ratio of sulphuric acid to water in the first container is 3 : 2, that in the second container is 7 : 3 and in the third container 11: 4. If all the liquids are mixed together, then the ratio of sulphuric acid to water in the mixture will be (a) 61 : 29 (c) 60 : 29

(a) 200 mL (c) 150 mL

with vegetable oil costing ` 50 per kg. A shopkeeper sells the mixture at ` 96 per kg, thereby making a profit of 20%. In what ratio does he mix the two?

(b) 61 : 28 (d) 59 : 29

are water and 10 part milk. How much of the mixture must be drawn off and replaced with water so that the mixture may be half water and half milk? 1 3

(b)

1 7

(c)

1 5

(d)

1 8

(a) 1 : 2 (c) 3 : 1

` 5 per kg respectively are mixed in order to produce a mixture having the rate of ` 4.60 per kg. What should be the amount of the second type of oil if the amount of the first type of oil in the mixture is 40 kg?

after mixing it with water sells it again at the same price. How many of water does he mix in every litre of milk if he makes a profit of 25%?

(a) 75 kg (c) 60 kg

(b) 200 mL (d) 30 mL

9. A barrel contains a mixture of wine and water in

1 4

(b)

1 3

(c)

1 2

(d)

(b) 50 kg (d) 40 kg

14. 400 students took a mock exam in Delhi 60% of the

boys and 80% of the girls cleared the cut off in the examination. If the total percentage of students qualifying is 65%, how many girls appeared in the examination?

the ratio 3 : 1. How much fraction of the mixture must be drawn off and substituted by water so that the ratio of wine and water in the resultant mixture in the barrel become 1 : 1? (a)

(b) 3 : 2 (d) 2 : 3

13. Two types of oils having the rates of ` 4 per kg and

8. A man buys milk at a certain price per litre and

(a) 250 mL (c) 150 mL

(b) 250 mL (d) 20 mL

12. Pure ghee costs ` 100 per kg. After adulterating it

7. A container is filled with liquid, 6 part of which

(a)

(b) ` 12.00 (d) ` 8.00

(a) 100 (c) 150

2 3

(b) 120 (d) 300

ANSWERS 1. (b) 11. (a)

2. (c) 12. (b)

3. (b) 13. (c)

4. (a) 14. (a)

5. (c)

6. (a)

7. (c)

8. (a)

9. (a)

10. (a)

Hints & Solutions 1. Quantity of water in mixture of 20 kg = 10% of 20 kg = 2 kg Weight of new mixture = 25 kg Quantity of water in new mixture = 25 − (20 − 2) = 25 − 18 = 7 kg 7 × 100 ∴ Required percentage = 25 = 7 × 4 = 28%

2. Let price of the second variety of rice be ` x per kg. Then, total cost of first variety of rice = 60 × 32 = ` 1920 and total cost of second variety of rice = 48 × x = ` 48x ∴Total SP of both varities of rice = (60 + 48) × 28 = 108 × 28 = ` 3024 Since, there is neither profit nor loss, therefore,

https://sscstudy.com/

https://sscstudy.com/

NUMERICAL ABILITY

1920 + 48x = 3024 ⇒ 48x = 1104 ∴ x = 23 Thus, price of second variety of rice is ` 23 per kg. 3. Let the total quantity of pulses = 5 kg SP of the mixture= ` 22 × 5 = ` 110 Profit = 10% Q ∴ CP of the mixture = ` 100 Q Cost of the first quantity = 2 × 14 = ` 28 Cost of the second quality = ` 72 72 ∴ CP of the second quality = 3 = ` 24 4. The quantity of paint = 18 L and quantity of oil = 2 L Now, 2 L of mixture is taken out and 2 L of oil is mixed. ∴Now, the quantity of paint 9 81 L = 18 − = 5 5 and quantity of oil 1 19 L =2 + 2 − = 5 5 Now, this process is performed again. Then, the quantity of paint 81 81 729 L = − = 5 50 50 and quantity of oil 19 19 271 L = − +2= 5 50 50 Hence, the ratio of paint and oil = 729 : 271 5. Let x L of several litres of acid were drawn off initially ∴ Remaining acid in the vessel = (54 − x) L and quantity of water in the vessel = x L Now, x L of mixture is drawn off ∴ Quantity of acid drawn off  54 − x  = × x L  54  and quantity of water drawn off x2 L = 54 Now, the quantity of acid   54 − x  = 54 − x −   x L  54   

(54 − x) x = 24 54 ⇒ x2 − 108x + 1620 = 0 ⇒ x = 90, 18 ∴ 90 > 54 So, x = 90 is ruled out Hence, x = 18 6. Amount of Sulphuric acid in the 3 7 11 61 new mixture = + + = 5 10 15 30 ∴

54 − x −

and amount of water in the new 2 3 4 29 mixture = + + = 5 10 15 30 Ratio of sulphuric acid and water in the new mixture 61 29 = = 61 : 29 : 30 30 7. Let the container initially contains 16 L of liquid. Let ‘a’ L of liquid be replaced with water. Quantity of water in the new 6a   mixture = 6 − + a L   16 Quantity of milk in the new 10a   mixture = 10 −  L  16  10a 6a ∴ + a = 10 − 6− 16 16 ⇒ 96 − 6a + 16a = 160 − 10a ⇒ 96 + 10a = 160 − 10a ⇒ 20a = 64 64 16 a= = ⇒ 20 5 ∴ Part of mixture replaced 1 16 1 = × = 16 5 5 8. Let 1 L of milk is bought for ` 810. Let x L of water is added to it, so that (1 + x) L of the mixture is sold at ` 10 per litre. ∴ CP of (1 + x) L = ` 10 and SP of (1 + x) L = ` 10 (1 + x) Profit per cent 1 ∴ 100x = 25 ⇒ x = 4 ∴ 250 mL of water should be mixed in every litre of milk. x−8 30 10. = 20 8. 50 − x Where, x is the CP of the mix per kg. ⇒ 25.50 − 3x = 2x − 15 ⇒ 5x = 25. 50 + 16

https://sscstudy.com/

57

41. 50 = ` 8. 30 5 ∴ SP of mix per kg at 20% profit 8. 30 × 120 = = 9. 96 100 = ` 10.00 (approx.)



x=

11. Let the price of milk per litre be ` 1. ∴ SP of adulterated milk per litres = ` 1 ∴ CP of adulterated milk per litres 1 × 100 5 = =` (100 + 20) 6 5 1 1− Quantity of water 6 =6 Q = Quantity of milk 5 − 0 5 6 6 ∴ Quantity of water with 1 L of milk = 200 mL 12. CP of mixture per kg 100 × 96 = ` 80 = (100 + 20) Quantity of pure ghee Quantity of vegetable oil 50 − 80 = 80 − 100 30 = 3 :2 ∴ Required ratio = 20 13. Given, first type of oil in the mixture = 40 kg By mixture and alligation rule, Type-I `4

Type-II `5 ` 4.6 0.6

0.4

∴ Required ratio = 2 : 3 Let the quantity of type-I and type-II be 2x and 3x respectively. 2x = 40 x = 20 ∴ Quantity of second type oil = 3 × 20 = 60 kg 14. Given, total students = 400 By mixture and alligation rule, Girls 80%

Boys 60% 65% 15

5

∴ Required ratio = 3 : 1 Number of girls appeared in 1 examination = × 400 = 100 4

https://sscstudy.com/

58

CUET (UG) Section III : General Test

CHAPTER 12

Time and Work In this chapter conceptual clarity of relationship between working efficiency and time is very important to understand. Working efficiency is the work done by an individual in one day and this efficiency is inversely proportional to the number of days to complete a work. It means that a person who takes less days to complete a work is said to be more efficient than a person who takes more days to complete the same work.

Important Tips/Formulae If A can do a piece of work in n days, then the piece of work completed by A in one day is

1 . n

 nm  If A complete a piece of work in n days and B in m days, then they together can complete the work in   days.  n + m 1 1 If a pipe A can fill a tank in x h and a pipe B can empty the full tank in y h (where y > x), then net part filled in 1 h =  −  . x y M1 D1 H1 M2 D2 H2 ; Here M1 , M2 = Number of men = W1 W2 D1 , D2 = Number of days, H1 , H2 = Number of hours and W1 , W2 = Works

Solved Examples 1. If A and B together can finish a piece of work in

20 days, B and C in 10 days and C and A in 12 days, then A, B and C jointly can finish the same work in 2 days 7 4 (c) 8 days 7 (a) 4

(b) 30 days (d)

7 days 60

1 20 1 (B + C )’s 1 day work = 10 1 (C + A )’s 1 day work = 12 1 1 1 2(A + B + C )’s 1 day work = + + 20 10 12 3+ 6+ 5 = 60 14 7 = = 60 30

Sol. (c) (A + B )’s 1 day work =

7 1 × 30 2 7 = 60

∴ (A + B + C )’s 1 day work =

Hence, A, B and C can finish the work together in 4 or 8 days. 7

60 days 7

2. In a fort there was sufficient food for 200 soldiers

for 31 days. After 27 days 120 soldiers left the fort. For how many extra days will the rest of the food last for the remaining soldiers? (a) 10 (c) 12

(b) 8 (d) 9

Sol. (a) Suppose extra days = x Then, 200 × 31 = 200 × 27 + 80 × x ⇒ 6200 = 5400 + 80x 800 x= = 10 days ⇒ 80

https://sscstudy.com/

https://sscstudy.com/

59

NUMERICAL ABILITY

Alternate Method

4. Two pipes A and B can fill a cistern in 12 min and

16 min, respectively. Both are opened together at the end of 4 min, B is turned off. In how much time will the cistern be full?

Required number of days 200 (31 − 27) i.e., D= 200 − 120 200 × 4 = 10 days = 80

(a) 8 min

3. If 10 men or 20 boys can make 260 mats in 20

days, then how many mats will be made by 8 men and 4 boys in 20 days? (a) 250 (c) 255

(b) 10 min

(c) 9 min

(d) 11 min

Sol. (c) Part filled by (A + B ) in 4 min

(b) 280 (d) 260

Sol. (d) Let the number of mats be x. 10 men ≡ 20 boys ⇒ (1 man ≡ 2 boys) (8 men + 4 boys) ≡ (8 × 2 + 4) ≡ 20 boys ⇒ In 20 days, 20 boys make 260 mats.

7 7 1 1 =  + × 4=  ×4=  12 16  12 48 7 5 Remaining part =  1 −  =  12  12 1 part is filled by A in 1 min. 12 5 5 part is filled by A in  12 ×  = 5 min  12 12  ∴Total time taken to fill the cistern = (4 + 5) = 9 min

Practice Questions 1. 30 men can produce 1500 units in 24 days working

6 h a day. In how many days, can 18 men produce 1800 units working 8 h a day? (a) 18

(b) 32

(c) 36

(d) 45

2. 4 men and 6 childern can complete a work in

8 days while 3 men and 7 childern can complete the same work in 10 days. If 20 childern only work, the work will be completed in (a) 20 days (c) 15 days

(b) 22 days (d) 18 days

together but B left 2 days before the completion of work and C left 5 days before the completion of work. The share of A from the assured money is (a) ` 2700 (c) ` 1800

7. A and B can do a piece of work in 72 days, B and C

can do it in 120 days, and A and C can do it in 90 days. When A, B and C work together, how much work is finished by them in 3 days? (a)

3. A can do a piece of work in 10 days and B can do the

same piece of work in 20 days. They start the work together, but after 5 days A leaves. B will do the remaining piece of work in (a) 5 days (c) 6 days

4. A and B undertake to do a piece of work for ` 720. A

alone can do it in 8 days and B alone can do it in 12 days. With the help of C they finish it in 4 days. Then, the share of C (in `) is (a) 120 (c) 360

(b) 300 (d) 240

20 min and 10 min, respectively. When the tank is empty, all the three pipes are opened. If A, B and C discharge chemical solutions P, Q and R respectively, then the part of solution R in the liquid in the tank after 3 min is 5 (b) 11

6 (c) 11

(b)

1 30

(c)

1 20

(d)

1 10

8. A is twice as good a workman as B and together

they finish a piece of work in 14 days. The number of days taken by A alone to finish the work is (b) 21 days (d) 42 days

9. A is thrice as good a workman as B and therefore

is able to finish a job in 40 days less than B. Working together, they can do it in (a) 14 days (c) 20 days

(b) 13 days (d) 15 days

10. A and B can do a piece of work in 8 days, B and C

5. Three pipes A, B and C can fill a tank in 30 min,

8 (a) 11

1 40

(a) 11 days (c) 28 days

(b) 10 days (d) 8 days

(b) ` 540 (d) ` 600

7 (d) 11

6. A, B and C can do a piece of work in 20, 24 and

30 days, respectively. They undertook to do the piece of work for ` 5400. They begin the work

can do it in 24 days, while C and A can do it in 8 days. In how many days can C do it alone? (a) 60 (c) 30

(b) 40 (d) 10

11. A coach helper repairs a coach in 12 days. His

technician completes the same job in 18 days. If both of them work together, in how many days would the job be completed? 5 36 (c) 7.2

(a)

https://sscstudy.com/

(b) 7.3 (d) 7

4 7

https://sscstudy.com/

60

CUET (UG) Section III : General Test

12. 15 trackmen complete rail renewal job in 12 days.

20. Rani and Sneha working separately can finish a job

How many days would 18 trackmen take to complete the same job? (a) 10 (c) 15

in 8 h and 12 h respectively. If they work for an hour alternately, Rani beginning at 9 : 00 am when will the job be finished?

(b) 18 (d) 17

(a) 7 : 30 pm (c) 6 : 30 pm

13. A man can do a work in 15 days. His father do this

work in 20 days and man’s son do this work in 25 days. If all do the work together, how many days will they take?

21. After working for 8 days, Anil finds that only 1/3 of

the work has been done. He employs Rakesh who is 60% efficient as Anil. How many more days will Anil take to complete the job?

(a) Less than 6 days (b) 6 days (c) Approximately 6.4 days (d) More than 10 days

(a) 5 (c) 10

14. If 6 men and 8 boys can do a piece of work in

10 days, while 26 men and 48 boys can do the same in 2 days, the time taken by 15 men and 20 boys in doing the same type of work will be (a) 4 days (c) 6 days

days respectively. In what time will they do it together?

(b) 5 days (d) 7 days

3 days 7 2 (c) 3 days 7

(b)

1 10

(c)

7 15

(d)

23. A can do 1/3 of the work in 5 days and B can do 2/5

8 15

of the work in 10 days. In how many days both A and B together can do the work?

16. 8 horses’ food is equal to 6 cows’ food. How many

3 4 3 (c) 9 8

(b) 16 (d) 18

17. X can complete a piece of work in 30 h and Y can

20 days 9 (c) 3 days

11 days 9 (d) None of these

(b)

(a)

18. A can do 3/4 of a work in 12 days. In how many

days can he finish 1/8 of the work? (b) 2 (d) 4

25. 12 men can complete a work in 18 days. 6 days

after they started working, 4 men joined them. How many days will all of them take to finish the remaining work?

19. If 18 men and 10 boys can do in one day as much

work as 10 men and 22 boys, the amount that a men should be paid per day, if a boy gets ` 10 per day (a) ` 7.5 (c) ` 20

(d) 10

efficient as C, if A, B and C work together how long would they take to complete a job that B takes 10 days to complete?

(b) 14.33 (d) 16.33

(a) 1 (c) 3

4 5

24. A is thrice as efficient as B and B is twice as

do it in 24 h. In how many hours will it be completed if they work together? (a) 13.33 (c) 15.33

(b) 8

(a) 7

cows can eat the food meant for 20 horses? (a) 15 (c) 17

3 days 7 4 (d) 3 days 7 (b) 3

(a) 2

work on it together for 4 days, then the fraction of the work that is left is 1 4

(b) 8 (d) 12

22. A, B and C can do a job alone in 6, 12 and 24

15. A can do a work in 15 days and B in 20 days. If they

(a)

(b) 7 : 00 pm (d) 6 : 00 pm

(a) 9 (c) 12

(b) ` 12.5 (d) ` 15

(b) 10 (d) 15

ANSWERS 1. 11. 21.

(c) (c) (c)

2. 12. 22.

(a) (a) (b)

3. 13. 23.

(a) (a) (c)

4. 14. 24.

(a) (a) (a)

5. 15. 25.

(c) (d) (a)

6. 16. 26.

(a) (a) (d)

https://sscstudy.com/

7. 17. 27.

(b) (a) (a)

8. 18. 28.

(b) (b) (d)

9. 19. 29.

(d) (d) (b)

10. 20. 30.

(a) (c) (c)

https://sscstudy.com/

NUMERICAL ABILITY

61

Hints & Solutions 1. Here, M1 = 30, W1 = 1500, D1 = 24 , T1 = 6, M 2 = 18, W 2 = 1800, D2 = ?, T2 = 8 Using, M1 T1 D1 W 2 = M 2 T2 D2 W1 30 × 6 × 24 × 1800 = 18 × 8 × D2 × 1500 30 × 6 × 24 × 1800 ⇒ D2 = 18 × 8 × 1500 = 36 days 2. Let 1 man’s one day’s work be x and 1 child's one day's work be y there, 1 ...(i) 4x + 6 y = 8 1 ...(ii) 3x + 7 y = 10 Solving both the equations, 1 y= 400 Therefore, 1 child can complete the work in 400 days and thus, 20 children can complete the work 400 in i.e. 20 days. 20 3. Let B will do the remaining work in x days. According of the question, x+5 5 + =1 10 20 10 + x + 5 ⇒ =1 20 ⇒ x + 15 = 20 ∴ x = 5 days 1 4. C can do the work = 1 1 1   − −   4 8 12 1 = = 24 days  6 − 3 − 2    24  Ratio of one day work of A, B and C 1 1 1 : = 3 :2 :1 = : 8 12 24 1 ∴ Share of C = × 720 (3 + 2 + 1) 1 = × 720 = `120 6 5. Total quantity of solutions P, Q and R from A, B and C respectively, after 3 min 3 3 3 3 × 11 11 = + + = = 30 20 10 60 20

Quantity of solution R in 3 min 3 = 10 ∴ Part of solution R 3 3 × 20 6 10 = = = 11 10 × 11 11 20 6. Let the number of days to complete the work be x, then x x−2 x−5 + + =1 20 24 30 6x + 5 (x − 2) + 4 (x − 5) =1 ⇒ 120 ⇒ 6x + 5x + 4x = 120 + 10 + 20 ⇒ 15x = 150 ∴ x = 10 10 1 ∴ Work done by A = = 20 2 ∴ Share of A from the assured money 1 = × 5400 = ` 2700 2 7. Work done by A and B together in 1 1 day = 72 Work done by B and C together 1 in 1 day = 120 and work done by C and A 1 together in 1 day = 90 ⇒ 2( A + B + C )’s 1 day’s work 1 1  1 = + +   72 120 90

9. Ratio in work efficiency of A and B = 3 : 1 and then ratio in time taken by A and B = 1 : 3 According to question, 3x − x = 40 2x = 40 ⇒ x = 20 ∴ A takes 20 days and B takes 30 days. ∴ If they work together, the time 20 × 60 taken by A and B = 20 × 60 = 15 days 10. According to question, 1 8 1 (B + C )’s 1 day’s work = 24 7 (C + A )’s 1 day’s work = 60 ∴ 2 ( A + B + C )’s 1 day’s work 1 1 7 17 = + + = 8 24 60 60 17 ( A + B + C )’s 1 day’s work = 120 17 1 1 C’s 1 day’s work = − = 120 8 60 ∴ C can complete the work in 60 days. 11. Both work together x × y 12 × 18 12 × 18 = = = 30 x + y 12 + 18 ( A + B)’s 1 day’s work =

= 7.2 days 12. T1 = 15 trackmen, D1 = 12 days

⇒ ( A + B + C )’s 1 day’s work 1 1 1 1 =  + +  2  72 120 90

T2 = 18 trackmen , D2 = ? According to the question,

∴ ( A + B + C )’s 2 day’s work 1  5 + 3 + 4 1 =2×   = 2  360  30



8. Let A completes the work in x days, then B completes the work in 2x days. A and B together completes the work in = 14 days x + 2x = 14 x + 2x 3 ⇒ x = × 14 2 = 21 days

https://sscstudy.com/

T1 D1 = T2D2 15 × 12 = 18 × D2 15 × 12 D2 = ⇒ = 10 days 18 1 13. A man 1 day’s work = 15 1 Man’s father 1 day’s work = 20 1 Man’s son 1 day’s work = 25 If three do together, then 1 day’s 1 1 1 work = + + 15 20 25 30 + 15 + 12 57 = = 300 300

https://sscstudy.com/

62

CUET (UG) Section III : General Test

Hence, they do the work 300 days together in 57 or 5.26 days. 14. 6 men and 8 boys can do a work = 10 days 60 men and 80 boys can do a work = 1 day Similarly, 26 men and 48 boys can do a work = 2 days 60 men + 80 boys = 52 men + 96 boys 8 men = 16 boys 1 man = 2 boys 6 men + 8 boys (4 men) can do a work = 10 days 10 men can do a work = 10 days 15 men + 20 boys (10 men) will complete the work = x days 25 men will complete the work = x days M1 D1 = M 2D2 10 × 10 = 25 × x 100 x= ∴ 25 ⇒ x = 4 days 1 15. One day work of A = 15 1 One day work of B = 20 One day work of both 1 1 = + 15 20 4+3 7 = = 60 60 7 7 4 day work of both = ×4 = 60 15 7 8 = ∴ Remaining work = 1 − 15 15 16. Food of 8 horses’ = Food of 6 cows’ Food of 1 horse = Food of

6 cow 8

6 × 20 8 = 15 cows 1 17. 1 h work of X = 30 1 1 h work of Y = 24 1 1 1 h work of both = + 30 24 Food of 20 horses =

4+5 9 3 = = 120 120 40 Both complete the work 40 = = 13. 33 h 3 =

18. Using formula, M1 D1 H 1 M 2D2H 2 = W1 W2 12 D2 ⇒ = 3 1 4 8 1 4 D2 = 12 × × 8 3 D2 = 2 ∴ He can finish in 2 days. 19. 18 men + 10 boys = 10 men + 22 boys Man 3 = ⇒ Boy 2 ⇒ Their wages paid will be in the ratio 3 : 2. Hence, if a boy gets ` 10 per day a mean get ` 15. 20. Capacity of Rani and Sneha per hour 1 1 respectively. = and 12 8 Total work done by them in 1 h 1 1 5 per hour = + = 8 12 24 1 48 ∴ Full work = 1 × = 9 h, then 6 5 of work will left and now turn is of Sneha. Sneha will do the remaining work in 30 min. Total time taken = 9 h and 30 min Time = 9 + 9 h and 30 min = 18 : 30 h Rani finished the job in 6 : 30 pm. 21. If efficiency of Anil is taken as 1, then efficiency of Rakesh = 0.6 ∴After employing Rakesh, total effective man; man × efficiency = 1 × 1 + 0.6 × 1 = 1 + 0.6 = 1.06 Using the formula Man1 × Efficiency1 × Days1 Work1

https://sscstudy.com/

Man 2 × Efficiency 2 × Days 2 Work2 1 × 1 × 8 1.6 × x ⇒ = 1 1  1 −   3 3 2 1 × 1.6x = × 8 ⇒ 3 3 ⇒ x = 10 days ∴10 days more to compete the job. 1 22. Taken time = 1 1 1 + + 6 12 24 1 = 4+2+1 24 1 24 = = 7 7 24 3 = 3 days 7 =

23. A does one work in 15 days B does one work in 25 days ∴( A + B)’s one day work 1 1 = + 25 25 5+3 8 = = 75 75 Hence, ( A + B) will complete one 3 work in 9 days. 8 24. Since, A = 3B and B = 2C ∴ B takes 10 days ⇒ C takes 20 days and A takes 10 days. 3 ∴ If they all work together, then 3 1 1 9 + + = 10 10 20 20 20 days. Hence, time taken is 9 1 25. 12 men’s 6 days work = 3 2 Remaining work = 3 m1 × d1 × w2 = m2 × d2 × w1 2 1 12 × 1 × = 16 × d2 × 3 18 12 × 2 × 18 = 9 days d2 = 3 × 16

https://sscstudy.com/

NUMERICAL ABILITY

63

CHAPTER 13

Time, Speed and Distance The concept of time, speed and distance is related to a particular object in motion. A set of typically asked questions from the topic ‘time, speed and distance’ that acquaints you to different concepts in the topic including relative speed, average speed, different units for measurement of time, speed and distance and the conversion of these units. Also includes questions on boats and stream and train and platform.

Time Time is defined as quantity, which governs the order or sequence of an occurrence. In the absence of time, the actual sequence of any occurrence or incident would be lost. If we did not have the concept of time, we would not be able know in what period or in what order something took place. Unit of Time Hour (h) and second (s) are mostly taken as the unit of time.

Speed Speed is defind as the distance covered per unit time. It is the rate at which the distance is covered. Unit of Speed Though we commonly take km/h. As units of speed.

Distance When an object is moving with a certain speed in a particular time, the displacement made by an object is called the distance. Unit of Distance kilometre (km) and metre (m) is usually taken as the unit of distance.

Relationship between Time, Speed and Distance Relationship between time, distance and speed is expressed by Speed =

Distance Time

or Distance = Speed × Time

This expression shown that 1. Speed is directly proportional to distance. If the speed is doubled, then distance travelled in the same time, will also be doubled.

2. Distance and time are directly proportional. If distance to be travelled is doubled, then the time taken would also be doubled at the same speed. 3. Time is inversely proportional to speed. If the distance remains the same and speed is doubled, then time taken to travel the same distance becomes half of the original time taken at the original speed.

Average Speed When a certain distance is covered at speed A and the same distance is covered at speed B, then the average 2AB . speed during the whole journey is given by A+ B e.g. A person goes to Delhi from Mumbai at the speed of 60 km/h and comes back at the speed of 50 km/h. Calculate the average speed of the person for the entire trip. Sol. Average Speed =

2 × 60 × 50 60 + 50 [QA = 60 km/h, B = 50 km/h]

6000 = = 54. 54 km/h 110

Relative Speed Relative speed is the speed of a moving object in relation to other moving object. Let two objects are in motion and their speeds are a and b, respectively, then (a) Relative speed = a + b (if two objects are moving in opposite directions) (b) Relative speed = a − b (if two objects are moving in same direction and a > b)

https://sscstudy.com/

https://sscstudy.com/

64

CUET (UG) Section III : General Test

e.g., Two persons are moving in the direction opposite to each other. The speeds of the both persons are 5 km/h and 3 km/h, respectively. Find the relative speed of the two persons in respect of each other. Sol. Required relative speed = 5 + 3 = 8 km/h

Concept of Relative Speed in Motion of Trains Some important points in problems on trains 1. If two trains of length x km and y km are moving in opposite directions at u km/h and v km/h, then time  x + y taken by the trains to cross each other =   h  u + v 2. If two trains of length x km and y km are moving in the same direction at u km/h and v km/h are, where u > v , then time taken by faster train to cross the  x + y slower train =   h  u − v

Boats and Streams The problems of boats and streams are also based on the basic relation of speed, distance and time Distance Speed = i. e. , Time In there questions, the direction along the stream (water) is called downstream and direction against the stream is called upstream. If the speed of a boat in still water is x km/h and the speed of the stream is y km/h, then Downstream speed = ( x + y ) km/h Upstream speed = ( x − y ) km/h ˜ From the above relationship we conclude, if the downstream speed is u km/h and upstream speed v km/h then,  u + v Speed of boat in still water =   km/h  2  u − v  Speed of stream =   km/h  2 

Solved Examples 1. The speed of a bus is 72 km/h. The distance

covered by the bus in 5 s is (a) 50 m

(b) 74.5 m

Sol. (d) T =

(c) 100 m (d) 60 m 5 Sol. (c) Speed of bus in m/s = 72 × = 20 m/s 18



∴Distance travelled in 5s = 20 × 5 (Speed × Time) = 100 m



2. Two men start together to walk a certain distance,

one at 4 km/h and another at 3 km/h. The former arrives half an hour before the latter. Find the distance. (a) 6 km

(b) 9 km

(c) 8 km

(d) 7 km

Sol. (a) Let the distance = x km 4x − 3x 1 12 x x 1 − = ⇒ = ⇒x = 2 3 4 2 12 2 x = 6 km

By given condition, ∴

3. If a train 110 m long passes a telegraph pole in 3 s,

then the time taken by it to cross a railway platform 165 m long will be (a) 6.50 s

(b) 8.5 s (c) 7.5 s (d) 6.55 s Distance 110 Sol. (c) Speed = m/s = Time 3 110 + 165 275 × 3 Time taken to cross the platform = = 110/3 110 = 7.5 s

directions, cross each other in 6 s. The speed of train A is 126 km/h, while that of train B is 90 km/h. If the length of train A is 160 m, what is the length of train B? (b) 120 m

(c) 220 m

160 + L2

6=

(126 + 90) 6 × 216 ×



5 18

5 = 160 + L2 18 L2 = 200 m

5. A train takes 18 s to pass completely through a

station 162 m long and 15 s through another station 120 m long. Find the length of the train. (a) 80 m (c) 85 m

(b) 90 m (d) 95 m 162 + l 120 + l and 15 = Sol. (b) 18 = 5 5 ∴ l − 185 + 162 = 0 and l − 155 + 120 = 0 On solving the above two equations, we have l = 90 m

6. A train 100 m long completely passes a man

walking in the same direction at 6 km/h in 5 s and a car travelling in the same direction in 6 s. At what speed was the car travelling? (a) 15 km/h (c) 18 km/h

(b) 20 km/h (d) 16 km/h

Sol. (c) In case of man,

4. Two trains A and B, travelling in opposite

(a) 150 m

L1 + L2 S1 + S2

(d) 200 m

100 × 18 ⇒ x = 78 km/h (x − 6) × 5 100 × 18 In case of car, 6 = (78 − y)5 5=



https://sscstudy.com/

y = (78 − 60) = 18 km/h

https://sscstudy.com/

NUMERICAL ABILITY

65

Practice Questions 1. A motorcycle covers 40 km with a speed of

11. Nalanda and Nawada are two towns. Sabir goes

20 km/h. Find the speed of the motorcycle for the next 40 km journey so that the average speed of the whole journey will be 30 km/h.

from Nalanda to Nawada at 30 km/h and comes back to the starting point at 70 km/h. What is the average speed during the whole journey?

(a) 70 km/h (c) 60 km/h

(a) 12 km/h

(b) 52.5 km/h (d) 60.5 km/h

half is completed at 22 km/h and the second half at 26 km/h. Find the distance. (b) 284 km (d) 288 km

3. A man takes 6 h 30 min in walking to a

certain place and riding back. He would have gained 2 h 10 min by riding both ways. How long would he take to walk both ways? (a) 8 h 20 min (c) 8 h 40 min

(b) 4 h 10 min (d) 4 h 20 min

4. Raghubir after travelling 84 km, found that if he

travelled 5 km an hour more, he would take 5 h less, he actually travelled at a rate of (a) 7 km/h (c) 5 km/h

(b) 10 km/h (d) 6 km/h

1 km in 3 h, the distance 5 covered by him in 5 h is

5. If a man covers 10 (a) 16 km (c) 18 km

(b) 15 km (d) 17 km

6. A man covers a certain distance on scooter. Had he

moved 3 km/h faster, he would have taken 40 min less. If he had moved 2 km/h slower, he would have token 40 min more. The distance (in km) is (a) 42.5 (c) 37.5

(b) 36 (d) 40

7. A man walking with 3/4 of his usual speed, reaches

office 20 min late. His usual time is (a) 50 min (c) 70 min

office 6 min late. He arrived 6 min early, when he increased his speed by 2 km/h. The distance of his office from the starting place is (b) 7 km (d) 16 km

a certain distance in 3 h 45 min. If he covers the same distance on cycle, cycling at the rate of 16.5 km/h, the time taken by him is

10. If a man runs at 2m/s, how many kilometres does (a) 8.4 (c) 9.6

(b) 6.9 (d) 7.4

(a) 10 s

(b) 20 s

(c) 400 s

(d) 17 s

13. A railway officer standing on a railway bridge

which is 200 m long finds that the train crosses the bridge in 19 s but himself in 9 s . Find the length of the train. (a) 135 m

(b) 180 m

(c) 72 m

(d) 90 m

1 14. Two buses, one of them takes 7 h to travel 2 300 km and another takes 9 h to travel 450 km. Find the ratio of speed of two buses. (a) 2 : 3

(b) 4 : 3

(c) 4 : 5 (d) 8 : 9

15. A train 110 m long is running at the speed of

72 km/h to pass a 132 m long platform in how many times? (a) 9.8 s

(b) 12.1 s

(c) 12.42 s

(d) 14.3 s

16. A boatman can row his boat at a speed of 8 km/h in

still water. If a river flows at a speed of 2 km/h, then how long will the boatman take to row his boat 1200 m with the direction of the current? (a) 7.2 min

(b) 7.4 min

(c) 7.6 min

(d) 7.8 min

17. Two cyclists start from the same place in opposite

directions. One goes towards north at 18 km/h and the other goes towards south at 20 km/h. What time will they take to be 47.5 km apart? 1 (a) 1 h 4 (c) 3 h

(b) 2 h (d) None of these

24 km/h and comes down with an average speed of 36 km/h. The distance travelled in both the cases being the same, the average speed for the entire journey is (a) 30 km/h (c) 32 km/h

(b) 28.8 km/h (d) None of these

72 km/h. If it crosses a tunnel in 1 min, then the length of the tunnel is (a) 650 m (c) 550 m

(b) 500 m (d) 700 m

20. Two trains of length 120 m and 80 m are running

(b) 54.55 min (d) 45.55 min

he run in 1 h 20 min?

much time will it pass a platform 200 m long?

19. A train 700 m long is running at the speed of

9. Walking at the rate of 4 km an hour, a man covers

(a) 55.45 min (c) 55.44 min

(d) 42 km/h

18. A man goes uphill with an average speed of

(b) 80 min (d) 60 min

8. When a person cycled at 10 km/h he arrived at his

(a) 6 km (c) 12 km

(c) 24 km/h

12. A train 200 m long is running at 72 km/h. In how

2. A bus can complete a journey in 12 h. The first

(a) 280 km (c) 286 km

(b) 60 km/h

in the same direction with velocities of 40 km/h and 50 km/h respectively. The time taken by them to cross each other is (a) 60 s (c) 72 s

https://sscstudy.com/

(b) 75 s (d) 80 s

https://sscstudy.com/

66

CUET (UG) Section III : General Test

21. A motor boat takes 2 h to travel a distance of 9 km

down the current and it takes 6 h to travel the same distance against the current. The speed of the boat in still water and that of the current (in km/h) respectively are (a) 3, 2 (c) 3, 1.5

(b) 3.5, 2.5 (d) 3, 1

motor cycle rider starts from P towards Q at 8 pm at a speed of 40 km/h. At the same time another motor cycle rider starts from Q towards P at 50 km/h. At what time will they meet? (b) 10 : 40 pm (d) 10 : 30 pm

23. A man standing on a railway platform observes

that a train going in one direction takes 4 s to pass him. Another train of same length going in the opposite direction takes 5 s to pass him. The time taken (in seconds) by the two trains to cross each other will be

49 9 50 (c) 9

40 9 31 (d) 9

distance of 1 km. But due to the slippery ground, his speed reduced by q km/h ( p > q) . If he takes r h to cover the distance, then 1 (b) = p+ q r 1 (d) = p−q r

(c) r = p − q

the remaining 40 km in 5 h. His average speed for whole journey is

27. A is twice as fast as B and B is thrice as fast as C.

The journey covered by C in 42 min will be covered by A in (d) 14 min

28. A man covers half of his journey at 6 km/h and the

remaining half at 3 km/h. His average speed is (a) 4.5 km/h (c) 4 km/h

(b) 3 km/h (d) 9 km/h

(b) 480 km (d) 700 km

33. The ratio of the speeds of three cars is 2 : 3 : 4.

What is the ratio of the times taken by them in covering the same distance? (b) 4 : 3 : 2 (d) 6 : 4 : 3

its normal speed it would have taken 2 h less to cover 300 km. What is its normal speed? (a) 20 km/h

(b) 25 km/h

(c) 30 km/h

(d) 45 km/h

35. A 200 m long train crosses a platform of double its

length in 36 s. The speed of the train is (b) 48 km/h

(c) 64 km/h

(d) 66 km/h

25 km/h. It will cross a man coming from the opposite direction at 2 km/h in (a) 36 s

(b) 32 s

(c) 28 s

(d) 24 s

telegraph pole. How long would it take to cross a platform 110 m long? (a) 24 s

(b) 31 s

(a) 46.5 km/h (c) 37.6 km/h

(c) 21 s

(d) 33 s

(b) 37.5 km/h (d) 39.6 km/h

39. A train is moving at a speed of 132 km/h. If the

length of the train is 110 m, how long will it take to cross a railway platform 165 m long? (a) 7.5 s

(b) 15 s

(c) 10 s

(d) 5 s

40. A train crosses a platform 100 m long in 60 s at a

(a) 8 s

destination late by 11 min. But, if it runs at 50 km/h, it is late by 5 min only. The correct time for the train to complete its journey is (b) 21 min (d) 19 min

(a) 420 km (c) 640 km

speed of 45 km/h. The time taken by the train to cross an electric pole is

29. If a train runs at 40 km/h, it reaches its

(a) 15 min (c) 13 min

is increased by 4 km/h. This distance could be covered in 7 1 h. This distance is 2

m long platform in 17 s and a 210 m long bridge in 25 s. The speed of the train is

(d) 8 km/h

(c) 63 min

32. A car covers a certain distance in 8 h. If the speed

38. A train running at a uniform speed crosses a 122

(b) 7 km/h

(b) 28 min

(b) 8 : 30 am (d) 8 : 30 pm

37. A train running at 36 km/h takes 10 s to pass a

26. A man completes 30 km of a journey at 6 km/h and

(a) 7 min

(a) 7 : 45 am (c) 7 : 14 am

(a) 60 km/h

(b) 30 m/s (d) 50 m/s

4 (a) 6 km/h 11 1 (c) 7 km/h 2

Ram walks from A to B at 4 km/h and Shyam walks from B to A at 6 km/h. Both start at 7 am. At what time will they meet?

36. A train 270 m long is moving at a speed of

25. A train is going at a speed of 180 km/h. Its speed is (a) 15 m/s (c) 40 m/s

(b) 19 min (d) 18 min

34. If the speed of a train is increased by 5 km/h from

24. A boy is running at a speed of p km/h to cover a

1 pq (a) = r p+ q

(a) 17 min (c) 16 min

(a) 2 : 3 : 4 (c) 4 : 3 : 6

(b)

(a)

runs 21 m the thief 15 m in a minute. In what time will the constable catch the thief?

31. The distance between two places A and B is 15 km.

22. Distance between two towns P and Q is 240 km. A

(a) 9 : 45 pm (c) 11 pm

30. A constable is 114 m behind a thief. The constable

(b) 52 s

(c) 1 min

(d) 40 s

41. Two trains 105 m and 90 m long run at the speeds

of 45 km/h and 72 km/h respectively in opposite directions on parallel tracks. How much time do they take to cross each other? (a) 5 s

https://sscstudy.com/

(b) 6 s

(c) 7 s

(d) 8 s

https://sscstudy.com/

NUMERICAL ABILITY

67

ANSWERS 1. 11. 21. 31. 41.

(c) (d) (c) (b) (b)

2. 12. 22. 32.

(c) (b) (b) (b)

3. 13. 23. 33.

(c) (b) (b) (d)

4. 14. 24. 34.

5. 15. 25. 35.

(a) (c) (d) (b)

(d) (b) (d) (a)

6. 16. 26. 36.

(d) (a) (b) (a)

7. 17. 27. 37.

(d) (a) (a) (c)

8. 18. 28. 38.

(c) (b) (c) (d)

9. 19. 29. 39.

(b) (b) (d) (a)

10. 20. 30. 40.

(c) (c) (b) (b)

Hints & Solutions 1. Let speed of the motor cycle for the next 40 km journey be x km/h. Then, 2 × 20 × x 30 = 20 + x ⇒ 3(20 + x) = 4x ⇒ 60 + 3x = 4x ⇒ x = 60 km/h 2. Let distance of the journey be d km. d d Then, + = 12 2 × 22 2 × 26 d d ⇒ + = 48 11 13 24d = 48 ⇒ 143 d = 286 km ⇒ 4. Let the speed of Raghubir be x km/h. According to the question, 84 84 − =5 x x+5 ⇒ ⇒ ⇒ ⇒ ⇒ ⇒ ⇒ ⇒ ∴

1 1  84  − =5 x x + 5)  (   x + 5 − x 84   =5  x(x + 5)  84 × 5 =5 x(x + 5) x(x + 5) = 84 x + 5x − 84 = 0 x(x + 12) − 7(x + 12) = 0 (x + 12) (x − 7) = 0 x− 7 =0 x = 7 km/h 2

5. Distance covered by man in 3h 1 = 10 km 5 Distance covered by man in 1 h 51 17 km = = 5 ×3 5 Distance covered by man in 5 h 17 will be = × 5 = 17 km 5

6. Let distance and original speed of the man be d km and s km/h. Then, d d 2 − = s s+3 3 d (s + 3 − s) 2 = ⇒ 3 s (s + 3) ⇒ and ⇒

9 d = 2s (s + 3) ...(i) d d 2 − = s−2 s 3 d (s − s + 2) 2 = s (s − 2) 3



3 d = s (s − 2) ...(ii) From Eqs. (i) and (ii), we get 3s (s − 2) = 2s (s + 3) ⇒ 3s2 − 6s = 2s2 + 6s ⇒ s2 = 12s ⇒ s = 12 From Eq. (ii), we get 3 d = 12 (12 − 2) ∴

d = 40 km

7. Let usual speed and usual time taken by the man are S km/h and T h, respectively. ...(i) ∴ D = ST According to the question, we get 3 1  ...(ii) D = S ⋅ T +   4 3 From Eqs. (i) and (ii), we get ST = ⇒ ∴

3 1  S ⋅ T +   4 3

4T = 3T + 1 T = 1 h = 60 min

8. Let the distance of his office from the starting point be x km. By given condition, x 6 x 6 − = + 10 60 (10 + 2) 60

https://sscstudy.com/

x x 12 − = 10 12 60 12 × 60 x= = 12 km 60 9. Speed = 4 km/h 3 h 4 ∴ Distance covered by man 3 = 4 × 3 = 15 km 4 15 h ∴Required time = 16. 5 15 = × 60 min 16. 5 = 54.55 min Time = 3 h 45 min = 3

10. Speed of man = 2 m/s 2 × 18 36 = km/ h = km/ h 5 5 80 4 Time = 1 h 20 min = h= h 60 3 Distance = Time× Speed 36 4 48 = × = = 9.6 km 5 3 5 11. Average speed of Sabir 2xy 2 × 30 × 70 = = x+ y 30 + 70 4200 = = 42 km/h 100 12. Length of train = 200 m Length of platform = 200 m Total distance cover by train to pass the platform = 400 m Speed of train = 72km / h 72 × 5 = = 20 m/s 18 Time taken by train to pass the platform Distance 400 = = = 20 s Speed 20 13. Let length of train = x m According to the question, x + 200 Speed = 19

...(i)

https://sscstudy.com/

68

CUET (UG) Section III : General Test

x ...(ii) 9 From Eqs. (i) and (ii), x x + 200 = 9 19 ⇒ 19x − 9x = 1800 ⇒ 10x = 1800 ⇒ x = 180 m Hence, length of train is 180 m. 300 14. First bus speed = ×2 15 = 40 km/h 450 Second bus speed = 9 = 50 km/h ∴ Required ratio = 40 : 50 = 4 : 5 Speed =

15. Speed of the train = 72 km/h 5 m/s = 72 × 18 = 20 m/s 110 + 132 ∴Required time = 20 242 = 12 .1 s = 20 16. Speed of boatman in still water = 8 km/h Speed of river = 2 km/h Speed of boatman with the direction of current = 8 + 2 = 10 km/h Distance cover by boat in the direction of current = 1200 m Distance Time = Speed 1200 = 10 × 1000 60 1200 × 60 72 = = 10 × 1000 10 = 7 . 2 min 17. Since, they are in opposite direction. So, total distance apart = 18 + 20 = 38 km For 47.5 km apart, it takes time 1 = × 47.5 38 1 =1 h 4 2xy 18. Average speed = x+ y 2 × 24 × 36 = 24 + 36 = 28 . 8 km/h

5 = 20 m/s 18 Let the length of tunnel be x m. 700 + x Then, = 60 20 ⇒ x = 500 m

19. Speed = 72 ×

20. Since, the train is running in same direction therefore relative speed = (50 − 40) km/h 5 25 m/s = 10 × = 18 9 ∴ Required time = Time taken to cover 25 m/s (120 + 80) at 9 9 = 200 × = 72 s 25 21. Rate of downstream 9 = = 4 .5 km/h 2 9 Rate of upstream = = 1 .5 km/h 6 ∴Rate in still water 1 = (4 .5 + 1 .5) = 3 km/h 2 Rate of the current 1 = (4 .5 − 1 .5) = 1 .5 km/h 2 22. Suppose they meet x h after 8 pm. Then, sum of distance covered by them in hours = 240 km ∴ 40x + 50x = 240 240 = 2 h 40 min ⇒ x= 90 Hence, they will meet in 10:40 pm. 23. Let the length of each train be x m. x Then, speed of first train = m/s 4 and speed of second train x = m/s 5  x x Relative speed =  +  m/s  4 5 9x m/s = 20 ∴ Time taken to cross each other = Time taken to cover 2x m  9x at   m/s  20 20 40 s = 2x × = 9x 9

https://sscstudy.com/

24. Actual speed of boy = ( p − q) km/h Time taken to cover 1 km 1 = p−q 1 ∴ =r p−q 1 ⇒ = p−q r 5  25. 180 km/h = 180 ×  m/s  18 = 50 m/s 26. Total journey = (30 + 40) = 70 km  30  Total time taken =  + 5 6  = 10 h 70 km/h Average speed = 10 = 7 km/h 27. Let C ’s speed be x m/min. B’s speed = 3x m/min A’s speed = 6x m/min Ratio of speeds of A and C = 6x : x = 6 : 1 ∴ 6 : 1 = 42 : y 42 ⇒ y= = 7 min 6 ∴A takes 7 min to cover it. 2xy km/h 28. Average speed = x+ y 2 ×6 ×3 = 6+3 36 = = 4 km/h 9 29. Let the distance travelled by the train is x km. Then, x x − = 60 40 × 1000 50 × 1000 60 60 60x 60x − =6 ⇒ 40000 50000 60  x x 6 ⇒  −  = 10000  4 5 1 x ⇒ = 1000 20 ⇒ x = 20 km ∴The correct time for the train to complete its journey 20 = × 60 − 11 40 = 30 − 11 = 19 min

https://sscstudy.com/

NUMERICAL ABILITY

30. (21 − 15) = 6 m is covered in 1 min 114 m will be covered in 1   × 114 min = 19 min 6  31. Suppose they meet after x h. Then, 4x + 6x = 15 ⇒ 10x = 15 ⇒ x = 1.5 h So, they meet at 8 : 30 am. 32. Let the required distance be x km. x x Then, − =4 15 / 2 8 2x x − =4 ⇒ 15 8 ⇒ 16x − 15x = 480 ⇒ x = 480 km 1 1 1 33. Required ratio = : : 2 3 4 = 6 :4 :3 34. Let the normal speed be x km/h. 300 300 − =2 x (x + 5) 1 1 1 ⇒ − = x x + 5 150 1 x+5−x ⇒ = x(x + 5) 150 ⇒ x(x + 5) = 750 ⇒ x2 + 5x − 750 = 0 ⇒ x2 + 30x − 25x − 750 = 0 ⇒ x(x + 30) − 25(x + 30) = 0 ⇒ (x − 25)(x + 30) = 0 ∴ x = 25 km/h

35. Length of train = 200 m and length of platform = 2 × 200 m = 400 m 200 + 400 Speed of the train = 36 600 m/s = 36 600 18 = × 36 5 = 60 km/h 36. Relative speed = (25 + 2) = 27 km/h 5 m/s = 27 × 18 15 m/s = 2 Required time 270 270 × 2 s = 36 s = = 15 15 2 5  37. Speed of train = 36 ×  m/s  18 = 10 m/s Length of the train = (10 × 10) m = 100 m Time taken to cross the platform 100 + 110 210 = = = 21 s 10 10 38. Let the length of train be x m/s. x + 122 x + 210 = 17 25 ⇒ 17x + 3570 = 25x + 3050 ⇒ 8x = 520 ⇒ x = 65 ∴Length of the train = 65 m 65 + 122 Speed of train = m/s 17

https://sscstudy.com/

69

187 m/s 17  187 18 = ×  km/h  17 5 =

= 39.6 km/h 39. Speed of train 5  110  m/s = 132 ×  =  18 3 Time taken to cross the platform 110 + 165  275 × 3 = =   110  110 / 3 15 = s 2 = 7.5 s 40. Let the length of the train be x m. 25 5  Speed = 45 ×  m/s = m/s  18 2 x + 100 25 ∴ = 60 2 ⇒ 2x + 200 = 1500 ⇒ 2x = 1300 ⇒ x = 650 Time taken to cross a pole 2  = 650 ×  = 52 s  25 41. Sum of the lengths of trains = (105 + 90) m = 195 m Relative speed = (72 + 45) = 117 km/h 5 65 m/s = 117 × = 18 2 Time taken to cross each other 2  = 195 ×  = 6 s  65

https://sscstudy.com/

70

CUET (UG) Section III : General Test

CHAPTER 14

Area and Perimeter In this chapter, we often have to deal with the problem of finding the areas of plane figures. As we know that, plane figures have only length and breadth (i.e., 2D only), hence this chapter can be featured as 2D mensuration too.

Area Unit Unit of its is same as the unit of side i.e., m, cm etc.

It is the space enclosed within the boundary of a 2D figure. Unit sq cm, sq m (i.e., square units) etc.

Triangle An enclosed figure formed by three line segment. Quadrilateral A close figure having four sides.

Perimeter

Circle A plane figure enclosed by a curve on which every point is equally distant from a fixed point.

Sum of length of all the sides of 2D figure is known as its perimeter.

2D Figures with their Postulates Different Types of Triangle ●

Equilateral Triangles Its all three sides are equal. Side = a, Height = h

a

Related Formulae

Different Types of Triangle

Area = 3 4 a 2 Height, h = 3 2 a Perimeter = 3a Each angle = 60°



Scalene Triangle Its all sides are unequal.

a

a

b

c a, b, c = Unequal sides of the triangle

a

Isosceles Triangles Its two sides are equal. a = Equal sides b = Third unequal side h = Height

b 4

4a 2 − b2

Height, h =

a

a h D b

C



b a 2 −    2

2

1 4a 2 − b2 2 Perimeter = a + a + b = 2a + b, ∠ABC = ∠ACB BD = DC =

A

B

Area =

Right Angled Triangle One of the angle in this triangle is 90°. h = Hypotenuse, b = Base p = Perpendicular

h p b

https://sscstudy.com/

Area =

s (s − a ) (s − b) (s − c) 1 = ×c×h 2

a+ b+ c 2 Perimeter = a + b + c

Where, s =

h

h



Related Formulae

1 × b× p 2 Perimeter = p + b + h h 2 = p 2 + b2 Area =

https://sscstudy.com/

NUMERICAL ABILITY

Different Types of Quadrilateral ●

Related Formulae

Parallelogram It is a quadrilateral with opposite sides parallel and equal. A b B a

a

h

b

D

C

a, b → Opposite parallel sides, h = Height ●

Trapezium Quadrilateral with one of the pair of opposite sides equal is called trapezium. a c

Area = Base × Height = b × h Perimeter = 2(a + b) ∠ABC = ∠ADC and ∠BAD = ∠BCD Area (∆ ADC ) = Area ( ∆ABC )

d

h

1 (Sum of parallel sides) × Height 2 1 = (a + b)h 2 Perimeter = a + b + c + d Area =

b a, b → Opposite parallel sides, h = Height ●

Rhombus It is a parallelogram whose all the four sides are equal. a O

a

d2

d1

a

1 × d1 × d 2 2 1 2 2 Sides (a ) = d1 + d 2 2 Perimeter = 4a 2 2 4a 2 = d1 + d 2 Area =

a

a = Sides, d1 , d 2 = Diagonals ●

Rectangle It is a parallelogram with equal opposite and each angle is equal to 90°. l A B d

b D



l 2 + b2

C

d = Diagonal

Square It is a parallelogram with all 4 sides equal and each angle is equal to 90°. a A B d

a D

Circle It is a plane figure enclosed by a curve on which every point is equally distant from a fixed point called centre inside the curve.

O r

A

B l

d2 2

Perimeter = 4 × Side = 4a Diagonal (d ) = a 2

C

a

r

Area = (Side) 2 = a 2 =

a

a = Sides, d = Diagonal ●

Diagonal (d ) =

b l

l = Length, b = Breadth,

Area = Length × Breadth = l × b Perimeter = 2(l + b)

Area = π r 2 Circumference (Perimeter) = 2 πr Diameter = 2r πrθ Length of the arc (l) = 180° πr 2 θ Area of sector AOB = 360° r = Radius 22 π= 7

https://sscstudy.com/

71

https://sscstudy.com/

72

CUET (UG) Section III : General Test

Regular Polygon



In a regular polygons, all sides and all interior angles are equal. 360° Each exterior angle = (n = Number of sides of n polygon) 360° Each interior angle = 180° − n   n ( n − 1) Number of diagonals of a polygon =  − n 2  



The area of the largest inscribed in a semicircle of radius r is equal to r 2 . Area of a square inscribed in a circle of radius r is equal to 2 r 2 .

Some Common Polygon Number of Sides (n)

Note ◆

If the length of a rectangle are increased by a% and b% ab  respectively, then area will be increased by  a + b +  %.  100 

Polygon

Area

5

Pentagon

5 a2

3 4

6

Hexagon

6a 2

3 4

7

Heptagon

8

Octagon

9

Nanogon

10

Decagon

2 ( 2 + 1) a 2

Solved Examples 1. A rectangular garden is 100 m long and 60 m

broad. It is surrounded by a 5m wide road. What is the area of the road? (a) 1600 m 2

(b) 1200 m 2

(c) 1000 m 2

(d) 1700 m 2

Alternate Method xy  Percentage increase in area =  x + y +   100  Here, x = 10 and y = 8

Sol. (d) Area of the rectangular garden ABCD = 100 × 60 = 6000 m 2 and area of the rectangular garden with road EFGH = (100 + 2 × 5) × (60 + 2 × 5) = 110 × 70 = 7700 m 2 E

10 × 8 4 = 18 % 5 100

3. A wire in the form of a square is cut and bent in

the form of a circle. If the area of the square is 110 cm2 , what is the area of the circle?

F A

= 10 + 8 +

(a) 160 cm 2 (c) 120 cm 2

B

(b) 140 cm 2 (d) 180 cm 2

2 Sol. (b) Area of the square = 110 cm

∴One side of the square = 110 cm ∴Perimeter of the square = 4 × 110 cm But the perimeter of the square = Circumference of the circle ∴ Circumference of the circle = 4 × 110 cm

C

D

G

H

∴ Area of road = 7700 − 6000 = 1700 m

2

2. If the length and breadth of a rectangle are

increased by 10% and 8% respectively, then by what per cent does the area of the rectangle increase? (a) 16

2 3

(b) 14

2 7

(c) 18

4 5

(d) 18

∴ Radius of the circle = =

2 5

Sol. (c) Let the original length and breadth of the rectangle are x and y. ∴Its original area = xy x × 110 11x Increased length = = 100 10 108 27 y and increased breadth = y × = 100 25 11x 27 y 297xy ∴Resulting area = × = 10 25 250 297xy 47xy − xy = ∴Increase in area = 250 250 4 47xy 100 94 ∴Percentage increase in area = × = = 18 % 5 250 xy 5

4 110 2× π 4 110 × 7 7 110 cm = 2 × 22 11 2

 7 110  ∴ Area of the circle = π ×    11  22 49 × 110 = × = 140 cm 2 7 11 × 11

Alternate Method Perimeter of the square = 4 110 cm ∴

https://sscstudy.com/

Area of the circle =

(Perimeter of the square)2 4p

=

(4 110 )2 × 7 4 × 22

=

16 × 110 × 7 = 140 cm 2 4 × 22

https://sscstudy.com/

NUMERICAL ABILITY

4. The radius of a circle is 11 cm. What is the area of

the square inscribed in the circle? (a) 212 cm 2 (c) 242 cm 2

(b) 232 cm 2 (d) 244 cm 2

Sol. (c)

73

Diagonal of the square = Diameter of the circle = 2 × 11 = 22 cm 1 ∴Area of the square = (Diagonal)2 2 1 = × 22 × 22 = 242 cm 2 2

Alternate Method Area of the square inscribed in a circle = 2r 2 r = 11= 2 × (11)2 = 242 cm 2

Here,

Practice Questions 1. If the ratio of the areas of two squares is 1 : 4, the ratio of their perimeters is (a) 1 : 6 (c) 1 : 2

(b) 1 : 8 (d) 1 : 4

(a) 126 cm

2. Perimeter of a triangle is 24 m and circumference of its incircle is 44 m. Find the area of the triangle. (a) 42 sq m (c) 48 sq m

(b) 24 sq m (d) 84 sq m

3. The ratio of the length of the parallel sides of a trapezium is 3 : 2. The shortest distance between them is 15 cm. If the area of the trapezium is 450 cm 2, then sum of lengths of the parallel sides is (a) 15 cm (c) 42 cm

(b) 36 cm (d) 60 cm

(b) 60 cm 2 (d) 96 cm 2

5. An equilateral triangle and a regular hexagon have the same perimeter. The ratio of the area of the triangle to that of the hexagon is (a) 3 : 2 (c) 1 : 2

(b) 162 cm

(c) 198 cm

(d) 251 cm

11. Tiling work of rectangular hall 60 m long and 40 m broad is to be completed with a square tile of 0.4 m side. If each tile cost ` 5, find the total cost of the tiles. (a) ` 60000

(b) ` 65000

(c) ` 75000

(d) ` 12000

12. A horse is placed for grazing inside a square field 12 m long and is tethered to one corner by a rope 8 m long. The area it can graze is (a) 50.18 sq m (c) 50.38 sq m

4. A parallelogram has sides 15 cm and 7 cm long. The length of one of the diagonals is 20 cm. The area of then parallelogram is (a) 42 cm 2 (c) 84 cm 2

10. A semicircular shaped window has diameter of 63 cm. 22  Its perimeter equals to  π =   7

(b) 50.28 sq m (d) 50.48 sq m

13. The diameters of two concentric circles are 8 cm and 10 cm. The area of the region between them is (a) 2π sq cm (c) 36π sq cm

(b) 4π sq cm (d) 9π sq cm

14. The diagonal of square field is 50 m. The area (in m 2) of the field is (a) 625

(b) 1250

(c) 2500

(d) 5000

15. The area of the shaded region in the given figure is

(b) 2 : 3 (d) 1 : 4

6. If the edge of a cube is increased by 100%, then the surface area of the cube is increased by (a) 100%

(b) 200%

(c) 300%

(d) 400%

7. A street of width 10 m surrounds from outside a rectangular garden whose measurement is 200 m × 180 m. The area of the path (in sq m) is (a) 8000 (c) 7500

(b) 7000 (d) 8200

45°

45°

a2 a2 (a) 2a 2 ( π − 2) (b) ( π − 2) (c) a 2 ( π − 1) (d) ( π − 1) 2 2

16. The area of the shaded region shown in the given figure is

8. The length of the three sides of a right angled triangle are ( x − 2) cm, x cm and ( x + 2) cm, respectively. Then, the value of x is (b) 8

(c) 4

(d) 0

14 cm

(a) 10

7 cm

9. The lengths of two sides of an isosceles triangle are 15 and 22, respectively. What are the possible values of perimeter? (a) 52 or 59 (c) 15 or 37

(b) 52 or 60 (d) 37 or 29

14 cm

7 cm 7 cm

28 cm

(a) 515 cm 2 (c) 505 cm 2

https://sscstudy.com/

(b) 535 cm 2 (d) 525 cm 2

https://sscstudy.com/

74

CUET (UG) Section III : General Test

17. A square field with side 30 m is surrounded by a path of uniform width. If the area of the path is 256 m 2, the width of the path is (a) 14 m (c) 4 m

(b) 16 m (d) 2 m

1 3

(c) 9

(a) 8 (c) 12

19. The area of the four walls of a room is 128 sq m. The length is equal to the width and the height is 4 m. The area of the floor of the room is (a) 32 sq m (c) 64 sq m

(b)

(d) 3

1 times as 2 high as long. If the cost of carpeting the floor at ` 3 per sq m, is ` 144, the height (in m) of the room is

(b) 3 3 sq cm 5 3 (d) sq cm 2

(c) 3 6 sq cm

(a) 2

21. A room is half as broad as it is high and 1

18. An equilateral triangle of side 6 cm has its corners cut off to form a regular hexagon. The area of this hexagon is (a) 6 3 sq cm

20. If the radius of a circle is tripled, its perimeter will become how many times of its previous perimeter?

(b) 10 (d) 16

22. A rectangular plot 90 m × 50 m has two 10 m wide roads running in the middle of it, one parallel to the length and the other parallel to the breadth. Area of the plot used in roads is (a) 1300 sq m (c) 1500 sq m

(b) 49 sq m (d) 81 sq m

(b) 1400 sq m (d) 1200 sq m

ANSWERS 1. 11. 21.

(c) (c) (c)

2. 12. 22.

(d) (b) (a)

3. 13.

(d) (d)

4. 14.

(c) (b)

5. 15.

(b) (b)

6. 16.

(c) (d)

7. 17.

(a) (d)

8. 18.

(b) (a)

9. 19.

(a) (c)

10. 20.

(b) (d)

Hints & Solutions 1. Ratio of perimeters = Ratio of areas = 1 : 4 = 1: 2 2. If inradius of the triangle be r, there 2πr = 44 44 r= =7m ⇒ 2π ∆ Using r = S 24 ⇒ ∆ = 7× 2 = 7 × 12 = 84 m2

= 21 × 6 × 14 × 1 = 42 cm 2

15 cm

20 cm

D

B

7 cm

C

∴ Area of the parallelogram = 2 × Area of ∆ ABC = 2 × 42 = 84 cm 2 5. Let the side of an equilateral triangle is a and side of hexagon is b. According to question, Perimeter of equilateral triangle = Perimeter of hexagon 3a = 6b a =2 b 3 2 a ∴ Ratio of the areas = 4 3 2 6× b 4 2 a2 1  a = 2=   6  b 6b 1 2 2 = (2) = 6 3

https://sscstudy.com/

6. The required surface area increased by  100 + x 2  =   − 1 × 100%    100   100 + 100 2  =   − 1 × 100%  100     200 2  =   − 1 × 100%    100  = [(2)2 − 1] × 100% = (4 − 1) ×100% = 300% 7.

10 200m 10

180m

3. Let the length of the parallel sides of a trapezium be 3x cm, 2x cm. Given, Area of trapezium = 450 1 × (2x + 3x) × 15 = 450 2 450 × 2 = 60 cm 5x = 15 ∴ Required sum = 60 cm 15 + 7 + 20 4. In ∆ ABC, S = = 21 2 Area of ∆ ABC = 21(21 − 15)(21 − 7)(21 − 20)

A

10

10

∴ The area of the path = (200 + 10 × 2) × (180 + 10 × 2) − 200 × 180 = 220 × 200 − 200 × 180 = 44000 − 36000 = 8000 m 2

https://sscstudy.com/

NUMERICAL ABILITY

8. In right angled triangle (Hypotenuse)2 = (Base)2 + (Perpendicular)2 2 ⇒ (x + 2) = x2 + (x − 2)2 2 ⇒ x + 4 x + 4 = x2 + x2 − 4 x + 4 ⇒ x2 = 8 x ⇒ x=8 9. Perimeter of isosceles triangle = 15 + 15 + 22 or 15 + 22 + 22 = 52 or 59 units 10. Perimeter of semicircular shaped window = (πr + 2r ) cm = r (π + 2) cm 63  22  = + 2 cm   2  7 63 36 = 162 cm = × 2 7 60 × 40 11. Number of tiles = 0.4 × 0.4 = 15000 Total cost of the tiles = 15000 × 5 = ` 75000 1 12. The required area = [π (8)2] 4 1 22 = × × 64 = 50. 28 sq m 4 7 13. The area of the region between two concentric circles = π (R2 − r 2) = π (52 − 42) = π (25 − 16) = 9π cm 2 14. Area of the square field 1 = (Diagonal)2 2 1 = (50)2 2 = 1250 sq m 15. If one side of the triangle is x, then in right angled triangle x2 + x2 = 4 a 2 x=a 2 ∴ ∴ Area of the right angled triangle 1 = × a 2 × a 2 = a2 2

and area of the semicircle 1 = πa 2 2 ∴Area of the shaded region 1 = πa 2 − a 2 2 1 = a 2(π − 2) 2 16. Area of the whole external rectangle = 28 × 14 = 392 cm 2 Area of the whole semicircle 1 22 = × × 14 × 14 2 7 = 308 cm 2 ∴ Area of the whole external figure = 392 + 308 = 700 cm 2 and area of the whole inner figure 1 22 = 14 × 7 + × ×7×7 2 7 = 98 + 77 = 175 cm 2 ∴ Area of the shaded region = 700 − 175 = 525 cm 2 17. Let the breadth of the path x m. ∴(30 + 2x)2 − (30)2 = 256 ⇒ (30 + 2x + 30)(30 + 2x − 30) = 256 30 m

⇒ (60 + 2x) × 2x = 256 ⇒ x2 + 30x − 64 = 0 ⇒ (x − 2)(x + 32) = 0 ∴ x = 2 or −32 So, the breadth of the path be 2 m. 18. Q One side of an equilateral triangle = 6 cm ∴One side of the regular hexagon = 2 cm

https://sscstudy.com/

75

∴ Area of the hexagon 3 3 3 = × (6)2 − × (2)2 4 4 3 = [36 − 12] 4 3 = × 24 4 = 6 3 cm 2 19. Let the length of the room be x m. Q 128 = 2 × 4 (x + x) ∴ x=8m So, area of the floor = 64 m 2 20. Let the radius of the initial circle be r. ∴Radius of the resulting circle = 3r ∴ Perimeter of the initial circle = 2πr and perimeter of the resulting circle = 2π (3r ) = 6πr 6 πr =3 ∴ Required ratio = 2 πr 21. Let the height of the room be x m. 2 Length = x × ∴ 3 2x m = 3 x and Breadth = m 2 2x x × ∴ Area of the floor = 3 2 x2 sq m = 3 2 x Q × 3 = 144 3 ⇒ x = 12 m 22. Area of road = Width of road × [Length of plot + Breadth of plot − Width of road] = 10 × [90 + 50 − 10] = 10 × 130 = 1300 sq m

https://sscstudy.com/

76

CUET (UG) Section III : General Test

CHAPTER 15

Volume and Surface Area In this chapter, we have to deal with the problem of finding the volume and surface area of solid figures. As we know that, solid figures have length, breadth and height (thickness). Hence, this chapter can be featured as 3D and mensuration too.

Volume

Surface Area

It is the space occupied within the boundary of a 3D figure. Unit cu cm, cu m (i.e., cube units) etc.

It is the total area that can be measured on the entire surface. This can only be measured, if the object is a 3D object surface area is measured in square unit.

Important Formulae Related to Solid Figures Name

Figure

Lateral/Curved Surface Area

h

Cuboid

Total Surface Area

Volume

Nomenclature

2h (l + b)

2 (lb + bh + h l )

lbh

4a 2

6a 2

a3

l = Length b = Breadth h = Height

b I

a

Cube

a = Edge

a a

b h

Right prism

(Perimeter of base) × Height

2 (Area of base) + Lateral surface area

Area of base × Height

r

Right circular cylinder

h

2πrh

2πr (r + h )

https://sscstudy.com/

πr 2h

r = Radius of base h = Height of the cylinder

https://sscstudy.com/

NUMERICAL ABILITY

Name

Figure

Lateral/Curved Surface Area

Total Surface Area

(Perimeter of the base) × (Slant height)

Area of the base + Lateral surface area

1 (Area of the base) 3 × Height

πrl

πr (l + r )

1 πr 2h 3

h = Height r = Radius l = Slant height



4 πr 2

4 πr 3 3

r = Radius

2 πr 2

3 πr 2

2 πr 3 3

r = Radius



4 π (R 2 − r 2 )

4 π (R 3 − r 3 ) 3

R = Outer radius r = Inner radius

l

h

Right pyramid r

Right circular cone

Volume

l

h

Nomenclature

r

r

Sphere

Hemisphere r

R

Spherical shell

77

r

Solved Examples 1. If the surface of a cube is 216 sq cm, its volume

will be (a) 108 sq cm (c) 216 sq cm

If the capacity of the tank is 21 hectolitre, then the height is

(b) 36 cu cm (d) 216 cu cm

(a) 0.9 m

2 Sol. (d) Surface of a cube = 6 × (Side)

∴ ∴

Sol. (c) Volume = Area × h = 15 × 4 = 60 cu m

(d) 10 m

Capacity = l × b × h ⇒ 21 . = 21 . × 2× h 2.1 h= = 0.5 m 2.1 × 2

the per cent decrease in the surface area of the cube. (a) 64

15 sq m and the height of the room be 4 m, then how much air is in the room? (c) 60 cu m

(c) 0.7 m

4. If each edge of a cube is decreased by 40%. Find

2. If the area of the floor of a rectangular room be

(a) 11 cu m

(b) 0.5 m

. cu m Sol. (b) Capacity = 21 hectolitre = 2100 L = 21

∴ 216 = 6 × x2 Where, x is the side of cube. 216 = 36 x2 = 6 ∴ x = 36 = 6 cm ∴Volume of the cube = 6 × 6 × 6 = 216 cu cm

15 cu m (b) 4 (d) 19 cu m

3. One rectangular tank is 2.1 m long and 2 m broad.

(b) 84

(c) 80

(d) 74

Sol. (a) According to the formula, Percentage decrease in surface area  (− 40)2   (1600)  =  2 (− 40) +  =  − 80 + 100  100    = (− 80 + 16) = − 64% Negative sign shows that decrease takes place here.

https://sscstudy.com/

https://sscstudy.com/

78

CUET (UG) Section III : General Test

Practice Questions 1. The base of right prism is an equilateral triangle with a side of 7 m and its height is 24 m. Find its volume. (a) 509 m3 (c) 529 m3

(b) 1018 m3 (d) 519 m3

2. To raise the height of a low land 48 m long and 31.5 m broad to 6.5 dm, a ditch 27 m long and 18.2 m broad was dug in a side plot, the depth of the ditch will be (a) 5 m (c) 1 m

(b) 7 m (d) 2 m

(b) 15.5 kg (d) 18.5 kg

(b) 2 : 3 (d) 4 : 3

(b) 2 : 3 (d) 3 : 4

6. A sphere and a cube have equal surface areas. The ratio of the volume of the sphere to that of the cube is (a) (c)

π: 6 2: π

(b) (d)

6: π π:3

7. A sphere exactly fits inside a hollow cylinder closed at both ends. The ratio of the volume of the empty space in the cylinder to the volume of the sphere is (a) 2 : 1 (c) 2 : 3

(b) 1 : 2 (d) 3 : 2

8. If the length of longest rod that can be placed within the cuboid is 5 5 m long and the sum of lenght breadth and height is 19 m long, then find the whole surface area of that cuboid. (a) 236 m 2 (c) 125 m 2

(b) 256 m 2 (d) 361 m 2

9. The number of spherical bullets that can be made out of a solid cube of lead whose edge measures 44 cm, 22  each bullet being of 4 cm diameter, is  take π =   7 (a) 2541 (c) 2514

(b) 2451 (d) 2415

10. The length of longest pole that can be placed in a room of 12 m long, 8 m broad and 9 m high is (a) 12 m (c) 19 m

(b) 17 m (d) 21 m

(b) 33.1% (d) 42%

13. A sphere of radius 2 cm is put into water contained in a cylinder of radius 4 cm. If the sphere is completely immersed in the water, the water level in the cylinder rises by (b)

1 cm 3

(c)

1 cm 2

(d)

2 cm 3

14. A sphere and a right circular cylinder have the same radius r. If their volumes are equal, the height of the cylinder is (a)

5. A sphere and a cylinder have equal volume and equal radius. The ratio of the curved surface area of the cylinder to that of the sphere is (a) 4 : 3 (c) 3 : 2

(b) 14 cm (d) 16 cm

12. If the radius of base and height of a cone are increased by 10%, then the volume of the cone is increased by

(a) 2 cm

4. The radii of a sphere and a right circular cylinder are equal and their curved surface areas are also equal. The ratio of their volumes is (a) 3 : 4 (c) 3 : 2

(a) 10 cm (c) 18 cm

(a) 30% (c) 40%

3. Weight of a solid metal sphere of radius 4 cm is 4 kg. The weight of a hollow sphere made with same metal, whose outer diameter is 16 cm and inner diameter is 12 cm, is (a) 20.5 kg (c) 16.5 kg

11. Three cubes of iron of edges 9 cm, 12 cm and 15 cm respectively are melted to form a large single cube. The edge of the new cube is

4 r 3

(b)

3 r 4

(c)

2 r 3

(d)

3 r 2

15. If the ratio of surface areas of two sphere is 9 : 16, then the ratio of their volume is (a) 3 : 4 (c) 27 : 64

(b) 9 : 16 (d) 81 : 256

16. The radius of the base and height of a cone are 3 cm and 5 cm respectively whereas the radius of the base and height of a cylinder are 2 cm and 4 cm respectively. The ratio of the volume of the cone to that of the cylinder is (a) 15 : 8 (c) 15 : 16

(b) 45 : 16 (d) 1 : 3

17. A rectangular block 6 cm × 42 cm × 45 cm is cut up into exact number of equal cubes. The least possible number of cubes will be (a) 30

(b) 210

(c) 330

(d) 420

18. Two cylindrical buckets have their diameters in the ratio 3 : 1 and their heights are as 1 : 3. Their volumes are in the ratio (a) 1 : 2

(b) 2 : 3

(c) 3 : 1

(d) 3 : 4

19. A sphere is cut into two hemispheres. One of them is used as a bowl. It takes 8 bowlfuls of this to fill a conical vessel of height 12 cm and radius 6 cm. The radius of the sphere is (a) 2 cm (c) 4 cm

(b) 3 cm (d) 6 cm

20. A cone of height 7 m and of base radius 3 m is carved from a rectangular block of wood of dimensions 10 m × 5 m × 4 m. The percentage of volume of the block left out is (a) 67% (c) 34%

https://sscstudy.com/

(b) 66% (d) 33%

https://sscstudy.com/

NUMERICAL ABILITY

21. Three cubes of metal whose edges are in the ratio 3 : 4 : 5 are melted to form a single cube whose diagonal is 12 3 cm. The edges of the three cubes (in cm) are (a) 9, 12, 15 (c) 6, 8, 10

23. The surface areas of a cylinder, a cone and a hemisphere of smae radii are equal. The ratio between height of the cylinder and cone is

(b) 15, 20, 25 (d) 8, 10, 12

(b) 1 : 2 3 (d) 3 : 2

(a) 2 3 : 1 (c) 2 : 3

22. A solid cylinder of diameter 14 mm and length 25 mm has a volume 3850 mm 3. If the length were doubled and the diameter halved, the new volume would be (a) 1172 mm3 (c) 3850 mm3

79

24. If the side of two cubes are in the ratio 3 : 1, the ratio of their total surface areas is

(b) 1925 mm3 (d) 7700 mm3

(a) 3 : 1 (c) 9 : 1

(b) 8 : 1 (d) 12 : 1

ANSWERS 1. 11. 21.

(a) (c) (c)

2. 12. 22.

(d) (b) (b)

3. 13. 23.

(d) (d) (b)

4. 14. 24.

(b) (a) (c)

5. 15.

(c) (c)

6. 16.

(b) (c)

7. 17.

(b) (d)

(a) (c)

8. 18.

9. 19.

(a) (b)

10. 20.

(b) (a)

Hints & Solutions 1. Area of base of right prism 3 49 3 2 m (7)2 = 4 4 Volume of right prism = Area of base × Height 49 3 = × 24 4 = 6 × 49 × 3 = 294 × 1.732 = 509.222 ≈ 509 m3 =

2. Let the depth of the ditch be h m. According to the question, 6. 5 48 × 31. 5 × 10 h= 27 × 18.2 9828 = =2m 4914 3. Volume of solid sphere of radius 4 4 cm = π (4)3 3 Volume of hollow sphere 4 = π [(8)3 − (6)3 ] 3 4 Q Weight of π (4)3 cm3 = 4 kg 3 4 ∴ Weight of π [(8)3 − (6)3 ] cm3 3 4 4 = ⋅ π [(8)3 − (6)3 ] 4 π (4)3 3 3 4 (512 − 216) = 18.5 kg = 43

4. Given, 4πr 2 = 2πrh ⇒

h = 2r

Now, required ratio =

4 3 πr : πr 2h 3

= 4r : 3h = 4r : 6r

(Q h = 2r )

= 2 :3 5. According to question, Volume of sphere = Volume of cylinder 4 3 πr = πr 2h 3 4 h= r 3 ∴ Required ratio =

Curved surface of sphere Curved surface of cylinder

=

4 πr 2 = 2πrh

3 4 πr 2 = 4  2 2πr  r 3 

6. Let radius of sphere = r and side of cube = a According to question, Surface area of sphere = Surface area of cube 2

4πr = 6a 2 r 3 = a 2π 4 3 πr ∴ Ratio of volumes = 3 3 a

https://sscstudy.com/

=

3 4  r 4  3   π  = π 3  a 3  2π

=

4π × 3 3 = 3 ×2 2 × π π

3

6 π

7. Radius of the sphere = Radius of the base of the cylinder and height of the cylinder = Diameter of the sphere = 2r 4 Volume of sphere = πr3 3

and volume of the cylinder = πr 2(2r ) = 2πr3 ∴ Volume of the empty space 4 = 2πr3 − πr3 3 2 3 = πr 3 2πr3 4 πr3 ∴ Required ratio = : 3 3 = 1 :2 8. Given, l + b + h = 19 and

2

2

2

l + b + h =5 5

...(i) ...(ii)

On squaring Eq. (i), we get (l + b + h )2 = (19)2 l2 + b2 + h 2 + 2(lb + bh + hl) ...(iii) = 361

https://sscstudy.com/

80

CUET (UG) Section III : General Test

Now, on squaring Eq. (ii), ( l2 + b2 + h 2 )2 = (5 5 )2 l2 + b2 + h 2 = 125 Now putting the obtaining value in Eq. (iii), we get 125 + 2(lb + bh + hl) = 361 2(lb + bh + hl) = 361 − 125 Whole surface area of cuboid = 236 m 2 9. Total number of spherical bullets Volume of solid cube = Volume of 1 bullet 44 × 44 × 44 = = 2541 4 22 × ×2 ×2 ×2 3 7 10. Length of the longest pole =

122 + 82 + 92

= =

144 + 64 + 81 289 = 17 m

32 (Q r = 4 cm) 3 2 h = cm ⇒ 3 4 14. Volume of the sphere = πr3 3 and volume of the cylinder = πr 2h 4 Q πr 2h = πr3 3 4 h= r ∴ 3 (4)2h =

15. Let r1 and r2 be the radii of spheres. 4πr12 9 = ∴ 4πr22 16 r12 9 r 3 = ⇒ 1 = r2 4 r22 16



4 3 4 3 πr1 : πr2 = r13 : r23 3 3 = 33 : 43 = 27 : 64 1 16. Volume of the cone = π (3)2 × 5 3 = 15π cm3 ∴

11. Volume of the new cube = (9)3 + (12)3 + (15)3 = 729 + 1728 + 3375 = 5832 cm3 ∴One side of the new cube = 18 cm 12. Let the original radius and height be r and h, respectively. ∴ Volume of the original cone 1 = πr 2h 3 and increased volume 2

1  110r   110h  π  ×   100  3  100  1 = π × 1.331 r 2h 3 ∴Percentage of increase 1 2 πr h (1.331 − 1) =3 × 100 1 2 πr h 3 = 33.1% 13. Volume of the sphere 4 32π cm3 = π (2)3 = 3 3 Let the water raised h m when a sphere is immersed in it. Volume of water (cylinder) =

= Volume of sphere 32π π r 2h = 3

and volume of the cylinder = π (2)2 × 4 = 16 π cm3 ∴ Required ratio = 15 : 16 17. For the least number of cubes, the edge of the cube will be largest. ∴ HCF of 6, 42 and 45 = 3 ∴ Volume of 1 cube = 3 × 3 × 3 = 27 cm3 ∴ Required number of cubes 6 × 42 × 45 = = 420 27 18. Let the diameter and height of one bucket 3x m and h m. ∴The diameter and height of other bucket will x m and 3 h m. 2

2

 3x  x ∴V1 : V 2 = π   × h : π   ⋅ 3 h  2  2 2

=

2

9 πx h 3 πx h : = 3 :1 4 4

19. Volume of the cone 1 = π × 36 × 12 3 = 144π cm3 If the radius of the sphere be r cm, then

https://sscstudy.com/

2 3 πr × 8 = 144π 3 ∴ r = 3 cm 20. Volume of the block left  1 22  = (10 × 5 × 4) −  × × 3 × 3 × 7 3  7 = 200 − 66 = 134 m3

134 × 100 200 = 67%

∴Required percentage =

21. Let the edges of the three cubes be 3x cm, 4x cm and 5x cm. ∴ Volume of the resulting cube = (3x)3 + (4x)3 + (5x)3 = 216 x3 ∴Edge of the resulting cube = 6x cm ∴Diagonal the resulting cube = 6x 3 cm 6x 3 = 12 3 12 3 ∴ x= = 2 cm 6 3 ∴The edges of three cubes are 6 cm, 8 cm and 10 cm. 22. Q Diameter = 14 mm ∴ Diameter of new cylinder = 7 mm ∴ Length of new cylinder = 50 mm 2  7 π   × 50 = x  2 ∴ x = 1925 mm3 23. Let the radius of each solid be r and the heights of the cylinder and the cone be h1 and h2 respectively. 2πr 2 + 2πrh1 = 3πr 2 r ∴ h1 = 2 and also 3πr 2 = πr h22 + r 2 + πr 2 ⇒ ⇒ ⇒ ∴ ∴

πr h22 + r 2 = 2π r 2 h22 + r 2 = 4 r 2 h22 = 3 r 2 h2 = 3 r r h1 = 2 = 1 :2 3 h2 3r

24. The ratio of their total surface (3)2 areas = = 9 :1 (1)2

https://sscstudy.com/

NUMERICAL ABILITY

81

CHAPTER 16

Algebra Polynomial f ( x ) = a0x n + a1x n − 1 +..... + an ( a0 ≠ 0) is called a polynomial in variable x, where a0 , a1 , ... , an are real numbers and n is a non-negative integer, is called degree of polynomial. e.g. Polynomial ( x − a ) is a degree of 1 and polynomial x 2 − 7x + 12 is a degree of 2. Note

¿

Ex. 1 Find the quotient and the remainder when x4 + 1 is divide by x − 1. (a) x3 + x2 + x + 1, 2 (c) x3 + x2 − x + 1, 3

Sol. (a) Using long division method, x3 + x2 + x + 1 x − 1) x4 + 1 x4 − x3 − + x3 + 1 x3 − x2 − +

If f (x) = 0, then it is said to be polynomial equation.

Fundamental Operations on Polynomials Some operations based on polynomials are discussed below 1. Addition of Polynomials Polynomials can be added by arranging their like terms and combining them. 2. Subtraction of Polynomials Polynomials can be subtracted by arranging their like terms and by changing sign of each term of the polynomial to be subtracted and then added. 3. Multiplication of Polynomials We know that, (i) the product of two factors with like signs is positive and product of unlike signs is negative. (ii) if x is any variable and m , n are positive integers, then x m × x n = x m + n Thus, x3 × x 6 = x (3 + 6) = x 9. 4. Division of a Polynomial by a Polynomial The following steps are given below Firstly, arrange the terms of the dividend and divisor in descending order of their degrees. Divide the first term of the dividend by the first term of the divisor to obtain the first term of the quotient. Multiply all the terms of the divisor by the first term of the quotient and subtract the result from the dividend. Consider the remainder (if any) as a new dividend and proceed as before. Repeat this process till we obtain a remainder which is either 0 or a polynomial of degree less than the degree of the divisor. ●







(b) x3 + x2 − x + 1, 2 (d) None of these

x2 + 1 x2 − x − + x+ 1 x −1 − + 2 3

2

Hence, quotient = x + x + x + 1and remainder = 2.

Linear Equations In One Variable The expression of the form ax + b = 0, where a and b are real numbers and a ≠ 0, is a linear polynomial of one variable and equation involving only linear polynomial are called linear equations of one variable. e.g. 5x + 8 = 9 − x is a linear equation in one variable. Graph of linear equation of one variable is a straight line, which is either parallel to the horizontal and vertical axis. Linear equation in one variable has unique solution. ●



In Two Variables An equation of the form ax + by + c = 0, where a , b, c, ∈ R , a ≠ 0, b ≠ 0 and here x , y are variables is called a linear equation in two variables. e.g. 2x + 3 y = 5, 2 x + 3 y = 0,



2a + 3b = 0 are linear equations in two variables.

https://sscstudy.com/

https://sscstudy.com/

82







CUET (UG) Section III : General Test

The linear equation in two variables ax + by + c = 0 has an infinite number of solutions. The graph of equation ax + by + c = 0 is a straight line, so it is called as linear equation. Every point on graph of ax + by + c = 0 gives it solution.

2 3 5 + = , where x−3 x− 4 x x ≠ 3, x ≠ 4 and x ≠ 0



Ex. 4 Which option is correct, for the following pair of

1 (b) 3 2

equations? x + 2 y − 4 = 0 and 3 x + 6 y − 12 = 0 (a) Consistent (b) Consistent (dependent) (c) Inconsistent (d) None of these

(d) None of these

Sol. (a) Given that,

2 3 5 + = x−3 x−4 x

2(x − 4) + 3(x − 3) 5 = (x − 3) (x − 4) x



(5x − 17) x = 5 (x2 − 7x + 12)



5x2 − 17x = 5x2 − 35x + 60 1 60 =3 18x = 60 ⇒ x = 3 18



If

Hence, it represent a pair of parallel lines.

Ex. 2 Solve 1 (a) 3 3 1 (c) 3 4

a1 b1 c1 = = , then system has infinite solutions and a2 b2 c2 represents overlapping lines. Inconsistent System The given system will be a b c inconsistent, if 1 = 1 ≠ 1 and do not have any solution. a2 b2 c2 ●

Sol. (b) Given, pair of linear equations is x + 2 y − 4 = 0 and 3x + 6 y − 12 = 0 On comparing with standard form of pair of linear equations, we get a1 = 1, b1 = 2, c1 = − 4

1 So, x = 3 is a solution of the given equation. 3

and Now,

Ex. 3 The length of a rectangle is 8 cm more than its and

breadth. If the perimeter of the rectangle is 68 cm, its length and breadth are respectively (a) 21 cm, 13 cm (c) 23 cm, 13 cm

(b) 22 cm, 21 cm (d) 24 cm, 21 cm

Clearly,

Hence, the given pair of linear equations is consistent (dependent).

Sol. (a) Let the breadth of the rectangle be x cm. Then, its length = (x + 8) cm ∴ Perimeter of rectangle = 2 [x + (x + 8)]

Factor and Factorisation

= 4x + 16



According to the given condition, 4x + 16 = 68 ⇒

a2 = 3, b2 = 6, c2 = − 12 a1 1 b1 2 1 = , = = a2 3 b2 6 3 c1 −4 1 = = c2 − 12 3 a1 b1 c1 1 = = = a2 b2 c2 3

4x = 52 ⇒ x = 13



∴Breadth of rectangle = 13 cm and length of rectangle = 13 + 8 = 21cm .

A polynomial g( x ) is called a factor of polynomial p( x ), if g ( x ) divides p( x ) exactly. To express polynomial as the product of polynomials of degree less than that of the given polynomial is called as factorisation.

Consistency of the System of Linear Equations

Factorisation by Common Factors

A set of linear equations is said to be consistent, if there exists atleast one solution for these equation. A set of linear equations is said to be inconsistent, if there is no solution for these equation. Let us consider a system of two linear equations as shown. a1x + b1 y + c1 = 0 and a2x + b2 y + c2 = 0.

A factor which occurs in each terms, is called the common factor. e.g. Factorise 16x 2 y + 4xy We have, 16x 2 y = 2 × 2 × 2 × 2 × x × x × y and 4xy = 2 × 2 × x × y Here, 2 × 2 × x × y is common in these two terms.

Consistent System The above system will be consistent, a b a b c if 1 ≠ 1 or 1 = 1 = 1 a2 b2 a2 b2 c2 a1 b1 If ≠ , then system has unique solution and a2 b2 represents a pair of intersecting lines. ●

Factorisation by Splitting Middle Term Let factors of the quadratic polynomial ax 2 + bx + c be ( px + q ) and (r x + s). Then, ax 2 + bx + c = ( px + q ) (r x + s) = prx 2 + ( ps + qr )x + qs On comparing the coefficients of x 2, x and constant terms from both sides, we get a = pr, b = ps + qr and c = qs. Here, b is the sum of two numbers ps and qr, whose product is ( ps)( qr ) = ( pr )( qs ) = ac.

https://sscstudy.com/

https://sscstudy.com/

NUMERICAL ABILITY

Thus, to factorise ax 2 + bx + c, write b as the sum of two numbers, whose product is ac. 2

2

Note To factorise ax + bx − c and ax − bx − c, write b as the difference of two numbers whose product is (– ac).



Ex. 5 Factors of 2x + 7 x + 3 are ●

a = 2, b = 7

(a) − 7 (c) − 6

c=3 ac = 2 × 3 = 6

If q (x) is divisible by 2x − 1, then (2x − 1) is a factor of q (x). 1 Consider 2x − 1 = 0 ⇒ x = 2 1 On putting x = in q(x), we have 2

Clearly, pair 1 and 6 gives 1+ 6 = 7 = b 2

2x + 7x + 3 = 2x2 + (1+ 6)x + 3 = 2x2 + x + 6x + 3 = x(2x + 1) + 3(2x + 1)

3

Factorisation by Algebraic Identities



Sometimes, we do a factorisation with the help of algebraic identities, which are given below. 1. ( a 2 − b2 ) = ( a + b)( a − b)



2. ( a + b)2 = a 2 + b2 + 2ab and ( a − b)2 = a 2 + b2 − 2ab



2

3. ( a + b) − ( a − b) = 4ab and ( a + b)2 + ( a − b)2 = 2 ( a 2 + b2 ) 5. ( a + b)3 = a3 + b3 + 3ab ( a + b) 7. ( a3 + b3 ) = ( a + b)( a 2 + b2 − ab) 2



2

8. ( a − b ) = ( a − b)( a + b + ab)



9. a3 + b3 + c3 − 3abc = ( a + b + c)( a 2 + b2 + c2 − ab − bc − ac)

5 p = − 35



p=−7

The second degree equation of polynomial is called quadratic equation. The general quadratic equation is given by ax 2 + bx + c = 0, where a , b, c are real numbers and a ≠ 0.

Roots of a Quadratic Equation

10. If a + b + c = 0, then a3 + b3 + c3 = 3abc

A value of a variable which satisfies the particular quadratic equation is called root of that equation or solution of the equation. e.g. Let the equation is x 2 − 6x + 8 = 0.

Ex. 6 Factorise 8 a 3 − 343 b3 (a) (2a + 7b) (4a 2 + 14ab + 49b2 ) (b) (2a − 7b) (4a 2 + 14ab + 49b2 ) (c) (2a − 7b) (4a 2 − 14ab + 49b2 ) (d) None of the above

Here, we take x = 2 , then 22 − 6 ( 2) + 8 = 0 So, x = 2 is a root of the quadratic equation.

Sol. (b) 8a3 − 343b3 = (2a )3 − (7b)3 = (2a − 7b) [(2a )2 + (2a ) (7b) + (7b)2 ] = (2a − 7b) (4a 2 + 14ab + 49b2 )

Factorisation by Using Theorems 1. Remainder Theorem ●

5 p + 35 = 0



Quadratic Equation

6. ( a − b)3 = a3 − b3 − 3ab ( a − b) 3

1 1 11 2× + p × + + p + 3 = 0 8 4 2 1 p 11 + + + p+ 3 = 0 4 4 2 1+ p + 22 + 4 p + 12 =0 4



4. ( a + b + c)2 = a 2 + b2 + c2 + 2 ( ab + bc + ca )

3

2

1 1 1 1 q   = 2 ×   + p   + 11  + p + 3 = 0  2  2  2  2

= (2x + 1) (x + 3)

2

(b) 7 (d) 5

Sol. (a) Let q (x) = 2x3 + px2 + 11x + p + 3

So, all possible pairs of factors of 6 are 1 and 6, 2 and 3.



Let p( x ) be a polynomial in x of degree not less than one and α be a real number. If p(α ) = 0, then ( x − α ) is factor of p( x ).

Ex. 7 The value of p, if (2x − 1) is a factor of 2x 3 + px2 + 11x + p + 3, is

On comparing with ax2 + bx + c, we get

Now,

Remainder can be evaluated by substituting, x = α in p(x).

Note If (x − α ) is a factor of p (x ), then p(α ) = 0.

Sol. (b) Given polynomial is 2x2 + 7x + 3

and

¿

2. Factor Theorem ●

(b) (2x + 1) (x + 3) (d) (2x − 2) (x − 3)

If p( x ) is divided by ( x − α ), then remainder is f(α ).

Note

2

(a) (x + 2) (x + 1) (c) (x + 3) (2x − 1)

83

Let p( x ) be the polynomial in x of degree not less than one and α be a real number.

Solution of a Quadratic Equation The solution of a quadratic equation can be find by two methods. 1. By Factorisation Method Let the quadratic equation be ax 2 + bx + c = 0. If the factors of ax 2 + bx + c are ( x + α ) ( x + β ), then the solution is x = − α , − β.

https://sscstudy.com/

https://sscstudy.com/

84

CUET (UG) Section III : General Test

2. By Quadratic Formula If given equation is ax 2 + bx + c = 0, then roots of a quadratic equation can be determined by the formula

Sol. (c) Given equation is 2x2 + 14x + 9 = 0. Then, x=

− b ± b2 − 4ac x= 2a

=

This formula is known as Sridharacharya Formula, where ( b2 − 4ac) is the discriminant (4) of the equation.

Nature of Roots of a Quadratic Equation

If D > 0, then the two roots are real and unequal. If D = 0, then the two roots are real and equal. If D < 0, then there are no real roots. If D > 0 and D is perfect square, then roots are rational. 5. If D > 0 and D is not a perfect square, then roots are irrational. 1. 2. 3. 4.

¿

If one of the roots of the quadratic equation is a + then its another root will be a − b.

b,

Sum and Products of the Roots Let α , β be the roots of the equation ax 2 + bx + c = 0

−7 + 31 2 −7 + 31 −7 − 31 (c) , 2 2

(b)

−7 − 31 2

− 14 ± 124 4

− 14 ± 2 31 − 7 ± 31 = 4 2 − 7 + 31 − 7 − 31 and ⋅ 2 2

1 Ex. 9 If  a −  = 6, then ( a4 + 1 / a4 ) = ? a



(a) 1444 (c) 34

(b)38 (d) 1442

Sol. (d) a −

1 =6 a 2

1 On squaring both side  a −  = (6)2  a 1 ⇒ a 2 + 2 − 2 = 36 a 1 a 2 + 2 = 38 ⇒ a  a 2 + 1  = 382    a2  1 a 4 + 4 + 2 = 1444 a 1 a 4 + 4 = 1442 a

⇒ ⇒

If the roots of equation are given to us say α and β, then S = Sum of roots = α + β

(a)

=

2

Formation of a Quadratic Equation

Ex. 8 Solve the equation 2x + 14 x + 9 = 0 .

196 − 72 4



b a c 2. The product of the roots , α ⋅ β = a

2

− 14 ±

Again squaring both side

1. The sum of the roots, α + β = −

and P = Product of roots = αβ ∴ The quadratic equation will be x 2 − (α + β )x + αβ = 0 or x 2 − Sx + P = 0

2 (2)

∴The roots are

Let D = b2 − 4ac be the discriminant of the quadratic equation ax 2 + bx + c = 0.

Note

=

(14)2 − 4 (2) (9)

− 14 ±

Ex. 10 Divide 36 into two parts, such that 5 times of the first part is more than 8 times of the second part by 24? (a) 20, 16 (c) 26, 10

(b) 24, 12 (d) 22,14

Sol. (b) Let two parts are ‘x’ and ‘y’ 5x − 8 y = 24

then

x + y = 36

and

On multiplying with 8 both side, 8x + 8 y = 288 Adding in Eq. (i) 13x = 288 + 24 ⇒ 13x = 312

(d) None of these

x = 24 and y = 36 − 24 = 12

https://sscstudy.com/

…(i)

https://sscstudy.com/

85

NUMERICAL ABILITY

Practice Questions 1. The degree of polynomial 336x2 + 210x + 42 is (a) 3 (c) 42

11. If ax + by = 3, bx − ay = 4 and x2 + y2 = 1, then the

value of a2 + b2 is

(b) 4 (d) 2

(a) 25 (c) 27

2. If 2x2 + ax + b, when divided by x − 3, leaves a

remainder of 31 and x2 + bx + a, when divided by x − 3, leaves a remainder of 24, then a + b equals (a) −23 (c) 7

12. If 3 x − 2 = 2 3 + 4, then the value of x is (a) 2 (1 − 3 ) (c) 1 + 3

(b) −7 (d) 23

3 x + 6 11x − 8 x 3x x + 7 , then the value = − − + 4 24 8 24 3 of x is

[( s − a)2 + ( s − b)2 + ( s − c)2 + s2 ] = ? (a) (a 2 + b2 + c2 ) (c) (s2 − a 2 − b2 − c2 )

(b) (4s2 − a 2 − b2 − c2 ) (d) (s2 + a 2 + b2 + c2 )

14. The value of y in the solution of the equation

2x +

(c)

5. The sum of a number and its reciprocal is −12.

20, then the numbers are (a) 5, 15 (c) 3, 9 (a) x = 2 , y = 5 (b) x = 5, y = 5 (c) x = 5, y = 2 (d) x = 0, y = 3

(b) 1 (d) 9

9. If y = − 1, then the value of 1 + (1 / y) + (1 / y2 ) + (1 / y 3)

+ (1 / y4 ) + (1 / y5 ) is

19. The solution of the system of linear equations

0 .4 x + 0 .3 y = 1 . 7 and 0 . 7 x − 0 . 2 y = 0 . 8 is

(b) 0 (d) 2

(a) x = 3, y = 2 (c) x = 2 , y = 3

10. If x and y are positive with x − y = 2 and xy = 24,



1 6

(d)

(b) x = 2 , y = − 3 (d) None of these

1 1 20. If  x +  :  x −  = 5 : 4, then the value of x is

1 1 + is equal to x y

(c)

(b) 4, 12 (d) None of these

18. If x + y = 7 and 3 x − 2 y = 11, then

exactly divisible by x + 2 but not divisible by x + 3.

1 12

(b) 9, 2 (d) 6, 5

17. If one number is thrice the other and their sum is

8. Find the values of k for which x2 + 5kx + k2 + 5 is

(b)

(b) 5 (d) None of these

(a) 8, 3 (c) 7, 4

(b) −10 (d) 2

5 12

1 2

product is 30, then the numbers are

7. Solve for x;x ∈N: ( x − 4)2 − 36 = 0 .

(a)

1 4 3 (d) 7

(b)

16. The sum of the two numbers is 11 and their

(b) 7 (d) 6

(a) −1 (c) 1

= 8 is

(a) 0.5 (c) 0.25

3 −1 = a + b 3 ; then a2 + b2 = ? 3 +1

(a) Both 1 and 9 (c) Neither 1 nor 9

y

then the number is

(b) −1728 (d) −1692

(a) −2 (c) 10

= 2x −

15. If 5 is added to twice of a number it becomes 6,

What would be the sum of cubes of the two (the number and its reciprocal)?

(a) 8 (c) 5

y

(a) 0

(b) 12 (d) −12

(a) −1764 (c) −1681

(b)

(c) 3

answer and ( −2) marks are given for every incorrect answer, Rakesh answered all the questions and scored 30 marks though he got 10 correct answers. How many incorrect answers had he attempted? (a) 10 (c) −10

3 2 1 (d) 3

(a) − 3

4. In a test, ( +5) marks are given for every correct

then

(b) 2 (1 + 3 ) (d) 1 − 3

13. If

3. If a + b + c = 2s, then

6. If

(b) 26 (d) 28

x 

x

(b) ± 1 (d) ± 3

(a) 0 (c) ± 2

25 6

ANSWERS 1. 11.

(d) (a)

2. 12.

(c) (b)

3. 13.

(a) (c)

4. 14.

(a) (a)

5. 15.

(d) (a)

6. 16.

(c) (d)

https://sscstudy.com/

7. 17.

(c) (a)

8. 18.

(d) (c)

9. 19.

(b) (c)

10. 20.

(a) (d)

https://sscstudy.com/

86

CUET (UG) Section III : General Test

Hints & Solutions 1. We know, degree of ax2 + bx + c is 2. So, degree of 336x2 + 210x + 42 is 2. 2. If 2x2 + ax + b in divided by (x − 3) then remainder is 31 and 2x2 + xa + b − 31 in divisible by (x − 3) thus x = 3 is a solution of 2x2 + ax + b − 31 2(3)2 + 3a + b − 3 = 0 18 + 3a + b − 31 = 0 …(i) 3a + b = 13 Now if x2 + bx + a in divided by (x − 3) and remainder is 24 then x2 + bx + a − 24 = 0 (3)2 + 3x + a − 24 = 0 [Q x = 3] 9 + 3x + a − 24 = 0 …(ii) 3x + a = 15 On Solving Eqs. (i) and (ii) b = 4 on putting Eq. (i) a = 3 thus a + b =4+3= 7 3. [(s − a )2 + (s − b)2 + (s − c)2 + s2] ⇒ s2 + a 2 − 2sa + s2 + b2 − 2sb + s2 + c2 − 2sc + s2 [Q (a − b)2 = a 2 + b2 − 2b] 2 ⇒ 4s − 2(sa + sb + sc) + a 2 + b2 + c2 ⇒ 4s2 − 2s × 2s + a 2 + b2 + c2 [Q a + b + c = 2s Given ] 2 2 ⇒ 4s − 4s + a 2 + b2 + c2 ⇒ a 2 + b2 + c2 4. Let he attempted ‘x’ correct answer and y incorrect answer, then 5x − 2 y = 30 5(10) − 2 y = 30 [x = 10 [given ]] 50 − 2 y = 30 50 − 30 y= ⇒ y = 10 2 5. Let the number is x then 1 x + = −12 x On taking cube both side 1 (x + )3 = (−12)3 x 1 1 3 x + 3 + 3(x + ) = −1728 x x [Q (a + b)3 = a3 + b3 + 3ab (a + b)]

x3 +

6.

1 + 3(−12) = −1728 x3 1 x3 + 3 − 36 = −1728 x 1 x3 + 3 = −1692 x

3 −1 =a+b 3 3 −1 By using componendo and dividendo ( 3 − 1) ( 3 − 1) =a+b 3 ( 3 + 1 ) ( 3 − 1) ( 3 − 1 )2 =a+b 3 3 −1 [Q a 2 − b2 = (a + b)(a + b)] 3 + 1 −2 3 =a+b 3 2 4 −2 3 =a+b 3 2 2− 3 = a + b 3 a = 2 and b = −1 Thus, a 2 + b2 = 22 + (−1)2 a 2 + b2 = 5

7. (x − 4)2 − 36 = 0 ⇒ (x − 4)2 = 36 (x − 4)2 = (6)2 On taking square root both side x − 4 = 6 ⇒ x = 10 8. If x2 + 5kx + k2 + 5 is divisible by (x + 2) then x = −2 is a solution then (−2)2 + 5(−2)k + k2 + 5 = 0 k2 − 10k + 9 = 0 (k − 1)(k − 9) = 0 k = 1 and 9 also given that (x + 3) in not a solution then (−3)2 + 5(−3)(k) + k2 + 5 ≠ 0 9 + k2 − 15k + 5 ≠ 0 k2 − 14k − k + 14 ≠ 0 (k − 1)(k − 14) ≠ 0 k ≠ 1 and 14; thus only k = 9 9. Now,  1  1   1   1   1  1 +   +  2 +  3  +  4  +  5   y  y   y   y   y   1   1   1  =1+  +  +  (−1)  (−1)2  (−1)3   1   1  + +   (−1)4   (−1)5 

[Q put y = − 1]

= 1 −1 + 1 −1 + 1 −1 =0

https://sscstudy.com/

10. We have, …(i) x − y =2 …(ii) xy = 24 ⇒ y ( y + 2) = 24 [Q from Eq. (i). x = y + 2] ⇒ y2 + 2 y − 24 = 0 ⇒ y = 4, y = − 6 but x and y are positive, so y = 4 and x = y + 2 =4 + 2 ⇒ x =6 1 1 1 1 5 ∴ + = + = x y 4 6 12 11. Given equations are …(i) ax + by = 3 …(ii) bx − ay = 4 and …(iii) x 2 + y2 = 1 On squaring Eqs. (i) and (ii) and then adding, we get a 2x2 + b2y2 + 2ax by + a 2y2 + b2x2 − 2ax by = 9 + 16 ⇒ a 2 (x2 + y2) + b2 (x2 + y2) + 2ax by −2axby = 25 ⇒ a 2 × 1 + b2 × 1 = 25 [put x2 + y2 = 1] 2 2 ∴ a + b = 25 12. Given that, 3 x − 2 = 2 3 + 4 ⇒ ⇒ ⇒ ⇒

3 x=2 3 + 6 2 3+6 x= 3 2 3+6 3 x= × 3 3 x = 2 (1 + 3 )

13. Given that, 3x + 6 11x − 8 x 3x x + 7 − + = − 8 24 3 4 24 9x + 18 − 11x + 8 + 8x ∴ 24 18x − x − 7 = 24 ⇒ 6x + 26 = 17x − 7 ⇒ 11x = 33 ⇒ x = 3 14. Since, 2x + y = 8 and 2x − y = 8 3 ⇒ x+ y= 2 3 and x− y= 2 ⇒ 2 x + 2y = 3

https://sscstudy.com/

NUMERICAL ABILITY

and

2 x − 2y = 3

3 On solving, we get x = , y = 0 2 15. Let the number be x. ∴

2x+ 5 =6 ⇒ x=

1 = 0.5 2

16. Let the two numbers be x and y. …(i) ∴ x + y = 11 and xy = 30 2 2 Now, (x − y) = (x + y) − 4xy = (11)2 − 4 × 30 = 121 − 120 = 1 …(ii) ⇒ x− y=1 On solving Eqs. (i) and (ii), we get x = 6, y = 5 17. Let the two numbers be x and y. ∴ x = 3 y and x + y = 20

⇒ 3 y + y = 20 ⇒ 4 y = 20 ⇒ y = 5 and x = 15 Hence, two numbers are 5 and 15. 18. Given equations are x + y = 7 and 3x − 2 y = 11 On multiplying Eq. (i) by 2 and then adding Eq. (ii), we get 5x = 25 ⇒ x=5 ∴ 5+ y=7 ⇒ y=2 19. Given system of linear equations are 4x 3 y 17 7x 2 y 8 and + = − = 10 10 10 10 10 10

https://sscstudy.com/

87

…(i) ∴ 4x + 3 y = 17 and …(ii) 7x − 2 y = 8 On solving Eqs. (i) and (ii), we get x = 2 and y = 3 1 x+ x =5 20. Given, 1 4 x− x 1 1   4 × x +  =5 x −  ⇒   x x ⇒ ⇒ ⇒ ∴

4 5 = 5x − x x 4 5 5x − 4x = + x x 9 x = ⇒ x2 = 9 x x= ± 9 =±3 4x +

https://sscstudy.com/

88

CUET (UG) Section III : General Test

CHAPTER 17

Geometry Plane Geometry

Acute Angled Triangle A triangle each of whose angle is less than 90° is called an acute angled triangle.

Plane geometry is about flat shapes like line, circle and triangle etc. These types of shapes can be easily drawn on a piece of paper.

3. Circle

1. Lines and Angles Two lines in the same plane are said to be parallel, if they never meet. A line which cuts a pair of parallel line is called a transversal.

A circle is a set of points which are equidistant from a given point. The given point is known as the centre of that circle. 1 Side , Inradius (OD) = × Height = 3 2 3

E 1 2 B 4 3 5

C 8

O

D

6

D

Circumradius (OA) =

7

2 Side × Height = 3 3

F

A

Here, two parallel lines AB and CD are cut by a transversal i. e. , EF. Then, The corresponding angles are ∠1 = ∠ 5, ∠2 = ∠6, ∠4 = ∠8 and ∠3 = ∠7. The alternate angles are ∠1 = ∠7, ∠2 = ∠8, ∠3 = ∠5 and ∠4 = ∠6.

2. Triangles A figure bounded by three straight lines is called a triangle. The small of all interior angles of a triangle is 180°.

Types of Triangle Equilateral Triangle A triangle having all sides equal

is called an equilateral triangle and each angle equal to 60°. Scalene Triangle A triangle having all sides of different length is called a scalene triangle. Isosceles Triangle A triangle having two sides equal is called an isosceles triangle. Right Angled Triangle A triangle one of whose angles measures 90° is called a right angled triangle. Obtuse Angled Triangle A triangle one of whose angles lies between 90° and 180° is called an obtuse angled triangle.

O

4. Quadrilateral It is a plane figure bounded by four straight lines. The sum of the internal angles of a quadrilateral is equal to 360°.

Parallelogram A quadrilateral in which the opposite sides are equal and parallel, is called a parallelogram. D

C

Breadth (B)

A

O

A

Length (L)

B

The opposite angles are equal in magnitudes. The digonals of a parallelogram are not equal in magnitudes, but they bisect each other. AC ≠ BD but AO = OC and OB = OD.

https://sscstudy.com/

https://sscstudy.com/

NUMERICAL ABILITY

89

The coordinates of any point on X-axis are of the form ( x , 0).

Rectangle A parallelogram in which the adjacent sides are perpendicular to each other, is called a rectangle.

The coordinates of any point on Y -axis are of the form ( 0, y ).

The diagonals of a rectangle are of equal magnitudes and bisect each other i. e. , AC = BD and OA = OB = OC = OD.

Distance Formula Distance between two points If A ( x1 , y1 ) and B( x2 , y2 ) are two points, then

Square D C A parallelogram in which all the sides are equal and perpendicular to each other, is called a square. O The diagonals bisect each other at right angles and form four isosceles right B A angled triangles. The diagonals of a square are of equal mangnitudes i. e. , AC = BD.

Polygon

B (x2, y2)

Y

C

A (x1, y1)

O

X

E

D

AB = ( x2 − x1 )2 + ( y2 − y1 )2 = ( x1 − x2 )2 + ( y1 − y2 )2 For example The distance between A( 1, 2) and B( 5, 6) is

Regular polygon A polygon in which all the sides are

equal and also all the interior angles are equal, is called a regular polygon.

AB = ( 5 − 1)2 + ( 6 − 2)2 = 42 + 42 = 32 = 4 2 units

Sum of all interior angles = ( n − 2) × 180° = ( 2n − 4) × 90°

Area of a Triangle

Each interior angle = 180° − Exterior angle

If A( x1 , y1 ), B( x2 , y2 ) and C( x3 , y3 ) are three vertices of a ∆ABC, then its area is given by 1 Area of ∆ ABC = [x1 ( y2 − y3 ) + x2( y3 − y1 ) + x3 ( y1 − y2 )] 2

360°   Each exterior angle =   (in degrees)  Number of sides Sum of all exterior angle = 360° (always constant). n( n − 3) Number of diagonals of polygon of n sides = 2

Y 4 B (0, 4) 3

Coordinate Geometry

2

It is a system of geometry, where the position of points on the plane is described by using an ordered pair of numbers.

Quadrants The X and Y-axes divide the cartesian plane into four regions referred as quadrants. The table of sign conventions of coordinates in various quadrants is given below

1 O (0, 0)

1st 2nd quadrant quadrant 3rd quadrant

2

3

4

A (5, 0) X 5

For example If we have to find the area of a triangle having the vertices (0, 0), (5, 0) and (0, 4), then

Y

X′

1

O

∴ Area of triangle = X

4th quadrant

Y′

Quadrant

Region

Sign of (x, y)

Example

I

XOY

(+ , + )

(2, 3)

II

YOX ′

(− , + )

(−2, 4)

III

X ′ OY ′

(− , − )

(−1, − 2)

IV

Y ′ OX

(+ , − )

(1, − 3)

=

1 { 0 ( 0 − 4) + 5 ( 4 − 0) + 0 ( 0 − 0)} 2 1 × 20 = 10 sq units 2

Q ( x1 , y1 ) = ( 0, 0), ( x2 , y2 ) = ( 5, 0) and ( x3 , y3 ) = ( 0, 4)

Collinearity of Three Points Three points A ( x 1 , y1 ), B( x2 , y2 ) and C( x3 , y3 ) are collinear, if (i) Area of ∆ABC is 0, i.e. x1( y2 − y3 ) + x2( y3 − y1 ) + x3 ( y1 − y2 ) = 0. (ii) Slope of AB = Slope of BC = Slope of AC

The coordinates of point O (origin) are taken as ( 0, 0).

https://sscstudy.com/

https://sscstudy.com/

90

CUET (UG) Section III : General Test

(iii) Distance between A and B + Distance between B and C = Distance between A and C

Centroid of a Triangle A (x1, y1)

Centroid is the point of intersection of all the three medians of a triangle.

G

If A( x1 , y1 ), B( x2 , y2 ) and C( x3 , y3 ) are the vertices of B C (x2, y2) (x3, y3) ∆ABC, then the coordinates of its centroid are 1  1  3 ( x1 + x2 + x3 ), 3 ( y1 + y2 + y3 ).

Section formulae Let A( x1 , y1 ) and B( x2 , y2 ) be two points on the cartesian plane. Let point P ( x , y ) divides the line AB in the ratio of m : n internally. mx2 + nx1 my2 + ny1 Then, x= ,y= m+n m+n If P divides AB externally, then mx2 − nx1 my2 − ny1 x= ,y= m−n m−n If P is the mid-point of AB, then x1 + x2 y + y2 ,y= 1 x= 2 2

Solved Examples Ex. 1 A chord AB is drawn in a circle with centre O and radius 5 cm. If the shortest distance between centre and chord is 4 cm, find the length of chord AB ?

Ex. 3 In the given figure, AB and CD are parallel lines. If ∠EGB = 50° , find ∠CHG. (1) 120°

(2) 130°

(3) 125°

(1) 6 cm (2) 5 cm (3) 4 cm (4) 3 cm Sol. (1) In the adjoining figure AO = 5 cm (radius)

E 50° G

A

cm 5 x

D

C

4 cm C

x

F

B

Sol. (2) ∠AGH = ∠EGB

OC = 4 cm (shortest distance between centre and chord) Let length of chord AB be 2x, then AC = x. In ∆AOB, AO 2 = AC 2 + OC 2 ⇒ (5)2 = x2 + (4)2 ⇒ 25 = x2 + 16 x = 25 − 16 = 9 = 3 cm ∴ ∴ Length of chord AB = 2x = 2 × 3 = 6 cm

Ex. 2 In the figure given, ∠ BAC : ∠ ABC = 2 : 3. Find the

[Vertically opposite angles]

∠AGH = 50° Now, ∠ AGH + ∠ CHG = 180° [Interior angles on the same side of the transversal are supplementary.] ∴ 50° + ∠ CHG = 180° ⇒ ∠ CHG = 180° − 50° = 130° .

Ex 4. An angle θ° is one-fourth of its supplementary angle. What is the measure of the angle θ°? (1) 36° (2) 34° (3) 32° (4) 31° Sol. (1) If the sum of two angles is 180°, the angles are said to be supplementary. ∴ The supplementary angle of θ° is (180° − θ° ). 1 Given that, θ° = (180° − θ° ) 4 ⇒ 4 θ° = 180° − θ° ⇒ 5 θ° = 180° 180° θ° = = 36° ⇒ 5

measure of ∠ ABC. A

120° B

B

H

O

A

(4) 140°

C

(1) Obtuse angle (2) Acute angle (3) Right angle (4) None of these Sol. (2) Let ∠ A = 2x and ∠ B = 3x

Ex. 5 The number of diagonals in a 27-sided polygon is

Then, 2x + 3x = 120° [Exterior angle is equal to the sum of the interior opposite angles] ⇒ 5x = 120° ⇒ x = 24° ∴ ∠ ABC = 3x = 3 × 24° = 72°

(1) 324 (2) 325 (3) 322 (4) 320 Sol. (1) Number of diagonals of polygon of n sides (n )(n − 3) = 2 Number of diagonals of polygon of 27 sides 27 × 24 = = 324 2

https://sscstudy.com/

https://sscstudy.com/

NUMERICAL ABILITY

Ex. 6 Find the distance between the points A ( − 6, 8) and B ( 4, − 8). (1) 18.86 units (2) 16.76 units (3) 11.77 units (4) 16.76 units Solution (1) Here, A (− 6, 8) = A (x1 , y1 ) and B (4, − 8) = B (x2 , y2 ) So, x1 = − 6, y1 = 8, ∴ Required distance, AB =

x2 = 4 and

2

(x2 − x1 ) + ( y2 − y1 )

91

Ex. 7 Find the area of ∆ABC, whose vertices are A (8, − 4), B (3, 6) and C ( − 2, 4). (1) 20 sq units (2) 30 sq units (3) 35 sq units (4) 29 sq units Solution (2) Here, A (8, − 4), then x1 = 8 , y1 = − 4 B (3, 6), then x2 = 3, y2 = 6 C (− 2, 4), then x3 = − 2, y3 = 4

y2 = − 8

∴ Area of ∆ABC 1 {x1 ( y2 − y3 ) + x2 ( y3 − y1 ) + x3 ( y1 − y2 )} 2 1 = { 8 (6 − 4) + 3 (4 − (− 4)) + (−2)(− 4 − 6)} 2 1 1 = {16 + 24 + 20} = × 60 = 30 sq units 2 2

2

=

=

{4 − (− 6)}2 + (− 8 − 8)2

=

(4 + 6)2 + (− 16)2 =

=

~ 18.86 units 356 −

100 + 256

Practice Questions 1. In the given figure, if l ||m, then find the value of x

6. In the given figure, ABCD is a parallelogram in

which ∠BAD = 75° and ∠CBD = 60°. Then, ∠BDC is equal to

(in degrees). A

l

D

l

C

100° O

(x + 5)° 30°

m

m

B

(1) 105° (3) 110°

A

(1) 60°

2. In a ∆ABC, ∠A = 90° , ∠C = 55° and AD ⊥ BC. What

is the value of ∠BAD? (1) 60°

(3) 55°

B

(2) 75°

(3) 45°

(4) 50°

A

7.

(2) 45°

60°

75°

(2) 100° (4) 115°

D y

x

(4) 35°

3. In the figure given below, ∠PQR = 90° and QL is a

120º

median, PQ = 5 cm and QR = 12 cm. Then, QL is equal to

E

F

z

P

90º

50º

5 cm

B L

In the figure given above, ABCD is a trapezium. EF is parallel to AD and BC. Then, ∠y is equal to (1) 30°

Q

(1) 5 cm (3) 6 cm

C

12 cm

R

(2) 45°

(3) 60°

8. In the given figure, AB||CD. If ∠CAB = 80° and

(2) 5.5 cm (4) 6.5 cm

∠EFC = 25°, then ∠CEF is equal to

4. An angle which is less than 360° and more than

180°, is called (1) a reflex angle (3) an acute angle

(2) a straight angle (4) an obtuse angle

80°

C

25°

A

and the longest side of the triangle are respectively (2) AC and BC (4) AB and AC

F

D

B

5. In a ∆ABC, ∠A : ∠B : ∠C = 2 : 4 : 3. The shortest side (1) AC and AB (3) BC and AC

(4) 65°

E

(1) 65°

https://sscstudy.com/

(2) 55°

(3) 45°

(4) 75°

https://sscstudy.com/

92

CUET (UG) Section III : General Test

9. AB is the diameter of a circle with centre O and P is

a point on it. If ∠POA = 120°, then the value of ∠PBO is (1) 30°

(2) 50°

(3) 60°

(4) 40°

10. Three angles of a quadrilateral are 80°, 95° and

112°. Its fourth angle is (1) 78°

(2) 73°

(3) 85°

(4) 100°

11. What is the value of θ ?

Information (I)

(4) 15

AB = 10 cm, then length of AG is

(1)

5 3 cm 3

(2)

10 3 cm 3

(3) 5 3 cm

(4) 10 3 cm

(2) 60°

(3) 90°

(4) 120°

complement. Find the greater angle. (1) 30° B

(2) 60°

(3) 45°

(4) 75°

22. If the distance between the points ( x, 0) and ( −7, 0)

is 10 units, then the possible values of x are

23. The distance between the points ( 4, − 8) and ( k, 0) is

10. Find k.

12. Find the area of the triangle formed with the three

straight lines represented by (i) x + y = 0; (ii) 3 x = 5 y; and (iii) y = 3 x − 12 (2) 20 units

(3) 12 units

(4) 16 units

13. Find the ratio in which line 3 x + 2 y = 17 divides the

line segment joined by points (2,5) and (5,2). (2) 1 : 2

(2) − 3 and 17 (4) − 3 and −17

(1) 3 and 17 (3) 3 and −17

Neither I nor II is sufficient Either I or II is sufficient Only I is sufficient Only II is sufficient

T

(3) 16

21. An angle is 10° more than one-third of its

(II) 0 < θ < 90°

14.

(2) 12

19. G is the centroid of the equilateral ∆ABC. If

(1) 30°

A

(1) 1 : 3

(1) 9

3, then find the value of the largest angle.

θ

(1) 15 units

∆XYZ and perimeter of ∆PQR is 4 : 9 and if PQ = 27 cm, then what is the length of XY (in cm)?

20. If the angles of a triangle are in the ratio of 1 : 2 :

C 30° O

(1) (2) (3) (4)

18. ∆XYZ is similar to ∆PQR. If ratio of perimeter of

(3) 2 : 5

(4) 3 : 4

(1) k = 6 or −2 (3) k = 10 or −4

(2) k = 10 or −2 (4) k = 6 or −4

24. Coordinates of a point is (0, 1) and ordinate of an

another point is − 3. If distance between both the points is 5, then abscissa of second point is (1) 3

(2) − 3

(3) ± 3

(4) 1

25. What is the reflection of the point (6, − 3) in the

line y = 2 ?

U

(1) (−2, − 3) (3) (−6, 7)

(2) (6, 7) (4) (− 2, 3)

26. If the point ( x, y) is equidistant from points (7, 1) P Q

and (3, 5), then find (x − y).

R S

In the given diagram , TU || PS and points Q and R lie on PS. Also, ∠PQT = x ° , ∠RQT = ( x − 50) º and ∠TUR = ( x + 25) ° What is the measure of ∠URS? (1) 130°

(2) 140°

(3) 135°

(4) 115°

15. The ratio of the measures of the interior angles of

a regular octagon to that of a regular dodecagon is (1) 8 : 12

(2) 12 : 8 (3) 9 : 10

(4) 4 : 5

16. If one of the acute angles of a right-angled triangle

is 55°, what is the measure of the other acute angle? (1) 35°

(2) 40°

17.

(3) 30°

(4) 25°

A

B C

(1) 2

(4) 8

27. The vertices of a triangle are A( 4, 4), B (3, − 2) and

C ( − 3, 16). The area of the triangle is (1) 30 sq units (3) 27 sq units

(2) 36 sq units (4) 40 sq units

28. Two vertices of an equilateral triangle are origin

and (4, 0) What is the area of the triangle? 3 sq units

(1) 4 sq units

(2)

(3) 4 3 sq units

(4) 2 3 sq units

29. If the graph of the equation 2x + 3 y = 6 form a

triangle with coordinates axes, then the area of triangle will be (1) 2 sq units (3) 6 sq units

(2) 3 sq units (4) 1 sq unit

collinear, then k equals (1) 2

(2) 4

(3) 6

(4) 9

31. If two vertices of a triangle are (5, 4) and (–2, 4)

D

In the figure given above, ∠BAE = 30° , ∠ABE = 80° and ∠DBE = 50°. What is measure of ∠BCE? (2) 10°

(3) 6

30. If the points A (1, − 1), B ( 5, 2 ) and C( k, 5) are

E

(1) 20°

(2) 4

(3) 25°

(4) 5°

and centroid is (5, 6), then third vertex is (1) (12, 10) (3) (− 10, 12)

https://sscstudy.com/

(2) (10, 12) (4) (12, − 10)

https://sscstudy.com/

93

NUMERICAL ABILITY

32. A point C divides the line AB, where A(1, 3) and

35. The area of the triangle with vertices at

( a, b + c), ( b, c + a) and ( c, a + b) is

B (2, 7), in the ratio of 3 : 4. The coordinates of C are 5 (1)  , 5 3 

3 (2) (− 2, − 9) (3)  , 5 5 

(1) a − b − c (3) 0

10 33  (4)  ,   7 7

(2) ab + bc + ca (4) a + b + c

ANSWERS

33. Point A( 4, 2) divides segment BC in the ratio 2 : 5.

Coordinates of B are (2, 6) and C are (9, y). What is the value of y ? (1) 8

(2) – 8

(3) 6

1 6 11 16 21 26 31

(4) – 6

34. In what ratio, the line made by joining the points

A ( − 4, − 3) and B (5, 2) intersects X-axis? (1) 3 : 2 (3) − 3 : 2

(2) 2 : 3 (4) − 2 : 3

(1) (3) (3) (1) (2) (1) (1)

2 7 12 17 22 27 32

(3) (3) (3) (1) (3) (3) (4)

3 8 13 18 23 28 33

4 9 14 19 24 29 34

(4) (2) (2) (2) (2) (3) (2)

5 10 15 20 25 30 35

(1) (3) (2) (2) (3) (2) (1)

(3) (2) (3) (3) (2) (4) (3)

Hints & Solutions

L

5 90º Q

12

R

PR ...(i) PL = LR = ∴ 2 In ∆PQR, (PR )2 = (PQ )2 + (QR )2 [by Pythagoras theorem] = (5)2 + (12)2 = 25 + 144 = 169 = (13)2 2 ⇒ PR = (13)2 ⇒ PR = 13 Now, by theorem, if L is the mid-point of the hypotenuse PR of a right angled ∆PQR, then 1 1 QL = PR = (13) = 6.5 cm 2 2

4. An angle which is less than 360° and more than 180°, is called a reflex angle.

5. Let

∠A = 2x

∠B = 4x and ∠C = 3x We know, ∠A + ∠B + ∠C = 180° ∴ 2x + 4x + 3x = 180° ⇒ 9x = 180° ⇒ x = 20° Now, ∠A = 40°, ∠B = 80° and ∠C = 60° Hence, the shortest side of triangle = side opposite to the smallest angle = BC and the longest side of

6. ∠C = A = 75° [opposite angles of parallelogram] In ∆BCD, ∠CBD + ∠BCD + ∠BDC = 180° ⇒ 60° + 75° + ∠BDC = 180° ⇒ 135°+∠BDC = 180° ∠BDC = 45°

8. Let ∠CEF = x° Now, AB ||CD and AF is a transversal. ∴ ∠DCF = ∠CAB = 80° [corresponding angles] In ∆CEF, side EC has been produced to D. ⇒ x + 25 = 80° ⇒ x = 55°

10. Let the fourth angle be x°. 80 + 95 + 112 + x = 360 287 + x = 360 x = (360 − 287) = 73°

Then, ⇒ ⇒

11. From Statement (1) ∠C 30 = 90 + = 105° 2 2 Hence, only statement (1) is sufficient.

θ = 90 +

12. Given equation x + y = 0; 3x = 5 y, and y = 3x − 12 C 2

P

3x –1

QR = 12 cm and QL is a median.

triangle = side opposite to the longest angle = AC.

3x=5y

y=

3. Given that, PQ = 5 cm,

A

x+y=0

B

https://sscstudy.com/

Coordinate of Vertex A (3,− 3) = (x1 y1 ) Coordinate of vertex B (0, 0) = (x2 , y2 ) Coordinate of vertex C (5, 3) = (x3 , y3 ) Area of ∆ABC 1 = {x1 ( y2 − y3 ) + x2 ( y3 − y1 ) 2 + x3 ( y1 − y2 )} 1 = {3(0 − 3) + 0(3 + 3) + 5(−3 − 0)} 2 1 1 = {−9 − 15} = × (−24) = 12 units 2 2

13. Let the given lives divides m1 : m2 at point C ∴Coordinate of point C 3x+2y=17

A

m2

m1 C

B (5, 2)

m1 × 5 + m2 × 2 , m1 + m2 m × 2 + m2 × 5 y= 1 m1 + m2

x=

This point satisfying the given equation  5m + 2m2   2m + 5m2  3 1  + 2 1  = 17  m1 + m2   m1 + m2  ⇒15m1 + 6m2 + 4m1 + 10m2 = 17m1 + 17m2 ⇒ 19m1 − 17m1 = 17m1 − 16m2

https://sscstudy.com/

94

CUET (UG) Section III : General Test



K (PQ + QR + PR ) 4 4 = ⇒k = PQ + QR + PR 9 9 4 XY 4 or = ⇒ XY = × 27 9 PQ 9

2m1 = m2 m1 1 = m2 2

⇒ ⇒



m1 : m2 ⇒1 : 2



14. Given ∠PQT = x° , ∠RQT = (x − 50)° , ∠TUR = (x + 25)° ∠PQT + ∠RQT = 180° x + x − 50 = 180



2x = 230



x = 115°

∠URQ = 180°−140 = 40°



∠URS = 180°−∠URQ = 180° − 40° = 140° (8 − 2) × 180° 8 15. ∴Required ratio = (12 − 2) × 180° 12 (n − 2) × 180° [Qinterior angle = ] n 180 × 6 × 12 9 = = 10 × 8 × 180 10

16. ∴Other acute angle = 180°−90° − 55° = 180°−145° = 35°

17. Given, A 30°

80° 50°

C

A

10

⇒ (115 + 25) + ∠URQ = 180°

B

Altitude = Median

B

Q ∠TUR + ∠URQ = 180° ⇒

19. In equilateral triangle,

E

D

∠AEB = 180° − 80° − 30° = 70° ⇒ ∠EBC = 180° − 80° − 50 = 50° = ∠CED ∠AEB = 180°−80°−30° = 70° ∴ ∠CEB = 180°−70°−50° = 60° ∴ ∠BCE = 180°−100°−60° = 20°

18. Given, ∆XYZ − ∆PQR

G

P



Z

away from line y = 2.

X′

C

So, length of altitude, AD 3 3 a= = × 10 = 5 3 2 2 Now, 2 2 × 5 3 10 3 cm AG = × AD = = 3 3 3

20. According to the questions, largest angle =

180 × 3 180 = × 3 = 90° (1 + 2 + 3) 6

21. Let the other angle be x°. According to question, 1 x = 10°+ (90°− x) 3 1 x = 10° + 30°− x ⇒ 3 1 x + × = 40° ⇒ 3 4x = 40° ⇒ x = 30° ⇒ 3 ∴The greater angle is 90° − 30° = 60°. (x, 0) and (− 7, 0) = 10 units Here, x1 = x, y1 = 0, x2 = − 7 and y2 = 0 ∴Required distance = ⇒

(x2 − x1 )2 + ( y2 − y1 )2 (− 7 − x)2 + (0 − 0)2 = 10

⇒ ± (x + 7) = 10 If x + 7 = 10, then x = 3 If − (x + 7 ) = 10, then x = −17

23. Here, (k − 4)2 + (0 + 8)2 = (10)2 [Q distance =

Y

25. Here, the point (6, − 3) is five units 4 3 2 1

22. Given, distance between the points

X

Then, (x − 0)2 + (−3 − 1)2 = 52 ⇒ x2 + 16 = 25 ⇒ x2 = 9 ⇒ x=±3

cm



XY = 12 cm

24. Let the abscissa be x.

Q

R

XY YZ XZ = = = K (say) PQ QR PR

According to the question, XY + YZ + XZ 4 = PQ + QR + PR 9

(x2 − x1 )2 + ( y2 − y1 )2 ]

⇒ k 2 + 16 − 8k + 64 = 100 ⇒ k 2 − 8k − 20 = 0 2 ⇒ k − 10k + 2k − 20 = 0 ⇒ k (k − 10) + 2(k − 10) = 0 ⇒ (k + 2)(k − 10) = 0 ⇒ k = −2, k = 10 Hence, the value of k is 10 or – 2.

https://sscstudy.com/

Y y=2

X –3 –2 –1 –1 1 2 3 4 5 6 –2 –3 (6, –3) –4 Y′

So, its reflection point will also be 5 units away from the line y = 2. ∴ Required point = (6, 7)

26. Let P (x, y) be equidistant from points A (7, 1) and B (3, 5). Then, AP = BP , so AP 2 = BP 2 ∴ (x − 7)2 + ( y − 1)2 = (x − 3)2 + ( y − 5)2 2 2 ⇒ x − 14x + 49 + y − 2 y + 1 = x2 − 6x + 9 + y2 − 10 y + 25 ⇒ 8x − 8 y = 16 ∴ After solving, x − y = 2

27. Let x1 = 4, x2 = 3, x3 = − 3, y1 = 4, y2 = − 2 and y3 = 16 ∴ Area of triangle 1 = {x1 ( y2 − y3 ) + x2 ( y3 − y1 ) 2 + x3 ( y1 − y2 )} 1 = [4 (− 2 − 16) + 3 (16 − 4) 2 + (− 3 ) {4 − (− 2)}] 1 = [4 × (− 18) + 3 × 12 + (− 3 ) (6)] 2 1 = (− 72 + 36 − 18) 2 1 1 = × (− 54) = × 54 = 27 sq units 2 2 [neglecting negative sign] 28. Since, triangle is equilateral. ∴ AB = BC = CA = 4 4 ⇒ BD = = 2 2 A 4

(0, 0) O B

2

D 4

C(4, 0)

https://sscstudy.com/

Numerical Ability

In ∆ ADC, AD 2 = 42 − 22 = 16 − 4 = 12 ⇒ AD = 2 3 1 ∴ Area of ∆ABC = × BC × AD 2 1 = × 4× 2 3 2 = 4 3 sq units

29. Q2x + 3 y = 6 2x 3 y x y + =1 ⇒ + =1 6 6 3 2 Comparing with equation of line x y + = 1, we get a b Intercept at X-axis = 3 and intercept at Y -axis = 2 ⇒

Y B (0, 2) A (3, 0) (0, 0) O

∴ Area of ∆OAB =

X

1 × 3 × 2 = 3 sq 2

units

30. Given, x1 = 1, x2 = 5, x3 = k, y1 = − 1, y2 = 2 and y3 = 5. Since, A, B and C are collinear. ∴ Area of triangle = 0 ⇒ {x1 ( y2 − y3 ) + x2 ( y3 − y1 )

⇒ ⇒ ⇒ ∴

+ x3 ( y1 − y2 )} = 0 {1 (2 − 5 ) + 5 (5 − (− 1) + k (− 1 − 2)} = 0 {− 3 + 30 − 3k } = 0 3k = 27 k=9

31. Let the third vertex be (x, y). ∴Coordinates of centroid x + x2 + x3 y1 + y2 + y3  =  1 ,    3 3 Given, x1 = x, x2 = 5, x3 = − 2, y1 = y, y2 = 4, y3 = 4 and centroid = (5, 6) x+ 5− 2 5= ∴ 3 y + 4+ 4 and 6= 3 ⇒ x = 12 and y = 10 32. Given, m = 3, n = 4, x1 = 1, x2 = 2, y1 = 3 and y2 = 7 ∴ Coordinates of C mx2 + nx1 my2 + ny1  , =    m+ n m+ n  3 × 2 + 4 × 1 3 × 7 + 4 × 3 , =     3+ 4 3+ 4 10 33 =  ,   7 7

33. Using section formula, i.e. if a line is divided by a point in certain ratio (m : n ), then coordinates of point (x, y)

https://sscstudy.com/

95

mx2 + nx1 my2 + ny1 , y= m+ n m+ n 2 y + 5(6) 2= 2+ 5

x= ∴ ⇒ ⇒

2 y = −30 + 14 ⇒ 2 y = −16 y = −8

34. We know that, y-coordinate is zero on X-axis. Given, y1 = − 3, y2 = 2 m y 2 + n y1 y= ∴ m+ n ⇒ ⇒ ⇒

0=

m (2) + n (−3) m+ n

2m − 3n = 0 m 3 = n 2

35. Here, x1 = a , y1 = b + c x2 = b, y2 = c + a x3 = c, y3 = a + b Area of triangle 1 = [x1 ( y2 − y3 ) + x2 ( y3 − y1 ) 2 + x3 ( y1 − y2 )] 1 = [a (c + a − a − b) + b(a + b − b − c) 2 + c(b + c − c − a )] 1 = [a × (c − b) + b(a − c) + c(b − a )] 2 1 = [ac − ab + ba − bc + cb − ca ] = 0 2

https://sscstudy.com/

96

CUET (UG) Section III : General Test

CHAPTER 18

Data Interpretation Data Interpretation as the same suggests tests your skills to understand data presented in different forms like bar graphs, tables, charts, line graphs etc. The questions are given with a set of data and candidate is required to deduce the required results from the set of data. In this section, the most important thing that an individual must be able to do is to calculate fast and accurately. Adequate practice should generally set an individual well known about the course for cracking this section.

Objectives of Data Interpretation The objectives of data interpretation is as follows To test the analytical ability of the candidate. Examine the candidate’s ability to derive the useful Information from a bulk of informations. Testing the decision making ability from a given situation. Ability to check the data usability to find the solution of a problem.

Data Interpretation It can be defined as applying statistical procedure to analyse specific facts from a study or body of research. Data It is the term used to refer to the row numbers (facts and figures). Discrete Data variables Continuous

Generally, in DI, questions are asked in exams on the following topics 1. Pie chart/ Circle graph 2. Bar chart 3. Line graph

portion of entire pie chart. In this, data can be plotted with respect to only one parameter. Pie charts are useful for representing (i) percentage of various elements with respect to total quantity. (ii) proportions of various elements with respect to total quantity. (iii) shares of various elements for a particular quantity. Note In the questions of pie chart, the total quantity distributed over

a total angle of 360°.

Bar Chart It is a chart with rectangular bars with length proportional to the value which they represent. In this method of data representation, the data is plotted as bars on the X and Y-axes, where X-axis represents a discreate variable and Y-axis represents the scale for the variables. Following are the various bar charts (i) Simple Bar Chart This type of chart relates to only one variable. (ii) Subdivided Bar Chart This chart is used for represent various parts of the total magnitude of a given variable. (iii) Multiple Bar Chart In this, two or more bars are constructed adjoining to each other to represent either different components of a total or to show multiple variables.

Line Graph

Pie Chart/Circle Graph Pie chart is a special technique of data represented in the form of circle. It is divided into various sections or sector, each representing a different category and shows the

It indicates the variation of a quantity with respect to the two parameters plotted on X and Y-axes. Line graph simplifies the data as it gives a pictorial representation of data and then it is very useful in determining the trends and rate of change.

https://sscstudy.com/

https://sscstudy.com/

97

NUMERICAL ABILITY

Types of Line Graph

2. What is the difference (in tonne) between average

demand and average production of the five companies taken together?

Single Line Graph Used for single variable representation. Multiple Line Graph Used for more than one variable representation.

(a) 320

= 2100 − 1680 = 420 tonne

Directions (Q.Nos. 3 and 4) The following line diagram represents the yearly sales figures of a company in the years 2001-2010. Examine the diagram and answer the questions. 10

Directions (Q.Nos. 1 and 2) The following graph shows the demand and production of cotton by 5 companies A, B, C, D and E. Study the graph and answer the given questions.

8 7 6 5 4 3 2 1 2001 2002 2003 20042005 2006 2007 2008 2009 2010

1200

900

1800

600

Sales (in ` crore)

2700 1500

2400 1500

9

Production (in tonne) 3300

3000

(d) 1050

3000 + 600 + 2400 + 1200 + 3300  =     5 1500 1800 900 2700 + + + + 1500  −     5

Solved Examples

Demand (in tonne)

(c) 2100

Sol. (b) Required difference

Important Points Slope of graph represents the absolute growth and not the percentage growth. Simple average growth is found out by simple interest method. Cumulative average growth rate is found by compound interest method. Cumulative average growth is different from simple average growth in the sense that simple average growth is the growth between two points of time.

(b) 420

Years

3. By what per cent did the sales in 2008 decrease in

comparison to the sales in 2006? A

B

C Companies

D

(a) 20

E

1. What is the ratio of companies having more

demand than production to those having more production than demand? (a) 2 : 3 (c) 3 : 2

Sol. (c) Required ratio = 3 : 2

(b) 4 : 1 (d) 1 : 4

(b) 18

(c) 16

 Sol. (c) Percentage decrease =  

2 3

(d) 15

6− 5 2 50 % = 16 % × 100 =  3 3 6

4. The ratio of sales in 2002 to that in 2007 is (a) 2 : 3 (c) 1 : 1

(b) 1 : 3 (d) 3 : 2

Sol. (b) Required ratio = 2 : 6 = 1 : 3

https://sscstudy.com/

2 3

https://sscstudy.com/

98

CUET (UG) Section III : General Test

Practice Questions 1. The line graph below shows the number of houses sold each month by a real estate agent for the first six months of the year. Between which two months did sales increase the most?

Illiterate female 8%

Number of Houses

y Real Estate Sales 11 10 9 8 7 6 5 4 3 2 1 x′ 0 Jan Feb Mar Apr May Jun y′ Months

4. Study the following bie chart carufully and answer the question given below. Percentage of literate and illiterate males and females in a city having a population 250000

(a) April-May (c) January-February

Literate male 35% Illiterate male 24% Literate female 33%

What is the difference between the number of literate males and literate females?

x

(a) 5000

(b) May-June (d) March-April

2. The pie-diagram shows the expenditure incurred on the preparation of a book by a publisher, under various heads. A. Paper 20% B. Printing 25% C. Binding etc., 30% D. Miscellaneous 10% E. Royalty 15%

(b) 500

Agriculture

81°

(b) D and E

(c) A and E

(d) B and E

3. Study the following in formation carefully and answer the question given below. Percentage share of goods transported through Air, Road, Rail and Sea in five countries India, China, Korea, Japan, Iran. Given the total weight of goods transported in the 5 countries in a year are 1086, 3140, 1855, 2360 and 1465 thousand tonne respectively. 120

Sea

Rail

Road

80 60 40 20 India China Korea Japan

Iran

Which country transported the maximum quantity of goods by road? (a) Iran

(b) Japan

(c) China

5. The amount proposed on agriculture is more than that on industries and minerals by (a) 7.5%

(b) 10%

(c) 12%

(d) 12.5%

6. The amount (in ` crore) proposed on irrigation and power is less than that on industries and minerals by (a) 3000 (c) 2000

(b) 3500 (d) 2500

Directions (Q. Nos. 7-10) Study the following graph and give the answers of the following questions. Production of Fertilizers by a Company (in 10000 tonne) Over the Years 1995-2002 100

Air

100

0

Education Roads and Communications

Which two expenditures together will form an angle of 108° at the centre of the diagram? (a) A and D

Industries and Minerals

54°

A 20%

Production (in 10000 tonnes)

E 15%

Irrigation and Power

45° 108°

B 25%

0% D1

(d) 1500

Directions (Q.Nos. 5 and 6) The adjoining pie chart represents the proposed outlay of the fifth-five year plan of ` 40000 (in crores). Examine the chart and answer the questions.

72°

C 30%

(c) 75000

(d) Korea

https://sscstudy.com/

90 80

80 75 65

70

60

60 50

40

45

50

40 30 20

25

10 0

1995 1996 1997 1998 1999 2000 2001 2002

Years

https://sscstudy.com/

99

NUMERICAL ABILITY

7. What was the percentage decline in the production of fertilizers from 1997 to 1998? (b) 30

(c) 25

(d) 20

y

8. In how many years was the production, fertilizers more than the average production of the given years? (a) 1 (c) 3

(b) 2 (d) 4

9. In which year was the percentage increase in production as compared to the previous year, the maximum? (a) 2002 (c) 1996

(b) 2001 (d) 1997

10. The ratio of total production of fertilizers in the years 1996 and 1997 to that of total production in the years 1995, 1998 and 2000 is (a) 5 : 6 (c) 20 : 29

(b) 6 : 5 (d) 13 : 24

Directions (Q. Nos. 11-14) Study the following graph and answer the questions given below. Production of Salt by a Company (in 1000 tonne) Over the Years

10

22 20 18 16 14 12 8 6 4 2 0

x

15. How many students are coming from that locality? (a) 500

(b) 600

(c) 560

(d) 660

(b) 340

(c) 140

(d) 440

17. What is the percentage of students using Bus from that locality?

100

(a) 22

90 80

14 33

(b) 18

2 3

(c) 22

8 11

(d) 22

18. What is the ratio of the students using their means of transport as Car with those using Rickshaw?

70 60

(a) 7 : 2

50

(b) 8 : 3

(c) 2 : 7

(d) 3 : 8

Directions (Q. Nos. 19-22) The following diagram shows the percentage of population of Hindus, Sikhs and Muslims with respect to total population in a town during 2005 to 2008. Study the diagrams and answer the question.

40 30 20 10

75

2001 2002 2003 2004 2005 2006 2007 2008

Years

(b) 125 (d) 220

12. In how many of the given years was the production of salt more than the average production of the given years? (a) 1 (c) 3

(b) 2 (d) 4

13. The average production of 2004 and 2005 was exactly equal to the average production of which of the following pairs of years? (a) 2006, 2007 (c) 2002, 2006

(b) 2005, 2006 (d) 2001, 2005

14. What was the percentage decline in the production of salt from 2003 to 2004? (b) 180

Sikhs

(c) 62.4

(d) 107

https://sscstudy.com/

Muslims 65

65 60

60

Percentage of Total Population

(a) 55.5 (c) 150

Hindus

70

11. What was the percentage increase in production of salt in 2008 compared to that of 2001?

(a) 64.2

Bus Moped Bicycle Rickshaw Model of Travel

Car

(a) 240

110

Production (in 1000 tonnes)

24

16. How many students use Bicycle and Rickshaw combined?

120

0

(1 unit =10 students)

1 3

Number of Students

(a) 33

Directions (Q.Nos. 15-18) The following bar diagram represents the use of different modes of travel to school by students in a certain locality of the town. Study the graph and answer the questions.

55

55 50 45

45 40

35

35 30 25

30 25

25

20

20 15 10 5 0

15

15

10

HSM 2005

HSM HSM 2006 2007 Years

HSM 2008

https://sscstudy.com/

100

CUET (UG) Section III : General Test

19. If the total population in 2007 was 80 lakh, then the number of Hindus in 2007 was (in lakh) (a) 25

(b) 16

(c) 18

26. Which train has the second highest number of passengers?

(d) 20

(a) A

20. Percentage decrease in Hindu population form 2005 to 2008 is (a) 50 (c) 25

(b) 40 (d) 15

(b) 15 (d) 30

22. If the total number of Hindus in 2008 was 12 lakh, the number of Muslims in 2008 was (in lakh) (a) 18 (c) 24

(c) S

(b) 49 (d) 33

Directions (Q. Nos. 28-32) The pie chart provided below gives the distribution of land (in a village) under various food crops. Study the pie chart carefully and answer the questions based on it.

(b) 12 (d) 16

Wheat Rice

Directions (Q.Nos. 23-27) Study the following pie chart carefully to answer the questions.

Barley

72°

36° Jowar 18° 18° 45° Bajra

Percentage of Passenger Travelling in Six Different Trains

Train Q 19% Train R 9%

Train L 15%

(a) 18 (c) 15

Train S 24%

(a) wheat, rice and maize (c) wheat, rice and bajra

23. What was the approximate average number of passengers in train S, train M and train L together?

(b) wheat, rice and jowar (d) rice, barley and maize

30. The ratio of the land used for rice and barley is (a) 3 : 1 (c) 2 : 1

(b) 1641 (d) 1671

24. If in train R, 34% of the passengers are females and 26% are children, what is the number of males in that train?

(b) 1 : 2 (d) 3 : 2

31. If 10% of the land reserved for rice be distributed to wheat and barley in the ratio 2 : 1, then the angle corresponding to wheat in the new pie chart will be (a) 38.4° (c) 75.6°

(b) 316 (d) 318

25. The number of passengers in train Q is approximately what percentage of the total number of passengers in trains A and R? (a) 90 (c) 75

(b) 12 (d) 20

29. The combination of three crops which contribute to more than 50% of the total area under the food crops is

Total Number of Passengers = 8500

(a) 306 (c) 308

99°

28. If the total area under bajra was three hundred acre, then the total area (in hundred acre) under rice and barley together is

Train M 20%

(a) 1521 (c) 1651

72°

Others

Maize

Train A 13%

(d) M

27. How many more per cent (approximately) number of passengers are there in train M as compared to the number of passengers in train L? (a) 29 (c) 43

21. Difference of percentage of population of Hindus in 2005 and 2008 is (a) 20 (c) 25

(b) Q

(b) 76.8° (d) 45.5°

32. If the production of rice is 5 times that of jowar and the production of jowar is 2 times that of bajra, then the ratio between the yield per acre of rice and bajra is (a) 5 : 2 (c) 4 : 1

(b) 70 (d) 86

(b) 3 : 1 (d) 6 : 1

ANSWERS 1. 11. 21. 31.

(d) (b) (b) (b)

2. 12. 22. 32.

(a) (c) (d) (a)

3. 13. 23.

(a) (b) (d)

4. 14. 24.

(a) (a) (a)

5. 15. 25.

(b) (d) (d)

6. 16. 26.

(a) (b) (d)

https://sscstudy.com/

7. 17. 27.

(c) (c) (d)

8. 18. 28.

(d) (d) (a)

9. 19. 29.

(d) (d) (a)

10. 20. 30.

(a) (a) (c)

https://sscstudy.com/

NUMERICAL ABILITY

101

Hints & Solutions 1. From the line graph, it is clear that from March-April, the sales increased the most. 2. In percentage form 108° is 100 equivalent to 108°× = 30% 360° and from the pie diagram it is clear that expenditures of A and D together make 30%. 3. It is clear from the graph that Iran had maximum transportation of good via road. 4. Required difference = (35 − 33)% of 250000 = 2% of 250000 250000 × 2 = 100 = 2500 × 2 = 5000 5. Amount Spend on agriculture 108° = × 40000 = ` 12000 360° Amount Spend on Industries and minerals 72° = × 40000 = `8000 360° ∴Required percentage 12000 − 8000 = × 100 = 10% 40000 6. Required amount 72° − 45° × 40000 = 360° 27° = × 40000 = ` 3000 360° 7. Required percentage decline 60 − 45 = × 100 60 = 25% 8. Average production 25 + 40 + 60 + 45 + 65 + 50 =

+ 75 + 80 8

440 = 55 8 ∴ Required number of year = 4 =

9. Percentage increase in 1997 60 − 40 (maximum) = = 50% 40 10. Total production in years 1996 and 1997 = 40 + 60 = 100 and total production in years 1995, 1998 and 2000 = 25 + 45 + 50 = 120

100 5 = 120 6 90 − 40 11. Per cent increase = × 100 40 = 125% ∴ Required ratio =

12. Average production 40 + 30 + 70 + 25 + 55 + 50 + 80 + 90 = 8 440 = = 55 5 i.e. (70, 80, 90) > 50 13. Average production of 2004 and 2005 25 + 55 = = 40 2 As, average production of 2005 and 2006 30 + 50 = = 40 2 70 − 25 14. Required decline = × 100 70 = 64.2% (approx.) 15. Number of students coming from the locality = 6 + 15 + 11 + 18 + 16 = 66 units = 66 × 10 = 660 16. Number of students use bicycle and rickshaw = 18 × 10 + 16 × 10 = 180 + 160 = 340 17. Required percentage 15 8 = × 100 = 22 % 66 11 18. Required ratio = 6 : 16 = 3 : 8 19. Total number of Hindus in 2007 25 = 2000000 = 8000000 × 100 20. Required decrease percentage 30 − 15 = × 100 = 50% 30 21. Required difference = (30 − 15) % = 15% 22. According to the question, 15% = 1200000 1200000 × 20 ∴ 20% = 15 = 1600000

https://sscstudy.com/

23. Required average number of passenger 1 = [(24 + 20 + 15 )% of 8500] 3 1 8500 × 59 = × = 1671 3 100 24. Number of passengers in train R 8500 × 9 = = 765 100 ∴ Number of males = (100 − 34 − 26)% of 765 765 × 40 = = 306 100 25. Required per cent 19 = × 100 = 86% (13 + 9) 26. M, it is clear from the pie chart. 27. Required per cent  20 − 15  = × 100 = 33%  15  28. Corresponding angle for rice and barley = 72° + 36° = 108° Q 18° = 300 acre 300 ∴ 1° = 18 300 × 108 108° = ∴ 18 = 1800 acre 29. Q 100% = 360° ∴ 50% = 180° Now, wheat + rice + maize = 72° + 72° + 45° = 189° > 180° 30. Required ratio = 72° : 36° = 2 : 1 31. 10% of 72 = 7.2° ∴ Increase in the corresponding angle of wheat 2 = × 7.2 = 4.8° 3 New corresponding angle for wheat = 72° : 4. 8° = 76.8° 32. If the production of bajra be x tonne, then Production of jowar = 2x tonne Production of rice = 10x tonne ∴ Required ratio 10x x : = = 5 :2 72 18

https://sscstudy.com/

QUANTITATIVE REASONING

3

Quantitative Reasoning Quantitative reasoning involves the ability to solve various problems through mathematical ability along with logical and mental ability. Quantitative reasoning test is designed to test the ability of a candidate to solve the various Mathematical problems which are encountered in day to day life. To solve the problems on Quantitative reasoning, a candidate should have a knowledge of concepts of Arithmetic or basic Mathematics. Examples given below, will give you a better idea about the types of questions asked in various examinations.

Ex. 01 A man climbing up a wall of 24 m high. He climbs 16 m in a day but slipped back by 3 m 40 cm in the evening. How far had the man reached on that day? (a) 11.4 m (b) 12.6 m (c) 12 m 40 cm (d) 19 m 40 cm Sol. (b) Distance covered by man in a day = (16 - 3.4) m = 12.6 m

Ex. 02 The weights of 4 boxes are 90, 40, 80 and 50 kilograms. Which of the following cannot be the total weight, in kilograms, of any combination of these boxes and in a combination a box can be used only once? (a) 200 (c) 180

(b) 260 (d) 170 Sol. (a) For option (b), 90 + 40 + 80 + 50 = 260

Ex. 03 A shepherd had 17 sheeps. All but nine died. How many was he left with? (b) 9 (d) 8

Sol. (b) According to the question, ‘All but nine died’. This statement means that ‘All except nine died’ i.e. nine sheeps remained alive and others died. So, shepherd was left with 9 sheeps.

Ex. 04 John used to buy petrol at the rate of ` 80 per litre till last month. Now he buys it at the rate of ` 85 per litre. By what percentage did the petrol price increase as compared to last month? (b) 8.35% (d) 10%

(a) 48

(b) 16

(c) 56

(d) 32 80 ´ 80 = 64 100 Now, let the number of questions answered correctly = x \ Number of questions answered wrongly = 64 - x According to the question, x ´ 1 + (64 - x) x - 1 = 32 x - 64 + x = 32 Þ 2x = 64 + 32 Þ 2x = 96 \ x = 48 \ Number of questions answered correctly = 48

Sol. (a) Question attempted = 80% of 80 =

Ex. 06 How many odd numbered pages are there in a book of 1089 pages? (b) 545

(c) 544

(d) 546

Sol. (b) Odd pages in the book are 1, 3, 5, 7, 9, … 1089.

But in option (a), the sum of any numbers given in question is not 200.

(a) 6.25% (c) 5.5%

5 ´ 100 = 6.25% 80

Ex. 05 In an exam of 80 questions, a correct answer is given 1 mark, a wrong answer is given -1 mark and if a question is not attempted there are zero marks. If a student attempted only 80% of the questions and got 32 marks, then how many questions did he answer correctly?

(a) 542

For option (c), 90 + 40 + 50 = 180 For option (d), 90 + 80 = 170

(a) 17 (c) Nil

Sol. (a) Price increased = 85 - 80 = ` 5 \ Percentage increased =

Here, every alternate page is odd starting from 1. So, if the total number of pages was an even numbered, then the half number of pages will be odd numbered and half will be even. But, here the total number of pages is odd. Total number of pages + 1 So, required number of pages = 2 1089 + 1 1090 = 545 = = 2 2

Ex. 07 At the end of a business conference all the ten people present, shake hands with each other only once. How many handshakes were there altogether? (a) 20

(b) 45 (c) 55 (d) 90 Sol. (b) Clearly, the total number of hand shakes = 9 + 8 + 7 + 6 + 5 + 4 + 3 + 2 + 1 = 45 It can also be calculated with the help of formula. n (n - 1) Total number of handshakes = 2 where, n = number of people 10 (10 - 1) 10 ´ 9 90 Total number of handshakes = = = = 45 2 2 2

https://sscstudy.com/

https://sscstudy.com/

4

CUET (UG) Section III : General Test

Ex. 08 There are deer and peacocks in a zoo. By counting heads they are 80. The number of their legs is 200. How many peacocks are there? (a) 60

(b) 50

(c) 20

Students who can play only Flute and Guitar = 10 - (3 + 2 + 4) = 10 - 9 = 1 \ Students who can play only Flute = 12 - (3 + 3 + 1)

(d) 30

Sol. (a) Assuming deer = d and peacock = p ´4 Head ® d + p = 80 ¾ ¾ ¾® 4d + 4 p = 320l

…(i)

Legs ® 4d + 2 p = 200 ® 4d + 2 p = 200

…(ii)

(As, a deer has 4 legs and a peacock has 2 legs) From subtracting Eq. (ii) from Eq. (i), we get 120 2 p = 120 Þ p = = 60 2

Ex. 09 In a music class, 12 students can play flute, 11 can play guitar and 10 can play violin. 6 students can play flute as well as guitar and 3 out of these can also play violin. 3 students can play only guitar and 4 students can play only violin. How many students can play only flute? (a) 4

(b) 5 (c) 6 Sol. (b) According to the given information,

= 12 - 7 = 5

Ex. 10 A survey of 100 candidates with respect to their choice of icecream flavour-vanilla, chocolate and strawberry produced the following information. 50 candidates like vanilla, 43 like chocolate, 28 like strawberry, 13 like vanilla and chocolate, 11 like chocolate and strawberry, 12 like strawberry and vanilla and 5 like all the three flavours. How many candidates like chocolate and strawberry, but not vanilla? (a) 24

(b) 10

(c) 32

(d) 6

Sol. (d) According to the given information, 13

100 Vanilla (50)

(d) 7

Chocolate (43)

5 12

11

4

6 Strawberry (28)

Flute (12)

5 1

3 3 4

Guitar (11)

3 2

Since, number of candidates who like chocolate and strawberry = 11 And, number of candidates who like all the three flavours (chocolate, vanilla, strawberry) = 5

Violin (10)

Students who can play only Guitar and Violin = 11 - (3 + 3 + 3) = 11 - 9 = 2

\ Number of candidates who like chocolate and strawberry, but not vanilla = 11 - 5 = 6

Practice Questions 1. The sum of all the 3-digit numbers which are formed by the digits 1, 2 and 3 without repetition of digits, is (a) 1233 (b) 1321 (e) None of these

(c) 1323

(d) 1332

2. A florist had 133 roses. She sold 5/7 of them. How many roses had she left? (a) 58

(b) 38

(c) 57

(d) 19

3. ‘4 is even and 8 is odd’. What is the truth value of this? (a) Given statement is true (c) 32

(b) Given statements is false (d) Cannot be determined

4. At a party, the number of girls is half the number of

(c) is equal to Yellow balls minus Green balls (d) Cannot be ascertained

6. Two horses A and B run at a speed of 3 : 2 ratio in the first lap; during the second lap the ratio differs by 4 : 7; during the third lap the ratio differs by 8:9. What is the difference in ratio of speed altogether between the two horses? (a) 4

(b) 8

(c) 24

(d) 2

month after month as follows. Find the number of students in June from the following information.

(d) 22

5. There are some balls of Red, Green and Yellow colour lying on a table. There are as many Red balls as there are Yellow balls. There are twice as many Yellow balls as there are Green ones. The number of Red balls is (a) is equal to the sum of Yellow and Green balls (b) is double the number of Green balls

(c) 3

7. The number of students in an art class is increasing

boys. After an hours, five boys leave the party and three girls join the party. How many people were present at the party an hour before? (a) 16

(b) 1

(a) 16

https://sscstudy.com/

Month

Number of students

January

1

February

2

March

4

April

7

May

11

June

? (b) 13

(c) 15

(d) 14

https://sscstudy.com/

QUANTITATIVE REASONING

8. The heights of three towers are in the ratio 5 : 6 : 7. If a spider takes 15 min to climb the smallest tower, how much time will it take to climb the highest one? (a) 15 min

(b) 18 min

(c) 21 min

(d) 54 min

Which of the following cannot be the total weight, in kilograms, of any combination of these boxes and in a combination a box can be used only once. (a) 300

(b) 230

(c) 220

(d) 290

days. Find the price in the 3rd week of February.

Price

20

(a) 140

60 (b) 300

Jan. Jan. Feb. 1st 1st 3rd week week week 40

120

Feb. 3rd week

100

(c) 180

(d) 320

meeting all people shake hand to each other. Find the number of handshakes at the end of a meeting? (b) 28

(c) 48

(d) 64

12. A person is given 1 rupee for shooting at the target and if he misses, then he has to give 1 rupee. If he gets 30 rupees for 100 chances then how many chances did he miss? (a) 25

(b) 35

(c) 40

(d) 45

13. There were a total of 10 bicycles and tricycles. If the total number of wheels was 24, how many tricycles were there? (a) 2

(b) 6

(c) 18

(d) 4

14. P is greater than Q by 60% and greater than R by 30%. The ratio of Q and R is (a) 1 : 2 (c) 13 : 16

(b) 2 : 1 (d) 16 : 13

14 more than twice the number of heads. Find the number of cows? (b) 7 (d) 12

same speed, 2 h and 40 min to complete a particular task about how long will it take one worker, working at the same speed, to complete the same task alone? (a) 1 h 20 min (c) 5 h

(b) 4 h 40 min (d) 5 h 20 min

had ` 500 with him while Rahul had ` 240. Sonu spent twice as much as Rahul on shopping. Now, Sonu has three times as much money as is left with Rahul. How much money did Sonu spend? (b) ` 60 (d) ` 120

(b) 1400

(c) 1900

(d) 1200

20. The flowers kept in a basket doubles in every one minute. If the basket gets completely filled by flowers in 30 min, then in how many minutes 1 th of the 4 basket was filled with flowers? (a) 15 min

(b) 28 min

(c)

15 min 2

(d)

45 min 2

21. Some birds are sitting on two branches A and B. If one bird of the branch A fly away and sit on branch B, then the number of birds on both the branches will become equal. But if a bird from branch B flies and sits on the branch A then the number of birds on branch A is double of branch B. What number of birds were sitting on branch A at the beginning? (a) 3

(b) 4

(c) 5

(d) 7

22. A factory produced 1858509 cassettes in the month of January, 7623 more cassettes in the month of February and owing to short supply of electricity produced 25838 less cassettes in March than in February. Find the total production in all? (a) 5557312 (c) 5564935

(b) 5983245 (d) 5608988

23. Which number will be in the middle if the following numbers are arranged in descending order? 4456, 4465, 4655, 4665, 4565 (b) 4465

(c) 4565

(d) 4655

24. Two persons A and B get the same salary. Their basic pay are different. The allowances of A and B are 65% and 80% of the basic pay respectively. What is the ratio of the basic pay? (b) 7 : 5 (d) 12 : 11

25. A train is running 3 min late and further being late 3 s/min. Find how long this train will be delayed one hour? (a) 1140 min (c) 1160 min

(b) 1150 min (d) 1200 min

26. A machine cuts the wooden log into 10 m pieces and

17. Sonu and his friend Rahul went for shopping. Sonu

(a) ` 220 (c) ` 440

(d) 90

of female workers in the company is 800, what is the number of male workers in the company?

(a) 17 : 15 (c) 11 : 10

16. If it takes two workers, working separately but at the

(c) 75

19. In a company, 60% workers are males. If the number

(a) 4456

15. In a group of cows and hens, the number of legs are

(a) 5 (c) 10

(b) 60

?

11. 8 people are present in a meeting. In the end of

(a) 56

notes, five-rupee notes and ten-rupees notes. The number of notes of each denomination is equal. What is the total number of notes that he has?

(a) 1600

10. The price of onions is shown below for every fiftheen Period Dec. Dec. 3rd 1st week week

18. A man has ` 480 in the denominations of one-rupee

(a) 45

9. The weights of 4 boxes are 80, 60, 90 and 70 kg.

5

take 6 s to cut a 10 m piece. Find the time to cut the 3 km long wooden log into pieces? (a) 174 s

(b) 180 s

(c) 1794 s

(d) 1800 s

27. A tailor has to cut 10 shirt pieces of equal length from a roll of fabric. He cuts at the rate of 45 shirt pieces in a minute. How many rolls would be cut in 24 min? (a) 120

https://sscstudy.com/

(b) 108

(c) 84

(d) 72

https://sscstudy.com/

6

CUET (UG) Section III : General Test

28. In a zoo, there are rabbits and pigeons. If heads are counted, there are 200 heads and if legs are counted there are 580 legs. How many pigeons are there? (a) 90

(b) 100

(c) 110

(d) 120

29. In a plane, line X is perpendicular to line Y and parallel to line Z, line U is perpendicular to both lines V and W, line X is perpendicular to line V. Which one of the following statement is correct? (a) Z, U and W are parallel (b) X, V and Y are parallel (c) Z, V and U are all perpendicular to W (d) Y, V and W are parallel

(c) 11

(d) 15

The number of boys is less than 2 the number of girls. Father gives ` 10 to boys and ` 20 to girls as a gift. If total amount spent is ` 280, find the number of boys. (b) 10 (d) 14

(b) 60 (d) 70

(b) 8 (d) 12

34. Consider the following venn-diagram.

(b) 1.0

Hindi

(c) 6.8

(d) 7.8

on doordarshan, 44% people read newspaper and 24% people read newspaper and watch doordarshan both. How many percentage of people neither read newspaper nor watch doordarshan. (b) 18%

(c) 24

(d) 68

(a) 60

(b) 61

(c) 65

(d) 71

Directions (Q. Nos. 39-41) Read the following information and answer the following questions. In a class of 40 students, 28 can speak Tamil and 30 can speak Telugu. All students can speak at least one of the two languages.

(a) 8

(b)10

(c) 12

(d) 14

40. Find the minimum number of students who can speak both Tamil and Telugu. (a) 12

(b) 15

(c) 18

(d) 22

(c) 10%

(a) 8

(b) 10

(c) 12

(d) 14

42. A travel agent surveyed 100 people to find out how many of them had visited the cities of Mumbai and Bengaluru. 31 people had visited Mumbai, 26 people had visited Bengaluru and 12 people had visited both cities. Find the number of people who had visited neither Mumbai nor Bengaluru. (b) 55

(c) 19

(d) 45

43. A survey of 500 TV viewers produced the following

Math

35. According to a survey report, 62% people watch news

(a) 8%

(b) 32

38. How many students like only one vegetable?

(a) 12

500 candidates appeared in an examination comprising of tests in English, Hindi and Mathematics. The diagram gives the number of candidates who failed in different tests. What is the percentage of the candidates who failed in atleast two subject? (a) 0.078

(a) 16

Telugu?

animals are herded by a herdsman. The number of bulls are twice the number of cows. If total numbers of heads is 186 less than total number of legs (with herdsman). Find the number of herdsmen?

English

(d) 4

41. Find the number of persons who can speak only

33. A group has some cows, bulls and 45 hens. Every 15

50 12 12 5 30 10 75

(c) 16

Tamil.

examination. If C got 5 marks less than B, D got 10 marks more than B and 20 marks less than A and E got 22 marks more than B and B got 40 marks. Then, how many marks did A get?

(a) 6 (c) 10

(b) 18

39. Find the number of students who can speak only

32. 5 students A, B, C, D and E are present in an

(a) 52 (c) 64

cauliflower is

cabbage?

31. A father invites some boys and girls on his son’s birthday.

(a) 8 (c) 12

36. The difference between the people who like carrot and

37. What is the percentage of students that do not like

by 2 or 3.’’ This statement is false, if the number which I am thinking. (b) 8

In a group of 75 students, 12 like only cabbage, 15 like only cauliflower, 21 like only carrot, 12 like both carrot and cabbage, 13 like only capsicum and 2 like both capsicum and cauliflower.

(a) 6

30. Marry said, ‘‘a number which I am thinking is divisible

(a) 6

Directions (Q. Nos. 36-38) Consider the following information and answer the questions based on it.

(d) 0%

information. 285 viewers watch football games, 195 viewers watch hockey games, 115 viewers watch basketball game, 45 viewers watch football and basketball games, 70 viewers watch football and hockey games, 50 viewers watch hockey and basketball games and 50 viewers do not watch any of three games. How many viewers watch exactly one of the three games? (a) 440 (c) 205

(b) 365 (d) 325

44. In an airline, hot drinks and iced tea were served to 120 passengers. 75 passengers enjoyed hot drinks and 62 enjoyed iced tea. If 40 enjoyed both, then how many passengers enjoyed none of the beverages? (a) 15 (c) 23

https://sscstudy.com/

(b) 12 (d) 25

https://sscstudy.com/

ANSWERS 1. 11. 21. 31. 41.

(d) (b) (d) (a) (c)

2. 12. 22. 32. 42.

(b) (b) (c) (d) (b)

3. 13. 23. 33. 43.

(b) (d) (c) (a) (d)

4. 14. 24. 34. 44.

(c) (c) (d) (d) (c)

5. 15. 25. 35.

(b) (b) (a) (b)

6. 16. 26. 36.

(c) (d) (c) (c)

7. 17. 27. 37.

(a) (c) (b) (d)

8. 18. 28. 38.

(c) (d) (c) (b)

9. 19. 29. 39.

(d) (d) (d) (b)

10. 20. 30. 40.

(b) (b) (c) (c)

Hints & Solutions 1. (d) The 3-digit numbers which are formed by digits 1, 2 and 3 without repetition of digits are given below 312 + 321 + 123 + 231 + 132 + 213 = 1332 5 2. (b) Florist sold = 133 ´ 7 = 95 roses \Remaining roses = 133 - 95 = 38 3. (b) 4 is even but 8 is not odd. So, the statement is false. 4. (c) Let the number of boys = 2x \Number of girls = x \Total number of persons = 2x + x = 3x So, the total number of persons must be divided by 3. \From the given alternatives, only 24 is divided by 3. Hence, 24 people were present at the party an hour before. 5. (b) Let the number of Green balls = x \Number of Yellow balls = 2x \Number of Red balls = 2x It is clear from above that the number of Red balls is twice the number of Green balls. 6. (c) Difference between ratio of speed of both horses = (2 + 7 + 9) - (3 + 4 + 8) = 18 - 15 = 3 \Required difference = 3 7. (a) January Þ 1 February Þ 1 + 1 = 2 March Þ 2 + 2 = 4 April Þ 4 + 3 = 7 May Þ 7 + 4 = 11 \June Þ 11 + 5 = 16 8. (c) Spider climbs 5x units 15 min \Spider will climb 7x units in

15 ´ 7 = 21 min 5 9. (d) For option (a), = 80 + 60 + 90 + 70 = 300 For option (b), = 80 + 60 + 90 = 230 For option (c), = 60 + 90 + 70 = 220 And in option (d) the sum of any numbers given in option is 290. So, option (d) cannot be the total weight, in kg, of any combination of these boxes. 10. (b) In each month the price of onions in 3rd week in 3 times to the price of onions in 1st week. So, the price of onions in 3rd week of Feb = 100 ´ 3 = 300 11. (b) Number of handshakes n (n - 1) = 2 Here, n = 8 \Required number of 8(8 - 1) handshakes = 2 8´7 = = 28 2 12. (b) Assuming the number of chances missed by the person be x. Number of chances to shoot the target = 100 - x According to the question, (100 - x ) - x = 30 Þ 100 - 2x = 30 Þ 2x = 100 - 30 = 70 70 x= = 35 2 13. (d) Let the required number of tricycles = x Then, the number of bicycles = 10 - x

https://sscstudy.com/

According to the question, 3x + 2(10 - x) = 24 [Q Tricycles = 3 wheels; Bicycle = 2 wheels] Þ x + 20 = 24 Þ x=4 Therefore, the required number of tricycles x = 4 14. (c) Given, P = 160% of Q = 130% of R Þ Þ

160% of Q = 130% of R 160 130 Q´ =R´ 100 100

Þ

Q ´ 16 = R ´ 13 Q 13 = R 16

Þ

Q : R = 13 : 16

Þ

15. (b) Let the number of hens be x and cows be y. Number of legs in the group = 2x + 4 y Number of heads in the group =x+ y According to the question, legs = 2 (heads) + 14 2x + 4 y = 2(x + y) + 14 Þ 2x + 4 y = 2x + 2 y + 14 Þ 2x - 2x + 4 y - 2 y = 14 Þ 2 y = 14 y=7 (therefore number of cows = 7) 16. (d) Given that the two workers, working separately but at the same speed takes 2 h and 40 min to complete a particular task. Then, one worker, working at the same speed, to complete the same task will take double time i.e. 5 h 20 min. 17. (c) Let expenditure of Rahul = `x and expenditure of Sonu = ` 2x Now, 500 - 2x = 3 (240 - x )

https://sscstudy.com/

8

CUET (UG) Section III : General Test

Þ 500 - 2x = 720 - 3x Þ x = ` 220 \ Expenditure of Sonu = 2x = `440 18. (d) Let the number of notes of each denomination = x Total amount = 480 x + 5x + 10x = 480 Þ 16x = 480 Þ x = 30 So, required total number of notes = x + x + x [number of notes of each denomination is equal] Þ 3x Þ 3 ´ 30 Þ 90 19. (d) Let the total number of workers = x Given, number of female workers = 800 Then, according to the question, 60 of x + 800 = x 100 60x 800 x= 800 Þ x = \ 100 0.4 8000 = = 2000 4 Number of male workers 60 = ´ 2000 = 1200 100 20. (b) Q 30 min ® 1

[Full] 1 \ (30 - 1)min Þ 29 min ® 2 1 \ (29 - 1)min Þ 28 min ® 4 Therefore, the expected time is 28 min.

21. (d) Assuming the number of birds on A = x and B = y In Ist stage, x - 1 = y + 1 …(i) In IInd stage, x + 1 = 2( y - 1) …(ii) From Eqs. (i) and (ii), we get (x + 1) = 2(x - 2 - 1) Þ x + 1 = 2 (x - 3) Þ x + 1 = 2 x -6 Þ 2x - x = 7 Þ x=7 Therefore, total number of birds on branch ‘A’ = 7. 22. (c) Production in January = 1858509 Production in February = 1858509 + 7623 = 1866132 Production in March = 1866132 - 25838 = 1840294

\Total production = 1858509 + 1866132 + 1840294 = 5564935 23. (c) According to the question, arrangement of numbers in descending order 4665, 4655, 4565, 4465, 4456 \ Number in middle = 4565 24. (d) Suppose basic pay of A = ` x Basic pay of B = ` y Ratio between basic pays x ´ 165 y ´ 180 Þ = 100 100 x 180 Þ = y 165 12 = = 12 : 11 11 25. (a) Given, the train is running 3 min late. Therefore, to be delayed by one hour it has to be delayed = 60 - 3 = 57 min. Q The train is being late 3 sec or 3 min in one minute. 60 \The train will be late by 57 min in 57 ´ 1 57 ´ 60 = = 3 3 60 = 1140 min 26. (c) The length of the wooden log = 3 km = 3 ´ 1000 = 3000 m The length of the piece cut by machine = 10 m \Total number of pieces cut by 3000 the machine = = 300 10 To cut 300 pieces the machine has to cut 299 pieces because in the last step when machine cut 299th piece then only one piece i.e. 300th piece will remain. Q Time required to cut a piece = 6 sec \Time required to cut 299 pieces = 299 ´ 6 = 1794 s 27. (b) Shirt pieces cut by the tailor in 24 min 24 ´ 45 = 1080 Q Number of shirt pieces in a roll = 10 \Required number of rolls 1080 = 108 = 10

https://sscstudy.com/

28. (c) Let the number of rabbits is x and the number of pigeons is y. According to the question, …(i) x + y = 200 and …(ii) 4x + 2 y = 580 (Q A rabbit has 4 legs and a piegon has 2 legs) or 2x + y = 290 …(iii) From Eqs. (i) and (iii), x = 90 and y = 110 Hence, number of piegons = 110 29. (d) According to the question, W U

Z

X Y

V

Clearly, Y, V and W are parallel. 30. (c) From the given options, number 6, 8 and 15 are divisible by either 2 or 3. Only number 11, is not divisible by 2 or 3. Hence, the required number is 11. 31. (a) Let the number of girls = x \Number of boys = (x - 2) According to the question, (x - 2) ´ 10 + x ´ 20 = 280 Þ 10x - 20 + 20x = 280 Þ 30x = 280 + 20 Þ 30x = 300 Þ x = 10 \Number of boys = 10 - 2 = 8 32. (d) Given, B got = 40 marks Then, C got = 40 - 5 = 35 marks D got = 40 + 10 = 50 marks A got = 50 + 20 = 70 marks E got = 40 + 22 = 62 marks Hence, it is clear from above that A got 70 marks in the examination. 33. (a) Let the number of cows be x, number of bulls be y and the number of herds man be z. Then, according to the question, x + y + 45 + z =z 16 Þ x + y + 45 = 16z - z (Q y = 2x) Þ x + 2x + 45 = 15z …(i) Þ 3x + 45 = 15z and (3x + z + 45) = (4 ´ 3x + 2 ´ z + 2 ´ 45) - 186 Þ 15z + z = 12x + 2z + 90 - 186 Þ 16z - 2z - 12x = -96

https://sscstudy.com/

GENERAL MENTAL ABILITY

(15z - 45) = -96 3 Þ 14z - 4 ´ (15z - 45) = -96 Þ 14z - 60z + 180 = - 96 Þ - 46z = -276 \ z =6 Hence, it is clear from above that there are 6 herds man. Þ 14z - 12 ´

34. (d) According to the question, Total number of candidates = 500 Candidates failed in Math and Hindi = 12 Candidates failed in Math and English = 12 Candidate failed in Hindi and English = 10 Candidates failed in three subjects = 5 So, number of candidates who failed in atleast two subjects = 12 + 12 + 10 + 5 = 39 \Percent candidates failed in atleast two subjects 39 39 = ´ 100 = = 7.8 500 5 35. (b) 18%

Doordarshan

38% 24% 20%

Newspaper

Total number of people who watch doordarshan and who read newspaper (A È B ) = A + B - (A Ç B ) = 62 + 44 - 24 = 82% Hence, percentage of people who neither read newspaper nor watch doordarshan = 100 - 82 = 18% Sol. (Q. Nos. 36-38) According to the information 12 12 21 Cabbage

15 2 13

Carrot Cauliflower Capsian

36. (c) Required difference = (12 + 21) - (15 + 2) = 33 - 17 = 16 37. (d) Students who do not like cabbage = 75 - (12 + 12) = 51

51 ´ 100 75 = 68%

\ Required percent =

38. (b) Number of students who like only one vegetable = 12 + 21 + 15 + 13 = 61 Sol. (Q. Nos. 39-41) According to the question, Tamil

Telugu

10 18 12

Let x students can speak both Tamil and Telugu. So, only Tamil = 28 - x Only Telugu = 30 - x According to the question, Þ (28 - x) + (30 - x) + x = 40 Þ 58 - x = 40 x = 18 39. (b) Number of students who can speak only Tamil = 28 - x = 28 - 18 = 10 40. (c) Students who can speak both Tamil and Telugu x Þ 18. 41. (c) Number of students who can speak only Telugu 30 - 18 Þ 12. 42. (b) Number of people who visited Mumbai = 31 Number of people who visited Bengaluru = 26 Number of people who visited both cities = 12 And, total number of people = 100 \Number of people who visited neither Mumbai nor Bengaluru = 100 - {(31 + 26) - 12} = 100 - 57 + 12 = 112 - 57 = 55 43. (d) Total number of people, n (P ) = 500 People who watch Basketball, n (B) = 115 People who watch Football, n (F ) = 285 People who watch Hockey, n (H ) = 195 People who watch Basketball and Hockey, n (B Ç H ) = 50.

https://sscstudy.com/

9

People who watch Football and Hockey, n (H Ç F ) = 70 People who watch Basketball and Football, n (B Ç F ) = 45 People who do not watch any games, n (H È B È F ) = 50 Now, n (H È B È F ) = n (P ) - n (H È B È F )¢ Þ n (H È B È F ) = n (P ) - [n (H ) + n (B) + n (F ) - n (H Ç B) - n (H Ç F ) - n (B Ç F ) + n (H Ç B Ç F )] Þ 50 = 500 - [285 + 195 + 115 -70 - 50 -45 + n (H Ç B Ç F )] Þ 50 = 500 - 430 - n (H Ç B Ç F ) Þ n (H Ç B Ç F ) = 70 - 50 = 20 \20 people watch all three games. Number of people who only watch Football = 285 - (50 + 20 + 25) = 190. Number of people who only watch Hockey = 195 - (50 + 20 + 30) = 195 - 100 = 95 Number of people who only watch Basketball = 115 - (25 + 20 + 30) = 115 - 75 = 40 \Number of people who watch exactly one of the three games = 190 + 95 + 40 = 325 44. (c) Total passenger n (P ) = 120 Number of passenger enjoyed hot drinks n (H ) = 75 Number of passenger enjoyed iced tea n (I ) = 62 Number of passenger who enjoyed both n (I Ç H ) = 40 Number of passenger who enjoyed at least one beverage n (H È I ) = n (H ) + n (I ) - n (H Ç I ) = 75 + 62 - 40 = 97 \Number of passenger enjoyed none of the beverages = n (P ) - n (H Ç I ) = 120 - 97 = 23

https://sscstudy.com/

LOGICAL AND ANALYTICAL REASONING

3

Logical and Analytical Reasoning Analytical thinking involves objective analysis of facts to form a judgement. Analytical reasoning questions are designed to assess the ability to consider a group of facts and rules and determine what could or must be true. Analytical reasoning questions test a range of deductive reasoning skills. These include ●







Comprehending the basic structure of a set of relationships by determining a complete solution to the problem posed. Reasoning with conditional (‘‘if-then’’) statements and recognising logically equivalent formulation of such statements. Inferring what could be true or must be true from given facts and rules. Recognising when two statements are logically equivalent in context by identifying a condition or rule that could replace one of the original conditions while still resulting in the same possible outcomes.

In this chapter we will discuss six types of problems 1. Syllogism 2. Statement and conclusion 3. Statement and Argument 4. Statement and Assumptions 5. Assertion and Reason 6. Cause and Effects

Propositions Proposition is a statement giving a relation between two terms. It is also known as premise.

Classification of Proposition This classification is based upon the quantity and quality of proposition. Here, quantity denotes whether the proposition is universal or particular and quality denotes the proposition is affirmative or negative. There are four types of propositions which are as follows: Propositions

Quantity

Quality

A

All A are B

Universal

Affirmative

E

No A are B

Universal

Negative

I

Some A are B

Particular

Affirmative

O

Some A are not B Particular

Venn Diagram Representation of Four Propositions Types of Propositions

Venn Diagram

A : All S is P

E : No S is P

In this type of questions, a set of statements along with two or more conclusions is given on the basis of these statements candidates are required to check the correctness of conclusions whether the conclusions follows from statement or not.

S p

S P

or

or

It is deductive argument in which conclusion has to be drawn from the given statements.

Either

or

P S

or

P S S

Always Some S are P [All S are P]

Some S are P [All S are P] Some S are not P [All P are S]

P S

O : Some S are not P

https://sscstudy.com/

p

S

Either

Syllogism is a greek word which means inference or deduction.

Always

S P

I : Some S are P

1. Syllogism

Negative

Some S are not P [Some P are not S]

Some S are not P [All P are S] p Some S are not P [No S are P]

https://sscstudy.com/

4

CUET (UG) Section III : General Test

Directions (Ex. Nos. 1 and 2) In each of the following

Sol. We can draw all possible cases as given below

questions two statements along with a set of conclusions is given. You have to take the statements to be true even if they seem to be at variance from the commonly known facts and decide which of the following conclusions logically follows from the given statements Give answer

Ex. 1 Statements Some birds are Donkeys. All donkeys are stupid. I. All birds are stupid. II. Some birds are stupid. Sol. (b) Donkeys

Birds

Buses

Scooters (a)

Cars (b)

Statements

Some walls are doors. Some doors are coats. Some costs are chairs.

Conclusions

I. Some chairs are doors. II. Some coats are walls. III. No Chair is doors.

(a) Only II follows (c) Either I or III follows

II. ✓

Here, only Conclusion II follows.

(b) Only III follows (d) Only I follows

Sol. (c)

Ex. 2 Statements All teachers are students.

Walls Doors Coats Chairs

No student is girl. Conclusions

Buses

Ex. 3 Some statements are given in following question followed by some conclusions. You have to take the given statements to be true even, if they seems to be at variance from commonly known facts. Then answer the following question.

Stupid

Conclusions I. ✗

Scooters

Here, using both diagrammatical representations we can conclude either ‘Some cars are buses’ or ‘No cars are buses’ Hence, atleast one of the conclusions must be true.

(a) if only Conclusion I follows (b) if only Conclusion II follows (c) if neither Conclusion I nor II follows (d) if both Conclusions I and II follow

Conclusions

Car

Conclusions

I. All girls are students. II. Some girls are student.

I. ×

II. ×

III. ×

or

I or III makes complementary pairs. Either I or III follows.

Sol. (c) Teachers

2. Statement and Conclusions

Girls

In these type of questions a statement is given followed by some conclusions. The candidate is required to go through the statements throughly and then decide which of the given conclusion follows on its basis.

Students

Conclusions I. ✗

II. ✗

Hence, neither Conclusion I nor II follows.

Complementary Pair of Conclusions (‘either’ or Situation) In drawing inferences from given statements, students are required to be more attentive to select complementary pair of conclusions, where neither of the conclusions is definitely true but a combinations of both makes a complement argument pair. Let us consider the example

Some cars are scooters. Some scooters are buses. Conclusions I. Some cars are buses. II. No cars are buses. Statements

A statement is a formal account of certain facts, views, problems or situations expressed in words. A conclusion is a belief or an opinion that is the result of reasoning out a given statement.

Directions (Ex. Nos. 4 and 5) In the following questions a statement is followed by two conclusions I and II. Taking the statement to be true decide which of the given conclutions definitely follows from the given statement. Ex. 4 Statement In a one day cricket match, the total runs made by a team were 200. Out of these, 160 runs were made by spinners. Conclusions I. 80% of the team consists of spinners. II. The opening batsmen were spinners.

https://sscstudy.com/

https://sscstudy.com/

LOGICAL AND ANALYTICAL REASONING

(a) Only Conclusion I follows (b) Only Conclusion II follows (c) Neither I nor II follows (d) Both I and II follow

5

Give answer

Sol. (c) According to the statement, 80% of the total runs were made by spinners. So, I does not follow. Nothing about the opening batsmen is mentioned in the statement. So, II also does not follow.

Ex. 5 Statement followed by some conclusions are given below. Statements I. Knowledge is not a skill. It is learning and experience. II. The illiterate are not those who cannot read and write but those who cannot learn. Conclusions I. Those who cannot read and write are literate. II. Knowledge and skill are two different things. Find which of the given conclusions logically follows from the given statements. (a) Only Conclusion I follows (b) Only Conclusion II follows (c) Both I and II follow (d) Neither I nor II follows

Sol. (b) From Statement I, knowledge is learning and experience, it is not a skill. Hence, knowledge and skill are two different things. So, Conclusion II follows. Statement II does not mean that those who cannot read and write are literate. So, Conclusion II does not follows.

3. Statement and Argument An argument is a fact or set of facts that is being given by a person to support an idea, statement or theory in its favour or against it. In this topic, a statement followed by certain arguments is given. The statement is concerned with an issue. These issues may be political, social or economical. The arguments are in favour or against the given statement. Generally, the two arguments are contrary to each other and refer to the positive and negative results of the action as mentioned in the statement issue.

Directions (Ex. Nos. 6 and 7) Study the following instructions carefully and then answer the questions that follow. In making decisions about important questions, it is desirable to be able to distinguish between ‘strong’ and ‘weak’ arguments so far as they relate to the questions. ‘Weak’ arguments may not be directly related to the question and may be of minor importance or may be related to the trivial aspect of the question. Each question below is followed by two arguments numbered I and II. You have to decide which of the arguments is a ‘strong’ argument and which is a ‘weak’ argument?

(a) if only Argument I is strong (b) if only Argument II is strong (c) if neither I not II is strong (d) if both I and II are strong

Ex. 6 Statement Should luxury hotels be banned in India? Arguments I. Yes, these are place form where international criminals operate. II. No, affluent foreign tourists will have no place to stay. Sol. (b) The luxury hotels are symbols of country’s development and a place for staying the affluent foreign tourists. So. argument II is a strong one. Argument I is a weak argument because ban on luxury hotels is not a way to stop the international criminals.

Ex. 7 Statement Should there be no examination upto Std IX in all the schools in India? Arguments I. No, students need to go through the process of giving examinations right from young age. II. Yes, this will help students to think laterally and achieve their creative pursuits. Sol. (d) Argument I is strong as school is the ground where we prepare for the future battles of life. Argument II is strong as examinations kill our creativity, turning us all into more clerks.

4. Statement and Assumptions In this, a statement is given and assumptions are drawn from it. An assumption is something assumed, supposed and taken for granted. The implicity of the two assumptions is in question. The implication means the hidden meaning, something which is derived from and as such based upon the statement.

Directions (Ex. Nos. 8 and 9) In each question below is given a statement followed by two Assumptions I and II. You have to consider the statement and the following assumptions and decide which of the assumptions is implicit in the statement. Give answer (a) if only I is implicit (c) if only II is implicit

(b) if neither I nor II is implicit (d) if both I and II are implicit

Ex. 8 Statement The Prime minister is expected to announce an expansion in his ministry shortly. A newspaper report. Assumptions I. The newspaper has quoted authentic sources. II. The newspaper has reliable sources for the news. Sol. (c) The newspaper has not quoted any authentic sources, so I is not implicit, but II is implicit.

https://sscstudy.com/

https://sscstudy.com/

6

CUET (UG) Section III : General Test

Ex. 9 Statement Education and social change are complementary; and leads to a steady loss of tradition on people’s mind.

Ex. 12 Assertion (A) We feel colder on mountains than on plains.

Assumptions

Sol. (a) Both A and R are true and R is the correct

I. There is a direct relationship between education and social change and decline of tradition on people’s mind. II. Traditions can make priceless contribution to society. Sol. (a) Only Assumption I is implicit in the given statement. Assumption II is a generalised conclusion.

5. Assertion and Reason

Reason (R) Temperature decreases with altitude. explanation of A. Above the sea level, temperature decreases with an increase in altitude which makes mountain peaks colder.

Ex. 13 Assertion (A) Plants convert light energy into chemical energy during the process of photosynthesis. Reason (R) Sugar produced from photosynthesis by plants is used by human beings as a source of energy. Sol. (a) Both A and R are true, but R is not the correct

Assertion is a strong and forceful statement or claim made in regard with a thing, element for its use and effects. Reason means a fact, event or statement that provides an explanation to the assertion. This topic is basically designed to judge the candidate’s technical knowledge and his ability to reason out correctly. In the questions, two statements are given. Out of these two statements, one is the Assertion (a) and other is the Reason (R). It is required to analyse whether the reason is an optimum and correct explanation of the assertion. Some times both assertion and reason are correctly stated facts but the reason does not correctly explain the assertion. So, different possibilities can exist between these two statements and accordingly the correct answer is marked from the given alternatives.

Directions (Ex. Nos. 10-13) Each of these questions has an Assertion (A) and a Reason (R). Give answer (a) if both ‘A’ and ‘R’ are true and ‘R’ is the correct explanation of ‘A’ (b) if both ‘A’ and ‘R’ are true but ‘R’ is not the correct explanation of ‘A’ (c) if ‘A’ is true but ‘R’ is false (d) if ‘A’ is false but ‘R’ is true

Ex. 10 Assertion (A) A body weights less when immersed in water. Reason (R) Newton’s law explains the above phenomenon. Sol. (c) A is true and R is false. A body weights less when immersed in water but this law was given by Archimedes’, not by Newton.

Ex. 11 Assertion (A) The steam engine was invented by James Watt. Reason (R) There was a problem of taking out water from flooded mines. Sol. (a) Both A and R are true and R is the correct explanation of A. The problem of pumping out water from the flooded mines required the need of a self-working engine which led James Watt to invent the same.

explanation of A.

6. Caused and Effects Cause is the logical or scientific reason of an event that has occured and Effects to this cause are the consequences of that event. In this type of questions, two statements are given and the student has to identify whether they are independent causes or effects of independents causes or a common cause etc., and accordingly have to select the answer options.

Directions (Ex. Nos. 14 and 15) In each of these questions, two Statements I and II are given. These may have a cause and effect relationship or may have independent causes or be the effects of independent causes. Give answer (a) if Statement I is the causes and Statement II is its effect (b) if Statement II is the cause and Statement I is its effect (c) if both Statements I and II are effects of independent causes (d) if both Statements I and II are effects of some common cause

Ex. 14 I. The prices of petroleum products dropped marginally last week. II. The State Government reduced the tax on petroleum products last week. Sol. (b) State Government reduced the tax on petroleum products last week, that is why the price of petroleum products dropped marginally, so Statement II is the cause and Statement I is the effect.

Ex. 15 I. Many people visited the religious place during the weekend. II. Few people visited the religious place during the week days. Sol. (c) Both statements are effects of independent causes.

https://sscstudy.com/

https://sscstudy.com/

LOGICAL AND ANALYTICAL REASONING

Practice Questions Directions (Q. Nos. 1-20) In the following questions certain statements and conclusions are given. You have to take statements as true even if they seems to be at variance from commonly known facts, then answer the following questions. 1. Statement Medals are awards. Conclusions I. All awards are not medals. II. All medals received are called awards. (a) Only Conclusion I follows (b) Only Conclusion II follows (c) Both Conclusions I and II follow (d) Neither Conclusion I nor II follows

2. Statements 1. Some fruits are vegetables. 2. All vegetables are plants. Conclusions I. Some plants are vegetables. II. Some fruits are plants. (a) Only Conclusion I follows (b) Only Conclusion II follows (c) Both I and II follow (d) Neither of them follows

3. Statements 1. All animals are dogs. 2. All dogs are birds. Conclusions I. All animals are birds. II. All birds are animals. (a) Only Conclusion I follows (b) Only Conclusion II follows (c) Both Conclusions I and II follow (d) Neither Conclusion I nor II follows

4. Statements 1. Some books are mobiles. 2. Some calculators are mobiles. Conclusions I. Some mobiles are calculators. II. Some mobiles are books. (a) Only Conclusion I follows (b) Only Conclusion I follows (c) Both Conclusions I and II follow (d) Neither Conclusion I nor II follows

5. Statement 1. No flower is a pot. 2. No pot is a garden. Conclusion I. No flower is a garden. II. All garden are pots.

(a) Only Conclusion I follows (b) Only Conclusion II follows (c) Both Conclusion I and II follow (d) Neither Conclusion I nor II follows

6. Statements 1. Some food are sweet. 2. Some food are sour. Conclusions I. All food are either sweet or sour. II. Some sweets are sour. (a) Only Conclusion I follows (b) Only Conclusion II follows (c) Both Conclusions I and II follow (d) Neither Conclusion I nor II follows

7. Statements 1. All children are students. 2. All students are players. Conclusions I. All players are students. II. All children are players. (a) Only Conclusion I follows (b) Only Conclusion II follows (c) Both Conclusion I and II follow (d) Neither Conclusion I nor II follows

8. Statements All Student are boys. No boy is dull. Conclusions I. There are no girl in the class. II. No student is dull. (a) Only Conclusion I follows (b) Only Conclusion II follows (c) Both Conclusions I and II follow (d) Neither Conclusion I nor II follows

9. Statements 1. No girl is a parrot. 2. Sowmya is girl. Conclusions I. Sowmya is not a parrot. II. All girls are not Sowmya. (a) Only Conclusion I follows (b) Only Conclusion II follows (c) Either Conclusion I or II follows (d) Niether Conclusion I nor II follows

10. Statements 1. Most players are men. 2. Some men are singers. Conclusions I. Some singers are men. II. Some players are singers.

https://sscstudy.com/

7

https://sscstudy.com/

8

CUET (UG) Section III : General Test

(a) Only Conclusion I follows (b) Only Conclusion II follows (c) Neither Conclusion I nor II follows (d) Both Conclusions I and II follow

16. Statements 1. Some yeas are decades. 2. All centuries are decades. Conclusions I. Some centuries and years. II. Some decades are years. III. No century is a year.

11. Statements 1. All cupboards are watches. 2. All watches are costly. Conclusions I. All cupboards are costly. II. Some costly things are cupboards.

(a) Conclusion I and II follow. (b) Neither Conclusion I nor II follow (c) Only Conclusion I follows (d) Only Conclusion II follows

(a) Only Conclusion I follows (b) Only Conclusion II follows (c) Both of them follow (d) Neither of them follows

17. Statements

12. Statements 1. All English movies are violent. 2. Some people like watching English movies. Conclusions I. All people watching English movies like violence. II. All people who like violence watch English movies. (a) Only I follows (c) Neither I nor II follows

(b) Only II follows (d) Both I and II follow

1. No cow is a chair. 2. All chairs are tables. Conclusions I. Some tables are chairs II. Some tables are cows. III. Some chairs are cows. IV. No table is a cow. (a) Either II or III follow (c) Only I follows

(b) Either II or IV and I follow (d) All conclusions follow

18. Statements

1. Some human creature are angles. 2. All angles are doctors. Conclusions I. Some human creatures are doctors. II. Some doctors are human creatures.

1. Some buds are flowers. 2. All flowers are trees. 3. All trees are leaves. Conclusions I. Some leaves are buds. II. All flowers are leaves.

(a) Only I follows (b) Only II follows (c) Both I and II follow (d) Neither I nor II follows

(a) Only Conclusion I follows (b) Only Conclusion II follows (c) Both I and II follow (d) Neither I nor II follows

13. Statements

19. Below are given statements. You have to take the

14. Statements

given statements to be true even if they seem to be at variance with the commonly known facts and then decide which of the given conclusions logically follows from the given statements. Statements 1. All rats are hills. 2. All hills are rivers.

1. All teachers are experienced. 2. Some teachers are spinsters. Conclusions I. Some experienced are spinsters. II. Some spinsters are experienced. (a) Only Conclusions II follows (b) Either Conclusion I or II follows (c) Both Conclusions I and II follow (d) Only Conclusion I follows

(a) Some rivers are rats (c) All hills are rats

20. Read the given statements carefully and answer the

15. Statements

question. Statements 1. All teachers get angry. 2. Some teachers are sad. 3. Sad people may cry. Which of the following conclusion is true?

1. Blue is black and some black is red. 2. All red is green but not yellow. Conclusions I. Some Blue is Green. II. No Black is Yellow. III. Some Black is not Yellow (a) Only I and II follow (c) Only I and III follow

(b) No river is a hill (d) No river is a rat

(b) Only II and IV follow (d) Only I, II and IV follow

(a) All sad people cry (b) Some teachers may cry (c) All angry people are teachers (d) All sad people get angry

https://sscstudy.com/

https://sscstudy.com/

LOGICAL AND ANALYTICAL REASONING

Directions (Q. Nos. 21-28) In the following questions, each question has a statement followed by two conclusions. Taking the statement to be true, decide which of the given conclusions definitely follows from the given statement. Indicate your answer as Give answer (a) if only I follows (c) if neither I nor II follows

(b) if only II follows (d) if both I and II follow

21. Statement Good health is dependent on right eating habits. Most of the people do not follow any rule regarding eating. Conclusions I. Most of the people have poor health. II. People are ignorant of proper eating habits.

22. Statement Black cloud follows thunder. Rains follow thunder. Conclusions I. Thunder is the cause of rain. II. Black cloud is the cause of thunder.

23. Statement Workers feel highly motivated when they get sense of involvement by participating in the management of companies. Conclusions I. Workers should be motivated to produce more. II. Workers should be allowed to participate in the management of companies.

24. Statement Industrial revolution which first of all started in Europe has brought about modern age. Conclusions I. Disparity between rich and poor results in revolution. II. Revolution overhauls society.

25. Statement America’s defence secretary reiterated that they would continue to supply arms to Pakistan. Conclusions I. Pakistan is incapable of manufacturing arms. II. It would ensure peace in the region. 26. Statement Success cannot be achieved without hard work. Conclusions I. Every hardworking person is successful. II. Every successful person is hardworking. 27. Statement Unlike Aryabhata, moon is a natural satellite of the Earth. Conclusions I. Aryabhatta is not a satellite. II. Moon is star and Aryabhatta is a satellite. Find which of the given conclusions logically follows from the given statement. 28. Statement Customer service cannot be enforced. It has to come from within. Conclusions I. Customer service should be voluntary. II. Employees do not serve customers.

9

29. A statement follows by some conclusions are given below. Statement Based on his performance, Rajesh got a poor rating in his office. Conclusions I. Rajesh did not perform well. II. The rating given to Rajesh was not up to the mark. Find which of the given conclusions logically follow from the given statements. (a) Only Conclusion I follows (b) Only Conclusion II follows (c) Both I and II follow (d) Neither I nor II follows

30. Statements followed by some conclusions are given below. Statements 1. Depleting natural resources is a major concern in our country. 2. Unless we go for renewable energy sources in a big way increase in population being a heavy burden on the energy resources, will reduce the quality of life. Conclusions I. Depleting natural impacts quality of life. II. Use of renewable energy sources is a solution to counter increase in population. Find which of the gives conclusions logically follows from the given statements. (a) Only Conclusion I follows (b) Only Conclusion II follows (c) Both I and II follow (d) Neither I and II follows

Directions (Q. Nos. 31-35) In making decisions about important questions, it is desirable to distinguish between a ‘strong’ argument and a ‘weak’ argument. A ‘strong’ argument must be both important and directly related to the question. A ‘weak’ argument may not be directly related to the question and may be of minor importance or may be related to the trivial aspect of the question. Each question below is followed by two arguments numbered I and II. You have to decide which of the arguments is ‘strong’ and which is ‘weak’. Give answer (a) if only Argument I is strong (b) if only Argument II is strong (c) if neither I nor II is strong (d) if both I and II are strong

31. Statement Should there be a total ban on use of plastic bags? Arguments I. No, instead the thickness of plastic bags, which can be used without much damage to the environment, should be specified. II. Yes, use of plastic bags causes various problems like water pollution and water logging and hence, it is necessary to ban it.

https://sscstudy.com/

https://sscstudy.com/

10

CUET (UG) Section III : General Test

32. Statement Should coal engines be replaced by electric engines in trains? Arguments I. Yes, coal engines cause a lot of pollution. II. No, India does not produce enough electricity to fulfil even the domestic needs.

33. Statement should English be the medium of instruction for higher education in India? Arguments I. Yes, even in advanced countries like UK and USA, the medium of instruction is English for higher education. II. Yes, English is much widely spoken language in the world today.

34. Statement Should there be only a uniform rate of income tax irrespective of the level of income? Arguments I. Yes, this will substantially reduce the work of the officials of the income tax department. II. No, this will reduce government tax collection to a large extent.

35. Statement Should all the factories in the cities be shifted to the outskirts, far away from the main city? Arguments I. Yes, this is an essential Step for controlling pollution in the city. II. No, such a Step will lead to lot of inconvenience to the employees of the factories and their families as well.

Directions (Q. Nos. 36-40) In each question below is given a statement followed by two Assumptions I and II. An assumption is something supposed or taken for granted. You have to consider the statement and the following assumptions and decide which of the assumptions is implicit in the statement. Give answer (a) if Assumption I is implicit (b) if Assumption II is implicit (c) if neither I nor II is implicit (d) if both I and II are implicit

38. Statement In Mumbai, railway trains are indispensable for people in the suburbs to reach their places of work on time. Assumptions I Railway trains are the only mode of transport available in the suburbs of Mumbai. II Only railway trains run punctually. 39. Statement The mangoes are too cheap to be good. Assumptions I When the mango crop is abundant, the prices go down. II The lower the selling price, the inferior is the quality of the commodity. 40. Statement Of all the newspapers published in Delhi, ‘The Time and Space’ has the largest number of readers. Assumptions I Volume of readership of all the newspapers in Delhi is known. II No newspaper in Delhi other than ‘The Time and Space’ has large readership.

Directions (Q. Nos. 41-44) Each of these questions has an Assertion (A) and a Reason (R). Give answer (a) if both ‘A’ and ‘R’ are true and ‘R’ is the correct explanation of ‘A’ (b) if both ‘A’ and ‘R’ are true but ‘R’ is not the correct explanation of ‘A’ (c) if ‘A’ is true but ‘R’ is false (d) if ‘A’ is false but ‘R’ is true

41. Assertion (A) India’s ‘Republic Day’ falls on 26th

42. 43.

44.

36. Statement Apart from the entertainment value of television, its educational value cannot be ignored. Assumptions I People take the television to be means of entertainment only. II The educational value of television is not realised properly.

37. Statement Ten candidates who were on the waiting list could finally be admitted to the course. Assumptions I Wait-listed candidates get admission with difficulty. II A large number of candidates were on the waiting list.

45.

January. Reason (R) Constitution of India, declaring India as a ‘Republic’, came into force on 26th January 1950. Assertion (A) India is a democracy. Reason (R) India is a developing country. Assertion (A) In India, the judiciary is independent of the executive. Reason (R) Judiciary favours the government and helps in the implementation of its plans. Assertion (A) A salt water fish drinks sea water whereas a fresh water fish never drinks water. Reason (R) A salt water fish is hypertonic to its environment while a fresh water fish is not hypertonic to its environment. An Assertion (A) and Reason (R) are given below. Assertion (A) Leakages in household gas cylinders can be detected. Reason (R) LPG has a strong smell. Choose the correct option. (a) Both A and R are true and R is the correct explanation of A (b) Both A and R are true and R is not the correct explanation of A (c) Both A and R are false (d) A is true but R is false

https://sscstudy.com/

https://sscstudy.com/

LOGICAL AND ANALYTICAL REASONING

46. An Assertion (A) and Reason (R) are given below. Assertion (A) Beri - Beri is a viral infection. Reason (R) Vitamin deficiency causes diseases. Choose the correct option. (a) (b) (c) (d)

A is false but R is true A is true but R is false Both A and R are false Both A and R are true and R is the correct explanation of A

47. An Assertion (A) and a Reason (R) are given below. Assertion (A) We prefer to wear white clothes in winter. Reason (R) White clothes are good reflectors of heat. Choose the correct option. (a) A is true but R is false (b) A is false but R is true (c) Both A and R are true and R is the correct explanation of (A) (d) Both A and R are true and R is not the correct explanation of A

48. An Assertion (A) and a Reason (R) are given below. Assertion (A) Rainwater harvesting reduces soil crosion. Reason (R) Rainwater harvesting is not important for conservation of water. Choose the correct option. (a) Both A and R are true and R is the correct explanation of A (b) Both A and R are true, but R is not the correct explanation of A (c) A is true, but R is false (d) A is false, but R is true

49. An Assertion (A) and a Reason (R) are given below. Assertion (A) Penguins are birds, found in the hottest regions of the earth. Reason (R) Birds in hot regions do not have wings. Choose the correct option. (a) Both A and R are true and R is the correct explanation of A (b) Both A and R are ture, but r is not the correct explanation of A (c) A is true, but R is false (d) Both A and R are false

50. An Assertion (A) and a Reason (R) are given below. Assertion (A) Perennial rivers mostly originate from the Himalayas. Reason (R) The fountains in the Himalayas feed the rivers. Choose the correct option. (a) Both A and R are true and R is the correct explanation of A (b) Both A and R are true, but R is not the correct explanation of A (c) A is true, but R is false (d) A is false, but R is true

11

Directions (Q. Nos. 51-60) Below in each question are given two Statements A and B. These statements may be either independent causes or may be effects of independent causes or of a common cause. One of these statements may be the effect of the other statement. Read both the statements and decide which of the following answer choices correctly depicts the relationship between these two statements. Give answer (a) if Statement A is the cause and Statement B is its effect (b) if Statement B is the cause and Statement A is its effect (c) if both statements are effects of independent causes (d) if both statements are effects of some common cause

51. A. Domestic price of petrol has gone down. 52.

53. 54.

55.

56.

B. International price of crude oil has decreased. A. Ahmed is a healthy boy. B. His mother is very particular about the food he eats. A. Ravi died while on the way to the hospital. B. A car dashed into the motorcycle Ravi was driving. A. The average day temperature of the city has increased by about 2 degrees in the current year over the average of past ten years. B. More people living in rural areas of the state have started migrating to the urban areas in comparison with the earlier year. A. Most of the shopkeepers in the locality closed their shops for the second continuous day. B. Two groups of people living in the locality have been fighting with each other with bricks and stones, forcing people to stay indoors. A. The Government has decided to increase the prices of LPG cylinders with immediate effect. B. The Government has decided to increase the prices of Kerosine with immediate effect.

57. A. Party ‘X’ won clear majority in the recently held B.

58. A. B.

59. A.

B.

60. A.

state assembly elections. Of late, there was unrest in public and also among the members of the ruling party of the state. Staff members of the university decided to go on strike in protest during the examinations. The university administration made all the arrangements for smooth conduct of examination with the help of outsiders. In the university examination, overall performance of students from college ‘X’ was better than that of students from college ‘Y’. Majority of the students depend upon coaching classes for university examinations. The Government of state ‘X’ decided to ban working of women in night shifts and also in late evening hours.

B. The percentage of working women has a significant rise in the last one decade.

https://sscstudy.com/

https://sscstudy.com/

12

CUET (UG) Section III : General Test

ANSWERS 1. 11. 21. 31. 41. 51.

2. 12. 22. 32. 42. 52.

(b) (c) (a) (d) (a) (b)

3. 13. 23. 33. 43. 53.

(c) (a) (b) (d) (b) (b)

(a) (c) (d) (b) (c) (b)

4. 14. 24. 34. 44. 54.

(c) (c) (b) (b) (a) (c)

5. 15. 25. 35. 45. 55.

(d) (c) (c) (a) (a) (b)

6. 16. 26. 36. 46. 56.

(d) (d) (b) (d) (a) (d)

7. 17. 27. 37. 47. 57.

(b) (b) (c) (a) (b) (d)

8. 18. 28. 38. 48. 58.

(c) (c) (a) (c) (c) (a)

9. 19. 29. 39. 49. 59.

10. 20. 30. 40. 50. 60.

(a) (a) (c) (b) (d) (c)

(a) (b) (c) (a) (c) (c)

Hints & Solutions 1. (b)

or

medals awards

Conclusions I.ü II. ✗ From the above figure, only I follows.

7. (b) medals

awards

Students

13. (c)

Children

Conclusions I. û II. ü So, only Conclusion II follows.

Human Creature

Players

2. (c) Fruits

Doctors

Conclusions I. û II. ü From the above figure, Only Conclusion II follows.

Vegetables

Conclusions I. ü II. ü From the above figure, both I and II follow.

8. (c)

14. (c)

Plants

Teacher

Spinsters

Boys

Birds

3. (a)

Dull

Students

Conclusions I. ü II. ü Both the conclusions follow.

Angles

Experienced

Conclusions I. ü II. ü From the above figure, Both Conclusions I and II follow.

Animals

9. (a)

Dog

Girls S

Conclusions I. ü II. ü From the above figure. Both Conclusion I and II follow.

Parrot

15. (c) Yellow

From the above figure. Only Conclusion I follows.

4. (c)

Books

From above figure, only Conclusion I follows.

Moblies

Calculators

Conclusions I. ü II.ü From the above figure. Both Conclusions I and II follow.

5. (d)

Flower

Pot

Conclusions

I. ✗

I. ✗

Blue Black

Singers

11. (c)

Pot

Conclusions I.ü II. ✗ III. ü From the above figure. Both Conclusions I and III follow. Y

Costly

Conclusions I. ü II. ü Form the above figure, Both of the Conclusion follow.

II. ✗

From the above figure. Neither Conclusion I nor II follows.

Sweet

or

Liking

Violent

https://sscstudy.com/

Y

C D

Conclusions I. ✗ II. ü III. ✗ From above it is clear that only Conclusion II follows.

17. (b) Cow

12. (a) English movie

C D

Watches

II. ✗

Food

Red

16. (d)

Garden

Neither Conclusion I nor II follow.

Conclusions

Men

Conclusions I. ü II. ✗ Form above figure. Only Conclusion I follows.

Form the above figure.

Sour

Players

Cupboard

Flower

6. (d)

10. (a)

Garden

or

Green

S = Sowmya

Chair Table

Conclusions I. ü II. ü III. ✗ IV. ü From above figure either II or IV and I follow.

https://sscstudy.com/

LOGICAL AND ANALYTICAL REASONING

or

Cow

Also, it is clearly given in the statement that moon is a natural satellite. Hence, neither I nor II follows.

Chair

28. (a) From the statement it is clear that, customer service should be voluntrary. Hence, only Conclusion I follows.

Table

18. (c)

Buds

29. (c) Rajesh got a poor rating because of his performance so it is clear that Rajesh did not perform well. It is also clear from the statement that the rating given to Rajesh was not up to the mark. Hence, both Conclusions I and II follow.

Flowers Tree Leaves

30. (c) It is clear that, depleting resources impacts quality of life and use of renewable energy sources is a solution to counter increase in population.

Conclusions I. ü II. ü Both the conclusions follow.

19. (a)

31. (d) Argument I is strong as it takes a wise, reconciliatory approach to the problem. Argument II is also strong as water pollution etc., may severally harm mankind. Rats

32. (d) Both the arguments are logical and

Hills

directly related to the statement and hence, are strong arguments.

Leaves

33. (b) Anything successful in other countries may not succeed in India. However, since English is much widely spoken language in the world today and hence, should be adopted, is a strong idea. Hence, Argument II is the strong argument.

Clearly, Some rivers are rats.

20. (b) Teachers

Sad

Angry

Cry

Here, sad people many cry. So, all sad people cry does not definitely follow. Clearly, Some teachers may cry.

21. (a) Conclusion I definitely follows. Conclusion II is not confirmed by the given statement.

22. (b) Only Conclusion II follows. Conclusion I is wrong as rains are the cause of thunder.

23. (d) Both Conclusions I and II follow from the statement as involvement of workers boosts production and benefits the companies.

24. (b) Only Conclusion II follows. Conclusion I is not related to the statement.

25. (c) Neither Conclusion I nor conclusion II follows. Both the conclusions are unrelated to the statement.

26. (b) Success cannot be achieved without hardwork it means that every successful person is hardworking. Hence, only Conclusion II follows.

27. (c) It is clear from the statement that Aryabhata is a satellite but it is not a natural satellite.

13

43. (c) A is true but R is false. As in India, the judiciary is completely independent of the executive the government has no interference in the judicial affairs. 44. (a) Assertion is true, and this happens because a salt water fish is hypertonic to its environment while a fresh water fish is not hypertonic to its environment. 45. (a) LPG contains ethyl mercaptan which has strong smell. Because of this, smell leakages in household gas cylindren can be detected. 46. (a) Beri-Beri is a disease in which the body doesn’t have enough vitamin B1 (thiamin). Hence A is false but R is true. 47. (b) We prefer to wear dark clothes in winter as they absorb the heat and keep the body warm. However, white clothes are good reflectors of heat and are worn in summer. 48. (c) A is true, but R is false One of the main reason of rain water harvesting is conserve water for future. 49. (d) Both A and R are false. 50. (c) A is true, but R is false Perennial rivers usually originate from mountainous snowy regions or glaciers. 51. (b) The effect of decrement in International price of crude oil is the domestic price of petrol also gone down.

34. (b) Argument I is weak because reduction of work load of IT officials is not too desirable a motive. Argument II is strong as reduced tax collection will have a bad impact on state activities.

52. (b) Since, Ahmed’s mother take care of what he eats, Ahmed has a good health.

35. (a) Argument I is strong because pollution

54. (c) The two statements seem to have contrary causes.

control is highly desirable. Argument II is weak.

36. (d) Both Assumptions I and II are implicit. 37. (a) Assumption II is not implicit in the statement. Only assumption I is implicit. 38. (c) Neither Assumption I nor Assumption II is implicit in the statement because of the word ‘only’. 39. (b) Assumption I is not implicit in the statement, because it states the principle of supply and prices. Only Assumption II is implicit which states the principle of price quality parity. 40. (a) Assumption I is implicit in the statement because the largest readership may be established only if the volume of readership of all the newspapers is known. 41. (a) Both A and R are true and R is the correct explanation of A. 42. (b) Both A and R are true but R is not the correct explanation of A.

https://sscstudy.com/

53. (b) Ravi died because of the accident.

55. (b) The fighting has led to the closure of the shops. 56. (d) It seems the price of Petroleum has increased in general. Alternatively, subsidies may have been reduced, again a cause common to both the statements. 57. (d) It can be safely assumed that party ‘X’ was earlier in the opposition and has benefitted from anti-incumbancy. But A and B are consequences of a common cause, i.e. bad governance by the ruling party. 58. (a) Since, the staff members have gone on strike, the help of outsiders has been sought. 59. (c) A seems to have happened as X is a better college. B seems to be the result of the falling standard of teaching in colleges. 60. (c) A might have happened because harassment of woman is on the rise while B seems to be the result of a change in gender role perception.

https://sscstudy.com/

PRACTICE SET 01

3

CUET (UG) Section III : General Test

Practice Paper 01 Instructions 60 questions to be attempted out of 75. 1. Which state became the first state to launch India’s first Night Navigation mobile application for ferry services? (a) Haryana (c) Rajasthan

(b) Bihar (d) Assam

2. National Institute of Urban Affairs (NIUA) signed MoU with which institution to collaborate on ‘Sustainable Cities India programme’? (a) NITI Aayog (b) World Economic Forum (c) World Bank (d) UNDP

3. Which financial company has partnered with International Air Transport Association (IATA) to launch ‘IATA Pay’ payment platform for airline industry in India? (a) DBS Bank (b) State Bank of India (c) Standard Chartered (d) HSBC

4. India’s first Biosafety level-3 containment mobile laboratory has been inaugurated in (a) Jaipur (Rajasthan) (b) Nashik (Maharashtra) (c) Chennai (Tamil Nadu) (d) Gurugram (Haryana)

5. Which country has topped the medal table of Winter Olympics Games 2022 held in Beijing? (a) India (c) Germany

(b) Norway (d) China

6. First Governor of Reserve Bank of India is (a) CD Deshmukh (b) Hilton Young (c) Edwin Lutyens (d) Osborne Smith

Time : 60 Min

7. Who was the first woman to receive Jnanpith Award? (a) Ashapurna Devi (b) Mahaswetah Devi (c) Amrita Pritam (d) Mahadevi Verma

8. The longest river bridge of India is (a) Dibang River bridge (Roing-Dambuk) (b) Mahatma Gandhi Setu (Patna-Hajipur) (c) Bandra-Worli Sea Link (Mumbai) (d) Bhupen Hazarika Setu (Tinsukia)

9. Which is the largest river Island of the world located in India? (a) Dibru-saikhua (c) Umananda

(b) Majuli (d) None of these

10. Which among the following is most populous city of India? (a) Mumbai (c) Kolkata

(b) New Delhi (d) Bengaluru

11. The first woman to get the Bharat Ratna Award is (a) Mother Teresa (b) Indira Gandhi (c) Lata Mangeshkar (d) Sarojini Naidu

12. Which boxer is nicknamed ‘The Real Deal’? (a) Mike Tyson (b) Mohammed Ali (c) Evander Holyfield (d) Joe Louis

13. Jnanpith Award is conferred in the field of (a) Literature (c) Drama

(b) History (d) Dance

14. Who received both Bharat Ratna and Nishan-e-Pakistan? (a) Lal Bahadur Shastri (b) Ayub Khan (c) Morarji Desai (d) Maulana Abdul Kalam Azad

https://sscstudy.com/

15. Which Indian won the 2014 Nobel Prize for Peace? (a) Kailash Satyarthi (b) Malala Yousafzai (c) Sanjiv Chaturvedi (d) Anshu Gupta

16. Army Training Command is located in (a) Chandigarh (c) Jaipur

(b) Shimla (d) Pune

17. Admiral is the Chief of which force of Indian Defence? (a) Indian Navy (b) Indian Army (c) Indian Air Force (d) Coast Guard

18. From where was Mangalyaan launched? (a) Chennai (b) Sriharikota (c) Trombay (d) Gopalpur, on sea

19. Who developed Arjun Tank? (a) DRDO (b) Ministry of Defence (c) Russian Army (d) Indian Army

20. India procured T90 tank from which country? (a) China (c) France

(b) Russia (d) USA

21. Which is the first missile which has been developed in India? (a) Akash (c) Agni

(b) Prithvi (d) Trishul

22. Air Force Academy is located in (a) Hyderabad (c) Kakinada

(b) Coimbatore (d) Mumbai

23. Indian and Russian Scientists successfully test fired a Supersonic cruise missile named (a) GSAT-1 (c) Trishul

(b) INSAT-3B (d) Brahmos

https://sscstudy.com/

4

CUET (UG) Section III : General Test

24. Nuclear powered attack submarine of India is (a) INS Kitten (c) INS Kolkata

(b) INS Chennai (d) INS Arihant

25. Which is first Indian ship to have a super structure of carbon fibre composite material? (a) INS Sindurakshak (b) INS Kiltan (c) INS Kalvari (d) INS Chakra

26. Select the option that is related to the third number in the same way as the second number is related to the first number and the sixth number is related to the fifth number. 72 : 108 : : 84 : ? : : 102 : 153 (a) 144

(b) 126

(c) 117

(d) 135

27. Identify the diagram that best represents the relationship among the given classes. Planet, Earth, Venus

(a)

(b)

(c)

(d)

28. In the given Venn diagram, the triangle represents students playing table tennis,the rectangle represents students playing badminton, the circle represents female students and the pentagon represents students playing football. The numbers given in the diagram represent the number of persons in that particular category.

22 24

7 12 3 9

6

24

11

26

Who is the father of K? (a) O

(b) L

(c) M

(d) N

30. Select the letter-cluster that can replace the question mark (?) in the following series. aYd, fTi, kOn, pJs, ? (a) VeX

(b) uEw

(c) uFw

(d) uEx

31. Four letter-clusters have been given, out of which three are alike in same manner, while one is different. Select the odd letter-cluster. (a) TVW (c) LNP

: : : :

Happiness Peace Hear Rage

(b) 152927949 (d) 152927947

34. Select the number that can replace the question mark (?) in the following series. 40, 37, 43, 34, 46, ? (a) 31 (c) 51

(b) 41 (d) 61

35. Which of the option figures is the exact mirror image of the given figure when the mirror is held at the right side? qv56jk89lm 6 9 (a) m l 98 k j 6 5 v q (b) m l 8 k j 5v q q v56 j k89 lm ( c) (d) m 9 8 k j 65 q l

(a) 22

(b) 18

(c) 7

(d) 9

29. There is a family of five members K, L, M, N and O. Among them, there is one married couple. O is unmarried and is the brother of K. N is the sister of O. M is the only married female and the mother of N. L and O are the only males in the group.

36. Select the option that is related to the third word in the same way as the second word is related to the first word. Ministers : Council : : Sailors : ? (a) Sea (c) Captain

(b) Ship (d) Crew

37. The two given expression on both the side of the ‘=’ sign will have the same value if two numbers from

https://sscstudy.com/

38. Select the set of letters that when sequentially placed in the blanks of the given letters series will complete the series. f_hg_fh_gf_hg_fh_g (b) g, h, f, g, h, f (d) h, f, g, h, f, g

39. In a certain code language, U is written as C, K is written as H. L is written as U. N is written as E. S is written as L. E is written as K and C is written as N. How will, ‘KNUCKLES’ be written as in that language? (b) CHUECKN (d) HECNHULK

40. Two positions of the same dice are shown. Select the number that will be on the face opposite to the one showing 6. 6

3 1

5

(a) 4 (c) 1

3

5

(b) 5 (d) 3

41. The sequence of folding a piece of paper and the manner in which the folded paper has been cut is shown in the following figures. How would this paper look when unfolded? Question Figures

Answer Figures

v

How many female students play both table tennis and badminton?

(b) 5, 7 (d) 24, 36

(a) KECNKUHL (c) HECNHUKL

33. In each letter of the English alphabet is assigned an odd numerical value in increasing order, such as A=1, B =3 and so on, then what will be the code of HONEY? (a) 132725747 (c) 132725745

(a) 6, 3 (c) 4, 7

(a) f, g, h, f, g, h (c) g, f, g, f, h, f

(b) FHJ (D) DFH

32. Select the option in which the words share the same relationship as that shared by the given pair of words. Blunder : Error (a) Euphoria (b) War (c) Speak (d) Anger

either side or both side are interchanged. Select the correct numbers to be interchanged from the given options. 3 + 5 × 4 − 24 ÷ 3 = 7 × 4 − 3 + 36 ÷ 6

(a)

(b)

(c)

(d)

42. Select the option that is related to the third word in the same way as the second is related to the first word. Medicine : Disease : : Food : ? (a) Energy (c) Thirst

(b) Nutrition (d) Hunger

https://sscstudy.com/

5

PRACTICE SET 01

43. Study the given pattern carefully and select the number that can replace the question mark (?) in it. 6 40 8

21 500 25

(a) 98 (c) 91

52. If the diagonal of a square is 12 cm, then what is the area (in cm 2 ) of the square? (a) 72 (c) 36

14 ? 7 (b) 84 (d) 78

44. How many rectangles are there in the given figures?

(b) 72 2 (d) 36 2

53. A can do a piece of work in 12 days and B in 15 days. With help of C, they finish the work in 4 days. In how many days C alone can do the same work? (a) 10

(b) 12

(c) 8

(d) 15

54. A is twice as efficient as B and together they finish a piece of work in 18 days. How many days will A take to complete the work alone? (a) 27 (c) 36 (a) 34 (c) 30

(b) 32 (d) 35

45. Select the figure that can replace the question mark (?) in the following series. Question Figures

?

(a)

(b)

(c)

(d) 3

46. What is the value of 22 ? (a) 128 (c) 256

(b) 64 (d) 512

47. What is the sum of all natural numbers between 1 and 150 which are multiples of 3? (a) 3675 (c) 3735

(b) 4235 (d) 4415

48. What is the sum of first 25 odd natural number? (a) 475 (c) 600

(b) 575 (d) 625

49. What is the HCF of 12, 15 and 48? (a) 5 (c) 3

(b) 1 (d) 6

50. How many numbers less than 100 are multiples of both 3 and 4? (a) 7 (c) 6

(b) 8 (d) 12

51. Which of the following number is not divisible by 4? (a) 113424 (c) 314250

55. If chairs are bought at ` 500 each and sold at ` 700 each, then what will be the profit percentage? (a) 10% (c) 30%

(b) 213552 (d) 52644

(b) 20% (d) 40%

56. An article is sold at 10% profit instead of 10% loss, the man gains ` 20 more. What is the cost price (in `) of that article? (a) 50 (c) 200

Answer Figures

(b) 24 (d) 42

(b) 100 (d) 400

57. If P got 20% marks less than Q, then the marks of Q is how much per cent more than P? (a) 20 (c) 25

(b) 10 (d) 15

58. If the price of a commodity is decreased by 20% and its consumption is increased by 40%, then what will be the increase or decrease in the expenditure of the commodity? (a) 12% decrease (c) 20% increase

(b) 12% increase (d) 20% decrease

59. Speeds of a boat along the current and against the current are 16 km/h and 10 km/h respectively. What is the speed of boat (in km/h) in still water? (a) 4 (c) 13

(b) 12 (d) 14

60. A bus travels some distance at a speed of 12 km/h and returns at a speed of 8 km/h. If the total time taken by the bus is 20 h, then what is the distance (in km)? (a) 80 (c) 96

(b) 84 (d) 92

https://sscstudy.com/

61. A sum was invested on simple interest at a certain rate for 3 yr. If the interest rate is increased by 5%, then the interest increases by ` 225. What is the sum (in `) invested? (a) 1000 (c) 1750

(b) 1500 (d) 2000

62. If P : Q = 2 : 3 and P : R = 4 : 5, then what is (P + Q) : (Q + R)? (a) 20 : 27 (c) 10 : 11

(b) 20 : 17 (d) 20 : 29

x 5 = , then what is the ratio of y 4 (5x + 6 y) and (5x − 2 y)?

63. If

(a) 49 : 17 (c) 37 : 17

(b) 49 : 19 (d) 37 : 19

64. The mean of 17 numbers is 12. If two numbers 9 and 15 are removed, then what will be the mean of remaining number? (a) 11 (c) 13

(b) 12 (d) 14

65. What is the HCF of 8, 12 and 15? (a) 1 (c) 3

(b) 2 (d) 120

66. If a number is multiplied by three-fourth of itself, then the value thus obtained is 10800. What is that number? (a) 210 (c) 120

(b)180 (d) 160

67. The number of boys in a class is three times the number of girls. Which of the following numbers cannot represent the total number of children in the class? (a) 44 (c) 40

(b) 48 (d) 42

68. Neela is now three times as old as her daughter Leela. Ten years back, Neela was five times as old as Leela. The age of Leela is (a) 15 yr (c) 30 yr

(b) 25 yr (d) 20 yr

69. A shepherd had 17 sheeps. All but nine died. How many was he left with ? (a) 17

(b) 9

(c) Nil

(d) 8

70. In a zoo, there are Rabbits and Pigeons. If heads are counted, there are 200 and if legs are counted, there are 580. How many pigeons are there? (a) 90

(b) 100

(c) 110

(d) 120

https://sscstudy.com/

6

CUET (UG) Section III : General Test

Directions (Q. Nos. 71 and 72) In each of the following questions two statements alongwith a set of conclusions is given. You have to take the statements to be true even if they seem to be at variance from the commonly known facts and decide which of the following conclusions logically follows from the given statements. 71. Statements Some stars are planet. All planets are galaxies. Conclusions I. Some stars are galaxies. II. Some galaxies are planet.

(a) Only Conclusion I follows (b) Only Conclusion II follows (c) Only Conclusion III follows (d) All Conclusions follow

II. Discard old ideas because they are old.

Directions (Q. Nos. 73 and 74) In the following questions a statement is followed by two conclusions I and II. Taking the statement to be true decide which of the given conclustions difinitely follows from the given statement. 73. Statement In a one day cricket match, the total runs made by a team were 200. Out of these, 160 runs were made by spinners. Conclusions I. 80% of the team consists of spinners. II. The opening batsmen were spinners.

(a) Only Conclusion I follows (b) Only Conclusion II follows (c) Neither Conclusion I nor II follows (d) Both Conclusions I and II follow

72. Statements All teachers are students. No student is girl.

(a) Only Conclusion I follows (b) Only Conclusion II follows (c) Neither I nor II follows (d) Both I and II follow

Conclusions I. All girls are students. II. Some girls are students. III. Some students are teachers.

(a) Only Conclusion I follows (b) Only Conclusion II follows (c) Either I or II follows (d) Both I and II follow

Direction (Q. No. 75) In given question below is given a statement followed by two Assumptions I and II. You have to consider the statement and the following assumptions and decide which of the assumptions is implicit in the statement. Give Answer (a) If only I is implicit (b) If neither I nor II is implicit (c) If only II is implicit (d) If both I and II are implicit

75. Statement The Prime Minister is expected to announce an expansion in his Ministry shortly. A newspaper report. Assumptions I. The newspaper has quoted authentic sources. II. The newspaper has reliable sources for the news.

74. Statement The old order changed yielding place to new. Conclusions I. Change is the law of nature.

ANSWERS 1. 11. 21. 31. 41. 51. 61.

(d) (b) (b) (a) (b) (c) (b)

71.

(d)

2. 12. 22. 32. 42. 52. 62. 72.

(b) (c) (a) (a) (d) (a) (c) (c)

3. 13. 23. 33. 43. 53. 63. 73.

(c) (a) (d) (b) (c) (a) (a) (c)

4. 14. 24. 34. 44. 54. 64. 74.

(b) (c) (d) (a) (d) (a) (b) (a)

5. 15. 25. 35. 45. 55. 65. 75.

(b) (a) (b) (d) (a) (d) (a)

6. 16. 26. 36. 46. 56. 66.

(d) (b) (b) (d) (c) (b) (c)

(c)

https://sscstudy.com/

7. 17. 27. 37. 47. 57. 67.

(a) (a) (d) (b) (a) (c) (d)

8. 18. 28. 38. 48. 58. 68.

(d) (b) (c) (b) (d) (b) (d)

9. 19. 29. 39. 49. 59. 69.

(b) (a) (b) (c) (c) (c) (b)

10. 20. 30. 40. 50. 60. 70.

(a) (b) (d) (c) (b) (c) (c)

https://sscstudy.com/

PRACTICE SET 02

7

CUET (UG) Section III : General Test

Practice Paper 02 Instructions 60 questions to be attempted out of 75. 1. India has launched which operation to bring back its nationals from Ukraine? (a) Operation Yamuna (b) Operation Saraswati (c) Operation Ganga (d) Operation Bharat

2. India, along with which country, agreed on ‘Roadmap on Blue Economy and Ocean Governance’? (a) The USA (b) The UAE (c) France (d) Japan

3. Which bank has collaborated with NPCI to launch exclusive MSME RuPay Credit Card for the Micro, Small and Medium Enterprises (MSMEs) industry? (a) Punjab National Bank (b) Bank of Baroda (c) State Bank of India (d) Union Bank of India

4. With which country’s Air Force, India has organised bilateral air exercise titled ‘Eastern Bridge-VI’ at Jodhpur (Rajasthan)? (a) Qatar (c) Kuwait

(b) Oman (d) Saudi Arabia

5. Which sports federation has suspended Vladimir Putin as Honorary President? (a) International Judo Federation (b) International Volleyball Federation (c) International Football Federation (d) International Chess Federation

6. Which programme is launched for the development of a comprehensive range of missiles in India? (a) DRDO (c) PRAKRAM

(b) IGMDP (d) IRSP

Time : 60 Min

7. Which agency in India is responsible for development of technology for All Military Wings? (a) IGMPP (b) Ministry of Defence (c) Home Ministry (d) DRDO

8. Which Fighter Aircraft is bought from France? (a) Dhruv (c) Mig-29

(b) Rafale (d) Sukhoi-30

9. Sukhoi-30 is procured from which country? (a) USA (c) France

(b) Russia (d) Germany

10. Which helicopter is developed by Hindustan Aeronautics Limited? (a) Dhruv (c) Nishant

(b) Tejas (d) Ajaya

11. The Headquarter of International Atomic Energy Agency is in (a) Geneva (c) Vienna

(b) Paris (d) Washington

12. When did India join the United Nations? (a) 1945 (c) 1950

(b) 1947 (d) 1954

13. The International Court of Justice sits in (a) Geneva (c) Vienna

(b) The Hague (d) Rome

14. The Non-Permanent members of the Security Council are elected for (a) 1 year (c) 3 years

(b) 2 years (d) 6 months

15. Earth Summit was sponsored by (a) UNESCO (c) WHO

(b) UNCED (d) UNICEF

https://sscstudy.com/

16. ‘Queensberry rules’ is the code followed in which sport? (a) Tennis (c) Boxing

(b) Cricket (d) Equestrian

17. With which sport is the term ‘Tee’ connected? (a) Hockey (c) Golf

(b) Polo (d) Badminton

18. Which two teams played the first official international cricket match? (a) England and Australia (b) England and West Indies (c) USA and Canada (d) Australia and India

19. The famous tennis player Steffi Graf belongs to which among the following countries? (a) USA (c) Germany

(b) England (d) Switzerland

20. How high is the badminton net at the center? (a) 5 feet (c) 5.5 feet

(b) 5.1 feet (d) 4.8 feet

21. Four pairs of words are given. Find the odd one out. (a) (b) (c) (d)

5th June : World Environment Day 22nd April : Earth Day 22nd March : World Water Day 22nd May : World Sparrow Day

22. With which sport is Karnam Malleswari associated? (a) Tennis (c) Athletics

(b) Swimming (d) Weightlifting

23. The ‘Constitution Day’ of India is observed on (a) 24th November (b) 25th November (c) 26th November (d) 27th November

24. Which is the capital of Cyprus? (a) Nicosia (c) Lamaca

(b) Polis (d) Aradipou

https://sscstudy.com/

8

CUET (UG) Section III : General Test

25. World Tuberculosis (TB) day is observed on? (a) 28th March (c) 24th May

(c) BOC (d) COD

27. Select the correct mirror image of the given figure when a mirror is placed on its right side. Question Figure

(b)

(c)

(d)

28. In a certain code language “CAUGHT” is written as “326212087”. Then in this language “SOLDER” is written as (a) 1812122459 (c) 2012152358

(a)

(b)

(c)

31. Select the option in which the numbers are related in the same way as are the numbers in the given set. (7, 98, 196) (a) (11, 154, 308) (c) (15, 190, 380)

(b) (20, 267, 520) (d) (18, 185, 360)

32. Which two signs and two numbers should be interchanged to make the given equation correct? 11 × 7 ÷ 35 − 64 + 56 = 47

33. Which of the following Venn diagram best represents the relationship between the following classes? Income tax payers, Employees, Males

(b) 1915124359 (d) 1912122359

29. In the question given below an unfolded dice is given. You are required to select the correct answer choice (s) which is/are formed by folding the unfolded dice. Question Figure

(a)

(b)

(c)

3

(a) 30 (c) 28

6

(b) 24 (d) 14

(a) Paternal uncle (b) Paternal grandfather (c) Maternal uncle (d) Brother

38. T is 5 yr older to U. U is 8 yr younger to V. V is 3 yr older to W. W is 9 yr younger to X. Who are of same age? (a) W and T (c) V and X

(b) W and V (d) X and T

39. From the given answer figures, select the one in which the question figure is hidden/embedded. Question Figure

Answer Figures

(a)

(b)

(c)

(d)

40. Which answer figure will completes the pattern in the question figure? Question Figure

Answer Figures

4

32 ? 348

37. Pointing to the photograph of Sanchi, Nitin said, “Her mother’s father’s son’s wife is my mother-inlaw’s only daughter”. How is Nitin related to Sachin’s mother?

?

1

18 22 296

(d)

34. Select the figure that can replace the question mark (?) in the following series. Question Figures

2

21 19 299

(d)

(a) + and − ; 7 and 11 (b) × and ÷ ; 35 and 56 (c) × and ÷ ; 35 and 11 (d) + and − ; 35 and 11

Answer Figures

(a)

36. Study the given pattern carefully and select the number that can replace the question mark (?) in it.

(b) 24th March (d) 28th May

26. Select the letter-cluster that can replace the question mark (?) in the following series. SAT, VEW, YIZ, ?, EUF (a) BUK (b) FIC

Answer Figures

5

Answer Figures 6 1 3

(a)

5 1 6

(b)

(a) Only A (c) Only A and B

2 3 6

(c)

1 4 2

(d)

(b) Only B (d) Only C and D

30. The sequence of folding a piece of paper and manner in which the folded paper has been cut is shown in the following figures. How would this paper look when unfolded?

(a)

(b)

(c)

(d)

35. Select the option that is embedded in the given figure X (rotation is not allowed). Question Figure

?

Answer Figures

(a)

(b)

(c)

(d)

41. How many rectangles are there in the given figure?

Answer Figures

Question Figures

(a)

(b)

(c)

https://sscstudy.com/

(d)

(a) 4

(b) 3

(c) 5

(d) 6

https://sscstudy.com/

9

PRACTICE SET 02

(a) 8-136 (c) 6-102

(b) 17-289 (d) 19-361

45. In the following question, select the odd letters from the given alternatives. (a) MN (c) FU

(b) NM (d) ZA

46. If the difference of two numbers is 4 and the difference of their squares is 64, then which is the smaller of the two number? (a) 10 (c) 4

(b) 6 (d) 8

47. What is the value of 0.03010? 301 100 301 (c) 10000

(a)

301 1000 301 (d) 100000

(b)

48. A number when divided by 12 leaves remainder 9 is obtained. What will be the remainder when the same number is divided by 3? (a) 1 (c) 2

(b) 0 (d) 4

49. What is the LCM of 4, 6 and 9? (a) 18 (c) 1

(b) 36 (d) 42

50. Working 12 h a day, Vaibhav can complete a piece of work in 28 days. How many hours a day should he work so as to finish the work in 21 days? (a) 12 (c) 15

(b) 16 (d) 18

51. If the price of an article decreases by 25%, then to restore its former value by how much per cent should the new price be increased? (a) 25 (c) 33.33

(b) 20 (d) 10

52. If the price of a commodity is decreased by 40% and its

(a) 20 (c) 45

(b) 40 (d) 60

55. If 6/7 of P = 4/5 of Q, then what is P :Q ? (a) 7 : 5 (c) 14 :15

(b) 14 : 7 (d) 8: 7

56. What is the net discount for two successive discounts of 20% and 20%? (a) 36 (c) 42

(b) 40 (d) 44

57. A trader allows two successive discounts of 20% and 10% while selling an article. If he gets ` 720 for that article, then what is the marked price of the article? (a) 1000 (c) 960

(b) 1150 (d) 1200

58. If the ratio of edges of two cubes is 2 : 3, then what is the ratio of the volume of the cubes? (a) 8 : 27 (c) 2 : 3

(b) 4: 9 (d) 4 : 27

59. A is twice as good as B and together they finish a piece of work in 24 days. In how many days can A alone to finish the work? (a) 27 (c) 42

(b) 36 (d) 48

60. A man gains 10% by selling and article for a certain price. If he sells it at double the price, then what will be the profit percentage? (a) 110 (c) 150

(b) 120 (d) 140

61. If chairs are bought at ` 400 each and sold at ` 500 each, then what will be the profit percentage? (a) 10% (c) 25%

(b) 20% (d) 60%

https://sscstudy.com/

375

425

200

2016

0

250

2014

54. The marked price of an article is twice the cost price. For a gain of 20%, what should be the discount percentage?

425

2015

(b) 16 (d) 28

395

2012

44. In the following question, select the odd number-pair from the given alternatives.

(a) 14 (c) 18

525

400 525

2013

(b) 32 (d) 11

53. The marked price of an article is 60% more than its cost price. If a discount of 20% is given, then what is the profit percentage?

City P City Q 1000 860 920 770 760 800 700 670 575 575 575 625 600 475

2010

(a) 22 (c) 44

(b) 28% increase (d) 20% decrease

2011

43. If 15 C 5 D 45 = 135 and 13 C 4 D 12 = 39, then 11 C 33 D 66 =?

(a) 28% decrease (c) 20% increase

2008

(b) 9 (d) 27

Directions (Q. Nos. 62-65) The line chart given below shows the number of cars stolen by City P and Q for the years 2008 to 2016.

2009

(a) 15 (c) 12

consumption is increased by 20%, then what will be the increase or decrease in the expenditure of the commodity?

Number of cars stolen

42. If ‘+’ means ‘÷’, ‘×’ means ‘−’, ‘−’ means ‘+’ and ‘÷’ means ‘×’, then 16 − 5 × 2 + 4 ÷ 12 = ?

Year

62. In which year from 2008 to 2016, the combined number of stolen cars from City P and Q is maximum? (a) 2008 (c) 2012

(b) 2009 (d) 2016

63. In how many years the number of cars stolen from City P is more than the number of cars stolen from City Q? (a) 1 (c) 3

(b) 2 (d) 5

64. What is the percentage decrease in the number of car stolen in year 2015 from year 2014 from City P ? (a) 50 (c) 25

(b) 20 (d) 33.33

65. The number of stolen cars from the two cities in year 2015 is what per cent of the total number of cars stolen from the two cities in 2016? (a) 82.5 (c) 87.5

(b) 86.5 (d) 79.5

66. How many odd numbered pages are there in a book of 1089 pages ? (a) 542 (c) 544

(b) 545 (d) 546

67. There were a total of 10 bicycles and tricycles. If the total number of wheels was 24, how many tricycles were there ? (a) 2 (c) 18

(b) 6 (d) 4

68. A monkey climbs a 12 m high pole. In the first minute he climbs 2 m and slips down in the next minute by 1 m. If this continues, then in how minutes will the monkey climb at the top of the pole ? (a) 10 (c) 12

(b) 21 (d) 13

https://sscstudy.com/

10

CUET (UG) Section III : General Test

69. A group of 1200 persons consisting of captains and soldiers is travelling in a train. For every 15 soldiers there is one captain. The number of captain in the group is (a) 85 (c) 75

(b) 80 (d) 70

70. A person is given ` 1 for shooting at the target and if he misses. Then he has to give ` 1. If he gets ` 30 for 100 chances, then how many chances did he miss ? (a) 25 (c) 40

(b) 35 (d) 45

(c) Both Conclusions I and II follow (d) Neither I nor II follows

72. Statements All flowers are trees. No fruit is tree. Conclusions I. No fruit is flower. II. Some trees are flowers. (a) Only Conclusion I follows (b) Only Conclusion II follows (c) Both Conclusions I and II follow (d) Neither I nor II follows

Directions (Q. Nos. 73 and 74) In the following questions, each question has a statement followed by two conclusions. Taking the statement to be true, decide which of the given conclusions definitely follows from the given statement. Indicate your answer as

Directions (Q. Nos. 71 and 72) In each of the following questions two statements followed by two or three conclusions are given. You have to take the given two statements to be true even if they seem to be at variance from commonly known facts. Read the conclusion and then decide which of the given conclusions logically follows from the two given statements.

(a) if only I follows (b) if only II follows (c) if neither I nor II follows (d) if both I and II follow

73. Statement Good health is dependent on right eating habits. Most of the people do not follow any rule regarding eating.

71. Statements Some books are tables. Some tables are mirrors. Conclusions I. Some mirrors are books. II. Some tables are books.

74. Statement Black cloud follows thunder. Rains follow thunder. Conclusions I. Thunder is the cause of rain. II. Black cloud is the cause of thunder.

Direction (Q. No. 75) In each question below is given a statement followed by two Assumptions I and II. An assumption is something supposed or taken for granted. You have to consider the statement and the following assumptions and decide which of the assumptions is implicit in the statement. Give Answer (a) If Assumption I is implicit (b) If Assumption II is implicit (c) If neither I nor II is implicit (d) If both I and II are implicit

75. Statement Of all the newspapers published in Delhi, ‘The Time and Space’ has the largest number of readers. Assumptions I. Volume of readership of all the newspapers in Delhi is known.

Conclusions I. Most of the people have poor health. II. People are ignorant of proper eating habits.

(a) Only Conclusion I follows (b) Only Conclusion II follows

II. No newspaper in Delhi other than ‘The Time and Space’ has large readership.

ANSWERS 1. 11. 21. 31. 41. 51. 61. 71.

(c) (c) (d) (a) (c) (c) (c) (b)

2. 12. 22. 32. 42. 52. 62. 72.

(c) (a) (d) (c) (a) (a) (b) (c)

3. 13. 23. 33. 43. 53. 63. 73.

(d) (b) (c) (a) (a) (d) (b) (a)

4. 14. 24. 34. 44. 54. 64. 74.

(b) (b) (a) (d) (d) (b) (d) (b)

5. 15. 25. 35. 45. 55. 65. 75.

(a) (b) (b) (b) (b) (c) (a) (a)

6. 16. 26. 36. 46. 56. 66.

(b) (c) (b) (d) (b) (a) (b)

https://sscstudy.com/

7. 17. 27. 37. 47. 57. 67.

(d) (c) (b) (d) (c) (a) (d)

8. 18. 28. 38. 48. 58. 68.

(b) (c) (d) (a) (b) (a) (b)

9. 19. 29. 39. 49. 59. 69.

(b) (c) (a) (d) (b) (b) (c)

10. 20. 30. 40. 50. 60. 70.

(a) (a) (b) (c) (b) (b) (b)

https://sscstudy.com/

PRACTICE SET 03

11

CUET (UG) Section III : General Test

Practice Paper 03 Instructions 60 questions to be attempted out of 75. 1. Which Union Ministry launched the ‘National Strategy for Additive Manufacturing’? (a) Ministry of Electronics and Information Technology (b) Ministry of MSME (c) Ministry of Commerce and Industry (d) Ministry of Science and Technology

2. France government has decided to withdraw its military from Mali after nine years, and shift its military base to which country? (a) Chad (c) Niger

(b) Senegal (d) Mauritania

3. Which bank has become the first to enter in the Metaverse? (a) Barclays (c) Citibank

(b) JPMorgan (d) UBS

4. Which country hosted multilateral exercise MILAN 2022? (a) India (c) The UK

(b) France (d) Malaysia

5. Who won Mexican Open 2022 men’s singles title? (a) Rafael Nadal (b) Cameron Norrie (c) Daniil Medvedev (d) Stefanos Tsitsipas

6. Who was the first man to walk in space? (a) M Yuri Gagarin (b) Alexey Leonov (c) Neil Arm Strong (d) Dennis Tito

7. Who was the first woman President of any country in the world? (a) Junko Tabei (b) Arunima Sinha (c) Maria Estela Peran (d) Margaret Thatcher

Time : 60 Min

8. Who was the first woman Cosmonaut in space? (a) Valentina Tereshkova (b) Junko Tabei (c) Marlyn Phipps (d) Louis Frechette

9. Which is the first country to make a Constitution? (a) Sweden (c) Japan

(b) Norway (d) USA

10. Which country is first to issue paper currency? (a) Australia (c) China

(b) USA (d) New Zealand

11. The famous Newspaper of Germany is known as (a) The Age (c) Die Welt

(b) Herald Tribune (d) The Press

12. Novosti is a major News Agency of which country? (a) USA (c) China

(b) Russia (d) Germany

13. UK’s famous News Agency is (a) Associated Press (b) Reuters (c) Europa Press (d) PTI

14. Anna Karenina is the famous work of (a) Stephen Hawking (b) Charles Dickens (c) Sidney Sheldon (d) Leo Tolstoy

15. Charles Dickens famous work is (a) Bleak House (c) Candide

(b) Blood Cive (d) Care of the Soul

16. Which award is considered the most prestigious awards in cinema world? (a) Grammy Award (b) Booker Prize (c) Jnanpith Award (d) Oscar Award

https://sscstudy.com/

17. Grammy Award is given in the field of (a) Literature (c) Science

(b) Music (d) Invention

18. In which of the fields Nobel Prize is not given? (a) Physics (c) Chemistry

(b) Mathematics (d) Medicine

19. Who received Booker Prize for the Book ‘Half a Life’? (a) Arundhati Roy (b) Anita Desai (c) VS Naipaul (d) Kiran Bedi

20. Who was the first recipient of Dhyan Chand Award in 2002? (a) Sachin Tendulkar (b) Aparna Ghosh (c) Arjun Chopra (d) Vishwanathan Anand

21. Which was the oldest Aircraft Carrier of India? (a) INS Viraat (b) INS Vikrant (c) INS Vikramaditya (d) INS Mysore

22. Which is an Air to Air Missile? (a) Prithvi(b) Agni (c) Akash (d) Astra

23. Which of the following is a Surface to Air Missile? (a) Trishul (b) K-15 Sagarika (c) Brahmos (d) Agni

24. Nuclear explosive devices were tested in India at (a) Sriharikota (b) Bengaluru (c) Pokharan (d) Kanchipuram

https://sscstudy.com/

12

CUET (UG) Section III : General Test

25. What is the name of the Light Combat Aircraft developed by India indigenously? (a) Brahmos (c) Astra

(b) Chetak (d) Tejas

26. In the following question, select the number which can be placed at the sign of question mark (?) from the given alternatives. 21 19 299

18 22 296

32. Which answer figure will complete the pattern in the question figure? Question Figure

(a) 64

?

(b) 49

(c) 25

(d) 16

40. If a mirror is placed on the line AB, then which of the answer figures is the right image of the given figure? Question Figure

Answer Figures

32 ? 348

39. In the following question, select the related number from the given alternatives. 100 : 121 : : 36 : ?

A 4 3

2 B

(a) 30 (c) 28

(b) 14 (d) 24

(a)

27. How many triangles are there in the given figure?

(a) 4 (c) 5

(b) 6 (d) 7

(b) VWUKMPIY (d) UXTKNQIY

29. In a certain code language, ‘round’ is written as ‘tall’, ‘tall’ is written as ‘red’, ‘red’ is written as ‘wood’, ‘wood’ is written as ‘bread’ and ‘bread’ is written as ‘soil’. What do we eat in breakfast? (a) soil (c) bread

30. The present ages of P and Q are 15 yr and 21 yr respectively. What will be the respective ratio of the ages of P and Q after 6 yr? (a) 5 : 7 (c) 3 : 4

(b) 5 : 6 (d) 7 : 9

31. From the given answer figures, select the one in which the question figure is hidden/embedded. Question Figure

(b) 3.75 (d) 3.25

(b) bbca

(c) caca

(d) aacc

36. In the following question, select the odd letter from the given alternatives. (a) QP (c) XY

(c)

(d)

Answer Figures

(a)

(b)

(c)

(d)

41. A piece of paper is folded and punched as shown below in the question figures. From the given answer figures, indicate how it will appear when opened? Question Figures

Answer Figures

35. In the following question, which one of letters when sequentially placed at the gaps in the given letter series shall complete it? _b c a b _ a b c _ b _

(b) DC (d) ML

(a)

(b)

(c)

(d)

42. If ‘J’ menas ‘+’, ‘K’ means ‘−’, ‘T’ means ‘×’ and ‘U’ means ‘÷’ then 18 T 3 U 27 J 2 = ? (a) 6 (c) 4

(b) 2 (d) 3

43. If 8 × 16 × 9 = 8169 and

37. Identify the diagram that best represent the relationship among the given classes. Haryana, Chandigarh, Uttar Pradesh

9 × 23 × 4 = 9234, then 4 × 10 × 11 = ? (a) 11014 (c) 14610

(b) 41011 (d) 10114

44. In the following question, select the odd word-pair from the given alternatives. (b)

(c)

(d)

38. In the following question, select the related word from the given alternatives. Liver : Organ : : Earth : ?

Answer Figures

(b)

(b) 21543 (d) 12534

(a) 3.5 (c) 4.25

(a)

(a)

(d)

34. In the following question, select the missing number from the given series (need to write options). 104, 52, 26, 13, 6. 5, ?

(a) acac

(b) wood (d) tall

(c)

33. Arrange the given words in the sequence in which they occur in the dictionary. 1. Paper 2. Parade 3. Postal 4. Praise 5. Picture (a) 12543 (c) 21534

28. In a certain code language ‘SANCTION’ is coded as ‘XFSHODJI’. Then in the same code language ‘PROFOUND’ is coded as? (a) UWTKKPIZ (c) UWTKJPIY

(b)

(a) Round (c) Home

(b) Brown (d) Planet

https://sscstudy.com/

(a) Ground : Play (c) Body : Hand

(b) Solar system : Sun (d) Tree : Stem

45. In the following question, select the related letters from the given alternatives. EI : AE : : UO : ? (a) PK (c) PJ

(b) QJ (d) QK

https://sscstudy.com/

13

PRACTICE SET 03

46. Which of the following number is divisible by 11? (a) 59609 (c) 23581

(b) 45332 (d) 44433

47. What is the value of [( a −2b3 ) ÷ ( a1b−1 )] × [( a 2b−4 ) ÷ ( a −1b2 )] ? (a) b2 (c) a 2

(b) 1/b2 (d) a 2b2

(b) 1

(c) 0

(d) 2

49. How many times digit ‘5’ appears in the number from 1 to 100? (a) 20

(b) 21

(c) 19

(d) 18

50. What is the value of 162 + 172 + 182 + ... 252 ? (a) 4325 (c) 4105

(b) 4465 (d) 4285

51. If a number is divided by 30 then it leaves 17 as a remainder. What will be the remainder when the same number is divided by 10? (a) 7 (c) 1

(b) 3 (d) 2

52. The marked price of an article is 50% more than its cost price. If a discount of 10% is given, then what is the profit percentage? (a) 25 (c) 35

(b) 30 (d) 20

53. The marked price of an article is ` 750 and a customer pays ` 600 for it. What is the discount percentage? (a) 20 (c) 15

(b) 30 (d) 10

54. P can do a piece of work in 36 days. If Q is 50% more efficient than P, then in how many days can Q do the same work? (a) 18 (c) 12

(b) 24 (d) 21

55. P is twice as good as Q and together they finish a piece of work in 36 days. The number of days taken by P alone to finish the work? (a) 48 (c) 54

(b) 60 (d) 72

56. If each edge of a cube is increased by 10%, then the percentage increase in its surface area is (a) 22% (c) 20%

(b) 19% (d) 21%

(a) 40 (c) 50

(b) 37.5 (d) 56.25

58. Average of 9 consecutive numbers is 37. What is the smallest number out of these 9? (a) 26 (c) 29

48. What is the unit of digit of 5124 × 1245 ? (a) 5

57. If the cost price of 5 articles is equal to the selling price of 8 articles, then what is the loss percentage (in %)?

(b) 38 (d) 33

59. Monthly income of X and Y is in ratio of 5 : 11 respectively. The difference in their income is ` 24000. What will be the monthly income of Y? (a) ` 44000 (c) ` 22000

(b) ` 20000 (d) ` 33000

60. Total runs scored by three players A, B, C and C are 1800. Ratio of runs scored by A and B is 5 : 7 and B and C is 3 : 2. How much runs are scored by A? (a) 756 (c) 562

(b) 612 (d) 540

61. Selling price of an article is ` 2695. If 23% discount is given, then what is the marked price (in `)? (a) 3700 (c) 3500

(b) 3100 (d) 3800

62. If x : y = 3 : 2 and x + y = 90, then the value of ( x − y) is (a) 16 (c) 18

(b) 14 (d) 12

63. If the length of a rectangle is increased by 25%, then by how much per cent breadth should be reduced so that the area remains the same? (a) 15 (c) 12.5

(b) 20 (d) 25

64. A train is moving at a speed of 84 km/h and covers a certain distance in 12 h. If the same distance is to be covered in 14 h, then what will be the speed (in km/h) of the train? (a) 72 (c) 66

(b) 76 (d) 60

65. An amount of ` 35000 is invested in a scheme of compound interest. If rate of interest is 20% per annum, then what will be the amount (in `) obtained after 2 yr? (a) 56200 (c) 48600

(b) 50400 (d) 42500

https://sscstudy.com/

66. A factory produced 1858509 cassettes in the month of January, 7623 more cassettes in the month of February and owing to short supply of electricity produced 25838 less cassettes in March than in February. Find the total production in all? (a) 5557312 (c) 5564935

(b) 5983245 (d) 5608988

67. The sum of the ages of 4 children born at the intervals of 4 yr is 48. Find the age of the youngest child. (a) 4 yr (c) 6 yr

(b) 5 yr (d) 7 yr

68. Mr. and Mrs. Gopal have two daughters and each daughter has one brother. How many persons are there in the family ? (a) 8

(b) 6

(c) 7

(d) 5

69. In a group of equal number of cows and herdsmen, the number of legs was 28 less than four times the number of heads. The number of herdsmen was (a) 7 (c) 21

(b) 28 (d) 14

70. A book has 300 pages and each page has 20 lines of 10 words each. How many words are there in the book altogether ? (a) 6000 (c) 66000

(b) 60000 (d) 600000

Directions (Q. Nos. 71 and 72) In each of the following questions two statements followed by two or three conclusions are given. You have to take the given two statements to be true even if they seem to be at variance from commonly known facts. Read the conclusion and then decide which of the given conclusions logically follows from the two given statements. 71. Statements Some pins are clips. Some clips are pens. Conclusions I. Some pins are pens. II. Some pens are clips. (a) Only conclusion I follows (b) Only conclusion II follows (c) Both conclusions I and II follow (d) Neither I nor II follows

72. Statements Some horses are lions. All lions are foxes.

https://sscstudy.com/

14

CUET (UG) Section III : General Test

Conclusions I. No fox is horse. II. Some foxes are lions.

73. Statement Workers feel highly motivated when they get sense of involvement by participating in the management of companies.

(a) Only Conclusion I follows (b) Only Conclusion II follows (c) Both Conclusions I and II follow (d) Neither I nor II follows

Conclusions I. Workers should be motivated to produce more. II. Workers should be allowed to participate in the management of companies.

Directions (Q. Nos. 73 and 74) In the following questions, each question has a statement followed by two conclusions. Taking the statement to be true, decide which of the given conclusions definitely follows from the given statement. Indicate your answer as

74. Statement Industrial revolution which first of all started in Europe has brought about modern age. Conclusions I. Disparity between rich and poor results in revolution. II. Revolution overhauls society.

(a) If only I follows (b) If only II follows (c) If neither I nor II follows (d) If both I and II follows

75. Two statements are given followed by two Conclusions I and II. You have to consider the statement to be true, even if it seems to be at variance from commonly known facts. You are to decide which of the given conclusions can definitely be drawn from the given statement. Indicate your answer. Statements All children are students. All students are players. Conclusions I. All cricketers are students. II. All children are players. (a) (b) (c) (d)

Only Conclusion II follows Both Conclusions I and II follow Neither Conclusion I nor II follows Only Conclusion I follows

ANSWERS 1. 11. 21. 31. 41. 51. 61. 71.

(a) (c) (a) (d) (a) (a) (c) (b)

2. 12. 22. 32. 42. 52. 62. 72.

(c) (b) (d) (d) (c) (c) (c) (b)

3. 13. 23. 33. 43. 53. 63. 73.

(b) (b) (a) (d) (b) (a) (b) (d)

4. 14. 24. 34. 44. 54. 64. 74.

(a) (d) (c) (d) (a) (b) (a) (b)

5. 15. 25. 35. 45. 55. 65. 75.

(a) (a) (d) (a) (d) (c) (b) (a)

6. 16. 26. 36. 46. 56. 66.

(b) (d) (b) (c) (a) (d) (c)

https://sscstudy.com/

7. 17. 27. 37. 47. 57. 67.

(c) (b) (b) (d) (b) (b) (c)

8. 18. 28. 38. 48. 58. 68.

(a) (b) (c) (d) (c) (d) (d)

9. 19. 29. 39. 49. 59. 69.

(d) (b) (a) (b) (b) (a) (d)

10. 20. 30. 40. 50. 60. 70.

(c) (b) (d) (c) (d) (d) (b)

https://sscstudy.com/

PRACTICE SET 04

15

CUET (UG) Section III : General Test

Practice Paper 04 Instructions 60 questions to be attempted out of 75. 1. National Means-cum-Merit Scholarship Scheme (NMMSS) has been extended till which year? (a) 2023-24 (c) 2029-30

(b) 2025-26 (d) 2031-32

2. Which country will become the first country to adopt India’s UPI platform? (a) Nepal (c) Bangladesh

(b) Myanmar (d) Sri Lanka

3. Which city in India has highest number of dollar-millionaire households according to the Hurun India Wealth Report 2021? (a) Bengaluru (c) Delhi

(b) Kolkata (d) Mumbai

4. Which regiment’s units were honoured with the President’s Colours on 23rd February? (a) Rajputana Rifles (b) Parachute Regiment (c) Sikh Regiment (d) Jat Regiment

5. Which Indian city has been elected as the host of International Olympic Committee’s (IOC) session 2023? (a) New Delhi (c) Chennai

(b) Mumbai (d) Hyderabad

6. Which is the 1st Spaceship landed on Mars? (a) Chandran-I (c) Columbus-IV

(b) Viking-I (d) Atlanta-III

7. Who was the first cloned human baby of world? (a) Dolly (c) Oliver

(b) Eve (d) Jack

8. Which is the largest sea port of the world? (a) Shanghai (c) Los Angels

(b) Mumbai (d) San Diago

Time : 60 Min

9. Male is the capital of which Country? (a) Mauritius (c) Maldives

(b) Lakshadweep (d) Malaysia

10. Vientiane is the capital city of which country? (a) Fiji (c) Laos

(b) Ecuador (d) Togo

11. Which is the latest country to be admitted as new member state of UN? (a) Switzerland (c) South Sudan

(b) East Timor (d) Montenegro

12. Where is the permanent Secretariat of the SAARC? (a) New Delhi (c) Columbo

(b) Islamabad (d) Kathmandu

13. Where is the headquarter of INTERPOL? (a) Paris (c) Lyons

(b) London (d) Geneva

14. Headquarter of World Health Organisation (WHO) is situated at (a) Hague (c) Geneva

(b) Oslo (d) New York

15. The Panchatantra fables are thought to be composed by (a) Mullah Nasruddin (b) Vishnu Sharma (c) King Sudharshan (d) Tenali Raman

16. Sendy Lodge Stadium is located in which country? (a) USA (c) Scotland

(b) Australia (d) England

17. Twickenham Stadium is related to which sport? (a) Football (b) Cricket (c) Rugby Football (d) Boxing

https://sscstudy.com/

18. Hopmen Cup is the famous Tournament of which sport? (a) Lawn Tennis (c) Badminton

(b) Table Tennis (d) Golf

19. Term ‘Spikers’ is related to which sport? (a) Football (c) Billiards

(b) Hand ball (d) Volleyball

20. Who was the first person to stay in Rashtrapati Bhawan? (a) William Bentinck (b) Lord Irwin (c) C Rajagopalachari (d) Lord Canning

21. What is the range of Agni V missiles? (a) 5000 km (c) 8000 km

(b) 6000 km (d) 10000 km

22. First aircraft carrier built in India is (a) INS Viraat (c) INS Arihant

(b) INS Vikrant (d) INS Kalvart

23. Barak–8 is a missile which is codeveloped by India and (a) Israel (c) US

(b) Japan (d) France

24. India’s National flag was adopted on (a) 22nd July, 1947 (b) 26th January, 1950 (c) 24th January, 1950 (d) 15th August, 1947

25. When was national calendar adopted? (a) 22nd July, 1947 (b) 24th January, 1950 (c) 26th January, 1950 (d) 22nd March, 1957

26. Which of the following Venn diagram best represents the relationship between the following classes?

https://sscstudy.com/

16

CUET (UG) Section III : General Test

Cricketers, Players, Human beings

(a)

(b)

(c)

(d)

27. Four number-pairs have been given out of the which three are alike in same manner and one is different. Select the number-pair that is different from the others. (a) 4209 : 15 (c) 8101 : 10

(b) 5121 : 12 (d) 7456 : 22

28. The sequence of folding a piece of paper and the manner in which the folded paper has been cut is shown in the following figures. How would this paper look when unfolded? Question Figures

32. Select the option in which the words share the same relationship as that shared by the given words pair Player : Team (a) Flower : Bouquet (b) Ant : Bee (c) Student : Teacher (d) Purpose : Crowd

33. Four words have been given, out of which three are alike in same manner and one is different. Select the odd word. (a) Kidney (c) Renin

(b) Pancreas (d) Liver

34. How many triangles are there in the given figure?

(c)

(d)

29. In a certain code language ‘AUSTERE’ is coded as ‘13102595’. How will ‘SETTING’ be coded as in that language? (a) 11544957 (c) 10522957

(b) 10522948 (d) 11522867

30. Select the correct mirror image of the given figure when a vertical mirrors is placed on the right side of the figure. Question Figure A

B

Answer Figures

(a)

(b)

(c)

(d)

31. Study the given pattern carefully and select the number that can replace the question mark (?) in it. 42 18 252 (a) 45 (c) 40

37 24 296

? 36 540 (b) 55 (d) 50

(b) FIBQ (d) GIAQ

39. Select the letter that can replace the question mark (?) in the following series. A, T, E, U, I, V, O, W, ?, X (a) R

(b) M

(c) U

(d) Z

40. Select the number that can replace the question mark (?) in the following series. 45, 47, 53, 65, 85? (a) 115

(a) 194 (a) 25 (c) 27

(b)

(a) FJCQ (c) FICR

(b) 130

(c) 145

(d) 105

41. Select the option that is related to the third number in the same way as the second number is related to the first number. 13 : 109 : : 24 : ?

Answer Figures

(a)

38. Select the letter-cluster that can replace the question mark (?) in the following series. TULG, WRPC, ZOTY, CLXU, ?

(b) 24 (d) 26

35. Select the figure that can replace the question mark (?) in the following series. Question Figures

? Answer Figures

(a)

(b)

(c)

(d)

36. A + B means ‘A, is the mother of B’; A − B means ‘A is the brother of B’ ; A × B means ‘A is the father of B’; A ÷ B means ‘A is the daughter of B’. If P − K × Y − J ÷ S + R, then which of the following statements is not correct? (a) J is daughter of P. (b) P is paternal uncle of R. (c) K is husband of S. (d) Y is son of S.

37. Four letter-clusters have been given, out of which three are alike in same manner and one is different. Select the odd letter-cluster. (a) PSVX (c) JMPS

(b) ZCFI (d) ORUX

https://sscstudy.com/

(b) 201

(c) 197

(d) 216

42. Select the option in which the numbers are related in the same way as the numbers in the given set. (8, 7, 407) (a) (12, 8, 555) (c) (11, 12, 450)

(b) (10, 8, 576) (d) (9, 6, 339)

43. Which two signs and two numbers should be interchanged to make the given equation correct? 28 − 32 ÷ 2 × 8 + 34 = 132 (a) × and − ; 32 and 34 (b) + and − ; 28 and 32 (c) + and − ; 32 and 34 (d) × and − ; 8 and 2

44. ‘Criticise’ is related to ‘Condemn’ in the same way as ‘Solidarity’ is related to’ ……… . (a) Unity (c) Guilty

(b) Atrocious (d) Prosperity

45. Select the option in which the given figure X is embedded (rotation is not allowed). Question Figure

Answer Figures

(a)

(b)

(c)

(d)

https://sscstudy.com/

PRACTICE SET 04

46. A can do a piece of work in 20 days and B can do the same piece of work in 30 days. They start working together and work for 5 days and then both leave the work C alone finished the remaining work in 14 days. In How many days will C alone finish the whole work? (b) 18 (d) 42

47. Marked price of an article is 40% more than its cost price. If a discount of 45% is given, then what will be the loss percentage? (a) 23 (c) 19

(b) 29 (d) 25

48. If 123457Y is completely divisible by 8, then what will be the digit in place of Y? (a) 4 (c) 8

(b) 5 (d) 6

49. A number when divided by 18 leaves remainder 15. What is the remainder when the same number is divided by 6 ? (a) 3 (c) 1

(b) 2 (d) 4

50. What is the value of 1 2 1 3 8 +5 +4 +3 ? 7 7 7 7 (a) 22 (c) 24

(b) 23 (d) 21

51. What is the value of 13 + 14 ..... 20? (a) 182 (c) 132

(b) 144 (d) 124

52. What is the digit in unit’s place of the product 25 × 252 × 3 7? (a) 1 (c) 0

(b) 6 (d) 1

54. A man spends 80% of his income and saves the rest. If his income and spending both increases by 10%, then what is the percentage change in his savings? (a) 10% increase (b) 5% decrease (c) 5% increase (d) 15% decrease

(b) 30 (d) 50

56. If some articles are bought at ` 10 each and sold at `7 each, then what is the loss percentage? (a) 60% (c) 25%

(b) 16.67% (d) 30%

(a) 40 (c) 20

(b) 25 (d) 10

58. Average of 9 numbers is 20. If a number 30 is also included, then what will be the average of these 10 numbers? (a) 20.5 (c) 19.5

(b) 21 (d) 21.5

(b) 9 : 4 (d) 7 : 3

60. A man gains 25% by selling an article at a certain price. If he sells the same article at half the price, then what is the loss percentage? (a) 62.5 (c) 37.5

(b) 27.5 (d) 32.5

61. What is the value of 19 + 8 3 ? (a) 4 + 3 (c) 8 + 3

(b) 4 − 3 (d) 8 − 3

Directions (Q. Nos. 62-65) The bar chart given below shows the stock of 6 different types of shoes in a store. 350 300 250 200 150 100 50 0

300

120 90

S2

S3

S4

60 S5

Type of shoes

https://sscstudy.com/

S6

(b) 30 (d) 25

63. What is the average number of shoes of all 6 types of shoes? (a) 166.67 (c) 116.66

(b) 333.33 (d) 163.33

64. S4 is what per cent of S3? (b) 80 (d) 75

65. S2 are how much per cent more than S5? (a) 350 (c) 250

(b) 150 (d) 300

66. In a company 60% workers are males. If the number of female workers in the company is 800. What is the number of male workers in the company ? (b) 1600 (d) 1400

67. Aarna cut a cake into two halves and cuts one half into smaller pieces of equal size. Each of the small pieces is twenty grams in weight. If she has seven pieces of the cake in all with her, how heavy was the original cake ? (a) 120 g (c) 240 g

(b) 140 g (d) 280 g

68. A certain number of horses and an equal number of men are going somewhere. Half of the owner are on their horse’s back while the remaining ones are walking along leading their horses. If the number of legs walking on the ground is 70, how many horses are there ? (a) 10 (c) 14

(b) 12 (d) 16

69. Mani is double the age of Prabhu, Ramona is half the age of Prabhu. If Mani is sixty years old, find out the age of Ramona. (a) 20 yr (c) 10 yr

220 210

S1

(a) 60 (c) 20

(a) 1200 (c) 1900

59. B starts some business by investing ` 90000. After 4 months, D joins business by investing ` 80000. At the end of the year, in what ratio will they share the profit? (a) 10 : 7 (c) 27 : 16

62. S6 are what per cent of total number of shoes?

(a) 90 (c) 70

57. By selling 50 m of cloth, a person gains the cost price of 20 m of cloth. What is his gain per cent?

(b) 2 (d) 5

53. Which smallest number to be subtracted from 300, so that the resulting number is completely divisible by 9? (a) 5 (c) 3

(a) 20 (c) 40

Stock

(a) 24 (c) 36

55. A man saves 30% of his income in 1 yr. If he wants to save the same amount in 8 months, then by how much percentage should he increase his monthly savings?

17

(b) 15 yr (d) 24 yr

70. John’s age is 42 yr and Kelvin’s age is 26 yr and how many years ago was Kelvin’s age half of John’s age ? (a) 6 yr (c) 10 yr

(b) 4 yr (d) 8 yr

https://sscstudy.com/

18

CUET (UG) Section III : General Test

Directions (Q. Nos. 71 and 72) In each of the following questions two statements followed by two or three conclusions are given. You have to take the given two statements to be true even if they seem to be at variance from commonly known facts. Read the conclusion and then decide which of the given conclusions logically follows from the two given statements. 71. Statements All wheels are brakes. Some gears are wheels. Conclusions I. No brake is gear. II. Some gears are brakes.

(c) Both Conclusions I and II follow (d) Neither I nor II follows

Assumptions I. Wait-listed candidates get admission with difficulty. II. A large number of candidates were on the waiting list.

Directions (Q. Nos. 73 and 74) In each question below is given a statement followed by two Assumptions I and II. An assumption is something supposed or taken for granted. You have to consider the statement and the following assumptions and decide which of the assumptions is implicit in the statement. Given answer (a) If Assumption I is implicit (b) If Assumption II is implicit (c) If neither I nor II is implicit (d) If both I and II are implicit

75. A statement is given, followed by two Conclusions I and II. You have to consider the statement to be true even, if it seem to be at variance from commonly known facts. You have to decide which of the given conclusions, if any, follow from the given statement. Statement The Government of India has recently announced several concessions and offered attractive package tours to foreign visitors.

73. Statement Apart from the entertainment value of television, its educational value cannot be ignored.

(a) Only Conclusion I follows (b) Only Conclusion II follows (c) Both Conclusions I and II follow (d) Neither I nor II follows

Conclusions I. Now more number of foreign tourists will visit India. II. The Government of India seems to be serious in attracting tourists.

Assumptions I. People take the television to be means of entertainment only. II. The educational value of television is not realised properly.

72. Statements All men are dogs. All dogs are cats. Conclusions I. All men are cats. II. All cats are men.

(a) (b) (c) (d)

74. Statement Ten candidates who were on the waiting list could finally be admitted to the course.

(a) Only Conclusion I follows (b) Only Conclusion II follows

Only Conclusion I follows Only Conclusion II follows Neither Conclusion I nor II follows Both Conclusions I and II follow

ANSWERS 1. 11. 21. 31. 41. 51. 61. 71.

(b) (c) (a) (a) (c) (c) (a) (b)

2. 12. 22. 32. 42. 52. 62. 72.

(a) (d) (b) (a) (d) (c) (b) (a)

3. 13. 23. 33. 43. 53. 63. 73.

(d) (c) (a) (c) (c) (c) (c) (d)

4. 14. 24. 34. 44. 54. 64. 74.

(b) (c) (a) (a) (a) (a) (d) (a)

5. 15. 25. 35. 45. 55. 65. 75.

(b) (b) (d) (d) (c) (d) (c) (a)

6. 16. 26. 36. 46. 56. 66.

(b) (c) (c) (a) (a) (d) (a)

https://sscstudy.com/

7. 17. 27. 37. 47. 57. 67.

(b) (c) (b) (a) (a) (a) (c)

8. 18. 28. 38. 48. 58. 68.

(a) (a) (b) (b) (d) (b) (c)

9. 19. 29. 39. 49. 59. 69.

(c) (d) (c) (c) (a) (c) (b)

10. 20. 30. 40. 50. 60. 70.

(c) (b) (b) (a) (d) (c) (c)

https://sscstudy.com/

PRACTICE SET 05

19

CUET (UG) Section III : General Test

Practice Paper 05 Instructions 60 questions to be attempted out of 75. 1. The Smart Card Arms License and Shastra App has been launched by which the police department of which State/UT? (a) Uttar Pradesh (c) Delhi

(b) Maharashtra (d) Tamil Nadu

2. The World Sustainable Development Summit is an annual flagship event of which organisation? (a) (b) (c) (d)

World Wide Fund for Nature-India Confederation of Indian Industry The Energy and Resources Institute Quality Council of India

3. Name the bank which has recently acquired the highest stake in India Debt Resolution Company Ltd (IDRCL)? (a) Canara Bank (b) State Bank of India (c) Punjab National Bank (d) Union Bank of India

4. Which nation has successfully tested its new naval air defence system called ‘C-Dome’? (a) Israel (c) Turkey

(b) Iran (d) North Korea

5. Sadia Tariq won a gold medal for India in which event? (a) Wushu (c) Boxing

(b) Fencing (d) Weightlifting

6. Name the capital of Uganda? (a) Mogadishu (c) Lusaka

(b) Kampala (d) Bulenga

7. Denmark Legislature is called as (a) Shora (c) Tasongidu

(b) Diet (d) Folketing

8. White paper is the official paper of which government? (a) Japan (c) India

(b) France (d) Iran

Time : 60 Min

9. Official report of the Government of Japan is known as (a) Blue Book (c) Yellow Book

(b) Grey Book (d) Green Book

10. What does ‘Satyameva Jayate’ mean? (a) ‘Truth alone triumphs’ (b) ‘True Faith is Rare’ (c) ‘Truth is Divine’ (d) ‘Truth is a Treasure’

11. Which among the following is not an official language of United Nation? (a) Russian (c) Arabic

(b) German (d) French

12. The Indian recipient of Noble Peace Prize is (a) Hargovind Khurana (b) Rabindranath Tagore (c) Mother Teresa (d) Amartya Sen

13. First Nobel Prize in Economics was given in (a) 1967 (c) 1970

(b) 1968 (d) 1969

14. Pulitzer Prize is given in which field? (a) Music (c) Literature

(b) Movie (d) Sport

15. The Magsaysay Award for Social Service was instituted by (a) The Government of India (b) The UNO (c) The ILO (d) Philippines Government

16. Who is the first Indian to fly a plane? (a) JRD Tata (b) Mihir Sen (c) Saifuddin Kitchlew (d) WC Banerjee

https://sscstudy.com/

17. Who was the first Indian to travel in space? (a) Rakesh Sharma (b) Kalpana Chawla (c) Ravish Malhotra (d) Sunita Williams

18. The first Central Agricultural University of India was constructed in (a) Manipur (c) Bihar

(b) Uttar Pradesh (d) Assam

19. Rukmini Satellite is dedicated to which purpose? (a) Education (b) Remote Sensing (c) Agriculture (d) Defence

20. Central tobacco research Institute is locate at (a) Rajahmundry (c) Hyderabad

(b) Coimbatore (d) Pune

21. Who wrote the National song of India? (a) Rabindranath Tagore (b) Bankim Chandra Chatterjee (c) Mohammed Iqbal (d) Chitragupta

22. Which is the largest fresh water lake in the world? (a) Lake Victoria (c) Lake Superior

(b) Lake Erie (d) Lake Ontario

23. National Good Governance Day is observed in India on (a) 24th December (b) 25th December (c) 26th December (d) 31st December

24. Which is the fastest land animal in the World? (a) Dog (c) Tiger

(b) Cheetah (d) Horse

https://sscstudy.com/

20

CUET (UG) Section III : General Test

25. When is International day for Preservation of Ozone layer observed? (a) 16th September (b) 4th July (c) 23rd January (d) 1st May

26. Study the given pattern carefully and select the number that can replace the question mark (?) in it.

(a) 9

?

13

49

9

17

69

13

11

59

(b) 5

(c) 10

(d) 21

(b) GNS (d) GMS

28. A + B means ‘A is the husband of B’; A − B means, ‘B is the sister of A’ A × B means ‘A is the mother of B’; A ÷ B means ‘B is the son of A’. If P + R × T − Q ÷ S + U, then how is P related to S? (a) Maternal grandfather (b) Father-in-law (c) Paternal grandfather (d) Uncle

29. Four number-pairs have been given, out of which three are alike in some manner and one is different. Select the number-pair that is different from the rest. (a) 11 : 121 (c) 17 : 289

(b) 13 : 169 (d) 15 : 250

30. Select the number that can replace the question mark (?) in the following series. 5, 10, 26, ?, 122, 170 (a) 82 (c) 65

(b) 16132420 (d) 16136220

(a) Clarinet (c) Xylophone

(b) Trumpet (d) Saxophone

35. ‘Astronomy’ is related to ‘Stars’ in the same way as ‘Agronomy’ is related to ‘………’. (a) Mines (c) Planets

(a) − and ÷ (c) × and −

(b) ÷ and × (d) + and ×

37. Three different positions of the same dice are shown. Select the symbol that will be on the face opposite to the one showing +. %

# $

*

(a) # (c) @

# @

+

%

(b) 4, 3, 5, 1, 2 (d) 3, 5, 1, 2, 4

40. In the given Venn diagram, the ‘rectangle’ represents travellers who like to travel by plane, the ‘circle’ represents ‘travellers who like to travel by bus’, and the ‘triangle’ ‘represents travellers who like to travel to train’. The numbers given in the diagram represents the number of travellers in that particular category.

(b) Crops (d) Emotions

36. Which two signs should be interchanged to make the given equation correct? 12 + 81 − 27 × 9 ÷ 3 = 36

@

(a) 3, 5, 1, 4, 2 (c) 2, 3, 5, 1, 4

10 5

2 9

11 6

13

How many travellers like to travel either by train or plane but not by bus? (a) 36 (c) 30

(b) 27 (d) 29

41. Select the option that depicts how the given transparent sheet of paper would appear if it is folded at the dotted line. Question Figure

(b) * (d) $

38. Select the correct mirror image of the given figure when a mirror is placed on the right of the figure. Question Figure

Answer Figures

(a)

(b)

(c)

(d)

42. How many triangles are there in the given figure?

Answer Figures

(b) GCV (d) FBL

32. Select the option in which the words share the same relationship as that shared by the given pair of words. North : South (a) Frown : Smile (b) Whole : Total

(a) 16125020 (c) 16128620

3. Million 4. Octillion 5. Trillion

(b) 50 (d) 77

31. Four letter-clusters have been given, out of which three are alike in some manner and one is different. Select the odd letter-cluster. (a) BJT (c) DDP

33. In a certain code language ‘PAGE’ is coded as ‘161495’. How will ‘PART’ be coded as in that language?

34. Four words have been given, out of which three are alike in some manner and one is different. Select the odd word.

27. Select the letter-cluster that can replace the question mark (?) in the following series. ADG, CGK, EJO, ?, IPW (a) GMZ (c) GNZ

(c) Ductile : Bendable (d) Integrity : Honesty

(a)

(b)

(c)

(d)

39. Arrange the following words in a logical and meaningful order. 1. Quadrillion 2. Quintillion

https://sscstudy.com/

(a) 32 (c) 33

(b) 30 (d) 44

43. Select the option that is embedded in the given figure. (Rotation is not allowed)

https://sscstudy.com/

PRACTICE SET 05

Question Figure

49. The length of two parallel sides of trapezium are 30 cm and 40 cm. If the area of the trapezium is 350 cm 2, then what is the value (in cm) of its height? (a) 8 (c) 15

Answer Figures

(a)

(b) 10 (d) 12

50. P, Q and R undertook a work for `48000. Together P and Q complete 5/12th part of the work. What is the share (in `) of R ?

(b)

(a) 21000 (c) 27000

(c)

51. When 80 is subtracted from 20% of a number, the result is 30. What is the value of the number?

(d)

44. Select the figure that can replace the question mark (?) in the following series. Question Figures J

Q M

V

?

(a)

B

(b)

BA

A

(c)

(d)

45. Select the number that can replace the question mark (?) in the following series. 62, 66, 74, 90, ? (a) 106 (c) 116

(b) 120 (d) 122

46. The ratio of two positive numbers is 9 : 11. Their product is 6336. What is the smallest number? (a) 32 (c) 88

(b) 72 (d) 48

47. If a shopkeeper marks the price of goods 40% more than their cost price and allows a discount of 40%, then what is his gain or loss percent? (a) 16% loss (c) 10% loss

(b) 16% profit (d) 12% profit

48. If the difference between discount of 35% and two successive discounts of 20% on a certain bill is `3, then what is the amount (in `) of the bill? (a) 250 (c) 350

(b) 300 (d) 400

(a) 330 (c) 550

(b) 440 (d) 220

52. If the rice is sold at ` 48 per kg, then there would be a 20% loss. To earn a profit of 20% what should be the price of rice (per kg)? (a) 72 (c) 78

Answer Figures AB

(b) 28000 (d) 31000

(b) 76 (d) 84

53. A number is first decreased by 30% and then increased by 30%. If the number so obtained is 72 less than the original number, then what is the value of the original number? (a) 720 (c) 960

(b) 800 (d) 1080

54. What is the unit digit of 342 × 743 + 175? (a) 1 (c) 3

(b) 2 (d) 7

55. Which one is the largest fraction among 3/4, 7/8 and 5/6? (a) 3/4 (c) 5/6

(b) 7/8 (d) All are equals

56. What is the value of 2 + 2 −1 + 2 2 + 2 −2 ? (a) 27/4 (c) 9/5

(b) 9/4 (d) 25/4

57. If 1062 = 11236 , then what is the value of 112.36 + 11236? (a) 106.6 (c) 116.6

(b) 100.6 (d) 126.6

58. Which of the given values is completely divisible by 18? (a) 1642 (c) 7218

(b) 3612 (d) 2427

https://sscstudy.com/

21

59. X alone can complete a work in 6 days and Y alone can complete the same work in 30 days. If X and Y work together, then in how many days work will be completed? (a) 5 (c) 6

(b) 4 (d) 4.5

60. The population of a town increases at the rate of 15% per annum. If the present population is 108445 of town, then what was the population 2 yr ago? (a) 72000 (b) 79000 (c) 82000 (d) 85000

61. A 450 m long train crosses a bridge 650 m long in 36 s. What is speed (in km/h) of the train? (a) 110 (c) 150

(b) 125 (d) 95

62. A car travels at a speed of 25 m/s for 8 hours. What is the distance (in km) travelled by the car? (a) 360 (c) 450

(b) 720 (d) 900

63. If the ratio of the cost price and selling price of an article is 5 : 6, then what will be the profit percentage? (a) 10 (c) 15

(b) 12 (d) 20

64. The average of 6 numbers is 18. If one number is excluded, the average becomes 17. What is the excluded number? (a) 22 (c) 21

(b) 23 (d) 20

65. If 2/3 of P = 1 / 5 of Q, then what is P : Q? (a) 3 : 5 (c) 3 : 7

(b) 3 : 10 (d) 3 : 9

66. The total age of a mother and her daugher is 60 yr. The difference between their age is 30 yr. Find out the age of mother. (a) 40 yr (c) 45 yr

(b) 50 yr (d) 55 yr

67. Which number will be in the middle if the following numbers are arranged in descending order? 4456, 4465, 4655, 4665, 4565 (a) 4456 (c) 4565

(b) 4465 (d) 4655

https://sscstudy.com/

22

CUET (UG) Section III : General Test

Directions (Q. Nos. 68-70) Read the following information and answer the following questions. In a class of 40 students, 28 can speak Tamil and 30 can speak Telugu. All students can speak at least one of the two languages. 68. Find the number of students who can speak only Tamil. (a) 8 (c) 12

69. Find the minimum number of students who can speak both Tamil and Telugu. (a) 12 (c) 18

(b) 15 (d) 22

(b) 10 (d) 14

Directions (Q. Nos. 73 and 74) In each

Directions (Q. Nos. 71 and 72) In each of the following questions two statements followed by two or three conclusions are given. You have to take the given two statements to be true even if they seem to be at variance from commonly known facts. Read the conclusion and then decide which of the given conclusions logically follows from the two given statements. 71. Statements All pens are roads. All roads are houses.

74. Statement The mangoes are too cheap to be good. Assumptions I. When the mango crop is abundant, the prices go down. II. The lower the selling price, the inferior is the quality of the commodity.

(a) Only Conclusion I and II follows (b) Only Conclusion II and III follows (c) Both Conclusions I and III follow (d) No conclusion follow

70. Find the number of persons who can speak only Telugu? (a) 8 (c) 12

(a) Only Conclusion I and II follows (b) Only Conclusion II and III follows (c) Both Conclusions I and III follow (d) No conclusion follow

72. Statements All pens are chalks. All chairs are chalks. Conclusions I. Some pens are chairs. II. Some chalks are pens. III. Some chalks are chairs.

(b) 10 (d) 14

Assumptions I. Railway trains are the only mode of transport available in the suburbs of Mumbai. II. Only railway trains run punctually.

Conclusions I. All houses are pens. II. Some houses are pens. III. All pens are houses.

question below is given a statement followed by two Assumptions I and II. An assumption is something supposed or taken for granted. You have to consider the statement and the following assumptions and decide which of the assumptions is implicit in the statement. Give Answer

75. Two statements are given followed by two conclusions I and II. You have to consider the statements to be true even if they seem to be at variance from commonly known facts. You have to decide which of the given conclusions, if any, follow from the given statements. Indicate your answer. Statements All animals are dogs. All dogs are birds. Conclusions I. All animals are birds. II. All birds are animals.

(a) If Assumption I is implicit (b) If Assumption II is implicit (c) If neither I nor II is implicit (d) If both I and II are implicit

(a) (b) (c) (d)

73. Statement In Mumbai, railway trains are indispensable for people in the suburbs to reach their places of work on time.

Only Conclusion I follows Only Conclusion II follows Both Conclusion I and II follows Neither conclusion I nor II follows

ANSWERS 1. 11. 21. 31. 41. 51. 61. 71.

(c) (b) (b) (b) (b) (c) (a) (b)

2. 12. 22. 32. 42. 52. 62. 72.

(c) (c) (c) (a) (c) (a) (b) (b)

3. 13. 23. 33. 43. 53. 63. 73.

(a) (d) (b) (b) (c) (b) (d) (c)

4. 14. 24. 34. 44. 54. 64. 74.

(a) (c) (b) (c) (b) (a) (d) (b)

5. 15. 25. 35. 45. 55. 65. 75.

(a) (d) (a) (b) (d) (b) (b) (a)

6. 16. 26. 36. 46. 56. 66.

(b) (a) (b) (a) (b) (a) (c)

https://sscstudy.com/

7. 17. 27. 37. 47. 57. 67.

(d) (a) (d) (c) (a) (c) (c)

8. 18. 28. 38. 48. 58. 68.

(c) (a) (a) (a) (b) (c) (b)

9. 19. 29. 39. 49. 59. 69.

(b) (d) (d) (d) (b) (a) (c)

10. 20. 30. 40. 50. 60. 70.

(a) (a) (b) (c) (b) (c) (c)